You are on page 1of 970

fiziks

Forum for CSIR-UGC JRF/NET, GATE, IIT-JAM, GRE in PHYSICAL SCIENCES

SYLLABUS FOR GATE EXAMINATION IN PHYSICS (PH)


Mathematical Physics
Linear vector space; matrices; vector calculus; linear differential equations; elements of
complex analysis; Laplace transforms, Fourier analysis, elementary ideas about tensors.

Classical Mechanics
Conservation laws; central forces, Kepler problem and planetary motion; collisions and scattering
in laboratory and centre of mass frames; mechanics of system of particles; rigid body dynamics;
moment of inertia tensor; noninertial frames and pseudo forces; variational principle; Lagrange's
and Hamilton's formalisms; equation of motion, cyclic coordinates, Poisson bracket; periodic
motion, small oscillations, normal modes; special theory of relativity - Lorentz transformations,
relativistic kinematics, mass-energy equivalence.

Electromagnetic Theory
Solution of electrostatic and magnetostatic problems including boundary value problems;
dielectrics and conductors; Biot-Savart's and Ampere's laws; Faraday's law; Maxwell's equations;
scalar and vector potentials; Coulomb and Lorentz gauges; Electromagnetic waves and their
reflection, refraction, interference, diffraction and polarization. Poynting vector, Poynting
theorem, energy and momentum of electromagnetic waves; radiation from a moving charge.

Quantum Mechanics
Physical basis of quantum mechanics; uncertainty principle; Schrodinger equation; one, two and
three dimensional potential problems; particle in a box, harmonic oscillator, hydrogen atom;
linear vectors and operators in Hilbert space; angular momentum and spin; addition of angular
momenta; time independent perturbation theory; elementary scattering theory.

Thermodynamics and Statistical Physics


Laws of thermodynamics; macrostates and microstates; phase space; probability ensembles;
partition function, free energy, calculation of thermodynamic quantities; classical and quantum
statistics; degenerate Fermi gas; black body radiation and Planck's distribution law; Bose-Einstein
condensation; first and second order phase transitions, critical point.

Atomic and Molecular Physics


Spectra of one- and many-electron atoms; LS and jj coupling; hyperfine structure; Zeeman and
Stark effects; electric dipole transitions and selection rules; X-ray spectra; rotational and
vibrational spectra of diatomic molecules; electronic transition in diatomic molecules, Franck-
Condon principle; Raman effect; NMR and ESR; lasers.

fiziks c/o Anand Institute of mathematics, 28-B/6 Jia Sarai


Near IIT, Hauz Khas, New Delhi, PIN- 110016 (INDIA)
Phone: 011-32718565, +91-9871145498
Website: http://www.physicsbyfiziks.com 1
Email: fiziks.physics@gmail.com
fiziks
Forum for CSIR-UGC JRF/NET, GATE, IIT-JAM, GRE in PHYSICAL SCIENCES

Solid State Physics


Elements of crystallography; diffraction methods for structure determination; bonding in solids;
elastic properties of solids; defects in crystals; lattice vibrations and thermal properties of solids;
free electron theory; band theory of solids; metals, semiconductors and insulators; transport
properties; optical, dielectric and magnetic properties of solids; elements of superconductivity.

Nuclear and Particle Physics


Nuclear radii and charge distributions, nuclear binding energy, Electric and magnetic moments;
nuclear models, liquid drop model - semi-empirical mass formula, Fermi gas model of nucleus,
nuclear shell model; nuclear force and two nucleon problem; Alpha decay, Beta-decay,
electromagnetic transitions in nuclei; Rutherford scattering, nuclear reactions conservation
laws; fission and fusion; particle accelerators and detectors; elementary particles, photons,
baryons, mesons and leptons; quark model.

Electronics
Network analysis; semiconductor devices; Bipolar Junction Transistors, Field Effect Transistors,
amplifier and oscillator circuits; operational amplifier, negative feedback circuits , active filters
and oscillators; rectifier circuits, regulated power supplies; basic digital logic circuits, sequential
circuits, flip-flops, counters, registers, A/D and D/A conversion

fiziks c/o Anand Institute of mathematics, 28-B/6 Jia Sarai


Near IIT, Hauz Khas, New Delhi, PIN- 110016 (INDIA)
Phone: 011-32718565, +91-9871145498
Website: http://www.physicsbyfiziks.com 2
Email: fiziks.physics@gmail.com
fiziks
Institute for NET/JRF, GATE, IIT‐JAM, JEST, TIFR and GRE in PHYSICAL SCIENCES 
 
MATEMATICAL PHYSICS SOLUTIONS
GATE-2010
Q1. Consider an anti-symmetric tensor Pij with indices i and j running from 1 to 5. The
number of independent components of the tensor is
(a) 3 (b) 10 (c) 9 (d) 6
Ans: (b)
Solution: The number of independent components of the tensor

= N  N   1 25  5  10 N  5
1 2
2 2
e 2 sin z 
Q2. The value of the integral  dz , where the contour C is the unit circle: z  2  1 ,
C z2

is
(a) 2πi (b) 4πi (c) πi (d) 0
Ans: (d)
e z sin z
Solution: Pole is at z  0 , Circle z  2  1   dz  2i  0  0 .
C z2
2 3 0
 
Q3. The eigenvalues of the matrix  3 2 0  are
0 1 
 0

(a) 5, 2, -2 (b) -5, -1, -1 (c) 5, 1, -1 (d) -5, 1, 1


Ans: (c)
Solution: The characteristic equation of the matrix A , A  I  0

2 3 0
 A  I  3 2 0  0  3  52    5  0    5,1,  1
0 0 1 

0 for x  3,
Q4. If f  x    then the Laplace transform of f(x) is
x  3 for x  3

(a) s 2 e sx (b) s 2 e sx (c) s 2 (d) s 2 e  sx

                                                                                
Head office  Branch office 
 
fiziks, H.No. 23, G.F, Jia Sarai,  Anand Institute of Mathematics, 
 
Near IIT, Hauz Khas, New Delhi‐16  28‐B/6, Jia Sarai, Near IIT 
 
Phone: 011‐26865455/+91‐9871145498 Hauz Khas, New Delhi‐16 
                                                   
                                             Website: www.physicsbyfiziks.com                                                                                          
                                                            Email: fiziks.physics@gmail.com                                                                   1 
fiziks
Institute for NET/JRF, GATE, IIT‐JAM, JEST, TIFR and GRE in PHYSICAL SCIENCES 
 
Ans: (d)
 3  
Solution: L f  x    e  sx
f  x  dx   e s x
f  x  dx   e  sx
f  x  dx    x  3 e  sx dx
0 0 3 3

   
e  sx  e  sx  1  e  sx 
L f  x    x  3
1
  1    dx  0   e  sx dx    2  sx
 s e
s 3 3  s  s3 s   s 3

d2y
Q5. The solution of the differential equation for y t  :  y  2 cosh(t ) , subject to the
dt 2
dy
initial conditions y 0   0 and  0 , is
dt t  0

cosh t   t sinh t  (b)  sinh t   t cosh t 


1
(a)
2
(c) t cosh t  (d) t sinh t 
Ans: (d)
Solution: For C.F D 2  1y  0  m  1  C.F .  C1e t  C 2 e t

 e t  e t 
P.I .  2
1
2 cosh t   2 2
1
  2
1
et  2
1
 t t
e t  e t   e t    
D 1 D 1  2  D 1 D 1 2 2

t t t t
 y  C1e t  C 2 e t  e  e  y 0   0  C1  C 2  0
2 2
dy t 1 t 1
 C1e t  C 2 e t  e t  e t  e t  e t
dt 2 2 2 2
dy 1 1
 0  C1  C 2  0   0   0  C1  C 2  0
dt t 0 2 2

Since C1  C 2  0 and C1  C 2  0  C1  0, C 2  0 .
t t t t
Thus  y  e  e  y  t sinh t
2 2

                                                                                
Head office  Branch office 
 
fiziks, H.No. 23, G.F, Jia Sarai,  Anand Institute of Mathematics, 
 
Near IIT, Hauz Khas, New Delhi‐16  28‐B/6, Jia Sarai, Near IIT 
 
Phone: 011‐26865455/+91‐9871145498 Hauz Khas, New Delhi‐16 
                                                   
                                             Website: www.physicsbyfiziks.com                                                                                          
                                                            Email: fiziks.physics@gmail.com                                                                   2 
fiziks
Institute for NET/JRF, GATE, IIT‐JAM, JEST, TIFR and GRE in PHYSICAL SCIENCES 
 
GATE-2011
Q6. Two matrices A and B are said to be similar if B = P-1AP for some invertible matrix P.
Which of the following statements is NOT TRUE?
(a) DetA = DetB (b) Trace of A = Trace of B
(c) A and B have the same eigenvectors (d) A and B have the same eigenvalues
Ans: (c)
Solution: If A and P be square matrices of the same type and if P be invertible then matrices A
and B = P-1AP have the same characteristic roots
Then B  I  P 1 AP  P 1IP  P 1  A  I P where I is identity matrix.

B  I  P 1  A  I P  P 1 A  I P  A  I P 1 P  A  I PP 1  A  I

Thus the matrices A and B (= P-1AP) have the same characteristic equation and hence
characteristic roots of eigen values. Since the sum of the eigen values of a matrix and
product of eigen values of a matrix is equal to the determinant of matrix hence third
alternative is incorrect.

Q7. If a force F is derivable from a potential function V(r), where r is the distance from the
origin of the coordinate system, it follows that

(a)   F  0 (b)   F  0 (c)  V  0 (d)  2 V  0


Ans: (a)
 
Solution: Since F is derivative from potential V(r) and F  V r 

   F    V  0 .  
Q8. A 33 matrix has elements such that its trace is 11 and its determinant is 36. The
eigenvalues of the matrix are all known to be positive integers. The largest eigenvalues of
the matrix is
(a) 18 (b) 12 (c) 9 (d) 6
Ans: (c)
Solution: We know that for any matrix
1. The product of eigenvalues is equal to determinant of that matrix.

                                                                                
Head office  Branch office 
 
fiziks, H.No. 23, G.F, Jia Sarai,  Anand Institute of Mathematics, 
 
Near IIT, Hauz Khas, New Delhi‐16  28‐B/6, Jia Sarai, Near IIT 
 
Phone: 011‐26865455/+91‐9871145498 Hauz Khas, New Delhi‐16 
                                                   
                                             Website: www.physicsbyfiziks.com                                                                                          
                                                            Email: fiziks.physics@gmail.com                                                                   3 
fiziks
Institute for NET/JRF, GATE, IIT‐JAM, JEST, TIFR and GRE in PHYSICAL SCIENCES 
 
2. 1   2  3  .......  Trace of matrix
1   2  3  11 and 12 3  36 . Hence the largest eigen value of the matrix is 9.
Q9. The unit vector normal to the surface x2 + y2 – z = 1 at the point P(1, 1, 1) is
iˆ  ˆj  kˆ 2iˆ  ˆj  kˆ iˆ  2 ˆj  kˆ 2iˆ  2 ˆj  kˆ
(a) (b) (c) (d)
3 6 6 3

Ans: (d)
Solution: The equation of the system is f x, y, z   x 2  y 2  z  1  0
   ˆ  ˆ 2
The gradient of the above function is f   iˆ  j  k x  y 2  z  1
 x y z 

 2 xiˆ  2 yˆj  lˆ

f 2iˆ  2 ˆj  kˆ
Hence unit normal vector at (1, 1, 1)  .
f 3

Q10. Consider a cylinder of height h and radius a, closed at both ends, centered at the origin.

Let r  iˆx  ˆjy  kˆz be the position vector and n̂ a unit vector normal to the surface. The

surface integral  r  nˆ ds over the closed surface of the cylinder is


S
z

O y

x
(a) 2πa2 (a + h) (b) 3πa2h (c) 2 πa2h (d) zero
Ans: (b)

 
Solution:  r.nˆ ds   .r d  3 d  3a 2 h
S V V

dy x
Q11. The solutions to the differential equation  are a family of
dx y 1

(a) circles with different radii


(b) circles with different centres

                                                                                
Head office  Branch office 
 
fiziks, H.No. 23, G.F, Jia Sarai,  Anand Institute of Mathematics, 
 
Near IIT, Hauz Khas, New Delhi‐16  28‐B/6, Jia Sarai, Near IIT 
 
Phone: 011‐26865455/+91‐9871145498 Hauz Khas, New Delhi‐16 
                                                   
                                             Website: www.physicsbyfiziks.com                                                                                          
                                                            Email: fiziks.physics@gmail.com                                                                   4 
fiziks
Institute for NET/JRF, GATE, IIT‐JAM, JEST, TIFR and GRE in PHYSICAL SCIENCES 
 
(c) straight lines with different slopes
(d) straight lines with different intercepts on the y-axis
Ans: (a)
dy x x2 y2
Solution:   xdx  ydy  dy  0    y  C1  x 2  y 2  2 y  2C1
dx y 1 2 2

  x  0    y  1  2C1  1  C
2 2

which is family of circles with different radii.


z sin z
Q12. Which of the following statements is TRUE for the function f  z   ?
 z   2
(a) f  z  is analytic everywhere in the complex plane

(b) f  z  has a zero at z  

(c) f  z  has a pole of order 2 at z  

(d) f  z  has a simple pole at z  

Ans: (c)
z sin z
Solution: f  z   has a pole of order 2 at z  
z  
2

Q13. Consider a counterclockwise circular contour z 1 about the origin. Let

z sin z
f z   , then the integral  f z dz over this contour is
 z   2
(a) –iπ (b) zero (c) iπ (d) 2iπ
Ans: (b)

                                                                                
Head office  Branch office 
 
fiziks, H.No. 23, G.F, Jia Sarai,  Anand Institute of Mathematics, 
 
Near IIT, Hauz Khas, New Delhi‐16  28‐B/6, Jia Sarai, Near IIT 
 
Phone: 011‐26865455/+91‐9871145498 Hauz Khas, New Delhi‐16 
                                                   
                                             Website: www.physicsbyfiziks.com                                                                                          
                                                            Email: fiziks.physics@gmail.com                                                                   5 
fiziks
Institute for NET/JRF, GATE, IIT‐JAM, JEST, TIFR and GRE in PHYSICAL SCIENCES 
 
GATE-2012
 
Q14. Identify the correct statement for the following vectors a  3iˆ  2 ˆj and b  iˆ  2 ˆj
 
(a) The vectors a and b are linearly independent
 
(b) The vectors a and b are linearly dependent
 
(c) The vectors a and b are orthogonal
 
(d) The vectors a and b are normalized
Ans: (a)
   
Solution: If a  3iˆ  2 ˆj, b  iˆ  2 ˆj are linearly dependent a  mb  0, for some values of m but
3 + m = 0 and 2 + 2m = 0 do not have any solution. So they are linearly independent.
   
a  b  0 (Not orthogonal); a  b  0 (Not normalized)
Q15. The number of independent components of the symmetric tensor Aij with indices
i, j  1, 2,3 is
(a) 1 (b) 3 (c) 6 (d) 9
 A11 A12 A13 
Ans: (c) For symmetric tensor Aij   A21 A22 A23 
 A31 A32 A33 

 A12  A21 , A23  A32 , A13  A33 , hence there are six independent components.

0 1 0
Q16. The eigenvalues of the matrix 1 0 1  are
0 0 
 1

(a) 0, 1, 1 (b) 0, 2 , 2


1 1
(c) , ,0 (d) 2 , 2 ,0
2 2
Ans: (b)
 1 0
Solution: A  I  0  1   
1  0   2  1    0    0,  2 ,  2
0 1 

                                                                                
Head office  Branch office 
 
fiziks, H.No. 23, G.F, Jia Sarai,  Anand Institute of Mathematics, 
 
Near IIT, Hauz Khas, New Delhi‐16  28‐B/6, Jia Sarai, Near IIT 
 
Phone: 011‐26865455/+91‐9871145498 Hauz Khas, New Delhi‐16 
                                                   
                                             Website: www.physicsbyfiziks.com                                                                                          
                                                            Email: fiziks.physics@gmail.com                                                                   6 
fiziks
Institute for NET/JRF, GATE, IIT‐JAM, JEST, TIFR and GRE in PHYSICAL SCIENCES 
 
GATE-2013
    
Q17. If A and B are constant vectors, then  A  B  r is   
    
(a) A  B (b) A  B (c) r (d) zero
Ans: (d)

Solution: Let A  A0  xˆ  yˆ  zˆ  , B  B0  xˆ  yˆ  zˆ  and r  xxˆ  yyˆ  zzˆ .
    
B  r  xˆ  z  y B0  yˆ  z  x B0  zˆ  y  x B0   A  B  r  0 .   
16 z
Q18. For the function f  z   , the residue at the pole z  1 is (your answer
z  3z  12
should be an integer) ____________.
Ans: 3

1 d 21   z  1 16 z 
2

Solution: At z  1 pole is of order 2 so residue is   =3


2  1 dz 21   z  3 z  12 
  z 1

 4  1  1
Q19. The degenerate eigenvalue of the matrix  1 4  1 is (your answer should be an
 1  1 4 

integer) ____________
Ans: 2,5,5

4   1 1   1 1 1 
 1 4  
1   (4   ) 0 5   0  = (4   )(5   )2  0    2,5,5 .

 1 1 4    0 0 5   

Q20. The number of distinct ways of placing four indistinguishable balls into five
distinguishable boxes is ___________.
Ans: 120
Solution: 4  C 45 =120 ways

                                                                                
Head office  Branch office 
 
fiziks, H.No. 23, G.F, Jia Sarai,  Anand Institute of Mathematics, 
 
Near IIT, Hauz Khas, New Delhi‐16  28‐B/6, Jia Sarai, Near IIT 
 
Phone: 011‐26865455/+91‐9871145498 Hauz Khas, New Delhi‐16 
                                                   
                                             Website: www.physicsbyfiziks.com                                                                                          
                                                            Email: fiziks.physics@gmail.com                                                                   7 
fiziks
Institute for NET/JRF, GATE, IIT‐JAM, JEST, TIFR and GRE in PHYSICAL SCIENCES 
 
GATE-2014
Q21. The unit vector perpendicular to the surface x 2  y 2  z 2  3 at the point (1, 1, 1) is
xˆ  yˆ  zˆ xˆ  yˆ  zˆ xˆ  yˆ  zˆ xˆ  yˆ  zˆ
(a) (b) (c) (d)
3 3 3 3
Ans: (d)

Solution: Let f  x 2  y 2  z 2  3  0   f  2 xxˆ  2 yyˆ  2 zzˆ

f 2 xˆ  2 yˆ  2 zˆ xˆ  yˆ  zˆ
 nˆ   at 1,1,1  
f 12 3

Q22. The matrix


1  1 1  i
A   is
3 1  i  1 
(a) orthogonal (b) symmetric (c) anti-symmetric (d) Unitary
unitary
unitary A† A  I
Q23. The value of the integral
z2
C e z  1 dz
where C is the circle z  4 , is

(a) 2 i (b) 2 2 i (c) 4 3i (d) 4 2 i


Ans. (c)
Solution: Pole e z  1  e z  ei 2 m 1 where m  0,1, 2,3.....

  z 2
Residue Res z i    i   2
 z  e

Similarly, z  i , Res   2

I  2 i  2   2   4 3i

                                                                                
Head office  Branch office 
 
fiziks, H.No. 23, G.F, Jia Sarai,  Anand Institute of Mathematics, 
 
Near IIT, Hauz Khas, New Delhi‐16  28‐B/6, Jia Sarai, Near IIT 
 
Phone: 011‐26865455/+91‐9871145498 Hauz Khas, New Delhi‐16 
                                                   
                                             Website: www.physicsbyfiziks.com                                                                                          
                                                            Email: fiziks.physics@gmail.com                                                                   8 
fiziks
Institute for NET/JRF, GATE, IIT‐JAM, JEST, TIFR and GRE in PHYSICAL SCIENCES 
 
d y
2
Q24. The solution of the differential equation 2  y  0 , subject to the boundary conditions
dt

y 0   1 and y    0 is
(a) cos t  sin t (b) cosh t  sinh t
(c) cos t  sin t (d) cosh t  sinh t
Ans: (d)
Soluiton:
D 2  1  0  D  1  y  t   c1et  c2 e  t

Applying boundary condition


y 0   1  1  c1  c2 and y     0  0  c1e  c2 e 

 c1  0, c2  1

 y  t   e t  y  t   cosh t  sinh t

GATE-2015
1
Q25. Consider a complex function f  z   . Which one of the following
 1
z  z   cos  zx 
 2
statements is correct?
(a) f  z  has simple poles at z  0 and z  
1
2
(b) f  z  has second order pole at z  
1
2
(c) f  z  has infinite number of second order poles
(d) f  z  has all simple poles
Ans.: (a)
1
Solution: f  z 
 1
z  z   cos  z 
 2

 lim  z  a  f  z   finite and  0


n
For nth order pole
z a

                                                                                
Head office  Branch office 
 
fiziks, H.No. 23, G.F, Jia Sarai,  Anand Institute of Mathematics, 
 
Near IIT, Hauz Khas, New Delhi‐16  28‐B/6, Jia Sarai, Near IIT 
 
Phone: 011‐26865455/+91‐9871145498 Hauz Khas, New Delhi‐16 
                                                   
                                             Website: www.physicsbyfiziks.com                                                                                          
                                                            Email: fiziks.physics@gmail.com                                                                   9 
fiziks
Institute for NET/JRF, GATE, IIT‐JAM, JEST, TIFR and GRE in PHYSICAL SCIENCES 
 
At z  0
lim zf  z   finite  z  0 is a simple pole.
z 0

1
At z  
2
2
 1  1
z  z 
lim  2
 lim  2
 lim
1
 1 z cos z z  1.cos z  z.   sin z 
1 1 1
 cos z
z  z 
2 zz  2 2
 2
1 1 2
 lim1
   finite
z  cos z  z sin z  
2
2
1
 f  z  has second order pole at z  
2
3
Q26. The value of  t 2 3t  6 dt is_______________ (upto one decimal place)
0

Ans.: 1.33
3 3 3
1 4
Solution:  t   3t  6  dt   t  3  t  2   dt   t 2  t  2  dt 
2 2

0 0
30 3

If f  x   e  x and g  x   x e  x , then
2 2
Q27.

(a) f and g are differentiable everywhere


(b) f is differentiable everywhere but g is not
(c) g is differentiable everywhere but f is not
(d) g is discontinuous at x  0
Ans. (b)

Solution: f ( x)  e  x is differentiable but g ( x)  x e  x is not differentiable.


2 2

 xe  x ; x  0
2

g ( x)  
 x2
 xe ; x  0
2
 x h
Left hand Limit lim g  x  h     x  h  e
h 0

                                                                                
Head office  Branch office 
 
fiziks, H.No. 23, G.F, Jia Sarai,  Anand Institute of Mathematics, 
 
Near IIT, Hauz Khas, New Delhi‐16  28‐B/6, Jia Sarai, Near IIT 
 
Phone: 011‐26865455/+91‐9871145498 Hauz Khas, New Delhi‐16 
                                                   
                                             Website: www.physicsbyfiziks.com                                                                                          
                                                            Email: fiziks.physics@gmail.com                                                                   10 
fiziks
Institute for NET/JRF, GATE, IIT‐JAM, JEST, TIFR and GRE in PHYSICAL SCIENCES 
 
2
 xh
Right hand Limit lim g  x  h    x  h  e
h 0

 lim g  x  h   lim g  x  h 
h 0 h o

Q28. Consider w  f  z   u  x, y   iv x, y  to be an analytic function in a domain D . Which


one of the following options is NOT correct?
(a) u  x, y  satisfies Laplace equation in D
(b) v x, y  satisfies Laplace equation in D
z2

(c)  f z dz is dependent on the choice of the contour between z and z


z1
1 2 in D

(d) f  z  can be Taylor expended in D


Ans.: (c)
Solution: w  f ( z )  u  x, y   iv  x, y  to be an analytic function in a domain D,  zz12 f ( z )dz is

independent on the choice of the contour between z1 and z2 in D .

 1 for t  0
Q29. The Heaviside function is defined as H t    and its Fourier transform is
 1 for t  0

given by  2i /  . The Fourier transform of


1
H t  1 / 2  H t  1 / 2 is
2
   
sin   cos 
(a) 2 (b) 2
 
2 2
 
(c) sin   (d) 0
2
Ans.: (a)

Solution: H  f    h t  e
 i 2 ft
dt


2i
For a function h  t  , H  f   

                                                                                
Head office  Branch office 
 
fiziks, H.No. 23, G.F, Jia Sarai,  Anand Institute of Mathematics, 
 
Near IIT, Hauz Khas, New Delhi‐16  28‐B/6, Jia Sarai, Near IIT 
 
Phone: 011‐26865455/+91‐9871145498 Hauz Khas, New Delhi‐16 
                                                   
                                             Website: www.physicsbyfiziks.com                                                                                          
                                                            Email: fiziks.physics@gmail.com                                                                   11 
fiziks
Institute for NET/JRF, GATE, IIT‐JAM, JEST, TIFR and GRE in PHYSICAL SCIENCES 
 
For h  t  t0  , Fourier Transform is e  i 2 ft0
Hf

Shifting theorem

1   1   1   1  i 2 
 i  2i 1  i 2 
 i  2i
For  h  t  
 h t     e  e 2
   e  e 2
 i
2   2   2  2    2i   

 
sin  
1
The Fourier transform of  H  t  1/ 2   H  t  1/ 2    2 .
2 
2
m2
A function y  z  satisfies the ordinary differential equation y 
1
Q30. y  2 y  0, where
z z
m  0, 1, 2, 3, ..... Consider the four statements P, Q, R, S as given below.

P: z m and z  m are linearly independent solutions for all values of m


Q: z m and z  m are linearly independent solutions for all values of m  0
R: ln z and 1 are linearly independent solutions for m  0
S: z m and ln z are linearly independent solutions for all values of m
The correct option for the combination of valid statements is
(a) P, R and S only (b) P and R only (c) Q and R only (d) R and S only
Ans.: (c)
1 m2
Solution: y   y  2 y  0  z 2 y  zy  m 2 y  0

z z
d
m  0,1, 2,3,.... z  ex , D 
dx
If m  0 ; z 2 y  zy  0

 D  D  1  D  y  0   D 2  D  D  y  0  D 2  m 2 y  0  
y  c1  c2 x D  m

y  c1  c2 ln z c1e mx  c2 e  mx

R is correct. c1em log z  c2 e m log z

                                                                                
Head office  Branch office 
 
fiziks, H.No. 23, G.F, Jia Sarai,  Anand Institute of Mathematics, 
 
Near IIT, Hauz Khas, New Delhi‐16  28‐B/6, Jia Sarai, Near IIT 
 
Phone: 011‐26865455/+91‐9871145498 Hauz Khas, New Delhi‐16 
                                                   
                                             Website: www.physicsbyfiziks.com                                                                                          
                                                            Email: fiziks.physics@gmail.com                                                                   12 
fiziks
Institute for NET/JRF, GATE, IIT‐JAM, JEST, TIFR and GRE in PHYSICAL SCIENCES 
 
or if m  0, m  0

y  c1 cosh  m log  z    ic2 sinh  m log  x   m0

GATE-2016
dy
Q31. Consider the linear differential equation  xy . If y  2 at x  0 , then the value of y at
dx
x  2 is given by
(a) e 2 (b) 2e 2 (c) e 2 (d) 2e 2
Ans.: (d)
dy 1 x2
 xy  dy  xdx  ln y   ln c  y  ce x / 2
2
Solution:
dx y 2

If y  2 at x  0  c  2  y  2e x
2
/2
.

The value of y at x  2 is given by y  2e2


Q32. Which of the following is an analytic function of z everywhere in the complex plane?

(a) z 2 (b) z *  2
(c) z
2
(d) z

Ans.: (a)

Solution: z 2   x  iy   x 2  y 2  i  2 xy   u  x 2  y 2 and v  2 xy
2

u v v u
Cauchy Riemann equations   2 x,   2 y satisfies.
x y x y

The direction of f for a scalar field f  x, y, z   x 2  xy  z 2 at the point P1,1,2  is
1 1
Q33.
2 2

(a)
 ˆj  2kˆ (b)
 ˆj  2kˆ (c)
 ˆj  2kˆ (d)
 ˆj  2kˆ
5 5 5 5
Ans.: (b)
  
 f  ˆj  2kˆ
Solution: f   x  y  iˆ  xjˆ  zkˆ  nˆ     
 f  5
 1,1,2

                                                                                
Head office  Branch office 
 
fiziks, H.No. 23, G.F, Jia Sarai,  Anand Institute of Mathematics, 
 
Near IIT, Hauz Khas, New Delhi‐16  28‐B/6, Jia Sarai, Near IIT 
 
Phone: 011‐26865455/+91‐9871145498 Hauz Khas, New Delhi‐16 
                                                   
                                             Website: www.physicsbyfiziks.com                                                                                          
                                                            Email: fiziks.physics@gmail.com                                                                   13 
fiziks
Institute for NET/JRF, GATE, IIT‐JAM, JEST, TIFR and GRE in PHYSICAL SCIENCES 
 
Q34. A periodic function f  x  of period 2 is defined in the interval    x   

 1,    x  0
f x   
 1, 0 x 

The appropriate Fourier series expansion for f  x  !is

4
(a) f  x    sin x  sin 3x  / 3  sin 5 x  / 5  ...
 
4
(b) f  x    sin x  sin 3x  / 3  sin 5 x  / 5  ..
 
4
(c) f  x    cos x  cos 3x  / 3  cos 5 x  / 5  ...
 
4
(d) f  x    cos x  cos 3x  / 3  cos 5 x  / 5  ...
 
Ans.: (a)
 1,    x  0
Solution: f  x   
 1, 0 x 

Let f  x   a0    an cos nx  bn sin nx 
n 1

1 
 a0 
2   f  x dx

 
  f  x dx  2     1 dx   1 dx   2   x    x   0
1 1 0 1 
 a0 
0

2   0  0

This can also be seen without integration, since the area under the curve of f x  between
 to  is zero.
1 
f  x  cos nxdx

 an 


1   sin nx   sin nx  
0
1 0 

 1 cos nxdx  0 1 cos nxdx     
  
 an     0
   n   n 0 

1 
f  x  sin nxdx

 bn 

                                                                                
Head office  Branch office 
 
fiziks, H.No. 23, G.F, Jia Sarai,  Anand Institute of Mathematics, 
 
Near IIT, Hauz Khas, New Delhi‐16  28‐B/6, Jia Sarai, Near IIT 
 
Phone: 011‐26865455/+91‐9871145498 Hauz Khas, New Delhi‐16 
                                                   
                                             Website: www.physicsbyfiziks.com                                                                                          
                                                            Email: fiziks.physics@gmail.com                                                                   14 
fiziks
Institute for NET/JRF, GATE, IIT‐JAM, JEST, TIFR and GRE in PHYSICAL SCIENCES 
 

 bn     1 sin nxdx   1 sin nxdx 
1 0

   0 

 cos nx   1  1  1  1 1  1  2 2  1n 



1   cos nx 
0 n n

 bn               
   n   n 0    n n n n    n n 

4
If n is even bn  0 and If n is odd bn  .
n
4 1 1 
Thus Fourier series is f  x   sin x  sin 3x  sin 5 x  ...
 3 5 
GATE-2017

dz
Q35. The contour integral  1  z 2
evaluated along a contour going from  to  along the

real axis and closed in the lower half-plane circle is equal to………….. (up to two
decimal places).
Ans. : 

1 1 1
Solution: C 1  z 2 dz   1  x 2 dx  C 1  z 2 dz
Poles, 1  z 2  0
z  i
z  i is inside C
1 1 1
 Res  z  i   lim  z  i   
z  i  z  i  z  i  i  i 2i

1 1
 dx    2 i  

1 x 2
2i
(Since here we use lower half plane i.e. we traversed in clockwise direction hence we
have to take 2 i )

                                                                                
Head office  Branch office 
 
fiziks, H.No. 23, G.F, Jia Sarai,  Anand Institute of Mathematics, 
 
Near IIT, Hauz Khas, New Delhi‐16  28‐B/6, Jia Sarai, Near IIT 
 
Phone: 011‐26865455/+91‐9871145498 Hauz Khas, New Delhi‐16 
                                                   
                                             Website: www.physicsbyfiziks.com                                                                                          
                                                            Email: fiziks.physics@gmail.com                                                                   15 
fiziks
Institute for NET/JRF, GATE, IIT‐JAM, JEST, TIFR and GRE in PHYSICAL SCIENCES 
 
Q36. The coefficient of e in the Fourier expansion of u  x   A sin 2  x  for k  2 is
ikx

A A A A
(a) (b) (c) (d)
4 4 2 2
Ans.: (b)
ei x  e  i x
Solution: We can write sin  x  
2i
ei 2 x  2  e2i x
Hence, sin  x  2

 4 
Since, 2   k , hence
e  ikm  2  eikx
sin 2  x  
 4 
Hence

A
ck    sin  x  dx
2

8 

  
A   ikx ikx 
  
 ikx
 e e dx  2 e dx  e  ikx eikx dx 
8     
  
A  2ikx 
  
 ikx
 e dx  2 e dx  dx 
8     
The first two integrals are zero and the third integral has the value 2 .
Thus
A A
ck    2   
8 4
Q37. The imaginary part of an analytic complex function is v  x, y   2 xy  3 y . The real part of

the function is zero at the origin. The value of the real part of the function at 1  i is
……………... (up to two decimal places)
Ans. : 3
Solution: The imaginary part of the given analytic function is v  x , y   2 xy  3 y . From the

Cauchy – Riemann condition


                                                                                
Head office  Branch office 
 
fiziks, H.No. 23, G.F, Jia Sarai,  Anand Institute of Mathematics, 
 
Near IIT, Hauz Khas, New Delhi‐16  28‐B/6, Jia Sarai, Near IIT 
 
Phone: 011‐26865455/+91‐9871145498 Hauz Khas, New Delhi‐16 
                                                   
                                             Website: www.physicsbyfiziks.com                                                                                          
                                                            Email: fiziks.physics@gmail.com                                                                   16 
fiziks
Institute for NET/JRF, GATE, IIT‐JAM, JEST, TIFR and GRE in PHYSICAL SCIENCES 
 
v u
  2x  3
y x
Integrating partially gives
u  x , y   x 2  3x  g  y 

From the second Cauchy – Riemann condition


u v
 , we obtain
y x
dg  y 
 2 y  g  y    y 2  c
dy

Hence u  x , y   x 2  3 x  y 2  c

Since the real part of the analytic function is zero at the origin. Hence
0  000c  c  0

Thus u  x , y   x 2  3 x  y 2

Thus f  z    x 2  3x  y 2   i  2 xy  3 y 

Thus the value of real part when z  1  i , that is x  1 and y  1 is 1  3 1  1  3 .
2

Q38. Let X be a column vector of dimension n  1 with at least one non-zero entry. The
number of non-zero eigenvalues of the matrix M  XX T is
(a) 0 (b) n (c) 1 (d) n  1
Ans. : (c)
0
0
 
a 
Solution: Let X    then X T   0 0... a... 0
0
0
 
 0 

                                                                                
Head office  Branch office 
 
fiziks, H.No. 23, G.F, Jia Sarai,  Anand Institute of Mathematics, 
 
Near IIT, Hauz Khas, New Delhi‐16  28‐B/6, Jia Sarai, Near IIT 
 
Phone: 011‐26865455/+91‐9871145498 Hauz Khas, New Delhi‐16 
                                                   
                                             Website: www.physicsbyfiziks.com                                                                                          
                                                            Email: fiziks.physics@gmail.com                                                                   17 
fiziks
Institute for NET/JRF, GATE, IIT‐JAM, JEST, TIFR and GRE in PHYSICAL SCIENCES 
 
Here X is an n 1 column vector with the entry in the i th row equal to a. X T is a row

vector having entry in the i th column equal to a. Then XX T is an n 1 matrix having the

entry in the i th row and i th column equal to a 2 .


Hence
0 0 0...0...0 0 
0 0 0...0...0 0 
 ith row
XX  0 0 0...0...0 0 
T
.....................
..................... 
0 0 0...0...0 0 

ith row
Since this matrix is diagonal its eigenvalues are a 2 , 0, 0.....0 . Hence the number of

nonzero eigenvalues of the matrix XX T is 1 .

dy  
Q39. Consider the differential equation  y tan  x   cos  x  . If y  0   0, y   is
dx 3
…………... (up to two decimal places)
Ans.: 0.52
The given differential equation is a linear differential equation of the form
dy
 p  x  y  cos x
dx

Integrating factor  e 
p  x  dx

Thus integrating factor  e 


tan x dx

 I  F  eln sec x  sec x


Thus the general solution of the given differential equation is
y  sec x   sec x  cos xdx  c

 y sec x  x  c -(i)

It is given that y  0   0 . Hence

0  sec 0  0  c  c  0

                                                                                
Head office  Branch office 
 
fiziks, H.No. 23, G.F, Jia Sarai,  Anand Institute of Mathematics, 
 
Near IIT, Hauz Khas, New Delhi‐16  28‐B/6, Jia Sarai, Near IIT 
 
Phone: 011‐26865455/+91‐9871145498 Hauz Khas, New Delhi‐16 
                                                   
                                             Website: www.physicsbyfiziks.com                                                                                          
                                                            Email: fiziks.physics@gmail.com                                                                   18 
fiziks
Institute for NET/JRF, GATE, IIT‐JAM, JEST, TIFR and GRE in PHYSICAL SCIENCES 
 
Thus the solution satisfying the given condition is
x
y sec x  x  y 
sec x
 
Thus the value of y   is
3
 /3  /3 
y    0  52
sec  / 3 2 6

                                                                                
Head office  Branch office 
 
fiziks, H.No. 23, G.F, Jia Sarai,  Anand Institute of Mathematics, 
 
Near IIT, Hauz Khas, New Delhi‐16  28‐B/6, Jia Sarai, Near IIT 
 
Phone: 011‐26865455/+91‐9871145498 Hauz Khas, New Delhi‐16 
                                                   
                                             Website: www.physicsbyfiziks.com                                                                                          
                                                            Email: fiziks.physics@gmail.com                                                                   19 
fiziks
Institute for NET/JRF, GATE, IIT‐JAM, JEST, TIFR and GRE in PHYSICAL SCIENCES 
 
CLASSICAL MECHANICS SOLUTIONS
GATE- 2010
Q1. For the set of all Lorentz transformations with velocities along the x-axis consider the two
statements given below:
P: If L is a Lorentz transformation then, L-1 is also a Lorentz transformation.
Q: If L1 and L2 are Lorentz transformations then, L1L2 is necessarily a Lorentz
transformation.
Choose the correct option
(A) P is true and Q is false (B) Both P and Q are true
(C) Both P and Q are false (D) P is false and Q is true
Ans: (b)
1 2  3
Q2. A particle is placed in a region with the potential V x   kx  x , where k, λ > 0.
2 3
Then,
k
(A) x = 0 and x  are points of stable equilibrium

k
(B) x = 0 is a point of stable equilibrium and x  is a point of unstable equilibrium

k
(C) x = 0 and x  are points of unstable equilibrium

(D) There are no points of stable or unstable equilibrium
Ans: (b)
1 2 x 3 V k
Solution: V  kx    kx  x 2  0  x  0, x  .
2 3 x 
 2V
  k  2x
x 2
 2V k  2V
 At x  0,   ve (Stable) and  At x  ,  ve (unstable)
x 2  x 2

                                                                                
Head office  Branch office 
 
fiziks, H.No. 23, G.F, Jia Sarai,  Anand Institute of Mathematics, 
 
Near IIT, Hauz Khas, New Delhi‐16  28‐B/6, Jia Sarai, Near IIT 
 
Phone: 011‐26865455/+91‐9871145498 Hauz Khas, New Delhi‐16 
                                                   
                                             Website: www.physicsbyfiziks.com                                                                                          
                                                            Email: fiziks.physics@gmail.com                                                                   1 
fiziks
Institute for NET/JRF, GATE, IIT‐JAM, JEST, TIFR and GRE in PHYSICAL SCIENCES 
 
0
Q3. A π meson at rest decays into two photons, which move along the x-axis. They are both
detected simultaneously after a time, t = 10s. In an inertial frame moving with a velocity
V = 0.6c in the direction of one of the photons, the time interval between the two
detections is
(A) 15 s (B) 0 s (C) 10 s (D) 20 s
Ans: (a)
v v
1 1
c 1  0.6 c
Solution: t1  t 0  10  10  2  20sec , t 2  t 0
v 1  0.6 v
1 1
c c

1  0.6 1
 10  10   5sec
1  0.6 2
 t1  t 2  15sec
Statement for Linked Answer Questions 4 and 5:
1 2 2
The Lagrangian for a simple pendulum is given by L  ml   mgl 1  cos  
2
Q4. Hamilton’s equations are then given by
p p
(A) p   mgl sin  ;   2 (B) p   mgl sin  ;   2
ml ml
p g p
(C) p   m;    (D) p     ;  
m l ml
Ans: (b)
P2 H H P
Solution: H   mgl 1  cos     P  P  mglsin ;       2 .Q5. The
2ml 2
 P ml

Poisson bracket between θ and  is

 
(A)  ,  1   1
(B)  ,  2
ml

 
(C)  , 
1
m
 
(D)  , 
g
l
Ans: (b)
                                                                                
Head office  Branch office 
 
fiziks, H.No. 23, G.F, Jia Sarai,  Anand Institute of Mathematics, 
 
Near IIT, Hauz Khas, New Delhi‐16  28‐B/6, Jia Sarai, Near IIT 
 
Phone: 011‐26865455/+91‐9871145498 Hauz Khas, New Delhi‐16 
                                                   
                                             Website: www.physicsbyfiziks.com                                                                                          
                                                            Email: fiziks.physics@gmail.com                                                                   2 
fiziks
Institute for NET/JRF, GATE, IIT‐JAM, JEST, TIFR and GRE in PHYSICAL SCIENCES 
 

   P  P 1   
 ,   ,  2  where    2 .  2  
 P  1 1
  1  2  0  2 .
 ml  ml ml   P P   ml ml

GATE- 2011
1  q
Q6. A particle is moving under the action of a generalized potential V q, q   . The
q2

magnitude of the generalized force is


21  q  21  q  2 q
(A) (B) (C) (D)
q3 q3 q3 q3

Ans: (c)
d  V  V 2
Solution:     Fq  Fq  3 .
dt  q  q q
Q7. Two bodies of mass m and 2m are connected by a spring constant k. The frequency of the
normal mode is
(A) 3k / 2m (B) k/m (C) 2k / 3m (D) k / 2m

Ans: (a)

k k 3k 2mm 2m
Solution: m k
2m     where reduce mass    .
 2m 2m 2m  m 3
3
Q8. Let (p, q) and (P, Q) be two pairs of canonical variables. The transformation
Q  q  cosp  , P  q  sinp 

is canonical for
(A) α = 2, β = 1/2 (B) α = 2, β =2 (C) α = 1, β = 1 (D) α = 1/2, β = 2
Ans: (d)
Q P Q P
Solution:    1
q p p q

 q  1 cosp   q   cosp   q    sin p   q  1 sin p   1

q 2 1  cos 2 p  sin 2 p   1  q 2 1  1    ,   2 .


1
2

                                                                                
Head office  Branch office 
 
fiziks, H.No. 23, G.F, Jia Sarai,  Anand Institute of Mathematics, 
 
Near IIT, Hauz Khas, New Delhi‐16  28‐B/6, Jia Sarai, Near IIT 
 
Phone: 011‐26865455/+91‐9871145498 Hauz Khas, New Delhi‐16 
                                                   
                                             Website: www.physicsbyfiziks.com                                                                                          
                                                            Email: fiziks.physics@gmail.com                                                                   3 
fiziks
Institute for NET/JRF, GATE, IIT‐JAM, JEST, TIFR and GRE in PHYSICAL SCIENCES 
 
Q9. Two particles each of rest mass m collide head-on and stick together. Before collision, the
speed of each mass was 0.6 times the speed of light in free space. The mass of the final
entity is
(A) 5m / 4 (B) 2m (C) 5m / 2 (D) 25 m / 8
Ans: (c)
Solution: From conservation of energy
mc 2 mc 2 2mc 2
  m1c 2   m1c 2
2
v2
v2
v
1 1 1
c2 c2 c2

Since v  0.6c  m1  5m / 2

GATE- 2012
Q10. In a central force field, the trajectory of a particle of mass m and angular momentum L in
plane polar coordinates is given by,
1 m
 1   cos  
r L2
where, ε is the eccentricity of the particle’s motion. Which one of the following choice
for ε gives rise to a parabolic trajectory?
(a) ε = 0 (b) ε = 1 (c) 0 < ε < 1 (d) ε > 1
Ans: (b)
l m
Solution:  1   cos   for parabolic trajectory   1 .
r l2
Q11. A particle of unit mass moves along the x-axis under the influence of a potential,
V  x   x x  2 . The particle is found to be in stable equilibrium at the point x = 2. The
2

time period of oscillation of the particle is


 3
(a) (b)  (c) (d) 2
2 2
Ans: (b)

                                                                                
Head office  Branch office 
 
fiziks, H.No. 23, G.F, Jia Sarai,  Anand Institute of Mathematics, 
 
Near IIT, Hauz Khas, New Delhi‐16  28‐B/6, Jia Sarai, Near IIT 
 
Phone: 011‐26865455/+91‐9871145498 Hauz Khas, New Delhi‐16 
                                                   
                                             Website: www.physicsbyfiziks.com                                                                                          
                                                            Email: fiziks.physics@gmail.com                                                                   4 
fiziks
Institute for NET/JRF, GATE, IIT‐JAM, JEST, TIFR and GRE in PHYSICAL SCIENCES 
 
V 2
V  x   x x  2     x  2   2 x x  2   0  x  2, x 
2 2

x 3

 2V  2V
 2 x  2 2
 2 x  2   2 x   2 2  4
x 2 x 2

 2V 2
    2 T  .
x 2 x2
T

Q12. A rod of proper length l0 oriented parallel to the x-axis moves with speed 2c/3 along the
x-axis in the S-frame, where c is the speed of the light in free space. The observer is also
moving along the x-axis with speed c/2 with respect to the S-frame. The length of the rod
as measured by the observer is
(a) 0.35l0 (b) 0.48l0 (c) 0.87l0 (d) 0.97l0
Ans: (d)

u2x
Solution: l  l0 1   0.97 l0
c2
Q13. A particle of mass m is attached to fixed point O by a weightless inextensible string of
length a. It is rotating under the gravity as shown in the figure. The
z
Lagrangian of the particle is

1
 
L ,    ma 2  2  sin 2  2  mga cos  where θ and  are the
2 a

polar angles. The Hamiltonian of the particles is m

1  2 p2  g
(a) H   p    mga cos  (b)
2ma 2  sin 2
 
 

1  2 p2 
H  p    mga cos
2ma 2  sin 2
 
 

(c) H 
1
2ma 2
 p2  p2   mga cos  (d) H 
1
2ma 2
 p2  p2   mga cos 

Ans: (b)
                                                                                
Head office  Branch office 
 
fiziks, H.No. 23, G.F, Jia Sarai,  Anand Institute of Mathematics, 
 
Near IIT, Hauz Khas, New Delhi‐16  28‐B/6, Jia Sarai, Near IIT 
 
Phone: 011‐26865455/+91‐9871145498 Hauz Khas, New Delhi‐16 
                                                   
                                             Website: www.physicsbyfiziks.com                                                                                          
                                                            Email: fiziks.physics@gmail.com                                                                   5 
fiziks
Institute for NET/JRF, GATE, IIT‐JAM, JEST, TIFR and GRE in PHYSICAL SCIENCES 
 
1
Solution: H  P  P  L  P   P   ma 2  2  sin 2  2  mga cos 
2
 
L P L P
 P  ma 2   P     2 and P    ma 2 sin 2    
 ma  ma 2 sin 2 

Put the value of  and 

 2
 
2
2 
P P 1 2   P 
P
H  P   P    ma    sin     mga cos
ma 2 ma 2 sin 2  2   ma 2   ma 2 sin 2   
 

P2 P2 P2 P2


H     mga cos
ma 2 2ma 2 ma 2 sin 2  2ma 2 sin 2 

1  2 P2 
H  P    mga cos 
2ma 2  sin 2  
 

Statement for Linked Answer Questions 14 and 15:


Q14. A particle of mass m slides under the gravity without friction along the parabolic path
y  ax 2 axis shown in the figure. Here a is a constant.
y

x
The Lagrangian for this particle is given by

m1  4a 2 x 2 x 2  mgax 2


1 2 1
(a) L  mx  mgax 2 (b) L 
2 2

(c) L 
1 2
2
mx  mgax 2 (d) L 
1
2
 
m 1  4a 2 x 2 x 2  mgax 2

Ans: (d)

                                                                                
Head office  Branch office 
 
fiziks, H.No. 23, G.F, Jia Sarai,  Anand Institute of Mathematics, 
 
Near IIT, Hauz Khas, New Delhi‐16  28‐B/6, Jia Sarai, Near IIT 
 
Phone: 011‐26865455/+91‐9871145498 Hauz Khas, New Delhi‐16 
                                                   
                                             Website: www.physicsbyfiziks.com                                                                                          
                                                            Email: fiziks.physics@gmail.com                                                                   6 
fiziks
Institute for NET/JRF, GATE, IIT‐JAM, JEST, TIFR and GRE in PHYSICAL SCIENCES 
 
1
Solution: Equation of constrain is given by y  ax 2 , K.E T  m  x 2  y 2 
2
1 1
y  2axx  T  m  x 2  4ax 2 x 2   mx 2 1  4ax 2 
2 2

V   mgy   mgax 2 . Since particle is moving downward direction so potential V is

negative.

 L  T V  L 
1
2
 
m 1  4a 2 x 2 x 2  mgax 2

Q15. The Lagrange’s equation of motion of the particle for above question is given by
(a) x  2 gax (b) m 1  4a 2 x 2  
x  2mgax  4ma 2 xx 2

 
(c) m 1  4a 2 x 2 x  2mgax  4ma 2 xx 2 (d) x  2 gax
Ans: (c)
d  dL  dL
Solution:    m(1  4a 2 x 2 ) 
x  4ma 2 xx 2  2mgax
dt  dx  dx

GATE- 2013
Q16. In the most general case, which one of the following quantities is NOT a second order
tensor?
(a) Stress (b) Strain
(c) Moment of inertia (d) Pressure
Ans: (b)
Solution: Strain is not a tensor.
Q17. An electron is moving with a velocity of 0.85c in the same direction as that of a moving
photon. The relative velocity of the electron with respect to photon is
(a) c (b)  c
(c) 0.15c (d)  0.15c
Ans: (b)

                                                                                
Head office  Branch office 
 
fiziks, H.No. 23, G.F, Jia Sarai,  Anand Institute of Mathematics, 
 
Near IIT, Hauz Khas, New Delhi‐16  28‐B/6, Jia Sarai, Near IIT 
 
Phone: 011‐26865455/+91‐9871145498 Hauz Khas, New Delhi‐16 
                                                   
                                             Website: www.physicsbyfiziks.com                                                                                          
                                                            Email: fiziks.physics@gmail.com                                                                   7 
fiziks
Institute for NET/JRF, GATE, IIT‐JAM, JEST, TIFR and GRE in PHYSICAL SCIENCES 
 
Q18. The Lagrangian of a system with one degree of freedom q is given by L  q 2  q 2 ,
where  and  are non-zero constants. If p q denotes the canonical momentum

conjugate to q then which one of the following statements is CORRECT?


(a) p q  2  q and it is a conserved quantity.

(b) p q  2  q and it is not a conserved quantity.

(c) p q  2 q and it is a conserved quantity.

(d) p q  2q and it is not a conserved quantity.

Ans: (d)
L L
Solution:  pq but 0
q q
Q19. The relativistic form of Newton’s second law of motion is

mc dv m c 2  v 2 dv
(a) F  (b) F 
c 2  v 2 dt c dt

mc 2 dv c 2  v 2 dv
(c) F  (d) F  m
c 2  v 2 dt c2 dt
Ans:
mv dP dv 1  1 1 2v dv
Solution: P  F m   mv     3/ 2
 2
v2 dt dt v2  2   v2  c dt
1 1 2  1 2 
c2 c  c 

   
 v 2 
2  2 
dv 1  1 dv  1 v 
F m 1 c m 2c 2
 
v 2  2 1  v   dt   v 2  3 2
2
dt 
1  2   
2 
c   c   1  c 2  
  

                                                                                
Head office  Branch office 
 
fiziks, H.No. 23, G.F, Jia Sarai,  Anand Institute of Mathematics, 
 
Near IIT, Hauz Khas, New Delhi‐16  28‐B/6, Jia Sarai, Near IIT 
 
Phone: 011‐26865455/+91‐9871145498 Hauz Khas, New Delhi‐16 
                                                   
                                             Website: www.physicsbyfiziks.com                                                                                          
                                                            Email: fiziks.physics@gmail.com                                                                   8 
fiziks
Institute for NET/JRF, GATE, IIT‐JAM, JEST, TIFR and GRE in PHYSICAL SCIENCES 
 
Q20. Consider two small blocks, each of mass M, attached to two identical springs. One of the
springs is attached to the wall, as shown in the figure. The spring constant of each spring
is k . The masses slide along the surface and the friction is negligible. The frequency of
one of the normal modes of the system is,

3 2 k
(a)
2 M

3 3 k
(b)
2 M
k k
3 5 k M M
(c)
2 M

3 6 k
(d)
2 M
Ans: (c)
1 2 1 2
Solution: T  mx1  mx 2 ,
2 2

V 
1 2 1
2
1 1 1

kx1  k  x 2  x1   kx12  k x 22  x12  2 x 2 x1  k 2 x 2  x 2  2 x 2 x1
2
2

2 2 2
  
m 0   2k k 
T  ; V   
 0 m  k k 

2k   2 m k 3 5 k
 0   2 k   2 m  k   2 m   k 2  0   
k k  m2
2 m

GATE- 2014
Q21. If the half-life of an elementary particle moving with speed 0.9c in the laboratory frame is
5  10 8 s, then the proper half-life is _______________ 10 8 s. c  3  10 8 m / s  
Ans: 2.18

                                                                                
Head office  Branch office 
 
fiziks, H.No. 23, G.F, Jia Sarai,  Anand Institute of Mathematics, 
 
Near IIT, Hauz Khas, New Delhi‐16  28‐B/6, Jia Sarai, Near IIT 
 
Phone: 011‐26865455/+91‐9871145498 Hauz Khas, New Delhi‐16 
                                                   
                                             Website: www.physicsbyfiziks.com                                                                                          
                                                            Email: fiziks.physics@gmail.com                                                                   9 
fiziks
Institute for NET/JRF, GATE, IIT‐JAM, JEST, TIFR and GRE in PHYSICAL SCIENCES 
 
t0 v2
Solution: t  , t0  t  1  2 = t0  5  10 8  .19  2.18 108 s
v2 c
1
c2
Q22. Two masses m and 3m are attached to the two ends of a massless spring with force
constant K . If m  100 g and K  0.3 N / m , then the natural angular frequency of
oscillation is ________ Hz .
Ans: 0.318

1 k m1.m2 3m.m 3m 4k
Solution: f       2  0.318 Hz
2  m1  m2 4m 4 3m
Q23. The Hamilton’s canonical equation of motion in terms of Poisson Brackets are
(a) q  q, H ; p  p, H  (b) q  H , q; p  H , p

(c) q  H , p; p  H , p (d) q  p, H ; p  q, H 


Ans: (a)
df f q f p f
Solution:  .  . 
dt q t p t t
df f H f H f df f
 .  .     f , H 
dt q p p q t dt t
dq dp
 q, H  and   p, H 
dt dt
Q24. A bead of mass m can slide without friction along a mass less rod kept at 45 o with the
vertical as shown in the figure. The rod is rotating about the vertical axis with a constant
angular speed  . At any instant r is the distance of the bead from the origin. The

momentum conjugate to r is

(a) mr
1
(b) mr
2
1 m
(c) mr
2 45 o
r
(d) 2mr

                                                                                
Head office  Branch office 
 
fiziks, H.No. 23, G.F, Jia Sarai,  Anand Institute of Mathematics, 
 
Near IIT, Hauz Khas, New Delhi‐16  28‐B/6, Jia Sarai, Near IIT 
 
Phone: 011‐26865455/+91‐9871145498 Hauz Khas, New Delhi‐16 
                                                   
                                             Website: www.physicsbyfiziks.com                                                                                          
                                                            Email: fiziks.physics@gmail.com                                                                   10 
fiziks
Institute for NET/JRF, GATE, IIT‐JAM, JEST, TIFR and GRE in PHYSICAL SCIENCES 
 
Ans: (a)
1
Solution: L  m(r 2  r 2 2  r 2 sin 2  2 )  mgr cos 
2

equation of constrain is   and it is given   
4
1 1 1
L m(r 2  r 2 2 )  mgr
2 2 2
L
the momentum conjugate to r is p r  = p r  mr
r
Q25. A particle of mass m is in a potential given by
a ar02
V r    
r 3r 3
when a and r0 are positive constants. When disturbed slightly from its stable equilibrium
position it undergoes a simple harmonic oscillation. The time period of oscillation is
3 3
mr03 m r0 2m r0 mr03
(a) 2 (b) 2 (c) 2 (d) 4
2a a a a
Ans: (a)
a ar02 V a 3ar02
Solution: V  r     3 for equilibrium   0 r   r0
r 3r r r 2 3r 4

 2V 2a 4ar02 2a 4ar02 2a
 3  5  3  5  3
r 2 r r r0
r0 r0 r0

 2V
r 2 mr03
  T  2
r0

m 2a
Q26. A planet of mass m moves in a circular orbit of radius r0 in the gravitational potential

k
V r    , where k is a positive constant. The orbit angular momentum of the planet is
r
(a) 2r0 km (b) 2r0 km (c) r0 km (d) r0 km

Ans: (d)
                                                                                
Head office  Branch office 
 
fiziks, H.No. 23, G.F, Jia Sarai,  Anand Institute of Mathematics, 
 
Near IIT, Hauz Khas, New Delhi‐16  28‐B/6, Jia Sarai, Near IIT 
 
Phone: 011‐26865455/+91‐9871145498 Hauz Khas, New Delhi‐16 
                                                   
                                             Website: www.physicsbyfiziks.com                                                                                          
                                                            Email: fiziks.physics@gmail.com                                                                   11 
fiziks
Institute for NET/JRF, GATE, IIT‐JAM, JEST, TIFR and GRE in PHYSICAL SCIENCES 
 
J 2
k dVeffect J 2
k
Solution: Veffctive  2
    3  2 =0 at r  r0
2mr r dr mr r
so J  r0 km

Q27. Given that the linear transformation of a generalized coordinate q and the corresponding
momentum p ,
Q  q  4ap
P  q  2p
is canonical, the value of the constant a is _________________
Ans: 0.5
Q P Q P
Solution: .  .  0  1.2  4a.1  0  a  0.5
q p p q

p2  q2
Q28. The Hamiltonian of particle of mass m is given by H   .which one of the
2m 2
following figure describes the motion of the particle in phase space?
(a) (b)
p p

q q

(c) (d)
p p

q q

Ans: (d)

                                                                                
Head office  Branch office 
 
fiziks, H.No. 23, G.F, Jia Sarai,  Anand Institute of Mathematics, 
 
Near IIT, Hauz Khas, New Delhi‐16  28‐B/6, Jia Sarai, Near IIT 
 
Phone: 011‐26865455/+91‐9871145498 Hauz Khas, New Delhi‐16 
                                                   
                                             Website: www.physicsbyfiziks.com                                                                                          
                                                            Email: fiziks.physics@gmail.com                                                                   12 
fiziks
Institute for NET/JRF, GATE, IIT‐JAM, JEST, TIFR and GRE in PHYSICAL SCIENCES 
 
GATE- 2015
Q29. A satellite is moving in a circular orbit around the Earth. If T ,V and E are its average
kinetic, average potential and total energies, respectively, then which one of the
following options is correct?
(a) V  2T ; E  T (b) V  T ; E  0
T T  3T T
(c) V   ;E  (d) V  ;E 
2 2 2 2
Ans.: (a)
n 1
Solution: From Virial theorem T  V where V  r n 1
2
k 1
V   V   n  2  V  2 T
r r

Q30. In an inertial frame S , two events A and B take place at ct A  0, rA  0  and
ct B  0, rB  2 yˆ  , respectively. The times at which these events take place in a frame
S  moving with a velocity 0.6cyˆ with respect to S are given by
3
(a) ct A  0; ct B   (b) ct A  0; ct B  0
2
3 1
(c) ct A  0; ct B  (d) ct A  0; ct B 
2 2
Ans.: (a)
Solution: Velocity of S ' with respect to S is v  .6c
v
tA  y
t A'  c2 for event A t A  0, y  0 so ct A'  0
v2
1 2
c
v
tB  y
c2 3
t B'  for event B t B  0, y  2 so ct B'  
v2 2
1 2
c

                                                                                
Head office  Branch office 
 
fiziks, H.No. 23, G.F, Jia Sarai,  Anand Institute of Mathematics, 
 
Near IIT, Hauz Khas, New Delhi‐16  28‐B/6, Jia Sarai, Near IIT 
 
Phone: 011‐26865455/+91‐9871145498 Hauz Khas, New Delhi‐16 
                                                   
                                             Website: www.physicsbyfiziks.com                                                                                          
                                                            Email: fiziks.physics@gmail.com                                                                   13 
fiziks
Institute for NET/JRF, GATE, IIT‐JAM, JEST, TIFR and GRE in PHYSICAL SCIENCES 
 
 
Q31. The Lagrangian for a particle of mass m at a position r moving with a velocity v is given
m  
by L  v 2  Cr .v  V r  , where V r  is a potential and C is a constant. If pc is the
2
canonical momentum, then its Hamiltonian is given by
1 
(a)  pc  Cr 2  V r  (b)
1 
 pc  Cr 2  V r 
2m 2m
pc2 1 2
(c)  V r  (d) pc  C 2 r 2  V r 
2m 2m
Ans.: (b)
m 2  
Solution: L  v  Cr.v  V  r  where v  r
2
m 2
H   r pc  L  rp
 c L where L  r  Cr.r  V  r 
2
L p  Cr
  pc   mr  Cr   r  c
r m
2
 p  Cr  m  pc  Cr   pc  Cr 
 H  c  pc     cr   V r 
 m  2 m   m 
2
 p  Cr  m  pc  Cr 
 H  c   pc  Cr     V r 
 m  2 m 

 p  Cr   p  Cr 
2 2
1
V r   pc  Cr   V  r 
2
H  c  c H 
m 2m 2m
 
Q32. The Hamiltonian for a system of two particles of masses m1 and m2 at r1 and r2 having
  1 1 C  
velocities v1 and v2 is given by H  m1v12  m2v22    2 zˆ   r1  r2  , wrong where
2 2  r1  r2 
C is constant. Which one of the following statements is correct?
(a) The total energy and total momentum are conserved
(b) Only the total energy is conserved
(c) The total energy and the z - component of the total angular momentum are conserved
(d) The total energy and total angular momentum are conserved
Ans.: (c)

                                                                                
Head office  Branch office 
 
fiziks, H.No. 23, G.F, Jia Sarai,  Anand Institute of Mathematics, 
 
Near IIT, Hauz Khas, New Delhi‐16  28‐B/6, Jia Sarai, Near IIT 
 
Phone: 011‐26865455/+91‐9871145498 Hauz Khas, New Delhi‐16 
                                                   
                                             Website: www.physicsbyfiziks.com                                                                                          
                                                            Email: fiziks.physics@gmail.com                                                                   14 
fiziks
Institute for NET/JRF, GATE, IIT‐JAM, JEST, TIFR and GRE in PHYSICAL SCIENCES 
 
 
Solution: Lagrangian is not function of time so energy is conserve and component of  r1  r2  are

Only in ẑ direction means potential is symmetric under  so Lz is conserve.

Q33. A particle of mass 0.01 kg falls freely in the earth’s gravitational field with an initial

velocity 0  10ms 1 . If the air exerts a frictional force of the form, f  kv , then for

k  0.05 Nm 1 s , the velocity (in ms 1 ) at time t  0.2 s is _________ (upto two decimal

places). (use g  10 ms 2 and e  2.72 )


Ans.: Data given is incorrect
u 0.2
dv dv k dv dv
Solution: m  mg  kv  g v   dt     dt
dt dt m k k
g v 10 g v 0
m m
u
m  k  m   k   10k  
   ln  g  v    t 0    ln  g  u    ln  g 
0.2
   0.2
k   m  10 k   m   m 

m  0.05   .05  
 ln 10  u   ln 10  10     0.2
k  0.01   .01  
m

k
ln 10  5u   ln  40   0.2
 ln  40  can not be defined. So given data are not correct.

Q34. Consider the motion of the Sun with respect to the rotation of the Earth about its axis. If
 
Fc and FCo denote the centrifugal and the Coriolis forces, respectively, acting on the
Sun, then
  
(a) Fc is radially outward and FCo  Fc
  
(b) Fc is radially inward and FCo  2 Fc
  
(c) Fc is radially outward and FCo  2 Fc
  
(d) Fc is radially outward and FCo  2 Fc
Ans.: (b)

                                                                                
Head office  Branch office 
 
fiziks, H.No. 23, G.F, Jia Sarai,  Anand Institute of Mathematics, 
 
Near IIT, Hauz Khas, New Delhi‐16  28‐B/6, Jia Sarai, Near IIT 
 
Phone: 011‐26865455/+91‐9871145498 Hauz Khas, New Delhi‐16 
                                                   
                                             Website: www.physicsbyfiziks.com                                                                                          
                                                            Email: fiziks.physics@gmail.com                                                                   15 
fiziks
Institute for NET/JRF, GATE, IIT‐JAM, JEST, TIFR and GRE in PHYSICAL SCIENCES 
 
Q35. A particle with rest mass M is at rest and decays into two particles of equal rest masses
3
M which move along the z axis. Their velocities are given by
10
   
(a) v1  v 2  0.8c zˆ (b) v1  v 2  0.8c zˆ
   
(c) v1  v 2  0.6c zˆ (d) v1  0.6c zˆ; v 2   0.8c zˆ
Ans.: (b)
3 3
Solution: M M M
10 10
From momentum conservation
   
0  P1  P 2  P1   P 2  P1  P2
From energy conservation E  E1  E2

3 Mc 2 3 Mc 2 3 Mc 2
 Mc 2    Mc 2 
10 v 2 10 v2 5 v2
1 2 1 2 1 2
c c c

 v2  9 v 2 16
1  2    2   v  0.8c
 v  25 v 25

GATE-2016
Q36. The kinetic energy of a particle of rest mass m0 is equal to its rest mass energy. Its

momentum in units of m0 c , where c is the speed of light in vacuum, is _______.


(Give your answer upto two decimal places
Ans. : 1.73

m0c 2
Solution:  2m0 c 2  E
2
v
1
c2
E 2  p 2 c 2  m02 c 4  4m02c 4  m02c 4  p 2 c 2  p  3m0 c  1.732m0 c

                                                                                
Head office  Branch office 
 
fiziks, H.No. 23, G.F, Jia Sarai,  Anand Institute of Mathematics, 
 
Near IIT, Hauz Khas, New Delhi‐16  28‐B/6, Jia Sarai, Near IIT 
 
Phone: 011‐26865455/+91‐9871145498 Hauz Khas, New Delhi‐16 
                                                   
                                             Website: www.physicsbyfiziks.com                                                                                          
                                                            Email: fiziks.physics@gmail.com                                                                   16 
fiziks
Institute for NET/JRF, GATE, IIT‐JAM, JEST, TIFR and GRE in PHYSICAL SCIENCES 
 
Q37. In an inertial frame of reference S , an observer finds two events occurring at the same
time at coordinates x1  0 and x 2  d A different inertial frame S  moves with velocity
v with respect to S along the positive x -axis. An observer in S  also notices these two
events and finds them to occur at times t1 and t 2 and at positions x1 and x2 respectively.
1
If t   t 2  t1 , x   x 2  x1 and   , which of the following statements is true?
v2
1 2
c
d
(a) t   0, x   d (b) t   0, x  

 vd  vd d
(c) t   , x   d (d) t   , x  
c2 c 2

Ans.: (c)
 vx   vx 
 t2  22   t1  21 
Solution: t2'  t1'   c  c   t '  t   vx it is given t  0, x  d
 v2   v2  c2
 1  2   1  2 
 c   c 
 vx
 t '  
c2
   
   
x  vt2    x1  vt1
x2  x1   2
' '   x '    x  vt  it is given t  0, x  d
 v2   v2 
 1  2   1  2 
 c   c 
 x '   d
Q38. The Lagrangian of a system is given by

L
1 2 2
2
 
ml   sin 2  2  mgl cos  , where m, l and g are constants.

Which of the following is conserved?


 
(a)  sin 2  (b)  sin  (c) (d)
sin  sin 2 

                                                                                
Head office  Branch office 
 
fiziks, H.No. 23, G.F, Jia Sarai,  Anand Institute of Mathematics, 
 
Near IIT, Hauz Khas, New Delhi‐16  28‐B/6, Jia Sarai, Near IIT 
 
Phone: 011‐26865455/+91‐9871145498 Hauz Khas, New Delhi‐16 
                                                   
                                             Website: www.physicsbyfiziks.com                                                                                          
                                                            Email: fiziks.physics@gmail.com                                                                   17 
fiziks
Institute for NET/JRF, GATE, IIT‐JAM, JEST, TIFR and GRE in PHYSICAL SCIENCES 
 
Ans.: (a)
L
Solution:  is cyclic coordinate so  p  ml 2 sin 2  is constant hence m, l and g are


constants. Then  sin 2 


Q39. A particle of rest mass M is moving along the positive x -direction. It decays into two
photons  1 and  2 as shown in the figure. The energy of  1 is 1 GeV and the energy of
GeV
 2 is 0.82 GeV . The value of M (in units of ) is ________. (Give your answer
c2
1
upto two decimal places)

M 450
600

2
Ans.: 1.40

Solution: p 2 c 2  M 2 c 4  E1  E2  1.82GeV

E1 E 1GeV 1 .82GeV 1 1.11GeV


p cos 1  2 cos  2   
c c c 2 c 2 c

 p 2 c 2  m 2c 4  3.312

 m 2 c 4  3.312  1.21  2.077


 m  2.076  1.40

                                                                                
Head office  Branch office 
 
fiziks, H.No. 23, G.F, Jia Sarai,  Anand Institute of Mathematics, 
 
Near IIT, Hauz Khas, New Delhi‐16  28‐B/6, Jia Sarai, Near IIT 
 
Phone: 011‐26865455/+91‐9871145498 Hauz Khas, New Delhi‐16 
                                                   
                                             Website: www.physicsbyfiziks.com                                                                                          
                                                            Email: fiziks.physics@gmail.com                                                                   18 
fiziks
Institute for NET/JRF, GATE, IIT‐JAM, JEST, TIFR and GRE in PHYSICAL SCIENCES 
 
GATE- 2017

2
1  dq  1  dq 
Q40. If the Lagrangian L0  m    m 2 q 2 is modified to L  L0   q   , which one
2  dt  2  dt 
of the following is TRUE?
(a) Both the canonical momentum and equation of motion do not change
(b) Canonical momentum changes, equation of motion does not change
(c) Canonical momentum does not change, equation of motion changes
(d) Both the canonical momentum and equation of motion change
Ans. : (b)
2
1  dq  1
Solution: For lagrangian L0  m    m 2 q 2 canonical momentum is p  mq and
2  dt  2

d  L   L 
       0 mq  m q  0
2
equation of motion is given by
dt  q   q 
2
 dq  1  dq  1
For langrangian L  L0   q    L  m    m 2 q 2   qq
 dt  2  dt  2
Canonical momentum is p  mq   q

d  L   L 
      0 mq  m q  0
2
Equation of motion
dt  q   q 
Q41. Two identical masses of 10 gm each are connected by a massless spring of spring
constant 1 N / m . The non-zero angular eigenfrequency of the system is…………rad/s.
(up to two decimal places)
Ans. : 14.14

k m 10 1
Solution:   , where     and k  1N / m   14.14
 2 2 1000 200

                                                                                
Head office  Branch office 
 
fiziks, H.No. 23, G.F, Jia Sarai,  Anand Institute of Mathematics, 
 
Near IIT, Hauz Khas, New Delhi‐16  28‐B/6, Jia Sarai, Near IIT 
 
Phone: 011‐26865455/+91‐9871145498 Hauz Khas, New Delhi‐16 
                                                   
                                             Website: www.physicsbyfiziks.com                                                                                          
                                                            Email: fiziks.physics@gmail.com                                                                   19 
fiziks
Institute for NET/JRF, GATE, IIT‐JAM, JEST, TIFR and GRE in PHYSICAL SCIENCES 
 
Q42. The phase space trajectory of an otherwise free particle bouncing between two hard walls
elastically in one dimension is a
(a) straight line (b) parabola
(c) rectangle (d) circle
Ans. : (c)

p2
Solution: E  p   2mE
2m
Q43. The Poisson bracket  x, xp y  ypx  is equal to

(a)  x (b) y (c) 2 px (d) p y

Ans. : (b)
Solution:  x, xp y  ypx    x, xp y    x, ypx   0  y  x, px   y

c
Q44. An object travels along the x -direction with velocity in a frame O . An observer in a
2
c
frame O sees the same object travelling with velocity . The relative velocity of O
4
with respect to O in units of c is…………….. (up to two decimal places).
Ans. : 0.28

c c u'  v
Solution: u x'  , v  u x  x '
2 4 uv
1  x2
c
c c

ux  2 4  2c  0.28c
c c 1 7
1 . . 2
2 4 c
Q45. A uniform solid cylinder is released on a horizontal surface with speed 5 m / s without
any rotation (slipping without rolling). The cylinder eventually starts rolling without
slipping. If the mass and radius of the cylinder are 10 gm and 1cm respectively, the final
linear velocity of the cylinder is……………… m / s . (up to two decimal places).
Ans. : 3.33
                                                                                
Head office  Branch office 
 
fiziks, H.No. 23, G.F, Jia Sarai,  Anand Institute of Mathematics, 
 
Near IIT, Hauz Khas, New Delhi‐16  28‐B/6, Jia Sarai, Near IIT 
 
Phone: 011‐26865455/+91‐9871145498 Hauz Khas, New Delhi‐16 
                                                   
                                             Website: www.physicsbyfiziks.com                                                                                          
                                                            Email: fiziks.physics@gmail.com                                                                   20 
fiziks
Institute for NET/JRF, GATE, IIT‐JAM, JEST, TIFR and GRE in PHYSICAL SCIENCES 
 
1 v 3 2 10
Solution: mvr  mvcm r  I cm  mvcm r  mr 2 cm  v  vcm  vcm  v   3.33m / sec
2 r 2 3 3
Q46. A person weighs wp at Earth’s north pole and we at the equator. Treating the Earth as a

perfect sphere of radius 6400 km , the value 100 


w p  we 
is………….. (up to two
wp

decimal places). (Take g  10 ms 2 ).


Ans. : 0.33
Solution: g p  g

ge  g   2 R

wp  we 2R
100  
wp g

Put the value of g  10 m / sec 2


2 2
 
T 24  3600
R  6400  103
wp  we
Then 100   0.33
wp

                                                                                
Head office  Branch office 
 
fiziks, H.No. 23, G.F, Jia Sarai,  Anand Institute of Mathematics, 
 
Near IIT, Hauz Khas, New Delhi‐16  28‐B/6, Jia Sarai, Near IIT 
 
Phone: 011‐26865455/+91‐9871145498 Hauz Khas, New Delhi‐16 
                                                   
                                             Website: www.physicsbyfiziks.com                                                                                          
                                                            Email: fiziks.physics@gmail.com                                                                   21 
fiziks
Institute for NET/JRF, GATE, IIT‐JAM, JEST, TIFR and GRE in PHYSICAL SCIENCES 
 
ELECTROMAGNETIC THEORY SOLUTIONS
GATE- 2010
Q1. An insulating sphere of radius a carries a charge density


 r   0 a 2  r 2 cos  ; r  a .
The leading order term for the electric field at a distance d, far away from the charge
distribution, is proportional to
(a) d-1 (b) d-2 (c) d-3 (d) d-4
Ans: (c)
1 1 
Solution: V r     d  2   cos d   ,
r V r 
a  2
Ist term,   d     0  a  r  cos   r sin  drd d  0
2 2 2

0 0 0

a  2
IInd term,   cos  d     0  a  r  cos   r sin  drd d  0 .
2 2 2 2

0 0 0

1 1
 V 2
 E 3
r r
Q2. Two magnetic dipoles of magnitude m each are placed in a plane as shown in figure.
m
The energy of interaction is given by
45 o 2
0 m 2
(a) Zero (b)
4d 3 d
3 0 m 2
3 0 m 2 45 o
(c) (d) 
2d 3 8d 3 m 1
Ans: (d)
0 
Solution: U  m1  m2  3m 1  rˆ m 2  rˆ ,
4r 3


   
Since m1  m2  m1  m2  0  U  0 3  3  m cos 45 0  m cos 45 0
4d
 

                                                                                
Head office  Branch office 
 
fiziks, H.No. 40 D, G.F, Jia Sarai,  Anand Institute of Mathematics, 
 
Near IIT, Hauz Khas, New Delhi‐16  28‐B/6, Jia Sarai, Near IIT 
 
Phone: 011‐26865455/+91‐9871145498 Hauz Khas, New Delhi‐16 
                                                   
                                             Website: www.physicsbyfiziks.com                                                                                          
                                                            Email: fiziks.physics@gmail.com                                                                   1 
fiziks
Institute for NET/JRF, GATE, IIT‐JAM, JEST, TIFR and GRE in PHYSICAL SCIENCES 
 
3 m 2
U   0 3 .
8 d
Statement for Linked Answer Questions 3 and 4:
Consider the propagation of electromagnetic waves in a linear, homogenous and isotropic
material medium with electric permittivity ε and magnetic permeability μ.

Q3. For a plane wave of angular frequency ω and propagation vector k propagating in the
medium Maxwell’s equations reduce to

(a) k  E  0; k  H  0; k  E   H ; k  H   E

(b) k  E  0; k  H  0; k  E   H ; k  H   E

(c) k  E  0; k  H  0; k  E   H ; k  H   E

(d) k  E  0; k  H  0; k  E   H ; k  H   E
Ans: (d)
Q4. If ε and μ assume negative values in a certain frequency range, then the directions of the

propagation vector k and the Poynting vector S in that frequency range are related as

(a) k and S are parallel

(b) k and S are anti-parallel

(c) k and S are perpendicular to each other

(d) k and S makes an angle that depends on the magnitude of |ε| and |μ|
Ans: (a)

                                                                                
Head office  Branch office 
 
fiziks, H.No. 40 D, G.F, Jia Sarai,  Anand Institute of Mathematics, 
 
Near IIT, Hauz Khas, New Delhi‐16  28‐B/6, Jia Sarai, Near IIT 
 
Phone: 011‐26865455/+91‐9871145498 Hauz Khas, New Delhi‐16 
                                                   
                                             Website: www.physicsbyfiziks.com                                                                                          
                                                            Email: fiziks.physics@gmail.com                                                                   2 
fiziks
Institute for NET/JRF, GATE, IIT‐JAM, JEST, TIFR and GRE in PHYSICAL SCIENCES 
 
Q5. Consider a conducting loop of radius a and total loop resistance R placed in a region with
a magnetic field B thereby enclosing a flux 0. The loop is connected to an electronic
circuit as shown, the capacitor being initially uncharged

 C


 

   
 Vout

 



If the loop is pulled out of the region of the magnetic field at a constant speed u, the final
output voltage Vout is independent of
(a) 0 (b) u (c) R (d) C
Ans: (a)
GATE-2011

Q6. If a force F is derivable from a potential function V(r), where r is the distance from the
origin of the coordinate system, it follows that

(a)   F  0 (b)   F  0 (c) V  0 (d)  2 V  0


Ans: (a)
Q7. Tow charges q and 2q are placed along the x-axis in front of a grounded, infinite
conducting plane, as shown in the figure. They
are located respectively at a distance of 0.5 m and
1.5 m from the plane. The force acting on the
0 .5 m q 2q
charge q is   x
1 .5 m
7q 2
1 1 2
(a) (b) 2q
4 0 2 4 0

1 1 q2
(c) q2 (d)
4 0 4 0 2
Ans: (a)
Solution: Using method of Images we can draw equivalent figure as shown below:

                                                                                
Head office  Branch office 
 
fiziks, H.No. 40 D, G.F, Jia Sarai,  Anand Institute of Mathematics, 
 
Near IIT, Hauz Khas, New Delhi‐16  28‐B/6, Jia Sarai, Near IIT 
 
Phone: 011‐26865455/+91‐9871145498 Hauz Khas, New Delhi‐16 
                                                   
                                             Website: www.physicsbyfiziks.com                                                                                          
                                                            Email: fiziks.physics@gmail.com                                                                   3 
fiziks
Institute for NET/JRF, GATE, IIT‐JAM, JEST, TIFR and GRE in PHYSICAL SCIENCES 
 
 2q q 0 .5 m 0 .5 m q 2q
    x
1 .5 m 1 .5 m

q  2q q 2q  q 7q 1 7q 2
F       
4 0  12 12  2 2  4 0 2 4 0 2

Q8. A uniform surface current is flowing in the positive y-direction over an infinite sheet
lying in x-y plane. The direction of the magnetic field is
(a) along iˆ for z > 0 and along  iˆ for z < 0
(b) along k̂ for z > 0 and along  k̂ for z < 0
(c) along  iˆ for z > 0 and along iˆ for z < 0
(d) along  k̂ for z > 0 and along k̂ for z < 0
Ans: (a)
Q9. A magnetic dipole of dipole moment m is placed in a non-uniform magnetic field B . If

the position vector of the dipole is r , the torque acting on the dipole about the origin is

(a) r  m  B  (b) r   m  B 
(c) m  B (d) m  B  r   m  B  
Ans: (c)

Q10. A spherical conductor of radius a is placed in a uniform electric field E  E 0 kˆ . The


potential at a point P(r, θ) for r > a, is given by
E0 a 3
Φ(r, θ) = constant – E 0 r cos   cos 
r2
where r is the distance of P from the centre O of the sphere and θ is the angle OP makes
with the z-axis P
The charge density on the sphere at θ = 30o is r
(a) 3 3 0 E 0 / 2 (b) 3 0 E 0 / 2 
O k̂

                                                                                
Head office  Branch office 
 
fiziks, H.No. 40 D, G.F, Jia Sarai,  Anand Institute of Mathematics, 
 
Near IIT, Hauz Khas, New Delhi‐16  28‐B/6, Jia Sarai, Near IIT 
 
Phone: 011‐26865455/+91‐9871145498 Hauz Khas, New Delhi‐16 
                                                   
                                             Website: www.physicsbyfiziks.com                                                                                          
                                                            Email: fiziks.physics@gmail.com                                                                   4 
fiziks
Institute for NET/JRF, GATE, IIT‐JAM, JEST, TIFR and GRE in PHYSICAL SCIENCES 
 
(c) 3 0 E 0 / 2 (d))  0 E 0 / 2
Ans: (a)

V  2E a 3 
Solution:    0   0  E 0 cos   03 cos   .
r r a  r  r a

3 3
   0  E 0 cos   2 E 0 cos      3E 0  0 cos   3E 0  0 cos 30 0   0 E0
2
Q11. Which of the following expressions for a vector potential A DOES NOT represent a
uniform magnetic field of magnitude B0 along the z-direction?
(a) A  0, B0 x,0 (b) A   B0 y,0,0 

 B0 x B0 y   B0 y B0 x 
(c) A   , ,0  (d) A    , ,0 
 2 2   2 2 
Ans: (c)
 
Solution: B    A .

Statement for Linked Questions 12 and 13:


A plane electromagnetic wave has the magnetic field given by
 k 
B x, y, z , t   B0 sin x  y    t  kˆ
 2 

where k is the wave number and iˆ, ˆj and kˆ are the Cartesian unit vectors in x, y and z
directions respectively.

Q12. The electric field E  x, y, z , t  corresponding to the above wave is given by


(a) cB0 sin  x  y 
k  iˆ  ˆj
  t
  
(b) cB0 sin  x  y 
k  iˆ  ˆj
  t
 
 2  2  2  2
 k   k 
(c) cB0 sin  x  y    t  iˆ (d) cB0 sin  x  y    t  ˆj
 2   2 
Ans: (a)

                                                                                
Head office  Branch office 
 
fiziks, H.No. 40 D, G.F, Jia Sarai,  Anand Institute of Mathematics, 
 
Near IIT, Hauz Khas, New Delhi‐16  28‐B/6, Jia Sarai, Near IIT 
 
Phone: 011‐26865455/+91‐9871145498 Hauz Khas, New Delhi‐16 
                                                   
                                             Website: www.physicsbyfiziks.com                                                                                          
                                                            Email: fiziks.physics@gmail.com                                                                   5 
fiziks
Institute for NET/JRF, GATE, IIT‐JAM, JEST, TIFR and GRE in PHYSICAL SCIENCES 
 
c  k  xˆ  yˆ    x  y k
 c 
Solution: E   k  B   
k k
   B0 sin 
 
 t  zˆ 
2  2  
  k  xˆ  yˆ 
E  cB0 sin  x  y   t 
 2  2

Q13. The average Poynting vector is given by

(a)

cB02 iˆ  ˆj  (b) 
cB02 iˆ  ˆj   (c)
cB02 iˆ  ˆj  (d) 
cB02 iˆ  ˆj  
2 0 2 2 0 2 2 0 2 2 0 2
Ans: (d)
 cB02 cB02  xˆ  yˆ   cB02  xˆ  yˆ 
Solution: S  ˆ
k     
2 0 2 0  2  2 0  2 

GATE-2012
Q14. The space-time dependence of the electric field of a linearly polarized light in free space
is given by xE0 cost  kz  where E0, ω and k are the amplitude, the angular frequency
and the wavevector, respectively. The time average energy density associated with the
electric field is
1 1
(a)  0 E 02 (b)  0 E 02 (c)  0 E 02 (d) 2 0 E 02
4 2
Ans: (a)
1 1 1
Solution: u E   0 E 2   0 E 2 cos 2 wt  kz   u E   0 E 02
2 2 4
Q15. A plane electromagnetic wave traveling in free space is incident normally on a glass plate
of refractive index 3/2. If there is no absorption by the glass, its reflectivity is
(a) 4% (b) 16% (c) 20% (d) 50%
Ans: (a)

                                                                                
Head office  Branch office 
 
fiziks, H.No. 40 D, G.F, Jia Sarai,  Anand Institute of Mathematics, 
 
Near IIT, Hauz Khas, New Delhi‐16  28‐B/6, Jia Sarai, Near IIT 
 
Phone: 011‐26865455/+91‐9871145498 Hauz Khas, New Delhi‐16 
                                                   
                                             Website: www.physicsbyfiziks.com                                                                                          
                                                            Email: fiziks.physics@gmail.com                                                                   6 
fiziks
Institute for NET/JRF, GATE, IIT‐JAM, JEST, TIFR and GRE in PHYSICAL SCIENCES 
 
2 2
 n  n2  1 3/ 2  1 4
Solution: R   1        .04 or 4%
 n1  n2  1 3/ 2  4 25

Q16. The electric and the magnetic field E  z , t  and B  z , t  , respectively corresponding to the

scalar potential   z , t   0 and vector potential A z , t   iˆtz are
   
(a) E  iˆz and B  -ĵt (b) E  iˆz and B  ĵt
   
(c) E  iˆz and B  -ĵt (d) E  iˆz and B  -ĵt
Ans: (d)
 
  A A   
Solution: E      iˆz , B    A   ˆjt .
t t
Q17. A plane polarized electromagnetic wave in free space at time t=0 is given
 
by E  x, y   10 ˆj expi6 x  8 z  . The magnetic field B x, z , t  is given by
 1
 
(a) B x, z , t   6kˆ  8iˆ expi 6 x  8 z  10ct 
c
 1
 
(b) B x, z , t   6kˆ  8iˆ expi 6 x  8 z  10ct 
c
 1
 
(c) B x, z , t   6kˆ  8iˆ expi 6 x  8 z  ct 
c
 1
 
(d) B x, z , t   6kˆ  8iˆ expi 6 x  8 z  ct 
c
Ans: (a)

 1  k  
 1

Solution: B  kˆ  E    E  
c c k
 1  6iˆ  8kˆ 
c  10 
 10


ˆjexp i k .r  t 

 
 1
 
B  6kˆ  8iˆ expi 6 x  8 z  10ct ,   10c.
c

                                                                                
Head office  Branch office 
 
fiziks, H.No. 40 D, G.F, Jia Sarai,  Anand Institute of Mathematics, 
 
Near IIT, Hauz Khas, New Delhi‐16  28‐B/6, Jia Sarai, Near IIT 
 
Phone: 011‐26865455/+91‐9871145498 Hauz Khas, New Delhi‐16 
                                                   
                                             Website: www.physicsbyfiziks.com                                                                                          
                                                            Email: fiziks.physics@gmail.com                                                                   7 
fiziks
Institute for NET/JRF, GATE, IIT‐JAM, JEST, TIFR and GRE in PHYSICAL SCIENCES 
 

Q18. Two infinitely extended homogeneous isotopic dielectric media (medium-1and medium-2
1 
with dielectric constant  2 and 2  5 , respectively)
0 0
medium - 1
meet at the z = 0 plane as shown in the figure. A uniform
electric field exists everywhere. For z ≥ 0, the electric field
 medium - 2 z=0
is given by E1  2iˆ  3 ˆj  5kˆ . The interface separating the
two media is charge free. The electric displacement vector
in the medium-2 is given by


(a) D 2   0 10iˆ  15 ˆj  10kˆ  
(b) D 2   0 10iˆ  15 ˆj  10kˆ 
(c) D 2   4iˆ  6 ˆj  10kˆ 
0 (d) D 2   4iˆ  6 ˆj  10kˆ 
0

Ans: (b)
 E1  E 2  E 2  2iˆ  3 ˆj

1  2  5 ˆ 
and  f  0  D1  D2  E 2  E1  k  2kˆ  E 2  2iˆ  3 ˆj  2kˆ
2 5
 

 D2   2 E 2   0 10iˆ  15 ˆj  10kˆ . 
GATE-2013
Q19. At a surface current, which one of the magnetostatic boundary condition is NOT
CORRECT?
(a) Normal component of the magnetic field is continuous.
(b) Normal component of the magnetic vector potential is continuous.
(c) Tangential component of the magnetic vector potential is continuous.
(d) Tangential component of the magnetic vector potential is not continuous.
Ans: (d)

                                                                                
Head office  Branch office 
 
fiziks, H.No. 40 D, G.F, Jia Sarai,  Anand Institute of Mathematics, 
 
Near IIT, Hauz Khas, New Delhi‐16  28‐B/6, Jia Sarai, Near IIT 
 
Phone: 011‐26865455/+91‐9871145498 Hauz Khas, New Delhi‐16 
                                                   
                                             Website: www.physicsbyfiziks.com                                                                                          
                                                            Email: fiziks.physics@gmail.com                                                                   8 
fiziks
Institute for NET/JRF, GATE, IIT‐JAM, JEST, TIFR and GRE in PHYSICAL SCIENCES 
 
Q20. Interference fringes are seen at an observation plane z  0 , by the superposition of two
 
  
 
plane waves A1 exp i k1  r  t and A2 exp i k 2  r  t , where A1 and A2 are real 
amplitudes. The condition for interference maximum is
     
 
(a) k1  k 2  r  2m  1 (b) k1  k 2  r  2m  
     
 
(c) k1  k 2  r  2m  1 (d) k1  k 2  r  2m  
Ans: (b)

Q21. For a scalar function  satisfying the Laplace equation,  has


(a) zero curl and non-zero divergence
(b) non-zero curl and zero divergence
(c) zero curl and zero divergence
(d) non-zero curl and non-zero divergence
Ans: (c)

 
 2  0  .   0 and      0 .  
Q22. A circularly polarized monochromatic plane wave is incident on a dielectric interface at
Brewaster angle. Which one of the following statements is correct?
(a) The reflected light is plane polarized in the plane of incidence and the transmitted
light is circularly polarized.
(b) The reflected light is plane polarized perpendicular to the plane of incidence and the
transmitted light is plane polarized in the plane of incidence.
(c) The reflected light is plane polarized perpendicular to the plane of incidence and the
transmitted light is elliptically polarized.
(d) There will be no reflected light and the transmitted light is circularly polarized.
Ans: (c)

                                                                                
Head office  Branch office 
 
fiziks, H.No. 40 D, G.F, Jia Sarai,  Anand Institute of Mathematics, 
 
Near IIT, Hauz Khas, New Delhi‐16  28‐B/6, Jia Sarai, Near IIT 
 
Phone: 011‐26865455/+91‐9871145498 Hauz Khas, New Delhi‐16 
                                                   
                                             Website: www.physicsbyfiziks.com                                                                                          
                                                            Email: fiziks.physics@gmail.com                                                                   9 
fiziks
Institute for NET/JRF, GATE, IIT‐JAM, JEST, TIFR and GRE in PHYSICAL SCIENCES 
 
Q23. A charge distribution has the charge density given by   Q x  x 0     x  x0  . For

this charge distribution the electric field at 2 x0 ,0,0

2Qxˆ Qxˆ Qxˆ Qxˆ


(a) (b) (c) (d)
9 0 x02 4 0 x03 4 0 x 02 16 0 x02
Ans:

Solution: Potential V  r  
1   x

a '

dx  
a  x'
 
a  x'
    
   a x3    
2
x dx x dx ....
4 0   a x x2 
  a 

First term, total charge


x0 x0

QT     x dx  Q    x   x 0 dx   Q    x   x 0 dx   Q  Q  0


 x0  x0

Second term, dipole moment


x0 x0

p   x   x dx  Q  x   x   x 0 dx   Q  x  x   x 0 dx   Qx 0  Q   x 0  2Qx 0


 x0  x0

2Qx 0 V 4Qx 0 4Qx 0 Q


V E xˆ  xˆ  xˆ  xˆ
4 0 x x 4 0 x 4 0 2 x 0  8 0 x 20
2 3 3

Q24. A monochromatic plane wave at oblique incidence undergoes reflection at a dielectric


interface. If kˆi , kˆr and n̂ are the unit vectors in the directions of incident wave, reflected
wave and the normal to the surface respectively, which one of the following expressions
is correct?
 
(a) kˆi  kˆr  nˆ  0  
(b) kˆi  kˆr  nˆ  0

(c) kˆ  nˆ  kˆ
i r 0 (d) kˆ  nˆ  kˆ
i r 0
Ans: (c)

                                                                                
Head office  Branch office 
 
fiziks, H.No. 40 D, G.F, Jia Sarai,  Anand Institute of Mathematics, 
 
Near IIT, Hauz Khas, New Delhi‐16  28‐B/6, Jia Sarai, Near IIT 
 
Phone: 011‐26865455/+91‐9871145498 Hauz Khas, New Delhi‐16 
                                                   
                                             Website: www.physicsbyfiziks.com                                                                                          
                                                            Email: fiziks.physics@gmail.com                                                                   10 
fiziks
Institute for NET/JRF, GATE, IIT‐JAM, JEST, TIFR and GRE in PHYSICAL SCIENCES 
 
Q25. In a constant magnetic field of 0.6 Tesla along the z direction, find the value of the path

integral  A  dl in the units of (Tesla m 2 ) on a square loop of side length 1 / 2 meters.  
The normal to the loop makes an angle of 60 0 to the z-axis, as shown in the figure. The
answer should be up to two decimal places. ___________

60 o

Ans: 0.15

 
2
  1  1
Solution:  A  dl     A .d a   B.d a  BA cos 60  0.6  
0
   0.15T .m 2
S S  2 2

GATE-2014

Q26. Which one of the following quantities is invariant under Lorentz transformation?
(a) Charge density (b) Charge (c) Current (d) Electric field
Ans: (b)
Q27. An unpolarized light wave is incident from air on a glass surface at the Brewster angle.
The angle between the reflected and the refracted wave is
(a) 0 o (b) 45 o (c) 90 o (d) 120 o
Ans: (c)

                                                                                
Head office  Branch office 
 
fiziks, H.No. 40 D, G.F, Jia Sarai,  Anand Institute of Mathematics, 
 
Near IIT, Hauz Khas, New Delhi‐16  28‐B/6, Jia Sarai, Near IIT 
 
Phone: 011‐26865455/+91‐9871145498 Hauz Khas, New Delhi‐16 
                                                   
                                             Website: www.physicsbyfiziks.com                                                                                          
                                                            Email: fiziks.physics@gmail.com                                                                   11 
fiziks
Institute for NET/JRF, GATE, IIT‐JAM, JEST, TIFR and GRE in PHYSICAL SCIENCES 
 
Q28. The electric field of a uniform plane wave propagating in a dielectric non-conducting

medium is given by E  xˆ 10 cos  6 107 t  0.4 z  V / m . The phase velocity of the

wave is _________ 10 8 m / s
Ans: 1.5
 6  107
v   1.5 108 m / sec
k 0.4

Q29. If the vector potential A  xxˆ  2 yyˆ  3zzˆ , satisfies the Coulomb gauge, the value of the
constant  is _______
Ans: 1
 
Solution: Coulomb gauge condition . A  0    2  3  0    1
Q30. A ray of light inside Region 1 in the xy -plane is incident at the semicircular boundary

 
that carries no free charges. The electric field at the point P  r0 ,  in plane polar
 4

coordinates is E1  7eˆr  3eˆ where êr and ê are the unit vectors. The emerging ray in

Region 2 has the electric field E 2 parallel to x -axis. If  1 and  2 are the dielectric

2
constants of Region-1 and Region-2 respectively then is ________
1
y

P r0 ,  / 4 

O
1 2 x
Region 1 Region 2

                                                                                
Head office  Branch office 
 
fiziks, H.No. 40 D, G.F, Jia Sarai,  Anand Institute of Mathematics, 
 
Near IIT, Hauz Khas, New Delhi‐16  28‐B/6, Jia Sarai, Near IIT 
 
Phone: 011‐26865455/+91‐9871145498 Hauz Khas, New Delhi‐16 
                                                   
                                             Website: www.physicsbyfiziks.com                                                                                          
                                                            Email: fiziks.physics@gmail.com                                                                   12 
fiziks
Institute for NET/JRF, GATE, IIT‐JAM, JEST, TIFR and GRE in PHYSICAL SCIENCES 
 
Ans: 2.32
 y
Solution:  E1  7er  3e
ˆ ˆ P r0 ,  / 4 

10  E2
 Ex   7eˆr  3eˆ  .xˆ  7 cos 45  3sin 45 
2
1 
2 E1

O
4 1 x
 E y   7eˆr  3eˆ  . yˆ  7 sin 45  3sin 45  2
2 Region 1 Region 2

 E   4
Thus E1 makes an angle   tan 1  y   tan 1    21.80
 Ex   10 
tan  2  2  tan 45
   2   2.32 where 1    450 and  2  450
tan 1  2  2 tan 23.2
Q31. The value of the magnetic field required to maintain non-relativistic protons of energy
1MeV in a circular orbit of radius 100 mm is _______Tesla
(Given: m p  1.67  1027 kg , e  1.6  1019 C )

Ans: 1.44

1.6  1019  B 2  0.1 1.6 1013  2 1.67 1027 


2 2
q2 B2 R2 13
E  1.6  10  B 
2

2 1.67  1027  1.6 1019   0.12


2
2m p

1013  2 1.67  1027  3.34  1040


B  2
  2.08  B  2.08 Tesla  1.44Tesla
1.6 10   0.01
38
1.6 1040

Q32. In an interference pattern formed by two coherent sources, the maximum and minimum
of the intensities are 9 I 0 and I 0 respectively. The intensities of the individual wave are

(a) 3I 0 and I 0 (b) 4 I 0 and I 0

(c) 5I 0 and 4 I 0 (d) 9 I 0 and I 0


Ans: (b)

   
2 2
Solution: I max  I1  I 2 and I min  I1  I 2

   
2 2
9I0  I1  I 2 and I 0  I1  I 2  I1  4 I 0 and I 2  I 0

                                                                                
Head office  Branch office 
 
fiziks, H.No. 40 D, G.F, Jia Sarai,  Anand Institute of Mathematics, 
 
Near IIT, Hauz Khas, New Delhi‐16  28‐B/6, Jia Sarai, Near IIT 
 
Phone: 011‐26865455/+91‐9871145498 Hauz Khas, New Delhi‐16 
                                                   
                                             Website: www.physicsbyfiziks.com                                                                                          
                                                            Email: fiziks.physics@gmail.com                                                                   13 
fiziks
Institute for NET/JRF, GATE, IIT‐JAM, JEST, TIFR and GRE in PHYSICAL SCIENCES 
 
Q33. The intensity of a laser in free space is 150mW / m 2 . The corresponding amplitude of the

electric field of the laser is _________


V
m
 0  8.854  10 12 C 2 / N .m 2 
Ans: 10.6

1 2I 2  150 103
Solution: I  c 0 E02  E0    10.6 V / m
2 c 0 3  108  8.854 1012

                                                                                
Head office  Branch office 
 
fiziks, H.No. 40 D, G.F, Jia Sarai,  Anand Institute of Mathematics, 
 
Near IIT, Hauz Khas, New Delhi‐16  28‐B/6, Jia Sarai, Near IIT 
 
Phone: 011‐26865455/+91‐9871145498 Hauz Khas, New Delhi‐16 
                                                   
                                             Website: www.physicsbyfiziks.com                                                                                          
                                                            Email: fiziks.physics@gmail.com                                                                   14 
fiziks
Institute for NET/JRF, GATE, IIT‐JAM, JEST, TIFR and GRE in PHYSICAL SCIENCES 
 
GATE-2015
Q34. A point charge is placed between two semi-infinite conducting plates which are inclined
at an angle of 30 o with respect to each other. The number of image charges
is___________.
Ans.: 11
360 360
Solution: n  1   1  11
 30
R  
Q35. Given that the magnetic flux through the closed loop PQRSP is  . If  A  dl   along
P
1

R 
PQR , the value of   dl along PSR is
P
A Q
R

S
(a)   1 (b) 1   (c)  1 (d) 1
Ans.: (b)
    R   P 
Solution:    s B.d a   A.dl   A  dl   A  dl
P R

R   R  
   1   A  dl   A  dl  1  
P P

Q15. The space between two plates of a capacitor carrying charges  Q and  Q is filled with
two different dielectric materials, as shown in the figure. Across the interface of the two
dielectric materials, which one of the following statements is correct?
 
(a) E and D are continuous
 
(b) E is continuous and D is discontinuous
 
(c) D is continuous and E is discontinuous Q Q
 
(d) E and D are discontinuous
Ans.: (d)

                                                                                
Head office  Branch office 
 
fiziks, H.No. 40 D, G.F, Jia Sarai,  Anand Institute of Mathematics, 
 
Near IIT, Hauz Khas, New Delhi‐16  28‐B/6, Jia Sarai, Near IIT 
 
Phone: 011‐26865455/+91‐9871145498 Hauz Khas, New Delhi‐16 
                                                   
                                             Website: www.physicsbyfiziks.com                                                                                          
                                                            Email: fiziks.physics@gmail.com                                                                   15 
fiziks
Institute for NET/JRF, GATE, IIT‐JAM, JEST, TIFR and GRE in PHYSICAL SCIENCES 
 
Q22. Four forces are given below in Cartesian and spherical polar coordinates
   r2  
(i) F1  K exp 2 rˆ 
(ii) F2  K x 3 yˆ  y 3 zˆ 
 R 
   ˆ 
(iii) F3  K x 3 xˆ  y 3 yˆ  (iv) F4  K  
r
where K is a constant Identify the correct option
(a) (iii) and (iv) are conservative but (i) and (ii)are not
(b) (i) and (ii) are conservative but (iii) and (iv) are not
(c) (ii) and (iii) are conservative but (i) and (iv) are not
(d) (i) and (iii) are conservative but (ii) and (iv) are not
Ans.: (d)

r rˆ r sin 


  1   
Solution:   F 1  0
r sin 
2
r  
 r2 
k exp   2  0 0
 R 

x y z
    
 F2   x  3ky 2  0   3ky 2 x
x y z
0 kx 3  ky 3

x y z
    
 F3  0
x y z
kx3 ky 3 0

                                                                                
Head office  Branch office 
 
fiziks, H.No. 40 D, G.F, Jia Sarai,  Anand Institute of Mathematics, 
 
Near IIT, Hauz Khas, New Delhi‐16  28‐B/6, Jia Sarai, Near IIT 
 
Phone: 011‐26865455/+91‐9871145498 Hauz Khas, New Delhi‐16 
                                                   
                                             Website: www.physicsbyfiziks.com                                                                                          
                                                            Email: fiziks.physics@gmail.com                                                                   16 
fiziks
Institute for NET/JRF, GATE, IIT‐JAM, JEST, TIFR and GRE in PHYSICAL SCIENCES 
 

r r r sin 


  1   
 F4  2  r  k cos  
r sin  r  
k
0 0 r sin  
r
Q23. A monochromatic plane wave (wavelength  600 nm ) E 0 expi kz   t  is incident
 
normally on a diffraction grating giving rise to a plane wave E1 exp i k1  r   t in the  
  1 3 
first order of diffraction. Here E1  E 0 and k1  k1  xˆ  zˆ  . The period (in m ) of
2 2 

the diffraction grating is ______________ (upto one decimal place)


Ans.: 1.2

Solution: d sin   n  d  n 1
sin 
  1 3  1
 k1  k1  xˆ  zˆ   sin      300
2 2  2

600
d  nm  1200 nm  1.2  m
sin 30
Q24. A long solenoid is embedded in a conducting medium and is insulated from the medium.
If the current through the solenoid is increased at a constant rate, the induced current in
the medium as a function of the radial distance r from the axis of the solenoid is
proportional to
1 1
(a) r 2 inside the solenoid and outside (b) r inside the solenoid and outside
r r2
1 1
(c) r 2 inside the solenoid and outside (d) r inside the solenoid and outside
r2 r
Ans.: (d)

  B 
Solution:  E  dl     da ;
t

                                                                                
Head office  Branch office 
 
fiziks, H.No. 40 D, G.F, Jia Sarai,  Anand Institute of Mathematics, 
 
Near IIT, Hauz Khas, New Delhi‐16  28‐B/6, Jia Sarai, Near IIT 
 
Phone: 011‐26865455/+91‐9871145498 Hauz Khas, New Delhi‐16 
                                                   
                                             Website: www.physicsbyfiziks.com                                                                                          
                                                            Email: fiziks.physics@gmail.com                                                                   17 
fiziks
Institute for NET/JRF, GATE, IIT‐JAM, JEST, TIFR and GRE in PHYSICAL SCIENCES 
 
For r  R
 dI
r
dI 2 r 2  1 dI
 E  2 r   0 n
dt 
r  0
2 r dr    0 n 
dt

2
 E   0 n r
2 dt

For r  R
 dI
R
dI 2 R 2  1 dI
 E  2 r   0 n
dt 
r  0
2 r dr    0 n 
dt

2
 E   0 n R 2
2r dt

Q25. A plane wave xˆ  iyˆ E 0 expikz  t  after passing through an optical element emerges

as xˆ  iyˆ E 0 expi kz  t  , where k and  are the wavevector and the angular
frequency, respectively. The optical element is a
(a) quarter wave plate (b) half wave plate
(c) polarizer (d) Faraday rotator
Ans.: (b)

 
Solution: Incident wave: x  i y E0 ei   E0 cos  x  E0 sin  y 
 
Left circular polarization with phase angle 1     ei

 
Emergent wave: x  i y E0 ei   E0 cos  xˆ  E0 sin  y 
 
Right circular polarization with phase angle 1     ei 0


Thus there is phase change of  and hence path difference of .
2

                                                                                
Head office  Branch office 
 
fiziks, H.No. 40 D, G.F, Jia Sarai,  Anand Institute of Mathematics, 
 
Near IIT, Hauz Khas, New Delhi‐16  28‐B/6, Jia Sarai, Near IIT 
 
Phone: 011‐26865455/+91‐9871145498 Hauz Khas, New Delhi‐16 
                                                   
                                             Website: www.physicsbyfiziks.com                                                                                          
                                                            Email: fiziks.physics@gmail.com                                                                   18 
fiziks
Institute for NET/JRF, GATE, IIT‐JAM, JEST, TIFR and GRE in PHYSICAL SCIENCES 
 
Q26. A charge  q is distributed uniformly over a sphere, with a positive charge q at its center
in (i). Also in (ii), a charge  q is distributed uniformly over an ellipsoid with a positive
charge q at its center. With respect to the origin of the coordinate system, which one of
the following statements is correct?

X X

Z Z

Y Y
(i ) (ii )
(a) The dipole moment is zero in both (i) and (ii)
(b) The dipole moment is non-zero in (i) but zero in (ii)
(c) The dipole moment is zero in (i) but non-zero in (ii)
(d) The dipole moment is non-zero in both (i) and (ii)
Ans.: (a)
 
Solution: p   qi ri  0 in both cases.

GATE-2016
Q27. Which of the following magnetic vector potentials gives rise to a uniform magnetic field
B0 kˆ ?

(a) B0 z kˆ (b)  B0 x ˆj (c)


B0
2

 yiˆ  xˆj  (d)
B0 ˆ ˆ
2

yi  xj 
Ans.: (c)
 
Solution: (a)   A  0
 
(b)   A   B0 kˆ
 
(c)   A  B0 kˆ
 
(d)   A  0

                                                                                
Head office  Branch office 
 
fiziks, H.No. 40 D, G.F, Jia Sarai,  Anand Institute of Mathematics, 
 
Near IIT, Hauz Khas, New Delhi‐16  28‐B/6, Jia Sarai, Near IIT 
 
Phone: 011‐26865455/+91‐9871145498 Hauz Khas, New Delhi‐16 
                                                   
                                             Website: www.physicsbyfiziks.com                                                                                          
                                                            Email: fiziks.physics@gmail.com                                                                   19 
fiziks
Institute for NET/JRF, GATE, IIT‐JAM, JEST, TIFR and GRE in PHYSICAL SCIENCES 
 
Q28. The magnitude of the magnetic dipole moment associated with a square shaped loop
carrying a steady current I is m . If this loop is changed to a circular shape with the same
pm
current I passing through it, the magnetic dipole moment becomes . The value of p

is ______.
Ans.: 4
Solution: Magnetic dipole moment associated with a square shaped loop (let side is a) carrying a
steady current I is m  Ia 2 .
Magnetic dipole moment associated with a circular shaped loop (let radius is r) carrying a
steady current I is m  I  r 2 .
2
2a 2a  4 Ia 2 4m
Here 4a  2 r  r   m  I  r  I      2

    
Q29. In a Young’s double slit experiment using light, the apparatus has two slits of unequal
widths. When only slit- 1 is open, the maximum observed intensity on the screen is 4 I 0 .

When only slit- 2 is open, the maximum observed intensity is I 0 . When both the slits are
open, an interference pattern appears on the screen. The ratio of the intensity of the
principal maximum to that of the nearest minimum is ________.
Ans.: 9

      2 I  I   9I
2 2
I max I1  I 2 4I0  I0 0 0
Solution:  0
9
    2 I  I  I
2 2 2
I min I1  I 2 4I0  I0 0
0 0

                                                                                
Head office  Branch office 
 
fiziks, H.No. 40 D, G.F, Jia Sarai,  Anand Institute of Mathematics, 
 
Near IIT, Hauz Khas, New Delhi‐16  28‐B/6, Jia Sarai, Near IIT 
 
Phone: 011‐26865455/+91‐9871145498 Hauz Khas, New Delhi‐16 
                                                   
                                             Website: www.physicsbyfiziks.com                                                                                          
                                                            Email: fiziks.physics@gmail.com                                                                   20 
fiziks
Institute for NET/JRF, GATE, IIT‐JAM, JEST, TIFR and GRE in PHYSICAL SCIENCES 
 
Q30. An infinite, conducting slab kept in a horizontal plane carries a uniform charge density  .
Another infinite slab of thickness t, made of a linear dielectric material of dielectric
constant k , is kept above the conducting slab. The bound charge density on the upper
surface of the dielectric slab is
   k  2  k  1
(a) (b) (c) (d)
2k k 2k k

Ans.: (d)  1
k  1 z
Solution: 
  
Electric field due to infinite, conducting slab inside the dielectric is E  zˆ  zˆ
  0k
     k  1    k  1
Polarisation P   0  e E   0  k  1 zˆ  zˆ   1  P.zˆ 
 0k k k
Q31. The electric field component of a plane electromagnetic wave travelling in vacuum is

given by E  z , t   E 0 coskz  t iˆ . The Poynting vector for the wave is

 c   c 
(a)  0  E 02 cos 2 kz  t  ˆj (b)  0  E 02 cos 2 kz  t kˆ
 2   2 

(c) c 0 E 02 cos 2 kz  t  ˆj (d) c 0 E 02 cos 2 kz  t kˆ


Ans.: (d)
  1  E
Solution: E  z , t   E 0 coskz  t iˆ  B  zˆ  E  z , t   0 cos  kz  t  ˆj
c c
The Poynting vector for the wave is
 1   E2
S
0
0 c

E  B  0 cos 2  kz  t  kˆ  c 0 E02 cos 2  kz  t  kˆ

                                                                                
Head office  Branch office 
 
fiziks, H.No. 40 D, G.F, Jia Sarai,  Anand Institute of Mathematics, 
 
Near IIT, Hauz Khas, New Delhi‐16  28‐B/6, Jia Sarai, Near IIT 
 
Phone: 011‐26865455/+91‐9871145498 Hauz Khas, New Delhi‐16 
                                                   
                                             Website: www.physicsbyfiziks.com                                                                                          
                                                            Email: fiziks.physics@gmail.com                                                                   21 
fiziks
Institute for NET/JRF, GATE, IIT‐JAM, JEST, TIFR and GRE in PHYSICAL SCIENCES 
 
Q32. The x  y plane is the boundary between free space and a magnetic material with relative

permeability  r . The magnetic field in the free space is Bx iˆ  Bz kˆ . The magnetic field in
the magnetic material is
(a) B x iˆ  B z kˆ (b) B x iˆ   r B z kˆ

1
(c) B x iˆ  B z kˆ (d)  r B x iˆ  B z kˆ
r
Ans.: (d)
B B
Solution: B1  Bz kˆ  B2 and H1  H1  1  2  B2  r B1  r Bx iˆ
0 0 r

The magnetic field in the magnetic material is  r B x iˆ  B z kˆ

GATE- 2017
Q33. Identical charges q are placed at five vertices of a regular hexagon of side a . The
magnitude of the electric field and the electrostatic potential at the centre of the hexagon
are respectively
q q
(a) 0, 0 (b) ,
4 0 a 2
4 0 a

q 5q 5q 5q
(c) , (d) ,
4 0 a 2
4 0 a 4 0 a 4 0 a
2

Ans. : (c)
q
q
Solution: The resultant field at P is E 
4 0 a 2 q q
5q a
The electrostatic potential at P is V 
4 0 a q P q

                                                                                
Head office  Branch office 
 
fiziks, H.No. 40 D, G.F, Jia Sarai,  Anand Institute of Mathematics, 
 
Near IIT, Hauz Khas, New Delhi‐16  28‐B/6, Jia Sarai, Near IIT 
 
Phone: 011‐26865455/+91‐9871145498 Hauz Khas, New Delhi‐16 
                                                   
                                             Website: www.physicsbyfiziks.com                                                                                          
                                                            Email: fiziks.physics@gmail.com                                                                   22 
fiziks
Institute for NET/JRF, GATE, IIT‐JAM, JEST, TIFR and GRE in PHYSICAL SCIENCES 
 
Q34. A parallel plate capacitor with square plates of side 1 m separated by 1 micro meter is
filled with a medium of dielectric constant of 10 . If the charges on the two plates are 1C
and 1C , the voltage across the capacitor is………….. kV . (up to two decimal places).

(  0  8.854  1012 F / m )
Ans. : 1.29
 0 r A qd 1 1 106
Solution: q  CV  V V    1.29kV
d  0 r A 8.854  1012  10  1
Q35. Light is incident from a medium of refractive index n  1.5 onto vacuum. The smallest
angle of incidence for which the light is not transmitted into vacuum is…………...
degrees. (up to two decimal places)
Ans. : 41.8
n2 1  1 
Solution: sin C    C  sin 1    C  41.8
n1 1.5  1.5 
Q36. A monochromatic plane wave in free space with electric field amplitude of 1 V / m is
normally incident on a fully reflecting mirror. The pressure exerted on the mirror
is……………… 1012 Pa . (up to two decimal places) (  0  8.854  1012 F / m )
Ans. : 8.85
2I 2 1
  c 0 E02   0 E02  8.854  1012  1  8.85  1012 Pa
2
Solution: P 
c c 2
Q37. Three charges  2C , 1C , 1C  are placed at the vertices of an equilateral triangle of side

1 m as shown in the figure. The component of the electric dipole moment about the
marked origin along the ŷ direction is……… C m .
y
2C

1m

0 1C 1C x
Ans. : 1.73 1.5m


Solution: p  11xˆ   1 2 xˆ   2 1.5 xˆ  1  0.25 yˆ 
                                                                                
Head office  Branch office 
 
fiziks, H.No. 40 D, G.F, Jia Sarai,  Anand Institute of Mathematics, 
 
Near IIT, Hauz Khas, New Delhi‐16  28‐B/6, Jia Sarai, Near IIT 
 
Phone: 011‐26865455/+91‐9871145498 Hauz Khas, New Delhi‐16 
                                                   
                                             Website: www.physicsbyfiziks.com                                                                                          
                                                            Email: fiziks.physics@gmail.com                                                                   23 
fiziks
Institute for NET/JRF, GATE, IIT‐JAM, JEST, TIFR and GRE in PHYSICAL SCIENCES 
 
Along the ŷ direction  2  1  0.25  1.73

Q38. An infinite solenoid carries a time varying current I  t   At 2 , with A  0 . The axis of

the solenoid is along the ẑ direction. r̂ and ˆ are the usual radial and polar directions in

cylindrical polar coordinates. B  Br rˆ  Bˆ  Bz zˆ is the magnetic field at a point outside
the solenoid. Which one of the following statements is true?
(a) Br  0, B  0, Bz  0 (b) Br  0, B  0, Bz  0

(c) Br  0, B  0, Bz  0 (d) Br  0, B  0, Bz  0
Ans. : (d)
Q39. A uniform volume charge density is placed inside a conductor (with ressistivity
1
102 m ). The charge density becomes of its original value after
 2.718
time……………….fem to seconds (up to two decimal places) (  0  8.854  1012 F / m )
Ans. : 88.54
 t  1
Solution:   t     0  e t /  0   t /  0  ln  ln 1
 0 2.718

0
t   8.854  1012  102  88.54  1015 sec  88.54 fs

Q40. Consider a metal with free electron density of 6 1022 cm 3 . The lowest frequency

electromagnetic radiation to which this metal is transparent is 1.38 1016 Hz . If this metal

had a free electron density of 1.8  1023 cm 3 instead, the lowest frequency

electromagnetic radiation to which it would be transparent is…………… 1016 Hz (up to


two decimal places).
Ans. : 2.39
Solution: Cut-off frequency is f  n .

f2 n n 1.8  1023
Thus  2  f 2  f1 2  f 2  1.38 1016  2.39 1016 Hz
f1 n1 n1 6  1022

                                                                                
Head office  Branch office 
 
fiziks, H.No. 40 D, G.F, Jia Sarai,  Anand Institute of Mathematics, 
 
Near IIT, Hauz Khas, New Delhi‐16  28‐B/6, Jia Sarai, Near IIT 
 
Phone: 011‐26865455/+91‐9871145498 Hauz Khas, New Delhi‐16 
                                                   
                                             Website: www.physicsbyfiziks.com                                                                                          
                                                            Email: fiziks.physics@gmail.com                                                                   24 
fiziks
Institute for NET/JRF, GATE, IIT‐JAM, JEST, TIFR and GRE in PHYSICAL SCIENCES 
 
QUANTUM MECHANICS SOLUTIONS
GATE- 2010
Q1. Which of the following is an allowed wavefunction for a particle in a bound state? N is
a constant and α, β > 0.
e r
(a)   N (b)   N 1  e r 
r3
(c)   Ne x e   x  (d)   0non - zero constant if r  R
2
 y2 z2
 if r  R
Ans: (c)
Q2. A particle of mass m is confined in the potential
 1
V  x    2 m x
2 2
for x  0
 for x  0
Let the wavefunction of the particle be given by
1 2
 x     0  1 ,
5 5
where  0 and  1 are the eigenfunctions of the ground state and the first excited state

respectively. The expectation value of the energy is


31 25 13 11
(a)  (b)  (c)  (d) 
10 10 10 10
Ans: (a)
Solution: For half parabolic potential
3 7 1 3 4 7 31
E0   , E1    E       .
2 2 5 2 5 2 10
2
Q3. For a spin-s particle, in the eigen basis of S , Sz the expectation value sm S z2 sm is

(a)

 2 s s  1  m 2  
(b)  2 s s  1  2m 2 
2

(c)  s s  1  m 2
2
 (d)  2 m 2
Ans: (a)
1 1
sm S   S   sm  sm S 2  S 2  S  S   S  S  sm
2
Solution: sm S z2 sm 
2 2
1
 sm S  S   S  S  sm 
2
2
2
s s  1  m 2  
                                                                                
Head office  Branch office 
 
fiziks, H.No. 23, G.F, Jia Sarai,  Anand Institute of Mathematics, 
 
Near IIT, Hauz Khas, New Delhi‐16  28‐B/6, Jia Sarai, Near IIT 
 
Phone: 011‐26865455/+91‐9871145498 Hauz Khas, New Delhi‐16 
                                                   
                                             Website: www.physicsbyfiziks.com                                                                                          
                                                            Email: fiziks.physics@gmail.com                                                                   1 
fiziks
Institute for NET/JRF, GATE, IIT‐JAM, JEST, TIFR and GRE in PHYSICAL SCIENCES 
 
Q4. A particle of mass m is confined in an infinite potential well:

V x   0 if 0  x  L,
 otherwise.
 2x 
It is subjected to a perturbing potential V p  x   Vo sin  
 L  V x 
within the well. Let E(a) and E(2) be corrections to the ground
state energy in the first and second order in V0, respectively. V p x 
0 L
Which of the following are true?
(a) E(a) = 0; E(2) < 0 (b) E(a) >
0; E(2) = 0
(c) E(a) = 0; E(2) depends on the sign of V0 (d) E(a) < 0; E(2) < 0
Ans: (a)
2
2
L
2x  m VP  1
Solution: E   V0 sin
1
1 dx  0 ; E12    E1  E m so E12  ve .
L0 L m 1 E1  Em

GATE- 2011
Q5. The quantum mechanical operator for the momentum of a particle moving in one
dimension is given by
d d  2 d 2
(a) i (b)  i (c) i (d) 
dx dx t 2m dx 2
Ans: (b)
Q6. An electron with energy E is incident from left on a potential barrier, given by
V ( x) = 0 for x < 0 V x 
= V0 for x > 0
V0
as shown in the figure. For E < V0, the space part of the
E
wavefunction for x > 0 is of the form
(a) eax (b) e-ax (c) eiax x
0
(d)e-iax
Ans: (b)
Solution:  E  V0 , so there is decaying wave function.

                                                                                
Head office  Branch office 
 
fiziks, H.No. 23, G.F, Jia Sarai,  Anand Institute of Mathematics, 
 
Near IIT, Hauz Khas, New Delhi‐16  28‐B/6, Jia Sarai, Near IIT 
 
Phone: 011‐26865455/+91‐9871145498 Hauz Khas, New Delhi‐16 
                                                   
                                             Website: www.physicsbyfiziks.com                                                                                          
                                                            Email: fiziks.physics@gmail.com                                                                   2 
fiziks
Institute for NET/JRF, GATE, IIT‐JAM, JEST, TIFR and GRE in PHYSICAL SCIENCES 
 
Q7. If Lx, Ly and Lz are respectively the x, y and z components of angular momentum operator
L. The commutator [Lx Ly, Lz] is equal to
(a) i L2x  L2y  (b) 2iLz (c) i L2x  L2y  (d) 0

Ans: (c)
 
Solution: Lx L y , Lz = Lx Ly Lz   Lx , Lz Ly = iL2x  L2y 

Q8. The normalized ground state wavefunciton of a hydrogen atom is given by


1 2
 r   e r / a , where a is the Bohr radius and r is the distance of the electron
4 a
3/ 2

1
from the nucleus, located at the origin. The expectation value is
r2
8 4 4 2
(a) (b) (c) (d)
a2 a2 a2 a2
Ans: (d)
  2
1 1 1 2  2ar 2
3  2
Solution:  r e dr   sin  d d  2
r2 4a 0 r 0 0
a

Q9. The normalized eigenstates of a particle in a one-dimensional potential well

V x   0 if 0  x  a
 otherwise

2  nx 
are given by  n  x   sin   , where n = 1, 2, 3,….
a  a 
The particle is subjected to a perturbation
 x  a
V '  x   Vo cos  for 0  x 
 a 2
=0 otherwise
Q10. The shift in the ground state energy due to the perturbation, in the first order perturbation
theory,
2Vo Vo Vo 2Vo
(a) (b) (c)  (d) 
3 3 3 3

                                                                                
Head office  Branch office 
 
fiziks, H.No. 23, G.F, Jia Sarai,  Anand Institute of Mathematics, 
 
Near IIT, Hauz Khas, New Delhi‐16  28‐B/6, Jia Sarai, Near IIT 
 
Phone: 011‐26865455/+91‐9871145498 Hauz Khas, New Delhi‐16 
                                                   
                                             Website: www.physicsbyfiziks.com                                                                                          
                                                            Email: fiziks.physics@gmail.com                                                                   3 
fiziks
Institute for NET/JRF, GATE, IIT‐JAM, JEST, TIFR and GRE in PHYSICAL SCIENCES 
 
Ans: (a)
a/2
3 x
sin
 x   x 
a/2 a/2
2 2 2V0
Solution: E11   1 V x  1dx    V0 a 
*
sin 2  V0 cos dx
a  a  a a  3
0 0 3
a 0

Common data questions Q-10 and Q-11


Q11. In a one-dimensional harmonic oscillator, φ0, φ1 and φ2 are respectively the ground, first
and the second excited states. These three states are normalized and are orthogonal to one
another. ψ1 and ψ2 are two states defined by
ψ1 = φ0 - 2φ1 + 3φ2 , ψ2 = φ0 – φ1 + αφ2
where α is a constant

Q12. The value of α which ψ2 is orthogonal to ψ1 is


(a) 2 (b) 1 (c) – 1 (d) – 2
Ans: (c)
Solution: For orthogonal condition scalar product (ψ2, ψ1)=0 so 1  2  3  0    1 .

Q13. For the value of α determined in Q30, the expectation value of energy of the oscillator in
the state ψ2 is
(a) ħω (b) 3 ħω/2 (c) 3 ħω (d) 9 ħω/2
Ans: (b)
 3 5
2 H 2  
Solution:  2  0  1  2 put   1 , H   2 2 2  3 
2 2 3 2

                                                                                
Head office  Branch office 
 
fiziks, H.No. 23, G.F, Jia Sarai,  Anand Institute of Mathematics, 
 
Near IIT, Hauz Khas, New Delhi‐16  28‐B/6, Jia Sarai, Near IIT 
 
Phone: 011‐26865455/+91‐9871145498 Hauz Khas, New Delhi‐16 
                                                   
                                             Website: www.physicsbyfiziks.com                                                                                          
                                                            Email: fiziks.physics@gmail.com                                                                   4 
fiziks
Institute for NET/JRF, GATE, IIT‐JAM, JEST, TIFR and GRE in PHYSICAL SCIENCES 
 
GATE- 2012

Q14. A particle of mass m is confined in a two dimensional square well potential of


dimension a. This potential V(x, y) is given by
V(x, y) = 0 for –a < x < a and –a < y < a
= ∞ elsewhere
The energy of the first excited state for this particle is given by,
 2 2 2 2  2 5 2  2 4 2  2
(a) (b) (c) (d)
ma 2 ma 2 8ma 2 ma 2
Ans: (c)
 2 2 5 2  2
 2 2
Solution: E   n  n 2 2
  n  n 
2
x
2
y  n x  1, n y  2 .
2m  2a 
x y 2
8ma 2 8ma 2
 
Q15. Consider the wavefunction    r1 , r2  s for a fermionic system consisting of two spin-
half particles. The spatial part of the wavefunction is given by
 
 r1 , r2  
1
1 r1  2 r2    2 r1 1 r2 
2
where 1and 2 are single particle states. The spin part χs of the wavefunction with spin
states   1 / 2 and - 1/2  should be
1 1
(a)     (b)     (c) αα (d) ββ
2 2
Ans: (b)
Solution: Since  r1 , r2  is symmetric the total wavefunction must be antisymmetric for fermions
so spin part must be antisymmetric.

                                                                                
Head office  Branch office 
 
fiziks, H.No. 23, G.F, Jia Sarai,  Anand Institute of Mathematics, 
 
Near IIT, Hauz Khas, New Delhi‐16  28‐B/6, Jia Sarai, Near IIT 
 
Phone: 011‐26865455/+91‐9871145498 Hauz Khas, New Delhi‐16 
                                                   
                                             Website: www.physicsbyfiziks.com                                                                                          
                                                            Email: fiziks.physics@gmail.com                                                                   5 
fiziks
Institute for NET/JRF, GATE, IIT‐JAM, JEST, TIFR and GRE in PHYSICAL SCIENCES 
 
Q16. A particle is constrained to move in a truncated harmonic potential well (x > 0) as shown
in the figure. Which one of the following statements is CORRECT?
(a) The parity of the first excited state is even
(b) The parity of the ground state is even Vx
1
(c) the ground state energy is 
2
7
(d) The first excited state energy is 
2
Ans: (d) x
3 7
Solution: There is only odd parity. Ground state is  and first excited  
2 2

Q17. Consider a system in the unperturbed state described by the Hamiltonian, H0  1 0 .
0 1 

The system is subjected to a perturbation of the form H'   


   , when δ<< 1. The
energy eigenvalues of the perturbed system using the first order perturbation
approximation are
(a) 1 and (1 + 2δ) (b) (1 + δ) and (1 - δ)
(c) (1+ 2δ) and (1 - 2δ) (d) (1+ δ) and (1 - 2δ)
Ans: (a)
Solution: H 0  H  , H0 is degenerate so after using degenerate perturbation through diagonalized

 2 0 1 0  2 0
H  one will get H    , H      .
0 0 0 1 0 0
So E  1  2 and 1 0 .

                                                                                
Head office  Branch office 
 
fiziks, H.No. 23, G.F, Jia Sarai,  Anand Institute of Mathematics, 
 
Near IIT, Hauz Khas, New Delhi‐16  28‐B/6, Jia Sarai, Near IIT 
 
Phone: 011‐26865455/+91‐9871145498 Hauz Khas, New Delhi‐16 
                                                   
                                             Website: www.physicsbyfiziks.com                                                                                          
                                                            Email: fiziks.physics@gmail.com                                                                   6 
fiziks
Institute for NET/JRF, GATE, IIT‐JAM, JEST, TIFR and GRE in PHYSICAL SCIENCES 
 
Q18. The ground state wavefunction for the hydrogen atom is given by
3/ 2
1  1 
 100    e  r / a0 , where a 0 is the Bohr radius. The plot of the radial probability
4  a 0 
density, P(r) for the hydrogen atom in the ground state is
(a) (b)

P(r) P(r)

r/a 0 r/a 0
(c) (d)

P(r) P(r)

r/a 0 r/a 0
Ans: (d)
3/ 2
1 1
Solution: The ground state is given by  100    e r / a0
4  a0 
Radial probability function
Pr 
1 1 2  2 r / a0
Pr    r =
2 2
r e
4 a0 r / a0
Common Data for Questions 19–20
The wavefunction of particle moving in free space is given by,   e ikx  2e ikx
Q19. The energy of the particle is
5 2 k 2 3 2 k 2 2k 2 2k 2
(a) (b) (c) (d)
2m 4m 2m m
Ans: (c)

                                                                                
Head office  Branch office 
 
fiziks, H.No. 23, G.F, Jia Sarai,  Anand Institute of Mathematics, 
 
Near IIT, Hauz Khas, New Delhi‐16  28‐B/6, Jia Sarai, Near IIT 
 
Phone: 011‐26865455/+91‐9871145498 Hauz Khas, New Delhi‐16 
                                                   
                                             Website: www.physicsbyfiziks.com                                                                                          
                                                            Email: fiziks.physics@gmail.com                                                                   7 
fiziks
Institute for NET/JRF, GATE, IIT‐JAM, JEST, TIFR and GRE in PHYSICAL SCIENCES 
 
   
2 2 2
Solution: H  E , H   ik ik e ikx  2 ik  ik e ikx
2m x 2
2m
 2 k 2 ikx 2 k 2
 H  (e  2e 2ikx )  
2m 2m

Q20. The probability current density for the real part of the wavefunction is
k k
(a) 1 (b) (c) (d) 0
m 2m
Ans: (d)
Solution: The real part of the wave function real  cos kx  2 cos kx
Current density for real part of wave function = 0

GATE- 2013
Q21. Which one of the following commutation relations is NOT CORRECT? Here, symbols
have their usual meanings.

(a) L2 , L z  0   
(b) L x , L y  iL z

(c) L z , L   L (d) L z , L   L


Ans: (d)
Q22. A proton is confined to a cubic box, whose sides have length 10 12 m . What is the
minimum kinetic energy of the proton? The mass of proton is 1.67  10 27 kg and

Planck’s constant is 6.63  10 34 Js .


(a) 1.1  10 17 J (b) 3.3  10 17 J (c) 9.9  10 17 J (d) 6.6  10 17 J
Ans: (c)
3 2  2
Solution: 2
 9.9 1017
2ma

                                                                                
Head office  Branch office 
 
fiziks, H.No. 23, G.F, Jia Sarai,  Anand Institute of Mathematics, 
 
Near IIT, Hauz Khas, New Delhi‐16  28‐B/6, Jia Sarai, Near IIT 
 
Phone: 011‐26865455/+91‐9871145498 Hauz Khas, New Delhi‐16 
                                                   
                                             Website: www.physicsbyfiziks.com                                                                                          
                                                            Email: fiziks.physics@gmail.com                                                                   8 
fiziks
Institute for NET/JRF, GATE, IIT‐JAM, JEST, TIFR and GRE in PHYSICAL SCIENCES 
 
Q23. A spin-half particle is in a linear superposition 0.8   0.6  of its spin-up and spin-

down states. If  and  are the eigenstates if  z then what is the expectation value

up to one decimal place, of the operator 10 z  5 x ? Here, symbols have their usual
meanings. _______________
Ans: 76
1  0   .8 
Solution:   .8   .6   .8   .6    
 0  1   .6 
1 0 0 1 10 5 
Operator A  10 z  5 x  10   5   A   
0 1 1 0  5 10 
10 5  .8 
A   A  .8 .6   = 88  12   76
 5 10  .6 
Q24. Consider the wave function Ae i k r r0 / r  , where A is the normalization constant.

For r  2r0 , the magnitude of probability current density up to two decimal places, in

 
units of A 2 k / m is. _____________
Ans:
2 2
 2 k 2 r k 2 r k 2 k
Solution: J    A 0 J A 0 J A
m r m 2r0 m 4m

Common data questions 25 and 26


5 2 0 
To the given unperturbed Hamiltonian 2 5 0
0 0 2

1 1 1
we add a small perturbation given by  1 1  1 where  is small quantity.

1  1 1 

                                                                                
Head office  Branch office 
 
fiziks, H.No. 23, G.F, Jia Sarai,  Anand Institute of Mathematics, 
 
Near IIT, Hauz Khas, New Delhi‐16  28‐B/6, Jia Sarai, Near IIT 
 
Phone: 011‐26865455/+91‐9871145498 Hauz Khas, New Delhi‐16 
                                                   
                                             Website: www.physicsbyfiziks.com                                                                                          
                                                            Email: fiziks.physics@gmail.com                                                                   9 
fiziks
Institute for NET/JRF, GATE, IIT‐JAM, JEST, TIFR and GRE in PHYSICAL SCIENCES 
 
Q25. The ground state eigenvector of the unperturbed Hamiltonian is

(a) 1 / 2 ,1 2 ,0  
(b) 1 / 2 ,1 / 2 ,0 
(c) 0,0,1 (d) 1,0,0 
Ans: (c)
5 2 0 1 1 1
   
H 0   2 5 0  , H P    1 1  1
0 0 2 1  1 1 
 
Eigen value of H 0 is E1  2, E2  3, E3  7 and the Eigen vector corresponds

0 1 1


  1   1  
to 1   0  , 2  1 , 3  1 .
1 2   2  
  0 0
Q26. A pair of eigenvalues of the perturbed Hamiltonian, using first order perturbation theory,
is
(a) 3  2 ,7  2 (b) 3  2 ,2  
(c) 3, 7  2 (d) 3, 2  2
Ans: (c)
Solution: E1  1 H P 1  1  E1  2  1

1 1 1 1 1
1   1   1  
E2  2 H P 2  1  1 0. 1 1  1.   1 = 0 0 1  1  0
2 1  1 1  2  0  2 0
     

1 1 1  1 1
1   1   1  
E3  3 H P 3  1 1 0  . 1 1 1 .  1  =  . 2 2 0. 1 
2 1 1 1  2  0  2  0
     
1
 E 3  4  2
2
E1  2  1 , E 2  3  0 , E3  7  2 .

                                                                                
Head office  Branch office 
 
fiziks, H.No. 23, G.F, Jia Sarai,  Anand Institute of Mathematics, 
 
Near IIT, Hauz Khas, New Delhi‐16  28‐B/6, Jia Sarai, Near IIT 
 
Phone: 011‐26865455/+91‐9871145498 Hauz Khas, New Delhi‐16 
                                                   
                                             Website: www.physicsbyfiziks.com                                                                                          
                                                            Email: fiziks.physics@gmail.com                                                                   10 
fiziks
Institute for NET/JRF, GATE, IIT‐JAM, JEST, TIFR and GRE in PHYSICAL SCIENCES 
 
GATE- 2014
Q27. The recoil momentum of an atom is p A when it emits an infrared photon of wavelength
1500 nm , and it is p B when it emits a photon of visible wavelength 500 nm . The ratio

pA
is
pB

(a) 1 : 1 (b) 1 : 3 (c) 1 : 3 (d) 3 : 2


Ans: (c)
h p A B  B 500
Solution: p  ,  ,  =1 : 3
 pB A  A 1500
Q28. The ground state and first excited state wave function of a one dimensional infinite
potential well are  1 and  2 respectively. When two spin-up electrons are placed in this

potential which one of the following with x1 and x2 denoting the position of the two
electrons correctly represents the space part of the ground state wave function of the
system?
1 1
(a)  1 x1  2 x1    1 x2  2 x2  (b)  1 x1  2 x2    1 x2  2 x1 
2 2
1 1
(c)  1 x1  2 x1    1 x2  2 x2  (d  1 x1  2 x 2    1 x 2  2 x1  )
2 2
Ans: (d)
Solution: From the given information only possible spin configuration is symmetric in nature so
space part will anti symmetric
1
 1 x1  2 x 2    1 x 2  2 x1 
2
 
Q29. If L is the orbital angular momentum and S is the spin angular momentum, then L.S
does not commute with
 
(a) S z (b) L2 (c) S 2 
(d) L  S 2

Ans: (d)

                                                                                
Head office  Branch office 
 
fiziks, H.No. 23, G.F, Jia Sarai,  Anand Institute of Mathematics, 
 
Near IIT, Hauz Khas, New Delhi‐16  28‐B/6, Jia Sarai, Near IIT 
 
Phone: 011‐26865455/+91‐9871145498 Hauz Khas, New Delhi‐16 
                                                   
                                             Website: www.physicsbyfiziks.com                                                                                          
                                                            Email: fiziks.physics@gmail.com                                                                   11 
fiziks
Institute for NET/JRF, GATE, IIT‐JAM, JEST, TIFR and GRE in PHYSICAL SCIENCES 
 
Q30. An electron in the ground state of the hydrogen atom has the wave function
 r 
  
 1
 r   e  a0 

a 03

where a0 is constant. The expectation value of the operator Qˆ  z 2  r 2 , where

z  r cos  is
 n  n  1! )
(Hint:  0
e ar r n dr 
a n 1

a n 1
 a 02  3a 02
(a) (b)  a 02 (c) (d)  2a02
2 2
Ans: (d)

Solution: Qˆ  z 2  r 2  a02  3a02  2a02

Q31. A particle of mass m is subjected to a potential

V  x, y  
1
2
 
m 2 x 2  y 2 ,    x  ,  y  

The state with energy 4  is g  fold degenerate. The value of g is______


Ans: 4
Solution: This is two isotropic dimensional harmonic oscillator the energy eigen value for nth
state is E n  (n  1) with degeneracy g n  (n  1) so degeneracy for 4  is 4 .
Q32. A hydrogen atom is in the state

8 3 4
  200   310   321
21 7 21
where n, l , m in  nl m denote the principal. Orbit and magnetic quantum numbers,

respectively. If L is the angular momentum operator, the average value of L2 is
_______  2
Ans: 2

                                                                                
Head office  Branch office 
 
fiziks, H.No. 23, G.F, Jia Sarai,  Anand Institute of Mathematics, 
 
Near IIT, Hauz Khas, New Delhi‐16  28‐B/6, Jia Sarai, Near IIT 
 
Phone: 011‐26865455/+91‐9871145498 Hauz Khas, New Delhi‐16 
                                                   
                                             Website: www.physicsbyfiziks.com                                                                                          
                                                            Email: fiziks.physics@gmail.com                                                                   12 
fiziks
Institute for NET/JRF, GATE, IIT‐JAM, JEST, TIFR and GRE in PHYSICAL SCIENCES 
 
Solution: If L will measure on state  the measurement is 0 2 , 2 2 and 6 2 with probability
2

8 3 4 3 4
, and so L2  2 2   6 2  = 2 2
21 7 21 7 21
1
Q33.  1and 2 are two orthogonal states of a spin system. It is given that
2
1 1 2 0
1      ,
3 0 3  1 

1  0
where   and   represent the spin-up and spin-down states, respectively. When the
0 1
system is in the state  2 its probability to be in the spin-up state is _______
2
Ans:
3
1 1 2 0 2 1 1  0
Solution: If is  1      , then  2      ,
3 0 3  1  3  0 3  1 

2
so probability that  2 is in up state is
3
Q34. A particle is confined to a one dimensional potential box, with the potential
V x   0, 0xa
 , otherwise
If particle is subjected to a perturbation within the box. W   x . Where  is small
constant, the first order correction to the ground state energy is
(a) 0 (b) a / 4 (c) a / 2 (d) a
Ans: (c)
Solution: First order energy correction is W   x the average value position in ground state

a
is x  so ans is a / 2
2

                                                                                
Head office  Branch office 
 
fiziks, H.No. 23, G.F, Jia Sarai,  Anand Institute of Mathematics, 
 
Near IIT, Hauz Khas, New Delhi‐16  28‐B/6, Jia Sarai, Near IIT 
 
Phone: 011‐26865455/+91‐9871145498 Hauz Khas, New Delhi‐16 
                                                   
                                             Website: www.physicsbyfiziks.com                                                                                          
                                                            Email: fiziks.physics@gmail.com                                                                   13 
fiziks
Institute for NET/JRF, GATE, IIT‐JAM, JEST, TIFR and GRE in PHYSICAL SCIENCES 
 
Q35. A one dimensional harmonic oscillator is in the superposition of number state n given

by
1 3
  2  3
2 2
The average energy of the oscillator in the given state is______  .
Ans: 3.25
1 5 3 7
.  .
Solution: Average energy will 4 2 4 2  3.25
1 3

4 4
Q36. If L and L are the angular momentum ladder operators then the expectation value of

L L  L L  in the state l  1, m  1 of an atom is _____  2

Ans: 2
Solution: L L  L L   2( L2  L2z )  2(l.(l  1)  m 2 ) 2 = 2 2
GATE- 2015
1  
Q37. An operator for a spin  particle is given by     B ,where
2
 B 
B  xˆ  yˆ  ,  denotes Pauli matrices and  is a constant. The eigenvalues of  are
2
B
(a)  (b)  B (c) 0, B (d) 0,   B
2
Ans.: (b)
   B
Solution: Aˆ     , B  xˆ  yˆ 
2

Aˆ    x Bx   y By   z Bz   Aˆ    x Bx   y By 

 0 1  B  0 i  B  ˆ B  0 1 i
Aˆ         A  
 1 0  2  i 0  2  2 1  i 0 

 B   1  i 
A   I  0     0      B
2 1  i  

                                                                                
Head office  Branch office 
 
fiziks, H.No. 23, G.F, Jia Sarai,  Anand Institute of Mathematics, 
 
Near IIT, Hauz Khas, New Delhi‐16  28‐B/6, Jia Sarai, Near IIT 
 
Phone: 011‐26865455/+91‐9871145498 Hauz Khas, New Delhi‐16 
                                                   
                                             Website: www.physicsbyfiziks.com                                                                                          
                                                            Email: fiziks.physics@gmail.com                                                                   14 
fiziks
Institute for NET/JRF, GATE, IIT‐JAM, JEST, TIFR and GRE in PHYSICAL SCIENCES 
 
1   
Q38. The Pauli matrices for three spin  particles are  1 ,  2 and  3 , respectively. The
2
  
dimension of the Hilbert space required to define an operator Oˆ   1   2   3 is_______
Ans.: 8
Solution:  2   3 has dimension of 4 and  1.  2   3 has dimension of 2  4  8
 
Q39. Let L and p be the angular and linear momentum operators, respectively, for a a particle.
The commutator L x , p y gives  
(a) i pz (b) 0 (c) i px (d) i pz
Ans.: (d)
Solution:  Lx , p y    ypz  zp y , p y    ypz , p y    zp y , p y    y, p y  pz

  p y , p y   0 and  z , p y   0

  Lx , p y   ipz   y, p y   i
3
Q40. Consider a system of eight non-interacting, identical quantum particles of spin  in a
2
one dimensional box of length L . The minimum excitation energy of the system, in units
 2 2
of is ________
2mL2
Ans.: 5
3  3 
Solution: spin  degeneracy   2S  1   2   1  4
2  2 
 22 4 2  2 20 2  2
Eground  4   4 
2mL2 2mL2 2mL2
st  22  22  22  2 2
I
Eexcited  4  3 4  1 9  25
2mL2 2mL2 2mL2 2mL2
st  22  22  2 2
Now minimum excitation energy E  Eexcited
I
 Eground  25  20 5
2mL2 2mL2 2mL2

                                                                                
Head office  Branch office 
 
fiziks, H.No. 23, G.F, Jia Sarai,  Anand Institute of Mathematics, 
 
Near IIT, Hauz Khas, New Delhi‐16  28‐B/6, Jia Sarai, Near IIT 
 
Phone: 011‐26865455/+91‐9871145498 Hauz Khas, New Delhi‐16 
                                                   
                                             Website: www.physicsbyfiziks.com                                                                                          
                                                            Email: fiziks.physics@gmail.com                                                                   15 
fiziks
Institute for NET/JRF, GATE, IIT‐JAM, JEST, TIFR and GRE in PHYSICAL SCIENCES 
 
Q41. A particle is confined in a box of length L as shown below.

V0
L/2
If the potential V0 is treated as a perturbation, including the first order correction, the
ground state energy is
 2 2  2  2 V0
(a) E   V0 (b) E  
2mL2 2mL2 2
 2  2 V0  2  2 V0
(c) E   (d) E  
2mL2 4 2mL2 2
Ans.: (d)
L
2
x L
x
Solution: E01   V0 sin 2 dx   0  sin 2 dx
0
L L L
2

L L
V0  2
2 x  V0   2 x  L  2
 E01  
2 0
1  cos
L 
dx 
2  x  sin  L  2 x 
   0

V0  2 2 V0
 E01  E 
2 2mL2 2
 
Q42. Let the Hamiltonian for two spin-½ particles of equal masses m , momenta p1 and p2
 
and positions r1 and r2 be H 
1 2
2m
p1 
1 2 1
2m
  
p 2  m 2 r12  r22  k 1   2 , where  1
2
 

and  2 denote the corresponding Pauli matrices,   0.1eV and k  0.2 eV . If the
ground state has net spin zero, then the energy (in eV ) is ___________
Ans.: -0.3

                                                                                
Head office  Branch office 
 
fiziks, H.No. 23, G.F, Jia Sarai,  Anand Institute of Mathematics, 
 
Near IIT, Hauz Khas, New Delhi‐16  28‐B/6, Jia Sarai, Near IIT 
 
Phone: 011‐26865455/+91‐9871145498 Hauz Khas, New Delhi‐16 
                                                   
                                             Website: www.physicsbyfiziks.com                                                                                          
                                                            Email: fiziks.physics@gmail.com                                                                   16 
fiziks
Institute for NET/JRF, GATE, IIT‐JAM, JEST, TIFR and GRE in PHYSICAL SCIENCES 
 
1 2 1 2 1  
Solution: H  p1  p2  m 2  r12  r22   k 1. 2
2m 2m 2
    2      2
   1   2     12   22  2 1. 2  2 1. 2     12   22
   
 2 1. 2  0  3I  3I  6 I   1. 2  3
3
Now energy E  2    k  3  3   0.1   0.2  3  0.3 eV
2
Q43. Suppose a linear harmonic oscillator of frequency  and mass m is in the state

1  i 
  
 0  e 2
 1  at t  0 where  0 and  1 are the ground and the first
2 


excited states, respectively. The value of  x  in the units of at t  0 is _____
m
Ans. : 0
1   
Solution:     0  e i 1 
2 2 

 
 x  
2m
 a  a     x  
2m
a   a  
1 i 2 1  i


a   e 0 and a    
 0  2 e 2
2 
2 2 

  1 1 i 2 1 1  i 2 
  x    e    e 1 1 
2m  2 2
0 0
2 2 

  x   0  0
2m
GATE-2016
Q44. Which of the following operators is Hermitian?
d d2 d2 d3
(a) (b) (c) i (d)
dx dx 2 dx 2 dx 3
Ans.: (b)

                                                                                
Head office  Branch office 
 
fiziks, H.No. 23, G.F, Jia Sarai,  Anand Institute of Mathematics, 
 
Near IIT, Hauz Khas, New Delhi‐16  28‐B/6, Jia Sarai, Near IIT 
 
Phone: 011‐26865455/+91‐9871145498 Hauz Khas, New Delhi‐16 
                                                   
                                             Website: www.physicsbyfiziks.com                                                                                          
                                                            Email: fiziks.physics@gmail.com                                                                   17 
fiziks
Institute for NET/JRF, GATE, IIT‐JAM, JEST, TIFR and GRE in PHYSICAL SCIENCES 
 

Q45. The scattering of particles by a potential can be analyzed by Born approximation. In


particular, if the scattered wave is replaced by an appropriate plane wave, the
corresponding Born approximation is known as the first Born approximation. Such an
approximation is valid for
(a) large incident energies and weak scattering potentials.
(b) large incident energies and strong scattering potentials.
(c) small incident energies and weak scattering potentials.
(d) small incident energies and strong scattering potentials.
Ans.: (a)
Q46. Consider an elastic scattering of particles in l  0 states. If the corresponding phase shift
 0 is 900 and the magnitude of the incident wave vector is equal to 2 fm 1 then the

total scattering cross section in units of fm 2 is _______.


Ans.: 2
4
Solution:     2l  1 sin  l for l  0 it is given  0  90 0 and k  2 fm 1
k 2 l 0
4
 sin 90  2
2
Q47. A hydrogen atom is in its ground state. In the presence of a uniform electric field

E  E0 zˆ , the leading order change in its energy is proportional to  E0  . The value of
n

the exponent n is _______.


Ans.: 2
Solution: First order energy correction is zero  1,0,0 E0 r cos   1,0,0  0
2
 n,l ,m E0 r cos   1,0,0
So one need to find correction of second   E02
n 1 E10  Em0

So value of n  2

                                                                                
Head office  Branch office 
 
fiziks, H.No. 23, G.F, Jia Sarai,  Anand Institute of Mathematics, 
 
Near IIT, Hauz Khas, New Delhi‐16  28‐B/6, Jia Sarai, Near IIT 
 
Phone: 011‐26865455/+91‐9871145498 Hauz Khas, New Delhi‐16 
                                                   
                                             Website: www.physicsbyfiziks.com                                                                                          
                                                            Email: fiziks.physics@gmail.com                                                                   18 
fiziks
Institute for NET/JRF, GATE, IIT‐JAM, JEST, TIFR and GRE in PHYSICAL SCIENCES 
 
 
Q48. If s1 and s2 are the spin operators of the two electrons of a He atom, the value of
 
s1 .s 2 for the ground state is

3 3 1 2
(a)   2 (b)   2 (c) 0 (d) 
2 4 4
Ans.: (b)
   1 1   s s  1 2  s1 s1  1 2  s 2 s 2  1 2
Solution: s  s1  s2 s1  , s1  , s  0,1 , s1 .s 2 
2 2 2
3 3
22  2  2
  4 4  3 2
For s  1, s1.s2 
2 4
3 3
02  2  2
  4 4   3 2
s  0, s1.s2 
2 4
Q49. A two-dimensional square rigid box of side L contains six non-interacting electrons at
T  0 K . The mass of the electron is m . The ground state energy of the system of
 2 2
electrons, in units of is _________.
2mL2
Ans.: 24

Solution: 2 
1 2

 12  2  2
 4
2 2

 12  2  2

24 2  2
2mL2 2mL2 2mL2
Q50.  x ,  y and  z are the Pauli matrices. The expression 2 x y   y x is equal to

(a)  3i z (b)  i z (c) i z (d) 3i z


Ans.: (c)
Solution: 2 x y   y x   x y   x y   y x   x y  i z

Q51. If x and p are the x components of the position and the momentum operators of a

particle respectively, the commutator x 2 , p 2 is  


(a) i  xp  px  (b) 2i  xp  px  (c) i  xp  px  (d) 2i  xp  px 
Ans.: (d)

                                                                                
Head office  Branch office 
 
fiziks, H.No. 23, G.F, Jia Sarai,  Anand Institute of Mathematics, 
 
Near IIT, Hauz Khas, New Delhi‐16  28‐B/6, Jia Sarai, Near IIT 
 
Phone: 011‐26865455/+91‐9871145498 Hauz Khas, New Delhi‐16 
                                                   
                                             Website: www.physicsbyfiziks.com                                                                                          
                                                            Email: fiziks.physics@gmail.com                                                                   19 
fiziks
Institute for NET/JRF, GATE, IIT‐JAM, JEST, TIFR and GRE in PHYSICAL SCIENCES 
 
2 2
 2
 
2
  
Solution: x , p  p x , p  x p p  2ipx  2ixp  2i  xp  px 

Q52. Let l , m be the simultaneous eigenstates of L2 and L z . Here L is the angular

momentum operator with Cartesian components L x , L y , L z , l is the angular momentum

quantum number and m is the azimuthal quantum number. The value of

1, 0 ( Lx  iLy ) 1, 1 is

(a) 0 (b)  (c) 2 (d) 3


Ans.: (c)

Solution: 1, 0 ( Lx  iLy 1, 1  1, 0 L 1, 1  2 1, 0 1, 0  2

Q53. For the parity operator P , which of the following statements is NOT true?
(a) P†  P (b) P 2   P (c) P 2  I (d) P †  P 1
Ans.: (b)
Q54. The state of a system is given by
  1  2  2  3 3

Where 1 , 2 and 3 form an orthonormal set. The probability of finding the system

in the state  2 is ________. (Give your answer upto two decimal places)

22 4 4 2
Ans probability that  in state 2      0.28
2 2
1 2 32
1  4  9 14 7
Q55. A particle of mass m and energy E , moving in the positive x
V0
direction, is incident on a step potential at x  0 , as indicated in the
E
figure. The height of the potential is V0 , where V0  E . At x  x0 ,

1
where x0  0 , the probability of finding the electron is times the
e x0 x  x0
2mV0  E 
probability of finding it at x  0 . If   , the value of x0 is
2
2 1 1 1
(a) (b) (c) (d)
  2 4
                                                                                
Head office  Branch office 
 
fiziks, H.No. 23, G.F, Jia Sarai,  Anand Institute of Mathematics, 
 
Near IIT, Hauz Khas, New Delhi‐16  28‐B/6, Jia Sarai, Near IIT 
 
Phone: 011‐26865455/+91‐9871145498 Hauz Khas, New Delhi‐16 
                                                   
                                             Website: www.physicsbyfiziks.com                                                                                          
                                                            Email: fiziks.physics@gmail.com                                                                   20 
fiziks
Institute for NET/JRF, GATE, IIT‐JAM, JEST, TIFR and GRE in PHYSICAL SCIENCES 
 
Ans.: (c)
1 1
Solution:  e 2 x0  e 1  e 2 x0  x0 
e 2
GATE-2017

Q56. The Compton wavelength of a proton is…………….. fm. (up to two decimal places).
Ans. : 3  108
Solution:  m p  1.67 1027 kg , h  6.626  1034 Js, e  1.602 1019 C , c  3 108 ms 1 

p2 1 2
Q57. A one dimensional simple harmonic oscillator with Hamiltonian H 0   kx is
2m 2
subjected to a small perturbation, H1   x   x 3   x 4 . The first order correction to the
ground state energy is dependent on
(a) only  (b)  and 
(c)  and  (d) only 
Ans. : (d)

Solution: H1   x   x 3   x 4 E1g   x   x3   x 4

x  0, x3  0, x 4  0
  
Q58. For the Hamiltonian H  a0 I  b . where a0  R, b is a real vector, I is the 2  2

identity matrix, and  are the Pauli matrices, the ground state energy is
(a) b (b) 2a0  b (c) a0  b (d) a0

Ans. : (c)
  1 0 0 1  0 i   1 0   a0  bz bx  iby 
Solution: a0 I  b .  a0    bx    by    bz   
0 1 1 0 i 0   0 1  bx  iby a0  bz 

   a0  bz bx  iby 
H  a0 I  b .   
 bx  iby a0  bz 

                                                                                
Head office  Branch office 
 
fiziks, H.No. 23, G.F, Jia Sarai,  Anand Institute of Mathematics, 
 
Near IIT, Hauz Khas, New Delhi‐16  28‐B/6, Jia Sarai, Near IIT 
 
Phone: 011‐26865455/+91‐9871145498 Hauz Khas, New Delhi‐16 
                                                   
                                             Website: www.physicsbyfiziks.com                                                                                          
                                                            Email: fiziks.physics@gmail.com                                                                   21 
fiziks
Institute for NET/JRF, GATE, IIT‐JAM, JEST, TIFR and GRE in PHYSICAL SCIENCES 
 
 a0  bz   bx  iby 
For eigen value  0
 b x  ib y a0  b z   

 a0  bz    a0  bz      bx2  by2   0
1  a0  b , 1  a0  b
Q59. The degeneracy of the third energy level of a 3-dimensional isotropic quantum harmonic
oscillator is
(a) 6 (b) 12 (c) 8 (d) 10
Ans. : (a)
Solution: First energy level is n  0
Second energy level is n  1
Third energy level is n  2

Degeneracy of third level


 n  1 n  2   3  4  6
2 2
Q60. A free electron of energy 1 eV is incident upon a one-dimensional finite potential step of
height 0.75 eV . The probability of its reflection from the barrier is…………. (up to two
decimal places).
Ans. : 0.11
2 2
 E  E  V0   1  0.25   1  0.5 2
Solution: R           0.11
  
 E  E  V0   1  0.25   1 0.5 

Q61. Consider a one-dimensional potential well of width 3nm . Using the uncertainty principle

 
 x  p   , an estimate of the minimum depth of the well such that it has at least one
 2

bound state for an electron is ( me  9.311031 kg , h  6.626 1034 Js,

e  1.602  1019 C )
(a) 1  eV (b) 1meV (c) 1eV (d) 1MeV
Ans. : (b)

                                                                                
Head office  Branch office 
 
fiziks, H.No. 23, G.F, Jia Sarai,  Anand Institute of Mathematics, 
 
Near IIT, Hauz Khas, New Delhi‐16  28‐B/6, Jia Sarai, Near IIT 
 
Phone: 011‐26865455/+91‐9871145498 Hauz Khas, New Delhi‐16 
                                                   
                                             Website: www.physicsbyfiziks.com                                                                                          
                                                            Email: fiziks.physics@gmail.com                                                                   22 
fiziks
Institute for NET/JRF, GATE, IIT‐JAM, JEST, TIFR and GRE in PHYSICAL SCIENCES 
 
2
p  
Solution: E  , p   p 
2m 2x 2a

so E 


2
h

2  6.6 10  34 2

 .0011019 J  1 meV
31 18
8ma 2
32 ma
2 2
32 10  9.3110  9 10

x e
2  x2
Q62. The integral dx is equal to……….. (up to two decimal places).
0

Ans. : 0.44

x
2
Solution: The given integral is 2
e  x dx
0

dt
Let x 2  t then 2 xdx  dt  dx 
2 t
Thus the given integral can be written as
  3 
dt 1 1 1
t e   e  t t1/ 2 dt   e  t t 2 dt
t

0 2 t 20 20

1 3 1 1 1 
       
2 2 2 2 2 4
Hence the value of the integral up to two decimal places is 0.44 .
Q63. Which one of the following operators is Hermitian?

(a) i
p x x
2
 x 2 px 
(b) i
p x x
2
 x 2 px 
2 2
(c) ei px a (d) e  i px a
Ans. : (a)

Solution: A  i
p x x
2
 x 2 px 
, A  i

 p x    x p    i  p x  x p 
x
2 † 2
x

x
2 2
x

2 2 2

                                                                                
Head office  Branch office 
 
fiziks, H.No. 23, G.F, Jia Sarai,  Anand Institute of Mathematics, 
 
Near IIT, Hauz Khas, New Delhi‐16  28‐B/6, Jia Sarai, Near IIT 
 
Phone: 011‐26865455/+91‐9871145498 Hauz Khas, New Delhi‐16 
                                                   
                                             Website: www.physicsbyfiziks.com                                                                                          
                                                            Email: fiziks.physics@gmail.com                                                                   23 
fiziks
Institute for NET/JRF, GATE, IIT‐JAM, JEST, TIFR and GRE in PHYSICAL SCIENCES 
 
THERMODYNAMICS AND STATISTICAL PHYSICS SOLUTIONS
GATE 2010
Q1. A system of N non-interacting classical point particles is constrained to move on the two-
dimensional surface of a sphere. The internal energy of the system is
3 1 5
(a) Nk BT (b) Nk BT (c) Nk BT (d) Nk BT
2 2 2
Ans: (c)
Solution: There are 2 N degree of freedom.
Nk BT Nk BT
The internal energy of the system is   Nk BT
2 2
Q2. Which of the following atoms cannot exhibit Bose-Einstein condensation, even in
principle?
(a) 1H1 (b) 4H2 (c) 23Na11 (d) 30K19
Ans: (d)
Solution: For Bose-Einstein condensation:
Number of electron + number of proton + number of neutron = Even
For 30 K19
Number of proton = 19, Number of electron = 19, Number of neutron = 11.
19 + 19 + 11 = 49 this is odd. So it will not exhibit Bose-Einstein condensation.
Q3. For a two-dimensional free electron gas, the electronic density n, and the Fermi energy
EF, are related by

2mE F  2
3
mE F
(a) n  (b) n 
3  2 3
 2

mE F 23
2
mE F  3 2
(c) n  (d) n 
2 2 
Ans: (c)
EF
2m f E   1 if E  E F
Solution: n   g(E)f (E)dE ,
0
g(E)dE 
h2
dE , at T=0
0 if E  E F

                                                                                
Head office  Branch office 
 
fiziks, H.No. 23, G.F, Jia Sarai,  Anand Institute of Mathematics, 
 
Near IIT, Hauz Khas, New Delhi‐16  28‐B/6, Jia Sarai, Near IIT 
 
Phone: 011‐26865455/+91‐9871145498 Hauz Khas, New Delhi‐16 
                                                   
                                             Website: www.physicsbyfiziks.com                                                                                          
                                                            Email: fiziks.physics@gmail.com                                                                   1 
fiziks
Institute for NET/JRF, GATE, IIT‐JAM, JEST, TIFR and GRE in PHYSICAL SCIENCES 
 
2mE F mE F
n 
h 2
2 2
Q4. Which among the following sets of Maxwell relations is correct? (U-internal energy, H-
enthalpy, A-Helmholtz free energy and G-Gibbs free energy)
 U   U   H   H 
(a) T    and P    (b) V    and T   
 V  S  S V  P  S  S  P

 G   G   A   A 
(c) P    and V    (d) P    and S   
 V T  P  S  S T  P V
Ans: (b)
 H   H 
Solution: dH  TdS  VdP     T,  V
 S  P  P  S

 2V k 0T 3
Q5. Partition function for a gas of photons is given as ln Z  . The specific heat of
45 3 C 3
the photon gas varies with temperature as
(a) (b)

CV CV

T T
(c) (d)

CV
CV

T
T
Ans: (a)
 ln z  U 
Solution: U  K B T 2 , CV     CV  T .
3

T  T  v

                                                                                
Head office  Branch office 
 
fiziks, H.No. 23, G.F, Jia Sarai,  Anand Institute of Mathematics, 
 
Near IIT, Hauz Khas, New Delhi‐16  28‐B/6, Jia Sarai, Near IIT 
 
Phone: 011‐26865455/+91‐9871145498 Hauz Khas, New Delhi‐16 
                                                   
                                             Website: www.physicsbyfiziks.com                                                                                          
                                                            Email: fiziks.physics@gmail.com                                                                   2 
fiziks
Institute for NET/JRF, GATE, IIT‐JAM, JEST, TIFR and GRE in PHYSICAL SCIENCES 
 
Q6. From Q. no. 5, the pressure of the photon gas is
 2 k B T 3  2 k B T 4  2 k B T 4  2 k B T 3 / 2
(a) (b) (c) (d)
15 3 C 3 8 3 C 3 45 3 C 3 45 3 C 3
Ans: (c)

2  k 0 T 
4
  ln z 
Solution: P  KT   
 V T 45 3C3
GATE 2011

Q7. A Carnot cycle operates on a working substance between two reservoir at temperatures T1
and T2 with T1 > T2. During each cycle, an amount of heat Q1 is extracted from the
reservoir at T1 and an amount Q2 is delivered in the reservoir at T2. Which of the
following statements is INCORRECT?
(a) Work done in one cycle is Q1 – Q2
Q1 Q2
(b) 
T1 T2

(c) Entropy of the hotter reservoir decreases


(d) Entropy of the universe (consisting of the working substance and the two reservoirs)
increases
Ans: (a)
Solution: Entropy of hotter reservoirs decreases.
Q8. In a first order phase transition, at the transition temperature, specific heat of the system
(a) diverges and its entropy remains the same
(b) diverges and its entropy has finite discontinuity
(c) remains unchanged and its entropy has finite discontinuity
(d) has finite discontinuity and its entropy diverges
Ans: (b)
Q9. A system of N non-interacting and distinguishable particle of spin 1 is in thermodynamic
equilibrium. The entropy of the system is
(a) 2kB ln N (b) 3kB ln N (c) NkB ln 2 (d) NkB ln 3

                                                                                
Head office  Branch office 
 
fiziks, H.No. 23, G.F, Jia Sarai,  Anand Institute of Mathematics, 
 
Near IIT, Hauz Khas, New Delhi‐16  28‐B/6, Jia Sarai, Near IIT 
 
Phone: 011‐26865455/+91‐9871145498 Hauz Khas, New Delhi‐16 
                                                   
                                             Website: www.physicsbyfiziks.com                                                                                          
                                                            Email: fiziks.physics@gmail.com                                                                   3 
fiziks
Institute for NET/JRF, GATE, IIT‐JAM, JEST, TIFR and GRE in PHYSICAL SCIENCES 
 
Ans: (d)
Solution: S  k B  ln  ,  =3 is number of microstate. S  1; Sz  1, 0, 1
i

The entropy of the system is NkB ln 3.


Q10. A system has two energy levels with energies ε and 2ε. The lower level is 4-fold
degenerate while the upper level is doubly degenerate. If there are N non-interacting
classical particles in the system, which is in thermodynamic equilibrium at a temperature
T, the fraction of particles in the upper level is
1 1
(a) (b)
1  e  / k BT 1  2e  / k BT
1 1
(c)  / k BT
(d)
2e  4e 2  / k B T 2e  / k BT
 4e 2  / k B T
Ans: (b)
Solution: Partition function Z  4e / kT  2e / kT
2e 2/ kT
P2  
1
/ kT  2/ kT

4e  2e 1  2e/ kT
GATE 2012

Q11. The isothermal compressibility,  of an ideal gas at temperature T0, and V0, is given by
1 V 1 V P P
(a)  (b) (c)  V0 (d) V0
V0 P T0 V0 P T0 V T0 V T0

Ans: (c)
 P 
Solution: Isothermal compressibility   V  
 V T
Q12. For an ideal Fermi gas in three dimensions, the electron velocity VF at the Fermi surface
is related to electron concentration n as,
(a) VF  n 2 / 3 (b) VF  n (c) VF  n1 / 2 (d) V F  n1 / 3
Ans: (d)
1
Solution: E F  mV F2  E F  n 2 / 3  VF2  n 2 / 3  VF  n1 / 3 .
2

                                                                                
Head office  Branch office 
 
fiziks, H.No. 23, G.F, Jia Sarai,  Anand Institute of Mathematics, 
 
Near IIT, Hauz Khas, New Delhi‐16  28‐B/6, Jia Sarai, Near IIT 
 
Phone: 011‐26865455/+91‐9871145498 Hauz Khas, New Delhi‐16 
                                                   
                                             Website: www.physicsbyfiziks.com                                                                                          
                                                            Email: fiziks.physics@gmail.com                                                                   4 
fiziks
Institute for NET/JRF, GATE, IIT‐JAM, JEST, TIFR and GRE in PHYSICAL SCIENCES 
 
Q13. A classical gas of molecules, each of mass m, is in thermal equilibrium at the
absolute temperature T. The velocity components of the molecules along the Cartesian

axes are v x , v y and v z . The mean value of v x  v y  is


2

k BT 3 k BT 1 k BT 2k B T
(a) (b) (c) (d)
m 2 m 2 m m
Ans: (d)

Solution: Vx  Vy 
2k B T
 V x2  V y2  2 V x  V y  Vx2  Vy2  2 Vx    Vy 
2

m
2k B T
 Vx    Vy  0 and Vx2  V y2  .
m
Q14. The total energy, E of an ideal non-relativistic Fermi gas in three dimensions is given by
N 5/3
E where N is the number of particles and V is the volume of the gas. Identify the
V 2/3
CORRECT equation of state (P being the pressure),
1 2 5
(a) PV  E (b) PV  E (c) PV  E (d) PV  E
3 3 3
Ans: (b)
5 5
 E  2  N 3 2N 3 2
Solution: P         PV   E.
 V  N 3  V  3 V23 3
Q15. Consider a system whose three energy levels are given by 0, ε and 2ε. The energy level ε
is two-fold degenerate and the other two are non-degenerate. The partition function of the
1
system with   is given by
k BT

(a) 1  2e   (b) 2e    e 2  (c) (1  e   ) 2 (d) 1  e    e 2 


Ans: (b)
Solution: E1  0, E 2   , E 3  2 ; g1  1, g 2  2, g 3  1 where g1 , g 2 and g 3 are degeneracy.

The partition function Z  g1e   E1  g 2 e   E2  g 3 e   E3  1  2e    e   2  1  e     2

                                                                                
Head office  Branch office 
 
fiziks, H.No. 23, G.F, Jia Sarai,  Anand Institute of Mathematics, 
 
Near IIT, Hauz Khas, New Delhi‐16  28‐B/6, Jia Sarai, Near IIT 
 
Phone: 011‐26865455/+91‐9871145498 Hauz Khas, New Delhi‐16 
                                                   
                                             Website: www.physicsbyfiziks.com                                                                                          
                                                            Email: fiziks.physics@gmail.com                                                                   5 
fiziks
Institute for NET/JRF, GATE, IIT‐JAM, JEST, TIFR and GRE in PHYSICAL SCIENCES 
 
GATE 2013

Q16. If Planck’s constant were zero, then the total energy contained in a box filled with
radiation of all frequencies at temperature T would be ( k is the Boltzmann constant and
T is nonzero)
3
(a) zero (b) Infinite (c) kT (d) kT
2
Ans: (d)
Solution: If Planck’s constant were zero, then the system behaved as a classical system and the
energy is kT .
Q17. Across a first order phase transition, the free energy is
(a) proportional to the temperature
(b) a discontinuous function of the temperature
(c) a continuous function of the temperature but its first derivative is discontinuous
(d) such that the first derivative with respect to temperature is continuous
Ans: (c)
Q18. Two gases separated by an impermeable but movable partition are allowed to freely
exchange energy. At equilibrium, the two sides will have the same
(a) pressure and temperature (b) volume and temperature
(c) pressure and volume (d) volume and energy
Ans: (a)
Q19. The entropy function of a system is given by S E   aEE0  E  where a and E0 are
positive constants. The temperature of the system is
(a) negative for some energies (b) increases monotonically with energy
(c) decreases monotonically with energy (d) Zero
Ans: (a)
Solution: From first and second law of thermodynamics
1  S  1
TdS  dU  PdV  dS   dU  PdV       E  U
T  E V T

                                                                                
Head office  Branch office 
 
fiziks, H.No. 23, G.F, Jia Sarai,  Anand Institute of Mathematics, 
 
Near IIT, Hauz Khas, New Delhi‐16  28‐B/6, Jia Sarai, Near IIT 
 
Phone: 011‐26865455/+91‐9871145498 Hauz Khas, New Delhi‐16 
                                                   
                                             Website: www.physicsbyfiziks.com                                                                                          
                                                            Email: fiziks.physics@gmail.com                                                                   6 
fiziks
Institute for NET/JRF, GATE, IIT‐JAM, JEST, TIFR and GRE in PHYSICAL SCIENCES 
 
 S 
S E   aE E0  E   
1
    E0  E    E    E0  2 E   T  .
 E V   E0  2 E 
Q20. Consider a linear collection of N independent spin ½ particles, each at a fixed location.
The entropy of this system is ( k is the Boltzmann constant)

(d) Nk ln 2 
1
(a) zero (b) Nk (c) Nk
2
Ans: (d)
Solution: There are two microstates possible for one so entropy is given by Nk ln 2 
Q21. Consider a gas of atoms obeying Maxwell-Boltzmann statistics. The average value of
   
e a p over all the moments p of each of the particles (where a is a constant vector and a
is the magnitude, m is the mass of each atom, T is temperature and k is Boltzmann’s
constant) is,
1 3
 a 2 mkT  a 2 mkT
(a) one (b) zero (c) e 2
(d) e 2

Ans: (c)
  

   f  p , p , p e dpx dp y dpz where f  px , p y , pz  is Maxwell probability


   
Solution:  e p.a   x y z
p .a

  

distribution at temperature T.
 p x2  p 2y  p z2
    
 Ae  Ae  Ae
py ay
e  
p.a
x
2 mkT
e px ax
dpx y
2 mkT
e dp y z
2 mkT
e pz az dpz
  

 ( a x2  a 2y  a z2 ) mkT  ( px  mkTax )2  ( p y  mkTa y )2  ( pz  mkTa z )2


    
e   e  Ae dx  Ay e dy  Ax e
p.a 2 2 mkT 2 mkT 2 mkT
x
  

 ( ax2  a 2y  a z2 ) mkT 1
   a 2 mkT
e p .a
e 2
.1.1.1 = e 2

Common Data for Questions 22 and 23: There are four energy levels E , 2E , 3 E and
4 E (where E  0 ). The canonical partition function of two particles is, if these particles
are
Q22. Two identical fermions

                                                                                
Head office  Branch office 
 
fiziks, H.No. 23, G.F, Jia Sarai,  Anand Institute of Mathematics, 
 
Near IIT, Hauz Khas, New Delhi‐16  28‐B/6, Jia Sarai, Near IIT 
 
Phone: 011‐26865455/+91‐9871145498 Hauz Khas, New Delhi‐16 
                                                   
                                             Website: www.physicsbyfiziks.com                                                                                          
                                                            Email: fiziks.physics@gmail.com                                                                   7 
fiziks
Institute for NET/JRF, GATE, IIT‐JAM, JEST, TIFR and GRE in PHYSICAL SCIENCES 
 
2 E 4 E 6 E 8 E
(a) e e e e
(b) e 3 E  e 4 E  e 5 E  e 6 E  e 7 E

(c) e  E  e  2 E  e 3 E  e  4 E 
2

(d) e 2 E  e 4 E  e 6 E  e 8 E
Ans: (b)
Solution: The possible value of Energy for two Fermions
E1  3E, E2  4 E , E3  5E, E4  6 E , E5  7 E

The partition function is Z  e 3E  e 4 E  2e 5 E  e 6 E  e 7 E


Q23. Two distinguishable particles
(a) e 2 E  e 4 E  e 6 E  e 8 E
(b) e 3 E  e 4 E  e 5 E  e 6 E  e 7 E

(c) e  E  e  2 E  e 3 E  e  4 E 
2

(d) e 2 E  e 4 E  e 6 E  e 8 E
Ans: (c)
Solution: When two particles are distinguishable then minimum value of Energy is 2 E and
maximum value is 8 E .

So from checking all four options Z  e  E  e 2 E  e 3 E  e 4 E 


2

GATE 2014

Q24. For a gas under isothermal condition its pressure p varies with volume V as P  V 5 / 3 .
The bulk modules B is proportional to
(a) V 1 / 2 (b) V 2 / 3 (c) V 3 / 5 (d) V 5 / 3
Ans: (d)
dP
Solution: P  KV 5 / 3 , B  V B  V 5 / 3
dV
Q25. At a given temperature T , the average energy per particle of a non-interacting gas of
two-dimensional classical harmonic oscillators is _________ k B T

                                                                                
Head office  Branch office 
 
fiziks, H.No. 23, G.F, Jia Sarai,  Anand Institute of Mathematics, 
 
Near IIT, Hauz Khas, New Delhi‐16  28‐B/6, Jia Sarai, Near IIT 
 
Phone: 011‐26865455/+91‐9871145498 Hauz Khas, New Delhi‐16 
                                                   
                                             Website: www.physicsbyfiziks.com                                                                                          
                                                            Email: fiziks.physics@gmail.com                                                                   8 
fiziks
Institute for NET/JRF, GATE, IIT‐JAM, JEST, TIFR and GRE in PHYSICAL SCIENCES 
 
( k B is the Boltzmann constant)

Ans: 2k B T
Q26. Which one of the following is a fermion?
(a)  particle (b) 4 Be 2 nucleus
(c) Hydrogen atom (d) deuteron
Ans (d)
Solution: Total number of particles: P  N  E  3
Q27. For a free electron gas in two dimensions the variations of the density of states. N E  as a
function of energy E , is best represented by
(a) (b)

N E  N E 

E E

(c) (d)

N E  N E 

E E

ans (c)
N E   E 0
Q28. For a system of two bosons each of which can occupy any of the two energy levels 0 and
1
 the mean energy of the system at temperature T with   is given by
k T

                                                                                
Head office  Branch office 
 
fiziks, H.No. 23, G.F, Jia Sarai,  Anand Institute of Mathematics, 
 
Near IIT, Hauz Khas, New Delhi‐16  28‐B/6, Jia Sarai, Near IIT 
 
Phone: 011‐26865455/+91‐9871145498 Hauz Khas, New Delhi‐16 
                                                   
                                             Website: www.physicsbyfiziks.com                                                                                          
                                                            Email: fiziks.physics@gmail.com                                                                   9 
fiziks
Institute for NET/JRF, GATE, IIT‐JAM, JEST, TIFR and GRE in PHYSICAL SCIENCES 
 
  2  
e  2 e 1   e  
(a) (b)
1  2e     e  2   2e     e  2  
2 e     e 2    e     2 e 2  
(c) (d)
2  e    e 2   2  e    e  2  
if both particle will in ground state the energy will 0 which is non degenerate
if one particle is in ground state and other is in first excited state then energy is  and
non degenerate
if both particle will in first excited state the energy will 2 which is non degenerate
then partition function is Z  1  exp   exp 2
exp    2 exp  2 
average value of energy
1  exp    exp  2 
no one ans. is correct .
Q29. Consider a system of 3 fermions which can occupy any of the 4 available energy states
with equal probability. The entropy of the system is
(a) k B ln 2 (b) 2k B ln 2 (c) 2k B ln 4 (d) 3k B ln 4
Ans: (b)
Solution: Number of ways that 3 fermions will adjust in 4 available energy is 4
C3  4 so

entropy is k B ln 4 = 2k B ln 2
GATE 2015
Q30. In Boss-Einstein condensates, the particles
(a) have strong interparticle attraction
(b) condense in real space
(c) have overlapping wavefunctions
(d) have large and positive chemical potential
Ans.: (c)
Solution: In Bose- Einstein condensates, the particles have overlapping wave function.

                                                                                
Head office  Branch office 
 
fiziks, H.No. 23, G.F, Jia Sarai,  Anand Institute of Mathematics, 
 
Near IIT, Hauz Khas, New Delhi‐16  28‐B/6, Jia Sarai, Near IIT 
 
Phone: 011‐26865455/+91‐9871145498 Hauz Khas, New Delhi‐16 
                                                   
                                             Website: www.physicsbyfiziks.com                                                                                          
                                                            Email: fiziks.physics@gmail.com                                                                   10 
fiziks
Institute for NET/JRF, GATE, IIT‐JAM, JEST, TIFR and GRE in PHYSICAL SCIENCES 
 
Q31. For a black body radiation in a cavity, photons are created and annihilated freely as a
result of emission and absorption by the walls of the cavity. This is because
(a) the chemical potential of the photons is zero
(b) photons obey Pauli exclusion principle
(c) photons are spin-1 particles
(d) the entropy of the photons is very large
Ans.: (a)
Solution: The chemical potential of photon is zero
1
Q32. Consider a system of N non-interacting spin  particles, each having a magnetic
2

moment  , is in a magnetic field B  Bzˆ . If E is the total energy of the system, the
number of accessible microstates  is given by
 E 
 N  !
N! B 
(a)   (b)   
1 E  1 E   E 
 N   !  N  !  N  !
2 B  2  B   B 

1 E  1 E  N!
(c)    N  !  N  ! (d)  
2 B  2  B   E 
 N  !
 B 
Ans.: (a)
1
Solution: Number of microstate is N
Cn1 where n1 is number of particle in  state and
2
n2  N  n1 is

1
Number of state in  state.
2
1 E  1 E 
n1   N   , n2   N 
2 B  2  B 
N
So number of microstate is
1 E  1 E 
 N   N
2 B  2   B 

                                                                                
Head office  Branch office 
 
fiziks, H.No. 23, G.F, Jia Sarai,  Anand Institute of Mathematics, 
 
Near IIT, Hauz Khas, New Delhi‐16  28‐B/6, Jia Sarai, Near IIT 
 
Phone: 011‐26865455/+91‐9871145498 Hauz Khas, New Delhi‐16 
                                                   
                                             Website: www.physicsbyfiziks.com                                                                                          
                                                            Email: fiziks.physics@gmail.com                                                                   11 
fiziks
Institute for NET/JRF, GATE, IIT‐JAM, JEST, TIFR and GRE in PHYSICAL SCIENCES 
 
Q33. The average energy U of a one dimensional quantum oscillator of frequency  and in
contact with a heat bath at temperature T is given by
1 1  1 1 
(a) U   coth    (b) U   sinh    
2 2  2 2 
1 1  1 1 
(c) U   tanh    (d) U   cosh   
2 2  2 2 
Ans.: (a)
  1
 Ei    n   
 1 1
Solution:  Z   e  e  2
where E   n     Z 
0  2    
2sinh  
 2 
 
   1      
U  ln Z  U   ln   U  coth  
        2  2 
2sinh  
  2  
Q34. The entropy of a gas containing N particles enclosed in a volume V is given by
 aVE 3 / 2 
S  Nk B ln  , where E is the total energy, a is a constant and k B is the
 N
5/ 2

Boltzmann constant. The chemical potential  of the system at a temperature T is given
by
  aVE 3 / 2  5   aVE 3 / 2  3
(a)    k B T ln    (b)    k B T ln   
  N   N
5/ 2 5/ 2
 2  2

  aVE 3/ 2  5    aVE 3 / 2  3
(c)   k BT ln  3/ 2 
  (d)   k B T ln   
  N   N
3/ 2
 2  2
Ans.: (a)
 3
  3

 G   aVE 2
  S  Nk ln  aVE 2

Solution:     S   Nk ln
 T  P
B  5  B  5 
 N2   N2 
   

                                                                                
Head office  Branch office 
 
fiziks, H.No. 23, G.F, Jia Sarai,  Anand Institute of Mathematics, 
 
Near IIT, Hauz Khas, New Delhi‐16  28‐B/6, Jia Sarai, Near IIT 
 
Phone: 011‐26865455/+91‐9871145498 Hauz Khas, New Delhi‐16 
                                                   
                                             Website: www.physicsbyfiziks.com                                                                                          
                                                            Email: fiziks.physics@gmail.com                                                                   12 
fiziks
Institute for NET/JRF, GATE, IIT‐JAM, JEST, TIFR and GRE in PHYSICAL SCIENCES 
 
 3

 aVE 
2

 G   Nk BT ln  5
  ln A
N2
  3
  5 
  aVE 2
 5
 3
 G     N 2  2 
      k BT ln  Nk BT . aVE 2 
 N 
5 3 7 
 N2 aVE 2 N 2 
 

  3
 
aVE
    k BT ln    5
2

 5  2
 
  N  
2

GATE-2016
Q35. The total power emitted by a spherical black body of radius R at a temperature T is P1 .
R
Let P2 be the total power emitted by another spherical black body of radius kept at
2
P1
temperature 2T . The ratio, is _______. (Give your answer upto two decimal places)
P2
Ans.: 0.25

p1 R12T14 R 2T 4 4 1
Solution: p  AT   2 4  4
   0.25
p2 R2 T2  R  2
16 4
   2T 
4

2
Q36. The entropy S of a system of N spins, which may align either in the upward or in the
downward direction, is given by S   k B N  p ln p  1  p  In(1  p)  . Here k B is the

Boltzmann constant. The probability of alignment in the upward direction is p. The value
of p, at which the entropy is maximum, is _______. (Give your answer upto one decimal
place)
Ans.: 0.5
Solution: S   k B N  p ln p  1  p  In(1  p) 

                                                                                
Head office  Branch office 
 
fiziks, H.No. 23, G.F, Jia Sarai,  Anand Institute of Mathematics, 
 
Near IIT, Hauz Khas, New Delhi‐16  28‐B/6, Jia Sarai, Near IIT 
 
Phone: 011‐26865455/+91‐9871145498 Hauz Khas, New Delhi‐16 
                                                   
                                             Website: www.physicsbyfiziks.com                                                                                          
                                                            Email: fiziks.physics@gmail.com                                                                   13 
fiziks
Institute for NET/JRF, GATE, IIT‐JAM, JEST, TIFR and GRE in PHYSICAL SCIENCES 
 
dS 1 1
For maximum entropy  0  ln p  p   ln 1  p   1  p    1  0
dp p 1 p

 p 
ln p  1  ln 1  p   1  0  ln    0  p  1  p  p  0.5
 1 p 
Q37. For a system at constant temperature and volume, which of the following statements is
correct at equilibrium?
(a) The Helmholtz free energy attains a local minimum.
(b) The Helmholtz free energy attains a local maximum.
(c) The Gibbs free energy attains a local minimum.
(d) The Gibbs free energy attains a local maximum.
Ans.: (a)
Solution: dF   SdT  PdV
Q38. N atoms of an ideal gas are enclosed in a container of volume V . The volume of the
container is changed to 4V , while keeping the total energy constant. The change in the
entropy of the gas, in units of Nk B ln 2 , is _______, where k B is the Boltzmann constant.
Ans.: 2
1
Solution: S1   Nk B ln1 S2   Nk B ln
4
S  S2  S1  Nk B ln 4  2 Nk B ln 2

Q39. Consider a system having three energy levels with energies 0, 2 and 3 ,with
respective degeneracies of 2,2 and 3 . Four bosons of spin zero have to be
accommodated in these levels such that the total energy of the system is 10  . The
number of ways in which it can be done is ______.
Ans.: 18
Solution: The system have energy 10  if out of four boson two boson are in energy level
2 and two boson are in energy level 3 and
ni  gi  1
W  n1  2, g1  2 and n2  2, g 2  3
i ni gi  1

                                                                                
Head office  Branch office 
 
fiziks, H.No. 23, G.F, Jia Sarai,  Anand Institute of Mathematics, 
 
Near IIT, Hauz Khas, New Delhi‐16  28‐B/6, Jia Sarai, Near IIT 
 
Phone: 011‐26865455/+91‐9871145498 Hauz Khas, New Delhi‐16 
                                                   
                                             Website: www.physicsbyfiziks.com                                                                                          
                                                            Email: fiziks.physics@gmail.com                                                                   14 
fiziks
Institute for NET/JRF, GATE, IIT‐JAM, JEST, TIFR and GRE in PHYSICAL SCIENCES 
 
2  2 1 2  3 1
  3  6  18
2 2 1 2 3 1
Q40. A two-level system has energies zero and E . The level with zero energy is non-
degenerate, while the level with energy E is triply degenerate. The mean energy of a
classical particle in this system at a temperature T is
E E E E
k BT k BT k BT k BT
Ee Ee 3Ee 3Ee
(a) E
(b) E
(c) E
(d) E

1  3e k BT
1 e k BT
1 e k BT
1  3e k BT
Ans.: (d)
Ei
 0 E E
 gi Ei e kT  
k BT
0e kT  3 E  e kT 3Ee
Solution: E  i
E
 0 E
 E
 i  
 gi e kT e kT  3  e kT 1  3e k BT
i

GATE-2017

Q41. Consider a triatomic molecule of the shape shown in the figure in three
dimensions. The heat capacity of this molecule at high temperature
(temperature much higher than the vibrational and rotational energy scales
of the molecule but lower than its bond dissociation energies) is:
3 9
(a) kB (b) 3k B (c) kB (d) 6k B
2 2
Ans. : (d)
Solution: If given molecule are low temperature i.e. atoms are attached to rigid rod then degree
6 k BT
of freedom is 6 so internal energy is is but at high temperature vibration mode
2
will active so there are three extra vibration mode will active so total enegy
U  3k BT  3k BT  6k BT

 U 
CV     6k B
 T V

                                                                                
Head office  Branch office 
 
fiziks, H.No. 23, G.F, Jia Sarai,  Anand Institute of Mathematics, 
 
Near IIT, Hauz Khas, New Delhi‐16  28‐B/6, Jia Sarai, Near IIT 
 
Phone: 011‐26865455/+91‐9871145498 Hauz Khas, New Delhi‐16 
                                                   
                                             Website: www.physicsbyfiziks.com                                                                                          
                                                            Email: fiziks.physics@gmail.com                                                                   15 
fiziks
Institute for NET/JRF, GATE, IIT‐JAM, JEST, TIFR and GRE in PHYSICAL SCIENCES 
 
Q42. A reversible Cannot engine is operated between temperatures T1 and T2 T2  T1  with a

photon gas as the working substance. The efficiency of the engine is


3/ 4 4/3
3T T T  T 
(a) 1  1 (b) 1  1 (c) 1   1  (d) 1   1 
4T2 T2  T2   T2 
Ans. : (b)
Solution: Efficiency of carnot enegine does not depends on nature of working substance rather
depends on temperature of source and sink
T1
  1
T2

Q43. Water freezes at 00 C at atmospheric pressure 1.01 105 Pa  . The densities of water and

ice at this temperature and pressure are 1000 kg / m3 and 934 kg / m3 respectively. The

latent heat of fusion is 3.34 105 J / kg . The pressure required for depressing the melting

temperature of ice by 100 C is…………… GPa . (up to two decimal places)


Ans. : 1105
p2
 dP  L L T2 dT L T
Solution:      dP    P2  P1  ln 2
 dT V T  v2  v1  P1  v2  v1  T1 T  v2  v1  T1
L T
 P2  P1  ln 2  1 105
 v2  v1  T1
Q44. Consider N non- interacting, distinguishable particles in a two-level system at
temperature T . The energies of the levels are 0 and  , where   0 . In the high
temperature limit  k BT    , what is the population of particles in the level with energy

?
N N 3N
(a) (b) N (c) (d)
2 4 4
Ans. : (a)

                                                                                
Head office  Branch office 
 
fiziks, H.No. 23, G.F, Jia Sarai,  Anand Institute of Mathematics, 
 
Near IIT, Hauz Khas, New Delhi‐16  28‐B/6, Jia Sarai, Near IIT 
 
Phone: 011‐26865455/+91‐9871145498 Hauz Khas, New Delhi‐16 
                                                   
                                             Website: www.physicsbyfiziks.com                                                                                          
                                                            Email: fiziks.physics@gmail.com                                                                   16 
fiziks
Institute for NET/JRF, GATE, IIT‐JAM, JEST, TIFR and GRE in PHYSICAL SCIENCES 
 

exp 
Solution: P     kT population of particle in the level with energy  is

1  exp
kT
 
exp exp
kT kT 1 N
NP     N for  k BT    NP     N N 
  11 2
1  exp 1  exp
kT kT
u
Q45. The energy density and pressure of a photon gas are given by u  aT 4 and P  . Where
3
T is the temperature and a is the radiation constant. The entropy per unit volume is given
by  aT 3 . The value of  is…………… (up to two decimal places)
Ans. : 1.33
 S   U 
Solution: TdS  dU  PdV  T     P
 V T  V T

 S  1  U  P aT 4 aT 4 4 3
         aT  1.33
 V T T  V T T T 3T 3
Q46. Consider two particles and two non-degenerate quantum levels 1 and 2. Level 1 always
contains a particle. Hence, what is the probability that level 2 also contains a particle for
each of the two cases:
(i) when the two particles are distinguishable and (ii) when the two particles are bosons?
1 1 1 1
(a) (i) and (ii) (b) (i) and (ii)
2 3 2 2
2 1
(c) (i) and (ii) (d) (i) 1 and (ii) 0
3 2
Ans. : (c) A
B
2
Solution: A B AB For distinguishable h So
3

A
1
A AA For indistinguishable h So
2

                                                                                
Head office  Branch office 
 
fiziks, H.No. 23, G.F, Jia Sarai,  Anand Institute of Mathematics, 
 
Near IIT, Hauz Khas, New Delhi‐16  28‐B/6, Jia Sarai, Near IIT 
 
Phone: 011‐26865455/+91‐9871145498 Hauz Khas, New Delhi‐16 
                                                   
                                             Website: www.physicsbyfiziks.com                                                                                          
                                                            Email: fiziks.physics@gmail.com                                                                   17 
fiziks
Institute for NET/JRF, GATE, IIT‐JAM, JEST, TIFR and GRE in PHYSICAL SCIENCES 
 
ELECTRONICS SOLUTIONS

GATE-2010
Q1. The voltage resolution of a 12-bit digital to analog converter (DAC), whose output varies
from  10 V to  10 V is, approximately
(a) 1 mV (b) 5 mV (c) 20 mV (d) 100 mV
Ans: (b)
20V
Solution: Voltage resolution=  4.8 mV
212  1
Q2. The figure shows a constant current source charging a capacitor that is initially
uncharged. Vout

If the switch is closed at t = 0, which of the following plots depicts correctly the output
voltage of the circuit as a function of time?
(a) (b)

Vout Vout

t t
(c) (d)

Vout Vout

t t

Ans: (d)
CdV0 I
Solution: I 0   V0  0 t
dt C

                                                                                
Head office  Branch office 
 
fiziks, H.No. 23, G.F, Jia Sarai,  Anand Institute of Mathematics, 
 
Near IIT, Hauz Khas, New Delhi‐16  28‐B/6, Jia Sarai, Near IIT 
 
Phone: 011‐26865455/+91‐9871145498 Hauz Khas, New Delhi‐16 
                                                   
                                             Website: www.physicsbyfiziks.com                                                                                          
                                                            Email: fiziks.physics@gmail.com                                                                   1 
fiziks
Institute for NET/JRF, GATE, IIT‐JAM, JEST, TIFR and GRE in PHYSICAL SCIENCES 
 
Q3. In one of the following circuits, negative feedback does not operate for a negative input.
Which one is it? The opamps are running from ± 15 V supplies.
(a) (b)
 

 

(c) (d)
 

 

Ans: (c)
Q4. For any set of inputs, A and B, the following circuits give the same output, Q, except one.
Which one is it?
(a)  (b) 
Q
 Q 

(c)  (d) 
Q
 Q 

Ans: (d)

                                                                                
Head office  Branch office 
 
fiziks, H.No. 23, G.F, Jia Sarai,  Anand Institute of Mathematics, 
 
Near IIT, Hauz Khas, New Delhi‐16  28‐B/6, Jia Sarai, Near IIT 
 
Phone: 011‐26865455/+91‐9871145498 Hauz Khas, New Delhi‐16 
                                                   
                                             Website: www.physicsbyfiziks.com                                                                                          
                                                            Email: fiziks.physics@gmail.com                                                                   2 
fiziks
Institute for NET/JRF, GATE, IIT‐JAM, JEST, TIFR and GRE in PHYSICAL SCIENCES 
 
GATE-2011

Q5. Which of the following statements is CORRECT for a common emitter amplifier
circuit?
(a) The output is taken from the emitter
(b) There is 180o phase shift between input and output voltages
(c) There is no phase shift between input and output voltages
(d) Both p-n junctions are forward biased
Ans: (b)
Q6. For an intrinsic semiconductor, me* and mh* are respectively the effective masses of
electrons and holes near the corresponding band edges. At a finite temperature the
position of the Fermi level
(a) depends on me* but not on mh* (b) depends on mh* but not on me*
(c) depends on both me* and mh* (d) depends neither on me* nor on mh*
Ans: (c)
Q7. In the following circuit, the voltage across and the current through the 2 kΩ resistance are

500 1k

20V 10V
2k
30V

(a) 20 V, 10 mA (b) 20 V, 5 mA (c) 10 V, 10 mA (d) 10 V, 5 mA


Ans: (d)

                                                                                
Head office  Branch office 
 
fiziks, H.No. 23, G.F, Jia Sarai,  Anand Institute of Mathematics, 
 
Near IIT, Hauz Khas, New Delhi‐16  28‐B/6, Jia Sarai, Near IIT 
 
Phone: 011‐26865455/+91‐9871145498 Hauz Khas, New Delhi‐16 
                                                   
                                             Website: www.physicsbyfiziks.com                                                                                          
                                                            Email: fiziks.physics@gmail.com                                                                   3 
fiziks
Institute for NET/JRF, GATE, IIT‐JAM, JEST, TIFR and GRE in PHYSICAL SCIENCES 
 
Q8. In the following circuit, Tr1 and Tr2 are identical transistors having VBE = 0.7 V. The
current passing through the transistor Tr2 is

100
Tr2
5V

Tr1

(a) 57 mA (b) 50 mA (c) 48 mA (d) 43 mA


Ans: (d)
5  0.7
Solution: Current through 100 , I   43 mA
100
I  I C  2 I B  I C  43 mA .
Q9. The following Boolean expression
Y  A  B  C  D  A  B  C  D  A  B  C  D  A  B  C  D  A  B  C  D  A  B  C  D can
be simplified to
(a) A  B  C  A  D (b) A  B  C  A  D
(c) A  B  C  A  D (d) A  B  C  A  D
Ans: (c)
CD CD CD CD
AD
AB 1 1
AB 1 1
AB
AB 1 1

AB C

                                                                                
Head office  Branch office 
 
fiziks, H.No. 23, G.F, Jia Sarai,  Anand Institute of Mathematics, 
 
Near IIT, Hauz Khas, New Delhi‐16  28‐B/6, Jia Sarai, Near IIT 
 
Phone: 011‐26865455/+91‐9871145498 Hauz Khas, New Delhi‐16 
                                                   
                                             Website: www.physicsbyfiziks.com                                                                                          
                                                            Email: fiziks.physics@gmail.com                                                                   4 
fiziks
Institute for NET/JRF, GATE, IIT‐JAM, JEST, TIFR and GRE in PHYSICAL SCIENCES 
 
Q10. Consider the following circuit 1k 4k
 10V

 Vout
Vin
 10V
Which of the following correctly represents the output Vout corresponding to the input
Vin?
 5V  5V
(a) (b)
 2V  2V
Vin Vin
 2V time  2V time
 5V  5V

 10V
 10V Vout
Vout time
- 10V
time
- 10V

(c)  5V (d)
 5V
 2V
Vin  2V
 2V time Vin
 2V time
 5V
 5V

 10V
 10V
Vout
Vout
time
time - 10V
- 10V

Ans: (a)
 1   1 
Solution: Vut     10  2V , Vlt     10  2V .
1 4  1 4 
                                                                                
Head office  Branch office 
 
fiziks, H.No. 23, G.F, Jia Sarai,  Anand Institute of Mathematics, 
 
Near IIT, Hauz Khas, New Delhi‐16  28‐B/6, Jia Sarai, Near IIT 
 
Phone: 011‐26865455/+91‐9871145498 Hauz Khas, New Delhi‐16 
                                                   
                                             Website: www.physicsbyfiziks.com                                                                                          
                                                            Email: fiziks.physics@gmail.com                                                                   5 
fiziks
Institute for NET/JRF, GATE, IIT‐JAM, JEST, TIFR and GRE in PHYSICAL SCIENCES 
 
GATE-2012
Q11. If the peak output voltage of a full wave rectifier is 10 V, its d.c. voltage is
(a) 10.0 V (b) 7.07 V (c) 6.36 V (d) 3.18 V
Ans: (c)
2Vm 2  10 14  10 70
Solution: Vdc      6.36V
 22 / 7 22 11
Q12. A Ge semiconductor is doped with acceptor impurity concentration of 1015 atoms/cm3.
For the given hole mobility of 1800 cm2/V-s, the resistivity of the material is
(a) 0.288 Ω cm (b) 0.694 Ω cm (c) 3.472 Ω cm (d) 6.944 Ω cm
Ans: (c)
1 1 1
Solution:     15  3.47 cm
 N A e u h 10  1.6  10 19  1800
Q13. Identify the CORRECT energy band diagram for silcon doped with Arsenic. Here CB,
VB, ED and EF are conduction band, valence band, impurity level and Fermi level,
respectively.
(a) (b)
CB CB
ED
ED
EF
EF

VB VB

(c) (d)
CB CB

EF EF
ED ED
VB VB

Ans: (b)
Solution: N-type material ( Si doped with AS ).
                                                                                
Head office  Branch office 
 
fiziks, H.No. 23, G.F, Jia Sarai,  Anand Institute of Mathematics, 
 
Near IIT, Hauz Khas, New Delhi‐16  28‐B/6, Jia Sarai, Near IIT 
 
Phone: 011‐26865455/+91‐9871145498 Hauz Khas, New Delhi‐16 
                                                   
                                             Website: www.physicsbyfiziks.com                                                                                          
                                                            Email: fiziks.physics@gmail.com                                                                   6 
fiziks
Institute for NET/JRF, GATE, IIT‐JAM, JEST, TIFR and GRE in PHYSICAL SCIENCES 
 
 10V
Q14. Consider the following OP-AMP circuit
Which one of the following correctly represents the Vin 

output Vout corresponding to the input Vin? 5V Vout



4k
- 10V
1k

(a) (b)
 5V  5V
Vin Vin
 1V  1V
0V t 0V t

 10V  10V
Vout Vout

t t

 10V  10V
(c) (d)
 5V  5V
Vin Vin

0V t 0V t

 10V  10V
Vout Vout

t t

 10V  10V

Ans: (a)
 1 
Solution: Voltage at inverting input V2     5  1V .
1 4 
When vin  1V , v0  VCC and when vin  1V , v0  VCC
                                                                                
Head office  Branch office 
 
fiziks, H.No. 23, G.F, Jia Sarai,  Anand Institute of Mathematics, 
 
Near IIT, Hauz Khas, New Delhi‐16  28‐B/6, Jia Sarai, Near IIT 
 
Phone: 011‐26865455/+91‐9871145498 Hauz Khas, New Delhi‐16 
                                                   
                                             Website: www.physicsbyfiziks.com                                                                                          
                                                            Email: fiziks.physics@gmail.com                                                                   7 
fiziks
Institute for NET/JRF, GATE, IIT‐JAM, JEST, TIFR and GRE in PHYSICAL SCIENCES 
 
Q15. In the following circuit, for the output voltage to be V0   V1  V2 / 2  the ratio R1/R2 is
(a) 1/2 R

(b) 1  VCC
R
(c) 2 V1 

Vo
(d) 3 
V2
R1
- VCC
Ans: (d) R2

Solution: When V2  0, v01  V1

 R   R2 
when V1  0, v02  1     V2
 R   R1  R2 
V2 R2 1 R
Since V0  V1   2   1 3
2 R1  R2 2 R2
Q16. In the following circuit, the voltage drop across the ideal diode in forward bias condition
is 0.7V. The current passing through the diode is
(a) 0.5 mA
12k
(b) 1.0 mA
 24 Volt

(c) 1.5 mA 

(d) 2.0 mA 6k 3.3 k

Ans: (b)
Solution: Let current through 12k is I and through diode is I D
Then 0 .7  I D  3 .3   I  I D   6 (1)
and  24  I  12  I  I D   6  0 (2)
From (1) and (2) I D  1mA.

                                                                                
Head office  Branch office 
 
fiziks, H.No. 23, G.F, Jia Sarai,  Anand Institute of Mathematics, 
 
Near IIT, Hauz Khas, New Delhi‐16  28‐B/6, Jia Sarai, Near IIT 
 
Phone: 011‐26865455/+91‐9871145498 Hauz Khas, New Delhi‐16 
                                                   
                                             Website: www.physicsbyfiziks.com                                                                                          
                                                            Email: fiziks.physics@gmail.com                                                                   8 
fiziks
Institute for NET/JRF, GATE, IIT‐JAM, JEST, TIFR and GRE in PHYSICAL SCIENCES 
 
Q17. Consider the following circuit in which the current gain βdc of the transistor is 100.
 15 V

100 k 900 

100 

Which one of the following correctly represents the load line (collector current IC with
respect to collector-emitter voltage VCE) and Q-point of this circuit?
(a) (b)
15 mA Q - point 13 mA Q - point
(2 V, 13 mA) (2 V, 10 mA)
IC →

IC →

VCE  15 V VCE  15 V

(c) (d)
15 mA 13 mA
Q - point Q - point
(7.5 V, 7.5 mA) (7.5 V, 6.5 mA)
IC →

IC →

VCE  15 V VCE  15 V

Ans: (a)
VCC  V BE 15  0.7 14.3
Solution: I B    mA.
RB  RE 100  10  100 100
3

I C  I B  14.3mA  13mA , VCE  VCC  I C RC  RE   15  900  100  13  10 3  2V .

VCC 15
I C , Sat    15 mA.
R C  R E 1000
                                                                                
Head office  Branch office 
 
fiziks, H.No. 23, G.F, Jia Sarai,  Anand Institute of Mathematics, 
 
Near IIT, Hauz Khas, New Delhi‐16  28‐B/6, Jia Sarai, Near IIT 
 
Phone: 011‐26865455/+91‐9871145498 Hauz Khas, New Delhi‐16 
                                                   
                                             Website: www.physicsbyfiziks.com                                                                                          
                                                            Email: fiziks.physics@gmail.com                                                                   9 
fiziks
Institute for NET/JRF, GATE, IIT‐JAM, JEST, TIFR and GRE in PHYSICAL SCIENCES 
 
GATE-2013
Q18. What should be the clock frequency of a 6  bit A / D converter so that its maximum
conserved time is 32s ?
(a) 1 MHz (b) 2 MHz (c) 0.5 MHz (d) 4 MHz
Ans: (c)
Q19. A phosphorous doped silicon semiconductor (doping density: 1017/cm3) is heated from
100C to 200C. Which one of the following statements is CORRECT?
(a) Position of Fermi level moves towards conduction band
(b) Position of dopant level moves towards conduction band
(c) Position of Fermi level moves towards middle of energy gap
(d) Position of dopant level moves towards middle of energy gap
Ans: (c)

Statement for Linked Answer Questions 20 and 21:


Consider the following circuit
Q20. For this circuit the frequency above which the gain will decrease by 20 dB per decade is
(a) 15.9 kHz (b) 1.2kHz 10 k 
Vin  
(c) 5.6 kHz (d) 22.5 kHz
Vout 
1000pF 
Ans: (a)
1 1 k
Solution: f H   16kHz
2RC

2 k
Q21. At 1.2 kHz the closed loop gain is
(a) 1 (b) 1.5 (c) 3 (d) 0.5
1  R F 
v0  R1 

Ans: (b)   1 .5
v in 2
1   f 

 fH 

                                                                                
Head office  Branch office 
 
fiziks, H.No. 23, G.F, Jia Sarai,  Anand Institute of Mathematics, 
 
Near IIT, Hauz Khas, New Delhi‐16  28‐B/6, Jia Sarai, Near IIT 
 
Phone: 011‐26865455/+91‐9871145498 Hauz Khas, New Delhi‐16 
                                                   
                                             Website: www.physicsbyfiziks.com                                                                                          
                                                            Email: fiziks.physics@gmail.com                                                                   10 
fiziks
Institute for NET/JRF, GATE, IIT‐JAM, JEST, TIFR and GRE in PHYSICAL SCIENCES 
 
GATE-2014
Q22. The input given to an ideal OP-AMP integrator circuit is
V

V0

t
t0
The correct output of the integrator circuit is
(a) V (b) V

V0 V0

t0 t t0 t

(c) V (d) V

V0
V0
t0 t
t0 t
Ans: (a)
Q23. The minimum number of flip-flops required to construct a mod-75 counter is
__________
Ans: 7
Q24. The donor concentration in a sample of n -type silicon is increased by a factor of 100.
The shift in the position of the Fermi level at 300K, assuming the sample to non
degenerate is ________ meV
k BT  25meV at 300 K 
Ans: 115.15
N   Nc   Nc 
Solution: EC  EF  kT ln  c  and EC  EF  kT ln    kT ln    kT ln 100 
 Nd   100 N d   Nd 
Thus shift is E  kT ln 100   25ln 100  meV  115.15 meV

                                                                                
Head office  Branch office 
 
fiziks, H.No. 23, G.F, Jia Sarai,  Anand Institute of Mathematics, 
 
Near IIT, Hauz Khas, New Delhi‐16  28‐B/6, Jia Sarai, Near IIT 
 
Phone: 011‐26865455/+91‐9871145498 Hauz Khas, New Delhi‐16 
                                                   
                                             Website: www.physicsbyfiziks.com                                                                                          
                                                            Email: fiziks.physics@gmail.com                                                                   11 
fiziks
Institute for NET/JRF, GATE, IIT‐JAM, JEST, TIFR and GRE in PHYSICAL SCIENCES 
 
Q25. The current gain of the transistor in the following circuit is  dc  100 . The value of

collector current I C is_________ mA 12V

3k  20  F
V0
150 k 
Vi
20  F

3k 

Ans: 1.6
VCC  VBE 12  0
Solution: I B    0.016 mA  I C   I B  1.6 mA
RB    RC  RE  150  100  3  3

Q26. In order to measure a maximum of 1V with a resolution of 1mV using a n  bit


A
converter working under the principle of ladder network the minimum value of n is
D
___________
Ans: 10
1
Solution: 1103   n  10
2 1 n

Q27. A low pass fliter is formed by a resistance R and a capacitance C . At the cut-off angular
1
frequency C  the voltage gain and the phase of the output voltage relative to the
RC
input voltage respectively are
(a) 0.71and 45 o (b) 0.71and  45 o (c) 0.5 and  90 o (d) 0.5 and 90 o
Ans: (b)
v0 XC 1 1
Solution:   
 1 1  jCR
vin R  X C R
XC
1 v 1 1 1  j 450
At   C   0   0  e
RC vin 1 j 2e j 45 2

                                                                                
Head office  Branch office 
 
fiziks, H.No. 23, G.F, Jia Sarai,  Anand Institute of Mathematics, 
 
Near IIT, Hauz Khas, New Delhi‐16  28‐B/6, Jia Sarai, Near IIT 
 
Phone: 011‐26865455/+91‐9871145498 Hauz Khas, New Delhi‐16 
                                                   
                                             Website: www.physicsbyfiziks.com                                                                                          
                                                            Email: fiziks.physics@gmail.com                                                                   12 
fiziks
Institute for NET/JRF, GATE, IIT‐JAM, JEST, TIFR and GRE in PHYSICAL SCIENCES 
 
GATE-2015
Q28. The band gap of an intrinsic semiconductor is E g  0.72 eV and mn*  6m*g . At 300 K ,

the Fermi level with respect to the edge of the valence band (in eV ) is at _______(upto
three decimal places) k B  1.38  10 23 JK 1
Ans.: 0.395
Ec  E 3  m* 
Solution: Ei   kT ln  h* 
2 4  mn 
  Ei  Ev  / kT  Eg / 2 kT
ni  NV e  Nc Nv e

  Ei  Ev  / kT N c  Eg / 2 kT N v Eg / 2 kT
 e i v  
E  E / kT
e  e e
Nv Nc
3
Ei  Ev  N v  Eg  mh*  4 Eg 3 E
 ln     ln  *   Ei  Ev  kT ln  6   g
kT  N c  2kT  me  2kT 4 2

3 0.72
 Ei  Ev   0.026 1.7917   0.3949eV  0.395 eV
4 2
Q29. Which one of the following DOES NOT represent an exclusive OR operation for inputs
A and B ?
(a)  A  B AB (b) AB  BA

(c)  A  B A  B  (d)  A  B AB


Ans.: (d)
Solution: (a) ( A  B) AB  ( A  B)( A  B)  AB  AB

(b) AB  AB
(c) AB  AB
(d)  A  B  AB  AB

                                                                                
Head office  Branch office 
 
fiziks, H.No. 23, G.F, Jia Sarai,  Anand Institute of Mathematics, 
 
Near IIT, Hauz Khas, New Delhi‐16  28‐B/6, Jia Sarai, Near IIT 
 
Phone: 011‐26865455/+91‐9871145498 Hauz Khas, New Delhi‐16 
                                                   
                                             Website: www.physicsbyfiziks.com                                                                                          
                                                            Email: fiziks.physics@gmail.com                                                                   13 
fiziks
Institute for NET/JRF, GATE, IIT‐JAM, JEST, TIFR and GRE in PHYSICAL SCIENCES 
 
Q30. Consider the circuit shown in the figure, where RC  1 . For an input signal Vi shown

below, choose the correct V0 from the options:


R
Vi
C
Vi 
V0 1

R

1 2 3 t

V0 V0
(a) (b)
1 1

1 2 3 t 1 2 3 t
1 1

(c) V0 (d) V
i

0 .1
1
1 2 3 t

 0 .1 3 t
1 2

Ans.: (b)
dvi 0  v0 dv dv
Solution: C   v0   RC in   in
dt R dt dt
vin  t  v0  1V and vin  t  v0  1V

                                                                                
Head office  Branch office 
 
fiziks, H.No. 23, G.F, Jia Sarai,  Anand Institute of Mathematics, 
 
Near IIT, Hauz Khas, New Delhi‐16  28‐B/6, Jia Sarai, Near IIT 
 
Phone: 011‐26865455/+91‐9871145498 Hauz Khas, New Delhi‐16 
                                                   
                                             Website: www.physicsbyfiziks.com                                                                                          
                                                            Email: fiziks.physics@gmail.com                                                                   14 
fiziks
Institute for NET/JRF, GATE, IIT‐JAM, JEST, TIFR and GRE in PHYSICAL SCIENCES 
 
Q31. In the simple current source shown in the figure, Q1 and Q2 are identical transistors with
current gain   100 and VBE  0.7 V
Vice  30 V

5 k I0

Q1 Q2

The current I 0 in mA is __________ (upto two decimal places)


Ans.: 5.86
Solution: VCC  I C RC  VBE  0

30  0.7 29.3
IC    5.86 mA
5 5
Q32. In the given circuit, if the open loop gain A  10 5 the feedback configurations and the
closed loop gain A f are Vi 
V0

9 k
1 k RL

(a) series-shunt, A f  9 (b) series-series, A f  10

(c) series-shunt, A f  10 (d) shunt-shunt, A f  10

Ans.: (c)
 R 
Solution: AF  1  F   1  9   10.
 R1 

                                                                                
Head office  Branch office 
 
fiziks, H.No. 23, G.F, Jia Sarai,  Anand Institute of Mathematics, 
 
Near IIT, Hauz Khas, New Delhi‐16  28‐B/6, Jia Sarai, Near IIT 
 
Phone: 011‐26865455/+91‐9871145498 Hauz Khas, New Delhi‐16 
                                                   
                                             Website: www.physicsbyfiziks.com                                                                                          
                                                            Email: fiziks.physics@gmail.com                                                                   15 
fiziks
Institute for NET/JRF, GATE, IIT‐JAM, JEST, TIFR and GRE in PHYSICAL SCIENCES 
 
Q33. In the given circuit, the voltage across the source resistor is 1 V . The drain voltage (in V )
is ___________ 25V

5k 

2 M
500 

Ans.: 15
1 1
Solution: VS  I D RS  I D  A  VD  VDD  I D RD  25   5000  VD  15V
500 500

GATE-2016
Q34. The number density of electrons in the conduction band of a semiconductor at a given
temperature is 2  1019 m 3 . Upon lightly doping this semiconductor with donor
impurities, the number density of conduction electrons at the same temperature
becomes 4  10 20 m 3 . The ratio of majority to minority charge carrier concentration is
________.
Ans : 400
Solution: Intrinsic carrier concentration is ni  2 1019 m 3

Majority carrier concentration is n  4 1020 m 3

ni2  2  10 
19 2

Minority carrier concentration is p    1018 m 3


n 4  10 20

n 4  1020
The ratio of majority to minority charge carrier concentration is   400
p 1018

                                                                                
Head office  Branch office 
 
fiziks, H.No. 23, G.F, Jia Sarai,  Anand Institute of Mathematics, 
 
Near IIT, Hauz Khas, New Delhi‐16  28‐B/6, Jia Sarai, Near IIT 
 
Phone: 011‐26865455/+91‐9871145498 Hauz Khas, New Delhi‐16 
                                                   
                                             Website: www.physicsbyfiziks.com                                                                                          
                                                            Email: fiziks.physics@gmail.com                                                                   16 
fiziks
Institute for NET/JRF, GATE, IIT‐JAM, JEST, TIFR and GRE in PHYSICAL SCIENCES 
 
Q35. For the digital circuit given below, the output X is

X
B
C

(a) A  B.C (b) A . B  C  (c) A . B  C  (d) A  B.C 


Ans.: (b)
Q36. For the transistor shown in the figure, assume V BE  0.7V and  dc  100 . If Vin  5V , Vout
(in Volts) is _________. (Give your answer upto one decimal place)

10V

3 k

Vin Vout

200 k 

1k 

Ans.: 5.7
Vin  VBE 5  0.7 4.3
Solution: I B    mA I C   I B  1.433 mA
RB   RE 200  100 300

Vout  VCC  I C RC  Vout  10  1.433  3  5.7 V

                                                                                
Head office  Branch office 
 
fiziks, H.No. 23, G.F, Jia Sarai,  Anand Institute of Mathematics, 
 
Near IIT, Hauz Khas, New Delhi‐16  28‐B/6, Jia Sarai, Near IIT 
 
Phone: 011‐26865455/+91‐9871145498 Hauz Khas, New Delhi‐16 
                                                   
                                             Website: www.physicsbyfiziks.com                                                                                          
                                                            Email: fiziks.physics@gmail.com                                                                   17 
fiziks
Institute for NET/JRF, GATE, IIT‐JAM, JEST, TIFR and GRE in PHYSICAL SCIENCES 
 
GATE-2017
Q37. The best resolution that a 7 bit A/D convertor with 5V full scale can achieve
is…………… mV . (up to two decimal places)
Ans. : 39.37
5
Solution: Resolution   39.37 mV
2 1
7

Q38. In the figure given below, the input to the primary of the transformer is a voltage varying
sinusoidally with time. The resistor R is connected to the centre tap of the secondary.
Which one of the following plots represents the voltage across the resistor R as a
function of time?
C

V V
(a) 0 (b) 0
t t

(c) V (d) V
0 0
t t

Ans. : (a)
Solution: Full wave rectifier with RC filter.
Q39. The minimum number of NAND gates required to construct an OR gate is:
(a) 2 (b) 4 (c) 5 (d) 3
Ans. : (d)

                                                                                
Head office  Branch office 
 
fiziks, H.No. 23, G.F, Jia Sarai,  Anand Institute of Mathematics, 
 
Near IIT, Hauz Khas, New Delhi‐16  28‐B/6, Jia Sarai, Near IIT 
 
Phone: 011‐26865455/+91‐9871145498 Hauz Khas, New Delhi‐16 
                                                   
                                             Website: www.physicsbyfiziks.com                                                                                          
                                                            Email: fiziks.physics@gmail.com                                                                   18 
fiziks
Institute for NET/JRF, GATE, IIT‐JAM, JEST, TIFR and GRE in PHYSICAL SCIENCES 
 
Q40. For the transistor amplifier circuit shown below with R1  10 k , R2  10 k , R3  1 k  ,

and   99 . Neglecting the emitter diode resistance, the input impedance of the amplifier
looking into the base for small ac signal is…………. k . (up to two decimal places)
VCC

R1
C
Vin B

E Vout
R2 R3

Ans. : 4.75
Solution: Z i  Z b  R where Z b   R3  99k  and R  R1  R2  5k 

 Z i  Z b  R  4.75k 
Q41. Consider an ideal operational amplifier as shown in the figure below with
R1  5 k , R2  1 k , RL  100 k  . For an applied input voltage V  10 mV , the current

passing through R2 is……………..  A . (up to two decimal places)




R1
V
RL
R2
Ans. : 10.0
V 10
Solution: I 2    10 A
R2 1

                                                                                
Head office  Branch office 
 
fiziks, H.No. 23, G.F, Jia Sarai,  Anand Institute of Mathematics, 
 
Near IIT, Hauz Khas, New Delhi‐16  28‐B/6, Jia Sarai, Near IIT 
 
Phone: 011‐26865455/+91‐9871145498 Hauz Khas, New Delhi‐16 
                                                   
                                             Website: www.physicsbyfiziks.com                                                                                          
                                                            Email: fiziks.physics@gmail.com                                                                   19 
fiziks
Institute for NET/JRF, GATE, IIT‐JAM, JEST, TIFR and GRE in PHYSICAL SCIENCES 
 
ATOMIC AND MOLECULAR PHYSICS SOLUTIONS

GATE-2010

Q1. To detect trace amounts of gaseous species in a mixture of gases, the preferred probing
tool is
(a) Ionization spectroscopy with X-rays (b) NMR spectroscopy
(c) ESR spectroscopy (d) Laser spectroscopy
Ans: (a)
Q2. A collection of N atoms is exposed to a strong resonant electromagnetic radiation with Ng
atoms in the ground state and Ne atoms in the excited state, such that
Ng+Ne=N. This collection of two-level atoms will have the following population
distribution:
(a) Ng << Ne (b) Ng >> Ne (c) Ng ≈ Ne ≈ N/2 (d) Ng – Ne ≈ N/2
Ans: (c)
Solution: In two level lair population inversion is possible to achieve at any power level. The
N
maximum possible situation can be N g  N e 
2
Q3. Two states of an atom have definite parities. An electric dipole transition between these
states is
(a) Allowed if both the sates have even parity
(b) Allowed if both the states have odd parity
(c) Allowed if the two states have opposite parities
(d) Not allowed unless a static electric field is applied
Ans: (c)
Q4. The spectrum of radiation emitted by a black body at a temperature 1000 K peaks in the
(a) Visible range of frequencies (b) Infrared range of frequencies
(c) Ultraviolet range of frequencies (d) Microwave range of frequencies
Ans: (a)

                                                                                
Head office  Branch office 
 
fiziks, H.No. 23, G.F, Jia Sarai,  Anand Institute of Mathematics, 
 
Near IIT, Hauz Khas, New Delhi‐16  28‐B/6, Jia Sarai, Near IIT 
 
Phone: 011‐26865455/+91‐9871145498 Hauz Khas, New Delhi‐16 
                                                   
                                             Website: www.physicsbyfiziks.com                                                                                          
                                                            Email: fiziks.physics@gmail.com                                                                   1 
fiziks
Institute for NET/JRF, GATE, IIT‐JAM, JEST, TIFR and GRE in PHYSICAL SCIENCES 
 
Q5. The three principal moments of inertia of a methanol (CH3OH) molecule have the
property Ix = Iy = I and Iz ≠ I. The rotation energy eigenvalues are
2  2 m12  1 1  2
(a) l l  1     (b) l l  1
2I 2 I  I z I  2I

 2 m12  1 1  2  2 m12  1 1 
(c)    (d) l l  1    
2 I  I z I  2I 2  I z I 
Ans: (a)
Solution: CH 3 OH is example of symmetric rotar where I x  I y  I z ( I x  I y  I and I z  I )

The classical expression for energy is E 


1 2
2I

J x  J y2 
1 2
2I z

Jz

This can be expressed in term of J 2  J x2  J y2  J z2 by adding and subtracting J z2

1 2  1 1 
E J     J z2 .
2I  2I z 2I 
2  2 m J2  1 1 
Quantum mechanically E  J J  1    
2I 2  I z I 
Q6. Match the typical spectra of stable molecules with the corresponding wave-number range
1. Electronic spectra (i) 106 cm-1 and above
2. Rotational spectra (ii) 105 – 106 cm-1
3. Molecule dissociation (iii) 108 – 102 cm-1
(a) 1 – ii, 2 – i, 3 – iii (b) 1 – ii, 2 – iii, 3 – i
(b) 1 – iii, 2 – ii, 3 – i (d) 1 – i, 2 – ii, 3 – iii
Ans: (b)

Q7. Consider the operations P : r   r (parity) and T: t → - t (time reversal). For the electric

and magnetic fields E and B , which of the following set of transformations is correct?

(a) P : E   E , B  B; (b) P : E  E , B  B;

T : E  E, B   B T : E  E, B  B

(c) P : E   E , B  B; (d) P : E  E , B   B;

                                                                                
Head office  Branch office 
 
fiziks, H.No. 23, G.F, Jia Sarai,  Anand Institute of Mathematics, 
 
Near IIT, Hauz Khas, New Delhi‐16  28‐B/6, Jia Sarai, Near IIT 
 
Phone: 011‐26865455/+91‐9871145498 Hauz Khas, New Delhi‐16 
                                                   
                                             Website: www.physicsbyfiziks.com                                                                                          
                                                            Email: fiziks.physics@gmail.com                                                                   2 
fiziks
Institute for NET/JRF, GATE, IIT‐JAM, JEST, TIFR and GRE in PHYSICAL SCIENCES 
 
T : E   E, B   B T : E  E, B  B

Ans: (b)
Common Data Questions 8 and 9:
In the presence of a weak magnetic field, atomic hydrogen undergoes the transition:
2
P1 / 2 1S1 / 2 , by emission of radiation

Q8. The number of distinct spectral lines that are observed in the resultant Zeeman spectrum
is
(a) 2 (b) 3 (c) 4 (d) 6
Ans: (c)
Solution: 2 p1 / 2  2 S 1 / 2 is sodium D1 lines and it has total 4 zeeman components.
Q9. The spectral line corresponding to the transition
 1  1
2
P1  m j   1S1 / 2  m j   
2 2  2
is observed along the direction of the applied magnetic field. The emitted electromagnetic
field is
(a) Circularly polarized (b) Linearly polarized
(c) Unpolarized (d) Not emitted along the magnetic field direction
Ans: (a)
 1  1
Solution: For 2
P1  m j   1S1 / 2  m j   
2 2  2

Here m j  1 gives   component. In longitudinal observation   is circularly

polarized.
GATE-2011

Q10. The population inversion in a two layer material CANNOT be achieved by optical
pumping because
(a) the rate of upward transitions is equal to the rate of downward transitions
(b) the upward transitions are forbidden but downward transitions are allowed
(c) the upward transitions are allowed but downward transitions are forbidden
                                                                                
Head office  Branch office 
 
fiziks, H.No. 23, G.F, Jia Sarai,  Anand Institute of Mathematics, 
 
Near IIT, Hauz Khas, New Delhi‐16  28‐B/6, Jia Sarai, Near IIT 
 
Phone: 011‐26865455/+91‐9871145498 Hauz Khas, New Delhi‐16 
                                                   
                                             Website: www.physicsbyfiziks.com                                                                                          
                                                            Email: fiziks.physics@gmail.com                                                                   3 
fiziks
Institute for NET/JRF, GATE, IIT‐JAM, JEST, TIFR and GRE in PHYSICAL SCIENCES 
 
(d) the spontaneous decay rate of the higher level is very low
Ans: (a)
Q11. A heavy symmetrical top is rotating about its own axis of symmetry (the z-axis). If I1, I2
and I3 are the principal moments of inertia along x, y and z axes respectively, then
(a) I2 = I3; I1 ≠ I2 (b) I1 = I3; I1 ≠ I2 (c) I1 = I2; I1 ≠ I3 (d) I1 ≠ I2 ≠ I3
Ans: (c)
Q12. A neutron passing through a detector is detected because of
(a) the ionization it produces (b) the scintillation light it produces
(c) the electron-hole pairs it produces
(d) the secondary particles produced in a nuclear reaction in the detector medium
Ans: (b)
Q13. An atom with one outer electron having orbital angular momentum l is placed in a weak
magnetic field. The number of energy levels into which the higher total angular
momentum state splits, is
(a) 2l + 2 (b) 2l + 1 (c) 2l (d) 2l – 1
Ans: (b)
Q14. For a multi-electron atom l, L and S specify the one-electron orbital angular momentum,
total orbital angular momentum and total spin angular momentum, respectively. The
selection rules for electric dipole transition between the two electronic energy levels,
specified by l, L and S are
(a) ∆L = 0, ±1; ∆S = 0; ∆l = 0, ±1 (b) ∆L = 0, ±1; ∆S = 0; ∆l = ±1
(c) ∆L = 0, ±1; ∆S = ±1; ∆l = 0, ±1 (d) ∆L = 0, ±1; ∆S = ±1; ∆l = ±1
Ans: (b)
Q15. The lifetime of an atomic state is 1 nanosecond. The natural line width of the spectral line
in the emission spectrum of this state is of the order of
(a) 10-10 eV (b) 10-9 eV (c) 10-6 eV (d) 10-4 eV
Ans: (c)
h 6.625  10 34 J  S 6.625  10 25
Solution: E  h   9
 19
eV  4.14  10 6 eV
t 10 1.6  10

                                                                                
Head office  Branch office 
 
fiziks, H.No. 23, G.F, Jia Sarai,  Anand Institute of Mathematics, 
 
Near IIT, Hauz Khas, New Delhi‐16  28‐B/6, Jia Sarai, Near IIT 
 
Phone: 011‐26865455/+91‐9871145498 Hauz Khas, New Delhi‐16 
                                                   
                                             Website: www.physicsbyfiziks.com                                                                                          
                                                            Email: fiziks.physics@gmail.com                                                                   4 
fiziks
Institute for NET/JRF, GATE, IIT‐JAM, JEST, TIFR and GRE in PHYSICAL SCIENCES 
 
Q16. The degeneracy of an excited state of nitrogen atom having electronic configuration
1s22s22p23d1 is
(a) 6 (b) 10 (c) 15 (d) 150
Ans: (b)
Solution: Degeneracy = 2 (2l + 1)
Q17. The far infrared rotational absorption spectrum of a diatomic molecule shows equilibrium
lines with spacing 20 cm-1. The position of the first Stokes line in the rotational Raman
spectrum of this molecule is
(a) 20 cm-1 (b) 40 cm-1 (c) 60 cm-1 (d) 120 cm-1
Ans: (c)
Solution: Given 2B = 20 cm-1  B = 10 cm-1
The position of the first stokes line in the rotational Raman spectrum = 6B
   6 B  6  10  60 cm 1 .
GATE-2012
Q18. The ground state of sodium atom ( 11 Na ) is a 2 S1 / 2 state. The difference in energy levels
arising in the presence of a weak external magnetic field B, given in terms of Bohr
magnet on,  B , is
(a)  B B (b) 2  B B (c) 4  B B (d) 6  B B
Ans: (b)
Solution: The energy separation in the Zeeman level is E  gM J  B B 

1
For 2 S1 / 2 state; g  2 and M J   . Therefore E1   B B  and E 2   B B  .
2
Thus E  2  B B
Q19. The first Stokes line of a rotational Raman spectrum is observed at 12.96 cm-1.
Considering the rigid rotor approximation, the rotational constant is given by
(a) 6.48 cm-1 (b) 3.24 cm-1 (c) 2.16 cm-1 (d) 1.62 cm-1
Ans: (c)
Solution: The first Stoke line of the Rotational Raman spectrum lies at = 6B

                                                                                
Head office  Branch office 
 
fiziks, H.No. 23, G.F, Jia Sarai,  Anand Institute of Mathematics, 
 
Near IIT, Hauz Khas, New Delhi‐16  28‐B/6, Jia Sarai, Near IIT 
 
Phone: 011‐26865455/+91‐9871145498 Hauz Khas, New Delhi‐16 
                                                   
                                             Website: www.physicsbyfiziks.com                                                                                          
                                                            Email: fiziks.physics@gmail.com                                                                   5 
fiziks
Institute for NET/JRF, GATE, IIT‐JAM, JEST, TIFR and GRE in PHYSICAL SCIENCES 
 
1 1
Thus 6 B  12.96 cm  B  1.16 cm .
Q20. Match the typical spectroscopic regions specified in Group I with the corresponding type
of transitions in Group II.
Group I Group II
(P) Infra-red region (i) electronic transitions involving valence electrons
(Q) Ultraviolet-visible region (ii) nuclear transitions
(R) X-ray region (iii) vibrational transitions of molecules
(S) γ-ray region (iv) transitions involving inner shell electrons
(a) (P, i); (Q, iii); (R, ii); (S, iv) (b) (P, ii); (Q, iv); (R, i); (S, iii)
(c) (P, iii); (Q, i); (R, iv); (S, ii) (d) (P, iv); (Q, i); (R, ii); (S, iii)
Ans: (c)
Q21. The term  j1 , j 2 J arising from 2s1 3d 1 electronic in j-j coupling scheme are

1 3  1 5  1 1  1 3
(a)  ,  and  ,  (b)  ,  and  , 
 2 2  2,1  2 2  3, 2  2 2 1, 0  2 2  2,1

1 1  1 5  3 1 1 5 
(c)  ,  and  ,  (d)  ,  and  , 
 2 2 1,0  2 2  3, 2  2 2  2,1  2 2  3, 2
Ans: (c)
Q22. The equilibrium vibration frequency for an oscillator is observed at 2990 cm-1. The ratio
of the frequencies corresponding to the first and the fundamental spectral lines is 1.96.
Considering the oscillator to be anharmonic, the anharmonicity constant is
(a) 0.005 (b) 0.02 (c) 0.05 (d) 0.1
Ans: (b)
2 e 1  3 xe  1  3xe 
Solution:  e 1  2 xe   2990cm 1 and  1.96cm 1   0.98cm 1
 e 1  2 xe  1  2 xe 
 xe  0.02 .

                                                                                
Head office  Branch office 
 
fiziks, H.No. 23, G.F, Jia Sarai,  Anand Institute of Mathematics, 
 
Near IIT, Hauz Khas, New Delhi‐16  28‐B/6, Jia Sarai, Near IIT 
 
Phone: 011‐26865455/+91‐9871145498 Hauz Khas, New Delhi‐16 
                                                   
                                             Website: www.physicsbyfiziks.com                                                                                          
                                                            Email: fiziks.physics@gmail.com                                                                   6 
fiziks
Institute for NET/JRF, GATE, IIT‐JAM, JEST, TIFR and GRE in PHYSICAL SCIENCES 
 
GATE-2013
Q23. The number of spectral lines allowed in the spectrum for the 3 2 D  3 2 P transition in
sodium is _____________.
Ans: 28
Solution: The numbers of Zeeman components for 2D5/2 → 2P3/2 transition = 12
The numbers of Zeeman components for 2D3/2 → 2P3/2 transition = 10
The numbers of Zeeman components for 2D3/2 → 2P1/2 transition = 6
Q24. In a normal Zeeman Effect experiment, spectral splitting of the line at the wavelength
643.8 nm corresponding to the transition 5 1 D2  5 1 P1 of cadmium atoms is to be
observed. The spectrometer has a resolution of 0.01 nm. Minimum magnetic field needed

to observe this is me  9.1  10 31 kg , e  1.6  19 C , c  3  10 8 m / s 
(a) 0.26T (b) 0.52T (c) 2.6T (d) 5.2T
Ans: (b)
Solution: Separation of Zeeman Components
eB 2 2 eB
      
4m c c 4m
4mc  4  3.14  9.1  10 31  3  10 8 0.01  10 9
B   0.514T
e 2 1.6  10 19 643.8  10 9
2
 
Q25. The spacing between vibrational energy levels in CO molecule is found to
be 8.44  10 2 eV . Given that the reduced mass of CO is 1.14  10 26 kg , Planck’s constant

is 6.626  10 34 Js and 1 eV  1.6  10 19 J . The force constant of the bond in CO
molecule is
(a) 1.87 N/m (b) 18.7 N/m (c) 187 N/m (d) 1870 N/m
Ans: (c)
Solution: The energy of the quantum harmonic oscillator is
 1
E  h  n  , n  0,1,2,........
 2
1 k
The frequency of oscillation is   .
2 
                                                                                
Head office  Branch office 
 
fiziks, H.No. 23, G.F, Jia Sarai,  Anand Institute of Mathematics, 
 
Near IIT, Hauz Khas, New Delhi‐16  28‐B/6, Jia Sarai, Near IIT 
 
Phone: 011‐26865455/+91‐9871145498 Hauz Khas, New Delhi‐16 
                                                   
                                             Website: www.physicsbyfiziks.com                                                                                          
                                                            Email: fiziks.physics@gmail.com                                                                   7 
fiziks
Institute for NET/JRF, GATE, IIT‐JAM, JEST, TIFR and GRE in PHYSICAL SCIENCES 
 
Where k = Spring constant and  = reduced mass
h k
The energy levels are equally spaced with energy separation of E  h 
2 
2 2
 2   2  3.14 
k  E     34
 8.44  10  2  1.6  10 19   1.14  10  26  186.7 N / m
 h   6.626  10 
GATE-2014
Q26. The number of normal Zeeman splitting components of 1 P  1D transition is
(a) 3 (b) 4 (c) 8 (d) 9
Ans: (a)
Solution: This is singlet transition.
Q27. The moment of inertia of a rigid diatomic molecule A is 6 times that of another rigid
diatomic molecule B . If the rotational energies of the two molecules are equal, then the
corresponding values of the rotational quantum numbers J A and J B are
(a) J A  2, J B  1 (b) J A  3, J B  1
(c) J A  5, J B  0 (d) J A  6, J B  1
Ans: (b)
J A  J A  1 I B I
Solution:   B
J B  J B  1 I A 6I B

J A  6, J B  1
Q28. The value of the magnetic field required to maintain non-relativistic protons of energy
1MeV in a circular orbit of radius 100mm is_______Tesla
(Given: m p  1.67  10 27 kg . e  1.6  10 19 C )

Ans: 1.44

mv 2 1 2mE
Solution:  qvB, E  mv 2  B   1.44
r 2 qr

                                                                                
Head office  Branch office 
 
fiziks, H.No. 23, G.F, Jia Sarai,  Anand Institute of Mathematics, 
 
Near IIT, Hauz Khas, New Delhi‐16  28‐B/6, Jia Sarai, Near IIT 
 
Phone: 011‐26865455/+91‐9871145498 Hauz Khas, New Delhi‐16 
                                                   
                                             Website: www.physicsbyfiziks.com                                                                                          
                                                            Email: fiziks.physics@gmail.com                                                                   8 
fiziks
Institute for NET/JRF, GATE, IIT‐JAM, JEST, TIFR and GRE in PHYSICAL SCIENCES 
 
3
Q29. Neutrons moving with speed 10 m / s are used for the determination of crystal structure.

If the Bragg angle for the first order diffraction is 30 o the interplannar spacing of the
0
crystal is ______ A
(Given: mn  1.675  1027 kg h  6.626 1034 J .s )
Ans: 4
h 0
Solution: 2d sin     d  4 A
mv
Q30. The emission wavelength for the transition D2  F3 is 3122 Ǻ. The ratio of population

of the final to the initial states at a temperature 5000 K is


h  6.626  10 34
J .s, c  3  10 8 m / s k B  1.380  10 23 J / K 

(a) 2.03  10 5 (b) 4.02  10 5 (c) 7.02  10 5 (d) 9.83  10 5


Ans: (c)

N F  2 J f  1   kBT 5 9.227641144
hc

Solution:  e  e  7.02 105


NI  2 J i  1 7

GATE-2015
Q31. In a rigid rotator of mass M , if the energy of the first excited state is ( 1 meV ), then the
fourth excited state energy (in meV ) is ____________.
Ans.: 6
Solution:  E  J  J  1 where J  0,1, 2,3..

E4 3  3  1
   E4  6 E1  6 meV where J  0,1, 2,3..
E1 11  1
Q32. The binding energy per molecule of NaCl (lattice parameter is 0.563 nm ) is 7.956 eV .
K
The repulsive term of the potential is of the form , where K is a constant. The value
r9
of the Madelung constant is ___________ (upto three decimal places)
(Electron charge e  1.6  10 19 C ;  0  8.854  10 12 C 2 N 1 m 2 )
Ans.: 1.68
                                                                                
Head office  Branch office 
 
fiziks, H.No. 23, G.F, Jia Sarai,  Anand Institute of Mathematics, 
 
Near IIT, Hauz Khas, New Delhi‐16  28‐B/6, Jia Sarai, Near IIT 
 
Phone: 011‐26865455/+91‐9871145498 Hauz Khas, New Delhi‐16 
                                                   
                                             Website: www.physicsbyfiziks.com                                                                                          
                                                            Email: fiziks.physics@gmail.com                                                                   9 
fiziks
Institute for NET/JRF, GATE, IIT‐JAM, JEST, TIFR and GRE in PHYSICAL SCIENCES 
 
Solution: The total energy of one ion due to the presence of all others in NaCl crystal is
(considering univalent ions)
Ae2 K
U r     n where A is Modelung Constant.
4 0 r r

The potential energy will be minimum at the equilibrium spacing r0 .

 dU   Ae 2 Kn  Ae2 r0n 1
Thus     n 1 
 0  K 
 dr  r  r0  4 0 r0 r0  4 0 n
2

Thus, Binding energy of molecule or lattice energy is

Ae 2 Ae2 r0n 1  Ae 2   n  1 
U 0  U r  r      
0
4 0 r0 4 0 nr0n  4 0 r0   n 
K
Given repulsive term of the potential is  n  9 
r9
Also binding energy per molecule is U 0  7.95 eV

4 0 r0 n
The Modelung constant is A  U 0  
e 2
n 1
Given, the lattice parameter is a  0.563 nm , thus, the interatomic separation is
a
r0   0.82 nm
2

 A  7.95 1.67  0 19


J

4  3.14  8.85  1012 C 2 N 1m 2  0.282 109   9
1.67 10 
2
19
J 8

7.95  4  3.14  8.85  0.282  9


 A 102  A  1.68
1.67  8
Q33. Match the phrases in Group I and Group II and identify the correct option.
Group I Group II
(P) Electron spin resonance (ESR) (i) radio frequency
(Q) Nuclear magnetic resonance (NMR) (ii) visible range frequency
(R) Transition between vibrational states of a molecule (iii) microwave frequency
(S) Electronic transition (iv) far-infrared range
                                                                                
Head office  Branch office 
 
fiziks, H.No. 23, G.F, Jia Sarai,  Anand Institute of Mathematics, 
 
Near IIT, Hauz Khas, New Delhi‐16  28‐B/6, Jia Sarai, Near IIT 
 
Phone: 011‐26865455/+91‐9871145498 Hauz Khas, New Delhi‐16 
                                                   
                                             Website: www.physicsbyfiziks.com                                                                                          
                                                            Email: fiziks.physics@gmail.com                                                                   10 
fiziks
Institute for NET/JRF, GATE, IIT‐JAM, JEST, TIFR and GRE in PHYSICAL SCIENCES 
 
(a) (P-i), (Q-ii), (R-iii), (S-iv) (b) (P-ii), (Q-i), (R-iv), (S-iii)
(c) (P-iii), (Q-iv), (R-i), (S-ii) (d) (P-iii), (Q-i), (R-iv), (S-ii)
Ans.: (d)
Solution: (P) Electron spin resonance (ESR) is achieved by Microwave frequency (iii)
(Q): Nuclear magnetic resonance (NMR) is achieved by Radio frequency (i)
(R): Transition between vibrational states of a molecule is achieved by radiation of far
infrared range (iv)
(S): Electronic transition is achieved by visible radiation (ii)
Q34. The excitation wavelength of laser in a Raman effect experiment is 546 nm . If the
Stokes’ line is observed at 552 nm , then the wavenumber of the anti-Stokes’ line (in

cm 1 ) is ___________
Ans.: 18514
Solution: Raman displacement is
1 1 1 1
   AS  0   0  S or     
AS 0 0 S
where  AS , 0 ,  S are wavelength of anti-stoke, exciting & stoke line.
From above relation we can write
1 1 1 1 1 2 1 1 2 S   0 0  S
          AS 
 AS 0 0 S  AS 0 S  AS 0  S 2 S   0

 AS 
 546 10 m  552 10 m 
9 9


546  552
109 m
 2  552 10 9
m  546 10 m  9
558

 AS  540.129 109 m  540.129 107 cm

1 1
Anti-stoke wavenumber is  AS    18514 cm 1
AS 7
540.129  10 cm

                                                                                
Head office  Branch office 
 
fiziks, H.No. 23, G.F, Jia Sarai,  Anand Institute of Mathematics, 
 
Near IIT, Hauz Khas, New Delhi‐16  28‐B/6, Jia Sarai, Near IIT 
 
Phone: 011‐26865455/+91‐9871145498 Hauz Khas, New Delhi‐16 
                                                   
                                             Website: www.physicsbyfiziks.com                                                                                          
                                                            Email: fiziks.physics@gmail.com                                                                   11 
fiziks
Institute for NET/JRF, GATE, IIT‐JAM, JEST, TIFR and GRE in PHYSICAL SCIENCES 
 
Q35. The number of permitted transitions from 2 P3 / 2  2 S1 / 2 in the presence of a weak
magnetic field is ________________
Ans. : 6
Solution: Zeeman splitting of 2 P3 / 2 and 2 S1/ 2 is shown below
MJ
3/2
2
P3/2 1/2

1/2
3/2
2
S1/ 2 1/2

1/2

The selection rule for Zeeman transactions are


M J  0,  1 0  0 if J  0
There are total six transition in accordance with above selection rules.

GATE-2016
17
Q36. The molecule O2
(a) Raman active but not NMR (nuclear magnetic resonance) active.
(b) Infrared active and Raman active but not NMR active.
(c) Raman active and NMR active.
(d) Only NMR active.
Ans.: (c)
17
Solution: (i) Molecule O2 can not absorb infrared as there is no change in dipole moment
17
during vibration. Thus O2 is infrared inactive.
17
(ii) Molecule O2 shows change in polaraziability during rotation. Thus it is Raman

active molecule.
17 5
(iii) The nucleus of O has spin , therefore it is NMR active.
2

                                                                                
Head office  Branch office 
 
fiziks, H.No. 23, G.F, Jia Sarai,  Anand Institute of Mathematics, 
 
Near IIT, Hauz Khas, New Delhi‐16  28‐B/6, Jia Sarai, Near IIT 
 
Phone: 011‐26865455/+91‐9871145498 Hauz Khas, New Delhi‐16 
                                                   
                                             Website: www.physicsbyfiziks.com                                                                                          
                                                            Email: fiziks.physics@gmail.com                                                                   12 
fiziks
Institute for NET/JRF, GATE, IIT‐JAM, JEST, TIFR and GRE in PHYSICAL SCIENCES 
 
Q37. There are four electrons in the 3d shell of an isolated atom. The total magnetic moment
of the atom in units of Bohr magneton is ________.
Ans.: 0
Solution: The configuration leads to S  2 and L  2
Since it is the case of less than half filled sub shell, thus according to Hund’s rules, lower
J will be in ground state. M L  2 1 0 1 2

 J  L  S  0 . Thus   0 3d 4 
Q38. Which of the following transitions is NOT allowed in the case of an atom, according to
the electric dipole radiation selection rule?
(a) 2 s  1s (b) 2 p  1s (c) 2 p  2s (d) 3d  2 p
Ans.: (a)
Solution: In electron dipole transition, l  1 . Thus in transition 2s  1s, l  0 . It violate the
selection rule and hence not allowed.
Q39. The number of spectroscopic terms resulting from the L.S coupling of a 3 p electron and
a 3d electron is _______.
Ans.: 12
1 1
Solution: For 3 p1 3d 1 : s1  , s2   S  0,1
2 2
l1  1, l2  2  L  1, 2,3

S  0, L  1  J  1  Term  1P1

S  0, L  2  J  2  Term  1D2

S  0, L  3  J  3  Term  1F3

S  1, L  1  J  0,1, 2  Terms  3P0 , 3P1 , 3 P2

S  1, L  2  J  1, 2,3  Terms  3D1 , 3D2 , 3 D3

S  1, L  3  J  2,3, 4  Terms  3F2 , 3F3 , 3 F4

Thus total number of spectroscopic terms are 12.

                                                                                
Head office  Branch office 
 
fiziks, H.No. 23, G.F, Jia Sarai,  Anand Institute of Mathematics, 
 
Near IIT, Hauz Khas, New Delhi‐16  28‐B/6, Jia Sarai, Near IIT 
 
Phone: 011‐26865455/+91‐9871145498 Hauz Khas, New Delhi‐16 
                                                   
                                             Website: www.physicsbyfiziks.com                                                                                          
                                                            Email: fiziks.physics@gmail.com                                                                   13 
fiziks
Institute for NET/JRF, GATE, IIT‐JAM, JEST, TIFR and GRE in PHYSICAL SCIENCES 
 
GATE-2017
Q40. The wavefunction of which orbital is spherically symmetric:
(a) px (b) p y (c) s (d) d xy

Ans. : (c)
Solution: For s orbital l  0
0.5 1
Q41. The total energy of an inert-gas crystal is given by E  R    (in eV ), where R
R12 R 6
is the inter-atomic spacing in Angstroms. The equilibrium separation between the atoms
is Angstroms. (up to two decimal places)
Ans. : 1
0.5 1
Solution: Given that E  R   
R12 R 6
For equilibrium separation
dE
0
dR
dE 12  0.5 6
   7 0
dR R13 R
1  6 
  R 6  6   0  R  1
R6
Q42. Which one of the following gases of diatomic molecules is Raman, infrared, and NMR
active?
(a) 1H - 1H (b) 12C - 16O (c) 1H - 35Cl (d) 16O- 16O
Ans. : (c)
Solution: (a) 1H  1H Infrared inactive
(b) 12C  16O NMR Inactive
(c) 1H  35Cl Raman, infrared & NMR active
(d) 16
O  16O Infrared , Raman inactive

                                                                                
Head office  Branch office 
 
fiziks, H.No. 23, G.F, Jia Sarai,  Anand Institute of Mathematics, 
 
Near IIT, Hauz Khas, New Delhi‐16  28‐B/6, Jia Sarai, Near IIT 
 
Phone: 011‐26865455/+91‐9871145498 Hauz Khas, New Delhi‐16 
                                                   
                                             Website: www.physicsbyfiziks.com                                                                                          
                                                            Email: fiziks.physics@gmail.com                                                                   14 
fiziks
Institute for NET/JRF, GATE, IIT‐JAM, JEST, TIFR and GRE in PHYSICAL SCIENCES 
 
Q43. Using Hund’s rule the total angular momentum quantum number J for the electronic
ground state of the nitrogen atom is
1 3
(a) (b) (c) 0 (d) 1
2 2
Ans. : (b)
Solution: N : 7 :1s 2 2 s 2 2 p 3

For p 3 : ML  1 0 1
  

 spectral term  251 LJ  4s3/ 2


Q44. Positronium is an atom made of an electron and a positron. Given the Bohr radius for the
ground state of the Hydrogen atom to be 0.53 Angstroms, the Bohr radius for the ground
state of positronium is…………Angstroms. (up to two decimal places).
Ans. : 1.06
m 
Solution: rn  a0  e 
  
me me m2 m
When    e  e
me  me 2me 2

 rn  2a0  2  0.53  1.06 A0

                                                                                
Head office  Branch office 
 
fiziks, H.No. 23, G.F, Jia Sarai,  Anand Institute of Mathematics, 
 
Near IIT, Hauz Khas, New Delhi‐16  28‐B/6, Jia Sarai, Near IIT 
 
Phone: 011‐26865455/+91‐9871145498 Hauz Khas, New Delhi‐16 
                                                   
                                             Website: www.physicsbyfiziks.com                                                                                          
                                                            Email: fiziks.physics@gmail.com                                                                   15 
fiziks
Institute for NET/JRF, GATE, IIT‐JAM, JEST, TIFR and GRE in PHYSICAL SCIENCES 
 
SOLID STATE PHYSICS SOLUTIONS

GATE-2010

Q1. The valence electrons do not directly determine the following property of a metal
(a) Electrical conductivity (b) Thermal conductivity
(c) Shear modulus (d) Metallic luster
Ans: (c)
Q2. Consider X-ray diffraction from a crystal with a face-centered-cubic (fcc) lattice. The
lattice plane for which there is NO diffraction peak is
(a) (2, 1, 2) (b) (1, 1, 1) (c) (2, 0, 0) (d) (3, 1, 1)
Ans: (a)
Q3. The Hall coefficient, RH, of sodium depends on
(a) The effective charge carrier mass and carrier density
(b) The charge carrier density and relaxation time
(c) The charge carrier density only
(d) The effective charge carrier mass
Ans: (c)

Q4. The Bloch theorem states that within a crystal, the wavefunction, ψ( r ), of an electron has
the form

 
(a)  r  u r e i k .r where u( r ) is an arbitrary function and k is an arbitrary vector

(b) r   u r e i G r
where u( r ) is an arbitrary function and G is a reciprocal lattice vector

(c)  r   u r e i G r
  
where u r  u r   ,  is a lattice vector and G is a reciprocal
lattice vector

    
(d)  r  u r e i k .r where u r  u r   ,  is a lattice vector and k is an arbitrary
vector
Ans: (d)

                                                                                
Head office  Branch office 
 
fiziks, H.No. 23, G.F, Jia Sarai,  Anand Institute of Mathematics, 
 
Near IIT, Hauz Khas, New Delhi‐16  28‐B/6, Jia Sarai, Near IIT 
 
Phone: 011‐26865455/+91‐9871145498 Hauz Khas, New Delhi‐16 
                                                   
                                             Website: www.physicsbyfiziks.com                                                                                          
                                                            Email: fiziks.physics@gmail.com                                                                   1 
fiziks
Institute for NET/JRF, GATE, IIT‐JAM, JEST, TIFR and GRE in PHYSICAL SCIENCES 
 
Q5. In an experiment involving a ferromagnetic medium, the following observations were
made. Which one of the plots does NOT correctly represent the property of the medium?
(TC is the Curie temperature)
(a) (b)

1 / TC 1/ T

(c) (d)

TC T TC T
Ans: (c)
Q6. The thermal conductivity of a given material reduces when it undergoes a transition from
its normal state to the superconducting state. The reason is:
(a) The Cooper pairs cannot transfer energy to the lattice
(b) Upon the formation of Cooper pairs, the lattice becomes less efficient in heat transfer
(c) The electrons in the normal state lose their ability to transfer heat because of their
coupling to the Cooper pairs
(d) The heat capacity increases on transition to the superconducting state leading to a
reduction in thermal conductivity
Ans: (d)

                                                                                
Head office  Branch office 
 
fiziks, H.No. 23, G.F, Jia Sarai,  Anand Institute of Mathematics, 
 
Near IIT, Hauz Khas, New Delhi‐16  28‐B/6, Jia Sarai, Near IIT 
 
Phone: 011‐26865455/+91‐9871145498 Hauz Khas, New Delhi‐16 
                                                   
                                             Website: www.physicsbyfiziks.com                                                                                          
                                                            Email: fiziks.physics@gmail.com                                                                   2 
fiziks
Institute for NET/JRF, GATE, IIT‐JAM, JEST, TIFR and GRE in PHYSICAL SCIENCES 
 
Q7. For a two-dimensional free electron gas, the electronic density n, and the Fermi energy
EF, are related by

2mE F  2 mE F  3
3 1 1
mE F mE F 2 3
(a) n  (b) n  (c) n  (d) n 
3 2  3  2 2 2 
Ans: (b)
Solution: For two dimensional gas, the number of possible k-states between k and k+dk is

2 2
 L   L 
g k dk    2 k dk  2  2 k dk it is multiplied by 2 for electron gas
 2   2 

2mE 2m 2m
Since k 2  2
 2k dk  2 dE  2 k dk  2 dE
  
2
 L  2m
 g E dE  2   2 dE
 2  
The total number of electrons at T  0 0 K is
EF EF 2 EF
2m  1  2m L2
N  g E dE  F E  
0
 g E dE  2 
0
 
 2  2   dE  2 
0

 2 4 2
 EF

m  2  N  
2
mE F
N  L2
E  E F     n  n 
 2
F
 2 m  L2  m
Q8. Far away from any of the resonance frequencies of a medium, the real part of the
dielectric permittivity is
(a) Always independent of frequency (b) Monotonically decreasing with frequency
(c) Monotonically increasing with frequency (d) A non-monotonic function of frequency
Ans: (a)
dipolar

 1 ionic
 2
electronics

frequency

                                                                                
Head office  Branch office 
 
fiziks, H.No. 23, G.F, Jia Sarai,  Anand Institute of Mathematics, 
 
Near IIT, Hauz Khas, New Delhi‐16  28‐B/6, Jia Sarai, Near IIT 
 
Phone: 011‐26865455/+91‐9871145498 Hauz Khas, New Delhi‐16 
                                                   
                                             Website: www.physicsbyfiziks.com                                                                                          
                                                            Email: fiziks.physics@gmail.com                                                                   3 
fiziks
Institute for NET/JRF, GATE, IIT‐JAM, JEST, TIFR and GRE in PHYSICAL SCIENCES 
 
GATE-2011

Q9. The temperature (T) dependence of magnetic susceptibility (χ) of a ferromagnetic


substance with a Curie temperature (Tc) is given by
C C
(a) , for T  Tc (b) , for T  Tc
T  Tc T  Tc

C C
(c) , for T  Tc (d) , for all temperatures
T  Tc T  Tc

where C is constant .
Ans: (b)
Q10. The order of magnitude of the energy gap of a typical superconductor is
(a) 1 MeV (b) 1 KeV (c) 1 eV (d) 1 meV
Ans: (d)
Q11. For a three-dimensional crystal having N primitive unit cells with a basis of p atoms, the
number of optical branches is
(a) 3 (b) 3p (c) 3p – 3 (d) 3N – 3p
Ans: (c)
Q12. For an intrinsic semiconductor, me* and mh* are respectively the effective masses of
electrons and holes near the corresponding band edges. At a finite temperature the
position of the Fermi level
(a) depends on me* but not on mh* (b) depends on mh* but not on me*
(c) depends on both me* and mh* (d) depends neither on me* nor on mh*
Ans: (c)
Ec  Ev 3  mh* 
Solution: The Fermi level for intrinsic semicondutor is E F   k B T ln * 
2 4  me 
Q13. A metal with body centered cubic (bcc) structure show the first (i.e. smallest angle)
diffraction peak at a Bragg angle of θ = 30o. The wavelength of X-ray used is 2.1 Ǻ. The
volume of the PRIMITIVE unit cell of the metal is
(a) 26.2 (Ǻ)3 (b) 13.1(Ǻ)3 (c) 9.3 (Ǻ)3 (d) 4.6 (Ǻ)3
Ans: (b)
                                                                                
Head office  Branch office 
 
fiziks, H.No. 23, G.F, Jia Sarai,  Anand Institute of Mathematics, 
 
Near IIT, Hauz Khas, New Delhi‐16  28‐B/6, Jia Sarai, Near IIT 
 
Phone: 011‐26865455/+91‐9871145498 Hauz Khas, New Delhi‐16 
                                                   
                                             Website: www.physicsbyfiziks.com                                                                                          
                                                            Email: fiziks.physics@gmail.com                                                                   4 
fiziks
Institute for NET/JRF, GATE, IIT‐JAM, JEST, TIFR and GRE in PHYSICAL SCIENCES 
 
a
Solution: According to Bragg’s law 2d sin    where d 
h2  k 2  l 2
For BCC structure the first diffraction peak appear for (110) plane.
a 2a
d   sin 30 0    2a sin 30 0  2.1A 0
2 2
1
 2 a  2.1A 0  a  2  2.1A 0  a  2.97 A 0 .
2

The volume primitive unit cell of BCC is volume 


a 3 26.2 0
2

2
A   3
 13.1 A 0  3

Common Data for Questions 14 and 15:


The tight binding energy dispersion (E-k) relation for electrons in a one-dimensional
array of atoms having lattice constant a and total length L is
E = E0 – β – 2γ cos (ka),
where E0, β and γ are constants and k is the wave vector.
Q14. The density of states of electrons (including spin degeneracy) in the band is given by
L L L L
(a) (b) (c) (d)
 a sin ka  2 a sin ka  2 a coska   a coska 
Ans: (b)
 L  1  L  1 L
Solution: D E   2   2  
 2  dE / dk  2  2a sin ka  2a sin ka 
Q15. The effective mass of electrons in the band is given by
2 2 2 2
(a) (b) (c) (d)
 a 2 coska  2 a 2 coska   a 2 sin ka  2 a 2 sin ka 
Ans: (b)
2 2 2
Solution: Effective mass m *   
 d 2E  2a 2  coska  2a 2 coska 
 2 
 dk 

                                                                                
Head office  Branch office 
 
fiziks, H.No. 23, G.F, Jia Sarai,  Anand Institute of Mathematics, 
 
Near IIT, Hauz Khas, New Delhi‐16  28‐B/6, Jia Sarai, Near IIT 
 
Phone: 011‐26865455/+91‐9871145498 Hauz Khas, New Delhi‐16 
                                                   
                                             Website: www.physicsbyfiziks.com                                                                                          
                                                            Email: fiziks.physics@gmail.com                                                                   5 
fiziks
Institute for NET/JRF, GATE, IIT‐JAM, JEST, TIFR and GRE in PHYSICAL SCIENCES 
 
GATE-2012

Q16. For an ideal Fermi gas in three dimensions, the electron velocity VF at the Fermi surface
is related to electron concentration n as,
(a) V F  n 2 / 3 (b) V F  n (c) VF  n1 / 2 (d) V F  n1 / 3
Ans: (d)

Solution: V F 

m

3 2 n
1/ 3

Q17. The total energy, E of an ideal non-relativistic Fermi gas in three dimensions is given by
N 5/3
E where N is the number of particles and V is the volume of the gas. Identify the
V 2/3
CORRECT equation of state (P being the pressure),
1 2 5
(a) PV  E (b) PV  E (c) PV  E (d) PV  E
3 3 3
Ans: (b)
Q18. Which one of the following CANNOT be explained by considering a harmonic
approximation for the lattice vibrations in solids?
(a) Deby’s T3 law (b) Dulong Petit’s law
(c) Optical branches in lattices (d) Thermal expansion
Ans: (d)
Solution: Thermal expansion in solid can only be explained if solid behave as a anharmonic
oscillator.
Q19. A simple cubic crystal with lattice parameter a c undergoes transition into a tetragonal

structure with lattice parameters at  bt  2ac and ct  2a c , below a certain


temperature. The ratio of the interplanar spacing of (1 0 1) planes for the cubic and the
tetragonal structure is

1 1 3 3
(a) (b) (c) (d)
6 6 8 8
Ans: (c)
                                                                                
Head office  Branch office 
 
fiziks, H.No. 23, G.F, Jia Sarai,  Anand Institute of Mathematics, 
 
Near IIT, Hauz Khas, New Delhi‐16  28‐B/6, Jia Sarai, Near IIT 
 
Phone: 011‐26865455/+91‐9871145498 Hauz Khas, New Delhi‐16 
                                                   
                                             Website: www.physicsbyfiziks.com                                                                                          
                                                            Email: fiziks.physics@gmail.com                                                                   6 
fiziks
Institute for NET/JRF, GATE, IIT‐JAM, JEST, TIFR and GRE in PHYSICAL SCIENCES 
 
a a
Solution: For Cubic Lattice d c   c
h2  k 2  l 2 2

a 2a c
For Tetragonal lattice d t  
h k
2
l 2 2
3
2
 2
a c
dc 3
Therefore the ratio is 
dt 8
Q20. Inverse susceptibility (1/χ) as a function of temperature, T for a material undergoing
paramagnetic to ferromagnetic transition is given in the figure, where O is the origin. The
values of the Curie constant, C, and the Weiss molecular field constant, λ, in CGS units,
are
1
600 K

(a) C  5  10 5 ,   3  10 2

(b) C  3  10 2 ,   5  10 5 O T

(c) C  3  10 2 ,   2  10 4
 2  10 4
(d) C  2  10 ,   3  10
4 2
(CGS unit)

Ans: (c)
1 T  TC 1
Solution:  and TC  C . Here TC  600 K and  2  10 4
 C 
Thus C  3  10 2 and   2  10 4 .

Common Data for Questions 21–22


The dispersion relation for a one dimensional monatomic crystal with lattice spacing a,
which interacts nearest neighbour harmonic potential is given by
Ka
  A sin
2
where A is a constant of appropriate unit.
Q21. The group velocity at the boundary of the first Brillouin zone is

                                                                                
Head office  Branch office 
 
fiziks, H.No. 23, G.F, Jia Sarai,  Anand Institute of Mathematics, 
 
Near IIT, Hauz Khas, New Delhi‐16  28‐B/6, Jia Sarai, Near IIT 
 
Phone: 011‐26865455/+91‐9871145498 Hauz Khas, New Delhi‐16 
                                                   
                                             Website: www.physicsbyfiziks.com                                                                                          
                                                            Email: fiziks.physics@gmail.com                                                                   7 
fiziks
Institute for NET/JRF, GATE, IIT‐JAM, JEST, TIFR and GRE in PHYSICAL SCIENCES 
 
Aa 2 1 Aa 2
(a) 0 (b) 1 (c) (d)
2 2 2
Ans: (a)
Solution: At the first Brillouin zone the frequency is maximum and the group velocity which is
the derivative of the angular frequency is zero.
Q22. The force constant between the nearest neighbour of the lattice is (M is the mass of the
atom)
MA 2 MA 2
(a) (b) (c) MA2 (d) 2MA2
4 2

Ans: (a)

4K MA 2
Solution: A  K
M 4
GATE-2013

Q23. A phosphorous doped silicon semiconductor (doping density: 1017/cm3) is heated from
100C to 200C. Which one of the following statements is CORRECT?
(a) Position of Fermi level moves towards conduction band
(b) Position of dopant level moves towards conduction band
(c) Position of Fermi level moves towards middle of energy gap
(d) Position of dopant level moves towards middle of energy gap
Ans: (c)
Solution: Phosphorous doped silicon semiconductors behave as a n-type semiconductor. In
n-type semiconductor Fermi level lies near conduction band and moves toward middle of
the band gap upon heating. At a very high temperature the Fermi level is near the middle
of the band gap and semiconductor behaves as intrinsic semiconductor.
Q24. Considering the BCS theory of superconductors, which one of the following statements is
NOT CORRECT? ( h is the Plank’s constant and e is the electronic charge)
(a) Presence of energy gap at temperature below the critical temperature
(b) Different critical temperature for isotopes
                                                                                
Head office  Branch office 
 
fiziks, H.No. 23, G.F, Jia Sarai,  Anand Institute of Mathematics, 
 
Near IIT, Hauz Khas, New Delhi‐16  28‐B/6, Jia Sarai, Near IIT 
 
Phone: 011‐26865455/+91‐9871145498 Hauz Khas, New Delhi‐16 
                                                   
                                             Website: www.physicsbyfiziks.com                                                                                          
                                                            Email: fiziks.physics@gmail.com                                                                   8 
fiziks
Institute for NET/JRF, GATE, IIT‐JAM, JEST, TIFR and GRE in PHYSICAL SCIENCES 
 
h
(c) Quantization of magnetic flux in superconduction ring in the unit of  
e
(d) Presence of Meissner effect
Ans: (c)
 h
Solution: Quantization of magnetic flux in superconduction ring in the unit of  
 2e 
Q25. Group I contains elementary excitations in solids. Group II gives the associated field with
these exciations. MATCH the excitations with their associated field and select your
answer as per codes given below.
Group I Group II
(P) phonon (i) photon + lattice vibration
(Q) plasmon (ii) electron +elastic deformation
(R) polaron (iii) collective electron oscillations
(S) polariton (iv) elastic wave
Codes
(a) P  iv , Q  iii , R  i , S  ii  (b) P  iv , Q  iii , R  ii , S  i 
(c) P  i , Q  iii , R  ii , S  iv  (d) P  iii , Q  iv , R  ii , S  i 
Ans: (b)
Solution: Phonon: Quantum of energy of the elastic wave in solid, produced due to the vibration
of atoms in solid.
Plasmon: Quantum of energy of the wave produced due to the oscillation of plasma,
which contains charged particles (positive ions and negative electrons or ions).
Polaron: A charge placed in a polarizable medium will be screened. The induced
polarization will follow the charge carrier when it is moving through the medium. The
carrier together with the induced polarization is considered as one entity, which is called
a polaron.
Polariton: A polariton is a quasiparticle resulting from the mixing of a photon with
phonon.

                                                                                
Head office  Branch office 
 
fiziks, H.No. 23, G.F, Jia Sarai,  Anand Institute of Mathematics, 
 
Near IIT, Hauz Khas, New Delhi‐16  28‐B/6, Jia Sarai, Near IIT 
 
Phone: 011‐26865455/+91‐9871145498 Hauz Khas, New Delhi‐16 
                                                   
                                             Website: www.physicsbyfiziks.com                                                                                          
                                                            Email: fiziks.physics@gmail.com                                                                   9 
fiziks
Institute for NET/JRF, GATE, IIT‐JAM, JEST, TIFR and GRE in PHYSICAL SCIENCES 
 
Q26.
  

A lattice has the following primitive vector in Å  : a  2 ˆj  kˆ , b  2 kˆ  iˆ , c  2 iˆ  ˆj .     
The reciprocal lattice corresponding to the above lattice is
 
(a) BCC lattice with cube edge of  Å -1
2
(b) BCC lattice with cube edge of 2 Å -1

 
(c) FCC lattice with cube edge of  Å -1
2
(d) FCC lattice with cube edge of 2 Å -1
Ans: (a)
Solution: The reciprocal lattice vectors are
 
 b c 
a  2     - î  ĵ  k̂ Å -1
a b c 2    
 
 ca 
b   2     î  ĵ  k̂ Å -1
a b c 2    
 
 a b 
c   2     î  ĵ  k̂ Å -1
a b c 2    
 e2 B
Q27. The total energy of an ionic solid is given by an expression E    9 where 
4 0 r r
is Madelung constant, r is the distance between the nearest neighbours in the crystal and
B is a constant. If r0 is the equilibrium separation between the nearest neighbours then
the value of B is
 e 2 r08  e 2 r08 2 e 2 r010  e 2 r010
(a) (b) (c) (d)
36 0 4 0 9 0 36 0
Ans: (a)
dE  e2 9B  e 2 r08
Solution: At r  r0 , 0  10  B
dr r  r0 4 0 r0 r0
2
36 0

                                                                                
Head office  Branch office 
 
fiziks, H.No. 23, G.F, Jia Sarai,  Anand Institute of Mathematics, 
 
Near IIT, Hauz Khas, New Delhi‐16  28‐B/6, Jia Sarai, Near IIT 
 
Phone: 011‐26865455/+91‐9871145498 Hauz Khas, New Delhi‐16 
                                                   
                                             Website: www.physicsbyfiziks.com                                                                                          
                                                            Email: fiziks.physics@gmail.com                                                                   10 
fiziks
Institute for NET/JRF, GATE, IIT‐JAM, JEST, TIFR and GRE in PHYSICAL SCIENCES 
 
GATE-2014
Q28. The Miller indices of a plane passing through the three points having coordinates (0, 0, 1)

1, 0, 0  1 , 1 , 1  are
2 2 4
(a) (212) (b) (111) (c) (121) (d) (211)
Ans: (a)
Q29. The plot of specifies heat versus temperature across the superconducting transition
temperature Tc  is most appropriately represented by
(a) (b)

Cp Cp

TC T TC T

(c) (d)

Cp Cp

TC T TC T
Ans: (a)
  
 
Solution: CV  e  2kT 

                                                                                
Head office  Branch office 
 
fiziks, H.No. 23, G.F, Jia Sarai,  Anand Institute of Mathematics, 
 
Near IIT, Hauz Khas, New Delhi‐16  28‐B/6, Jia Sarai, Near IIT 
 
Phone: 011‐26865455/+91‐9871145498 Hauz Khas, New Delhi‐16 
                                                   
                                             Website: www.physicsbyfiziks.com                                                                                          
                                                            Email: fiziks.physics@gmail.com                                                                   11 
fiziks
Institute for NET/JRF, GATE, IIT‐JAM, JEST, TIFR and GRE in PHYSICAL SCIENCES 
 
Q30. The energy  k for band electrons as a function of the wave vector k in the first Brillouin

  
zone    k   of a one dimensional monatomic lattice is shown as ( a is lattice
 a a
k
constant)

k
  /a O  /a

The variation of the group velocity vg is most appropriately represented by

(a) vg (b) vg

  /a k   /a k
O  /a O  /a

(c) (d)
vg vg

  /a k   /a k
O  /a O  /a

Ans: (b)
  ka  
Solution: E   E0    cos  
   
1 dE ka
Vg    sin
 dk 

                                                                                
Head office  Branch office 
 
fiziks, H.No. 23, G.F, Jia Sarai,  Anand Institute of Mathematics, 
 
Near IIT, Hauz Khas, New Delhi‐16  28‐B/6, Jia Sarai, Near IIT 
 
Phone: 011‐26865455/+91‐9871145498 Hauz Khas, New Delhi‐16 
                                                   
                                             Website: www.physicsbyfiziks.com                                                                                          
                                                            Email: fiziks.physics@gmail.com                                                                   12 
fiziks
Institute for NET/JRF, GATE, IIT‐JAM, JEST, TIFR and GRE in PHYSICAL SCIENCES 
 
Q31. For Nickel the number density is 8  10 atoms / cm 3 and electronic configuration is
23

1s 2 2 s 2 2 p 6 3s 2 3 p 6 3d 8 4s 2 . The value of the saturation magnetization of Nickel in its

ferromagnetic state is _____________  10 9 A / m .

(Given the value of Bohr magneton  B  9.21  10 21 Am 2 )


Ans: 4.42
Solution: Component of magnetic dipoles in a solid material are in the direction of external field.
M S  (Magnetic dipole moment)  B N ,

0.6  9.21 1021  Am 2  N (For N i :magnetic dipole moment  0.6

Fe :2.22, For Cu :1.2 )


 NA
N  8  1029 / m3  B  9.211021 A / m
AN

M S  0.6  9.211021  8 1029  4.42 109 A / m

GATE-2015
Q31. The energy dependence of the density of states for a two dimensional non-relativistic
electron gas is given by, g E   CE n , where C is constant. The value of n
is____________
Ans.: 0
Solution: We know that
g  E   E1/ 2 for 3  D , g  E   E 0 for 2  D , g  E   E 1/ 2 for 1  D

 n  0 for 2  D
Q32. The lattice parameters a, b, c of an orthorhombic crystal are related by a  2b  3c . In
units of a the interplanar separation between the 110  planes is ____________. (Upto
three decimal places)
Ans.: 0.447

                                                                                
Head office  Branch office 
 
fiziks, H.No. 23, G.F, Jia Sarai,  Anand Institute of Mathematics, 
 
Near IIT, Hauz Khas, New Delhi‐16  28‐B/6, Jia Sarai, Near IIT 
 
Phone: 011‐26865455/+91‐9871145498 Hauz Khas, New Delhi‐16 
                                                   
                                             Website: www.physicsbyfiziks.com                                                                                          
                                                            Email: fiziks.physics@gmail.com                                                                   13 
fiziks
Institute for NET/JRF, GATE, IIT‐JAM, JEST, TIFR and GRE in PHYSICAL SCIENCES 
 
1 1 a
Solution: d hkl   d110    0.447  a  2b  3c
2
h k 2
l 2
1 1 5
   0
a 2 b2 c2 a 2  a 2
 
2
Q33. The dispersion relation for phonons in a one dimensional monatomic Bravais lattice with
lattice spacing a and consisting of ions of masses M is given by

2c
 k   1  coska  , where  is the frequency of oscillation, k is the wavevector
M
and C is the spring constant. For the long wavelength modes   a  , the ratio of the
phase velocity to the group velocity is_________
Ans.: 1

2C
Solution:   k   1  cos  ka  
M 
For long wavelength modes   a 

 ka  2C   ka  
2 2
C
 cos  ka   1   k   1  1   a k
2 M  2  M

 C d C
Phase velocity vP  a and Group velocity vg  a
k M dk M
vP
 1
vg

Q34. In a Hall effect experiment, the hall voltage for an intrinsic semiconductor is negative.
This is because (symbols carry usual meaning)
(a) n  p (b) n  p (c)  *   h (d) m*  mn*
Ans.: (c)
Solution: The Hall voltage is V H  RH JB
where J : current density, B : magnetic field and R H : Hall constant

                                                                                
Head office  Branch office 
 
fiziks, H.No. 23, G.F, Jia Sarai,  Anand Institute of Mathematics, 
 
Near IIT, Hauz Khas, New Delhi‐16  28‐B/6, Jia Sarai, Near IIT 
 
Phone: 011‐26865455/+91‐9871145498 Hauz Khas, New Delhi‐16 
                                                   
                                             Website: www.physicsbyfiziks.com                                                                                          
                                                            Email: fiziks.physics@gmail.com                                                                   14 
fiziks
Institute for NET/JRF, GATE, IIT‐JAM, JEST, TIFR and GRE in PHYSICAL SCIENCES 
 
1 p  p  n n   p  n   n  p B
2 2 2 2 2

RH 
e  n  p  2   p  n  2  2  2 B 2
n p n p

1  p  n
For intrinsic semiconductor n  p  ni  RH 
eni  p   n

In Intrinsic semiconductor  n   p , therefore Hall voltage is negative.

Q35. Which one of the following represents the electron occupancy for a superconductor in its
normal and superconducting states?

Normal Superconducting Normal Superconducting


state state state state
(a) (b)
f E  f E  f E  f E 

E E E E

Normal Superconducting Normal Superconducting


(c) state state (d) state state

f E  f E  f E  f E 

Ans. : (d) E E E E
Solution: In normal slide, some states below Fermi levels are empty and equal number of states
above Fermi levels are filled. If material converts into a superconductor, electrons above
the Fermi Level makes cooper pair and they fall back below level Fermi level as same
energy released during cooper pair formation. Therefore, correct option is (d).
Q36. Given that the Fermi energy of gold is 5.54 eV , the number density of electrons is

__________ 1028 m3 (upto one decimal place)


(Mass of electron  9.11  10 31 kg ; h  6.626  10 34 j  s; 1 eV  1.6  10 19 j )
Ans.: 5.9
Solution: Relation between electron density (n) and Fermi energy E F  is
                                                                                
Head office  Branch office 
 
fiziks, H.No. 23, G.F, Jia Sarai,  Anand Institute of Mathematics, 
 
Near IIT, Hauz Khas, New Delhi‐16  28‐B/6, Jia Sarai, Near IIT 
 
Phone: 011‐26865455/+91‐9871145498 Hauz Khas, New Delhi‐16 
                                                   
                                             Website: www.physicsbyfiziks.com                                                                                          
                                                            Email: fiziks.physics@gmail.com                                                                   15 
fiziks
Institute for NET/JRF, GATE, IIT‐JAM, JEST, TIFR and GRE in PHYSICAL SCIENCES 
 
 2m 
3/ 2
2
EF  3 2 n  2/3
n
1
3 2 3
 EF3/ 2
2m

 2  9.110 kg 
31 3/ 2
1
  5.54  1.6 109 J 
3/ 2
n 
3   3.14  1.0546  10 J  s
2 34

1 2.45  1045  8.35 1028 3


n  m  0.59  10 29 m 3
29.61 1.17 10102
 n  5.9  1028 m 3
GATE-2016
Q37. Consider a metal which obeys the Sommerfeld model exactly. If E F is the Fermi energy
of the metal at T  0 K and R H is its Hall coefficient, which of the following statements
is correct?
3 2

(a) R H  E 2
F (b) R H  E 3
F

3

(c) RH  E F2 (d) R H is independent of EF .


Ans.: (c)
1
Solution: RH 
ne
3/ 2 3/ 2
2
   2m   EF 
2/3
where EF  3 2 n n 2   2 
 RH  EF3 / 2
2m    3 
Q38. A one-dimensional linear chain of atoms contains two types of atoms of masses m1 and
m2 (where m2  m1 ), arranged alternately. The distance between successive atoms is the
same. Assume that the harmonic approximation is valid. At the first Brillouin zone
boundary, which of the following statements is correct?
(a) The atoms of mass m 2 are at rest in the optical mode, while they vibrate in the
acoustical mode.
(b The atoms of mass m1 are at rest in the optical mode, while they vibrate in the
acoustical mode.

                                                                                
Head office  Branch office 
 
fiziks, H.No. 23, G.F, Jia Sarai,  Anand Institute of Mathematics, 
 
Near IIT, Hauz Khas, New Delhi‐16  28‐B/6, Jia Sarai, Near IIT 
 
Phone: 011‐26865455/+91‐9871145498 Hauz Khas, New Delhi‐16 
                                                   
                                             Website: www.physicsbyfiziks.com                                                                                          
                                                            Email: fiziks.physics@gmail.com                                                                   16 
fiziks
Institute for NET/JRF, GATE, IIT‐JAM, JEST, TIFR and GRE in PHYSICAL SCIENCES 
 
(c) Both types of atoms vibrate with equal amplitudes in the optical as well as in the
acoustical modes.
(d) Both types of atoms vibrate, but with unequal, non-zero amplitudes in the optical as
well as in the acoustical modes.
Ans.: (a)
Solution: In optical mode, at Brillouin zone boundary atom of heavier mass  m2  is at rest,

whereas in Acoustic mode, atoms of lighter mass  m1  is at rest.

 1 1 
2   
optical mode m
 1 m2 

2  2  / m1
Acoustic 1  2  / m2
mode
 
2a k 2a
Q39. A solid material is found to have a temperature independent magnetic susceptibility,
  C . Which of the following statements is correct?
(a) If C is positive, the material is a diamagnet.
(b) If C is positive, the material is a ferromagnet.
(c) If C is negative, the material could be a type I superconductor.
(d) If C is positive, the material could be a type I superconductor.
Ans.: (b)

Solution: Susceptibility is defined as    1 , where   0 is permeability of medium and
0
vacuum respectively.
(i) For Diamagnet;   0 , thus   0 i.e.  is negative

(ii) For Ferromagnet,   0 , thus   0 i.e.  is positive

(iii) For superconductor,   0 , thus   1


Thus best answer is (b)
                                                                                
Head office  Branch office 
 
fiziks, H.No. 23, G.F, Jia Sarai,  Anand Institute of Mathematics, 
 
Near IIT, Hauz Khas, New Delhi‐16  28‐B/6, Jia Sarai, Near IIT 
 
Phone: 011‐26865455/+91‐9871145498 Hauz Khas, New Delhi‐16 
                                                   
                                             Website: www.physicsbyfiziks.com                                                                                          
                                                            Email: fiziks.physics@gmail.com                                                                   17 
fiziks
Institute for NET/JRF, GATE, IIT‐JAM, JEST, TIFR and GRE in PHYSICAL SCIENCES 
 
Q40. Atoms, which can be assumed to be hard spheres of radius R , are arranged in an fcc
lattice with lattice constant a , such that each atom touches its nearest neighbours. Take
the center of one of the atoms as the origin. Another atom of radius r (assumed to be
 a 
hard sphere) is to be accommodated at a position  0, ,0  without distorting the lattice.
 2 
r
The maximum value of is ________. (Give your answer upto two decimal places)
R
Ans.: 0.41
 a 
Solution: The new atom location is  0, , 0  i.e. it is on the middle of y - axis.
 2 
z
a

2a

Position of new r
x
 a   a 
atom  0, , 0  a new atom at  0, ,0 
figure (i)  2   2 
2
figure (ii)
If new atom of radius r fit without distorting the original lattice, then from figure (ii) we get
a
 Rr (i)
2
4
whereas for FCC 2a  4 R  a  R  2 2R (ii)
2
Thus from (i) and (ii)

2 2
2
R  Rr   
2 1 R  r 
r
R
 2  1  1.414  1  0.414

                                                                                
Head office  Branch office 
 
fiziks, H.No. 23, G.F, Jia Sarai,  Anand Institute of Mathematics, 
 
Near IIT, Hauz Khas, New Delhi‐16  28‐B/6, Jia Sarai, Near IIT 
 
Phone: 011‐26865455/+91‐9871145498 Hauz Khas, New Delhi‐16 
                                                   
                                             Website: www.physicsbyfiziks.com                                                                                          
                                                            Email: fiziks.physics@gmail.com                                                                   18 
fiziks
Institute for NET/JRF, GATE, IIT‐JAM, JEST, TIFR and GRE in PHYSICAL SCIENCES 
 
Q41. The energy vs. wave vector E  k  relationship near the bottom of a band for a solid can

be approximated as E  Aka   Bka  , where the lattice constant a  2.1 A . The


2 4 0

values of A and B are 6.3  10 19 J and 3.2 10 20 J , respectively. At the bottom of
the conduction band, the ratio of the effective mass of the electron to the mass of free
electron is _______. (Give your answer upto two decimal places)
(Take   1.05  10 34 J  s , mass of free electron  9.110 31 kg )
Ans.: 0.22

Solution: E  A  ka   B  ka 
2 4

E 2 E
  2 Aa 2 k  4 Ba 4 k 3 and  2 Aa 2  12 Ba 4 k 2
k k 2

At the bottom of the band k  0


2 2
Thus effective mass m*  2 
 E / k 2 2 Aa 2
1.05  10 
34 2
J s 1.1025  1068
m  *
  0.01984  1029
 
2 39
2  6.3  1019 J  2.1  1010 m 55.57  10

m* 19.84  1032 kg
 19.84  1032 kg   31
 2.18  101  0.218  0.22
m 9.1  10 kg
Q42. The Fermi energies of two metals X and Y are 5 eV and 7 eV and their Debye
temperatures are 170 K and 340 K , respectively. The molar specific heats of these
metals at constant volume at low temperatures can be written as
CV  X   X T  AX T 3 and CV Y   Y T  AY T 3 where  and A are constants. Assuming
that the thermal effective mass of the electrons in the two metals are same, which of the
following is correct?
 X 7 AX  X 7 AX 1
(a)  , 8 (b)  , 
 Y 5 AY  Y 5 AY 8
 X 5 AX 1  X 5 AX
(c)  ,  (d)  , 8
 Y 7 AY 8  Y 7 AY

                                                                                
Head office  Branch office 
 
fiziks, H.No. 23, G.F, Jia Sarai,  Anand Institute of Mathematics, 
 
Near IIT, Hauz Khas, New Delhi‐16  28‐B/6, Jia Sarai, Near IIT 
 
Phone: 011‐26865455/+91‐9871145498 Hauz Khas, New Delhi‐16 
                                                   
                                             Website: www.physicsbyfiziks.com                                                                                          
                                                            Email: fiziks.physics@gmail.com                                                                   19 
fiziks
Institute for NET/JRF, GATE, IIT‐JAM, JEST, TIFR and GRE in PHYSICAL SCIENCES 
 
Ans.: (a)
Solution: Heat capacity is defined as CV   T  AT 3

3 2 1 12 4 1 1
where   Nk B  and A  Nk B  3  234 Nk B  3
2 EF 5 D D
3 2 1
Nk B 
x 2 EFx EF y 7eV 7
Thus,    
 y 3 Nk 2  1 EFx 5eV 5
B
2 EFy

1
234 Nk B 3
Ax  D3 x   Dy   340 3
  2  8
3
and    
Ay 234 Nk 1  D   170 
B 3  x 
 Dy

x 7 A
Thus,  and x  8
y 5 Ay

GATE-2017

Q43. The atomic mass and mass density of Sodium are 23 and 0.968 g cm 3 , respectively. The

number density of valence electrons is……………… 1022 cm 3 . (Up to two decimal

places) (Avogadro number, N A  6.022 1023 )

Ans. : 2.54
neff  M neff  NA
Solution:  n 
NA  a 3
a 3
M

where   0.968gcm 3 , N A  6.022 1023 , M  23g

0.968  6.022  1023


n  2.54  1022 cm 3
23

                                                                                
Head office  Branch office 
 
fiziks, H.No. 23, G.F, Jia Sarai,  Anand Institute of Mathematics, 
 
Near IIT, Hauz Khas, New Delhi‐16  28‐B/6, Jia Sarai, Near IIT 
 
Phone: 011‐26865455/+91‐9871145498 Hauz Khas, New Delhi‐16 
                                                   
                                             Website: www.physicsbyfiziks.com                                                                                          
                                                            Email: fiziks.physics@gmail.com                                                                   20 
fiziks
Institute for NET/JRF, GATE, IIT‐JAM, JEST, TIFR and GRE in PHYSICAL SCIENCES 
 
 2 x 
Q44. Consider a one dimensional lattice with a weak periodic potential U  x   U 0 cos  .
 a 
 
The gap at the edge of the Brillouin zone  k   is:
 a
U0 U0
(a) U 0 (b) (c) 2U 0 (d)
2 4
Ans. : (c)
 2 
Solution: U  U 0 cos  x
 a 
 2 a 
Energy at the edge of Brillouin Zone is U t  U 0 cos  . 
 a 
Energy at the k  0 is U b  U 0

 Band gap U  U t  U b  2U 0


Thus correct option is (c)
Q45. Consider a 2 - dimensional electron gas with a density of 1019 m2 . The Fermi energy of
the system is………………… eV (up to two decimal places).
( me  9.311031 kg , h  6.626 1034 Js, e  1.602  1019 C )
 A EF 2 AmEF
N   f  E  g  E dE  2  
1 2m  2 E 0 dE 
0 h 0 h2

 
2
nh 2 1019  6.6 1034
EF    0.37 1018 J  2.32eV
4 m 4  3.14  9.311031
Ans. : 2.34

1.055 1034 J  s 
2
2
  
Solution: EF     2 n   31
 2  3.142 1019
 2 m  2  9.31  10

0.3756 1018 J  0.2345 10 eV  2.34 eV

                                                                                
Head office  Branch office 
 
fiziks, H.No. 23, G.F, Jia Sarai,  Anand Institute of Mathematics, 
 
Near IIT, Hauz Khas, New Delhi‐16  28‐B/6, Jia Sarai, Near IIT 
 
Phone: 011‐26865455/+91‐9871145498 Hauz Khas, New Delhi‐16 
                                                   
                                             Website: www.physicsbyfiziks.com                                                                                          
                                                            Email: fiziks.physics@gmail.com                                                                   21 
fiziks
Institute for NET/JRF, GATE, IIT‐JAM, JEST, TIFR and GRE in PHYSICAL SCIENCES 
 
Q46.
  a
The real space primitive lattice vectors are a1  axˆ and a2  xˆ  3 yˆ . The reciprocal
2
 
 
space unit vectors b1 and b2 for this lattice are, respectively

2  yˆ  4 2  yˆ  4
(a)
a  xˆ   and yˆ (b)
a  xˆ   and yˆ
 3 a 3  3 a 3

2 4  xˆ  2 4  xˆ 
(c) xˆ and   yˆ  (d) xˆ and   yˆ 
a 3 a  3  a 3 a  3 
Ans. (a)

Solution:

a1  axˆ ,
 a
a2  xˆ  3 yˆ
2
 

assume, a3  zˆ

   a 
Now, a1   a2  a3   axˆ.  xˆ  3 yˆ  zˆ 
2 
 

a 2 xˆ
2
 a2

  yˆ  3xˆ   0  3  
2
3a 2
2

  
a2  a3  3 xˆ  yˆ  a2
b  2     2
a1   a2  a3  3 2
a
3
 2  yˆ 
b1   xˆ  
a  3
  
a3  a1 4
Similarly, b2  2     yˆ
a1   a2  a3  3a
Thus correct option is (a)

                                                                                
Head office  Branch office 
 
fiziks, H.No. 23, G.F, Jia Sarai,  Anand Institute of Mathematics, 
 
Near IIT, Hauz Khas, New Delhi‐16  28‐B/6, Jia Sarai, Near IIT 
 
Phone: 011‐26865455/+91‐9871145498 Hauz Khas, New Delhi‐16 
                                                   
                                             Website: www.physicsbyfiziks.com                                                                                          
                                                            Email: fiziks.physics@gmail.com                                                                   22 
fiziks
Institute for NET/JRF, GATE, IIT‐JAM, JEST, TIFR and GRE in PHYSICAL SCIENCES 
 
NUCLEAR AND PARTICLE PHYSICS SOLUTIONS

GATE-2010
Q1. The basic process underlying the neutron β-decay is
(a) d  u  e    e (b) d  u  e 
(c) s  u  e    e (d) u  d  e    e
Ans: (a)
Q2. In the nuclear shell model the spin parity of 15N is given by
1 1 3 3
(a) (b) (c) (d)
2 2 2 2
Ans: (a)
Solution: Z  7 ; s1 / 2   p3 / 2 4  p1 / 2 1
2
and N  8

1 1
 parity   1  1,
1
l  1, J  spin - parity   
2 2
Q3. Match the reactions on the left with the associated interactions on the right.
(1) π+ → μ+ +  (i) Strong

(2) π0 → γ + γ (ii) Electromagnetic


(3) π0 + n → π- + p (iii) Weak
(a) (1, iii), (2, ii), (3, i) (b) (1, i), (2, ii), (3, iii)
(c) (1, ii), (2, i), (3, iii) (d) (1, iii), (2, i), (3, ii)
Ans: (a)
Q4. The ground state wavefunction of deuteron is in a superposition of s and d states. Which
of the following is NOT true as a consequence?
(a) It has a non-zero quadruple moment
(b) The neutron-proton potential is non-central
(c) The orbital wavefunction is not spherically symmetric
(d) The Hamiltonian does not conserve the total angular momentum
Ans: (d)

                                                                                
Head office  Branch office 
 
fiziks, H.No. 23, G.F, Jia Sarai,  Anand Institute of Mathematics, 
 
Near IIT, Hauz Khas, New Delhi‐16  28‐B/6, Jia Sarai, Near IIT 
 
Phone: 011‐26865455/+91‐9871145498 Hauz Khas, New Delhi‐16 
                                                   
                                             Website: www.physicsbyfiziks.com                                                                                          
                                                            Email: fiziks.physics@gmail.com                                                                   1 
fiziks
Institute for NET/JRF, GATE, IIT‐JAM, JEST, TIFR and GRE in PHYSICAL SCIENCES 
 
228
Q5. The first three energy levels of Th90 are shown below

4 187 keV
2 57.5keV
0 0keV

The expected spin-parity and energy of the next level are given by
(a) (6+; 400 keV) (b) (6+; 300 keV) (c) (2+; 400 keV) (d) (4+; 300 keV)
Ans: (a)
E 2 J 2  J 2  1 E 66  1
Solution:   6   E 6  393keV
E1 J 1  J 1  1 E 4 44  1

GATE-2011

Q6. The semi-empirical mass formula for the binding energy of nucleus contains a surface
correction term. This term depends on the mass number A of the nucleus as
(a) A-1/3 (b) A1/3 (c) A2/3 (d) A
Ans: (c)
Q7. According to the single particles nuclear shell model, the spin-parity of the ground state
of 178 O is
   
1 3 3 5
(a) (b) (c) (d)
2 2 2 2
Ans: (d)
Solution: Z  8 and N  9; s1 / 2   p3 / 2 4  p1 / 2 2 d 5 / 2 1
2


5 5
l  2, J   parity   1  1, spin - parity   
2

2 2

                                                                                
Head office  Branch office 
 
fiziks, H.No. 23, G.F, Jia Sarai,  Anand Institute of Mathematics, 
 
Near IIT, Hauz Khas, New Delhi‐16  28‐B/6, Jia Sarai, Near IIT 
 
Phone: 011‐26865455/+91‐9871145498 Hauz Khas, New Delhi‐16 
                                                   
                                             Website: www.physicsbyfiziks.com                                                                                          
                                                            Email: fiziks.physics@gmail.com                                                                   2 
fiziks
Institute for NET/JRF, GATE, IIT‐JAM, JEST, TIFR and GRE in PHYSICAL SCIENCES 
 
Q8. In the β-decay of neutron n→ p + e + e , the anti-neutrino e , escapes detection. Its
-

existence is inferred from the measurement of


(a) energy distribution of electrons (b) angular distribution of electrons
(c) helicity distribution of electrons (d) forward-backward asymmetry of electrons
Ans: (a)
Q9. The isospin and the strangeness of   baryon are
(a) 1, -3 (b) 0, -3 (c) 1, 3 (d) 0, 3
Ans: (b)

GATE-2012

Q10. Deuteron has only one bound state with spin parity 1+, isospin 0 and electric quadrupole
moment 0.286 efm2. These data suggest that the nuclear forces are having
(a) only spin and isospin dependence
(b) no spin dependence and no tensor components
(c) spin dependence but no tensor components
(d) spin dependence along with tensor components
Ans: (d)

Q11. The quark content of   , K  ,   and p is indicated:

   uus ; K   su ;    u d ; p  uud .

In the process,    p  K     , considering strong interactions only, which of the


following statements is true?
(a) The process, is allowed because ∆S = 0
(b) The process is allowed because ∆I3 =0
(c) The process is not allowed because ∆S ≠ 0 and ∆I3 ≠ 0
(d) The process is not allowed because the baryon number is violated

                                                                                
Head office  Branch office 
 
fiziks, H.No. 23, G.F, Jia Sarai,  Anand Institute of Mathematics, 
 
Near IIT, Hauz Khas, New Delhi‐16  28‐B/6, Jia Sarai, Near IIT 
 
Phone: 011‐26865455/+91‐9871145498 Hauz Khas, New Delhi‐16 
                                                   
                                             Website: www.physicsbyfiziks.com                                                                                          
                                                            Email: fiziks.physics@gmail.com                                                                   3 
fiziks
Institute for NET/JRF, GATE, IIT‐JAM, JEST, TIFR and GRE in PHYSICAL SCIENCES 
 
Ans: (c)
Solution:    p  k    

S: 0 0 1 1 (not conserved)
1 1
I3 : 1   1 (not conserved)
2 2
For strong interaction S and I3 must conserve. Therefore this process is not allowed under
strong interaction
Q12. Which one of the following sets corresponds to fundamental particles?
(a) proton, electron and neutron
(b) proton, electron and photon
(c) electron, photon and neutrino
(d) quark, electron and meson
Ans: (a)
Q13. In case of a Geiger-Muller (GM) counter, which one of the following statement is
CORRECT?
(a) Multiplication factor of the detector is of the order of 1010
(b) Type of the particles detected can be identified
(c) Energy of the particles detected can be distinguished
(d) Operating voltage of the detector is few tens of Volts
Ans: (c)
Q14. Choose the CORRECT statement from the following
(a) Neutron interacts through electromagnetic interaction
(b) Electron does not interact through weak interaction
(c) Neutrino interacts through weak and electromagnetic interaction
(d) Quark interacts through strong interaction but not through weak interaction
Ans: (d)

                                                                                
Head office  Branch office 
 
fiziks, H.No. 23, G.F, Jia Sarai,  Anand Institute of Mathematics, 
 
Near IIT, Hauz Khas, New Delhi‐16  28‐B/6, Jia Sarai, Near IIT 
 
Phone: 011‐26865455/+91‐9871145498 Hauz Khas, New Delhi‐16 
                                                   
                                             Website: www.physicsbyfiziks.com                                                                                          
                                                            Email: fiziks.physics@gmail.com                                                                   4 
fiziks
Institute for NET/JRF, GATE, IIT‐JAM, JEST, TIFR and GRE in PHYSICAL SCIENCES 
 
GATE-2013
Q15. The decay process n  p   e   v e violates
(a) Baryon number (b) lepton number (c) isospin (d) strangeness
Ans: (c)
Q16. The isospin I  and baryon number B  of the up quark is
(a) I  1, B  1 (b) I  1, B  1 / 3
(c) I  1 / 2, B  1 (d) I  1 / 2, B  1 / 3
Ans: (d)
Q17. In the  decay process, the transition 2   3  , is
(a) allowed both by Fermi and Gamow-Teller selection rule
(b) allowed by Fermi and but not by Gamow-Teller selection rule
(c) not allowed by Fermi but allowed by Gamow-Teller selection rule
(d) not allowed both by Fermi and Gamow-Teller selection rule
Ans: (c)
Solution: According to Fermi Selection Rule:
I  0, Parity  No Change
According to Gammow-Teller Selection Rule:
I  0,1, Parity  No Change

In the  decay process, the transition 2   3  ,


I  1, Parity  No Change .
GATE-2014

Q18. Which one of the following is a fermions’?


(a)  -particle (b) 4 Be7 nucleus
(c) Hydrogen atom (d) deuteron

                                                                                
Head office  Branch office 
 
fiziks, H.No. 23, G.F, Jia Sarai,  Anand Institute of Mathematics, 
 
Near IIT, Hauz Khas, New Delhi‐16  28‐B/6, Jia Sarai, Near IIT 
 
Phone: 011‐26865455/+91‐9871145498 Hauz Khas, New Delhi‐16 
                                                   
                                             Website: www.physicsbyfiziks.com                                                                                          
                                                            Email: fiziks.physics@gmail.com                                                                   5 
fiziks
Institute for NET/JRF, GATE, IIT‐JAM, JEST, TIFR and GRE in PHYSICAL SCIENCES 
 
Ans: (b)
If a nucleus contains odd number of nucleons, it is fermions. If a nucleus contains even number
of nucleons, it is a boson.
Q19. Which one of the following three-quark states  qqq  denoted by X CANNOT be a

possible baryon? The corresponding electric charge is indicated in the superscript.


(a) X   (b) X  (c) X  (d) X  
Ans: (d)
Solution: X  qqq
2 2 2 6
X   uuu      2  two unit positive charge 
3 3 3 3
2 2 1 4 1
X   uud       1 single unit positive charge 
3 3 3 3 3
1 1 1
X   ddd       1 single unit negative charge 
3 3 3
X   Not possible with qqq  . So the correct option is (d)

Q20. Consider the process           . The minimum kinetic energy of the muons
  in the centre of mass frame required to produce the pion   pairs at rest is
______ MeV .
Ans: 81.7
Solution: Use conservation of energy and momentum in relativistic form.
m  105 MeV / c 2 and m  140 MeV / c 2

m   c
2 2

   m  c 2  m   m  2
 280 
2
MeV   210  MeV
2

E   E   163.3 MeV
2m 2 105

163.3
For pair it will be MeV  81.7 MeV
2

                                                                                
Head office  Branch office 
 
fiziks, H.No. 23, G.F, Jia Sarai,  Anand Institute of Mathematics, 
 
Near IIT, Hauz Khas, New Delhi‐16  28‐B/6, Jia Sarai, Near IIT 
 
Phone: 011‐26865455/+91‐9871145498 Hauz Khas, New Delhi‐16 
                                                   
                                             Website: www.physicsbyfiziks.com                                                                                          
                                                            Email: fiziks.physics@gmail.com                                                                   6 
fiziks
Institute for NET/JRF, GATE, IIT‐JAM, JEST, TIFR and GRE in PHYSICAL SCIENCES 
 
Q21. A nucleus X undergoes a first forbidden  -decay to nucleus Y . If the angular
7
momentum I  and parity P  , denoted by I P as for X , which of the following is a
2
possible I P value for Y ?
1 1 3 3
(a) (b) (c) (d)
2 2 2 2
Ans: (c)
For first forbidden  -decay; I  0,1 or 2 and Parity does change.

GATE-2015
Q22. The decay    e    is forbidden, because it violates
(a) momentum and lepton number conservations
(b) baryon and lepton number conservations
(c) angular momentum conservation
(d) lepton number conservation
Ans.: (d)
Solution:    e    . In this decay lepton number is not conserved.
Q23. A beam of X - ray of intensity I 0 is incident normally on a metal sheet of thickness

2 mm . The intensity of the transmitted beam is 0.025 I 0 . The linear absorption

 
coefficient of the metal sheet in m 1 is _______________ (upto one decimal place)
Ans.: 1844.4
x 1  0  1    1
Solution: I  I 0 e  ln    3
ln   3
ln  40 
x    2  10  0.025 I 0  2  10
2.303
3 
 log10 40  1.151 103  2  0.3010  1  1844.4 m 1
2  10
Q24. The mean kinetic energy of a nucleon in a nucleus of atomic weight A varies as An ,
where n is____________(upto two decimal places)

                                                                                
Head office  Branch office 
 
fiziks, H.No. 23, G.F, Jia Sarai,  Anand Institute of Mathematics, 
 
Near IIT, Hauz Khas, New Delhi‐16  28‐B/6, Jia Sarai, Near IIT 
 
Phone: 011‐26865455/+91‐9871145498 Hauz Khas, New Delhi‐16 
                                                   
                                             Website: www.physicsbyfiziks.com                                                                                          
                                                            Email: fiziks.physics@gmail.com                                                                   7 
fiziks
Institute for NET/JRF, GATE, IIT‐JAM, JEST, TIFR and GRE in PHYSICAL SCIENCES 
 
Ans.: -0.67
R 2  d 2 2 d  2 2
   2  4 r dr
2 R
0 2m  dr r dr   4   2  2  dr  4  4 R
Solution:  T   2m 0
 2 m
R R
4 R 3 / 3
 4 r 2 dr   2
4 r dr
0 0

2
R 1 1 1  2
 T  3
 2
 2
 2
 A 3
 n    0.667  0.67
R R  1
 3
R A
 0 
3 A3
 

152 152
Q25. The atomic masses of 63 Eu, 62 Sm, 11 H and neutron are 151.921749, 151.919756,
1.007825 and 1.008665 in atomic mass units (amu), respectively. Using the above
3
information, the Q - value of the reaction 152
63 Eu  n 152
62 Sm  p is ___________  10

amu (upto three decimal places)


Ans.: 2.833
Solution: Q  152.930414  152.927581  2.833  103 a.m.u.

1  
Q26. In the nuclear shell model, the potential is modeled as V r   m 2 r 2  L  S ,   0 .
2
The correct spin-parity and isospin assignments for the ground state of 13 C is
1  1 1  1 3 1 3  1
(a) ; (b) ; (c) ; (d) ;
2 2 2 2 2 2 2 2
Ans.: (a)
Solution: 13C6 , N  7, Z  6
2 4 1
      1
For N  7 ; 1S 1  1P3   P1   j  and l  1
 2  2  2 2

1
Thus spin- parity is   .
2

                                                                                
Head office  Branch office 
 
fiziks, H.No. 23, G.F, Jia Sarai,  Anand Institute of Mathematics, 
 
Near IIT, Hauz Khas, New Delhi‐16  28‐B/6, Jia Sarai, Near IIT 
 
Phone: 011‐26865455/+91‐9871145498 Hauz Khas, New Delhi‐16 
                                                   
                                             Website: www.physicsbyfiziks.com                                                                                          
                                                            Email: fiziks.physics@gmail.com                                                                   8 
fiziks
Institute for NET/JRF, GATE, IIT‐JAM, JEST, TIFR and GRE in PHYSICAL SCIENCES 
 
GATE-2016
Q27. In the SU 3 quark model, the triplet of mesons   ,  0 ,    has
(a) Isospin  0 , Strangeness  0 (b) Isospin  1 , Strangeness  0
1 1
(c) Isospin  , Strangeness  1 (d) Isospin  , Strangeness  1
2 2
Ans.: (b)
Solution:   ,  0 ,   are not strange particle thus strangness  0
Since meson group contain 3 particles, thus I  1

25 Mn  e  24 Cr  X . The particle X is
Consider the reaction 54 54
Q28.

(a)  (b)  e (c) n (d)  0


Ans.: (b)
Q29. Which of the following statements is NOT correct?
(a) A deuteron can be disintegrated by irradiating it with gamma rays of energy 4 MeV .
(b) A deuteron has no excited states.
(c) A deuteron has no electric quadrupole moment.
(d) The 1 S 0 state of deuteron cannot be formed.
Ans.: (c)
Q30. According to the nuclear shell model, the respective ground state spin-parity values of
15 17
8 O and 8 O nuclei are

1 1 1 5 3 5  3  1
(a) , (b) , (c) , (d) ,
2 2 2 2 2 2 2 2
Ans.: (d)
N  7 :  s1/ 2   p3/ 2   p1/ 2 
2 4 1
8 O ; Z  8 and N  7 ;
Solution: 15

1 1
 j and l  1 . Thus spin and parity   
2 2

N  9 :  s1/ 2   p3/ 2   p1/ 2   d5/ 2 


2 4 2 1
17
8 O ; Z  8 and N  9 ;

5 5
 j  and l  2 . Thus spin and parity   
2 2

                                                                                
Head office  Branch office 
 
fiziks, H.No. 23, G.F, Jia Sarai,  Anand Institute of Mathematics, 
 
Near IIT, Hauz Khas, New Delhi‐16  28‐B/6, Jia Sarai, Near IIT 
 
Phone: 011‐26865455/+91‐9871145498 Hauz Khas, New Delhi‐16 
                                                   
                                             Website: www.physicsbyfiziks.com                                                                                          
                                                            Email: fiziks.physics@gmail.com                                                                   9 
fiziks
Institute for NET/JRF, GATE, IIT‐JAM, JEST, TIFR and GRE in PHYSICAL SCIENCES 
 
GATE-2017

Q31. Which one of the following conservation laws is violated in the decay         
(a) Angular momentum (b) Total Lepton number
(c) Electric charge (d) Tau number
Ans. : (d)
Solution:        
q  1 1  1  1 conserved
L  1 1  1  1 conserved
L  1 0 0 0 Not conserved

1 1 1
spin = 1 conserved
2 2 2 2
Tau number is not conserved
Q32. Electromagnetic interactions are:
(a) C conserving
(b) C non-conserving but CP conserving
(c) CP non-conserving but CPT conserving
(d) CPT non-conserving
Ans. : (a)
Solution: In electromagnetic interaction C is conserved
CPT: Conserved in all interaction
CP: Conserved in EM and Strong interactions
13.6
En   eV 
n2
For n  1 E1  13.6ev Ground state

For n   E  0 Highest state


Thus correct option is (a)

                                                                                
Head office  Branch office 
 
fiziks, H.No. 23, G.F, Jia Sarai,  Anand Institute of Mathematics, 
 
Near IIT, Hauz Khas, New Delhi‐16  28‐B/6, Jia Sarai, Near IIT 
 
Phone: 011‐26865455/+91‐9871145498 Hauz Khas, New Delhi‐16 
                                                   
                                             Website: www.physicsbyfiziks.com                                                                                          
                                                            Email: fiziks.physics@gmail.com                                                                   10 
fiziks
Institute for NET/JRF, GATE, IIT‐JAM, JEST, TIFR and GRE in PHYSICAL SCIENCES 
 
Q33. In the nuclear reaction C6  e  N 7  X , the particle X is
13 13

(a) an electron (b) an anti-electron


(c) a muon (d) a pion
Ans. : (a)
Solution: 13C6  e  13N 7  X

 13C6  13N 7  X  e

Le  0 0 1 1

To conserve the Lepton number Le , x should be e 

Q34. J P for the ground state of the 13 C6 nucleus is

 3 3 1
(a) 1 (b) (c) (d)
2 2 2
Ans. : (d)
Solution: 13 C6 : Z  6, N  7

 s1/ 2   p3/ 2   p1/ 2 


2 4 1
N  7:

1 1
 j  and l  1 . Thus spin and parity   
2 2
Q35. The   decays at rest to   and v . Assuming the neutrino to be massless, the

momentum of the neutrino is…………….. MeV / c . (up to two decimal places)


( m  139 MeV / c 2 , m  105 MeV / c 2 )

Ans. : 29.84

Solution: E 
m 
2

 m2 c 2
 pc
2m

So p
m 2
  m2 c  
19321  11025
 29.84
2m 2  139

                                                                                
Head office  Branch office 
 
fiziks, H.No. 23, G.F, Jia Sarai,  Anand Institute of Mathematics, 
 
Near IIT, Hauz Khas, New Delhi‐16  28‐B/6, Jia Sarai, Near IIT 
 
Phone: 011‐26865455/+91‐9871145498 Hauz Khas, New Delhi‐16 
                                                   
                                             Website: www.physicsbyfiziks.com                                                                                          
                                                            Email: fiziks.physics@gmail.com                                                                   11 
fiziks
Institute for NET/JRF, GATE, IIT-JAM, JEST, TIFR and GRE in PHYSICAL SCIENCES

PH: PHYSICS

2014

GATE Test Paper

Duration: Three Hours Maximum Marks: 100

Q. 1 – Q. 25 carry one mark each.

Q1. Consider an anti-symmetric tensor Pij with indices i and j running from 1 to 5. The
number of independent components of the tensor is
(A) 3 (B) 10 (C) 9 (D) 6

Cos  sx 
Q2. The value of the integral  2 2
dx , is

k  x

 2  ks   ks  ks   ks
(A) e (B) e (C) e (D) e
k2 k2 k k
2 3 0
 
Q3. The eigenvalues of the matrix  3 2 0  are
0 0 1 

(A) 5, 2, -2 (B) -5, -1, -1 (C) 5, 1, -1 (D) -5, 1, 1

Q4. Which of the following is correct expression of probability current density in one
dimension .
i  d * * d  i  d * d 
(A) J x    .   .  (B) J x   .  * . 
2m  dx dx  2m  dx dx 
i   * d d *  i  * d d * 
(C) J x   .   .  (D) J x   .   . 
2m  dx dx  2m  dx dx 
3
Q.5 Three fermions of spin is adjusted in one dimensional harmonic oscillator of
2
frequency  what is energy of ground state configuration .
 3 5  7 
(A) (B) (C) (D)
2 2 2 2

Head office Branch office


fiziks, H.No. 23, G.F, Jia Sarai, Anand Institute of Mathematics,
Near IIT, Hauz Khas, New Delhi-16 28-B/6, Jia Sarai, Near IIT
Phone: 011-26865455/+91-9871145498 Hauz Khas, New Delhi-16
Website: www.physicsbyfiziks.com
Email: fiziks.physics@gmail.com 1
fiziks
Institute for NET/JRF, GATE, IIT-JAM, JEST, TIFR and GRE in PHYSICAL SCIENCES

Q6. Which of the following potential will give not give bound state for energy 0 E  .
k
(A) V ( x)  kx 2 (B) V ( r )   in three dimensional
r
(C) V ( x)  k x (D) V ( x)  V0 cosh x  V0

Q7. Consider Maxwell’s equations in vacuum in the absence of sources. If the solutions to

      
these equations are of the form  r , t   0 exp i k.r  t , B r , t   0 exp i k .r  t 
where  0 , 0 and k are constant vectors, then

 
(A) k .  0   0  0  
(B) k   0   0  0
(C) d dk  0 (D) k  c

Q8. Consider an electric field  existing in the interface between a conductor and free space.

Then the electric field  is


(A) external to the conductor and normal to the conductor’s surface
(B) internal to the conductor and normal to the conductor’s surface
(C) external to the conductor and tangential to the conductor’s surface
(D) both external and internal to the conductor and normal to the conductor’s surface

Q9. The xoy plane carries a uniform surface current of density   50 x̂ A/m. The magnetic
field (in Wb) at the point z = - 0.5 m is
(A) 10  10–6 Wb (B) 1  10 –6 Wb (C) π  10–6 Wb (D) 10π  10–6 Wb

Q10. A planet of mass m is moving in a circular orbit of radius ‘ a ’ under central force around
the sun of mass M, such that m << M. The kinetic energy of the planet is
GMm GMm
(A) (B) 
a a
GMm GMm
(C) (D) 
2a 2a

Head office Branch office


fiziks, H.No. 23, G.F, Jia Sarai, Anand Institute of Mathematics,
Near IIT, Hauz Khas, New Delhi-16 28-B/6, Jia Sarai, Near IIT
Phone: 011-26865455/+91-9871145498 Hauz Khas, New Delhi-16
Website: www.physicsbyfiziks.com
Email: fiziks.physics@gmail.com 2
fiziks
Institute for NET/JRF, GATE, IIT-JAM, JEST, TIFR and GRE in PHYSICAL SCIENCES

Q11. A bead of mass m is slide down smoothly on a wire of shape z = x2 in the (z, x) plane.
The Lagrangian of the bead is .
1 2 1 2
(A)
2
 
mx 1  x 2  mgx 2 (B)
2
 
mx 1  4 x 2  mgx 2

1 2 1 2
(C)
2
 
mx 1  x 2  mgx 2 (D)
2
 
mx 1  4 x 2  mgx 2

Q.12 The Lagrangian for a particle in cylindrical coordinates is given to be

1
L
2
 2

m  2   2 2  z 2 / 1  (  /    V (  )

where m is the mass of the particle and λ is a constant. Then which of the following is not
a conserved quantity?
(A) m (B) m 2
1

(C) mz / 1   /  2  (D)
2
 
m  2   2 2  z 2 / 1  (  /    V (  )
2

Q.13 Consider the Fermi-Dirac distribution function f E  at room temperature (300K) where

E refers to energy. If E F is the Fermi energy, which of the following is true?

(A) f E  is a step function


(B) f E F  has a value ½

(C) States with E  E F are filled completely

(D) f E  is large and tends to infinity as E decreases much below E F


2
Q.14 Find the average value of v x  iv y for ideal gas at temperature T

kT 2kT 3kT
(A) 0 (B) (C) (D)
m m 2m
Q15. ensemble of N three level systems with energies   1, 0, 1 is in thermal equilibrium at
temperature T. Let   kT  1 .
The partition function is given by
(A) (1+cosh2 ) (B) (1+cos2 ) (C) (1+2cosh2 ) (D) (1+cosh2)

Head office Branch office


fiziks, H.No. 23, G.F, Jia Sarai, Anand Institute of Mathematics,
Near IIT, Hauz Khas, New Delhi-16 28-B/6, Jia Sarai, Near IIT
Phone: 011-26865455/+91-9871145498 Hauz Khas, New Delhi-16
Website: www.physicsbyfiziks.com
Email: fiziks.physics@gmail.com 3
fiziks
Institute for NET/JRF, GATE, IIT-JAM, JEST, TIFR and GRE in PHYSICAL SCIENCES

Q16. For a cubic crystal the diffraction line from the planes with (h2+k2+l2) = 8 is observed at
an angle of diffraction 10.23 0. If only one line is observed at an angle lower than this, the
crystal structure is?
(A) Simple Cubic
(B) Body Centered Cubic
(C) Face Centered Cubic
(D) Diamond Cubic Lattice
Q17. Consider an ideal Fermi gas, with energy spectrum (p is momentum), contained in
a box of volume V in space of 3 dimension. Which of the following is correct PV relation
for this system?
(A) (B)

(C) (D)

Q18. Which of the following statement about superconductor is not correct?


(A) With appearance of Meisner effect, resistivity disappear
(B) When superconductivity is destroyed with magnetic field, the material will heat.
(C) Superconductivity occurs in material having high electrical resistivity
(D) Critical temperature (T c) decreases with isotopic mass

Q19. In a He-Ne laser, the laser transition takes place in:


(A) He only (B) Ne only
(C) Ne first, then in He (D) He first, then in Ne

Q20. All vibrations producing a change in the electric dipole moment of a molecule yield
(A) Raman spectra (B) Infrared spectra
(C) Ultra-violet spectra (D) X-ray spectra

Head office Branch office


fiziks, H.No. 23, G.F, Jia Sarai, Anand Institute of Mathematics,
Near IIT, Hauz Khas, New Delhi-16 28-B/6, Jia Sarai, Near IIT
Phone: 011-26865455/+91-9871145498 Hauz Khas, New Delhi-16
Website: www.physicsbyfiziks.com
Email: fiziks.physics@gmail.com 4
fiziks
Institute for NET/JRF, GATE, IIT-JAM, JEST, TIFR and GRE in PHYSICAL SCIENCES

Q21. Pick the WRONG statement.


(A) The nuclear force is independent of electric charge
 mc 
(B) The Yukawa potential is proportional to r 1 exp  r  , where r is the separation
 h 
between two nucleons.
(C) The range of nuclear force is of the order of 10-15 m – 10 -14m
(D) The nucleons interact among each other by the exchange of mesons.

Q.22 In the circuit shown in the figure the Thevenin voltage VTH and Thevenin resistance RTH
as seen by the load between point A and B are respectively

2 RTH
A A
 

12 A 4 3 VTH

 
B B

(A) 4V , 2 (B) 8V , 3 (C) 12V , 3 (D) 16V , 2

Q.23 How many maximum number of electrons can be adjusted in d orbital of hydrogen
atom …………………..

Q24. An amplifier has a voltage gain of 500 and input impedance 20 KΩ, without any
feedback. Now a negative feedback with β = 0.1 is applied. Its gain and input impedance
(K ohms) with feedback will respectively be …………………..

Q25. If the nuclear radius of 27Al is 3.6 Fermi, the approximate nuclear radius of 64Cu in Fermi
is …………………..

Head office Branch office


fiziks, H.No. 23, G.F, Jia Sarai, Anand Institute of Mathematics,
Near IIT, Hauz Khas, New Delhi-16 28-B/6, Jia Sarai, Near IIT
Phone: 011-26865455/+91-9871145498 Hauz Khas, New Delhi-16
Website: www.physicsbyfiziks.com
Email: fiziks.physics@gmail.com 5
fiziks
Institute for NET/JRF, GATE, IIT-JAM, JEST, TIFR and GRE in PHYSICAL SCIENCES

Q. 26 – Q. 55 carry two marks each.

d2y dy
Q.26 The solution of the differential equation for 2
 2  101 y  10.4e x , subject to the
dx dx
dy
initial conditions y  0   1.1 and  0.9 , is
dt x0

(A) e  x ( A cos10 x  B sin10 x ) (B) e  x ( A cos x  B sin x )

(C) e  x ( A cos10 x  B sin10 x ) (D) e x ( A cos10 x  B sin10 x)


Q.27 A binary star system consists of two stars S1 and S2, with masses m and 2m respectively
separated by a distance r. If both S1 and S2 individually follow circular orbits around the
centre of mass with instantaneous speeds v1 and v2 respectively, the speeds ratio v1/v2 is
…………………….
Q.28 A particle of unit mass moves along the x-axis under the influence of a potential,
2
V  x   x  x  4  . The particle is found to be in stable equilibrium at the point . The time

period of oscillation of the particle is


 3
(A) (B)  (C) (D) 2
2 2
Q.29 A parallel plate capacitor has circular plates of radius R. It is being charged by a current I.

Then the magnetic induction B at a point between the plates at a distance R/2 from the
axis of the capacitor is:
0 I ˆ 0 I ˆ 0 I ˆ 0 I ˆ
(a) B   (b) B   (c) B   (d) B  
2R 4R 6R 8R
1 0
Q30. Consider a system in the unperturbed state described by the Hamiltonian, H0   .
0 1 

0 
The system is subjected to a perturbation of the form H '   , when δ<< 1. The
 0 
energy eigenvalues of the perturbed system using the first order perturbation
approximation are
(A) 1 and (1 + 2δ) (B) (1 + δ) and (1 - δ)
Head office Branch office
fiziks, H.No. 23, G.F, Jia Sarai, Anand Institute of Mathematics,
Near IIT, Hauz Khas, New Delhi-16 28-B/6, Jia Sarai, Near IIT
Phone: 011-26865455/+91-9871145498 Hauz Khas, New Delhi-16
Website: www.physicsbyfiziks.com
Email: fiziks.physics@gmail.com 6
fiziks
Institute for NET/JRF, GATE, IIT-JAM, JEST, TIFR and GRE in PHYSICAL SCIENCES

(C) (1+2 δ) and (1 - δ) (D) (1+ δ) and (1 - 2δ)


Q31. Consider a two level quantum system with energies  1  0 and  1   . The specific heat
of the system is given by
 e  / k B T 2 e   / k BT
(A) (B)
k B T 1  e  / k BT 2 k B T 2 1  e  / kBT 

 2 e  / k B T 2 e   / k BT
(C)  (D)
1  e  / k B T 2
 k B T 2 1  e  / kBT 2

3 a
Q32. The internal energy of n moles of a gas is given by E  nRT  , where V is the
2 V
volume of the gas at temperature T and a is a positive constant. One mole of the gas in
state (T1, V1) is allowed to expand adiabatically into vacuum to a final state (T2, V2). The
temperature T2 is

a 1 1 2a 1 1
(A) 1     (B) 1    
R  V2 V1  3 R  V2 V1 

2a 1 1 1a 1 1
(C) 1     (D) 1    
3 R  V2 V1  3 R  V2 V1 

Q33. Consider the following circuit in which the current gain βdc of the transistor is 100
and VBE  0.7V . Which one of the following correctly represents the collector current IC
and collector-emitter voltage VCE?  15 V
(A) 12mA, 0.5V
(B) 14mA, 1V 100 k 900 

(C) 16mA, 2V
(D) 18mA, 3V

100 

Head office Branch office


fiziks, H.No. 23, G.F, Jia Sarai, Anand Institute of Mathematics,
Near IIT, Hauz Khas, New Delhi-16 28-B/6, Jia Sarai, Near IIT
Phone: 011-26865455/+91-9871145498 Hauz Khas, New Delhi-16
Website: www.physicsbyfiziks.com
Email: fiziks.physics@gmail.com 7
fiziks
Institute for NET/JRF, GATE, IIT-JAM, JEST, TIFR and GRE in PHYSICAL SCIENCES

Q.34 For the given circuit the frequency above which the gain will decrease by 20 dB per
decade is 10 k
Vin  
(A) 12 kHz
Vout 
(B) 16 kHz 1000pF 
(C) 18 kHz 1 k

(D) 20 kHz
2 k

Q.35 In the circuit shown, the ports Q1 and Q2 are in the state Q1 = 1, Q2 = 0. The circuit is
now subjected to three complete clock pulses. The state of these ports now becomes
Q1 Q2
(A) Q2 = 1, Q1 = 0
(B) Q2 = 0, Q1 = 1
1 J Q 1 J Q
(C) Q2 = 1, Q1 = 1 CLK CLK
(D) Q2 = 0, Q1 = 0 1 K Q 1 K Q

Q.36 There are only three bound states for a particle of mass m in a one-dimensional potential
well of the form shown in the figure. The depth V0 of the
potential satisfies V
a/2 a/2
2 2  2 9 2  2  2 2 2 2  2 x
(a)  V0  (b)  V0 
ma 2 2ma 2 ma 2 ma 2
2 2  2 8 2  2 2 2  2 50 2  2  V0
(c)  V0  (d)  V0 
ma 2 ma 2 ma 2 ma 2

Head office Branch office


fiziks, H.No. 23, G.F, Jia Sarai, Anand Institute of Mathematics,
Near IIT, Hauz Khas, New Delhi-16 28-B/6, Jia Sarai, Near IIT
Phone: 011-26865455/+91-9871145498 Hauz Khas, New Delhi-16
Website: www.physicsbyfiziks.com
Email: fiziks.physics@gmail.com 8
fiziks
Institute for NET/JRF, GATE, IIT-JAM, JEST, TIFR and GRE in PHYSICAL SCIENCES

3 2 3 2
Q.37 The equation of a surface of revolution is z   x  y
2 2

 2 
The unit normal to the surface at the point A ,0,1 is
 3 

3ˆ 2 ˆ 3ˆ 2 ˆ
(A) i k (B) i k
5 10 5 10

3ˆ 2 ˆ 3ˆ 2 ˆ
(C) i k (D) i k
5 5 10 10

0 for x  3,
Q.38 If f  x    then the Laplace transform of f(x) is
x  3 for x  3
(A) s-2e3s (B) s2e-3s (C) s-2 (D) s-2e-3s

Q.39 A hypothetical semiconductor has a conduction band that can be described by


Ecb = E1 – E2 cos (Ka)
and valence band that can be described by :
Evb = E3 – E4 sin2 (Ka/2)

Where E3 < (E1 – E2) and 


 
K
a a
The band gap of the material is
(A) E1-E2-E3+E4 (B) E1+E2- E3
(C) E1-E2+E4 (D) 2(E2-E4)
Q.40 The electron concentration in the conduction band of Ge at 77K is 10 12 cm-3. At this
temperature the electron and hole nobilities are equal, µ = 0.5 x 104 cm2 /V-sec. The
dielectric constant of Ge is 16.
If 100 volts is applied across a 1 cm cube of crystal, the current (mA) flowing in the
sample is …………………..

Head office Branch office


fiziks, H.No. 23, G.F, Jia Sarai, Anand Institute of Mathematics,
Near IIT, Hauz Khas, New Delhi-16 28-B/6, Jia Sarai, Near IIT
Phone: 011-26865455/+91-9871145498 Hauz Khas, New Delhi-16
Website: www.physicsbyfiziks.com
Email: fiziks.physics@gmail.com 9
fiziks
Institute for NET/JRF, GATE, IIT-JAM, JEST, TIFR and GRE in PHYSICAL SCIENCES

Q.41. Assuming a dispersion relation under long wavelength approximation for elastic
wave in one dimensional lattice of ferromagnetic system. The lattice specific heat at low
temperature varies as
(A) (B) (C) (D)

Q.42 According to the single particles nuclear shell model, the spin-parity of the ground state
of 178 O is
   
1 3 3 5
(A) (B) (C) (D)
2 2 2 2
Q.43 The disintegration energy is defined to be the difference in the rest energy between the
initial and final states. Consider the following process: 240
94 Pu 
236
92 U  42 He
The emitted α particle has a kinetic energy 5.17 MeV. The value of the disintegration
energy in MeV is ……………..
Q44. Which of the following fusion reaction give more energy? The nuclear mass of the different
nuclei is as follows
M( ) = 1.00783amu, M( ) = 2.01410amu, M( ) = 3.01605amu,

M( ) = 3.01603amu, M( ) = 4.02603amu, M( ) = 6.01512amu and


M(e) = 0.00055amu

(A) +
(B)
(C)
(D)
Q.45 If λB and λp is the wavelength of the series limit of Bracket and Pfund series respectively.
The ratio of λB and λp will be
16 25 9 9
(A) (B) (C) (D)
25 36 25 16

Head office Branch office


fiziks, H.No. 23, G.F, Jia Sarai, Anand Institute of Mathematics,
Near IIT, Hauz Khas, New Delhi-16 28-B/6, Jia Sarai, Near IIT
Phone: 011-26865455/+91-9871145498 Hauz Khas, New Delhi-16
Website: www.physicsbyfiziks.com
Email: fiziks.physics@gmail.com 10
fiziks
Institute for NET/JRF, GATE, IIT-JAM, JEST, TIFR and GRE in PHYSICAL SCIENCES

Q.46 A pure rotational Raman spectrum of a linear diatomic molecule is recorded using
electromagnetic radiation of frequency ve. The frequency of two consecutive stokes
(A) ve – 10 B, ve – 14 B (B) ve – 2 B, ve – 4 B
(C) ve + 10 B, ve + 14 B (D) ve + 2 B, ve + 4 B

Q.47 The first excited state of nucleus has spin parity 2+ and decays to 1+ ground state by
emitting gamma rays. The possible multipole radiation for the above transition are?
(A) E1, E2, E3 (B) M1, M2, M3
(C) M1, E2, M3 (D) E1, M2, E3.

Common Data Questions


Common Data for Questions 48 and 49
A hydrogen beam is prepared in the state

 1  Et   2  Et  3  Et 
 r , t   exp  i 1  1  r   exp  i 2   2  r   exp  i 3   3  r 
14    7    14   
where E1, E2, E3 are the energies of the ground state and the first excited state of the
  
hydrogen atom and 1 r , 2 r , 3 r  are their normalized wave functions respectively.
The beam is incident on a detector which measures their energy. Let E0 be the ionization
energy of the hydrogen atom.
E0
Q48. what will be probability such that energy is  E0 and  respectively .
9
1 3 1 9 1 1 3 3
(A) , (B) , (C) , (D) ,
14 14 14 14 14 14 14 14

Q49. The average energy measured by the detector is given by


3 3
(A) – E0 (B)  E0 (C) E0 (D) E0
14 14

Head office Branch office


fiziks, H.No. 23, G.F, Jia Sarai, Anand Institute of Mathematics,
Near IIT, Hauz Khas, New Delhi-16 28-B/6, Jia Sarai, Near IIT
Phone: 011-26865455/+91-9871145498 Hauz Khas, New Delhi-16
Website: www.physicsbyfiziks.com
Email: fiziks.physics@gmail.com 11
fiziks
Institute for NET/JRF, GATE, IIT-JAM, JEST, TIFR and GRE in PHYSICAL SCIENCES

Common Data for Questions 50 and 51


Consider a hetero nuclear diatomic molecule with moment of inertia I .
Q50. If N no of such molecule kept at equilibrium temperature T with j total angular
orbital quantum no then write the partition fuction at low temperature .
2 2 2
a) Z  exp(  ) b) Z  1  exp( ) c) Z  1  3exp( )
IkT IkT IkT
2 2
Z  1  3exp( )
IkT
Q51. what will average energy per molecule at higher temperature T
3kT
(A) kT (B) 2kT (C) (D)3 kT
2
Linked Answer Questions
Statement for Linked Answer Questions 52 and 53:
Three particles of equal mass m are connected by two identical mass less springs of
stiffness constant k as shown in the figure:
k k
m m m
Q.52 If x1 , x 2 and x 3 denote the horizontal displacements of the masses from their
respective equilibrium positions, the potential energy of the system is
1 1
(A)
2

k x12  x22  x32  (B)
2

k x12  x 22  x 32  x 2  x1  x3 
1 1
(C)
2

k x12  2 x 22  x32  2 x 2 x1  x3   (D)
2

k x12  2 x 22  x32  2 x 2  x1  x 3 
Q.53 The normal mode of frequency is given by

k k k 2k k 3k k k
(A) , (B) , (C) , (D) ,2 .
m m m m m m m m

Head office Branch office


fiziks, H.No. 23, G.F, Jia Sarai, Anand Institute of Mathematics,
Near IIT, Hauz Khas, New Delhi-16 28-B/6, Jia Sarai, Near IIT
Phone: 011-26865455/+91-9871145498 Hauz Khas, New Delhi-16
Website: www.physicsbyfiziks.com
Email: fiziks.physics@gmail.com 12
fiziks
Institute for NET/JRF, GATE, IIT-JAM, JEST, TIFR and GRE in PHYSICAL SCIENCES

Statement for Linked Answer Questions 54 and 55:


A long solenoid, of radius a, is driven by alternating current, so that the field inside is

sinusoidal: B  t   B0 cos t  zˆ . A circular loop of wire, of radius a and resistance R,
2
is placed inside the solenoid, and coaxial with it.

Q54. The Magnetic flux through the loop is,


1 2
(A)  a B0 sin  t  (B)  a 2 B0 sin t 
4
1 2
(C)  a 2 B0 cos  t  (D)  a B0 cos  t 
4

Q55. Then the amplitude of current induced in the loop is


 a 2 B0  a 2 B0  a 2 B0  a 2 B0
(A) (B) (C) (D)
2R 3R 4R 5R

Head office Branch office


fiziks, H.No. 23, G.F, Jia Sarai, Anand Institute of Mathematics,
Near IIT, Hauz Khas, New Delhi-16 28-B/6, Jia Sarai, Near IIT
Phone: 011-26865455/+91-9871145498 Hauz Khas, New Delhi-16
Website: www.physicsbyfiziks.com
Email: fiziks.physics@gmail.com 13
fiziks
Institute for NET/JRF, GATE, IIT-JAM, JEST, TIFR and GRE in PHYSICAL SCIENCES

General Aptitude (GA) Questions


Q.56 – Q.60 carry one mark each

Q.56 Choose the grammatically INCORRECT sentence:


(a) They gave us the money back less the service charges of Three Hundred rupees.
(b) This country’s expenditure is not less than that of Bangladesh.
(c) The committee initially asked for a funding of Fifty Lakh rupees, but later settled for a
lesser sum.
(d) This country’s expenditure on educational reforms is very less.

Q.57. Which one of the following options is the closest in meaning to the word given below?
Mitigate
(a) Diminish (b) Divulge (c) Dedicate (d) Denote

Q.58 Choose the most appropriate alternative from the options given below to complete the
following sentence:
Despite several ––––––––– the mission succeeded in its attempt to resolve the
conflict.
(a) attempts (b) setbacks (c) meetings (d) delegations

Q.59 T he cost function for a product in a firm is given by 5q2, where q is the amount of
production. The firm can sell the product at a market price of Rs50 per unit. The number
of units to be produced by the firm such that the profit is maximized is
(a) 5 (b) 10 (c) 15 (d) 25

Q.60 Choose the most appropriate alternative from the options given below to complete the
following sentence:
Suresh’s dog is the one ––––––––– was hurt in the stampede.
(a) that (b) which (c) who (d) whom

Head office Branch office


fiziks, H.No. 23, G.F, Jia Sarai, Anand Institute of Mathematics,
Near IIT, Hauz Khas, New Delhi-16 28-B/6, Jia Sarai, Near IIT
Phone: 011-26865455/+91-9871145498 Hauz Khas, New Delhi-16
Website: www.physicsbyfiziks.com
Email: fiziks.physics@gmail.com 14
fiziks
Institute for NET/JRF, GATE, IIT-JAM, JEST, TIFR and GRE in PHYSICAL SCIENCES

Q.61 – Q.65 carry two marks each.

Q.61 Which of the following assertions are CORRECT?


P: Adding 7 to each entry in a list adds 7 to the mean of the list
Q: Adding 7 to each entry in a list adds 7 to the standard deviation of the list
R: Doubling each entry in a list doubles the mean of the list
S: Doubling each entry in a list leaves the standard deviation of the list unchanged
(a) P, Q (b) Q, R (c) P, R (d) R, S

Q.62 An automobile plant contracted to buy shock absorbers from two suppliers X and Y. X
supplies
60% and Y supplies 40% of the shock absorbers. All shock absorbers are subjected to a
quality test. The ones that pass the quality test are considered reliable. Of X’s shock
absorbers, 96% are reliable. Of Y’s shock absorbers, 72% are reliable.
The probability that a randomly chosen shock absorber, which is found to be reliable, is
made by Y is
(a) 0.288 (b) 0.334 (c) 0.667 (d) 0.720

Q.63 A political party orders an arch for the entrance to the ground in which the annual
convention is
being held. The profile of the arch follows the equation y = 2x – 0.1x2 where y is the
height of the
arch in meters. The maximum possible height of the arch is
(a) 8 meters (b) 10 meters (c) 12 meters (d) 14 meters

Head office Branch office


fiziks, H.No. 23, G.F, Jia Sarai, Anand Institute of Mathematics,
Near IIT, Hauz Khas, New Delhi-16 28-B/6, Jia Sarai, Near IIT
Phone: 011-26865455/+91-9871145498 Hauz Khas, New Delhi-16
Website: www.physicsbyfiziks.com
Email: fiziks.physics@gmail.com 15
fiziks
Institute for NET/JRF, GATE, IIT-JAM, JEST, TIFR and GRE in PHYSICAL SCIENCES

Q.64 Wanted Temporary, Part-time persons for the post of Field Interviewer to conduct
personal interviews to collect and collate economic data. Requirements: High School-
pass, must be available for Day, Evening and Saturday work. Transportation paid,
expenses reimbursed.
Which one of the following is the best inference from the above advertisement?
(a) Gender-discriminatory
(b) Xenophobic
(c) Not designed to make the post attractive
(d) Not gender-discriminatory

Q.65 Given the sequence of terms, AD CG FK JP, the next term is


(a) OV (b) OW (c) PV (d) PW

Head office Branch office


fiziks, H.No. 23, G.F, Jia Sarai, Anand Institute of Mathematics,
Near IIT, Hauz Khas, New Delhi-16 28-B/6, Jia Sarai, Near IIT
Phone: 011-26865455/+91-9871145498 Hauz Khas, New Delhi-16
Website: www.physicsbyfiziks.com
Email: fiziks.physics@gmail.com 16
CSIR-UGC National Eligibility Test (NET) for Junior Research Fellowship
and Lecturer-ship

PHYSICAL SCIENCES

PART ‘A’ CORE


I. Mathematical Methods of Physics

Dimensional analysis. Vector algebra and vector calculus. Linear algebra, matrices, Cayley-Hamilton
Theorem. Eigenvalues and eigenvectors. Linear ordinary differential equations of first & second order,
Special functions (Hermite, Bessel, Laguerre and Legendre functions). Fourier series, Fourier and Laplace
transforms. Elements of complex analysis, analytic functions; Taylor & Laurent series; poles, residues
and evaluation of integrals. Elementary probability theory, random variables, binomial, Poisson and
normal distributions. Central limit theorem.

II. Classical Mechanics

Newton’s laws. Dynamical systems, Phase space dynamics, stability analysis. Central force motions.
Two body Collisions - scattering in laboratory and Centre of mass frames. Rigid body dynamics-
moment of inertia tensor. Non-inertial frames and pseudoforces. Variational principle. Generalized
coordinates. Lagrangian and Hamiltonian formalism and equations of motion. Conservation laws and
cyclic coordinates. Periodic motion: small oscillations, normal modes. Special theory of relativity-
Lorentz transformations, relativistic kinematics and mass–energy equivalence.

III. Electromagnetic Theory

Electrostatics: Gauss’s law and its applications, Laplace and Poisson equations, boundary value
problems. Magnetostatics: Biot-Savart law, Ampere's theorem. Electromagnetic induction. Maxwell's
equations in free space and linear isotropic media; boundary conditions on the fields at interfaces. Scalar
and vector potentials, gauge invariance. Electromagnetic waves in free space. Dielectrics and conductors.
Reflection and refraction, polarization, Fresnel’s law, interference, coherence, and diffraction. Dynamics
of charged particles in static and uniform electromagnetic fields.

IV. Quantum Mechanics

Wave-particle duality. Schrödinger equation (time-dependent and time-independent). Eigenvalue


problems (particle in a box, harmonic oscillator, etc.). Tunneling through a barrier. Wave-function in
coordinate and momentum representations. Commutators and Heisenberg uncertainty principle. Dirac
notation for state vectors. Motion in a central potential: orbital angular momentum, angular momentum
algebra, spin, addition of angular momenta; Hydrogen atom. Stern-Gerlach experiment. Time-
independent perturbation theory and applications. Variational method. Time dependent perturbation
theory and Fermi's golden rule, selection rules. Identical particles, Pauli exclusion principle, spin-statistics
connection.

V. Thermodynamic and Statistical Physics

Laws of thermodynamics and their consequences. Thermodynamic potentials, Maxwell relations,


chemical potential, phase equilibria. Phase space, micro- and macro-states. Micro-canonical, canonical
and grand-canonical ensembles and partition functions. Free energy and its connection with
thermodynamic quantities. Classical and quantum statistics. Ideal Bose and Fermi gases. Principle of
detailed balance. Blackbody radiation and Planck's distribution law.

VI. Electronics and Experimental Methods

Semiconductor devices (diodes, junctions, transistors, field effect devices, homo- and hetero-junction
devices), device structure, device characteristics, frequency dependence and applications. Opto-electronic
devices (solar cells, photo-detectors, LEDs). Operational amplifiers and their applications. Digital
techniques and applications (registers, counters, comparators and similar circuits). A/D and D/A
converters. Microprocessor and microcontroller basics.

Data interpretation and analysis. Precision and accuracy. Error analysis, propagation of errors. Least
squares fitting,

PART ‘B’ ADVANCED


I. Mathematical Methods of Physics

Green’s function. Partial differential equations (Laplace, wave and heat equations in two and three
dimensions). Elements of computational techniques: root of functions, interpolation, extrapolation,
integration by trapezoid and Simpson’s rule, Solution of first order differential equation using Runge-
Kutta method. Finite difference methods. Tensors. Introductory group theory: SU(2), O(3).

II. Classical Mechanics

Dynamical systems, Phase space dynamics, stability analysis. Poisson brackets and canonical
transformations. Symmetry, invariance and Noether’s theorem. Hamilton-Jacobi theory.

III. Electromagnetic Theory

Dispersion relations in plasma. Lorentz invariance of Maxwell’s equation. Transmission lines and wave
guides. Radiation- from moving charges and dipoles and retarded potentials.

IV. Quantum Mechanics

Spin-orbit coupling, fine structure. WKB approximation. Elementary theory of scattering: phase shifts,
partial waves, Born approximation. Relativistic quantum mechanics: Klein-Gordon and Dirac equations.
Semi-classical theory of radiation.

V. Thermodynamic and Statistical Physics

First- and second-order phase transitions. Diamagnetism, paramagnetism, and ferromagnetism. Ising
model. Bose-Einstein condensation. Diffusion equation. Random walk and Brownian motion.
Introduction to nonequilibrium processes.

VI. Electronics and Experimental Methods

Linear and nonlinear curve fitting, chi-square test. Transducers (temperature, pressure/vacuum, magnetic
fields, vibration, optical, and particle detectors). Measurement and control. Signal conditioning and
recovery. Impedance matching, amplification (Op-amp based, instrumentation amp, feedback), filtering
and noise reduction, shielding and grounding. Fourier transforms, lock-in detector, box-car integrator,
modulation techniques.

High frequency devices (including generators and detectors).

VII. Atomic & Molecular Physics

Quantum states of an electron in an atom. Electron spin. Spectrum of helium and alkali atom. Relativistic
corrections for energy levels of hydrogen atom, hyperfine structure and isotopic shift, width of spectrum
lines, LS & JJ couplings. Zeeman, Paschen-Bach & Stark effects. Electron spin resonance. Nuclear
magnetic resonance, chemical shift. Frank-Condon principle. Born-Oppenheimer approximation.
Electronic, rotational, vibrational and Raman spectra of diatomic molecules, selection rules. Lasers:
spontaneous and stimulated emission, Einstein A & B coefficients. Optical pumping, population
inversion, rate equation. Modes of resonators and coherence length.

VIII. Condensed Matter Physics

Bravais lattices. Reciprocal lattice. Diffraction and the structure factor. Bonding of solids. Elastic
properties, phonons, lattice specific heat. Free electron theory and electronic specific heat. Response and
relaxation phenomena. Drude model of electrical and thermal conductivity. Hall effect and
thermoelectric power. Electron motion in a periodic potential, band theory of solids: metals, insulators
and semiconductors. Superconductivity: type-I and type-II superconductors. Josephson junctions.
Superfluidity. Defects and dislocations. Ordered phases of matter: translational and orientational order,
kinds of liquid crystalline order. Quasi crystals.

IX. Nuclear and Particle Physics

Basic nuclear properties: size, shape and charge distribution, spin and parity. Binding energy, semi-
empirical mass formula, liquid drop model. Nature of the nuclear force, form of nucleon-nucleon
potential, charge-independence and charge-symmetry of nuclear forces. Deuteron problem. Evidence of
shell structure, single-particle shell model, its validity and limitations. Rotational spectra. Elementary
ideas of alpha, beta and gamma decays and their selection rules. Fission and fusion. Nuclear reactions,
reaction mechanism, compound nuclei and direct reactions.

Classification of fundamental forces. Elementary particles and their quantum numbers (charge, spin,
parity, isospin, strangeness, etc.). Gellmann-Nishijima formula. Quark model, baryons and mesons. C, P,
and T invariance. Application of symmetry arguments to particle reactions. Parity non-conservation in
weak interaction. Relativistic kinematics.
fiziks
Institute for NET/JRF, GATE, IIT‐JAM, M.Sc. Entrance, JEST, TIFR and GRE in Physics 
 
MATHEMATICAL PHYSICS SOLUTIONS
NET/JRF (JUNE-2011)
Q1. The value of the integral  dz z 2 e z , where C is an open contour in the complex z -plane as
C
lm z
shown in the figure below, is:
5 5
(a) e (b) e  0,1
e e
5 5 C
(c) e (d)  e
e e
Re z
Ans: (c)  1,0  1,0
Solution: If we complete the contour, then by Cauchy integral theorem
1 1
e   dzz e  0   dzz e    dzz 2 e z   z 2 e z  2 ze z  2e z  1 
1 5
 dzz e
2 z 2 z 2 z

1 C C 1
e
Q2. Which of the following matrices is an element of the group SU 2  ?
1 i 1 
 
 1 1
(a)   (b)  3 3
 0 1  1 1 i 
 
 3 3
 1 3
 
2  i i 
(c)   (d)  2 2 
 3 1 i  3 1 
 
 2 2 
Ans: (b)
   
Solution: SU 2  is a group defined as following: SU 2   
2 2
 :  ,   C ;     1
    
1 i 1 1 i 1
clearly (b) hold the property of SU 2  .   ,  and   ,  .
3 3 3 3
Note: SU 2  has wide applications in electroweak interaction covered in standard model
of particle physics.
  
Q3. Let a and b be two distinct three dimensional vectors. Then the component of b that is

perpendicular to a is given by
  
(a)
a ba  
(b)
  
b  a b 
(c)
 
a b b  (d)
 
b a a    
a2 b2 b2 a2
Ans: (a)
H.No. 40‐D, Ground Floor, Jia Sarai, Near IIT, Hauz Khas, New Delhi‐110016 
Phone: 011‐26865455/+91‐9871145498
Website: www.physicsbyfiziks.com  | Email: fiziks.physics@gmail.com  

fiziks
Institute for NET/JRF, GATE, IIT‐JAM, M.Sc. Entrance, JEST, TIFR and GRE in Physics 
 
 
Solution: a  b  ab sin  nˆ where n̂ is perpendicular to plane containing
 
a and b and pointing upwards.
b b sin  kˆ
  
  
a  a  b  ab sin  a  nˆ    a 2 b sin  kˆ

     
ˆ
b sin  k 
 a  a  b   ˆ
 b sin  k 
a  ba
.
  a

a2 a2
Q4. Let p n  x  (where n  0,1, 2, ...... ) be a polynomial of degree n with real coefficients,
4
defined in the interval 2  n  4 . If  pn  x  pm  x dx   nm , then
2

1 3 1
(a) p 0 x   and p1  x    3  x  (b) p0  x   and p1  x   3 3  x 
2 2 2

1 3 1 3
(c) p0  x   and p1  x   3  x  (d) p 0  x   and p1  x   3  x 
2 2 2 2
Ans: (d)
Solution: For n not equal to m kroneker delta become zero. One positive and one negative term
1
can make integral zero. So answer may be (c) or (d). Now take n  m  0 so p0  x  
2
and then integrate. (d) is correct option because it satisfies the equation Check by
integration and by orthogonal property of Legendre polynomial also.
Q5. Which of the following is an analytic function of the complex variable z  x  iy in the

domain z  2 ?

(a) 3  x  iy  (b) 1  x  iy  7  x  iy 
7 4 3

(c) 1  x  iy   7  x  iy  (d) x  iy  1
4 3 1/ 2

Ans: (b)
Solution: Put z  x  iy . If z  x  iy appears in any of the expressions then that expression is
1
non-analytic. For option (d) we have a branch point singularity as the power is which
2
is fractional. Hence only option (b) is analytic.

H.No. 40‐D, Ground Floor, Jia Sarai, Near IIT, Hauz Khas, New Delhi‐110016 
Phone: 011‐26865455/+91‐9871145498
Website: www.physicsbyfiziks.com  | Email: fiziks.physics@gmail.com  

fiziks
Institute for NET/JRF, GATE, IIT‐JAM, M.Sc. Entrance, JEST, TIFR and GRE in Physics 
 
 1 1 1
Q6. Consider the matrix M  1 1 1
 1 1 1
 
A. The eigenvalues of M are
(a) 0, 1, 2 (b) 0, 0, 3 (c) 1, 1, 1 (d) – 1, 1, 3
Ans: (b)
1   1 1 
Solution: For eigen values  1 1  1   0
 1 1 1   

1   1   2  1  1    1  11  1     0
1   1  2  2  1      0  2  2  3  22  2  0
3  32  0  2   3  0    0, 0, 3
For any n  n matrix having all elements unity eigenvalues are 0, 0, 0,..., n .
B. The exponential of M simplifies to (I is the 3  3 identity matrix)
 e3  1  M2
(a) e  I  
M
 M (b) e  I  M 
M

 3  2!

(c) e M  I  33 M (d) e M  e  1M


Ans: (a)
Solution: For e M , let us try to diagonalize matrix M using similarity transformation.

 2 1 1   x1  0

For   3 ,  1  2 1   x 2   0
 1 1  2  x3  0

  2 x1  x2  x 3  0 , x1  2 x 2  x3  0 , x1  x 2  2 x3  0

 3x 2  3x3  0 or x2  x3  x1  x 2  x3  k .

1
1 
Eigen vector is 1 , where k  1 .
3 
1

For   0 ,

H.No. 40‐D, Ground Floor, Jia Sarai, Near IIT, Hauz Khas, New Delhi‐110016 
Phone: 011‐26865455/+91‐9871145498
Website: www.physicsbyfiziks.com  | Email: fiziks.physics@gmail.com  

fiziks
Institute for NET/JRF, GATE, IIT‐JAM, M.Sc. Entrance, JEST, TIFR and GRE in Physics 
 
1 1 1  x1  0
1 1 1  x  0  x  x  x  0
  2   1 2 3

1 1 1  x3  0

 k1  1
  1  
Let x1  k1 , x2  k2 and x3   k1  k2  . Eigen vector is  k2    1 where k1  k 2  1 .
 k 1  k 2   2
   1 

1
1  
Let x1  k1 , x 2  k 2 and x3  k1  k 2  . Other Eigen vector 0 where k1  1, k 2  1 .
2 
 1

 0 1 1 1  2 1 
S   1 0 1  S  2  1  1  D  S 1 MS , M  SDS 1 .
  1

 1  1 1 1  1 1 

1 0 0 
eM  Se D S 1  e D  0 1 0   e M  1 
e3  1 M  
3
0 0 e 3 

NET/JRF (DEC-2011)
Q7. An unbiased dice is thrown three times successively. The probability that the numbers of
dots on the uppermost surface add up to 16 is
1 1 1 1
(a) (b) (c) (d)
16 36 108 216
Ans: (b)
Solution: We can get sum of dice as 16 in total six ways i.e. three ways (6, 5, 5) and three ways
(6, 6, 4).
Total number of ways for 3 dice having six faces  6  6  6
6 1
 
6  6  6 36

Q8. The generating function F x, t    Pn  x t n for the Legendre polynomials Pn  x 
n 0


is F x, t   1  2 xt  t 2 
1
2
. The value of P3  1 is
(a) 5 / 2 (b) 3 / 2 (c)  1 (d)  1
H.No. 40‐D, Ground Floor, Jia Sarai, Near IIT, Hauz Khas, New Delhi‐110016 
Phone: 011‐26865455/+91‐9871145498
Website: www.physicsbyfiziks.com  | Email: fiziks.physics@gmail.com  

fiziks
Institute for NET/JRF, GATE, IIT‐JAM, M.Sc. Entrance, JEST, TIFR and GRE in Physics 
 
Ans: (d)

Solution: P3 
1
2
  1
2
 1

5 x 3  3x  P3  1  5 1  3 1   5  3  1
3

2
Q9. The equation of the plane that is tangent to the surface xyz  8 at the point 1, 2, 4  is
(a) x  2 y  4 z  12 (b) 4 x  2 y  z  12
(c) x  4 y  2  0 (d) x  y  z  7
Ans: (b)
Solution: To get a normal at the surface, lets take the gradient

 xyx   yziˆ  zxˆj  kˆxy  8iˆ  4 ˆj  2kˆ

We want a plane perpendicular to this so: r  r0    8iˆ64 4 16ˆj 2k4ˆ  0 .


x 1iˆ   y  2 ˆj  z  4kˆ 8iˆ  4 ˆj  2kˆ  0  4 x  2 y  z  12 .
Q10.    
A 3  3 matrix M has Tr M   6, Tr M 2  26 and Tr M 3  90 . Which of the following
can be a possible set of eigenvalues of M ?
(a) 1,1, 4 (b)  1, 0, 7 (c)  1, 3, 4 (d) 2, 2, 2
Ans: (c)
  2 2
 
Solution: Tr M 2   1  3  4 also Tr M 3   1  3  4  90 .
2 3 3 3

Q11. Let x1 t  and x2 t  be two linearly independent solutions of the differential equation

d 2x dx t  dx t 
 2  f t x  0 and let wt   x1 t  2  x2 t  1 . If w0   1, then w1 is
dx
2
dt dt dt dt
given by
(a) 1 (b) e 2 (c) 1 / e (d) 1 / e 2
Ans: (d)
Solution: W t  is Wronskian of D.E.

W  e   e 2t  W 1  e 2 since P  2 .


 Pdt

H.No. 40‐D, Ground Floor, Jia Sarai, Near IIT, Hauz Khas, New Delhi‐110016 
Phone: 011‐26865455/+91‐9871145498
Website: www.physicsbyfiziks.com  | Email: fiziks.physics@gmail.com  

fiziks
Institute for NET/JRF, GATE, IIT‐JAM, M.Sc. Entrance, JEST, TIFR and GRE in Physics 
 
1 for 2n  x  2n  1
Q12. The graph of the function f  x   
0 for 2n  1  x  2n  2
~
where n  0,1, 2,...... is shown below. Its Laplace transform f s  is
f x 
1  es 1  es
(a) (b)
s s 1
1 1
(c) (d) x
s 1  e  s  s 1  e  s  0 1 2 3 4 5

Ans: (c)
 1 2 3
Solution: L f  x    e  sx
f  x  dx   e  sx
 1dx   e  sx
 0dx   e  sx  1dx  ......
0 0 1 2

1 3
 e  sx   e  sx 
   0     ...... 
1 s
e 1 
1 3 s

e  e  2 s  ......   
  s 0   s 2 s s


1
s
 1
 
 1  e  s  e 2 s  e 3 s  ........  1  e  s  e  2 s  e 3s  ....
s

a 1 1 
Since S   where r  e  s and a  1  S    .
1 r s  1  e  s 

Q13. The first few terms in the Taylor series expansion of the function f  x   sin x around

x are:
4

1     1   1   
2 3

(a)  
1  x     x     x   .....
2   4  2!  4  3!  4 

1   
2 3
 1  1 
(b) 1
  x     x     x   .....
2   4  2!  4  3!  4 

   1   
3

(c)  x     x   .....
 4  3!  4 

1  x 2 x3 
(d) 1  x   .....
2  2! 3! 

Ans: (b)
Solution: f  x   sin x

H.No. 40‐D, Ground Floor, Jia Sarai, Near IIT, Hauz Khas, New Delhi‐110016 
Phone: 011‐26865455/+91‐9871145498
Website: www.physicsbyfiziks.com  | Email: fiziks.physics@gmail.com  

fiziks
Institute for NET/JRF, GATE, IIT‐JAM, M.Sc. Entrance, JEST, TIFR and GRE in Physics 
 
  1    1    1
f   , f     cos  , f      sin  
4 2 4 4 2 4 4 2

1   
2 3
 1  1 
So Taylor’s series is given by 1   x     x     x   .....
2   4  2!  4  3!  4 

NET/JRF (JUNE-2012)

Q14. A vector perpendicular to any vector that lies on the plane defined by x  y  z  5 , is

(a) iˆ  ˆj (b) ˆj  kˆ (c) iˆ  ˆj  kˆ (d) 2iˆ  3 ˆj  5kˆ


Ans: (c)
   
Solution: Let   x  y  z  5     iˆ  ˆj  kˆ  x  y  z  5  iˆ  ˆj  kˆ .
 x y z 

1 2 3 
 
Q15. The eigen values of the matrix A   2 4 6  are
3 6 9 
 
(a) 1, 4, 9  (b) 0, 7, 7  (c) 0,1,13 (d) 0, 0,14 
Ans: (d)
1   2 3 
Solution: For eigenvalues A  I  0   2 4 6   0
 3 6 9   

1   4   9     36  229     18  312  34     0


1   4   9     361     49     36  9  0
3  142  0  2   14  0    0, 0, 14 .
1
Q16. The first few terms in the Laurent series for in the region 1  z  2 and
z  1z  2
around z  1 is

1  z  z 2  ....1  z  z  z  ....
2 3
1 1
 z  1  z   1  z   ....
2 3
(a) (b)
2  2 4 8  1 z

1  1 1  2 4 
(d) 2 z  1  5 z  1  7z  1  ....
2 3
(c)
z2 1  z  z 2  .... 1  z  z 2  ....

H.No. 40‐D, Ground Floor, Jia Sarai, Near IIT, Hauz Khas, New Delhi‐110016 
Phone: 011‐26865455/+91‐9871145498
Website: www.physicsbyfiziks.com  | Email: fiziks.physics@gmail.com  

fiziks
Institute for NET/JRF, GATE, IIT‐JAM, M.Sc. Entrance, JEST, TIFR and GRE in Physics 
 
Ans: (b)
1 1 1 1 1 1
 1  1  z 
1
Solution:     
z  1z  2 z  2 z  1 1  z  z  1  1 1  z

1 
 1  1  z  
 1 2  1  z 2   1 2  3 1  z 3 ...
     
1 z  2! 3! 


1
1 z

 z  1  z   1  z   ....
2 3

1 2
Q17. Let u  x, y   x  x  y 2  be the real part of analytic function f z  of the complex
2
variable z  x  i y . The imaginary part of f  z  is

(a) y  xy (b) xy (c) y (d) y 2  x 2


Ans: (a)

Solution: u  x, y   x 
1 2
2
 
x  y 2 , v  x, y   ?

u v u v
Check  and  .
x y y x
u v v
  ,  1 x , v  y  xy  f  x 
x y y
u v v
   y, v  yx  f  y 
y x x

y  xy  f  x   yx  f  y 

If f  x   0, f  y  y
v  xy  y
Q18. Let y  x  be a continuous real function in the range 0 and 2 , satisfying the

d2y dy  
inhomogeneous differential equation: sin x 2
 cos x  x  
dx dx  2
The value of dyldx at the point x   / 2
(a) is continuous (b) has a discontinuity of 3
(c) has a discontinuity of 1/3 (d) has a discontinuity of 1
Ans: (d)

H.No. 40‐D, Ground Floor, Jia Sarai, Near IIT, Hauz Khas, New Delhi‐110016 
Phone: 011‐26865455/+91‐9871145498
Website: www.physicsbyfiziks.com  | Email: fiziks.physics@gmail.com  

fiziks
Institute for NET/JRF, GATE, IIT‐JAM, M.Sc. Entrance, JEST, TIFR and GRE in Physics 
 
2
d y dy  
Solution: After dividing by sin x , 2
 cot x  cosec x    x  
dx dx  2

dy  dy   
Integrating both sides,   cot x dx   cosec x  x  dx
dx  dx   2
dy
 cot x  y   cosec 2 x  ydx  1
dx

Using Dirac delta property:  f x  x  x   f x  (it lies with the limit).


0 0

dy cos x
 y   y cosec 2 xdx  1 , at x   ; sin x  0 . So this is point of discontinuity.
dx sin x
Q19. A ball is picked at random from one of two boxes that contain 2 black and 3 white and 3
black and 4white balls respectively. What is the probability that it is white?
(a) 34 / 70 (b) 41 / 70 (c) 36 / 70 (d) 29 / 70
Ans: (b)
Solution: Probability of picking white ball
2 B 3W 3B 4W
3 4
From box I  and from box II 
5 7  
1  3 4  41
Probability of picking a white ball from either of the two boxes is   
2  5 7  70
Q20. The eigenvalues of the antisymmetric matrix,
 0  n3 n2 
 
A   n3 0  n1 
  n2 0 
 n1

where n1 , n 2 and n3 are the components of a unit vector, are


(a) 0, i,  i (b) 0,1,  1 (c) 0,1  i,  1,  i (d) 0, 0, 0
Ans: (a)
 0 n3 n2   0  n3 n2 
Solution: A   n3 0 n1    A   n3
 T
0  n1 
 n2 n1 0    n2 n1 0 

0    n3 n2 
 A   I   0,  n3 0  n1   0
  n2 n1 0   

H.No. 40‐D, Ground Floor, Jia Sarai, Near IIT, Hauz Khas, New Delhi‐110016 
Phone: 011‐26865455/+91‐9871145498
Website: www.physicsbyfiziks.com  | Email: fiziks.physics@gmail.com  

fiziks
Institute for NET/JRF, GATE, IIT‐JAM, M.Sc. Entrance, JEST, TIFR and GRE in Physics 
 
 1  0  2    n12  n22  n32  3   n12  n22  n32

but n12  n22  n32  1

so, 1  0 , 2  i , 3  i

A   AT (Antisymmetric). Eigenvalues are either zero or purely imaginary.


Q21. Which of the following limits exists?
 N 1   N 1 
(a) lim    ln N  (b) lim    ln N 
N 
 m 1 m  N 
 m 1 m 
 N
1  N
1
(c) lim    ln N  (d) lim 
N 
 m 1 m  N 
m 1 m

Ans: (b)
Q22. A bag contains many balls, each with a number painted on it. There are exactly n balls
which have the number n (namely one ball with 1, two balls with 2, and so on until N on
them). An experiment consists of choosing a ball at random, noting the number on it and
returning it to the bag. If the experiment is repeated a large number of times, the average
value the number will tend to
2N  1 N N 1 N  N  1
(a) (b) (c) (d)
3 2 2 2
Ans: (a)
N  N  1
Solution: Total number of balls 1  2  3  4  .....  N 
2
k
The probability for choosing a k th ball at random 
N  N  1
2
2 k 2 2 N  N  12 N  1
Average of it is given by k  k  P   
N  N  1 N N  1 6
2N 1 N  N  12 N  1
 where k 2  .
3 6

H.No. 40‐D, Ground Floor, Jia Sarai, Near IIT, Hauz Khas, New Delhi‐110016 
Phone: 011‐26865455/+91‐9871145498
Website: www.physicsbyfiziks.com  | Email: fiziks.physics@gmail.com  
10 
fiziks
Institute for NET/JRF, GATE, IIT‐JAM, M.Sc. Entrance, JEST, TIFR and GRE in Physics 
 
Q23. Consider a sinusoidal waveform of amplitude 1V and frequency f 0 . Starting from an
1
arbitrary initial time, the waveform is sampled at intervals of . If the corresponding
2 f0

Fourier spectrum peaks at a frequency f and an amplitude A , them

(a) f  2 f 0 and A  1V (b) f  2 f 0 and 0  A  1 V


f0 1
(c) f  0 and A  1V (d) f  and A  V
2 2
Ans: (b)
Solution: y  1sin 2 f 0t  . y
The fourier transform is:
1 IV
F y    f  f 0     f  f 0 
2
t
1 0 T
In Fourier domain f  f 0 , A  .
2

NET/JRF (DEC-2012)
 a b c 
Q24. The unit normal vector of the point  , ,  on the surface of the ellipsoid
 3 3 3
x2 y 2 z 2
   1 is
a2 b2 c2
bciˆ  caˆj  abkˆ aiˆ  bˆj  ckˆ
(a) (b)
a2  b2  c2 a2  b2  c2

biˆ  cˆj  akˆ iˆ  ˆj  kˆ


(c) (d)
a2  b2  c2 3
Ans: All the options given are incorrect.
x2 y 2 z 2
Solution: Here     1.
a 2 b2 c2


Unit normal vector is .


H.No. 40‐D, Ground Floor, Jia Sarai, Near IIT, Hauz Khas, New Delhi‐110016 
Phone: 011‐26865455/+91‐9871145498
Website: www.physicsbyfiziks.com  | Email: fiziks.physics@gmail.com  
11 
fiziks
Institute for NET/JRF, GATE, IIT‐JAM, M.Sc. Entrance, JEST, TIFR and GRE in Physics 
 
     x 2
y 2
z2  2 xiˆ 2 yˆj 2 zkˆ
So,    i  ˆj  kˆ    2  2  2  1  2  2  2
 x y z   a b c  a b c

2 ˆ 2 ˆ 2 ˆ
  a bc 
 i j k
 , ,
 3 3 3
 a 3 b 3 c 3

4 4 4 2 b 2c 2  a 2c 2  a 2c 2
    
3a 2 3b 2 3c 2 3 a 2b 2 c 2
2 ˆ 2 ˆ 2 ˆ
i j k
 a 3 b 3 c 3 bciˆ  caˆj  abkˆ
 
  a b c 
2 b 2c 2  c 2 a 2  a 2b 2 b 2c 2  c 2 a 2  a 2b 2
 , , 
 3 3 3 3 abc

Q25. Given a 2  2 unitary matrix U satisfying U †U  UU †  1 with det U  e i , one can


construct a unitary matrix V V †V  VV †  1 with det V  1 from it by

(a) multiplying U by e i / 2


(b) multiplying any single element of U by e  i
(c) multiplying any row or column of U by e i / 2
(d) multiplying U by e  i
Ans: (a)
Q26. The graph of the function f x  shown below is best described by
1.00
(a) The Bessel function J 0  x 
0.25
0.50
(b) cos x
0.25
f (x)

0.00
(c) e  x cos x
 0.25
 0.50
1
(d) cos x  0.75
x  1.00
0 1 2 3 4 5 6 7 8 9 10
Ans: (a) x
Q27. In a series of five Cricket matches, one of the captains calls “Heads” every time when the
toss is taken. The probability that he will win 3 times and lose 2 times is
(a) 1 / 8 (b) 5 / 8 (c) 3 / 16 (d) 5 / 16
Ans: (d)

H.No. 40‐D, Ground Floor, Jia Sarai, Near IIT, Hauz Khas, New Delhi‐110016 
Phone: 011‐26865455/+91‐9871145498
Website: www.physicsbyfiziks.com  | Email: fiziks.physics@gmail.com  
12 
fiziks
Institute for NET/JRF, GATE, IIT‐JAM, M.Sc. Entrance, JEST, TIFR and GRE in Physics 
 
3 53 2
1  1 5! 1 1 5!
Solution: P    1      .
2  2 3!5  3! 8  2  3!5  3!
1 5  4  3! 20 5 5
    
32 3! 2! 32  2 8  2 16

The probability of getting exactly k successes in n trials is given by probability mass


n!
p k  1  p  , k  successes, n  trials.
nk
function 
k !n  k !

Q28. The Taylor expansion of the function ln cosh x  , where x is real, about the point x  0
starts with the following terms:
1 2 1 4 1 2 1 4
(a)  x  x  .... (b) x  x  ....
2 12 2 12
1 2 1 4 1 2 1 4
(c)  x  x  .... (d) x  x  ....
2 6 2 6
Ans: (b)
e x  ex
Solution: cosh x  .Tailor’s series expansion of f  x  about x  a
2
f  a  f ''  a  f '''
f  x  f a   x  a   x  a  x  a   ... . Here a  0 .
2 3

1 ! 2 ! 3 !

e x  ex  1 e x  e x e x  e x
f  x   log    0 , f   x  x 0  x  x   x  tanh x  0
 2  x 0 e e 2 e  e x
2

f ' ' x  
e x
  
 ex e x  ex  e x  ex e x  ex    e x
 ex   e
2 x
 ex 
2

 1 tanh 2 x
e x
e x 2
 e x
e 
x 2

At x  0, f ''  x   1, f '''  x   0, f   x   2,...

1 2 1 4
 f x   x  x  .......
2 12
z 3 dz
Q29. The value of the integral  , where C is a closed contour defined by the
C z 2
 5z  6 
equation 2 z  5  0, traversed in the anti-clockwise direction, is

(a)  16 i (b) 16 i (c) 8 i (d) 2 i


Ans: (a)
H.No. 40‐D, Ground Floor, Jia Sarai, Near IIT, Hauz Khas, New Delhi‐110016 
Phone: 011‐26865455/+91‐9871145498
Website: www.physicsbyfiziks.com  | Email: fiziks.physics@gmail.com  
13 
fiziks
Institute for NET/JRF, GATE, IIT‐JAM, M.Sc. Entrance, JEST, TIFR and GRE in Physics 
 
Solution: z  5 z  6  0  z  2 z  3z  6  0  z  z  2   3 z  2  0  z  3, 2
2 2

2 z  5  z  2.5 , only 2 will be inside.

z3 8 z 3 dz
Residue   z  2    8   2  2 i  8  16 i
z  3z  2 z  2 2  3 c z  5z  6

NET/JRF (JUNE-2013)


tn
 H n x 
2
Q30. Given that  e t  2tx
n 0 n!
the value of H 4 0  is
(a) 12 (b) 6 (c) 24 (d) – 6
Ans: (a)
 
tn tn t4 t6
 H n x   e t  2tx   H n 0   e t  1  t 2  
2 2
Solution:
n 0 n! n 0 n! 2! 3!

H 4 0  4 t 4 4!
 t   H 4 0   12 .
4! 2! 2!
Q31. A unit vector n̂ on the xy -plane is at an angle of 120 o with respect to iˆ . The angle
 
between the vectors u  a iˆ  b nˆ and v  anˆ  b iˆ will be 60 o if

(a) b  3a / 2 (b) b  2a / 3 (c) b  a / 2 (d) b  a


Ans: (c)

Solution: u  aiˆ  bnˆ , v  anˆ  biˆ

  
 u  v  aiˆ  bnˆ  anˆ  biˆ  u v cos 60  a 2 iˆ  nˆ  ab  ba  b 2 nˆ.iˆ

a 2
 b 2  2ab cos120   cos 60  a
2
2
cos120  2ab  b 2 cos120

 2 2 1
2
1 2
2
2 1
 
1
 a  b  2ab    cos 60   a  b  2ab   a 2  b 2      a 2  b 2   2ab
ab
 2 2 2

5ab a
 a2  b2  b .
2 2

H.No. 40‐D, Ground Floor, Jia Sarai, Near IIT, Hauz Khas, New Delhi‐110016 
Phone: 011‐26865455/+91‐9871145498
Website: www.physicsbyfiziks.com  | Email: fiziks.physics@gmail.com  
14 
fiziks
Institute for NET/JRF, GATE, IIT‐JAM, M.Sc. Entrance, JEST, TIFR and GRE in Physics 
 
Q32. With z  x  iy, which of the following functions f  x, y  is NOT a (complex) analytic
function of z ?

(a) f  x, y    x  iy  8 4  x 2  y 2  2ixy 
3 7

(b) f  x, y    x  iy  1  x  iy 
7 3

(c) f  x, y   x 2  y 2  2ixy  3
5

(d) f  x, y   1  x  iy  2  x  iy 
4 6

Ans: (d)
Solution: f  x, y   1  x  iy  2  x  iy   1   x  iy  2  x  iy 
4 6 4 6

Due to present of z   x  iy 
Q33. The solution of the partial differential equation
2 2
u  x , t   u  x, t   0
t 2 x 2
satisfying the boundary conditions u 0, t   0  u L, t  and initial conditions

u  x,0   sin x / L  and u x, t  t 0  sin 2x / L  is
t

(a) sin  x / L  cos t / L   sin 2x / L  cos2 t / L 


L
2
(b) 2 sin x / L  cos t / L   sin x / L  cos2 t / L 

(c) sin x / L  cos2 t / L   sin 2x / L sin  t / L 


L

(d) sin x / L  cos t / L   sin 2x / L sin 2 t / L 


L
2
Ans: (d)
 2u  2u x u 2 x
Solution:  2  0 , u  x,0   sin and  sin
t 2
x L t t 0 L
This is a wave equation
 an t an  t   n x 
So solution is given by u  x, t     An cos  Bn sin  sin  
n  L L   L 

H.No. 40‐D, Ground Floor, Jia Sarai, Near IIT, Hauz Khas, New Delhi‐110016 
Phone: 011‐26865455/+91‐9871145498
Website: www.physicsbyfiziks.com  | Email: fiziks.physics@gmail.com  
15 
fiziks
Institute for NET/JRF, GATE, IIT‐JAM, M.Sc. Entrance, JEST, TIFR and GRE in Physics 
 
n x n x
L L
2 2
with An   f  x  sin dx, Bn   g  x  sin dx
L0 L an 0 L

 2u  2u x 2 x
Comparing a 2  2 , We have a  1 and f  x   sin , g  x   sin ,
t 2
x L L
 2 x 
 1  cos
x n x x L  dx  2  L  1 (let n  1 )
L L L
2 2 2
An   sin sin dx   sin 2 dx    
L0 L L L0 L L 0 2  L 2
 
2 x n x
L
2
Putting n  2 , Bn  
an 0
sin
L
 sin
L
dx

 4 x 
 1  cos
2 2 x L  dx  2  L  L
L L
2 2
2 0 2 0 
 sin dx   
L 2  2 2 2
 
Q34. The solution of the differential equation
dx
 x2
dt
with the initial condition x0   1 will blow up as t tends to
1
(a) 1 (b) 2 (c) (d) 
2
Ans: (a)
dx dx x 21 1
Solution:  x 2   2   dt  t C  t C
dt x  2 1 x
1 1 1
 x0  1   0  C  C  1   t 1  x  as t  1 , x blows up
1 x 1 t
1
Q35. The inverse Laplace transforms of is
s s  1
2

1 2 t 1 2
(a) t e (b) t  1  e t
2 2

t 1  e t 
1 2
(c) t 1  e  t (d)
2
Ans: (c)

H.No. 40‐D, Ground Floor, Jia Sarai, Near IIT, Hauz Khas, New Delhi‐110016 
Phone: 011‐26865455/+91‐9871145498
Website: www.physicsbyfiziks.com  | Email: fiziks.physics@gmail.com  
16 
fiziks
Institute for NET/JRF, GATE, IIT‐JAM, M.Sc. Entrance, JEST, TIFR and GRE in Physics 
 
 1  t t
Solution: f s  
1
 f t   e t  L1     e dt   e t 0   e t  1
t
   
s 1  ss  1  0
 1  t
 
   e  1 dt  e  t  
t
 L1  2 t t
 et  t  1 .
 s  s  1  0 0

Q36. The approximation cos   1 is valid up to 3 decimal places as long as  is less than:

(take 180 o /   57.29 o )


(a) 1.280 (b) 1.810 (c) 3.280 (d) 4.010
Ans: (b)
2 4 2
Solution: cos  1    .......  1 
2! 4! 2!

cos   1 when   1.81o   0.0314
100

NET/JRF -(DEC-2013)

Q37. If A  iˆyz  ˆjxz  kˆxy , then the integral  A  dl (where C is along the perimeter of a
C

rectangular area bounded by x  0, x  a and y  0, y  b ) is

(a) 
1 3
2
a  b3  (b)  ab 2  a 2 b  (c)  a 3  b 3  (d) 0

Ans: (d)

 A  d l     A.d a  0 since   A  0 .
C S

Q38. If A, B and C are non-zero Hermitian operators, which of the following relations must
be false?
(a) A, B   C (b) AB  BA  C (c) ABA  C (d) A  B  C
Ans: (a)
Solution:  A, B   C  AB  BA  C  ( AB  BA)†  C †

(( AB)†  ( BA)† )  C †  ( B † A† )  ( A† B † )  C †
Hence A,B and C are hermitian then
BA  AB  C  A, B   C

H.No. 40‐D, Ground Floor, Jia Sarai, Near IIT, Hauz Khas, New Delhi‐110016 
Phone: 011‐26865455/+91‐9871145498
Website: www.physicsbyfiziks.com  | Email: fiziks.physics@gmail.com  
17 
fiziks
Institute for NET/JRF, GATE, IIT‐JAM, M.Sc. Entrance, JEST, TIFR and GRE in Physics 
 
Q39. Which of the following functions cannot be the real part of a complex analytic function
of z  x  iy ?

(a) x 2 y (b) x 2  y 2 (c) x 3  3 xy 2 (d) 3x 2 y  y  y 3


Ans: (a)
Solution: Let x 2 y be real part of a complex function. Use Milne Thomson’s method to write
analytic complex function. The real part of that function should be (1) but that is not the
case. So this cannot be real part of an analytic function. Also,
z 2  x  iy   x 2  y 2  2ixy , Real part option (2)
2

z 3   x  iy   x 3  iy 3  3ixy  x  iy 
3

 x 3  iy 3  3ix 2 y  3xy 2 , Real part option (3)


Q40. The expression
 2 2 2 2  1
 2  2  2  2  2
 x
 1 x 2 x3 x 4  1 
 x  x2  x2  x2
2 3 4 
is proportional to
(a)   x1  x 2  x3  x 4  (b)   x1  x 2  x3   x 4 

(c) x12  x 22  x32  x 42  (d) x12  x 22  x32  x 42 


3 / 2 2

Ans: (b)
   1   2 x1
Solution:   2  
2 
 x1  x1  x 2  x3  x 4  x1  x2  x3  x4 
 2 2 2 2 2 2 2

 2
    
2
2 2 2
 2 x
 1  x2  x3  x4 1  2  2 x1  x1 x12  x22  x32  x42
 2  
x12
 
4
 x12  x22  x32  x42 
 

 2 3 4 1  1 2 3
 x 2  x 2  x 2  x 2 2  4 x 2  8x 2  2 x 2  x 2  x 2  x 2
 2  1  1 4 
 
x1  x 2  x3  x 4
2 2 2 2 3
  
x1  x 2  x3  x 4
2 2 2 2 3

Now similarly solving all and add up then we get
 2 2 2  2  1 1 1 1 
 2  2  2  2  2  2  2  2 
 x1 x 2 x3 x 4  x1 x 2 x3 x 4 

H.No. 40‐D, Ground Floor, Jia Sarai, Near IIT, Hauz Khas, New Delhi‐110016 
Phone: 011‐26865455/+91‐9871145498
Website: www.physicsbyfiziks.com  | Email: fiziks.physics@gmail.com  
18 
fiziks
Institute for NET/JRF, GATE, IIT‐JAM, M.Sc. Entrance, JEST, TIFR and GRE in Physics 
 


2 2

8 x1  x2  x3  x4  8 x1  x22  x32  x42
2 2 2

0
  
 
3
x12  x22  x32  x42

also if all x1 , x2 , x3 , x4 becomes zero it should be infinity.


 2 2 2 2  1
 x  x 2  x 2  x 2    x1     x 2     x3     x 4 

So  2  2  2  2  2
 x1 x 2 x3 x 4  1  2 3 4 
 
dx  dx
Q41. Given that the integral  2
0 y  x
2

2y
, the value of  y
0
2
 x2 
2
is

   
(a) (b) 3
(c) (d)
y 3
4y 8y 3
2 y3
Ans: (b)
 
1
Solution:  y
dx
 
dx
, pole is of 2nd order at x  iy , residue  1/ 4iy 3  
 x2   
2 2
0
2 2  y  x
2 2

1 1 
Integral     2 i  3 
2 4iy 4 y3  
Q42. The Fourier transform of the derivative of the Dirac  - function, namely   x  , is
proportional to
(a) 0 (b) 1 (c) sin k (d) ik
Ans: (d)
Solution: Fourier transform of   x 

H K     x e ikx dx  ike k 0   ik


Q43. Consider an n  nn  1 matrix A , in which Aij is the product of the indices i and j

(namely Aij  ij ). The matrix A

(a) has one degenerate eigevalue with degeneracy n  1


(b) has two degenerate eigenvalues with degeneracies 2 and n  2 
(c) has one degenerate eigenvalue with degeneracy n
(d) does not have any degenerate eigenvalue
Ans: (a)

H.No. 40‐D, Ground Floor, Jia Sarai, Near IIT, Hauz Khas, New Delhi‐110016 
Phone: 011‐26865455/+91‐9871145498
Website: www.physicsbyfiziks.com  | Email: fiziks.physics@gmail.com  
19 
fiziks
Institute for NET/JRF, GATE, IIT‐JAM, M.Sc. Entrance, JEST, TIFR and GRE in Physics 
 
1 2
Solution: If matrix is 2  2 let   then eigen value is given by
 2 4
1   2 
   0  (1   )(4   )  4  0    0,5
 2 4

 1 2 3
 
If If matrix is 3  3 let  2 4 6  then eigen value is given by
 3 6 9
 

1   2 3 
 
 2 4 6 0
 3 6 9   

1     4    9     36  2 18  2  9     3 12  3  4     0


1     2  13  36  36  2 18  18  2   312  12  3   0
 2  13   3  13 2  13  0   3  14 2  0    0, 0,   14
i.e. has one degenerate eigenvalue with degeneracy 2.
Thus one can generalized that for n dimensional matrix has one degenerate eigevalue
with degeneracy n  1 .
Q44. Three sets of data A, B and C from an experiment, represented by  , and  , are
plotted on a log-log scale. Each of these are fitted with straight lines as shown in the
figure. 1000

100
C
B
10

1 

 A

0.1 1 10 100 1000
0.1

The functional dependence y  x  for the sets A, B and C are respectively


x 1 1
(a) x , x and x 2 (b)  , x and 2 x (c) , x and x 2 (d) , x and x 2
2 x2 x
Ans: (d)

H.No. 40‐D, Ground Floor, Jia Sarai, Near IIT, Hauz Khas, New Delhi‐110016 
Phone: 011‐26865455/+91‐9871145498
Website: www.physicsbyfiziks.com  | Email: fiziks.physics@gmail.com  
20 
fiziks
Institute for NET/JRF, GATE, IIT‐JAM, M.Sc. Entrance, JEST, TIFR and GRE in Physics 
 
NET/JRF -(JUNE-2014)
Q45. Consider the differential equation
d 2x dx
2
2 x0
dt dt
with the initial conditions x0   0 and x 0   1 . The solution xt  attains its maximum
value when t is
(a) 1/2 (b) 1 (c) 2 (d) 
Ans: (b)
d 2x dx
 2  x  0  m 2  2m  1  0   m  1  0  m  1,  1
2
Solution: 2
dt dt
 x   c1  c2 t  e  t , since x  0   0  0  c1  x  c2te t

 x  c2  te t  e  t 

Since x  0   1  1  c2  x  te t

For maxima or minima x  0  x  te t  e t  0  x  e t 1  t 

 e  t  0, 1  t  0  t  , t  1
x  e  t  1  1  t  e  t  1  e  t   t  1 e  t  
 x 1  e 1  0e  t  0
Q46. Consider the matrix
 0 2i 3i 
 
M    2i 0 6i 
  3i  6i 0 
 
The eigenvalues of M are
(a)  5,  2, 7 (b)  7, 0, 7 (c)  4i, 2i, 2i (d) 2, 3, 6
Ans: (b)
 0 2i 3i   0 2i 3i 
    
Solution: M   2i 0 6i  , M   2i 0 6i 
 3i 6i 0   3i 6i 0 
   
M  M
Matrix is Hermitian so roots are real and trace = 0.
1  2  3  0, 1  2  3  0    7, 0, 7
H.No. 40‐D, Ground Floor, Jia Sarai, Near IIT, Hauz Khas, New Delhi‐110016 
Phone: 011‐26865455/+91‐9871145498
Website: www.physicsbyfiziks.com  | Email: fiziks.physics@gmail.com  
21 
fiziks
Institute for NET/JRF, GATE, IIT‐JAM, M.Sc. Entrance, JEST, TIFR and GRE in Physics 
 
1
Q47. If C is the contour defined by z  , the value of the integral
2
dz
 C sin 2 z

is
(a)  (b) 2 i (c) 0 (d)  i
Ans: (c)
1  1
Solution: f  z   2 z  
sin z  2
3 5
z z 1 1
sin z  z   ....  2
 2
3 5 sin z  z3 z5 
 z   .... 
 3 5 
2
1 1  z2 z4  dz
  2 
1   ....   0
2
sin z z  3 5  C sin 2 z

Q48. Given 

n 0

Pn  x t n  1  2 xt  t 2 1 / 2
, for t  1 , the value of P5  1 is

(a) 0.26 (b) 1 (c) 0.5 (d) 1


Ans: (d)
Solution: Pn  1  1 if n is odd  P5  1  1

Q49. The graph of a real periodic function f  x  for the range  ,   is f x 


shown in the figure.
Which of the following graphs represents the real part of its Fourier x
transform?
Re f k  Re f k 
(a) (b)

k k

(c) Re f k  (d) Re f k 

k k

H.No. 40‐D, Ground Floor, Jia Sarai, Near IIT, Hauz Khas, New Delhi‐110016 
Phone: 011‐26865455/+91‐9871145498
Website: www.physicsbyfiziks.com  | Email: fiziks.physics@gmail.com  
22 
fiziks
Institute for NET/JRF, GATE, IIT‐JAM, M.Sc. Entrance, JEST, TIFR and GRE in Physics 
 
Ans: (b)
Solution: This is cosine function
A
f  x   A cos x  F  k     k  k0     k  k0  
2
NET/JRF (DEC-2014)
 
Q50. Let r denote the position vector of any point in three-dimensional space, and r  r .

Then
      
(a)   r  0 and   r  r / r (b)   r  0 and  2 r  0
       
(c)   r  3 and  2 r  r / r 2 (d)   r  3 and   r  0
Ans: (d)

Solution: r  xxˆ  yyˆ  zzˆ
  x y z
  r      111  3
x y z

xˆ yˆ zˆ
   z y   x z   y x 
  r   / x  / y  / z  xˆ     yˆ     zˆ     0
x y z  y z   z x   x y 

a 0 0 1
   
Q51. The column vector  b  is a simultaneous eigenvector of A   0 1 0  and
a 1 0 0
   

0 1 1
 
B   1 0 1  if
1 1 0
 
(a) b  0 or a  0 (b) b  a or b  2a
(c) b  2a or b  a (d) b  a / 2 or b  a / 2
Ans: (b)
Solution: Let b  a

 0 0 1  a   a   0 1 1  a  a
         
 0 1 0  a    a  and  1 0 1  a   2  a 
 1 0 0  a   a   1 1 0  a  a
         

H.No. 40‐D, Ground Floor, Jia Sarai, Near IIT, Hauz Khas, New Delhi‐110016 
Phone: 011‐26865455/+91‐9871145498
Website: www.physicsbyfiziks.com  | Email: fiziks.physics@gmail.com  
23 
fiziks
Institute for NET/JRF, GATE, IIT‐JAM, M.Sc. Entrance, JEST, TIFR and GRE in Physics 
 
Let b  2a

0 0 1 a   a   0 1 1   a   a   a 
           
 0 1 0   2a    2a  and  1 0 1   2a    2a   1 2a 
1 0 0 a   a   1 1 0   a   a   a 
           
For other combination above relation is not possible.
sin 2 x 

Q52. The principal value of the integral  x3
dx is

(a)  2 (b)   (c)  (d) 2


Ans: (a)
ei 2 z
Solution: Let f  z  
z3
ei 2 z
lim  z  0  f  z   lim  z  0  3  1 finite and  0   z  0 is pole of order 3.
3 3

z 0 z 0 z

1 d2  3 e
i2z

Residue R  lim 2   z  0  3 
 2
2! z  0 dz  z 
 
 f  x  dx   iR   i  2   2 i  Im. Part  2   f  x  dx   2
 

Q53. The Laurent series expansion of the function f  z   e z  e1 / z about z  0 is given by

zn  n 1 1
n n! for all z   
 
(a) (b) n 0
z  n  only if 0  z  1
 z  n!

 1 1 zn
n0  z n  z n n n! only if z  1
 
(c)  for all 0  z   (d)
 n!
Ans: (c)
 z2   zn 1 1 1 
1
Solution: e  1  z   ....   
z
and e  1  
1/ z
2
 ....   n
 2!  n 0 n ! z 2! z n 0 z n !


 11
 f  z    e z  e1/ z     z n
  , for all 0  z  
n 0 z n  n!

H.No. 40‐D, Ground Floor, Jia Sarai, Near IIT, Hauz Khas, New Delhi‐110016 
Phone: 011‐26865455/+91‐9871145498
Website: www.physicsbyfiziks.com  | Email: fiziks.physics@gmail.com  
24 
fiziks
Institute for NET/JRF, GATE, IIT‐JAM, M.Sc. Entrance, JEST, TIFR and GRE in Physics 
 
Q54. Two independent random variables m and n , which can take the integer values
0, 1, 2, ...,  , follow the Poisson distribution, with distinct mean values  and 
respectively. Then
(a) the probability distribution of the random variable l  m  n is a binomial distribution.
(b) the probability distribution of the random variable r  m  n is also a Poisson
distribution.
(c) the variance of the random variable l  m  n is equal to   
(d) the mean value of the random variable r  m  n is equal to 0.
Ans: (c)
Solution:  l2   m2   n2   
1
Q55. Consider the function f z  
ln1  z  of a complex variable
z
z  re i r  0,       . The singularities of f  z  are as follows:
(a) branch points at z  1 and z   ; and a pole at z  0 only for 0    2
(b) branch points at z  1 and z   ; and a pole at z  0 for all  other than 0    2
(c) branch points at z  1 and z   ; and a pole at z  0 for all 
(d) branch points at z  0, z  1 and z   .
Ans: None of the above is correct

1 1 z 2 z3  z z2
Solution: For f  z   ln 1  z     z    .....   1    .....
z z 2 3  2 3
There is no principal part and when z  0 , f  z   1 . So there is removable singularity

at z  0 . Also z  1 and z   is Branch point.

 1 2 n 1
n

x
Q56. The function f  x      , satisfies the differential equation
n 0 n ! n  1 ! 2 

(a) x
d2 f
2

dx 2
x
df
dx

 x2 1 f  0  (b) x
d2 f
dx
2
2
 2x
df
dx

 x2 1 f  0 

(c) x
d2 f
2

dx 2
x
df
dx

 x2 1 f  0  (d) x
d2 f
dx
2
2
x
df
dx

 x2 1 f  0 
Ans: (c)

H.No. 40‐D, Ground Floor, Jia Sarai, Near IIT, Hauz Khas, New Delhi‐110016 
Phone: 011‐26865455/+91‐9871145498
Website: www.physicsbyfiziks.com  | Email: fiziks.physics@gmail.com  
25 
fiziks
Institute for NET/JRF, GATE, IIT‐JAM, M.Sc. Entrance, JEST, TIFR and GRE in Physics 
 
 1  x 
2 n 1
n

Solution: f  x     is generating function (Bessel Function of first kind)
n  0 n ! n  1 !  2 

d2 f df
 x  x 2  n 2 f  0 , put n  1 .
which satisfies the differential equation x 2
dx 2
dx
 
Q57. Let  and  be complex numbers. Which of the following sets of matrices forms a
group under matrix multiplication?
   1 
(a)   (b)   , where   1
0 0   
  *     2 2
(c)  *
 , where  * is real (d)   , where   
*
1
      
*

Ans: (d)
  2 2
Solution:      1
 
* *

  
 x , p , Lk  (where i j k is the Levi-Civita symbol, x , p, L are
3
Q58. The expression i jk i j
i , j , k 1

the position, momentum and angular momentum respectively, and A, B represents the
Poisson Bracket of A and B ) simplifies to
  
(a) 0 (b) 6 
(c) x , p  L   
(d) x  p
Ans: (b)
NET/JRF (JUNE-2015)
 dx
Q59. The value of integral  1  x 4

 
(a) (b) (c) 2 (d) 2
2 2
Ans. (a)
 dz
Solution:  1 z4
z R


 2 n 1
Now, pole z  e 4

i
1 1 1 1
n  0,  z0  e 4  i , n  2  z2  i
2 2 2 2

H.No. 40‐D, Ground Floor, Jia Sarai, Near IIT, Hauz Khas, New Delhi‐110016 
Phone: 011‐26865455/+91‐9871145498
Website: www.physicsbyfiziks.com  | Email: fiziks.physics@gmail.com  
26 
fiziks
Institute for NET/JRF, GATE, IIT‐JAM, M.Sc. Entrance, JEST, TIFR and GRE in Physics 
 
i 3
1 1 1 1
n  1  z1  e 4  i , n  3  z3   i
2 2 2 2
only z0 and z1 lies in contour
i
  1 1 1 
i.e., residue at  z  e 4    i 
  4 2 2
i 3
  1 1 1 
residue at  z  e 4   i 
  4 2 2
 dx 
now  x 1
4
 2 i Re S 
2
d 2x dx
Q60. Consider the differential equation 2
 3  2 x  0 . If x  0 at t  0 and x  1 at t  1 ,
dt dt
the value of x at t  2 is
(a) e 2  1 (b) e 2  e (c) e  2 (d) 2e
Ans. (b)
Solution: D 2  3D  2  0
 D  1 D  2   0  D  1, 2  x  c1e2t  c2et
using boundary condition x  0, t  0  c1  c2

again using boundary condition x  1, t  1

1 1 e2t 1 t
c2  , c1   x   e
ee 2
e e
2
e  e e  e2
2

again using t  2 then x  e 2  e


Q61. The Laplace transform of 6t 3  3sin 4t is
36 12 36 12
(a)  2 (b)  2
s 4
s  16 s 4
s  16
18 12 36 12
(c)  2 (d)  2
s 4
s  16 s 3
s  16
Ans. (a)
n 1
Solution: L 6t 3  3sin 4t   L t n  
s n 1

H.No. 40‐D, Ground Floor, Jia Sarai, Near IIT, Hauz Khas, New Delhi‐110016 
Phone: 011‐26865455/+91‐9871145498
Website: www.physicsbyfiziks.com  | Email: fiziks.physics@gmail.com  
27 
fiziks
Institute for NET/JRF, GATE, IIT‐JAM, M.Sc. Entrance, JEST, TIFR and GRE in Physics 
 
a
 L sin at   2
 s  a2 
6  4 3 4 36 12
L 6t 3  3sin 4t    2  4  2
s 4
s  16 s s  16
2 f 2  f
2
Q62. Let f  x, t  be a solution of the wave equation v in 1 -dimension. If at
t 2 x 2
f
t  0, f  x, 0   e  x and  x, 0   0 for all x , then f  x, t  for all future times t  0 is
2

t
described by

 x 2 v 2t 2 
(b) e  x vt 
2
(a) e
1  x vt 2 3  x  vt 2 1   x vt 2  x  vt  
2
(c) e  e (d) e  e
4 4 2  

Ans. (d)
2 f 2  f
2
Solution: For  v
t 2 x 2
f
 x, 0   0 and f  x, 0   e x
2

t
1 1
 
2
 f  x  vt   f  x  vt   . Therefore, solution is f  e  x vt   e  x  vt 
2
f 
2 2
NET/JRF (DEC-2015)
Q63. In the scattering of some elementary particles, the scattering cross-section  is found to
depend on the total energy E and the fundamental constants h (Planck’s constant) and c
(the speed of light in vacuum). Using dimensional analysis, the dependence of  on
these quantities is given by
2
hc hc  hc  hc
(a) (b) 3 / 2 (c)   (d)
E E E E
Ans.: (c)
Solution: The dimension of  is dimension of “Area”
h  Joul  sec
c  m / sec
E  Joul

H.No. 40‐D, Ground Floor, Jia Sarai, Near IIT, Hauz Khas, New Delhi‐110016 
Phone: 011‐26865455/+91‐9871145498
Website: www.physicsbyfiziks.com  | Email: fiziks.physics@gmail.com  
28 
fiziks
Institute for NET/JRF, GATE, IIT‐JAM, M.Sc. Entrance, JEST, TIFR and GRE in Physics 
 
2
 hc 
   m dimension of area
2

E
1
Q64. If y  , then x is
tanh  x 

 y 1   y 1  y 1 y 1
(a) ln   (b) ln   (c) ln (d) ln
 y 1   y 1  y 1 y 1
Ans.: (d)
1
Solution: y 
tanh x
e x  e x e2 x  1
y 
e x  e x e2 x  1
ye 2 x  y  e 2 x  1  ye 2 x  e 2 x  1  y  e 2 x  y  1  1  y 
1
 y 1  1  y 1  y  1 2
2 x  ln    x  ln    ln  
 y 1  2  y 1  y 1
z
Q65. The function of a complex variable z has
sin  z 2
(a) a simple pole at 0 and poles of order 2 at  n for n  1, 2,3...

(b) a simple pole at 0 and poles of order 2 at  n and i n for n  1, 2,3...

(c) poles of order 2 at  n , n  0,1, 2,3...


(d) poles of order 2 at  n , n  0,1, 2,3...
Ans. : (b)
z z
Solution: f  z   
sin  z 2
sin  z 2
 z2
 z2
sin  z 2
at z  0 , it is a simple pole since, lim 1
z 0  z2
Also, sin  z 2  sin n   z 2   n , z   n ,  i n

 
2 z
lim z  n . , exists. So its pole of order 2 .
z n sin  z 2

H.No. 40‐D, Ground Floor, Jia Sarai, Near IIT, Hauz Khas, New Delhi‐110016 
Phone: 011‐26865455/+91‐9871145498
Website: www.physicsbyfiziks.com  | Email: fiziks.physics@gmail.com  
29 
fiziks
Institute for NET/JRF, GATE, IIT‐JAM, M.Sc. Entrance, JEST, TIFR and GRE in Physics 
 

Q66. The Fourier transform of f  x  is f  k   dxeikx f  x  . 


If f  x     x      x      x  , where   x  is the Dirac delta-function (and prime

denotes derivative), what is f  k  ?

(a)   i  k  i k 2 (b)    k   k 2

(c)   i  k   k 2 (d) i   k  i k 2
Ans.: (c)

Solution: f  k    dx e   x      x      x  
ikx



   x  e dx  
ikx




 


    x  eikx dx   eikx  x     ike   x  dx   i  k
ikx

  

    x  e dx   k 2
ikx



dx
Q67. The solution of the differential equation  2 1  x 2 , with initial condition x  0 at
dt
t  0 is
   
 sin 2t , 0  t  4 sin 2t , 0  t  2
(a) x   (b) x  
sinh 2t ,   1, 
t t
 4  2

 
sin 2t , 0  t  4
(c) x   (d) x  1  cos 2t , t  0
 1, 
t
 4
Ans.: (c)
dx dx
Solution:  2 1  x2 ,  2dt , sin 1 x  2t  c , x  0, t  0 so, c  0  x  sin 2t
dt 1 x 2

x should not be greater than 1 at x  1

H.No. 40‐D, Ground Floor, Jia Sarai, Near IIT, Hauz Khas, New Delhi‐110016 
Phone: 011‐26865455/+91‐9871145498
Website: www.physicsbyfiziks.com  | Email: fiziks.physics@gmail.com  
30 
fiziks
Institute for NET/JRF, GATE, IIT‐JAM, M.Sc. Entrance, JEST, TIFR and GRE in Physics 
 
 
1  sin 2t , sin  sin 2t , t  .
2 4
 
sin 2t , 0t 
So, x 4
1 
, t
 4
Q68. The Hermite polynomial H n  x  , satisfies the differential equation

d 2Hn dH n
2
 2x  2nH n  x   0
dx dx
The corresponding generating function

1
G t, x    H n  x  t n , satisfies the equation
n 0 n !

 2G G G  2G G G
(a)  2x  2t 0 (b)  2x  2t 2 0
x 2
x t x 2
x t
 2G G G  2G G  2G
(c)  2x 2 0 (d)  2 x  2 0
x 2
x t x 2 x xt
Ans.: (a)
1 1 1
Solution: G  H n  x  t n , G   H n  x  t n , G   H n  x  t n
n! n! n!
G 1
 H n  x  n t n 1
t n!
Let’s check the options one by one
G G G
 2x  2t 0
x 2
x t
1 1 1
H n  x  t n  2 x H n  x  t n  2t H n  x  nt n 1
n! n! n!

H n  x   2 xH n  x   2 xH n  x   0 , which is Hermite Differential Equation.


8 1 1
Q69. The value of the integral  0 x 5
2
dx , valuated using Simpson’s rule with h  2 is
3
(a) 0.565 (b) 0.620 (c) 0.698 (d) 0.736

H.No. 40‐D, Ground Floor, Jia Sarai, Near IIT, Hauz Khas, New Delhi‐110016 
Phone: 011‐26865455/+91‐9871145498
Website: www.physicsbyfiziks.com  | Email: fiziks.physics@gmail.com  
31 
fiziks
Institute for NET/JRF, GATE, IIT‐JAM, M.Sc. Entrance, JEST, TIFR and GRE in Physics 
 
Ans.: (a)
2 1
 y0  4  y1  y2   2 y3  y4 
x
Solution: I  y
3 x 5
2

2 1 1 1 1 1 1
   4    2   0
3 5 9 4 21 69  5
2 1
2 1 
   0.5734  0.09523  0.0145 9
3 5  4 1
2 21
  0.2  0.5734  0.09523  0.0145 1
3 6
31
2 8 1
  0.8831  0.5887
3 69
Q70. Consider a random walker on a square lattice. At each step the walker moves to a nearest
neighbour site with equal probability for each of the four sites. The walker starts at the
origin and takes 3 steps. The probability that during this walk no site is visited more than
one is
(a) 12 / 27 (b) 27 / 64 (c) 3 / 8 (d) 9 /16
Ans.: (d)
Solution: Total number of ways  4  4  4
Number of preferred outcome  4  3  3
( Any four option in step-1 and only 3 option in step 2 &3 because he can not go to
previous position)
4  3 3 9
probability  
4  4  4 16

H.No. 40‐D, Ground Floor, Jia Sarai, Near IIT, Hauz Khas, New Delhi‐110016 
Phone: 011‐26865455/+91‐9871145498
Website: www.physicsbyfiziks.com  | Email: fiziks.physics@gmail.com  
32 
fiziks
Institute for NET/JRF, GATE, IIT‐JAM, M.Sc. Entrance, JEST, TIFR and GRE in Physics 
 
NET/JRF (JUNE-2016)
1
Q71. The radius of convergence of the Taylor series expansion of the function
cosh  x 

around x  0 , is

(a)  (b)  (c) (d) 1
2
Ans: (c)
1 e4 z  1
Q72. The value of the contour integral  cosh  z   2sinh  z  dz around the unit circle C
2 i 
C

traversed in the anti-clockwise direction, is


8 1
(a) 0 (b) 2 (c) (d)  tanh  
3 2
Ans: (c)
e4 z  1 e4 z  1 e4 z  1
Solution: f  z    z z 
cosh z  2sinh z e  e ez 3
2
 e z  e z   e z
2 2
 

 f z 

2e z e 4 z  1   2 e  e 
5z z

3  e 2z
 3  e  2z

ln 3
For pole at z  z0 ,3  e 2 z0  0  e 2 z0  3  z0 
2
It has simple pole at z0

Re  z0   lim  z  z0  f  z   lim  z  z0 

2 e5 z  e z 
z  z0 z  z0 3e 2z

 z  z0   2  5e5 z  e z   2  e5 z  e z  1  e5 z0  e z0 
 lim   
z  z0 2e 2 z  e
2 z0


 3  3 
5
9 3 3 8
         
 3   3  3
 
1 1 8
f  z  dz   2 i  Residue  
2 i  2 i 3

H.No. 40‐D, Ground Floor, Jia Sarai, Near IIT, Hauz Khas, New Delhi‐110016 
Phone: 011‐26865455/+91‐9871145498
Website: www.physicsbyfiziks.com  | Email: fiziks.physics@gmail.com  
33 
fiziks
Institute for NET/JRF, GATE, IIT‐JAM, M.Sc. Entrance, JEST, TIFR and GRE in Physics 
 
Q73. The Gauss hypergeometric function F  a, b, c, z  , defined by the Taylor series expansion
around z  0 as F  a, b, c, z  
 a  a  1 ...  a  n  1 b  b  1 ...  b  n  1

n 0 c  c  1 ...  c  n  1 n !
zn ,

satisfies the recursion relation


d c
(a) F  a, b, c; z   F  a  1, b  1, c  1; z 
dz ab
d c
(b) F  a, b, c; z   F  a  1, b  1, c  1; z 
dz ab
d ab
(c) F  a, b, c; z   F  a  1, b  1, c  1; z 
dz c
d ab
(d) F  a, b, c; z   F  a  1, b  1, c  1; z 
dz c
Ans: (d)
dF  a  a  1 ...  a  n  1 b  b  1 ...  b  n  1 n 1
Solution:  nz
dz n 0 c  c  1 ...  c  n  1 n !
 a  a  1 ...  a  n  1 b  b  1 ...  b  n  1 z n 1

n 0 c  c  1 ...  c  n  1 n 1

ab  a  1 ...  a  n  1 b  1 ...  b  n  1 z n 1



c
  c  1 ...  c  n  1 n 1

ab   a  1 ...  a   n  1  1  b  1 ... b   n  1  1 z n 1


 
c n 0  c  1 ... c   n  1  1 n 1

dF ab
 F  a  1, b  1, c  1, z 
dz c
Q74. Let X and Y be two independent random variables, each of which follow a normal
distribution with the same standard deviation  , but with means   and   ,
respectively. Then the sum X  Y follows a
(a) distribution with two peaks at   and mean 0 and standard deviation  2
(b) normal distribution with mean 0 and standard deviation 2
(c) distribution with two peaks at   and mean 0 and standard deviation 2

(d) normal distribution with mean 0 and standard deviation  2

H.No. 40‐D, Ground Floor, Jia Sarai, Near IIT, Hauz Khas, New Delhi‐110016 
Phone: 011‐26865455/+91‐9871145498
Website: www.physicsbyfiziks.com  | Email: fiziks.physics@gmail.com  
34 
fiziks
Institute for NET/JRF, GATE, IIT‐JAM, M.Sc. Entrance, JEST, TIFR and GRE in Physics 
 
Ans: (d)
Solution:     x   y      0

 12   x2   y2   2   2

   2
Q75. Using dimensional analysis, Planck defined a characteristic temperature TP from powers

of the gravitational constant G , Planck’s constant h , Boltzmann constant k B and the

speed of light c in vacuum. The expression for TP is proportional to

hc 5 hc3 G hk B2
(a) (b) 2 (c) (d)
k B2G kBG hc 4 k B2 Gc 3

Ans: (a)
E ML2T 2
Solution: E  h  h    ML2T 1
 T 1
E ML2T 2
E  k BT  k B    ML2T 2TP1
T TP

m1m2 MLT 2  L2
F G 2 G  2
 G  M 1 L3T 2
r M

 
5
hc 5 ML2T 1  LT 1 ML7T 6
   TP2  TP
k B2G M 2 L4T 4TP2  M 1 L3T 2 7 6 2
ML T TP

Q76. What is the Fourier transform  dxeikx f  x  of



dn
f  x    x   n
  x
n 1 dx

where   x  is the Dirac delta-function?

1 1 1 1
(a) (b) (c) (d)
1 i k 1 i k k i k i
Ans: (b)
  
dn dn
Solution: f  x     x    n   x    n   x      n   x 
n 1 dx n  0 dx n 0

 F   x    1  F   n   x     ik  F   x     ik 


n n

H.No. 40‐D, Ground Floor, Jia Sarai, Near IIT, Hauz Khas, New Delhi‐110016 
Phone: 011‐26865455/+91‐9871145498
Website: www.physicsbyfiziks.com  | Email: fiziks.physics@gmail.com  
35 
fiziks
Institute for NET/JRF, GATE, IIT‐JAM, M.Sc. Entrance, JEST, TIFR and GRE in Physics 
 

 f  x      n  x 
n 0


1 1
 F  f  x      ik   1  ik   ik    ik   .... 
n 2 3

n 0 1   ik  1  ik

Q77. In finding the roots of the polynomial f  x   3 x 3  4 x  5 using the iterative Newton-

Raphson method, the initial guess is taken to be x  2 . In the next iteration its value is
nearest to
(a) 1.671 (b) 1.656 (c) 1.559 (d) 1.551
Ans: (b)
Solution: f  x   3 x 3  4 x  5 ; f   x   9 x 2  4

3xn3  4 xn  5 3 x03  4 x0  5
xn 1  xn   x1  x0 
9 xn2  4 9 x02  4
3 8  4  2  5 11
Let x0  2  x1  2   2  x1  1.656
9 4  4 32

NET/JRF (DEC-2016)
1 3 2
 
Q78. The matrix M   3 1 0  satisfies the equation
0 0 1
 
(a) M 3  M 2  10M  12 I  0 (b) M 3  M 2  12 M  10 I  0
(c) M 3  M 2  10M  10 I  0 (d) M 3  M 2  10 M  10 I  0
Ans. : (c)
Solution: The characteristic equation is

1    3 2
3  1    0 0
0 0 1   
 1    1   1      3  3 1     0
    2  1    1  9 1     0   3  10   2  10  0

Thus the matrix M satisfies the equation

H.No. 40‐D, Ground Floor, Jia Sarai, Near IIT, Hauz Khas, New Delhi‐110016 
Phone: 011‐26865455/+91‐9871145498
Website: www.physicsbyfiziks.com  | Email: fiziks.physics@gmail.com  
36 
fiziks
Institute for NET/JRF, GATE, IIT‐JAM, M.Sc. Entrance, JEST, TIFR and GRE in Physics 
 
M  M  10 M  10 I  0 then the correct option is (c)
3 2

Q79. The Laplace transform of


t
 , 0t T
f t   T
 1 t T

is
 1  e  sT  1  e   sT
1  e   sT
1  e  sT

(a) (b) (c) (d)


s 2T s 2T s 2T s 2T
Ans. : (b)
Solution: we can write
t t t t
f  t   u0  t   uT  t    uT  t   1  uT  t    uT  t    uT  t   uT  t 
T T T T
Hence the transform of f  t  is

t     t  T   T  
L  f  t   L    L uT  t      L uT  t 
T    T  

1 e  sT  1 T  e  sT 1  e  sT
      2
s 2T T  s2 s  s sT
 1
Q80. The Fourier transform  dxf  x eikx of the function f  x   is
 x 2
2

 2k    2k
(a) 2 e (b) 2 e  2k
(c) e 2k
(d) e
2 2
Ans. : (d)
1 1  a k
Solution: Fourier transform of f  x   , a  0 is x eikx dx  e
x  a2
2 2
a 2
a
1  
x
2k
Hence eikx dx  e
2
a 2
2
Q81. Given the values sin 450  0.7071, sin 500  0.7660, sin 550  0.8192 and sin 600  0.8660 ,

the approximate value of sin 520 , computed by Newton’s forward difference method, is
(a) 0.804 (b) 0.776 (c) 0.788 (d) 0.798
Ans. : (c)

H.No. 40‐D, Ground Floor, Jia Sarai, Near IIT, Hauz Khas, New Delhi‐110016 
Phone: 011‐26865455/+91‐9871145498
Website: www.physicsbyfiziks.com  | Email: fiziks.physics@gmail.com  
37 
fiziks
Institute for NET/JRF, GATE, IIT‐JAM, M.Sc. Entrance, JEST, TIFR and GRE in Physics 
 
Solution: Given -
x y  f  x f  x  2 f  x  3 f  x 

450 0  7071

500 0  7660 0  0589 0  0057 0  0007

550 0  8192 0  0532 0  0064

600 0  8660 0  0468

h  5 , 520  x0  uh  450  uh

52  45 52  45 7
u  
h 5 5
From table we have
y0  0  7071, y0  0  0589 ,  2 y0  0  0057

 3 y0  0  0007 ,  4 y0  0,.....
y0  2 y0  3 y0
 sin  520   y0  u u  u  1  u  u  1 u  2   
1 2 3

7  0  0057  7  7   0  0007  7  7   7 
 0  7071   0  0589     1      1   2   0
5 2 55   6  5  5  5 
0  0399 0  0049
 0  7071  0  0825  
25 125
 0  7071  0  0825  0  0016  0  0000  0  7880
f 2 f
Q82. Let f  x, t  be a solution of the heat equation  D 2 in one dimension. The initial
t x
condition at t  0 is f  x, 0   e  x for   x   . Then for all t  0, f  x, t  is given by
2

 

2
[Useful integral: dx e  x  ]
 
2 2
1 
x
1 
x
(a) e 1 Dt (b) e 1 2 Dt
1  Dt 1  2 Dt
2
x2
1 
x

(c) e 1 4 Dt (d) e 1 Dt

1  4 Dt
Ans. : (c)
f 2 f
Solution: D 2 , t 0
t x
H.No. 40‐D, Ground Floor, Jia Sarai, Near IIT, Hauz Khas, New Delhi‐110016 
Phone: 011‐26865455/+91‐9871145498
Website: www.physicsbyfiziks.com  | Email: fiziks.physics@gmail.com  
38 
fiziks
Institute for NET/JRF, GATE, IIT‐JAM, M.Sc. Entrance, JEST, TIFR and GRE in Physics 
 
f  f
2
Let D  c 2 ,  c2 2
t x

Initial condition f  x , 0   e  x  g  x  ,    x  
2

   
2
 x  2 cz t
Now, g x  2cz t  e

The solution f  x , t  is given as


 
1   1  e  z dz
2
 x 2  4 c 2 z 2t  4 cxz t
f  x,t 
 x  2 cz t
e e
 z2 2
e dz 
   

  x 2 
 
2
e x   4 c 2 z 2t  4 cxz t  z 2  dz  e  
 1 4 c 2t z 2  4 cxz t

 e
  e

dz

e x  2   2  2cx t   2cx t  


2 2

 exp   1  4c t   z  2  z  1  4c 2t    1  4c 2t    1  4c 2t   dz
cx t
 2 2

       

 2 cx t 
2
4 c 2 x 2t  2 cx t 
2

e x
2   
 1 4 c 2t  z  2 1 4c t  dz  e 2  x2 4 c 2 x 2t 
 
 1 4 c 2t  z  2

 e

 1 4 tc 
e

e 1 4 c 2t
e

 1 4 tc 
dz

 x2

e 1 4 c 2t
1 
  2 
 2 1  4c 2t
2
x
1
f  x, t  
1  4 Dt
e1 4 Dt c 2
 D

3
Q83. A stable asymptotic solution of the equation xn 1  1  is x  2 . If we take
1  xn
n 1
xn  2 n and xn 1  2 n 1 , where n and n1 are both small, the ratio is
n
approximately
1 1 1 2
(a)  (b)  (c)  (d) 
2 4 3 3
Ans. (c)

H.No. 40‐D, Ground Floor, Jia Sarai, Near IIT, Hauz Khas, New Delhi‐110016 
Phone: 011‐26865455/+91‐9871145498
Website: www.physicsbyfiziks.com  | Email: fiziks.physics@gmail.com  
39 
fiziks
Institute for NET/JRF, GATE, IIT‐JAM, M.Sc. Entrance, JEST, TIFR and GRE in Physics 
 
NET/JRF (JUNE-2017)
Q84. Which of the following can not be the eigen values of a real 3  3 matrix
(a) 2i, 0,  2i (b) 1,1,1 (c) ei , e  i ,1 (d) i, 1, 0
Ans. : (d)
Solution: If the matrix is real then the complex eigen values always occurs with its complex
conjugate. In option (d) if i is an eigen value then i must also be an eigen value. But
i is not given in option, hence option (d) is incorrect.
Q85. Let u  x, y   e ax cos  by  be the real part of a function f  z   u  x, y   iv  x, y  of the

complex variable z  x  iy , where a, b are real constants and a  0 . The function f  z 

is complex analytic everywhere in the complex plane if and only if


(a) b  0 (b) b   a (c) b  2 a (d) b  a  2
Ans. : (b)
Solution: The function f  z  will be analytic everywhere in the complex plane if and only if it

satisfies the Cauchy Riemann equation in that region.


u v u v
  and 
x y y x
v
Hence ae ax cos  by   (i)
y
v
and be ax sin  by   (ii)
x
From equation (i)
ae ax sin  by 
v  x, y    c y (iii)
b
Differentiating partially with x gives

v a e sin  by 
2 ax

 (iv)
x b
From equation (iii) and (iv)
a 2 e ax sin  by 
be sin  by  
ax

b
 b2  a 2  b   a

H.No. 40‐D, Ground Floor, Jia Sarai, Near IIT, Hauz Khas, New Delhi‐110016 
Phone: 011‐26865455/+91‐9871145498
Website: www.physicsbyfiziks.com  | Email: fiziks.physics@gmail.com  
40 
fiziks
Institute for NET/JRF, GATE, IIT‐JAM, M.Sc. Entrance, JEST, TIFR and GRE in Physics 
 
i z / 2
ze
Q86. The integral  2 dz along the closed contour  shown in the figure is
 z 1
y

1 1 x

(a) 0 (b) 2 (c) 2 (d) 4 i


Ans. : (c)
zeiz / 2
Solution: f  z  
 z  1 z  1
For z  1 anti-clockwise
zei z / 2 2 i i / 2
I  2 i lim  e   iei / 2
z 1  z  1 2

For z  1

I  2 i lim
zei z / 2
 2 i 
 1 e  i / 2

  ie  i / 2
z 1  z  1  2 

Integral   i
e i / 2
 e  i / 2 
 2i  2 i 2 sin

 2
2i 2
dy cos  x
Q87. The function y  x  satisfies the differential equation x  2y  . If y 1  1 , the
dx x
value of y  2  is

(a)  (b) 1 (c) 1/ 2 (d) 1/ 4


Ans. : (d)
Solution: The given differential equation can be written as
dy 2 cos  x
 y
dx x x2
2
This is a linear differential equation with Integrating factor  e  x  x 2
dx

cos  x sin  x c
Hence y.x 2   x 2 . dx  c  y   2
x 2
 x2 x

H.No. 40‐D, Ground Floor, Jia Sarai, Near IIT, Hauz Khas, New Delhi‐110016 
Phone: 011‐26865455/+91‐9871145498
Website: www.physicsbyfiziks.com  | Email: fiziks.physics@gmail.com  
41 
fiziks
Institute for NET/JRF, GATE, IIT‐JAM, M.Sc. Entrance, JEST, TIFR and GRE in Physics 
 
sin  x 1
when x  1, y  1 hence c  1  y   2
 x2 x
1
hence, when x  2, y 
4
Q88. The random variable x    x    is distributed according to the normal distribution
x2
1 
P  x  e 2 2
. The probability density of the random variable y  x 2 is
2 2

1 2 1 2
(a) e  y / 2 , 0  y   (b) e  y / 2 , 0  y  
2 y 2
2 2 y 2

1 2 1 2
(c) e  y / 2 , 0  y   (d) e y /  , 0  y  
2 2
2 y 2

Ans. : (a)
x2
1 
Solution: p  x   e 2 2
,   x  
2 2
  x2
1 

 p  x  dx  1  
  2 2
e 2 2
dx  1

 x2  x2
1  2 1 
2 e 2 2
dx  1  e 2 2
dx  1
0 2 2 2 2 0

1
put x 2  y dy  2 xdx , dx 
2 y
 y  y
1 1 1 1 2 2
2
2 2
e
0
2 2

2 y
dy 
2 2

0 y
e dy 

y
1 
f  y  e 2 2
, 0  y  
2 y 2

H.No. 40‐D, Ground Floor, Jia Sarai, Near IIT, Hauz Khas, New Delhi‐110016 
Phone: 011‐26865455/+91‐9871145498
Website: www.physicsbyfiziks.com  | Email: fiziks.physics@gmail.com  
42 
fiziks
Institute for NET/JRF, GATE, IIT‐JAM, M.Sc. Entrance, JEST, TIFR and GRE in Physics 
 
Q89. The Green’s function satisfying
d2
g  x, x0     x  x0 
dx 2
with the boundary conditions g   L, x0   0  g  L, x0  , is

 1  1
 2 L  x0  L  x  L  ,  L  x  x0  2 L  x0  L  x  L  ,  L  x  x0
(a)  (b) 
 1  x  L  x  L  , x0  x  L  1  x  L  x  L  , x0  x  L
 2 L 0  2 L 0

 1
 2 L  L  x0  x  L  ,  L  x  x0
1
(c)  (d)  x  L  x  L  , L  x  L
 1  x  L  L  x  , x0  x  L
2L
 2 L 0
Ans. : (a)
d2
Solution: 2 g  x, x0     x  x0 
dx
boundary conditions:
g   L, x0   0  g  L, x0 

The homogeneous equation for Green’s function is


d2
g  x, x0   0
dx 2
Solution of above equation is
 Ax  B, x  x0
g  x, x0   
Cx  D, x  x0

Applying boundary condition


g   L, x0   0   AL  B  0  AL  B

g  L, x0   0  CL  D  0  CL  D

 A  x  L  , x  x0
Hence, g  x, x0   
C  x  L  , x  x0

From continuity of Green’s function at x  x0 we have

A  x0  L   C  x0  L 

H.No. 40‐D, Ground Floor, Jia Sarai, Near IIT, Hauz Khas, New Delhi‐110016 
Phone: 011‐26865455/+91‐9871145498
Website: www.physicsbyfiziks.com  | Email: fiziks.physics@gmail.com  
43 
fiziks
Institute for NET/JRF, GATE, IIT‐JAM, M.Sc. Entrance, JEST, TIFR and GRE in Physics 
 
 x L
AC 0 
 x0  L 
g
From discontinuity of derivative of Green’s function at x  x0
x
We have
g g
 1
x x  x0 x x  x0

g g
 C, A
x x  x0 x x  x0

 C  A  1  C  A 1
Thus, the required solution of Green’s function is given by
  x0  L  x  L 
 , x  x0
g  x, x0    2 L
  x0  L  x  L  , x  x0
 2L
 i z 
Q90. Let  x , y , z be the Pauli matrices and x x  y y  z  z  exp  
 2 
 i z 
 x x  y y  z z  exp   2 .
 
Then the coordinates are related as follows
 x   cos   sin  0  x   x   cos  sin  0  x 
         
(a)  y    sin  cos  0  y  (b)  y     sin  cos  0  y 
 z   0 0 1   z   z   0 0 1   z 
     

       
 cos 2 sin 0  cos 2  sin 0
 x   2  x   2
 x  x
           
(c)  y     sin cos 0  y  (d)  y    sin cos 0  y 
 2 2   2 2 
 z    z   z    z 
  0 0 1     0 0 1  
   
   
Ans. : (b)
0 1  0 i  1 0 
Solution:  x    , y    and  z   
1 0 i 0   0 1 
H.No. 40‐D, Ground Floor, Jia Sarai, Near IIT, Hauz Khas, New Delhi‐110016 
Phone: 011‐26865455/+91‐9871145498
Website: www.physicsbyfiziks.com  | Email: fiziks.physics@gmail.com  
44 
fiziks
Institute for NET/JRF, GATE, IIT‐JAM, M.Sc. Entrance, JEST, TIFR and GRE in Physics 
 
 z x  iy 
Hence, x x  y y  z z   
 x  iy z 

 z' x1  iy ' 
x ' x  y ' y  z ' z   
 x ' iy ' z ' 
i / 2  i / 2
 i z  e 0   i z  e 0 
exp  
   i / 2 
and exp  
  
 z   0 e   2   0 ei / 2 

 z x  iy   ei / 2 0  z x  iy   e i / 2 0 


Hence,     
 x  iy  z   0 e i / 2   x  iy z   0 ei / 2 

 z x  iy   z ei  x  iy  
 
  
 x  iy  z    e  i  x  iy  z 
Hence, z   z and x  iy  ei  x  iy 

Thus x  iy   cos   x   sin   y   i  cos   y   sin   x 

Thus x   cos   x   sin   y

And y '    sin   x   cos   y

 x   cos  sin  0  x 
    
Thus,  y     sin  cos  0  y 
 z   0 0 1   z 
  
Q91. Which of the following sets of 3  3 matrices (in which a and b are real numbers) forms
a group under matrix multiplication?
 1 0 a    1 a 0  
     
(a)  0 1 0  ; a, b    (b)  0 1 b  ; a, b   
 b 0 1     
    0 0 1  

 1 0 a    1 a 0  
     
(c)  0 1 b  ; a, b    (d)  b 1 0  ; a, b   
 0 0 1     
    0 0 1  
Ans. : (c)

H.No. 40‐D, Ground Floor, Jia Sarai, Near IIT, Hauz Khas, New Delhi‐110016 
Phone: 011‐26865455/+91‐9871145498
Website: www.physicsbyfiziks.com  | Email: fiziks.physics@gmail.com  
45 
fiziks
Institute for NET/JRF, GATE, IIT‐JAM, M.Sc. Entrance, JEST, TIFR and GRE in Physics 
 
Solution: In order to form the group the required matrix must satisfy the following conditions.
(a) For any three matrices A, B, C  G A  BC    AB  C

(b) There must exist an identity element I such that AI  IA  A


(c) There must exist an inverse element for each element belonging to G.
(d) For any two matrices A, B  G , the matrix A B  G (closer property).
All the given matrices satisfy property (a), if we put a  0 and b  0, we see that
property (b) also holds for all the given matrices.
If we put a  1 and b  1, in option (a) , two rows becomes identical and matrix in option
(a) is non-invertible. If we put a  1 and b  1, in option (d), two rows becomes identical
and the matrix is non-invertible.
Now only option (b) and (c) remains. For option (b) take two matrices and multiply
 1 a1 0   1 a2 0   1 a1  a2 a1b2 
    
 0 1 b1   0 1 b2    0 1 b1  b2 
0 0 1 0 0 
1  0  0 1 
 
we see that the resulting matrix does not satisfy closure property. For option (3) take two
matrices and multiply
 1 0 a1   1 0 a2   1 0 a2  a1 
    
 0 1 b1   0 1 b2    0 1 b2  b1 
0 0 1 0 0 1  0 0 1 
   
we see that the resulting matrix satisfy the closure property. Hence the correct option is (c)
Q92. A random variable n obeys Poisson statistics. The probability of finding n  0 is 106 .
The expectation value of n is nearest to
(a) 14 (b) 106 (c) e (d) 102
Ans. : (a)
n
Solution: In Poisson’s statistics the probability of finding the value n is given by P  n   e 
n!
The mean of Poisson’s statistics is  . From the question
0
P  0   10  10 
6 6
e    e    106
0!
Talking Log of both sides,    6 ln10    6 ln10
Hence the expectation value of n is   6  2.30  13.8  14

H.No. 40‐D, Ground Floor, Jia Sarai, Near IIT, Hauz Khas, New Delhi‐110016 
Phone: 011‐26865455/+91‐9871145498
Website: www.physicsbyfiziks.com  | Email: fiziks.physics@gmail.com  
46 
fiziks
Institute for NET/JRF, GATE, IIT‐JAM, M.Sc. Entrance, JEST, TIFR and GRE in Physics 
 
CLASSICAL MECHANICS SOLUTIONS

NET/JRF (JUNE-2011)
b
Q1. A particle of unit mass moves in a potential V  x   ax 2 , where a and b are positive
x2
constants. The angular frequency of small oscillations about the minimum of the potential
is
(a) 8b (b) 8a (c) 8a / b (d) 8b / a
Ans: (b)
1
b V 2b  b 4
Solution: V  x   ax 2  2   0  2ax  3  0  ax 4  b  0  x0    .
x x x a
k  2V
Since   , m  1 and k  2 where x0 is stable equilibrium point.
m x x  x0
1
 2V 6b 6b  b 4
Hence k  2  2a  4  2a   8a at x  x0    .
x x0 b a
a
Thus,   8a .
Q2. The acceleration due to gravity  g  on the surface of Earth is approximately 2.6 times
that on the surface of Mars. Given that the radius of Mars is about one half the radius of
Earth, the ratio of the escape velocity on Earth to that on Mars is approximately
(a) 1.1 (b) 1.3 (c) 2.3 (d) 5.2
Ans: (c)
Solution: Escape velocity = 2 gR
Escape velocity of Earth ge R e R g
  2.3 where e  2 and e  2.6.
Escape velocity of Mass gm R m Rm gm

Q3. The Hamiltonian of a system with n degrees of freedom is given by H q1 ,.....q n ; p1 ,....... p n ; t  ,

with an explicit dependence on the time t . Which of the following is correct?


(a) Different phase trajectories cannot intersect each other.
(b) H always represents the total energy of the system and is a constant of the motion.
(c) The equations q i  H / pi , p i  H / qi are not valid since H has explicit time
dependence.
(d) Any initial volume element in phase space remains unchanged in magnitude under
time evolution.
H.No. 40‐D, Ground Floor, Jia Sarai, Near IIT, Hauz Khas, New Delhi‐110016 
Phone: 011‐26865455/+91‐9871145498
Website: www.physicsbyfiziks.com  | Email: fiziks.physics@gmail.com  

fiziks
Institute for NET/JRF, GATE, IIT‐JAM, M.Sc. Entrance, JEST, TIFR and GRE in Physics 
 
Ans: (a)
Q4. The Lagrangian of a particle of charge e and mass m in applied electric and magnetic
1 2   
fields is given by L  mv  eA  v  e , where A and  are the vector and scalar
2
potentials corresponding to the magnetic and electric fields, respectively. Which of the
following statements is correct?
 
(a) The canonically conjugate momentum of the particle is given by p  mv

p2 e  
(b) The Hamiltonian of the particle is given by H   A  p  e
2m m
(c) L remains unchanged under a gauge transformation of the potentials
(d) Under a gauge transformation of the potentials, L changes by the total time derivative
Ans: (d)
L 
Solution: 2 V  
t 0

Q5. Consider the decay process         in the rest frame of the   . The masses of the

  ,   and   are M  , M  and zero respectively.

A. The energy of   is

(a)
M 2
 
 M 2 c 2
(b)
M 2
 
 M 2 c 2
(c)  M   M   c 2 (d) M M  c2
2M  2M
Ans. : (b)
Solution:        
From conservation of energy M  c2  E   E  .

E2  p 2 c 2  M 2 c 4 and E2  p 2 c 2 since momentum of   and   is same.

M 2 c 4
M c 2  E  E , M  2 c 4  E2  E2  E  E 
M c 2

E  E 
M 2 c 2
and E  E  M c  E 
2
M2  M 2 c 2
.
 
M 2M
B. The velocity of   is

(a)
M  M 2 c
2

(b)
M  M 2 c
2

(c)
M c
(d)
M c
M 2  M 2 M 2  M 2 M M
Ans: (a)
H.No. 40‐D, Ground Floor, Jia Sarai, Near IIT, Hauz Khas, New Delhi‐110016 
Phone: 011‐26865455/+91‐9871145498
Website: www.physicsbyfiziks.com  | Email: fiziks.physics@gmail.com  

fiziks
Institute for NET/JRF, GATE, IIT‐JAM, M.Sc. Entrance, JEST, TIFR and GRE in Physics 
 
Solution: Velocity of  

E 
M 2
  M 2  c 2

M c 2  v2 
 1  2  
4 M 2 M 2
2M   c   M 2  M 2 
2
v2
1
c2
v2 4M 2 M 2 v 2 M 4  M 4  2 M 2 M 2  4 M 2 M 2  M 2  M 2 
  1     v   2 2 
c.
c2  M 2  M 2  c M 2  M 2   M  M 
2 2 2
 

Q6. The Hamiltonian of a particle of unit mass moving in the xy -plane is given to be:
1 2 1 2
H  xp x  yp y  x  y in suitable units. The initial values are given to be
2 2

x0, y0  1,1 and  p x 0, p y 0   1 , 1  . During the motion, the curves traced out
2 2
by the particles in the xy-plane and the p x p y – plane are

(a) both straight lines


(b) a straight line and a hyperbola respectively
(c) a hyperbola and an ellipse, respectively
(d) both hyperbolas
Ans: (d)
1 2 1 2
Solution: H  xpx  yp y  x  y
2 2
Solving Hamiltonion equation of motion
H H
  p x  px  x   p x and   p y   p y  y   p y .
x y
H H
 x  x  x and  y   y  y .
px p y

After solving these four differential equation and eliminating time t and using boundary
1 1 1
condition one will get  x  and px 
y 2 py

H.No. 40‐D, Ground Floor, Jia Sarai, Near IIT, Hauz Khas, New Delhi‐110016 
Phone: 011‐26865455/+91‐9871145498
Website: www.physicsbyfiziks.com  | Email: fiziks.physics@gmail.com  

fiziks
Institute for NET/JRF, GATE, IIT‐JAM, M.Sc. Entrance, JEST, TIFR and GRE in Physics 
 
NET/JRF (DEC-2011)
Q7. A double pendulum consists of two point masses m attached by strings of length l as
shown in the figure: The kinetic energy of the pendulum is

(a)
2

1 2 2 2
ml  1   2 
l
(b)
2

ml 21   2  212 cos1   2 
1 2 2 2
 1

(c)
1 2 2
2

ml 1  222  212 cos1   2   m

l
(d) 
ml 2 1   2  212 cos 1   2 
1 2 2 2
2
 2

Ans: (b) m

Solution: Let co-ordinate  x1 , y1  and  x 2 , y 2  . K .E. 


1
2
 1
 
m x12  y 12  m x 22  y 22
2

x1  l sin  1 , y1  l cos 1  x1  l cos 11 , y1  l sin 11

x 2  l sin  1  l sin  2 , y 2  l cos 1  l cos  2

 
 x 2  l cos 11  l cos  2 2 , y 2  l  sin 11  l  sin  2 2
Put the value of x1 , y1 , x 2 , y 2 in K.E equation, one will get
1 2 2 2
T ml  21   2  212 cos 1   2   .
2
Q8. A constant force F is applied to a relativistic particle of rest mass m. If the particle starts
from rest at t = 0, its speed after a time t is

(a) Ft / m
 Ft 
(b) c tanh  
(c) c 1  e  Ft / mc  (d)
Fct
 mc  F 2t 2  m 2c 2
Ans: (d)
dp
Solution:  F  p  Ft  c . At t  0, p  0 so, c  0
dt
F
mu  t Fct
 p  Ft   Ft  u  m  .
u2  Ft 
2
F 2 t 2  m2 c 2
1 2 1  
c  mc 

H.No. 40‐D, Ground Floor, Jia Sarai, Near IIT, Hauz Khas, New Delhi‐110016 
Phone: 011‐26865455/+91‐9871145498
Website: www.physicsbyfiziks.com  | Email: fiziks.physics@gmail.com  

fiziks
Institute for NET/JRF, GATE, IIT‐JAM, M.Sc. Entrance, JEST, TIFR and GRE in Physics 
 
Q9. The potential of a diatomic molecule as a function of the distance r between the atoms is
a b
given by V r     12 . The value of the potential at equilibrium separation between
r 6
r
the atoms is:
(a)  4 a 2 / b (b)  2 a 2 / b (c)  a 2 / 2b (d)  a 2 / 4b
Ans: (d)
a b V 1  12b 
Solution: V r   
a 12b
 , for equilibrium  0     6    0  6 a  0
r 6 r 12 r r 7
r 13
r 
7
r 6 
1 1
12b  12b  6  2b  6
 6a  6  0  r    r  
r  6a   a 
 1

 2b  a b a2 a2 a2
V r      
6
      .
  a    2b   2b 2 2b 4b 4b
     
 a   a 
Q10. Two particles of identical mass move in circular orbits under a central potential

V r  
1 2
kr . Let l1 and l2 be the angular momenta and r1 , r2 be the radii of the orbits
2
l1 r
respectively. If  2 , the value of 1 is:
l2 r2

(a) 2 (b) 1 / 2 (c) 2 (d) 1 / 2


Ans: (a)
l2 1
Solution: Veff   kr 2 , where l is angular momentum.
2mr 2
2
Veff l2
Condition for circular orbit  0   3  kr  0  l 2  r 4  l  r 2 .
r mr
2
l r  r l r l
Thus 1   1   1  1  1  2 since 1  2 .
l2  r2  r2 l2 r2 l2

Q11. A particle of mass m moves inside a bowl. If the surface of the bowl is given by the

equation z 
1
2
 
a x 2  y 2 , where a is a constant, the Lagrangian of the particle is

(a)
1
2

m r 2  r 2 2  gar 2  (b)
1
2
 
m 1  a 2 r 2 r 2  r 2 2 
(c)
1
2

m r 2  r 2 2  r 2 sin 2  2  gar 2  1
 
(d) m 1  a 2 r 2 r 2  r 2 2  gar 2
2

H.No. 40‐D, Ground Floor, Jia Sarai, Near IIT, Hauz Khas, New Delhi‐110016 
Phone: 011‐26865455/+91‐9871145498
Website: www.physicsbyfiziks.com  | Email: fiziks.physics@gmail.com  

fiziks
Institute for NET/JRF, GATE, IIT‐JAM, M.Sc. Entrance, JEST, TIFR and GRE in Physics 
 
Ans: (d)

Solution: L 
1
2
  1

m x 2  y 2  z 2  mgz , where z  a x 2  y 2 .
2

a r2  .
1
It has cylindrical symmetry. Thus x  r cos  , y  r sin  , z 
2
x  r cos   r sin  , y  r sin   r cos  and z  a  rr  .

m 1  a 2 r 2  r 2  r 22  gar 2  .


1
So, L 
2
Q12. A planet of mass m moves in the inverse square central force field of the Sun of mass
M . If the semi-major and semi-minor axes of the orbit are a and b , respectively, the
total energy of the planet is:
GMm 1 1
(a)  (b)  GMm  
ab a b
GMm  1 1   ab 
(c)     (d)  GMm 

a b a  a  b 
2

Ans: (a)
Solution: Assume Sun is at the centre of elliptical orbit.
1 2 GMm 1 2 GMm
Conservation of energy mv1   mv2 
2 a 2 b v2

Conservation of momentum L  mv1a  mv2b b


v1
s a
 a
v2  v1  
 b
1 2 1 2 GMm GMm 1  2 2 a 
2
ba
mv1  mv2    m  v1  v1 2   GMm  
2 2 a b 2  b   ab 

1 2  b2  a 2   b  a 1 b 1
mv1    GMm    mv12  GMm   
 b   ab   a  b  a 
2
2 2

1 2 GMm b 1 GMm
E mv1   GMm 
2 a a b  a  a

GMm  b  GMm  b  b  a  GMm


   1     
a  b  a    a  b  a   b  a 

H.No. 40‐D, Ground Floor, Jia Sarai, Near IIT, Hauz Khas, New Delhi‐110016 
Phone: 011‐26865455/+91‐9871145498
Website: www.physicsbyfiziks.com  | Email: fiziks.physics@gmail.com  

fiziks
Institute for NET/JRF, GATE, IIT‐JAM, M.Sc. Entrance, JEST, TIFR and GRE in Physics 
 
Q13. An annulus of mass M made of a material of uniform density has inner and outer radii a
and b respectively. Its principle moment of inertia along the axis of symmetry
perpendicular to the plane of the annulus is:
1 b 4  a 4 
(a) M 2
2 b  a 2 
(b)
1
2

M b 2  a 2 
(c) M b 2  a 2   
1 1
(d) M b 2  a 2
2 2
Ans: (d)
Q14. The trajectory on the zpz - plane (phase-space trajectory) of a ball bouncing perfectly
elastically off a hard surface at z = 0 is given by approximately by (neglect friction):

PZ PZ
(a) (b)

z z

PZ PZ

(c) (d)

z z

Ans: (a)
Pz2 P2
Solution: H   mgz and E  z  mgz .
2m 2m

H.No. 40‐D, Ground Floor, Jia Sarai, Near IIT, Hauz Khas, New Delhi‐110016 
Phone: 011‐26865455/+91‐9871145498
Website: www.physicsbyfiziks.com  | Email: fiziks.physics@gmail.com  

fiziks
Institute for NET/JRF, GATE, IIT‐JAM, M.Sc. Entrance, JEST, TIFR and GRE in Physics 
 
NET/JRF (JUNE-2012)

Q15. The bob of a simple pendulum, which undergoes small oscillations, is immersed in water.
Which of the following figures best represents the phase space diagram for the pendulum?

p p
(a) (b)

x x

p p
(c) (d)

x x

Ans: (d)
Solution: When simple pendulum oscillates in water it is damped oscillation so amplitude
continuously decrease and finally it stops.
Q16. Two events separated by a (spatial) distance 9  109 m , are simultaneous in one inertial
frame. The time interval between these two events in a frame moving with a constant
speed 0.8 c (where the speed of light c  3  108 m / s ) is
(a) 60 s (b) 40 s (c) 20 s (d) 0 s
Ans: (b)
Solution: x2'  x1'  9  10 9 m and t 2'  t1'  0 . Then

   
 t '  v x'   t1  v x' 
 2 c2 2
t 2  t1  
 

1 1 ' '
' '

c 2   t  t  t 2  t1  v x 2  x1  v x 2  x1 .
' '
 
  2 
2 1
v2 c c2
2
 1 v
2
  1 v  v2 v2
    1  1  1 
 c2   c2  c2 c2 c2
Put v  0.8c  t 2  t1  40 sec

H.No. 40‐D, Ground Floor, Jia Sarai, Near IIT, Hauz Khas, New Delhi‐110016 
Phone: 011‐26865455/+91‐9871145498
Website: www.physicsbyfiziks.com  | Email: fiziks.physics@gmail.com  

fiziks
Institute for NET/JRF, GATE, IIT‐JAM, M.Sc. Entrance, JEST, TIFR and GRE in Physics 
 
x 2
Q17. If the Lagrangian of a particle moving in one dimensions is given by L   V  x  the
2x
Hamiltonian is
x 2 p2
xp  V  x   V x  x  V  x   V x 
1 2 1 2
(a) (b) (c) (d)
2 2x 2 2x
Ans: (a)
L x
Solution: Since H  p x x  L and  p x   p x  x  p x x .
x x

 px x 
2
x 2 p x2 x
H  p x x   V  x   H  px  px x   Vx  H   V x  .
2x 2x 2
Q18. A horizontal circular platform mutes with a constant angular velocity  directed
vertically upwards. A person seated at the centre shoots a bullet of mass m horizontally
with speed v. The acceleration of the bullet, in the reference frame of the shooter, is
(a) 2v  to his right (b) 2v  to his left
(c) v  to his right (d) v  to his left
Ans: (a)

Solution: Velocity of bullet = vˆj , Angular velocity= k̂ . There will be coriolis

 
force F  2m v   .

F  2mviˆ  a  2viˆ .
 
Q19. The Poisson bracket  r , p  has the value
 
(a) r p (b) rˆ  pˆ (c) 3 (d) 1

Ans: (b)
 
Solution: r  xiˆ  yjˆ  zkˆ , r   x 2  y 2  z 2  , p  p xiˆ  p y ˆj  p z kˆ ,
1/ 2


p   px2  p y2  pz2 
1/ 2

           
   r  p  r  p   r  p  r  p   r  p  r  p 
 r , p  =  x  p  p  x    y  p  p  y    z  p  p  y 
 x x   y y   z z 
 
x px y p y z pz rp
              rˆ  pˆ 
r p r p r p r p

H.No. 40‐D, Ground Floor, Jia Sarai, Near IIT, Hauz Khas, New Delhi‐110016 
Phone: 011‐26865455/+91‐9871145498
Website: www.physicsbyfiziks.com  | Email: fiziks.physics@gmail.com  

fiziks
Institute for NET/JRF, GATE, IIT‐JAM, M.Sc. Entrance, JEST, TIFR and GRE in Physics 
 
Q20. Consider the motion of a classical particle in a one dimensional double-well

potential V  x   
1 2
4
2

x  2 . If the particle is displaced infinitesimally from the minimum

on the x-axis (and friction is neglected), then


(a) the particle will execute simple harmonic motion in the right well with an angular

frequency   2

(b) the particle will execute simple harmonic motion in the right well with an angular

frequency   2

(c) the particle will switch between the right and left wells

(d) the particle will approach the bottom of the right well and settle there

Ans: (b)

Solution: V  x   x  2 2  V  2 x 2  2  2 x  0  x  0 , x   2 .
1 2
 
4 x 4
 2V  2V
 3 x 2
 2 . At x  0 ,  0 so V is maximum. Thus it is unstable point
x 2 x 2

 2V
 2V x 2 x  x0
 4 and it is stable equilibrium point with   2    1.
x 2 x 2

Q21. What is proper time interval between the occurrence of two events if in one inertial frame
events are separated by 7.5 108 m and occur 6.5 s apart?
(a) 6.50 s (b) 6.00 s (c) 5.75 s (d) 5.00 s
Ans: (b)
Solution: Proper time interval
2

 6.5     6 sec.
r2 7.5
t   t    2 
2 2

c  3 

H.No. 40‐D, Ground Floor, Jia Sarai, Near IIT, Hauz Khas, New Delhi‐110016 
Phone: 011‐26865455/+91‐9871145498
Website: www.physicsbyfiziks.com  | Email: fiziks.physics@gmail.com  
10 
fiziks
Institute for NET/JRF, GATE, IIT‐JAM, M.Sc. Entrance, JEST, TIFR and GRE in Physics 
 
NET/JRF (DEC-2012)

Q22. A solid cylinder of height H, radius R and density ρ, floats vertically on the surface of a
liquid of density  0 . The cylinder will be set into oscillatory motion when a small
instantaneous downward force is applied. The frequency of oscillation is

g  g g 0 g
(a) (b) (c) (d)
0 H 0 H 0H H
Ans: (d)
Solution: From Newton’s law of motion ma  mg  0 Agh where A is area of cross section,

m   AH .

0 gh 0 g
  AHa   AHg  0 Agh  a  1  
H H
Q23. Three particles of equal mass (m) are connected by two identical massless springs of
stiffness constant (K) as shown in the figure
K K

m m m
If x1, x2 and x3 denote the horizontal displacement of the masses from their respective
equilibrium positions the potential energy of the system is

(a)
1
2

K x12  x 22  x32  (b)
1
2

K x12  x 22  x32  x 2  x1  x3  
(c)
1
2

K x12  2 x 22  x32  2 x 2  x1  x3  (d)
1
2
 
K x12  2 x 22  2 x 2  x1  x3 
Ans: (c)
1 1
K  x2  x1   K  x3  x2  ,
2 2
Solution: V 
2 2

K  x22  x12  2 x2 x1   K  x32  x22  2 x3 x2   V  K  x12  2 x22  x32  2 x2  x1  x3  


1 1 1
V
2 2 2
Q24. A planet of mass m moves in the gravitational field of the Sun (mass M ). If the semi-
major and semi-minor axes of the orbit are a and b respectively, the angular momentum
of the planet is

2GMm 2 ab 2GMm 2 ab
(a) 2GMm 2 a  b  (b) 2GMm 2 a  b  (c) (d)
ab ab
H.No. 40‐D, Ground Floor, Jia Sarai, Near IIT, Hauz Khas, New Delhi‐110016 
Phone: 011‐26865455/+91‐9871145498
Website: www.physicsbyfiziks.com  | Email: fiziks.physics@gmail.com  
11 
fiziks
Institute for NET/JRF, GATE, IIT‐JAM, M.Sc. Entrance, JEST, TIFR and GRE in Physics 
 
Ans: (d)
Solution: Assume Sun is at the centre of elliptical orbit.
1 2 GMm 1 2 GMm
Conservation of energy mv1   mv2  v2
2 a 2 b
Conservation of momentum L  mv1a  mv2b b
v1
s a
 a
v2  v1  
 b

1 2 1 2 GMm GMm 1  2 2 a 
2
 b  a
mv1  mv2    m  v1  v1 2   GMm 
2 2 a b 2  b   ab 

1 2  b2  a 2   b  a 1  b 1
mv1    GMm    mv12  GMm   
2  b 2
  ab  2  a  b  a

 b 1
v1  2GM   
 a  b  a

 b  1  2GMab 2GMm 2 ab
L  mv1 a  m 2GM      a  m L
 a  b  a b  a ab

Q25. The Hamiltonian of a simple pendulum consisting of a mass m attached to a massless


p2
string of length l is H   mgl 1  cos   . If L denotes the Lagrangian, the value of
2ml 2
dL
is:
dt
2g g
(a)  p sin  (b)  p sin 2
l l
g
(c) p cos  (d) lp2 cos 
l
Ans: (a)
dL L p2
Solution:  L, H   where H   mgl 1  cos   .
dt t 2ml 2
H p ml 2 2
L   pi qi  H  p   H ,  
    , L  mgl 1  cos   .
i P ml 2 2

H.No. 40‐D, Ground Floor, Jia Sarai, Near IIT, Hauz Khas, New Delhi‐110016 
Phone: 011‐26865455/+91‐9871145498
Website: www.physicsbyfiziks.com  | Email: fiziks.physics@gmail.com  
12 
fiziks
Institute for NET/JRF, GATE, IIT‐JAM, M.Sc. Entrance, JEST, TIFR and GRE in Physics 
 
Hence we have to calculate L, H  which is only defined into phase space i.e. p and  .

p2
Then  L   mgl 1  cos  
2ml 2

L, H   L  H 
L H
 
2g
p sin  and
L
0
dL

2g
p sin 
 p p  l t dt l
Q26. Two bodies of equal mass m are connected by a massless rigid rod of length l lying in the
xy-plane with the centre of the rod at the origin. If this system is rotating about the z-axis
with a frequency ω, its angular momentum is
(a) ml 2 / 4 (b) ml 2 / 2 (c) ml 2 (d) 2ml 2
Ans: (b)
Solution: Since rod is massless i.e. M  0 .
l
Moment of inertia of the system I  m1r12  m2 r22 , m1  m2  m and r1  r2 
2
ml 2 ml 2 ml 2 ml 2
I  I . Angular momentum, J  I and J  .
4 4 2 2
Q27. Which of the following set of phase-space trajectories is not possible for a particle
obeying Hamilton’s equations of motion?
(a) (b)
P P

x x
(c) (d)
P P

x x
Ans: (b)
Solution: Phase curve does not cut each other
H.No. 40‐D, Ground Floor, Jia Sarai, Near IIT, Hauz Khas, New Delhi‐110016 
Phone: 011‐26865455/+91‐9871145498
Website: www.physicsbyfiziks.com  | Email: fiziks.physics@gmail.com  
13 
fiziks
Institute for NET/JRF, GATE, IIT‐JAM, M.Sc. Entrance, JEST, TIFR and GRE in Physics 
 
Q28. The muon has mass 105 MeV / c and mean life time 2.2  s in its rest frame. The mean
2

distance traversed by a muon of energy 315 MeV before decaying is approximately,

(a) 3  105 km (b) 2.2 cm (c) 6.6  m (d) 1.98 km


Ans: (d)
MeV
Solution: Since E  315MeV and m0  105 .
c2
m0 c 2 m0 c 2 105
E  mc 2  E   315   315   v  0.94c .
v2 v2 v2
1 2 1 2 1 2
c c c
t0 2.2  10 6
Now, t  , t 0  2.2 s  t   t  6.6  s
v2 8
1 2 1
c 9

Now the distance traversed by muon is vt  0.94c  6.6  10 6  1.86 km .

NET/JRF (JUNE-2013)

Q29. The area of a disc in its rest frame S is equal to 1 (in some units). The disc will appear
distorted to an observer O moving with a speed u with respect to S along the plane of the
disc. The area of the disc measured in the rest frame of the observer O is ( c is the speed
of light in vacuum)
1/ 2 1 / 2 1
 u2   u2   u2   u2 
(a) 1  2  (b) 1  2  (c) 1  2  (d) 1  2 
 c   c   c   c 
Ans: (a)
Solution: Area of disc from S frame is 1 i.e.  a 2  1 or  a  a  1
u2 u2 u2
Area of disc from S  frame is  a  b   a  a 1   1  1   1 
c2 c2 c2
u2
where b  a 1  .
c2

H.No. 40‐D, Ground Floor, Jia Sarai, Near IIT, Hauz Khas, New Delhi‐110016 
Phone: 011‐26865455/+91‐9871145498
Website: www.physicsbyfiziks.com  | Email: fiziks.physics@gmail.com  
14 
fiziks
Institute for NET/JRF, GATE, IIT‐JAM, M.Sc. Entrance, JEST, TIFR and GRE in Physics 
 
Q30. A planet of mass m and an angular momentum L moves in a circular orbit in a potential,
V r   k / r , where k is a constant. If it is slightly perturbed radially, the angular
frequency of radial oscillations is

(a) mk 2 / 2 L3 (b) mk 2 / L3 (c) 2mk 2 / L3 (d) 3mk 2 / L3


Ans: (b)
Veff
L2 k Veff L2 k
Solution: Veff   . For circular orbit    2 0
2mr 2
r r mr 3
r

L2 k L2 k
  2 . Thus r  r0    ,
mr 3
r mk m
r
d 2Veff 3L 2
2k 3L 2
2k 3m k 3
2m k
4
m k43 4 3
k   3      6
dr 2
mr 4
r r  r0  L2 
4
 L2 
3
L6 L6 L
r  r0
m   
 mk   mk 

d 2V
dr 2 r  r0 mk 2
   .
m L3
Q31. The number of degrees of freedom of a rigid body in d space-dimensions is
(a) 2d (b) 6 (c) d d  1 / 2 (d) d!
Ans: (c)
Q32. A system is governed by the Hamiltonian
1
 p x  ay 2  1  p x  bx 2
H 
2 2
where a and b are constants and p x , p y are momenta conjugate to x and y respectively.

For what values of a and b will the quantities  p x  3 y  and  p y  2 x  be conserved?

(a) a  3, b  2 (b) a  3, b  2


(c) a  2, b  3 (d) a  2, b  3
Ans: (d)
Solution: Poisson bracket  px  3 y, H   0 and
 p y  2 y , H   0
p y (b  3)  x(3b  b 2 )  0 and p x ( a  2)  y (2 a  a 2 )  0

 a  2, b  3

H.No. 40‐D, Ground Floor, Jia Sarai, Near IIT, Hauz Khas, New Delhi‐110016 
Phone: 011‐26865455/+91‐9871145498
Website: www.physicsbyfiziks.com  | Email: fiziks.physics@gmail.com  
15 
fiziks
Institute for NET/JRF, GATE, IIT‐JAM, M.Sc. Entrance, JEST, TIFR and GRE in Physics 
 
Q33. The Lagrangian of a particle of mass m moving in one dimension is given by
1 2
L mx  bx
2
where b is a positive constant. The coordinate of the particle xt  at time t is given by: (in
following c1 and c 2 are constants)
b 2
(a)  t  c1t  c 2 (b) c1t  c 2
2m
 bt   bt   bt   bt 
(c) c1 cos   c 2 sin   (d) c1 cosh   c 2 sinh  
m m m m
Ans: (a)
d  L  L d
Solution: Equation of motion    0  mx   b  0  mx  b  0  mx  b
dt  x  x dt

d 2x b dx b b t2
     t  c1  x    c1t  c2
dt 2 m dt m m 2

NET/JRF (DEC-2013)
Q34. Let A, B and C be functions of phase space variables (coordinates and momenta of a
mechanical system). If ,  represents the Poisson bracket, the value of
A, B, C  A, B, C is given by
(a) 0 (b) B, C , A (c) A, C , B (d) C , A, B
Ans: (d)
Solution: We know that Jacobi identity equation
A, B, C  B, C , A  C , A, B  0
Now A, B, C   A, B, C   B, C , A  C , A, B

z2
Q35. A particle moves in a potential V  x 2  y 2  . Which component(s) of the angular
2
momentum is/are constant(s) of motion?
(a) None (b) Lx , L y and L z (c) only L x and Ly (d) only L z

Ans: (d)

H.No. 40‐D, Ground Floor, Jia Sarai, Near IIT, Hauz Khas, New Delhi‐110016 
Phone: 011‐26865455/+91‐9871145498
Website: www.physicsbyfiziks.com  | Email: fiziks.physics@gmail.com  
16 
fiziks
Institute for NET/JRF, GATE, IIT‐JAM, M.Sc. Entrance, JEST, TIFR and GRE in Physics 
 
z2
Solution: A particle moves in a potential V  x 2  y 2 
2
r 2
V r ,  ,    r 2 sin 2  cos 2   r 2 sin 2  sin 2   cos 2 
2
r 2
V r ,  ,    r 2 sin 2   cos 2 
2
Now  is cyclic-co-ordinate  p  i.e Lz is constant of motion.

Q36. The Hamiltonian of a relativistic particle of rest mass m and momentum p is given

by H  p 2  m 2  V x  , in units in which the speed of light c  1 . The corresponding


Lagrangian is

(a) L  m 1  x 2  V  x  (b) L  m 1  x 2  V  x 

(c) L  1  mx 2  V  x  mx  V  x 


1 2
(d) L 
2
Ans: (b)
H
Solution: H  p 2  m 2  V x  
p
 x 
1 2p
1

 x p 2  m 2 
1/ 2
p
2
p 2
m 
2 2


xm
 p
1  x 2

Now L   xp  H  xp  H  xp


  p 2  m2  V  x 


xm
Put value p   L  m 1  x 2  V  x 
1  x 2

Q37. A pendulum consists of a ring of mass M and radius R suspended by a massless rigid
rod of length l attached to its rim. When the pendulum oscillates in the plane of the ring,
the time period of oscillation is

lR 2
(a) 2
g
(b) l 2
 R2 1/ 4

2 R 2  2 Rl  l 2 2
(c) 2
g R  l 
(d) 2R 2
 2 Rl  l 2  1/ 4

g
Ans: (c)
Solution: The moment of inertia about pivotal point is given by

H.No. 40‐D, Ground Floor, Jia Sarai, Near IIT, Hauz Khas, New Delhi‐110016 
Phone: 011‐26865455/+91‐9871145498
Website: www.physicsbyfiziks.com  | Email: fiziks.physics@gmail.com  
17 
fiziks
Institute for NET/JRF, GATE, IIT‐JAM, M.Sc. Entrance, JEST, TIFR and GRE in Physics 
 
I  I c.m  Md  MR  M (l  R)
2 2 2

If ring is displaced by angle  then potential energy is  Mg (l  R ) cos 


The Lagrangian is given by
1 2 1
L I  V ( ) = ( MR 2  M (l  R) 2 ) 2  Mg (l  R) cos 
2 2
d  L   L  2 
   0  ( MR  M (l  R) )  Mg (l  R ) sin   0
2

dt      

For small oscillation sin     ( MR 2  M (l  R) 2 )  Mg (l  R)  0

2 R 2  2 Rl  l 2
Time period is given by 2 .
g R  l 
Q38. Consider a particle of mass m attached to two identical springs y
each of length l and spring constant k (see the figure). The
equilibrium configuration is the one where the springs are
o
unstretched. There are no other external forces on the system. If the
particle is given a small displacement along the x -axis, which of
the following describes the equation of motion for small x
o
oscillations?
kx 3 kx 2
(a) mx  0 (b) mx  kx  0 (c) mx  2kx  0 (d) mx  0
l2 l
Ans: (a)
Solution: The lagrangian of system is given by y
1 2
L mx  V ( x)
2 x
o
The potential energy is given by
2 2
k 1
 k 1

2

V ( x)   x 2  l 2  2

 l    x2  l 2  2  l
 2  o x
2
 1


V ( x)  k  x 2  l 2  2  l
 
For small oscillation one can approximate potential by Taylor expansion

H.No. 40‐D, Ground Floor, Jia Sarai, Near IIT, Hauz Khas, New Delhi‐110016 
Phone: 011‐26865455/+91‐9871145498
Website: www.physicsbyfiziks.com  | Email: fiziks.physics@gmail.com  
18 
fiziks
Institute for NET/JRF, GATE, IIT‐JAM, M.Sc. Entrance, JEST, TIFR and GRE in Physics 
 
2
 1
 2
2 
 x 
2 2
  1 x 2 1 x 4  
V ( x)  kl  1  2   1  V ( x)  kl 1  2  4   1
2
 
 l   2 l 8 l  
 
2
kl 2  x   x 4
2
V ( x)     V ( x )  k  2 .
4  l2   4l 

1 2  x 4
So Lagrangian of system is given by L  mx  k  2 
2  4l 

d  L   L  kx 3
The Lagranges equation of motion      0  m
x   0.
dt  x   x  l2

NET/JRF (JUNE-2014)

Q39. The time period of a simple pendulum under the influence of the acceleration due to
gravity g is T . The bob is subjected to an additional acceleration of magnitude 3 g in
the horizontal direction. Assuming small oscillations, the mean position and time period
of oscillation, respectively, of the bob will be
(a) 0 o to the vertical and 3T (b) 30 o to the vertical and T / 2

(c) 60 o to the vertical and T / 2 (d) 0 o to the vertical and T / 3


Ans: (c)
l
Solution: T  2
g  T

g   3g 2  g 2  4g 2  2 g
3g
l l 1 T 
T   2  T   2   T  g
2g g 2 2 g

T cos  mg , T sin   3 mg  tan   3    60o

H.No. 40‐D, Ground Floor, Jia Sarai, Near IIT, Hauz Khas, New Delhi‐110016 
Phone: 011‐26865455/+91‐9871145498
Website: www.physicsbyfiziks.com  | Email: fiziks.physics@gmail.com  
19 
fiziks
Institute for NET/JRF, GATE, IIT‐JAM, M.Sc. Entrance, JEST, TIFR and GRE in Physics 
 
1 
Q40. A particle of mass m and coordinate q has the Lagrangian L  mq 2  qq 2 , where 
2 2
is a constant. The Hamiltonian for the system is given by
p 2 qp 2 p2
(a)  (b)
2m 2 m 2 2m  q 

p2 qp 2 pq
(c)  (d)
2m 2m  q 2 2

Ans: (b)
1 2  2
Solution: H   qp
  L where L  mq  qq
2 2
L p
 p  mq   qq  p  q  m   q   q 
q m  q

 H  qp
 L 
p2 1
 m
p2

 
q 
p2
 m   q  2  m   q 2 2  m   q 2
p2 p2
 H  qp
 L    m  q 
 m   q  2  m   q 2
p2 p2 p2
 H  qp
 L   H 
 m  q  2  m  q  2 m  q 

Q41. The coordinates and momenta xi , pi i  1, 2, 3 of a particle satisfy the canonical Poisson

bracket relations xi , p j    ij . If C1  x2 p3  x3 p2 and C2  x1 p2  x2 p1 are constants of

motion, and if C3  C1 , C 2   x1 p3  x3 p1 , then

(a) C2 , C3   C1 and C3 , C1  C2

(b) C2 , C3   C1 and C3 , C1  C2

(c) C2 , C3   C1 and C3 , C1   C2

(d) C2 , C3   C1 and C3 , C1  C2

Ans. : (d)
Solution: C1  x2 p3  x3 p2 , C2  x1 p2  x2 p1 , C3  x1 p3  x3 p1

H.No. 40‐D, Ground Floor, Jia Sarai, Near IIT, Hauz Khas, New Delhi‐110016 
Phone: 011‐26865455/+91‐9871145498
Website: www.physicsbyfiziks.com  | Email: fiziks.physics@gmail.com  
20 
fiziks
Institute for NET/JRF, GATE, IIT‐JAM, M.Sc. Entrance, JEST, TIFR and GRE in Physics 
 
 C C C C   C C C C   C C C C 
C2 , C3    2 3  2 3    2 3  2 3    2 3  2 3 
 x1 p1 p1 x1   x2 p2 p2 x2   x3 p3 p3 x3 

C2 , C3    p2 x3    x2  p3    0  x1  0    0  x1  0  p1    p2 x3  x2 p3   C1
 C3 C1 C3 C1   C3 C1 C3 C1   C3 C1 C3 C1 
C3 , C1        
 x1 p1 p1 x1   x2 p2 p2 x2   x3 p3 p3 x3 

C3 , C1   p3  0  x3  0    0  x3  0  p3    p1 x2  x1 p2     x1 p2  x2 p1   C2


Q42. The recently-discovered Higgs boson at the LHC experiment has a decay mode into a
photon and a Z boson. If the rest masses of the Higgs and Z boson are 125 GeV/c2 and

90 GeV/c2 respectively, and the decaying Higgs particle is at rest, the energy of the
photon will approximately be
(a) 35 3 GeV (b) 35 GeV (c) 30 GeV (d) 15 GeV
Ans. : (c)
Solution: H B  PH  Z B
   
From conservation of momentum 0  P1  P2  P1   P2  P1  P2

Now EH B  EPH  EZ B  EPH  EZ B  M H B c 2

E P2H  P12 c 2  0 and EZ2B  P22 c 2  M Z2B c 4


 EZ B  EPH  E ZB 
 EPH  M Z2B c 4  P1  P2

M Z2B c 4 M Z2B c 2
 EZ B  EPH    EZ B  EPH  M H B c 2
M HB c2 M HB

 2 EPH  M H B c  2
M z2B c 2
 EPH 
M 2
HB 
 M z2B c 2
M HB M HB

 125 125  90  90  c
4
 EPH    4  30.1GeV

 2 125  c
Q43. A canonical transformation relates the old coordinates q, p  to the new ones Q, P  by

the relations Q  q 2 and P  p / 2q . The corresponding time independent generating


function is
(a) P / q 2 (b) q 2 P (c) q 2 / P (d) qP 2
H.No. 40‐D, Ground Floor, Jia Sarai, Near IIT, Hauz Khas, New Delhi‐110016 
Phone: 011‐26865455/+91‐9871145498
Website: www.physicsbyfiziks.com  | Email: fiziks.physics@gmail.com  
21 
fiziks
Institute for NET/JRF, GATE, IIT‐JAM, M.Sc. Entrance, JEST, TIFR and GRE in Physics 
 
Ans. : (b)
Solution: Q  q 2 ; P  p / 2q
F2 F
 p  2  P  2q  F2  q 2 P  f ( P)
q q
F2
 Q  q 2  F2  q 2 P  f (q)
P
comparing both side f (q )  f ( P)  0  F2  q 2 P

NET/JRF (DEC-2014)
Q44. The equation of motion of a system described by the time-dependent Lagrangian
1 
L  e t  mx 2  V  x   is
2 
dV dV
(a) mx  mx  0 (b) mx  mx  0
dx dx
dV dV
(c) mx  mx  0 (d) mx  0
dx dx
Ans: (a)
1  L L V  t
Solution:  L  e t  mx 2  V  x     e t mx and  e
2  x x x
d  L  L d V  t V  t
     0   e t mx     t  mx e t 
e  mxe e 0
dt  x  x dt x x

 V   t V
 mx  m x   e  0  mx   mx  0
x x

A particle of mass m is moving in the potential V  x    ax 2  bx 4 where a, b are


1 1
Q45.
2 4
positive constants. The frequency of small oscillations about a point of stable equilibrium
is
(a) a/m (b) 2a / m (c) 3a / m (d) 6a / m
Ans: (b)
1 1
Solution: V  x    ax 2  bx 4
2 4

H.No. 40‐D, Ground Floor, Jia Sarai, Near IIT, Hauz Khas, New Delhi‐110016 
Phone: 011‐26865455/+91‐9871145498
Website: www.physicsbyfiziks.com  | Email: fiziks.physics@gmail.com  
22 
fiziks
Institute for NET/JRF, GATE, IIT‐JAM, M.Sc. Entrance, JEST, TIFR and GRE in Physics 
 
1
V  a 2
 0  ax  bx3  0  x  a  bx 2   0  x     , 0
x b
 2V  2V
  a  3bx 2  At x  0,  a (Negative so it is unstable point)
x 2
x 2
 2V a
  a  3b  2a (Positive so it is stable point)
x 2 b
1
a 2
x   
 b

 2V
  x 2  2a
m m
Q46. The radius of Earth is approximately 6400 km . The height h at which the acceleration
due to Earth’s gravity differs from g at the Earth’s surface by approximately 1 % is
(a) 64 km (b) 48 km (c) 32 km (d) 16 km
Ans: (c)
g 2h g 2h g 2h
Solution:  1  1     h  32 k .m.
g R g R g R
Q47. According to the special theory of relativity, the speed v of a free particle of mass m and
total energy E is:

mc 2 2 E  mc 2 
(a) v  c 1  (b) v  1  
E m  E 
2
 mc 2   mc 2 
(c) v  c 1    (d) v  c1  
 E   E 

Ans: (c)
2 2
v 2  mc 2 
mc 2 v2 m2c 4  mc 2 
Solution: E   1 2      1   v  c 1   
v2 c  E  c2 E2  E 
1 2
c

H.No. 40‐D, Ground Floor, Jia Sarai, Near IIT, Hauz Khas, New Delhi‐110016 
Phone: 011‐26865455/+91‐9871145498
Website: www.physicsbyfiziks.com  | Email: fiziks.physics@gmail.com  
23 
fiziks
Institute for NET/JRF, GATE, IIT‐JAM, M.Sc. Entrance, JEST, TIFR and GRE in Physics 
 
p2
Q48. The Hamiltonian of a classical particle moving in one dimension is H   q 4 where
2m
 is a positive constant and p and q are its momentum and position respectively. Given
that its total energy E  E 0 the available volume of phase space depends on E 0 as

(a) E 03 / 4 (b) E 0

(c) E0 (d) is independent of E 0

Ans: (a)
V q
p2
Solution: H    q4
2m E0
Phase area   p  dq q
p
1 2mE0
E 4
A   p  dq   2mE   
    E0 / 
1/4  E0 / 1/4
AE 1/2
E
1/4
 AE 3/4  2mE0
0 0 0

p2 1
Q49. A mechanical system is described by the Hamiltonian H q, p    m 2 q 2 . As a
2m 2
Q
result of the canonical transformation generated by F q, Q    , the Hamiltonian in
q
the new coordinate Q and momentum P becomes

1 2 2 m 2 2 1 2 2 m 2 2
(a) Q P  Q (b) Q P  P
2m 2 2m 2
1 2 m 2 2 1 2 4 m 2  2
(c) P  Q (d) Q P  P
2m 2 2m 2
Ans: (d)
p2 1 Q
Solution: H   m 2 q 2 , F  F1  q, Q   
2m 2 q
F1 Q
 p  2 p (i)
q q
F1 1 1
  P    P  q  (ii)
Q q P

H.No. 40‐D, Ground Floor, Jia Sarai, Near IIT, Hauz Khas, New Delhi‐110016 
Phone: 011‐26865455/+91‐9871145498
Website: www.physicsbyfiziks.com  | Email: fiziks.physics@gmail.com  
24 
fiziks
Institute for NET/JRF, GATE, IIT‐JAM, M.Sc. Entrance, JEST, TIFR and GRE in Physics 
 
1
From equation (i) and (ii)  p  QP 2 q 
P
p2 1 Q2 P4 1  1  1 2 4 1
H  m q 
2 2
 m 2  2   Q P  m 2 P 2
2m 2 2m 2  P  2m 2
Q50. The probe Mangalyaan was sent recently to explore the planet Mars. The inter-planetary
part of the trajectory is approximately a half-ellipse with the Earth (at the time of launch),
Sun and Mars (at the time the probe reaches the destination)
forming the major axis. Assuming that the orbits of Earth
and Mars are approximately circular with radii RE and Sun
Earth Mars
RM , respectively, the velocity (with respect to the Sun) of RE
RM
the probe during its voyage when it is at a distance
r RE  r  RM  from the Sun, neglecting the effect of Earth and Mars, is

R E  RM  R E  RM  r 
(a) 2GM (b) 2GM
r R E  R M  r  r R E  RM 

RE 2GM
(c) 2GM (d)
rRM r

Ans: (b)
Solution: Total energy E   K / 2a where 2a major axis and 2a  RE  RM .

1 2 GMm GMm  R  RM  r 
 v  2GM E
mv  
2 r  RE  RM  r  RE  RM 

H.No. 40‐D, Ground Floor, Jia Sarai, Near IIT, Hauz Khas, New Delhi‐110016 
Phone: 011‐26865455/+91‐9871145498
Website: www.physicsbyfiziks.com  | Email: fiziks.physics@gmail.com  
25 
fiziks
Institute for NET/JRF, GATE, IIT‐JAM, M.Sc. Entrance, JEST, TIFR and GRE in Physics 
 
NET/JRF (JUNE-2015)
Q51. A particle moves in two dimensions on the ellipse x 2  4 y 2  8 . At a particular instant it

is at the point  x, y    2,1 and the x -component of its velocity is 6 (in suitable units).

Then the y -component of its velocity is


(a) 3 (b) 2 (c) 1 (d) 4
Ans. (a)
dx dy
Solution:  x 2  4 y 2  8  2 x  8y 0
dt dt
 2 xvx  8 y v y  0  2  2  6  8 1 v y  0  v y  3

Q52. Consider three inertial frames of reference A, B and C . the frame B moves with a
c c
velocity with respect to A , and C moves with a velocity with respect to B in the
2 10
same direction. The velocity of C as measured in A is

3c 4c c 3c
(a) (b) (c) (d)
7 7 7 7
Ans. (b)
c c A S B S C u x
Solution: v  , u x 
2 10
C/2 C /10
u  v 4c
ux  x 
u vx 7
1 2
c
1 2
Q53. If the Lagrangian of a dynamical system in two dimensions is L  mx  mxy
 , then its
2
Hamiltonian is
1 1 2 1 1 2
(a) H  px p y  py (b) H  px p y  px
m 2m m 2m
1 1 2 1 1 2
(c) H  px p y  py (d) H  px p y  px
m 2m m 2m
Ans. (c)
1 2 L
Solution: L  mx  mxy
   mx  my  px (i)
2 x

H.No. 40‐D, Ground Floor, Jia Sarai, Near IIT, Hauz Khas, New Delhi‐110016 
Phone: 011‐26865455/+91‐9871145498
Website: www.physicsbyfiziks.com  | Email: fiziks.physics@gmail.com  
26 
fiziks
Institute for NET/JRF, GATE, IIT‐JAM, M.Sc. Entrance, JEST, TIFR and GRE in Physics 
 
L p
  mx  p y or x  y (ii)
y m
py px  p y
put x  in equation (i)  p y  my  px  y 
m m
1
H  px x  p y y  L  px x  p y y  mx 2  mxy

2
px p y p y2
put value of x and y  H  
m 2m
 
Q54. A particle of mass m moves in the one dimensional potential V  x   x3  x 4 where
3 4
 ,   0 . One of the equilibrium points is x  0 . The angular frequency of small
oscillations about the other equilibrium point is
2   
(a) (b) (c) (d)
3m m 12m 24m
Ans. (b)
  V 
Solution: V  x   x3  x4    x 2   x3  0  x0  
3 4 x 

 2V 2 k 
Spring constant k  2   ve     
x x  x0
 m m

Q55. A particle of unit mass moves in the xy -plane in such a way that x  t   y  t  and

y  t    x  t  . We can conclude that it is in a conservative force-field which can be

derived from the potential

(a)
2

1 2
x  y2  (b)
1 2
2

x  y2  (c) x  y (d) x  y

Ans. (a)
Solution:  x  y and y   x
 x  y   x
 y   x   y
and 
 xx 0
 y y 0
and 

that is possible for L 


1 2 1 2 1 2
2
1
mx  my  x  y 2  V  x 2  y 2
2 2 2
   

H.No. 40‐D, Ground Floor, Jia Sarai, Near IIT, Hauz Khas, New Delhi‐110016 
Phone: 011‐26865455/+91‐9871145498
Website: www.physicsbyfiziks.com  | Email: fiziks.physics@gmail.com  
27 
fiziks
Institute for NET/JRF, GATE, IIT‐JAM, M.Sc. Entrance, JEST, TIFR and GRE in Physics 
 
1
Q56. A particle moves in one dimension in the potential V  k  t  x 2 , where k  t  is a time
2
d
dependent parameter. Then V , the rate of change of the expectation value V of the
dt
potential energy is
1 dk 2 k 1 dk 2 1
(a) x  xp  px (b) x  p2
2 dt 2m 2 dt 2m
k 1 dk 2
(c) xp  px (d) x
2m 2 dt
Ans. (a)
p2 1
Solution: H   k t  x2
2m 2

d V 1 2 p
2
1 2 x 2 k
V  V , H     k t  x ,  k t  x    V , H 
dt t 2 2m 2  2 t

d 1 xp  px x 2 k x 2 k 1
V  k t   2    k  t  xp  px
dt 2 2m 2 t 2 t 2m

Q57. Let q and p be the canonical coordinate and momentum of a dynamical system. Which
of the following transformations is canonical?
1 2 1 2
1. Q1  q and P1  p
2 2
1 1
2. Q2   p  q  and P2   p  q 
2 2
(a) neither 1 nor 2 (b) both 1 and 2
(c) only 1 (d) only 2
Ans. (d)
q2 P2
Solution: For A : Q1  , P1 
2 2
Q P Q P
Q1 , P1   1 . 1  1 . 1  1 (Not canonical)
q p p q
1 1
For B : Q2   p  q  , P2   p  q 
2 2

Q2 , p2   1 (canonical)

H.No. 40‐D, Ground Floor, Jia Sarai, Near IIT, Hauz Khas, New Delhi‐110016 
Phone: 011‐26865455/+91‐9871145498
Website: www.physicsbyfiziks.com  | Email: fiziks.physics@gmail.com  
28 
fiziks
Institute for NET/JRF, GATE, IIT‐JAM, M.Sc. Entrance, JEST, TIFR and GRE in Physics 
 
Q58. Which of the following figures is a schematic representation of the phase space
trajectories (i.e., contours of constant energy) of a particle moving in a one-dimensional
1 2 1 4
potential V  x   x  x p
2 4
p

(a) (b)
x
x

p p

(c) (d)
x x

Ans. (a)
 x2 x4 V  x
Solution: V  x   
2 4
V x
 0  x  0, x  1
x
 2V
 ve for x  0 (unstable point)
x 2 E0
= + ve for x  1 (stable point)
E0 E0

H.No. 40‐D, Ground Floor, Jia Sarai, Near IIT, Hauz Khas, New Delhi‐110016 
Phone: 011‐26865455/+91‐9871145498
Website: www.physicsbyfiziks.com  | Email: fiziks.physics@gmail.com  
29 
fiziks
Institute for NET/JRF, GATE, IIT‐JAM, M.Sc. Entrance, JEST, TIFR and GRE in Physics 
 
NET/JRF (DEC-2015)
Q59. Two masses m each, are placed at the points  x, y    a, a  and   a,  a  and two

masses, 2m each, are placed at the points  a,  a  and  a, a  . The principal moments of

inertia of the system are


(a) 2m 2 , 4ma 2 (b) 4ma 2 ,8ma 2 (c) 4ma 2 , 4ma 2 (d) 8ma 2 ,8ma 2
Ans.: (b)

 
Solution: I xx   mi yi2  zi2   mi yi2  zi  0
i

 I xx  ma 2  ma 2  2ma 2  2ma 2  I xx  6ma 2

Similarly, I yy  6ma 2 and I zz  12ma 2

I xz  I zx  0, I yz  I zy  0

I xy  I yx   mi  xi yi   ma 2  ma 2  2ma 2  2ma 2  I xy  I yx  2ma 2


i

Moment of inertia tensor


 6ma 2 2ma 2 0 
 
I   2ma 2 6ma 2
0 
 0 0 12ma 2 

Eigen value of matrices is principal moment of inertia, which is given by
1  4ma 2  I x , 2  8ma 2  I y , 3  12ma 2  I z

So, I x  4ma 2 and I y  8ma 2

Q60. The Lagrangian of a system is given by


1 2 5 
L mq1  2mq22  k  q12  2q22  2q1q2 
2 4 
where m and k are positive constants. The frequencies of its normal modes are

(a)
k
,
2m m
3k
(b)
k
2m

13  73 
5k k k 6k
(c) , (d) ,
2m m 2m m
Ans.: (a)

H.No. 40‐D, Ground Floor, Jia Sarai, Near IIT, Hauz Khas, New Delhi‐110016 
Phone: 011‐26865455/+91‐9871145498
Website: www.physicsbyfiziks.com  | Email: fiziks.physics@gmail.com  
30 
fiziks
Institute for NET/JRF, GATE, IIT‐JAM, M.Sc. Entrance, JEST, TIFR and GRE in Physics 
 
1 5 
Solution: L  mq12  2mq22  k  q12  2q22  2q1 q2 
2 4 
1 2 4 2 k 10 2 
L mq1  mq2   q1  4q22  2q1 q2  2q2 q1 
2 2 2 4 

 10 
m 0  k 2 k 
T  , V  4
 0 4m   
 2k 4k 

The secular equation V   2 m  0

 10 
 4 k  m 2 k   0 ,  10 k   2 m  4k  4 2 m  4k 2  0
2

  
 4


 
 2 k 4k   4m 
2

 10k 2  10 2 km  4 2 km  4 4 m 2  4k 2  0
 3k 2  7 2 km  2 4 m 2  0  3k 2  6 2 km   2 km  2 4 m 2  0

k 3k
  
 k  2 2 m 3k   2 m  0   
2m
, 
m
Q61. Consider a particle of mass m moving with a speed v . If TR denotes the relativistic

kinetic energy and TN its non-relativistic approximation, then the value of


TR  TN  for
TR
v  0.01 c , is

(a) 1.25  105 (b) 5.0  105 (c) 7.5  105 (d) 1.0  104
Ans.: None of the options is correct.
1 m0 c 2
Solution: TN  m0 v 2 , TR  mc 2  m0 c 2   m0 c 2 ( v  0.01c )
2 v 2
1 2
c
 0.01
2
1 v2
TR  TN  m0 v 2
T 2 2 2
Now,  1 N  1  1  1
TR TR m0 c 2 c2 1
1
 m0 c 2
c 2

1   0.01
2
v 2
v 2
1 2 1 2
c c
TR  TN
 0.75
TR

H.No. 40‐D, Ground Floor, Jia Sarai, Near IIT, Hauz Khas, New Delhi‐110016 
Phone: 011‐26865455/+91‐9871145498
Website: www.physicsbyfiziks.com  | Email: fiziks.physics@gmail.com  
31 
fiziks
Institute for NET/JRF, GATE, IIT‐JAM, M.Sc. Entrance, JEST, TIFR and GRE in Physics 
 
Q62. A canonical transformation  p, q    P, Q  is performed on the Hamiltonian

p2 1 1
H  m 2 q 2 via the generating function, F  m q 2 cot Q . If Q  0   0 , which
2m 2 2
of the following graphs shows schematically the dependence of Q  t  on t ?

(a) (b)

Q t  Q t 

(c) (d)
Q t  Q t 

Ans.: (d)
p2 1 1
Solution: H   m 2 q 2 , F1  m q 2 cot Q
2m 2 2
F1 F1 F1
 p,   P, KH
q Q t  t 
F1
 p  m q cot Q …..(i)
q
t
F1 1
  P   m q 2 cosec 2Q   P
Q 2
1 P
 m q 2  …..(ii)
2 cosec 2Q
From (i) and (ii)
p  2m P cos Q
F1 F1
KH ;  0
t t
p2 1
KH   m 2 q 2 put the value of p and q
2m 2
 K  P using equation of motion Q and P

H.No. 40‐D, Ground Floor, Jia Sarai, Near IIT, Hauz Khas, New Delhi‐110016 
Phone: 011‐26865455/+91‐9871145498
Website: www.physicsbyfiziks.com  | Email: fiziks.physics@gmail.com  
32 
fiziks
Institute for NET/JRF, GATE, IIT‐JAM, M.Sc. Entrance, JEST, TIFR and GRE in Physics 
 
K
  P  0  P  0  P  constant
Q
K  K
Q    Q  Q  constant  P  constant 
P P
Q    Q  t    from boundary condition   0 
 Q  t
Therefore, option (d) is correct.
Q63. The Lagrangian of a particle moving in a plane s given in Cartesian coordinates as
L  xy
  x 2  y 2

In polar coordinates the expression for the canonical momentum pr (conjugate to the
radial coordinate r ) is
(a) r sin   r cos  (b) r cos   r sin 
(c) 2r cos   r sin 2 (d) r sin 2  r cos 2
Ans.: (d)
Solution: L  xy
  x 2  y 2  xy 
  x 2  y 2 
x  r cos  , y  r sin   x  r cos   r sin , y  r sin   r cos  

L  r 2 sin  cos   r 2 sin  cos   2  r r cos 2   rr


 sin 2 
L
Pr 
r

 2r sin  cos   r cos 2   sin 2  
 Pr  r sin 2  r cos 2

H.No. 40‐D, Ground Floor, Jia Sarai, Near IIT, Hauz Khas, New Delhi‐110016 
Phone: 011‐26865455/+91‐9871145498
Website: www.physicsbyfiziks.com  | Email: fiziks.physics@gmail.com  
33 
fiziks
Institute for NET/JRF, GATE, IIT‐JAM, M.Sc. Entrance, JEST, TIFR and GRE in Physics 
 
NET/JRF (JUNE-2016)
Q64. Let  x, t  and  x, t   be the coordinate systems used by the observers O and O ,

respectively. Observer O moves with a velocity v   c along their common positive x -


axis. If x  x  ct and x  x  ct are the linear combinations of the coordinates, the

Lorentz transformation relating O and O takes the form

x   x x   x 1  1 
(a) x  and x  (b) x  x and x  x
1  2
1  2 1  1 

x   x x   x 1  1 
(c) x  and x  (d) x  x and x  x
1  2 1  2 1  1 

Ans: (d)
Solution: x  x  ct 

 vx   v  v v v v
ct  2  x 1   ct 1   1 1 1
x  vt c  c c
        x c  ct c  c  x  ct 
v2 v2 v 2
v 2 v v v
1 2 1 2 1 2 1 2 1 1 1
c c c c c c c

1 
x  x
1 
 vx   v  v
ct  2  x  1   ct  1  
x  vt c   c  c
x  x  ct     
v2 v2 v2 v2
1 2 1 2 1 2 1 2
c c c c

v v
1 1
x  x c  ct c  x  1    x  ct   x  1   x
  
1
v
1
v 1  1 
c c
Q65. A ball of mass m , initially at rest, is dropped from a height of 5 meters. If the coefficient
of restitution is 0.9 , the speed of the ball just before it hits the floor the second time is
approximately (take g  9.8 m / s 2 )
(a) 9.80 m / s (b) 9.10 m / s (c) 8.91 m / s (d) 7.02 m / s
Ans: (c)
Solution: velocity just before hitting first time is
H.No. 40‐D, Ground Floor, Jia Sarai, Near IIT, Hauz Khas, New Delhi‐110016 
Phone: 011‐26865455/+91‐9871145498
Website: www.physicsbyfiziks.com  | Email: fiziks.physics@gmail.com  
34 
fiziks
Institute for NET/JRF, GATE, IIT‐JAM, M.Sc. Entrance, JEST, TIFR and GRE in Physics 
 
v1  2 gh  2  9.8  5  9.89 m / s

After hitting velocity will be  ev1  0.9  9.89

v2  8.9 m / s

velocity hitting before second time will be same as v2

Q66. The Hamiltonian of a system with generalized coordinate and momentum  q, p  is

H  p 2 q 2 . A solution of the Hamiltonian equation of motion is (in the following A and


B are constants)
A 2 At A 2 At
(a) p  Be 2 At , q e (b) p  Ae 2 At , q e
B B
A  At A A2t
(c) p  Ae At , q (d) p  2 Ae  A t , q
2
e e
B B
Ans: (a)
Solution: H  p 2 q 2
From Hamilton’s equation
H dp
  p   2 p 2 q (i)
q dt
H dq
 q   2 pq 2 (ii)
p dt
from equations (i) and (ii)
dp dq

p q
Integrating both sides, ln p   ln q  ln A
pq  A (iii)
from equation (i)
dp
 2 p 2 q  2 pA
dt
dp p
 p   2 Adt  ln B  ln B  2 At  p  Be
2 At

A 2 At
Putting this value of p in equation (iii) gives q  e
B
Hence, the correct option is (a)

H.No. 40‐D, Ground Floor, Jia Sarai, Near IIT, Hauz Khas, New Delhi‐110016 
Phone: 011‐26865455/+91‐9871145498
Website: www.physicsbyfiziks.com  | Email: fiziks.physics@gmail.com  
35 
fiziks
Institute for NET/JRF, GATE, IIT‐JAM, M.Sc. Entrance, JEST, TIFR and GRE in Physics 
 
Q67. A canonical transformation  q, p    Q, P  is made through the generating function

F  q, P   q 2 P on the Hamiltonian

p2 
H  q, p    q4
2 q 2
4

where  and  are constants. The equations of motion for  Q, P  are

P 4P  Q
(a) Q  and P    Q (b) Q  and P 
  2
P 2P 2 2P
(c) Q  and P    Q (d) Q  and P    Q
 Q 
Ans.: (b)
Solution: F  q, P   q 2 P

This is F2 type generating function so

F2 F
 p & 2 Q
q P
1 1
p  2qP & Q  q 2  q   Q  2 & p  2  Q  2 P

4QP 2  2 2 P 2  Q 2
H  Q, P    Q  
2 Q 4  4
H  4P H Q
  Q  Q  and   P  P  
P  Q 2
Q68. The Lagrangian of a system moving in three dimensions is

mx1  m  x22  x32   kx12  k  x2  x3 


1 2 1 1 2
L
2 2 2
The independent constants of motion is/are
(a) energy alone
(b) only energy, one component of the linear momentum and one component of the
angular momentum
(c) only energy, one component of the linear momentum
(d) only energy, one component of the angular momentum
Ans. : (a)

H.No. 40‐D, Ground Floor, Jia Sarai, Near IIT, Hauz Khas, New Delhi‐110016 
Phone: 011‐26865455/+91‐9871145498
Website: www.physicsbyfiziks.com  | Email: fiziks.physics@gmail.com  
36 
fiziks
Institute for NET/JRF, GATE, IIT‐JAM, M.Sc. Entrance, JEST, TIFR and GRE in Physics 
 
Solution: The motion is in 3D . So don’t get confine with x1 , x2 x3 they are actually x, y, z
Langrangian is then
L L L
mx  m  y 2  z 2   kx 2  k  y  z  , when
1 2 1 1 2
L  0,  0, 0
2 2 2 x y z
So, not any component at Linear momentum is conserve.
Now transform the Lagrangian to Hamiltonian
Px2 Py
2
P2 1 1
 z  kx 2  k  y  z 
2
H 
2m 4m 4m 2 2
H
 0 so energy is conserved
t
Now let us assume Lx  yPz  zPy

dLx L
  Lx , H   x
dt t

 Lx , H    yPz  zPy , H    y, H  Pz  y  Pz , H    z, H  Py  z  Py , H 
 Py2   1 2  Pz2   1 2
  Lx , H    y,  Pz  y  Pz , k  y  z     z ,  Py  z  Py , k  y  z  
 4m   2   4m   2 

Pz  1   P   1 
 2 Py  y 0  k .2  y  z     2 Py z   z 0  k .2  y  z  
4m  2   4m   2 

 k  y 2  yz   k  z 2  yz    k  y 2  z 2   k  z 2  y 2 

dLx dLy dL
  0 . Similarly  0 and  z  0
dt dt dt
Q69. For a particle of energy E and momentum p (in a frame F ), the rapidity y is defined
1  E  p3c 
as y  ln   . In a frame F  moving with velocity v   0, 0,  c  with respect to
2  E  p3c 
F , the rapidity y will be
1  1  
(a) y  y  ln 1   2 
1
(b) y  y  ln  
2 2  1  

 1    1  
(c) y  y  ln   (d) y  y  2 ln  
 1    1  
Ans: (b)

H.No. 40‐D, Ground Floor, Jia Sarai, Near IIT, Hauz Khas, New Delhi‐110016 
Phone: 011‐26865455/+91‐9871145498
Website: www.physicsbyfiziks.com  | Email: fiziks.physics@gmail.com  
37 
fiziks
Institute for NET/JRF, GATE, IIT‐JAM, M.Sc. Entrance, JEST, TIFR and GRE in Physics 
 
1  E  p3c 
Solution: y  ln  
2  E  p3c 

1  E   p3c 
Then y  ln  
2  E   p3c 

  E 
Where p3    p3  v  2   E     E  vp3 
  c 

 v 
1   E  p3c    E  p3c  
Put the value of p3 and E  one will get y  ln  c

2  E p c v E p c 
 3   3 
 c 

1   E  p3 c  1     1   E  p3 c   1  1   
ln    ln    ln  
2   E  p3 c  1     2   E  p3 c   2  1   

1  1   1  1  
y  ln    y  ln  
2  1   2  1  
NET/JRF (DEC-2016)
Q70. A ball of mass m is dropped from a tall building with zero initial velocity. In addition to
gravity, the ball experiences a damping force of the form  , where  is its
instantaneous velocity and  is a constant. Given the values m  10 kg ,   10 kg / s and

g  10 m / s 2 the distance travelled (in metres) in time t in seconds, is

(a) 10  t  1  e  t  (b) 10  t  1  e t 

(c) 5t 2  1  et  (d) 5t 2

Ans. : (b)
d2x dx
Solution: m  mg  
dt 2
dt
Putting the values of m,  and g and simplifying we obtain

d 2 x dx
  10
dt 2 dt
The general solution of this equation is x  t   c1  c2t  10e  t

Using the initial conditions x  0   0, x  0   0

H.No. 40‐D, Ground Floor, Jia Sarai, Near IIT, Hauz Khas, New Delhi‐110016 
Phone: 011‐26865455/+91‐9871145498
Website: www.physicsbyfiziks.com  | Email: fiziks.physics@gmail.com  
38 
fiziks
Institute for NET/JRF, GATE, IIT‐JAM, M.Sc. Entrance, JEST, TIFR and GRE in Physics 
 
We obtain, c1  10 and c2  10

Hence the required solution is x  t   10  t  1  e  t 

Q71. A relativistic particle moves with a constant velocity v with respect to the laboratory
frame. In time  , measured in the rest frame of the particle, the distance that it travels in
the laboratory frame is

c v2 v
(a) v (b) (c) v 1  (d)
v 2 c2 v2
1 2 1 2
c c
Ans. : (d)
Solution: From Particle x1  0 x2  0 tinitial  t1 t final  t2

x1  vt1 x2  vt2


x1  , x2  L
1  v2 / c2 1  v2 / c2
v
x2  x1 v  t2  t1 
x2  x1  
1  v2 / c2 1  v2 / c2
v  t2  t1  v
x  
1  v2 / c2 1  v2 / c2
Q72. A particle in two dimensions is in a potential V  x, y   x  2 y . Which of the following

(apart from the total energy of the particle) is also a constant of motion?
(a) p y  2 px (b) px  2 p y

(c) px  2 p y (d) p y  2 px

Ans. : (a)
Solution: V  x, y   x  2 y

px2 p y
2

H   x  2y
2m 2m
d  p y  2 px  
dt
  p y  2 px , H  
t
 p y  2 px 
  p y  2 px , H    p y  2 px , x  2 y    p y , 2 y    2 px , x   2  2  0

H.No. 40‐D, Ground Floor, Jia Sarai, Near IIT, Hauz Khas, New Delhi‐110016 
Phone: 011‐26865455/+91‐9871145498
Website: www.physicsbyfiziks.com  | Email: fiziks.physics@gmail.com  
39 
fiziks
Institute for NET/JRF, GATE, IIT‐JAM, M.Sc. Entrance, JEST, TIFR and GRE in Physics 
 
Q73. The dynamics of a particle governed by the Lagrangian
1 2 1 2
L mx  kx  kxxt
 describes
2 2
(a) an undamped simple harmonic oscillator
(b) a damped harmonic oscillator with a time varying damping factor
(c) an undamped harmonic oscillator with a time dependent frequency
(d) a free particle
Ans. : (d)
1 2 1 2
Solution: L  mx  kx  kx x t
2 2
L L
 mx  kxt ,  kx  kxt

x x
d  L  L
   0  mx  kxt   0  mx  0
  kx  kx  kxt
dt  x  x
So motion is equivalent to free particle
Q74. The parabolic coordinates  ,  are related to the Cartesian coordinates  x, y  by

x   and y  
1 2
2
   2  . The Lagrangian of a two-dimensional simple harmonic

oscillator of mass m and angular frequency  is

m    2   2  2   2  
1  2
(a)
2 
 
(b)
1
2



m  2   2    2   2   2  2   2  
1
4 
 
m  2   2   2   2   2 
1 1
(c)
2  2 
 1 
m  2   2   2   2   2 
1
(d)
2  4 
Ans. : (b)
Solution: For two dimensional Harmonic oscillation

m  x 2  y 2   m 2  x 2  y 2 
1 1
L
2 2

H.No. 40‐D, Ground Floor, Jia Sarai, Near IIT, Hauz Khas, New Delhi‐110016 
Phone: 011‐26865455/+91‐9871145498
Website: www.physicsbyfiziks.com  | Email: fiziks.physics@gmail.com  
40 
fiziks
Institute for NET/JRF, GATE, IIT‐JAM, M.Sc. Entrance, JEST, TIFR and GRE in Physics 
 
y   2   2 
1
x   ,
2
   ,
x   y    

 2 2 1 2 2 2
1  
        12 m   
2 2
L m    2
     
2  4

L
1
2
 
m  2 2   2 2   2 2   2 2  m 2  4   4  2 2 2 
1
8
1
 
m  2   2   2   2  m 2  2   2 
1 2

2 8
 
m  2   2   2   2   2  2   2  
1 1

2  4 
Q75. After a perfectly elastic collision of two identical balls, one of which was initially at rest,
the velocities of both the balls are non zero. The angle  between the final, velocities (in
the lab frame) is
  
(a)   (b)    (c) 0    (d)  
2 2 2
Ans. : (a)
v1
Solution: Angle between two particle 1   2  0
Conservation of momentum u 1
2
mu  mv1 cos 1  mv2 cos  2 (i)
v2
0  mv1 sin 1  mv2 sin  2 (ii)
conservation of kinetic energy
1 1 1
mu 2  mv12  mv22 (iii)
2 2 2
From (i) and (ii)
u 2  v12  v22  2v1v2  cos 1 cos  2  sin 1 sin  2 

u 2  v12  v22  2v1v2 cos 1   2  (iv)

u 2  v12  v22 (v)

v12  v22  v12  v22  2v1v2 cos 1   2 

H.No. 40‐D, Ground Floor, Jia Sarai, Near IIT, Hauz Khas, New Delhi‐110016 
Phone: 011‐26865455/+91‐9871145498
Website: www.physicsbyfiziks.com  | Email: fiziks.physics@gmail.com  
41 
fiziks
Institute for NET/JRF, GATE, IIT‐JAM, M.Sc. Entrance, JEST, TIFR and GRE in Physics 
 
 cos 1   2   0

 
1   2   
2 2
k
Q76. Consider circular orbits in a central force potential V  r    , where k  0 and
rn
0  n  2 . If the time period of a circular orbit of radius R is T1 and that of radius 2 R is

T2
T2 , then
T1
n 2 n
n 1
(a) 2 2 (b) 2 3 (c) 2 2 (d) 2n
Ans. : (c)
J2 k Veff J2 nk
Solution: Veff   ,    n 1  0
2mr 2
r n
r mr 3
r
m 2 2 r 4 nk 1  n  2  / 2
n
1
 J  mr 2   n 1
  2
 n2
   r  T  r 2
r 3
r r
n2
n
T2  2 R  2 1
  2 2
T1  R 
c
Q77. Consider a radioactive nucleus that is travelling at a speed with respect to the lab
2
frame. It emits  -rays of frequency v0 in its rest frame. There is a stationary detector,

(which is not on the path of the nucleus) in the lab. If a  -ray photon is emitted when the
nucleus is closest to the detector, its observed frequency at the detector is
3 1 1 2
(a) v0 (b) v0 (c) v0 (d) v0
2 3 2 3
Ans. : (a)

v2
Solution: v  v0 1  (If detector is not in the path at nucleus)
c2

1 3
v  v0 1   v0
4 2

H.No. 40‐D, Ground Floor, Jia Sarai, Near IIT, Hauz Khas, New Delhi‐110016 
Phone: 011‐26865455/+91‐9871145498
Website: www.physicsbyfiziks.com  | Email: fiziks.physics@gmail.com  
42 
fiziks
Institute for NET/JRF, GATE, IIT‐JAM, M.Sc. Entrance, JEST, TIFR and GRE in Physics 
 
NET/JRF (JUNE-2017)
Q78. The Hamiltonian for a system described by the generalised coordinate x and generalised
momentum p is

p2 1
H   x2 p    2 x2
2 1  2 x  2

where  ,  and  are constants. The corresponding Lagrangian is


1
 x   x 2  1  2 x    2 x 2
1 1 1
2
(a) (b) x 2   2 x 2   x 2 x
2 2 2 1  2  x  2


1 2
x   2 x  1  2 x    2 x 2
1 1 1
2
(c) (d) x 2   2 x 2   x 2 x
2 2 2 1  2  x  2

Ans. : (a)
p2 1
Solution: H  ax 2 p    2 x2 .
2 1  2 x  2

H p
 x  ax 2   p   x  ax 2  1  2  x  
p 1  2 x 
L  xP
 H

p2 1
 xP
  ax 2 P    2 x2 
1  2 x  2

 x  x   x
 x   x  1  2 x 
2 2 2
2
 1  2 x    x  x   x  1  2 x 
2 2

2 1  2  x 


 1  2  x   x   x 2   x   x 2 
 x   x2    1  2 x 2
 2  2

 1  2 x   x   x
 x   x  2 2
 x   x 
2 2
2
 2
1
  2 x 2  1  2 x 
2 2
1
  2 x2
2
Q79. An inertial observer sees two events E1 and E2 happening at the same location but 6  s

apart in time. Another observer moving with a constant velocity v (with respect to the
first one) sees the same events to be 9  s apart. The spatial distance between the events,
as measured by the second observer, is approximately
(a) 300 m (b) 1000 m (c) 2000 m (d) 2700 m
H.No. 40‐D, Ground Floor, Jia Sarai, Near IIT, Hauz Khas, New Delhi‐110016 
Phone: 011‐26865455/+91‐9871145498
Website: www.physicsbyfiziks.com  | Email: fiziks.physics@gmail.com  
43 
fiziks
Institute for NET/JRF, GATE, IIT‐JAM, M.Sc. Entrance, JEST, TIFR and GRE in Physics 
 
Ans. : (c)
Solution: x12  x11  0 , t21  t11  6 106 , t2  t1  9 106 , x2  x1  ?

t2  t2  9 106

 1 v 1   t '  v x' 
 t2  c 2 x2   1 c 2 1 
   9 106
 1  v / c  1 v / c
2 2 2 2

 
t2  t1 6 106
 9 106   9 106
1 v / c
2 2
1 v / c
2 2

5 v2
v c  1 2  2 / 3
9 c
 x2  vt2   x1  vt1 
 x2  x1     
 1 v / c   1 v / c 
2 2 2 2

v
 t2  t1 
1  v2 / c2

c    6 106  
5 9 5 9
 x2  x1    3 108   6 106
3 6 3 6
 9  5 102  20.12 102  2000m
Q80. A ball weighing 100 gm , released from a height of 5 m , bounces perfectly elastically off
a plate. The collision time between the ball and the plate is 0.5 s . The average force on
the plate is approximately
(a) 3 N (b) 2 N (c) 5 N (d) 4 N
Ans. : (d)
100
Solution: m   0.1 kg
1000
1 2
mgh  mv v  2 gh .
2
v  10 m / sec

change in momentum during collision,  mv     mv   2k .gm / sec

P 2
f    4N
t 0.5

H.No. 40‐D, Ground Floor, Jia Sarai, Near IIT, Hauz Khas, New Delhi‐110016 
Phone: 011‐26865455/+91‐9871145498
Website: www.physicsbyfiziks.com  | Email: fiziks.physics@gmail.com  
44 
fiziks
Institute for NET/JRF, GATE, IIT‐JAM, M.Sc. Entrance, JEST, TIFR and GRE in Physics 
 
Q81. A solid vertical rod, of length L and cross-sectional area A , is made of a material of
Young’s modulus Y . The rod is loaded with a mass M , and, as a result, extends by a
small amount  L in the equilibrium condition. The mass is then suddenly reduced to
M / 2 . As a result the rod will undergo longitudinal oscillation with an angular frequency
(a) 2YA / ML (b) YA / ML

(c) 2YA / M  L (d) YA / M  L


Ans. : (b)
Fl YAl
Solution: Y  F
Al l
YAl
For mass m , mg 
l
m YA   l 
For mass g  
2 l  2 
Equation (i) and (ii) is for equilibrium condition
Change in force will generate acceleration
 mg  YA  l  YA l
F    mg     l  
 2  l  2  2 l
m 2 l YA l

2 l 2

YA

ml
Q82. The Lagrangian of a free relativistic particle (in one dimension) of mass m is given by

L   m 1  x 2 where x  dx / dt . If such a particle is acted upon by a constant force in


the direction of its motion, the phase space trajectories obtained from the corresponding
Hamiltonian are
(a) ellipses (b) cycloids (c) hyperbolas (d) parabolas
Ans. : (c)
Solution: E 2  p 2 c 2  m02 c 4 .

dP P
For constant force  F , P  Ft  t 
dt F

H.No. 40‐D, Ground Floor, Jia Sarai, Near IIT, Hauz Khas, New Delhi‐110016 
Phone: 011‐26865455/+91‐9871145498
Website: www.physicsbyfiziks.com  | Email: fiziks.physics@gmail.com  
45 
fiziks
Institute for NET/JRF, GATE, IIT‐JAM, M.Sc. Entrance, JEST, TIFR and GRE in Physics 
 
mu
 Ft  u 
 F / m t .
1  u 2 / c2  Ft 
2

1  
 mc 

F
  t. mc 2 
2 
dx m F t tdt  Ft 
 x    1    1
dt  Ft 
2 m 0
 Ft 
2 F   mc  
1  1   
 
 mc   mc 

  P2 
2
 Fx

 2  1  1  2 4 
 mc   mc 

P 2  F 2 x 2  2mc 2 Fx   Fx  mc 2   m 2 c 4
2

 Fx  mc  2 2
 P 2  m 2 c 4 , which is equation of hyperbola.

Q83. A Hamiltonian system is described by the canonical coordinate q and canonical

momentum p . A new coordinate Q is defined as Q  t   q  t     p  t    , where t is

the time and  is a constant, that is, the new coordinate is a combination of the old
coordinate and momentum at a shifted time. The new canonical momentum P  t  can be

expressed as
(a) p  t     q  t    (b) p  t     q  t   

1 1
(c)  p  t     q  t     (d)  p  t     q  t    
2 2
Ans. : (d)
Solution: Given  q1 p   1

Q t   q t     p t   

If P is new momentum, then Q, P   1

For option (a), Q, P    q  p, p  q    q, p   q  q, q    p,  q    p, p 

 1   1  2

For option (b) Q, P    q  p, p  q   2

H.No. 40‐D, Ground Floor, Jia Sarai, Near IIT, Hauz Khas, New Delhi‐110016 
Phone: 011‐26865455/+91‐9871145498
Website: www.physicsbyfiziks.com  | Email: fiziks.physics@gmail.com  
46 
fiziks
Institute for NET/JRF, GATE, IIT‐JAM, M.Sc. Entrance, JEST, TIFR and GRE in Physics 
 
1 2
For option (c) Q, P    q  p, p  q    1 i.e. canonical transform
2 2
1 2
For option (d) Q, P    q  p, p  q    1
2 2
Option (c) and (d) are correct. But from translation symmetry option (d) is more suitable.
Q84. The energy of a one-dimensional system, governed by the Lagrangian
1 2 1 2n
L mx  kx
2 2
where k and n are two positive constants, is E0 . The time period of oscillation 
satisfies
1 1 1 n 1 n2 1 1 n
   
(a)   k n
(b)   k 2n
E02 n (c)   k 2n
E02 n (d)   k n E02 n
Ans. : (b)
H J J
Solution:   T  , Time Period T  , where J is Action variable
J H E
V  x

J   Pdx  4 2m  E  V  x  dx
0

1/ 2 n
x
 2E 
  
 k 
1
 2E   2E 
1/ 2 n
x
   
 k 
 
 k 
k 2n  2E   2E  2
1    
J 4 
0
2m  E  kx 2 n  dn  4 2mE
 2 

0
1
2E
x dx  k   k 

Don’t try to solve integration rather try to make E independent.


1 1
1/ 2 n 1
 2E   k  2n  2E  2n
J  4 2mE   1  t dt ,  x  t  dx  
2n
 Put   dt
 k  0  2E   k 
 1 n  1
1  
J  c 4 2m E  2n 
, where c   1  t 2 n dt
k 
1/ 2 n
0

 1 n
1 n 
J 1  1. 1
T  k  E
2 n  2n 
  k  E 2n
2 n
E

H.No. 40‐D, Ground Floor, Jia Sarai, Near IIT, Hauz Khas, New Delhi‐110016 
Phone: 011‐26865455/+91‐9871145498
Website: www.physicsbyfiziks.com  | Email: fiziks.physics@gmail.com  
47 
fiziks
Institute for NET/JRF, GATE, IIT‐JAM, M.Sc. Entrance, JEST, TIFR and GRE in Physics 
 
ELECTROMAGNETIC THEORY
NET/JRF–(JUNE-2011)
Q1. The electrostatic potential V(x, y) in free space in a region where the charge density ρ is
zero is given by V  x, y   4e 2 x  f  x   3 y 2 . Given that the x-component of the electric

field Ex, and V are zero at the origin, f  x  is

(a) 3x 2  4e 2 x  8 x (b) 3x 2  4e 2 x  16 x
(c) 4e 2 x  8 (d) 3x 2  4e 2 x
Ans: (d)
Solution: V  4e 2 x  f  x   3 y 2 . Since   0   2V  0  16e 2 x  f  x   6  0 .
 

Since E x  0 at origin  E  V  E x   8e 2 x  f  x  
E x 0, 0  8  f 0  0  f 0   8 .

Since V 0, 0   0  4  f 0   0  f 0   4

Solve equation 16e 2 x  f  x   6  0  f  x   6  16e 2 x  f x   6 x  8e 2 x  c1 , since

f   0   8  c1  8  c1  0 .

Again Integrate f  x   6 x  8e 2 x  f  x   3 x 2  4e 2 x  c 2

since f  0   4  c2  4  c2  0 . Thus f  x   3x 2  4e 2 x
   
Q2. For constant uniform electric and magnetic field E  E 0 and B  B0 , it is possible to

choose a gauge such that the scalar potential  and vector potential A are given by
 1      1  
(a)   0 and A  B0  r
2
  
(b)    E 0  r and A  B0  r
2

    
(c)    E0  r and A  0 (d)   0 and A   E 0 t
Ans: (a)
 
Solution: Let E  E 0  xˆ  yˆ  zˆ  and B  B0  xˆ  yˆ  zˆ  since they are constant vector.
  
Lorentz Gauge condition is   A   0 0
t

H.No. 40‐D, Ground Floor, Jia Sarai, Near IIT, Hauz Khas, New Delhi‐110016 
Phone: 011‐26865455/+91‐9871145498
Website: www.physicsbyfiziks.com  | Email: fiziks.physics@gmail.com  

fiziks
Institute for NET/JRF, GATE, IIT‐JAM, M.Sc. Entrance, JEST, TIFR and GRE in Physics 
 
 
since B  r  B0  z  y  xˆ  B0  z  x  yˆ  B0  y  x  zˆ 
     
(a)  0 and   A  0 (b)  0, and   A  0
t t
     
(c)  0 and   A  0 (d)  0 and   A  0
t t
Q3. A plane electromagnetic wave is propagating in a lossless dielectric. The electric field is
given by

 

 
E  x, y, z, t   E0  xˆ  Azˆ  exp ik0 ct  x  3 z  ,

where c is the speed of light in vacuum, E0 , A and k0 are constant and x̂ and ẑ are

unit vectors along the x - and z -axes. The relative dielectric constant of the medium  r
and the constant A are
1 1
(a)  r  4 and A   (b)  r  4 and A  
3 3

(c)  r  4 and A  3 (d)  r  4 and A   3


Ans: (a)

  
Solution: E  x, y, z , t   E 0  xˆ  Azˆ  exp ik 0  ct  x  3 z . 
 

Comparing with term e i k r t   k  k 0 xˆ  3 zˆ and   k 0 c .

 k0c c
Since v     Refractive index n   r  2   r  4.
k k  3k
2
0
2
0
2

  
Since k  nˆ  0  k 0 xˆ  3 zˆ   xˆ  Azˆ   0  k 0 1  A 3  0  A    1
3
A  Kr
Q4. A static, spherically symmetric charge distribution is given by  r   e where A
r
and K are positive constants. The electrostatic potential corresponding to this charge
distribution varies with r as

(a) re  Kr (b)
1  Kr
r
e (c)
1  Kr
r2
e (d)
1
r

1  e  Kr 
Ans: (b)
Solution: since  2V    /  0
H.No. 40‐D, Ground Floor, Jia Sarai, Near IIT, Hauz Khas, New Delhi‐110016 
Phone: 011‐26865455/+91‐9871145498
Website: www.physicsbyfiziks.com  | Email: fiziks.physics@gmail.com  

fiziks
Institute for NET/JRF, GATE, IIT‐JAM, M.Sc. Entrance, JEST, TIFR and GRE in Physics 
 
A 1   2 V 
 2V must be proportional to e  kr , where  2V  2 r .
r r r  r 
Q5. The magnetic field of the TE11 mode of a rectangular waveguide of dimensions a  b as

shown in the figure is given by H z  H 0 cos  0.3  x  cos  0.4  y  , where x and y are in
x
cm.
a
z
b
y

A. The dimensions of the waveguide are


(a) a  3.33 cm, b  2.50 cm (b) a  0.40 cm, b  0.30 cm
(c) a  0.80 cm, b  0.60 cm (d) a  1.66 cm, b  1.25 cm
Ans: (a)
Solution: Since H z  H 0 cos  0.3 x  cos  0.4 y 

m n
  0.3 where m  1 and  0.4 where n  1
a b
 a  3.33cm, b  2.50cm
B. The entire range of frequencies f for which the TE11 mode will propagate is

(a) 6.0 GHz  f  12.0 GHz (b) 7.5 GHz  f  9.0 GHz
(c) 7.5 GHz  f  12.0 GHz (d) 7.5GHz  f
Ans: (d)
2 2
c m n c 1 1
Solution: f m , n        f1,1   2  7.5 GH z .
2  a  b 2 a 2
b
For propagation, frequency of incident wave must be greater than cutoff frequency.

H.No. 40‐D, Ground Floor, Jia Sarai, Near IIT, Hauz Khas, New Delhi‐110016 
Phone: 011‐26865455/+91‐9871145498
Website: www.physicsbyfiziks.com  | Email: fiziks.physics@gmail.com  

fiziks
Institute for NET/JRF, GATE, IIT‐JAM, M.Sc. Entrance, JEST, TIFR and GRE in Physics 
 
NET/JRF -(DEC-2011)
Q6. Consider three polarizer’s P1 , P2 and P3 placed along an axis as shown in the figure.

P1 P2 P3
(unpolarized) 
I0

The pass axis of P1 and P3 are at right angles to each other while the pass axis of P2

makes an angle  with that of P1 . A beam of unpolarized light of intensity I 0 is incident

on P1 as shown. The intensity of light emerging from P3 is

I0 I0 I0
(a) 0 (b) (c) sin 2 2 (d) sin 2 2
2 8 4
Ans: (c)
Solution: I  I 0 cos 2  (Malus Law)

I0 I0 I0 I
 I1  , I2  cos 2  , I3  cos 2   cos 2 90     0 sin 2 2 .
2 2 2 8
Q7. Four equal point charges are kept fixed at the four vertices of a square. How many neutral
points (i.e. points where the electric field vanishes) will be found inside the square?
(a) 1 (b) 4 (c) 5 (d) 7
Ans: (a)
Solution: Inside the square, there is only one point where field vanishes.

Q8. A static charge distribution gives rise to an electric field of the form E   1  e  r / R   rˆ ,
r2
where  and R are positive constants. The charge contained within a sphere of radius R ,
centred at the origin is
e e2 R R2
(a)  0 (b)  0 (c) 4 0 (d)  0
R2 R2 e e
Ans: None of the options given are correct
   2
   
Solution: Qenc   0  E  da   0  1  e  r / R 2  r 2 sin ddrˆ   0    1  e  r / R sin dd

r 0 0

H.No. 40‐D, Ground Floor, Jia Sarai, Near IIT, Hauz Khas, New Delhi‐110016 
Phone: 011‐26865455/+91‐9871145498
Website: www.physicsbyfiziks.com  | Email: fiziks.physics@gmail.com  

fiziks
Institute for NET/JRF, GATE, IIT‐JAM, M.Sc. Entrance, JEST, TIFR and GRE in Physics 
 
 1
at r  R , Qenc  4 0 1   . So none of the options given are correct.
 e
Q9. In a Young’s double slit interference experiment, the slits are at a distance 2 L from each
other and the screen is at a distance D from the slits. If a glass slab of refractive index 
and thickness d is placed in the path of one of the beams, the minimum value of d for
the central fringe to be dark is
D D
(a) (b)
  1 D 2  L2   1L
 
(c) (d)
  1 2  1
Ans: (d)
n 
Solution: For central fringe to be dark,   1d  d 
2 2  1
Q10. Consider a solenoid of radius R with n turns per unit length, in which a time dependent
current I  I 0 sin t (where  R / c  1 ) flows. The magnitude of the electric field at a
perpendicular distance r  R from the axis of symmetry of the solenoid, is
1
(a) 0 (b)  0 nI 0 R 2 cos t
2r
1 1
(c)  0 nI 0 r sin t (d)  0 nI 0 r cos t
2 2
Ans: (d)
  
Solution: E  d l  
B
 t  d a ; B 
  0 nI t zˆ .
 dI
r
2r 2
dt r 0
 E  2r    0 n 2r d r     0 n  I 0  cos  t 
2
 1
 E     0 nI 0 r cos t
2
Q11. A constant electric current I in an infinitely long straight wire is suddenly switched on at
t  0 . The vector potential at a perpendicular distance r from the wire is given

by A 
kˆ 0 I  1
2

ln  ct  c 2 t 2  r 2
r
 . The electric field at a distance r   ct  is

H.No. 40‐D, Ground Floor, Jia Sarai, Near IIT, Hauz Khas, New Delhi‐110016 
Phone: 011‐26865455/+91‐9871145498
Website: www.physicsbyfiziks.com  | Email: fiziks.physics@gmail.com  

fiziks
Institute for NET/JRF, GATE, IIT‐JAM, M.Sc. Entrance, JEST, TIFR and GRE in Physics 
 
 I 1 ˆ ˆ
(a) 0 (b) 0
2 t 2
ij  
c 0 I c 0 I
(c)
1 ˆ ˆ
i j  (d)  kˆ
2 c t  r
2 2 2
2 2 c t  r
2 2 2

Ans: (d)
 
 A A   I r 1  2c 2 t 
c   .
Solution: E   
t

t
E 0
 
2 ct  c 2 t 2  r 2  r  
2 c 2t 2  r 2


  c 0 I
E kˆ
2 c t  r
2 2 2

NET/JRF -(JUNE-2012)
 1   10  
Q12. The magnetic field corresponding to the vector potential A  F  r  3 r , where F is
2 r
a constant vector, is
   30   30 
(a) F (b)  F (c) F  4 r (d) F  4 r
r r
Ans: (a)

   1   
    r  
Solution: B    A    F  r  10   3  . Since F is a constant vector, let
2  r 
xˆ yˆ zˆ
  
F  F0  xˆ  yˆ  zˆ  , F  r  F0 F0 F0  xˆ  z  y F0  yˆ  z  x F0  zˆ  y  x F0
x y z
xˆ yˆ zˆ
  

 F r   
x

y

z
 xˆF0  F0   yˆ  F0  F0   zˆF0  F0   2 F0  xˆ  yˆ  zˆ 
z  y F0 x  z F0
 y  x F0
  r
1   
  
   F  r  F0  xˆ  yˆ  zˆ   F ,   3  0 . Thus B  F
2 r
Q13. An electromagnetic wave is incident on a water-air interface. The phase of the
perpendicular component of the electric field, E  , of the reflected wave into the water is
found to remain the same for all angles of incidence. The phase of the magnetic field H
(a) does not change (b) changes by 3 / 2
(c) changes by  / 2 (d) changes by 
Ans: (d)

H.No. 40‐D, Ground Floor, Jia Sarai, Near IIT, Hauz Khas, New Delhi‐110016 
Phone: 011‐26865455/+91‐9871145498
Website: www.physicsbyfiziks.com  | Email: fiziks.physics@gmail.com  

fiziks
Institute for NET/JRF, GATE, IIT‐JAM, M.Sc. Entrance, JEST, TIFR and GRE in Physics 
 
Q14. The magnetic field at a distance R from a long straight wire carrying a steady current I
is proportional to
(a) IR (b) I / R 2 (c) I 2 / R 2 (d) I / R
Ans: (d)
Q15. Which of the following questions is Lorentz invariant?
2 2 2 2 2 2 2
(a) E  B (b) E  B (c) E  B (d) E B
Ans: (b)
Q16. Charges Q, Q and 2Q are placed on the vertices of an equilateral triangle ABC of
sides of length a , as shown in the figure. The dipole moment of this configuration of
charges, irrespective of the choice of origin, is - 2Q
ĵ C
(a)  2aQ iˆ a
a
(b)  3aQ ˆj
A B
Q a Q
(c)  3aQ ˆj
(d) 0 iˆ
Ans: (c)
Solution: Let coordinates of A is (l, m), then
 a   3a  ˆ 
 
   
p  qi ri   Q liˆ  mˆj  Q l  a iˆ  mˆj  2Q  l  iˆ   m   j
2  
 2  

      

p  Q liˆ  mˆj  Q l  a iˆ  mˆj  Q 2l  a iˆ  2m  3a ˆj  p   3aQˆj
mr
Q17. The vector potential A due to a magnetic moment m at a point r is given by A  .
r3

If m is directed along the positive z -axis, the x - component of the magnetic field, at the

point r , is
3myz 3mxy 3mxz 3mz 2  xy 
(a) (b)  (c) (d)
r5 r5 r5 r5
Ans: (c)

Solution: m  mzˆ and
   m
 
 
B    A  3 2 cos rˆ  sin ˆ  3 3m  rˆ rˆ  m
r r
1

 1  
 xxˆ  yyˆ  zzˆ  r  3mxz
B  3 3mzˆ     mzˆ   Bx  5
r   r r  r
H.No. 40‐D, Ground Floor, Jia Sarai, Near IIT, Hauz Khas, New Delhi‐110016 
Phone: 011‐26865455/+91‐9871145498
Website: www.physicsbyfiziks.com  | Email: fiziks.physics@gmail.com  

fiziks
Institute for NET/JRF, GATE, IIT‐JAM, M.Sc. Entrance, JEST, TIFR and GRE in Physics 
 
NET/JRF -(DEC-2012)
Q18. Three charges are located on the circumference of a circle of radius R as shown in the
figure below. The two charges Q subtend an angle 900 at the centre Q Q
of the circle. The charge q is symmetrically placed with respect to
the charges Q . If the electric field at the centre of the circle is zero,
what is the magnitude of Q ?
q
(a) q / 2 (b) 2q (c) 2q (d) 4q
Ans: (a)
1 Q 1 q
Solution: E1  E 2  and E3 
4 0 R 2
4 0 R 2
q
Resultant of E1 and E 2 is E  E12  E 22  2E1 , Thus E3  E  Q 
2
Q19. Consider a hollow charged shell of inner radius a and outer radius b . The volume
k
charge density is  r   ( k is constant) in the region a  r  b . The magnitude of the
r2
electric field produced at distance r  a is
k b  a 
(a) for all r  a ,
 0r 2
k b  a  kb
(b) for a  r  b and for r  b
 0r 2
 0r 2
k r  a  k b  a 
(c) for a  r  b and for r  b
 0r 2
 0r 2
k r  a  k b  a 
(d) for a  r  b and for r  b
 0a 2
 0r 2
Ans: (c)
  1 1 1 k
Solution: For r  a :  E.da  E (4 r 2 )  Qenc    dV   2 r sin  drd d
2

0 0 0 r

4 k r 4 k  k ra
E (4 r 2 )  a dr  (r  a)  E   rˆ
0 0  0  r 2 

H.No. 40‐D, Ground Floor, Jia Sarai, Near IIT, Hauz Khas, New Delhi‐110016 
Phone: 011‐26865455/+91‐9871145498
Website: www.physicsbyfiziks.com  | Email: fiziks.physics@gmail.com  

fiziks
Institute for NET/JRF, GATE, IIT‐JAM, M.Sc. Entrance, JEST, TIFR and GRE in Physics 
 
4 k b 4 k  k  ba 
For r  b : E 4 r 2  a dr  (b  a)  E   rˆ
0 0  0  r 2 
Q20. Consider the interference of two coherent electromagnetic waves whose electric field
 
vectors are given by E1  iˆE 0 cos  t and E 2  ˆjE 0 cos t    where  is the phase

0
difference. The intensity of the resulting wave is given by E 2 , where E 2 is the
2
time average of E 2 . The total intensity is
(a) 0 (b)  0 E 02 (c)  0 E 02 sin 2  (d)  0 E 02 cos 2 
Ans: (a)
Solution: Since waves are polarized in perpendicular direction hence there will be no
interference.
Q21. Four charges (two  q and two q ) are kept fixed at the four vertices of a square of side

a as shown. At the point P which is at a distance R from the centre  R  a  , the


q q
potential is proportional to
(a) 1/ R (b) 1/ R 2 a R P
(c) 1/ R 3
(d) 1/ R 4

q q
Ans: (c)
Solution: Given configuration is quadrupole.
Q22. A point charges q of mass m is kept at a distance d below a grounded infinite
conducting sheet which lies in the xy - plane. For what value of d will the charge
remains stationary?
(a) q / 4 mg 0 (b) q / mg 0

(c) There is no finite value of d (d) mg 0 / q

Ans: (a)
Solution: There is attractive force between point charge q and grounded conducting sheet that

1q2 q
can be calculate from method of images i.e.  mg  d 
4 0 2d  2
4 mg 0

H.No. 40‐D, Ground Floor, Jia Sarai, Near IIT, Hauz Khas, New Delhi‐110016 
Phone: 011‐26865455/+91‐9871145498
Website: www.physicsbyfiziks.com  | Email: fiziks.physics@gmail.com  

fiziks
Institute for NET/JRF, GATE, IIT‐JAM, M.Sc. Entrance, JEST, TIFR and GRE in Physics 
 
Q23. An infinite solenoid with its axis of symmetry along the z -direction carries a steady
current I .
 ẑ
The vector potential A at a distance R from the axis
(a) is constant inside and varies as R outside the solenoid
R
(b) varies as R inside and is constant outside the solenoid
1
(c) varies as inside and as R outside the solenoid
R
1
(d) varies as R inside and as outside the solenoid
R
Ans: (d)
Q24. Consider an infinite conducting sheet in the xy -plane with a time dependent current

density Kt iˆ , where K is a constant. The vector potential at  x, y , z  is given


0 K 
by A  ct  z 2 iˆ . The magnetic field B is
4c
 0 Kt ˆ  0 Kz ˆ 0 K 0 K
(a) j (b)  j (c)  ct  z iˆ (d)  ct  z  ˆj
2 2c 2c 2c
Ans: (d)
Ax  K
B   A  yˆ =  0 ct  z  ˆj
z 2c

Q25. When a charged particle emits electromagnetic radiation, the electric field E and the
 1   1 1
Poynting vector S  E  B at a larger distance r from emitter vary as n and m
0 r r
respectively. Which of the following choices for n and m are correct?
(a) n  1 and m  1 (b) n  2 and m  2
(c) n  1 and m  2 (d) n  2 and m  4
Ans: (c)

H.No. 40‐D, Ground Floor, Jia Sarai, Near IIT, Hauz Khas, New Delhi‐110016 
Phone: 011‐26865455/+91‐9871145498
Website: www.physicsbyfiziks.com  | Email: fiziks.physics@gmail.com  
10 
fiziks
Institute for NET/JRF, GATE, IIT‐JAM, M.Sc. Entrance, JEST, TIFR and GRE in Physics 
 
NET/JRF -(JUNE-2013)
Q26. A particle of charge e and mass m is located at the midpoint of the line joining two fixed
collinear dipoles with unit charges as shown in the figure. (The particle is constrained to
move only along the line joining the dipoles). Assuming that the length of the dipoles is
much shorter than their separation, the natural frequency of oscillation of the particle is

R R

  e, m  

2d 2d

6eR 2 6eR 6ed 2 6ed


(a) (b) (c) (d)
 0 md 5  0 md 4  0 mR 5  0 mR 4
Ans: (d) R R
Solution: Let us displace the charge particle by small
x
amount x at A . Then the resultant electric field at   e, m A  
point A is given by
2d 2d
2p  1 1  6d
E   3 
 x,
4 0  R  x  3
R  x    0 R 4
6ed k 6ed
F  eE   x . Then,    (where p  1 2d  2d )
 0 R 4
m  0 mR 4
Q27. A current I is created by a narrow beam of protons moving in vacuum with constant
 
velocity u . The direction and magnitude, respectively of the Poynting vector S outside the
beam at a radial distance r (much larger than the width of the beam) from the axis, are
   I2    I2
(a) S  u and S   (b) S ||  u  and S  
4 2  0 u r 2 4 2  0 u r 4
   I2    I2
(c) S || u and S   (d) S || u and S  
4 2  0 u r 2 4 2  0 u r 4

Ans: (c)

H.No. 40‐D, Ground Floor, Jia Sarai, Near IIT, Hauz Khas, New Delhi‐110016 
Phone: 011‐26865455/+91‐9871145498
Website: www.physicsbyfiziks.com  | Email: fiziks.physics@gmail.com  
11 
fiziks
Institute for NET/JRF, GATE, IIT‐JAM, M.Sc. Entrance, JEST, TIFR and GRE in Physics 
 
Solution: Let charge per unit length be  , hence I   u in z -direction.
0 I ˆ
The magnetic field at a distance r is B  .
2r
 I
The electric field at a distance r is E  rˆ  rˆ .
2 0 r 2 0 ur

EB I2
Hence Poynting vector S   zˆ
0 4 2  0 ur 2

Q28. If the electric and magnetic fields are unchanged when the potential A changes (in
  
suitable units) according to A  A  r̂ , where r  r t r̂ , then the scalar potential  must
simultaneously change to
(a)   r (b)   r (c)    r /  t (d)    r /  t
Ans: (c)

Solution: A  A    A  rˆ    /  r  1    r  C
V  V   / t V   r / t
Q29. Consider an axially symmetric static charge distribution of the form,
2
r 
   0  0  e  r / r0 cos 2 
r
The radial component of the dipole moment due to this charge distribution is
(a) 2 0 r04 (b)  0 r04 (c)  0 r04 (d)  0 r04 / 2
Ans: (a)
2
r 
Solution: p   r  r d      r    0  0  e  r  / r0 cos 2   r  2 sin dr dd
V  r 
  2

 dr  sin  d  cos 2  d  20 r04


 r  / r0
p r 0 0
2
r e
r  0 0 0

Q30. The components of a vector potential A   A0 , A1 , A2 , A3  are given by

A  k  xyz, yzt , zxt , xyt 

where k is a constant. The three components of the electric field are


(a) k  yz, zx, xy  (b) k  x, y, z  (c) 0, 0, 0  (d) k  xt , yt , zt 

H.No. 40‐D, Ground Floor, Jia Sarai, Near IIT, Hauz Khas, New Delhi‐110016 
Phone: 011‐26865455/+91‐9871145498
Website: www.physicsbyfiziks.com  | Email: fiziks.physics@gmail.com  
12 
fiziks
Institute for NET/JRF, GATE, IIT‐JAM, M.Sc. Entrance, JEST, TIFR and GRE in Physics 
 
Ans: (c)
Solution: A   , Ax , Ay , Az     kxyz, Ax  kyzt , Ay  kzxt , Az  kxyt

A
Since   k  yzxˆ  xzyˆ  xyzˆ  and  k  yzxˆ  xzyˆ  xyzˆ 
t

 A
E     k  yzxˆ  xzyˆ  xyzˆ   k  yzxˆ  xzyˆ  xyzˆ   0  E  0,0,0 
t
Q31. An oscillating current I t   I 0 exp it  flows in the direction of the y -axis through a

thin metal sheet of area 1.0 cm 2 kept in the xy -plane. The rate of total energy radiated
per unit area from the surfaces of the metal sheet at a distance of 100 m is

(a) I 0 / 12 0 c 3  (b) I 02 2 / 12 0 c 3 

(c) I 02 3 / 12 0 c 3  (d) I 02 4 / 12 0c 3 

Ans: (d)
NET/JRF -(DEC-2013)
Q32. A horizontal metal disc rotates about the vertical axis in a uniform magnetic field
pointing up as shown in the figure. A circuit is made by connecting one end A of a
resistor to the centre of the disc and the other end B to its edge through a sliding contact.
The current that flows through the resistor is
B
A B

(a) zero (b) DC from A to B


(c) DC from B to A (d) AC
Ans: (b)
Q33. The force between two long and parallel wires carrying currents I 1 and I 2 and separated
by a distance D is proportional to
(b) I 1  I 2  / D (c) I 1 I 2 / D 
2
(a) I 1 I 2 / D (d) I 1 I 2 / D 2
Ans: (a)

H.No. 40‐D, Ground Floor, Jia Sarai, Near IIT, Hauz Khas, New Delhi‐110016 
Phone: 011‐26865455/+91‐9871145498
Website: www.physicsbyfiziks.com  | Email: fiziks.physics@gmail.com  
13 
fiziks
Institute for NET/JRF, GATE, IIT‐JAM, M.Sc. Entrance, JEST, TIFR and GRE in Physics 
 
Q34. The electric field of an electromagnetic wave is given by

E  E 0 cos 0.3x  0.4 y  1000t kˆ .

The associated magnetic field B is
(a) 10 3 E 0 cos 0.3 x  0.4 y  1000t kˆ


(b) 10 4 E 0 cos 0.3 x  0.4 y  1000t  4iˆ  3 ˆj
(c) E cos 0.3 x  0.4 y  1000t 0.3iˆ  0.4 ˆj 
0

(d) 10 E cos 0.3 x  0.4 y  1000t 3iˆ  4 ˆj 


2
0

Ans: (b)

Solution: k    0.3xˆ  0.4 yˆ  ,   1000 
kE
 0.3 xˆ  0.4 yˆ   E 0 cos 0.3x  0.4 y  1000t kˆ
1
B 
 
 B  10 4 E 0 cos 0.3x  0.4 y  1000t  4iˆ  3 ˆj  
Q35. A point charge q is placed symmetrically at a distance d from two perpendicularly
placed grounded conducting infinite plates as shown in the figure. The net force on the
charge (in units of 1 / 4 0 ) is
q
(a)
q2
8d 2
 
2 2  1 away from the corner
d

d
(b)
q2
8d 2
 
2 2  1 towards the corner

q2
(c) towards the corner
2 2d 2
3q 2 F3
(d) away from the corner d d
8d 2 q q
Ans: (b) F1
F2 d
q2 q2
Solution: F 1  F 2  k and F 3  k
4d 2 8d 2 d
q 2 q q
Resultant of F 1 , F 2 is F12  F12  F22  2 2k . 2d
8d 2

H.No. 40‐D, Ground Floor, Jia Sarai, Near IIT, Hauz Khas, New Delhi‐110016 
Phone: 011‐26865455/+91‐9871145498
Website: www.physicsbyfiziks.com  | Email: fiziks.physics@gmail.com  
14 
fiziks
Institute for NET/JRF, GATE, IIT‐JAM, M.Sc. Entrance, JEST, TIFR and GRE in Physics 
 
q
 
2
Net force F  k 2 2 2  1 (towards the corner)
8d
Q36. If the electrostatic potential V r ,  ,   in a charge free region has the form

V  r ,  ,    f  r  cos  , then the functional form of f r  (in the following a and b are

constants) is:
b b b r
(a) ar 2  (b) ar  (c) ar  (d) a ln 
r r2 r b
Ans: (b)

1   V  1   V  1   2V 
Solution:  V  2  r 2  2  sin   2 2  2 0
2

r r  r  r sin      r sin    
1   2 f  1 
 r cos    2 sin  f    sin     0
r r  r  r sin  
2

cos   2 2 f f  f
  r 2  2r   2  2sin  cos    0
r2  r r  r sin 

2 f f
 r2  2r  2 f  r   0
r
2
r
b
f r   ar  satisfy the above equation.
r2
Q37. Let four point charges q,  q / 2, q and  q / 2 be placed at the vertices of a square of
side a . Let another point charge  q be placed at the centre of the square (see the figure).

q/2 q

q

q q/2
Let V r  be the electrostatic potential at a point P at a distance r  a from the centre
of the square. Then V 2r  / V r  is
1 1 1
(a) 1 (b) (c) (d)
2 4 8
Ans: (d)

H.No. 40‐D, Ground Floor, Jia Sarai, Near IIT, Hauz Khas, New Delhi‐110016 
Phone: 011‐26865455/+91‐9871145498
Website: www.physicsbyfiziks.com  | Email: fiziks.physics@gmail.com  
15 
fiziks
Institute for NET/JRF, GATE, IIT‐JAM, M.Sc. Entrance, JEST, TIFR and GRE in Physics 
 
q q
Solution: According to multipole expansion Qmono    q   q  q  0
2 2
 q q  q q q   q q  qq q 
p  q  xˆ  yˆ     xˆ  yˆ   0  q   xˆ  yˆ    xˆ  yˆ   0
2 2  2 2 2   2 2  22 2 
1 V 2r  1
Thus, V    .
r 3
V r  8

Q38.    
Let V , A and V , A denote two sets of scalar and vector potentials, and  is a scalar
function. Which of the following transformations leave the electric and magnetic fields
(and hence Maxwell’s equations) unchanged?
 
(a) A  A   and V   V  (b) A  A   and V   V  2
t t
 
(c) A  A   and V   V  (d) A  A   and V   V 
t t
Ans: (a)

JRF/NET–(JUNE-2014)

Q39. A time-dependent current I t   Ktzˆ (where K is a constant) is switched on at t  0 in
an infinite current-carrying wire. The magnetic vector potential at a perpendicular
distance a from the wire is given (for time t  a / c ) by

 0 K c t a  K
2 2 2
ct
ct  a 2  z 2 t
4 c  c t  a  dz a
(a) zˆ dz (b) zˆ 0
2 2 a 2  z 2 1 / 2
2
4  ct
2
 z2 1/ 2

c 2t 2  a 2
 K ct ct  a 2  z 2  K t
(c) zˆ 0  dz (d) zˆ 0  dz
4 c ct a 2  z 2 1 / 2 4  c 2t 2  a 2 a 2
 z2 
1/ 2

Ans: (a) I
dz
  
I  tr  0 
K t  R / c 
Solution: A  zˆ 0
4 

R
dz  zˆ
4 

R
dz z R

P
 0 K c t  a
2 2 2
ct  a 2  z 2 a
4 c  c2t 2 a2
 A  zˆ dz
 
1/2
a2  z2

H.No. 40‐D, Ground Floor, Jia Sarai, Near IIT, Hauz Khas, New Delhi‐110016 
Phone: 011‐26865455/+91‐9871145498
Website: www.physicsbyfiziks.com  | Email: fiziks.physics@gmail.com  
16 
fiziks
Institute for NET/JRF, GATE, IIT‐JAM, M.Sc. Entrance, JEST, TIFR and GRE in Physics 
 
Q40. A current i p flows through the primary coil of a transformer. The graph of i p t  as a

function of time t is shown in the figure below.


i p t 

1 2 3 t
Which of the following graphs represents the current i S in the secondary coil?
is  t  is  t 

(a) (b)
1 2 3 t

1 2 3 t

is  t  is  t 
(c) (d)

1 2 3t

1 2 3t
Ans: (c)
di p
Solution: is  
dt
Q41. If the electrostatic potential in spherical polar coordinates is
 r    0 e  r / r
0

where  0 and r0 are constants, then the charge density at a distance r  r0 will be

 0 0 e 0 0  0 0 2e 0 0
(a) (b) (c)  (d) 
er 0
2
2r02 er 0
2
r02
Ans: (a)

Solution:   2       0   2 
0

1   2   1   2 0  r / r0  1 0  2  r / r0
 2  2 r  2 r  e
r r  r  r r  r0
 2
r r r
r e  
 0

1 0  2 1  r / r0    1 2 
  2   r   e  2re  r / r0    0   e  r / r0  e  r / r0 
r r0 
2
r0  r0  r0 r 
H.No. 40‐D, Ground Floor, Jia Sarai, Near IIT, Hauz Khas, New Delhi‐110016 
Phone: 011‐26865455/+91‐9871145498
Website: www.physicsbyfiziks.com  | Email: fiziks.physics@gmail.com  
17 
fiziks
Institute for NET/JRF, GATE, IIT‐JAM, M.Sc. Entrance, JEST, TIFR and GRE in Physics 
 
  1 2      
At a distance r  r0 ,  2   0  e 1  e 1    20     0   20   02 0
r0  r0 r0  r0 e  r0 e  r0 e

Q42. If A  yziˆ  zxˆj  xykˆ and C is the circle of unit radius in the plane defined by z  1 ,
 
with the centre on the z - axis, then the value of the integral  A  d  is
C

 
(a) (b)  (c) (d) 0
2 4
Ans: (d)
iˆ ˆj kˆ
    
Solution:   A   iˆ  x  x   ˆj  y  y   kˆ  z  z   0
x y z
yz zx xy
    

Since  A  d      A  d a  0
C

S

Q43. Consider an electromagnetic wave at the interface between two homogenous dielectric
media of dielectric constants  1 and  2 . Assuming  2   1 and no charges on the surface,
 
the electric field vector E and the displacement vector D in the two media satisfy the
following inequalities
       
(a) E 2  E1 and D2  D1 (b) E 2  E1 and D2  D1
       
(c) E 2  E1 and D2  D1 (d) E 2  E1 and D2  D1

Ans. : (c)
Q44. A charge  e  is placed in vacuum at the point d ,0,0  , where d  0 . The region x  0
d 
is filled uniformly with a metal. The electric field at the point  ,0,0  is
2 
10e 10e
(a)  1, 0, 0 (b) 1, 0, 0
9 0 d 2
9 0 d 2
e e
(c) 1, 0, 0 (d)  1, 0, 0
 0 d 2  0 d 2
Ans: (b)

Solution: E

d
E
2 P
x
e d 0 d e

H.No. 40‐D, Ground Floor, Jia Sarai, Near IIT, Hauz Khas, New Delhi‐110016 
Phone: 011‐26865455/+91‐9871145498
Website: www.physicsbyfiziks.com  | Email: fiziks.physics@gmail.com  
18 
fiziks
Institute for NET/JRF, GATE, IIT‐JAM, M.Sc. Entrance, JEST, TIFR and GRE in Physics 
 
1 e 1 4e 1 e 1 4e
E   and E  
4 0  3d / 2  2
4 0 9d 2
4 0  d / 2  2
4 0 d 2

Thus resultant electric field at point P is


1 4e 1 4e 1 40e 1 10e  1 10e
E  E  E      E xˆ
4 0 9d 2
4 0 d 2
4 0 9d 2
9 0 d 2
9 0 d 2
Q45. A beam of light of frequency  is reflected from a dielectric-metal interface at normal
incidence. The refractive index of the dielectric medium is n and that of the metal is
n 2  n1  i  . If the beam is polarised parallel to the interface, then the phase change
experienced by the light upon reflection is
(a) tan 2 /   (b) tan 1 1 /   (c) tan 1 2 /   (d) tan 1 2  
Ans: (c)
 1     v c/n
Solution: Since E 0 R    E0 I where   1   1  i
 1  
 v 2 c / n 1  i  
 i      ei /2     i /2   

 E0 R   E     
E e E0 I where tan   .
  4  e   4  
 2  i 
0 I i 0 I

2
 
2
 2
2 2
Thus phase change     / 2     tan   cot      tan 1  
 
Q46. A thin, infinitely long solenoid placed along the z - axis contains a magnetic flux  .
Which of the following vector potentials corresponds to the magnetic field at an arbitrary
point x, y, z  ?

   
(a) Ax , Ay , Az    
y x
, ,0 
 2 x  y 2 x  y
2 2 2 2

   
(b) Ax , Ay , Az    
y x
, ,0 
 2 x  y  z 2 x  y  z
2 2 2 2 2 2

  x y  x y 
(c) Ax , Ay , Az     , ,0 
 2 x  y 2 x  y
2 2 2 2

   
(d) Ax , Ay , Az    
x y
, ,0 
 2 x  y 2 x  y
2 2 2 2

Ans: (a)
  
Solution: B    A  0
H.No. 40‐D, Ground Floor, Jia Sarai, Near IIT, Hauz Khas, New Delhi‐110016 
Phone: 011‐26865455/+91‐9871145498
Website: www.physicsbyfiziks.com  | Email: fiziks.physics@gmail.com  
19 
fiziks
Institute for NET/JRF, GATE, IIT‐JAM, M.Sc. Entrance, JEST, TIFR and GRE in Physics 
 
Q47. An electromagnetically-shielded room is designed so that at a frequency   10 7 rad/s
the intensity of the external radiation that penetrates the room is 1% of the incident

 10 6 m  is the conductivity of the shielding material, its


1 1
radiation. If  
2
minimum thickness should be (given that ln 10  2.3 )
(a) 4.60 mm (b) 2.30 mm (c) 0.23 mm (d) 0.46 mm
Ans. : (b)
1  I0 
Solution: I  I 0 e 2 z  z 
2  I 
ln

I0  1 1
where  100,     106  4  107  107  103
I 2 2 2
1
z ln 100   2.30 mm
2  103
Q48. A charged particle is at a distance d from an infinite conducting plane maintained at zero
potential. When released from rest, the particle reaches a speed u at a distance d / 2 from
the plane. At what distance from the plane will the particle reach the speed 2u ?
(a) d / 6 (b) d / 3 (c) d / 4 (d) d / 5
Ans: (d) x
d x
2
1 q d x2
A2
q 2
Solution: F  ma  m   2   2 where A  . P q
dt 2
4 0 4d 2
dt x 16 m 0
d
dv A dv A dx 1 d 2 d  A

dt
 2 v
x dt
 2
x dt

2 dt
v   
dt  x 
  0

v2 A A 1 1 d
   C at  x  d , v  0  C    v  2 A    .
2 x d x d q

 1 1 2A 1 1 d
Thus u  2 A    then 2u  2 A     x 
d /2 d  d x d 5

H.No. 40‐D, Ground Floor, Jia Sarai, Near IIT, Hauz Khas, New Delhi‐110016 
Phone: 011‐26865455/+91‐9871145498
Website: www.physicsbyfiziks.com  | Email: fiziks.physics@gmail.com  
20 
fiziks
Institute for NET/JRF, GATE, IIT‐JAM, M.Sc. Entrance, JEST, TIFR and GRE in Physics 
 
NET/JRF–(DEC-2014)
Q49. A charged particle moves in a helical path under the influence of a constant
magnetic field. The initial velocity is such that the component along the
magnetic field is twice the component in the plane normal to the magnetic l
field.
The ratio  / R of the pitch  to the radius R of the helical path is 2R
(a)  / 2 (b) 4 (c) 2 (d) 
Ans: (b)
Solution: v  2v

2 R 2 R l
Pitch of the helix l  vT  v  2v  4 R   4
v v R
Q50. A parallel beam of light of wavelength  is incident normally on a thin polymer film
with air on both sides. If the film has a refractive index n  1 , then second-order bright
fringes can be observed in reflection when the thickness of the film is
(a)  / 4n (b)  / 2n (c) 3 / 4n (d)  / n
Ans: (c)

Solution: For constructive interference: 2nd cos    2m  1
2
For normal incidence   0  and second order  m  1

 3
 2nd cos 0   2 1  1 d 
2 4n
Q51. A solid sphere of radius R has a charge density, given by
 ar 
 r    0 1  
 R
where r is the radial coordinate and  0 , a and R are positive constants. If the

magnitude of the electric field at r  R / 2 is 1.25 times that at r  R , then the value of a
is
(a) 2 (b) 1 (c) 1 / 2 (d) 1 / 4
Ans: (b)

H.No. 40‐D, Ground Floor, Jia Sarai, Near IIT, Hauz Khas, New Delhi‐110016 
Phone: 011‐26865455/+91‐9871145498
Website: www.physicsbyfiziks.com  | Email: fiziks.physics@gmail.com  
21 
fiziks
Institute for NET/JRF, GATE, IIT‐JAM, M.Sc. Entrance, JEST, TIFR and GRE in Physics 
 
r
  1  1  ar 
Solution:  E .d a  Qenc  E  4 r 2   0 1   4 r 2 dr
S
0 0 0  R 

40  2 ar 3  4 0  r 3 ar 4  0  r ar 2 
 r 

 0 0 
 E  4 r  2
r   dr     E    
R   0  3 4R   0  3 4R 

0  R / 2 aR 2 / 4  0  R aR 2 
 Er  R / 2  1.25 Er  R      1.25   
0  3 4R   0  3 4R 

 1 a  5  1 a   1 a   5 5a  5a a 5 1
            
 6 16  4  3 4   6 16   12 16  16 16 12 6
4a 5  2 a 3
     a 1
16 12 4 12
Q52. The electrostatic lines of force due to a system of four point charges
is sketched here. At large distance r , the leading asymptotic
behaviour of the electrostatic potential is proportional to
(a) r (b) r 1
(c) r 2 (d) r 3
Ans: (d)
1
Solution: The given electrostatic line of force is due to a quadrupole. So V  .
r3
Q53. A plane electromagnetic wave incident normally on the surface of a material is partially
reflected. Measurements on the standing wave in the region in front of the interface such
that the ratio of the electric field amplitude at the maxima and the minima is 5. The ratio
of the reflected intensity to the incident intensity is
(a) 4 / 9 (b) 2 / 3 (c) 2 / 5 (d) 1 / 5
Ans: (a)
E0 I  E0 R E 2
Solution:  5  E0 I  E0 R  5  E0 I  E0 R   6 E0 R  4 E0 I  0 R 
E0 I  E0 R E0 I 3
2
I E  4
 R   0R  
I I  E0 I  9

H.No. 40‐D, Ground Floor, Jia Sarai, Near IIT, Hauz Khas, New Delhi‐110016 
Phone: 011‐26865455/+91‐9871145498
Website: www.physicsbyfiziks.com  | Email: fiziks.physics@gmail.com  
22 
fiziks
Institute for NET/JRF, GATE, IIT‐JAM, M.Sc. Entrance, JEST, TIFR and GRE in Physics 
 

Q54. A non-relativistic particle of mass m and charge e , moving with a velocity v and

acceleration a , emits radiation of intensity I . What is the intensity of the radiation
 
emitted by a particle of mass m / 2 , charge 2e , velocity v / 2 and acceleration 2a ?
(a) 16 I (b) 8 I (c) 4 I (d) 2 I
Ans: (a)
q 2 a 2 sin 2  I 2 q22 a22 I 2 4e 2  4a 2
Solution:  I       16  I 2  16 I
r2 I1 q12 a12 I e2 a 2

NET/JRF–(JUNE-2015)
Q55. A Plane electromagnetic wave is travelling along the positive z -direction. The maximum
electric field along the x - direction is 10 V / m . The approximate maximum values of the
power per unit area and the magnetic induction B , respectively, are
(a) 3.3  107 watts / m 2 and 10 tesla
(b) 3.3  107 watts / m 2 and 3.3  108 tesla
(c) 0.265 watts / m 2 and 10 tesla

(d) 0.265 watts / m 2 and 3.3 108 tesla


Ans. (d)
P 1 1
 c 0 E02   3 108  8.86 1012  10   0.132 W / m 2
2
Solution: E0  10V / m, I 
A 2 2
E0 10
B0    3.3  108 Tesla
c 3 10 8

 
Q56.    
Which of the following transformations V , A  V ', A ' of the electrostatic potential

V and the vector potential A is a gauge transformation?
   

(a) V   V  ax, A  A  at kˆ  
(b) V   V  ax, A  A  at kˆ 
   

(c) V   V  ax, A  A  at iˆ  
(d) V   V  ax, A  A  at iˆ 
Ans. (d)
 
Solution: V   V     ax    axt  c
t t
  
   atiˆ  0 . Thus, A  A  atxˆ
H.No. 40‐D, Ground Floor, Jia Sarai, Near IIT, Hauz Khas, New Delhi‐110016 
Phone: 011‐26865455/+91‐9871145498
Website: www.physicsbyfiziks.com  | Email: fiziks.physics@gmail.com  
23 
fiziks
Institute for NET/JRF, GATE, IIT‐JAM, M.Sc. Entrance, JEST, TIFR and GRE in Physics 
 
Q57. Suppose the yz -plane forms a chargeless boundary between two media of permittivities
left and right where left :right  1: 2 , if the uniform electric field on the left is
 
 
Eleft  c iˆ  ˆj  kˆ (where c is a constant), then the electric field on the right Eright is


(a) c 2iˆ  ˆj  kˆ  
(b) c iˆ  2 ˆj  2kˆ 
1   1 1 
(c) c  iˆ  ˆj  kˆ  (d) c  iˆ  ˆj  kˆ 
2   2 2 
Ans. (c) y
1 2
 
Solution: E1  c ˆj  kˆ  E2

1 
D1  D2 1 E1 2 E2  E21  E1
2 x
1 ˆ  1 
 E2  ci  E2  c  iˆ  ˆj  kˆ 
2 2  z
Q58. A proton moves with a speed of 300 m / s in a circular orbit in the xy -plan in a magnetic
field 1 tesla along the positive z - direction. When an electric field of 1 V / m is applied
along the positive y -direction, the center of the circular orbit
(a) remains stationary
(b) moves at 1 m / s along the negative x  direction
(c) moves at 1 m / s along the positive z  direction
(d) moves at 1 m / s along the positive x  direction
z
Ans. (d)

Solution: change particle will deflect in  x -direction with B
E 1 y
v  1 m/ s . 
B 1 E
x
Q59. Consider a rectangular wave guide with transverse dimensions 2 m  1 m driven with an

angular frequency   109 rad / s . Which transverse electric TE  modes will propagate

in this wave guide?


(a) TE10 , TE01 and TE20 (b) TE01 , TE11 and TE20

(c) TE01 , TE10 and TE11 (d) TE01 , TE10 and TE22
H.No. 40‐D, Ground Floor, Jia Sarai, Near IIT, Hauz Khas, New Delhi‐110016 
Phone: 011‐26865455/+91‐9871145498
Website: www.physicsbyfiziks.com  | Email: fiziks.physics@gmail.com  
24 
fiziks
Institute for NET/JRF, GATE, IIT‐JAM, M.Sc. Entrance, JEST, TIFR and GRE in Physics 
 
Ans. (a)

m2 n2
Solution: mn  C 
a 2 b2
c 3 108  3.14
10    4.71 108 rod / sec
a 2
c 3 108  3.14
01    9.42 108 rod / sec
b 1
1 1
11  c  2  10.53 108 rod / sec
a b
2

2c
20   9.72 108 rod / sec
a
4 4
22  c  2  10.5  108 rod / sec
a b
2

Since   10 , 01 , 20

Q60. The electric and magnetic fields in the charge free region z  0 are given by
 
E  r , t   E0 e  k1z cos  k2 x  t  ˆj
  E
B  r , t   0 e  k1z  k1 sin  k2 x  t  iˆ  k2 cos  k2 x  t  kˆ 
  
where  , k1 and k2 are positive constants. The average energy flow in the x -direction is
E02 k2 2 k1z E02 k2 E02 k1 2 k1z 1
(a) e (b) e 2 k1z (c) e (d) c 0 E02 e 2 k1 z
20 0 20 2
Ans. (a)
 1   E 2 e 2 k1 z 
Solution: S 
0

EB  0
0  1
  
k cos  sin   kˆ  k2 cos 2  iˆ  , where   k2 x  t

 k E 2 e 2 k1 z E02 k2 2 k1 z
 S  2 0  e
2 0 20
Q61. A uniform magnetic field in the positive z -direction passes through a circular wire loop
of radius 1 cm and resistance 1  lying in the xy -plane. The field strength is reduced
from 10 tesla to 9 tesla in 1s . The charge transferred across any point in the wire is
approximately
(a) 3.1104 coulomb (b) 3.4 104 coulomb
(c) 4.2 104 coulomb (d) 5.2  104 coulomb
H.No. 40‐D, Ground Floor, Jia Sarai, Near IIT, Hauz Khas, New Delhi‐110016 
Phone: 011‐26865455/+91‐9871145498
Website: www.physicsbyfiziks.com  | Email: fiziks.physics@gmail.com  
25 
fiziks
Institute for NET/JRF, GATE, IIT‐JAM, M.Sc. Entrance, JEST, TIFR and GRE in Physics 
 
Ans. (a)
d dq  1 d A  r 2
Solution:    I     dq   dB  dB
dt dt R R dt R R

 
2
3.14  102
 dq   1  3.14  104 coulomb
1
Q62. A rod of length L carries a total charge Q distributed uniformly. If this is observed in a
frame moving with a speed v along the rod, the charge per unit length (as measured by
the moving observer) is

Q  v2  Q v2 Q Q
(a) 1  2  (b) 1 2 (c) (d)
L c  L c v2  v2 
L 1 L 1  2 
c2  c 
Ans. (c)
0 Q
Solution:   
v2
v2
1 L 1
c2 c2
NET/JRF–(DEC-2015)
Q63. A hollow metallic sphere of radius a , which is kept at a potential V0 has a charge Q at its
centre. The potential at a point outside the sphere, at a distance r from the centre, is
Q Va Q V a2 V0 a
(a) V0 (b)  0 (c)  02 (d)
4 0 r r 4 0 r r r
Ans.: (d)
Q
Solution: Let charge on conductor is Q , then V0 
4 0 a
Q Va
Now V V  0
4 0 r r
Q64. Consider a charge Q at the origin of 3 - dimensional coordinate system. The
flux of the electric field through the curved surface of a cone that has a height
h
h and a circular base of radius R (as shown in the figure) is
R Q
Q Q hQ QR
(a) (b) (c) (d)
0 2 0 R 0 2h 0
Ans.: (b)

H.No. 40‐D, Ground Floor, Jia Sarai, Near IIT, Hauz Khas, New Delhi‐110016 
Phone: 011‐26865455/+91‐9871145498
Website: www.physicsbyfiziks.com  | Email: fiziks.physics@gmail.com  
26 
fiziks
Institute for NET/JRF, GATE, IIT‐JAM, M.Sc. Entrance, JEST, TIFR and GRE in Physics 
 
Q65. Given a uniform magnetic field B  B0 kˆ (where B0 is a constant), a possible choice for
the magnetic vector potential A is

(a) B0 yiˆ (b)  B0 yiˆ 


(c) B0 xjˆ  yiˆ  
(d) B0 xiˆ  yjˆ 
Ans.: (b)
 
Solution: (a)   A   B0 kˆ
 
(b)   A  B0 kˆ
 
(c)   A  0
 
(d)   A  0
Q66. A beam of unpolarized light in a medium with dielectric constant 1 is reflected from a

plane interface formed with another medium of dielectric constant 2  3 1 . The two

media have identical magnetic permeability. If the angle of incidence is 600 , then the
reflected light
(a) is plane polarized perpendicular to the plane of incidence
(b) is plane polarized parallel to the plane of incidence
(c) is circularly polarized
(d) has the same polarization as the incident light
Ans.: (a)
n  I
Solution:  B  tan 1  2 
1
 n1 
2
 2 
 B  tan 1 
  
  tan
1
3  
 1
  B  600 (hence reflected light is plane polarized perpendicular to plane of incidence))

Q67. A small magnetic needle is kept at  0, 0  with its moment along the x -axis. Another

small magnetic needle is at the point 1,1 and is free to rotate in the xy - plane. In

equilibrium the angle  between their magnetic moments is such that


1
(a) tan   (b) tan   0 (c) tan   3 (d) tan   1
3
Ans.: (c)
H.No. 40‐D, Ground Floor, Jia Sarai, Near IIT, Hauz Khas, New Delhi‐110016 
Phone: 011‐26865455/+91‐9871145498
Website: www.physicsbyfiziks.com  | Email: fiziks.physics@gmail.com  
27 
fiziks
Institute for NET/JRF, GATE, IIT‐JAM, M.Sc. Entrance, JEST, TIFR and GRE in Physics 
 
      mm
 
Solution: U  0 3  m1  m2  3  m1  rˆ  m2  rˆ    U  0 1 3 2 cos   3cos 450 cos   450 
4 r 4 r
For stable position energy is minimum i.e.
 r̂ y
 mm  m
U 
3
 
2
 0  0 1 3 2   sin   sin   450   0 r
 4 r  2 
450 
 x
3  sin  cos   m1
 sin       tan   3
2 2 2 
so, option (c) is correct .

Q68. A dipole of moment p , oscillating at frequency  , radiates spherical waves. The vector
potential at large distance is
   eikr 
A  r   0 i p
4 r
1  
To order   the magnetic field B at a point r  rnˆ is
r
0  2  eikr 0  2  eikr
(a)    nˆ
n
ˆ  p (b)   n
ˆ  p 
4 C r 4 C r
0 2   eikr  0  2  eikr
(c)   k  nˆ  p  p (d)  p
4 r 4 C r
Ans.: (b)
 
Solution: Let p  pzˆ , then B must be in ˆ direction.

Check nˆ  p  rˆ  zˆ  ˆ . So, correct option is (b).
Q69. The frequency dependent dielectric constant of a material is given by
A
    1  2
0   2  i
where A is a positive constant, 0 is the resonant frequency and  is the damping

coefficient. For an electromagnetic wave of angular frequency   0 , which of the


following is true? (Assume that  1 ).
0
(a) There is negligible absorption of the wave
(b) The wave propagation is highly dispersive
(c) There is strong absorption of the electromagnetic wave
(d) The group velocity and the phase velocity will have opposite sign
H.No. 40‐D, Ground Floor, Jia Sarai, Near IIT, Hauz Khas, New Delhi‐110016 
Phone: 011‐26865455/+91‐9871145498
Website: www.physicsbyfiziks.com  | Email: fiziks.physics@gmail.com  
28 
fiziks
Institute for NET/JRF, GATE, IIT‐JAM, M.Sc. Entrance, JEST, TIFR and GRE in Physics 
 
Ans.: (a)
Solution: When   0 , there is negligible absorption of the wave.

NET/JRF–(JUNE-2016)
Q70. Four equal charges of Q , each are kept at the vertices of a square of side R . A particle
of mass m and charge Q is placed in the plane of the square at a short distance

a   R  from the centre. If the motion of the particle is confined to the plane, it will

undergo small oscillations with an angular frequency

Q2 Q2
(a) (b)
2 0 R 3 m  0 R 3m

2Q 2 Q2
(c) (d)
 0 R 3 m 4 0 R 3m

Ans: (c)
kQ Q Q
Solution: E1  E2   
 R  R2 
2
E2 E1 y
 a    
 2 4 
a x
Resultant field E12, y  2 E1 cos 
R /2
2kQ  R 2kQ  R
E12, y  a    a   Q R /2 Q
 2  2
3 3
 R  R2 
2 2  R2  2
 a      
 2 4   2 

4 2kQ  R
E12, y  a  
R 3
 2
Q R /2 Q
kQ R
Similarly; E3  E4   2 a
 R  R 
2 2

  a     
 2  4  E3 a E4

2kQ R 
Resultant E34, y  2 E3 cos    a
2 
3
 R  R 
2 2 2

  a    Q Q
 2  4 

H.No. 40‐D, Ground Floor, Jia Sarai, Near IIT, Hauz Khas, New Delhi‐110016 
Phone: 011‐26865455/+91‐9871145498
Website: www.physicsbyfiziks.com  | Email: fiziks.physics@gmail.com  
29 
fiziks
Institute for NET/JRF, GATE, IIT‐JAM, M.Sc. Entrance, JEST, TIFR and GRE in Physics 
 
4 2kQ  R 
 E34, y    a
R 3
2 

4 2kQ  R  R  8 2kQ
Resultant E     a     a    a
R 3
 2  2  R3

8 2 1 2 2Q
E  Qa  E   a
R 3
4 0  0 R 3

2 2Q 2 2 2Q 2
 F  QE   a
 0 R 3  0 mR 3
Q71. Two parallel plate capacitors, separated by distances x and 1.1x respectively, have a
dielectric material of dielectric constant 3.0 inserted between the plates and are
connected to a battery of voltage V . The difference in charge on the second capacitor
compared to the first is
(a) 66% (b) 20% (c) 3.3% (d) 10%
Ans: (d)
3 0 A 3 A
Solution: Q1  C1V1  V , Q2  C2V2  0 V
x 1.1x
 1  3 A
  1  0 V
Q2  Q1 x
 100%  
1.1 
 100  9%
Q1 3  A
0
V
x
Q72. The half space region x  0 and x  0 are filled with dielectric
x0 x0
media of dielectric constants 1 and  2 respectively. There is a 2 1 
uniform electric field in each part. In the right half, the electric field 1 E1

makes an angle 1 to the interface. The corresponding angle  2 in


the left half satisfies 2
(a) 1 sin  2   2 sin 1 (b) 1 tan  2   2 tan 1 
E2
(c) 1 tan 1   2 tan  2 (d) 1 sin 1   2 sin  2
Ans: (c)

H.No. 40‐D, Ground Floor, Jia Sarai, Near IIT, Hauz Khas, New Delhi‐110016 
Phone: 011‐26865455/+91‐9871145498
Website: www.physicsbyfiziks.com  | Email: fiziks.physics@gmail.com  
30 
fiziks
Institute for NET/JRF, GATE, IIT‐JAM, M.Sc. Entrance, JEST, TIFR and GRE in Physics 
 

E1
tan 1 E E 
Solution:
tan  2
 1  1
E E2

 E1  E2 
2

E2

E1  2 tan 1  2
D1  D2  1 E1   2 E2      1 tan 1   2 tan  2
E2 1

tan  2 1

Q73. The x - and z -components of a static magnetic field in a region are Bx  B0 x 2  y 2  


and Bz  0 , respectively. Which of the following solutions for its y -component is
consistent with the Maxwell equations?
(a) By  B0 xy (b) By  2 B0 xy

1 

(c) By   B0 x 2  y 2  (d) By  B0  x3  xy 2 
3 
Ans: (b)

 
Solution: Bx  B0 x 2  y 2 , Bz  0

  B By Bz By B


  B  0  x   0   x  2 B0 x  By  2 B0 xy
x y z y x
Q74. A magnetic field B is Bzˆ in the region x  0 and zero elsewhere. A rectangular loop, in
the xy -plane, of sides l (along the x -direction) and h (along the y - direction) is
inserted into the x  0 region from the x  0 region at constant velocity v  vxˆ . Which of
the following values of l and h will generate the largest EMF?
(a) l  8, h  3 (b) l  4, h  6 (c) l  6, h  4 (d) l  12, h  2
Ans: (b) z
Solution: m  Bhx h
l
 dm v
  Bvh  h
dt y

H.No. 40‐D, Ground Floor, Jia Sarai, Near IIT, Hauz Khas, New Delhi‐110016 
Phone: 011‐26865455/+91‐9871145498
Website: www.physicsbyfiziks.com  | Email: fiziks.physics@gmail.com  
31 
fiziks
Institute for NET/JRF, GATE, IIT‐JAM, M.Sc. Entrance, JEST, TIFR and GRE in Physics 
 
Q75. Consider a sphere S1 of radius R which carries a uniform charge
S2
R S
of density  . A smaller sphere S2 of radius a  is cut out and 1 P
2 
b 
removed from it. The centres of the two spheres are separated by r
 nR ˆ
the vector b  , as shown in the figure. The electric field at a
2
point P inside S2 is

R R  R a 
(a) nˆ (b)  r  na
ˆ  (c) nˆ (d) r
3 0 3 0 a 6 0 3 0 R
Ans: (c)
  
Solution: Electric field at P due to S1 is E1  r
3 0 S2
   S1 
Electric field at P due to S2 (assume   ) is E2  r r
3 0 
   b  P
        
Thus E  E1  E2   r  r  ;  b  r  r  r  r  b r
3 0
   R   R 
E b nˆ  b  nˆ 
3 0 6 0  2 
Q76. The value of the electric and magnetic fields in a particular reference frame (in Gaussian
units) are E  3xˆ  4 yˆ and B  3 zˆ respectively. An inertial observer moving with respect

to this frame measures the magnitude of the electric field to be E   4 . The magnitude of

the magnetic field B measured by him is

(a) 5 (b) 9 (c) 0 (d) 1


Ans: (c)
Solution:  E 2  B 2  E 2  B2  constant   9  16   9  16  B2  B  0

Q77. A loop of radius a , carrying a current I , is placed in a uniform magnetic field B . If the
 
normal to the loop is denoted by n̂ , the force F and the torque T on the loop are
     
(a) F  0 and T   a 2 I n̂ B (b) F  0 I  B
4
        1 
(c) F  0 I  B and T  I nˆ  B (d) F  0 and T  IB
4  0 0
H.No. 40‐D, Ground Floor, Jia Sarai, Near IIT, Hauz Khas, New Delhi‐110016 
Phone: 011‐26865455/+91‐9871145498
Website: www.physicsbyfiziks.com  | Email: fiziks.physics@gmail.com  
32 
fiziks
Institute for NET/JRF, GATE, IIT‐JAM, M.Sc. Entrance, JEST, TIFR and GRE in Physics 
 
Ans: (a)

Solution: In uniform field F  0
   
Torque T  m  B   a 2 Inˆ  B
Q78. A waveguide has a square cross-section of side 2a . For the TM modes of wave vector k ,
the transverse electromagnetic modes are obtained in terms of a function   x, y  which

obeys the equation


 2 2   2 
 2    2  k 2    x, y   0
 x y  c
2


with the boundary condition    a, y     x,  a   0 . The frequency  of the lowest

mode is given by
 4 2   2 
(a)  2  c 2  k 2  2  (b)  2  c 2  k 2  2 
 a   a 

 2 2   2 2 
(c)   c  k  2 
2 2
(d)   c  k  2 
2 2

 2a   4a 

Ans: (c)
Solution: c 2 k 2   2  mn
2
  2  c 2 k 2  mn
2

 m2 n2   1 1 
 mn
2
 c 2 2  2  2   112  c 2 2   
a b    2a   2a  
2 2

1 c 2 2 2 2 2 
 112  c 2 2     2
 c  k  
2a 2 2a 2  2a 2 

H.No. 40‐D, Ground Floor, Jia Sarai, Near IIT, Hauz Khas, New Delhi‐110016 
Phone: 011‐26865455/+91‐9871145498
Website: www.physicsbyfiziks.com  | Email: fiziks.physics@gmail.com  
33 
fiziks
Institute for NET/JRF, GATE, IIT‐JAM, M.Sc. Entrance, JEST, TIFR and GRE in Physics 
 
NET/JRF -(DEC-2016)
Q79. A screen has two slits, each of width w with their centres at a distance 2 w apart. It is
illuminated by a monochromatic plane wave travelling along the x -axis.
The intensity of the interference pattern, measured on a distant screen, at an angle
n
 to the x -axis is
w
w
(a) zero for n  1, 2,3... 
w x
(b) maximum for n  1, 2,3...
1 3 5 w
(c) maximum for n  , , ...
2 2 2
(d) zero for n  0 only
Ans. : (a)
Solution: maximum for n  0 and zero for n  1, 2,3... .
Q80. The electric field of an electromagnetic wave is

E  z , t   E0 cos  kz  t  iˆ  2 E0 sin  kz  t  ˆj

where  and k are positive constants. This represents


(a) a linearly polarised wave travelling in the positive z -direction
(b) a circularly polarised wave travelling in the negative z -direction
(c) an elliptically polarised wave travelling in the negative z -direction
(d) an unpolarised wave travelling in the positive z -direction
Ans. : (c)
Solution: Amplitude along iˆ is E0 and along ĵ is 2 E0 . So resultant wave is elliptically
polarised

H.No. 40‐D, Ground Floor, Jia Sarai, Near IIT, Hauz Khas, New Delhi‐110016 
Phone: 011‐26865455/+91‐9871145498
Website: www.physicsbyfiziks.com  | Email: fiziks.physics@gmail.com  
34 
fiziks
Institute for NET/JRF, GATE, IIT‐JAM, M.Sc. Entrance, JEST, TIFR and GRE in Physics 
 
Q81. A conducting circular disc of radius r and resistivity  rotates with an angular velocity
 in a magnetic field B perpendicular to it. A voltmeter is connected as shown in the
figure below. Assuming its internal resistance to be infinite, the reading on the voltmeter
(a) depends on  , B, r and 
B
(b) depends on  , B and r but not on 
(c) is zero because the flux through the loop is not
changing
r V
(d) is zero because a current the flows in the direction
of B
Ans. : (b)
Solution: Force experienced by charge is
  
 
F  q v  B and v  r

Q82. The charge per unit length of a circular wire of radius a in the xy -plane, with its centre at
the origin, is   0 cos  , where 0 is a constant and the angle  is measured from the

positive x -axis. The electric field at the centre of the circle is


 0  0
(a) E   iˆ (b) E  iˆ
4 0  4 0 
 0  0
(c) E   ˆj (d) E  kˆ
4 0  4 0 
Ans. : (a)
y
Solution: At centre O , direction of field is  x̂ . 0
So best option is (a)  0
o x
0

H.No. 40‐D, Ground Floor, Jia Sarai, Near IIT, Hauz Khas, New Delhi‐110016 
Phone: 011‐26865455/+91‐9871145498
Website: www.physicsbyfiziks.com  | Email: fiziks.physics@gmail.com  
35 
fiziks
Institute for NET/JRF, GATE, IIT‐JAM, M.Sc. Entrance, JEST, TIFR and GRE in Physics 
 
Q83. A pair of parallel glass plates separated by a distance d is illuminated by white light as
shown in the figure below. Also shown in the graph of the intensity of the reflected light
I as a function of the wavelength  recorded by a spectrometer.
1
spectrometer
0.8

Intensity
partially incident 0.6
reflecting mirror white light 0.4
0.2
0
d air gap 490 500 510 520 530
glass plates    m
Assuming that the interference takes place only between light reflected by the bottom
surface of the top plate and the top surface of bottom plate, the distance d is closest to
(a) 12  m (b) 24  m (c) 60  m (d) 120  m
Ans. : (d)
 1
Solution: For constructive interference of reflected light, 2d cos   n    .
 2
 495 m
First maxima occurs at   495 m ,   00 and n  0 . Thus, d    120 m
4 4
Q84. Suppose that free charges are present in a material of dielectric constant  10 and
resistivity   1011   m . Using Ohm’s law and the equation of continuity for charge, the
1
time required for the charge density inside the material to decay by is closest to
e
(a) 106 S (b) 106 S (c) 1012 S (d) 10 S
Ans. : (d)
t
 8.8 1012  10
0r 1
Solution:  f  t    f  0  e ;   

,  10sec ,  
  10 11

Q85. A particle with charge q moves with a uniform angular velocity  in a circular orbit of
radius a in the xy - plane, around a fixed charge  q , which is at the centre of the orbit at

 0, 0, 0  . Let the intensity of radiation at the point  0, 0, R  be I1 and at  2 R, 0, 0  be ‘ I 2

I2
The ratio for R  a , is
I1
1 1
(a) 4 (b) (c) (d) 8
4 8
H.No. 40‐D, Ground Floor, Jia Sarai, Near IIT, Hauz Khas, New Delhi‐110016 
Phone: 011‐26865455/+91‐9871145498
Website: www.physicsbyfiziks.com  | Email: fiziks.physics@gmail.com  
36 
fiziks
Institute for NET/JRF, GATE, IIT‐JAM, M.Sc. Entrance, JEST, TIFR and GRE in Physics 
 
Ans. : (c)
I 2 r13 R3 1
Solution:  3 
I1 r2  2 R  3
8

Q86. A parallel plate capacitor is formed by two circular conducting plates of radius a
separated by a distance d , where d  a . It is being slowly charged by a current that is
nearly constant. At an instant when the current is I , the magnetic induction between the
a
plates at a distance from the centre of the plate, is
2
0 I 0 I 0 I 0 I
(a) (b) (c) (d)
a 2 a a 4 a
Ans. : (d)
  Ir P
Solution: B  0 2 I a
2 a r
 I a
B  0 at r 
4 a 2
Q87. Two uniformly charged insulating solid spheres A and B , both of radius a , carry total
charges Q and Q , respectively. The spheres are placed touching each other as shown
in the figure.    
 A   B 
If the potential at the centre of the sphere A is VA and that at the
   
centre of B is VB then the difference VA  VB is

Q Q Q Q
(a) (b) (c) (d)
4 0 a 2 0 a 2 0 a 4 0 a
Ans. : (c)
3Q Q Q
Solution: VA   
8 0 a 4 0  2a  4 0 a

3Q Q Q
VB   
8 0 a 4 0  2a  4 0 a

Q
VA  VB 
2 0 a

H.No. 40‐D, Ground Floor, Jia Sarai, Near IIT, Hauz Khas, New Delhi‐110016 
Phone: 011‐26865455/+91‐9871145498
Website: www.physicsbyfiziks.com  | Email: fiziks.physics@gmail.com  
37 
fiziks
Institute for NET/JRF, GATE, IIT‐JAM, M.Sc. Entrance, JEST, TIFR and GRE in Physics 
 
NET/JRF -(JUNE -2017)
Q88. Two long hollow co-axial conducting cylinders of radii R1 and R2  R1  R2  are placed

in vacuum as shown in the figure below.

2 R1 2 R2

The inner cylinder carries a charge   per unit length and the outer cylinder carries a
charge  per unit length. The electrostatic energy per unit length of this system is
2 2
(a)
 0
ln  R2 / R1  (b)
4 0
 R22 / R12 

2 2
(c) ln  R2 / R1  (d) ln  R2 / R1 
4 0 2 0
Ans. : (c)
  
Solution: r  R1 , E1  0 ; R1  r  R2 , E2  rˆ
2 0 r

r  Rz , E3  0

0  Rz 2
W  E 2 dz  0
2 all spce 2 R1 4 2 02 r 2
 2 rldr

W 0 2 R2 1 2 R 
l
 
2 2 02 R1 r
dr  ln  2 
4 0  R1 

Q89. A set of N concentric circular loops of wire, each carrying a steady current I in the
same direction, is arranged in a plane. The radius of the first loop is r1  a and the radius

of the n th loop is given by rn  nrn 1 . The magnitude B of the magnetic field at the centre

of the circles in the limit N   , is


(a) 0 I  e 2  1 / 4 a (b) 0 I  e  1 /  a

(c) 0 I  e 2  1 / 8a (d) 0 I  e  1 / 2a

Ans. : (d)

H.No. 40‐D, Ground Floor, Jia Sarai, Near IIT, Hauz Khas, New Delhi‐110016 
Phone: 011‐26865455/+91‐9871145498
Website: www.physicsbyfiziks.com  | Email: fiziks.physics@gmail.com  
38 
fiziks
Institute for NET/JRF, GATE, IIT‐JAM, M.Sc. Entrance, JEST, TIFR and GRE in Physics 
 
 I1 1 1 1
Solution: B  0     ........ 
2  r1 r2 r3 rn 

r1  a

rn  nrn 1

r1  r0  a

r2  2r1  2a

r3  3r2  3.2a

r4  4r3  4.3.2a

0 I  1 1 1 
B 1     ....... 
2a  2 3.2 4.3.2 

0 I  1 N
B  
2a  n 1 n 

xn 
1 
1 
1
ex    e    1      e 1
n 0 n n 0 n n 1 n n 1 n

 N 1 I
lim     e  1  B  0  e  1
N 
 n l n  2a

Q90. An electromagnetic wave (of wavelength 0 in free space) travels through an absorbing

I
medium with dielectric permittivity given by    R  i I where  3 . If the skin
R
0
depth is , the ratio of the amplitude of electric field E to that of the magnetic field B ,
4
in the medium (in ohms) is
(a) 120  (b) 377 (c) 30 2  (d) 30 
Ans. : (d)
1 0 I 
Solution: d   ,  3
 4 R 
1/ 2
    
2

 1  
  
  
1
2   
 

H.No. 40‐D, Ground Floor, Jia Sarai, Near IIT, Hauz Khas, New Delhi‐110016 
Phone: 011‐26865455/+91‐9871145498
Website: www.physicsbyfiziks.com  | Email: fiziks.physics@gmail.com  
39 
fiziks
Institute for NET/JRF, GATE, IIT‐JAM, M.Sc. Entrance, JEST, TIFR and GRE in Physics 
 
  4 2 4
     
2 0  0
1/ 2
   
2

K  k       1  
2 2

     
 
E0   1 1
   
B0 K 2  4
1/ 2
    
2
2
2

   1      0
    
 
0 0  2 c / 0 c E c
     0
8 8 4 H0 4

E 4  107  3  108
   30
H0 4

Q91. The vector potential A  ke  at rrˆ (where a and k are constants) corresponding to an

electromagnetic field is changed to A   ke  at rrˆ . This will be a gauge transformation if
the corresponding change     in the scalar potential is

(a) akr 2e  at (b) 2akr 2 e at (c) akr 2e  at (d) 2akr 2e at


Ans. : (c)
Solution: Gauge Transformation
   
A  A   ,     
t
   
A'  A  2ke  at rrˆ    rˆ
r

   ke at r 2   kae  at r 2
t

  '     kae  at r 2
t
Q92. An electron is decelerated at a constant rate starting from an initial velocity u (where
u  c ) to u / 2 during which it travels a distance s . The amount of energy lost to
radiation is
0 e 2u 2 0 e 2u 2 0 e 2u 0 e 2u
(a) (b) (c) (d)
3 mc 2 s 6 mc 2 s 8 mcs 16 mcs
H.No. 40‐D, Ground Floor, Jia Sarai, Near IIT, Hauz Khas, New Delhi‐110016 
Phone: 011‐26865455/+91‐9871145498
Website: www.physicsbyfiziks.com  | Email: fiziks.physics@gmail.com  
40 
fiziks
Institute for NET/JRF, GATE, IIT‐JAM, M.Sc. Entrance, JEST, TIFR and GRE in Physics 
 
Ans. : (d)
0 q 2 a 2
Solution: Total power radiated P 
6 c
0 e 2 a 2  e2 a 2 u
Total energy radiated in time t is E  P  t  t  0 
6 c 6 c 2a
 u u 
 v  u  at  2  u  at  t  2a 

0 e2 au
E
12 c
E 2E
Fraction of initial K .E. lost due to radiation  
1
mu 2 mu 2
2
2 0e 2 au 0 e2 a
  
mu 2 12 c 6 mcu
 1 2 u 1 u2 u 2 u 2 3u 2 3u 2 
 s  ut  at  u   a      a 
 2 2a 2 4 a 2 2 a 8a 8a 8s 

0 e2 3u 2  e 2u
   0
6 mcu 8s 16 mcs
Q93. The figure below describes the arrangement of slits and screens in a Young’s double slit
experiment. The width of the slit in S1 is a and the slits in S2 are of negligible width.
S1 S2
screen

 b

d
If the wavelength of the light is  , the value of d for which the screen would be dark is
2 2 2
a b a ab ab
(a) b    1 (b)   1 (c)   (d)
 2  2  
Ans. : (d)

Solution: If the path difference Op2  Op1 
2

H.No. 40‐D, Ground Floor, Jia Sarai, Near IIT, Hauz Khas, New Delhi‐110016 
Phone: 011‐26865455/+91‐9871145498
Website: www.physicsbyfiziks.com  | Email: fiziks.physics@gmail.com  
41 
fiziks
Institute for NET/JRF, GATE, IIT‐JAM, M.Sc. Entrance, JEST, TIFR and GRE in Physics 
 
The minima of the interference pattern produced by O will fall on the maxima produced
1/ 2
 2  b a 2  1 b a
2

by O Now OP2   d      d    P1


  2 2   2d  2 2 
1/ 2
 2  b a 2  1 b a
2
a / 2 O b
OP1   d      d   
  2 2   2d  2 2  a/2 O

ab
 OP2  OP1   d  b, a  P2
2d
 ab ab
Thus  d  d
2 2d 
Q94. A constant current I is flowing in a piece of wire that is bent into a loop as shown in the
figure. y
2b

2b a a
2b
2a
2a O x
b b
The magnitude of the magnetic field at the point O is
0 I a 0 I  1 1 
(a) ln   (b)   
4 5  b  4 5  a b 
0 I  1  0 I  1 
(c)   (d)  
4 5  a  4 5  b 
Ans. : (b)
 I
Solution: B  0  sin  2  sin 1  ˆ
4 d
Magnetic field due to left and right segment of 2a
d 1 2
 I  2a 
B2 a  0  
4 a  5a  I
Field due to upper segment of 2a
0 I  a a 
   
4  2a   5a 5a 
H.No. 40‐D, Ground Floor, Jia Sarai, Near IIT, Hauz Khas, New Delhi‐110016 
Phone: 011‐26865455/+91‐9871145498
Website: www.physicsbyfiziks.com  | Email: fiziks.physics@gmail.com  
42 
fiziks
Institute for NET/JRF, GATE, IIT‐JAM, M.Sc. Entrance, JEST, TIFR and GRE in Physics 
 
 I 2 0 I 1
Net field B2 a  2  0   
4 a 5 4 a 5
0 I
B2 a  5  (inward)
4 a
0 I
similarly, B2b  5  (outward)
4 b
0 I 1 1
Net field B  B2 a  B2b  5  
4 a b
Q95. The charge distribution inside a material of conductivity  and permittivity  at initial
time t  0 is   r , 0   0 , a constant. At subsequent times   r , t  is given by

 t  1    t 
(a) 0 exp    (b) 0 1  exp   
  2    
0 t
(c) (d) 0 cosh
   t  
1  exp    
  
Ans. : (a)
       
Solution: J f   E , .E  f , .J f   f
 E
    
 .E   f  f    f
t t 
     
  f  t   0 exp   f    f  t   0 exp  t
     

H.No. 40‐D, Ground Floor, Jia Sarai, Near IIT, Hauz Khas, New Delhi‐110016 
Phone: 011‐26865455/+91‐9871145498
Website: www.physicsbyfiziks.com  | Email: fiziks.physics@gmail.com  
43 
fiziks
Institute for NET/JRF, GATE, IIT‐JAM, M.Sc. Entrance, JEST, TIFR and GRE in Physics 
 
QUANTUM MECHANICS SOLUTIONS
NET/JRF (JUNE-2011)
 1 
Q1. The wavefunction of a particle is given by    0  i1  where 0 and 1 are the
 2 
normalized eigenfunctions with energies E0 and E1 corresponding to the ground state

and first excited state, respectively. The expectation value of the Hamiltonian in the state
 is
E0 E0 E0  2 E1 E0  2 E1
(a)  E1 (b)  E1 (c) (d)
2 2 3 3
Ans: (d)
1  H E 0  2 E1
Solution:   0  i1 and H  
2  3

Q2. The energy levels of the non-relativistic electron in a hydrogen atom (i.e. in a Coulomb
potential V r   1 / r ) are given by E nlm  1 / n 2 , where n is the principal quantum

number, and the corresponding wave functions are given by  nlm , where l is the orbital

angular momentum quantum number and m is the magnetic quantum number. The spin
of the electron is not considered. Which of the following is a correct statement?
(a) There are exactly  2l  1 different wave functions nlm , for each Enlm .

(b) There are l  l  1 different wave functions  nlm , for each Enlm .

(c) Enlm does not depend on l and m for the Coulomb potential.

(d) There is a unique wave function  nlm for each Enlm .


Ans: (c)
  
Q3. The Hamiltonian of an electron in a constant magnetic field B is given by H    B .

where  is a positive constant and    1 ,  2 ,  3  denotes the Pauli matrices. Let

  B /  and I be the 2  2 unit matrix. Then the operator e i H t /  simplifies to


   
 t i  B  t i  B
(a) I cos  sin (b) I cos  t  sin  t
2 B 2 B
   
i  B i  B
(c) I sin  t  cos  t (d) I sin 2 t  cos 2 t
B B
H.No. 40‐D, Ground Floor, Jia Sarai, Near IIT, Hauz Khas, New Delhi‐110016 
Phone: 011‐26865455/+91‐9871145498
Website: www.physicsbyfiziks.com  | Email: fiziks.physics@gmail.com  

fiziks
Institute for NET/JRF, GATE, IIT‐JAM, M.Sc. Entrance, JEST, TIFR and GRE in Physics 
 
Ans: (b)
  
Solution: H   B where    1 ,  2 ,  3  are pauli spin matrices and B are constant magnetic
 
 


 
field.    1iˆ,  2 ˆj ,  3 kˆ , B  Bx iˆ  By ˆj  Bz kˆ and Hamiltonion H    B in matrices

form is given by
 Bz Bx  iBy 
H   .
 Bx  iBy  Bz 

Eigenvalue of given matrices are given by   B and   B . H matrices are not diagonals

so e i H t /  is equivalent to
 iBt 
1  e 0 
S   i  Bt 
S
 0  
 e 
where S is unitary matrices
 1 1 
 2 2 
and S 1  S   .
 1

1 
 
2 2
 1 1  i Bt  1 1 
 iBt   2  
e 0 
 2   e  0 
 2 2 
S 1   i  Bt 
S  , where   B /  .
 0   1 1    i  Bt 
  1 1 
 e      0 e    
 2 2   2 2

 cos  t i sin  t 
eiHt /    , which is equivalent to I cos  t  i x sin  t can be written
 i sin  t cos  t 
  
i  B i  B
as I cos  t  sin  t , where  x 
B B
Q4. If the perturbation H   ax , where a is a constant, is added to the infinite square well
potential
0 for 0  x  
V  x  
 otherwise.
The correction to the ground state energy, to first order in a , is
a a a
(a) (b) a (c) (d)
2 4 2
H.No. 40‐D, Ground Floor, Jia Sarai, Near IIT, Hauz Khas, New Delhi‐110016 
Phone: 011‐26865455/+91‐9871145498
Website: www.physicsbyfiziks.com  | Email: fiziks.physics@gmail.com  

fiziks
Institute for NET/JRF, GATE, IIT‐JAM, M.Sc. Entrance, JEST, TIFR and GRE in Physics 
 
Ans: (a)
 
a2 x a 2 x
Solution: E   H ' 0 dx 
1 *
0  x sin
2
dx   0  sin .
   
0
0 0
2

Q5. A particle in one dimension moves under the influence of a potential V  x   ax 6 , where
a is a real constant. For large n the quantized energy level En depends on n as:

(a) En ~ n3 (b) En ~ n 4 / 3 (c) En ~ n6 / 5 (d) En ~ n3 / 2


Ans: (d)
p x2 p x2
 ax 6 and p x  2mE  ax 6  2 .  
1
Solution: V  x   ax , H  6
 ax , E 
6

2m 2m
According to W.K.B approximation pdx  nh

  2m  E  ax 
1/ 2
6
dx  n

We can find this integration without solving the integration


Px
1 2mE
1/ 6
E
2 2 6 1/ 6
p p x 6
E  ax 6   1  x    at p x  0 .
x x
 E / a  E / a 
2m 2mE E / a a x
Area of Ellipse =  (semi major axis  semiminor axis)
 2mE
1
E
3
6
  2mE     n  E  n 2 .
a
1   
Q6. (A) In a system consisting of two spin particles labeled 1 and 2, let S 1   1 and
2 2

S 2    2  denote the corresponding spin operators. Here    x ,  y ,  z  and
  
2
 x ,  y ,  z are the three Pauli matrices.

In the standard basis the matrices for the operators S x1S y2  and S y1S x2  are respectively,

 2 1 0   2   1 0  2  i 0  2   i 0 
(a)  ,   (b)  ,  
4  0  1 4  0 1  4 0  i 4  0 i 
0 1 0 0  0 i 0 0
0 0 0  i 2 0 0 0  i    
2  0 0 i 0 ,   0 0 i 0  2 1 0 0 0  2 i 0 0 0
(c)  0  4 0 0 0
(d) ,
4 0  i 0 i 4 0 0 0  i 4 0 0 0 1
i 0 0 0  i
 0 0 0  
0
  
 0 i 0  0
 0 1 0 
H.No. 40‐D, Ground Floor, Jia Sarai, Near IIT, Hauz Khas, New Delhi‐110016 
Phone: 011‐26865455/+91‐9871145498
Website: www.physicsbyfiziks.com  | Email: fiziks.physics@gmail.com  

fiziks
Institute for NET/JRF, GATE, IIT‐JAM, M.Sc. Entrance, JEST, TIFR and GRE in Physics 
 
Ans: (c)
0 0 0 i 
 
2  0 1  0 i  2  0 0 i 0 
Solution: Sx1Sy2     
4 1 0 i 0  4 0 i 0 0 
 i 
0 
 0 0

0 0 0 i 
i   0  
2
0 1  02
0 i 0 
S y1 S x 2   i  
4 0  1 0 4 0 i 0 0 
 
i 0 0 0 
(B) These two operators satisfy the relation
 
(a) S x1S y2  , S y1S x2   S z1S z2   
(b) S x1S y2  , S y1S x2   0

(c) S  S   , S  S     iS  S  
x
1
y
2
y
1
x
2
z
1
z
2
(d) S  S   , S  S     0
x
1
y
2
y
1
x
2

Ans: (d)
Solution: We have matrix Sx1Sy2 and Sy1Sx2 from question 6(A) so commutation is given by

S   S   , S   S     0 .
x
1
y
2
y
1
x
2

NET/JRF (DEC-2011)

Q7. The energy of the first excited quantum state of a particle in the two-dimensional

potential V  x, y  
1
2
 
m 2 x 2  4 y 2 is

3 5
(a) 2 (b) 3 (c)  (d) 
2 2
Ans: (d)
 1  1
Solution: V  x, y   m 2 x 2  4 y 2   m 2 x 2  m 4 2 y 2 , E   n x     n y  2
1 1 1
2 2 2  2  2
 1 3
For ground state energy n x  0, n y  0  E   2 
2 2 2
3 5
First exited state energy n x  1, n y  0    
2 2

H.No. 40‐D, Ground Floor, Jia Sarai, Near IIT, Hauz Khas, New Delhi‐110016 
Phone: 011‐26865455/+91‐9871145498
Website: www.physicsbyfiziks.com  | Email: fiziks.physics@gmail.com  

fiziks
Institute for NET/JRF, GATE, IIT‐JAM, M.Sc. Entrance, JEST, TIFR and GRE in Physics 
 
Q8. Consider a particle in a one dimensional potential that satisfies V  x   V  x  . Let  0

and 1 denote the ground and the first excited states, respectively, and let

   0  0   1  1 be a normalized state with  0 and  1 being real constants. The

expectation value x of the position operator x in the state  is given by

(a)  02  0 x  0   12  1 x  1 (b)  0 1   0 x  1   1 x  0 
(c)  02   12 (d) 2 0 1
Ans: (b)
Solution: Since V  x   V  x  so potential is symmetric.

 0 x 0  0 ,  1 x 1  0

 x    0  0   1  1    0  0   1  1    01   0 x  1   1 x  0 

Q9. The perturbation H '  bx 4 , where b is a constant, is added to the one dimensional

harmonic oscillator potential V  x  


1
m 2 x 2 . Which of the following denotes the
2
correction to the ground state energy to first order in b ?
[Hint: The normalized ground state wave function of the one dimensional harmonic

 m 
1/ 4

is  0   e  m x
2
/ 2
oscillator potential  . You may use the following
  

 1
1
n
integral  x e 2n  ax 2
dx  a 2
 n   ].
  2

3b 2 3b 2 3b 2 15b 2


(a) (b) (c) (d)
4m 2  2 2m 2  2 2 m 2 2 4m 2  2
Ans: (a)

Solution: H '  bx 4 , V  x  
1
m 2 x 2 .
2
m x 2
 m 
1/ 4

Correction in ground state is given by E   0 H ' 0 1
where 0    e 2
.
  
0

H.No. 40‐D, Ground Floor, Jia Sarai, Near IIT, Hauz Khas, New Delhi‐110016 
Phone: 011‐26865455/+91‐9871145498
Website: www.physicsbyfiziks.com  | Email: fiziks.physics@gmail.com  

fiziks
Institute for NET/JRF, GATE, IIT‐JAM, M.Sc. Entrance, JEST, TIFR and GRE in Physics 
 
1 1
mx 2 
 m  2

 m  2 2 2  mx
 
 2
E   bx  0 dx     b x e dx    b  x e
1 * 4 4 
dx
     
0 0
 


 n 1/ 2  1
 e dx    n  2 
2 n x 2
It is given in the equation x

m
Thus n  2 and  

1 1 1
 2 
 m  2 2 2  mx  m  2  m 
   1
2 2
 E 01    b  x e dx  b   2  
   
       2
1 5
 m  2  m  2 5 3 b 2
 E  b1

     
.
2 4 m 2 2
0

Q10. Let 0 and 1 denote the normalized eigenstates corresponding to the ground and first

excited states of a one dimensional harmonic oscillator. The uncertainty  p in the

state
1
0  1  , is
2

(a) p  m / 2 (b) p  m / 2

(c) p  m (d) p  2m


Ans: (c)
m
Solution:  
1
2
 0  1  , p  i 2 a†  a  
a†  
1
2
 1 1  2 2  and a  
1
2
0  10 

p i
m
2

 a†  a   0 , p 2  
m † 2
2

a  a 2   2 N  1  
 m  †2 m  m  1 
p2  a  a2  2N  1   2N  1   2   1  m
2   2 2  2 

p  p2  p  m 
2

H.No. 40‐D, Ground Floor, Jia Sarai, Near IIT, Hauz Khas, New Delhi‐110016 
Phone: 011‐26865455/+91‐9871145498
Website: www.physicsbyfiziks.com  | Email: fiziks.physics@gmail.com  

fiziks
Institute for NET/JRF, GATE, IIT‐JAM, M.Sc. Entrance, JEST, TIFR and GRE in Physics 
 
Q11. The wave function of a particle at time t  0 is given by  0 
1
 u1  u 2  , where
2
u1 and u2 are the normalized eigenstates with eigenvalues E1 and E2

respectively, E 2  E1  . The shortest time after which  t  will become orthogonal to

 0 is

   2 2
(a) (b) (c) (d)
2E 2  E1  E 2  E1 E 2  E1 E 2  E1
Ans: (b)

1  
 iE1t  iE2t
Solution:   0  
1
 u1  u2    t    u1 e
2

 u2 e 

2 
 iE1t  iE2t
1 1
  t  is orthogonal to   0    0   t   0  e 
 e 
0
2 2
 iE1t  iE2t  iE1t  iE2t  E2  E1 
i
e  e  0e   e  e 
 1

 cos
 E2  E1  t  cos   t  
 E2  E1
Q12. A constant perturbation as shown in the figure below acts on a particle of mass m
confined in an infinite potential well between 0 and L .

V0 V0
2
0 L/2 L

The first-order correction to the ground state energy of the particle is


V0 3V0 V0 3V0
(a) (b) (c) (d)
2 4 4 2
Ans: (b)

H.No. 40‐D, Ground Floor, Jia Sarai, Near IIT, Hauz Khas, New Delhi‐110016 
Phone: 011‐26865455/+91‐9871145498
Website: www.physicsbyfiziks.com  | Email: fiziks.physics@gmail.com  

fiziks
Institute for NET/JRF, GATE, IIT‐JAM, M.Sc. Entrance, JEST, TIFR and GRE in Physics 
 
L

V0 2 2 x x
2 L
2
Solution: E   1 V p  1
1
1  sin dx   V0 sin 2 dx
0
2 L L L L L
2

V 1 2x  2V 1  2x 
2 L

E  0  1  cos
1
 dx  0  1  cos  dx
L 0 2 L  L L 2 L 
1

V0  L  2V0  L  V0 2V0 3V0


 E11    L    
2L  2  2L  2 4 4 4

NET/JRF (JUNE-2012)
Q13. The component along an arbitrary direction n̂ , with direction cosines n x , n y , n z  , of the
1
spin of a spin  particle is measured. The result is
2
(c)  n x  n y  n z 
  
(a) 0 (b)  n z (d) 
2 2 2
Ans: (d)
 0 1  0 i   1 0 
Solution: S x    , S y    , S z   
2 1 0 2 i 0  2 0 1

n  n x ˆi  n y ˆj  n z kˆ and n x2  n y2  n z2  1 , S  S x iˆ  S y ˆj  S z kˆ

   i   
0  0    0 
  2 +n  2 
n  S  nx  2  n 
  y
 i  z  
   0  0 
2  2   2 

 
n
   z2

n x  in y 
nS  2 
 
 n x  in y 

 nz 
2 2 
 
Let  is eigen value of n  S

nz



n x  in y 
2 2
0

n x  in y  
n z  
2 2

H.No. 40‐D, Ground Floor, Jia Sarai, Near IIT, Hauz Khas, New Delhi‐110016 
Phone: 011‐26865455/+91‐9871145498
Website: www.physicsbyfiziks.com  | Email: fiziks.physics@gmail.com  

fiziks
Institute for NET/JRF, GATE, IIT‐JAM, M.Sc. Entrance, JEST, TIFR and GRE in Physics 
 
 n 2 2  2 2
n   n   
n x  n y2   0 .
2
   z     z      n 2x  n 2y   0   z  2  
 2  2  4  4  4


2 2
4
  
n x  n y2  n z2  2  0     .
2
Q14. A particle of mass m is in a cubic box of size a . The potential inside the box
0  x  a,0  y  a,0  z  a  is zero and infinite outside. If the particle is in an

14 2  2
eigenstate of energy E  , its wavefunction is
2ma 2

3x 5y 6z 7x 4y 3z


3/ 2 3/ 2
2 2
(a)     sin sin sin (b)     sin sin sin
a a a a a a a a

4x 8y 2z x 2y 3z


3/ 2 3/ 2
2 2
(c)     sin sin sin (d)     sin sin sin
a a a a a a a a
Ans: (d)

   2 14 2  2
2
Solution: E nx ,n y ,nz  n x2  n y2  n z2 
2ma 2 2ma 2
 n x2  n y2  n z2  14  n x  1, n y  2, n z  3 .

Q15. Let  nlml denote the eigenfunctions of a Hamiltonian for a spherically symmetric

potential V r  . The wavefunction  


1
4
 
 210  5 211  10 211 is an eigenfunction

only of
(a) H , L2 and Lz (b) H and Lz (c) H and L2 (d) L2 and Lz
Ans: (c)
Solution: H  En

L2  l l  1 2 and Lz  m .

Q16. 
The commutator x 2 , p 2 is 
(a) 2ixp (b) 2i ( xp  px ) (c) 2ipx (d) 2i ( xp  px )
Ans: (b)
Solution:  x 2 , p 2   x  x, p 2    x, p 2  x  xp  x, p   x  x, p  p  p  x, p  x   x, p  px

H.No. 40‐D, Ground Floor, Jia Sarai, Near IIT, Hauz Khas, New Delhi‐110016 
Phone: 011‐26865455/+91‐9871145498
Website: www.physicsbyfiziks.com  | Email: fiziks.physics@gmail.com  

fiziks
Institute for NET/JRF, GATE, IIT‐JAM, M.Sc. Entrance, JEST, TIFR and GRE in Physics 
 
 
x , p  xpi   xi  p  pi x  i  px  2ixp  px  .
2 2

Q17. A free particle described by a plane wave and moving in the positive z -direction
undergoes scattering by a potential
V , if r  R
V r    0
0 , if r  R

If V0 is changed to 2V0 , keeping R fixed, then the differential scattering cross-section, in


the Born approximation.
(a) increases to four times the original value
(b) increases to twice the original value
(c) decreases to half the original value
(d) decreases to one fourth the original value
Ans: (a)
V0 , rR
Solution: V  r   
0, rR

Low energy scattering amplitude f  ,    


m 4
V R 3
2 0
2 3
2
d  2mV0 R 3 
And differential scattering is given by 1  f 
2

d  3 
2

d 2  2m  2V0  R 
2 2
3
 2mV0 R 3  d 1
Now V r   2V0 for r  R      4   4
d  3 2
  3
2
 d
Q18. A variational calculation is done with the normalized trial wavefunction

 x   5 / 2 a 2  x 2  for the one-dimensional potential well


15
4a
0 if x  a
V x   
 if x  a
The ground state energy is estimated to be
5 2 3 2 3 2 5 2
(a) (b) (c) (d)
3ma 2 2ma 2 5ma 2 4ma 2
Ans: (d)

Solution:   x  
15
5
a 2
 x2 , V x   0 , x  a and V  x    , x  a
4a 2

H.No. 40‐D, Ground Floor, Jia Sarai, Near IIT, Hauz Khas, New Delhi‐110016 
Phone: 011‐26865455/+91‐9871145498
Website: www.physicsbyfiziks.com  | Email: fiziks.physics@gmail.com  
10 
fiziks
Institute for NET/JRF, GATE, IIT‐JAM, M.Sc. Entrance, JEST, TIFR and GRE in Physics 
 
  
a 2 2
E  Hdx where H 
a
2m x 2

 15 2
a
    2 d 2  15 2   15   2
a

 a  4a5/ 2
E  a   x 2

 2  5/ 2 
a  x 2   dx    a 2  x 2  2dx
  2m dx  4a  
5
 16a 2m  a
a
15 2 2 15  2 4a 3 5 2
 E 
16a 5 2m a
 a 2
 x 2
 dx 
16a 5 m 3

4ma 2

Q19. A particle in one-dimension is in the potential


 , if x  0

V  x   V0 , if 0  x  l
0 , if x  l

If there is at least one bound state, the minimum depth of potential is
 2 2  2 2 2 2 2  2 2
(a) (b) (c) (d)
8ml 2 2ml 2 ml 2 ml 2
Ans: (a)
Solution: For bound state,  V0  E  0
Wave function in region I, I  0 ,  II  A sin kx  B cos kx , III  ce  x
  
2mV0  E  2m   E   V0 o
where k  ,  . l
2 2
Use Boundary condition at x  0 and x  l
(wave function is continuous and differential at x  0 and x  l ), one will get
k cot kl    kl cot kl  l     cot  where  l   , kl   .

2mV0l 2 
2  2 
2
1/ 2 o  3
 2mV0l 2   2 2
For one bound state     V0  . 2
 2
   8ml 2
2
2
Q20. Which of the following is a self-adjoint operator in the spherical polar coordinate
system r ,  ,   ?
i   i  
(a)  (b)  i (c)  (d)  i sin 
sin 2    sin   
Ans: (c)
H.No. 40‐D, Ground Floor, Jia Sarai, Near IIT, Hauz Khas, New Delhi‐110016 
Phone: 011‐26865455/+91‐9871145498
Website: www.physicsbyfiziks.com  | Email: fiziks.physics@gmail.com  
11 
fiziks
Institute for NET/JRF, GATE, IIT‐JAM, M.Sc. Entrance, JEST, TIFR and GRE in Physics 
 
i 
Solution: is Hermitian.
sin  
NET/JRF (DEC-2012)
Q21. Let v, p and E denote the speed, the magnitude of the momentum, and the energy of a
free particle of rest mass m. Then

(a) dE  constant (b) p = mv


dp

(c) v  cp (d) E = mc2


p m c
2 2 2

Ans: (c)
mv m2v 2 p2v2
Solution: p  mv  p   m v  p  2 , m  rest mass energy
2 2 2 2

v2 v2 c
1 2 1 2
c c

 p2  p2 pc
 v 2  m2  2   p 2  v 2  2 2 v
 c  m c  p2 p  m2c 2
2

c2
Q22. The wave function of a state of the Hydrogen atom is given by,
   200  2 211  3 210  2 211
where  nlm is the normalized eigen function of the state with quantum numbers n, l , m in

the usual notation. The expectation value of Lz in the state  is

15 11 3 
(a) (b) (c) (d)
6 6 8 8
Ans: (d)
1 2 3 2
Solution: Firstly normalize ,    200   211   210   211
16 16 16 16

P0  
1 9 10
  .
16 16 16
4 2
Probability of getting 1  i.e. P     and P      .
16 16
 Lz  
 0   1    1  
10 4 2 4 2 2
Now, Lz       
 16 16 16 16 16 16 8

H.No. 40‐D, Ground Floor, Jia Sarai, Near IIT, Hauz Khas, New Delhi‐110016 
Phone: 011‐26865455/+91‐9871145498
Website: www.physicsbyfiziks.com  | Email: fiziks.physics@gmail.com  
12 
fiziks
Institute for NET/JRF, GATE, IIT‐JAM, M.Sc. Entrance, JEST, TIFR and GRE in Physics 
 

The energy eigenvalues of a particle in the potential V  x  


1
Q23. m 2 x 2  ax are
2
 1 a2  1 a2
(a) En   n    (b) En   n   
 2 2m 2  2 2m 2

 1 a2  1
(c) En   n    (d) En   n  
 2 m 2  2
Ans: (a)
p x2 1
Solution: Hamiltonian  H  of Harmonic oscillator, H   m 2 x 2
2m 2
 1
Eigenvalue of this, E n   n  
 2

p x2 1 p2 1  2ax a2  a2
But here, H   m 2 x 2  ax  H  x  m 2  x 2    
2m 2 2m 2  m 2 m 2 4  2m 2
2
p2 1  a  a2
H  x  m 2  x  
2m 2  m 2  2m 2

 1 a2
Energy eigenvalue, E n   n   
 2 2m 2
Q24. If a particle is represented by the normalized wave function


 15 a 2  x 2


, for  a  x  a
  x   4a 5 / 2
0
 , otherwise

the uncertainty p in its momentum is

(a) 2 / 5a (b) 5 / 2a (c) 10 / a (d) 5 / 2a


Ans: (d)

  i 
Solution: p  p2  p
2
and p  x


a

15 a 2  x 2   i 
 p   4a 5/ 2
4a
15  2
5 / 2 xa  x 2 dx 
a

H.No. 40‐D, Ground Floor, Jia Sarai, Near IIT, Hauz Khas, New Delhi‐110016 
Phone: 011‐26865455/+91‐9871145498
Website: www.physicsbyfiziks.com  | Email: fiziks.physics@gmail.com  
13 
fiziks
Institute for NET/JRF, GATE, IIT‐JAM, M.Sc. Entrance, JEST, TIFR and GRE in Physics 
 
a
 i  a 2  x 2  2 x dx  ih 2 155
   a 
a
15
 2
x  x3 dx  0 , ( odd function)
16  a
5
 a 16a a

2 2
aa  x  x 2 a  x dx
a
15
p 2
   2 2 2 2

16a 5
a
 2 x3 
 
a
   215
  2   a 2
 x 2
dx   2

15
 2  a  x  
16a 5 a 16a 5  3  a

15  3 2a 3  2 15 2 3 1  15 2 2 5 2
   2  2a       5  2a 1     
2

16a 5  3  16 a  3  4a 2 3 2a 2

5 2 5
Now, p  p2  p  0 
2

2a 2 2a
Q25. Given the usual canonical commutation relations, the commutator A, B  of
A  i xp y  yp x  and B   yp z  zp y  is

(a)   xp z  p x z  (b)    xp z  p x z  (c)   xp z  p x z  (d)    xp z  p x z 


Ans: (c)

Solution: A, B   ixp y  iyp x ,  yp z  zp y  
A, B  ixp y , yp z   i yp x , yp z   ixp y , zp y   iyp x , zp y 
A, B  ixp y , yp z   0  0  iyp x , zp y   ixp y , yp z   iyp x , zp y 
A, B  ix p y , yp z   ix, yp z  p y  iy p x , zp y   iy, zp y p x
A, B  ix p y , yp z   0  0  iy, zp y p x   
 ix p y , yp z  i y, zp y p x 
 A, B   ix   i  pz  izi  px    xpz  zpx 
A, B  xp z  p x z 
Q26. Consider a system of three spins S1, S2 and S3 each of which can take values +1 and -1.
The energy of the system is given by E   J S1 S 2  S 2 S 3  S 3 S1  where J is a positive
constant. The minimum energy and the corresponding number of spin configuration are,
respectively,
(a) J and 1 (b) 3 J and 1 (c) 3 J and 2 (d) 6 J and 2
Ans: (c)

H.No. 40‐D, Ground Floor, Jia Sarai, Near IIT, Hauz Khas, New Delhi‐110016 
Phone: 011‐26865455/+91‐9871145498
Website: www.physicsbyfiziks.com  | Email: fiziks.physics@gmail.com  
14 
fiziks
Institute for NET/JRF, GATE, IIT‐JAM, M.Sc. Entrance, JEST, TIFR and GRE in Physics 
 
Solution: If we take S1  S2  S3  1 i.e.   
S1 S2 S3

Then energy, E   J 1  1  1  1  1  1  3J

Again S1  S2  S3  1 , then   

Energy  E   3J

So, minimum energy is  3J  and there are two spin configuration.

If we take   
S1 S2 S3

Then we get Maximum energy E  J .


1
Q27. The energies in the ground state and first excited state of a particle of mass m  in a
2
potential V  x  are 4 and 1 , respectively, (in units in which   1 ). If the

corresponding wavefunctions are related by  1  x    0  x sinh x, then the ground state


eigenfunction is
(a)  0  x   sec hx (b)  0  x   sec hx

(c)  0  x   sec h 2 x (d)  0  x   sec h 3 x


Ans: (c)
Solution: Given that ground state energy E0  4 , first excited state energy E1  1 and  0 ,  1
are corresponding wave functions.
1
Solving Schrödinger equation (use m  and   1 )
2
  2  2 0  2 0
 V 0  E0 0   V 0  4 0 ……(1)
2m x 2 x 2
 2  2 1  2 1
 V 1  E1 1   V 1  1 1 ……..(2)
2m x 2 x 2
Put  1   0 sinh x in equation (2) one will get

  2  
  20 .sinh x  2 0 cosh x  0 sinh x   V 0 sinh x   0 sinh x
 x x 
  2 0  
  2  2 0 coth x  0   V 0   0
 x x 
H.No. 40‐D, Ground Floor, Jia Sarai, Near IIT, Hauz Khas, New Delhi‐110016 
Phone: 011‐26865455/+91‐9871145498
Website: www.physicsbyfiziks.com  | Email: fiziks.physics@gmail.com  
15 
fiziks
Institute for NET/JRF, GATE, IIT‐JAM, M.Sc. Entrance, JEST, TIFR and GRE in Physics 
 
  0
2
  0  2 0
   V  0  2 coth x     using relation   V 0  4 0
  
2 0 0 2
 x  x x

 0 d 0
4 0  2 coth x  0   0   2 tanh xdx   0  sec h 2 x .
x 0

NET/JRF (JUNE-2013)
Q28. In a basis in which the z - component S z of the spin is diagonal, an electron is in a spin

 1  i  / 6 
state    . The probabilities that a measurement of S z will yield the values
 
 2 / 3 
 / 2 and   / 2 are, respectively,
(a) 1/ 2 and 1/ 2 (b) 2 / 3 and 1/ 3 (c) 1/ 4 and 3 / 4 (d) 1/ 3 and 2 / 3
Ans: (d)
1 0  
Solution: Eigen state of S z is 1    and 2    corresponds to Eigen value and 
0 1 2 2
respectively.
2 2
  1  1 i
2
2 1   2  2
P      , P   
 2   6 6 3  2   3

Q29. Consider the normalized state  of a particle in a one-dimensional harmonic oscillator:

  b1 0  b2 1

where 0 and 1 denote the ground and first excited states respectively, and b1 and b2

are real constants. The expectation value of the displacement x in the state  will be a
minimum when
1 1
(a) b2  0, b1  1 (b) b2  b1 (c) b2  b1 (d) b2  b1
2 2
Ans: (d)
Solution: x  b12 0 x 0  b22 1 x 1  2b1b2 0 x 1

Since 0 x 0  0 and 1 x 1  0  x  2b1b2 0 x 1 .

Min of x means min 2b1b2 . We know that b12  b22  1.

H.No. 40‐D, Ground Floor, Jia Sarai, Near IIT, Hauz Khas, New Delhi‐110016 
Phone: 011‐26865455/+91‐9871145498
Website: www.physicsbyfiziks.com  | Email: fiziks.physics@gmail.com  
16 
fiziks
Institute for NET/JRF, GATE, IIT‐JAM, M.Sc. Entrance, JEST, TIFR and GRE in Physics 
 
     
x min   b1  b2   b12  b22  0 x 1  b1  b2   1 0 x 1  1  b1  b2  0 x 1 will

2


2 2

be minimum and minimum value of 1   b1  b2   , there must be maximum of  b1  b2  ,


2 2

 
so  b1  b2
Q30. The un-normalized wavefunction of a particle in a spherically symmetric potential is
given by

  r   zf  r 

where f r  is a function of the radial variable r . The eigenvalue of the operator



L2 (namely the square of the orbital angular momentum) is
(a)  2 / 4 (b)  2 / 2 (c)  2 (d) 2 2
Ans: (d)
Solution:  r   zf r   r cos f r 

  r  Y10  ,    , L2  r   L2Y10  ,   , where l  1

L2  l l  1 2  11  1 2  2 2


Q31. If  nlm denotes the eigenfunciton of the Hamiltonian with a potential V  V r  then the

expectation value of the operator L2x  L2y in the state

1
  3 211  210  15 211 
5
is
(a) 39 2 / 25 (b) 13 2 / 25 (c) 2 2 (d) 26 2 / 25
Ans: (d)
Solution: L2x  L2y  L2  L2z  L2x  L2y  L2  L2z  L2  L2z

 9 1 15 
L2  L2z = 2 2    1 2   0 2  1 2 
 25 25 25 
24 2 50  24 2 26 2
L2  L2z  2 2      
25 25 25

H.No. 40‐D, Ground Floor, Jia Sarai, Near IIT, Hauz Khas, New Delhi‐110016 
Phone: 011‐26865455/+91‐9871145498
Website: www.physicsbyfiziks.com  | Email: fiziks.physics@gmail.com  
17 
fiziks
Institute for NET/JRF, GATE, IIT‐JAM, M.Sc. Entrance, JEST, TIFR and GRE in Physics 
 
Q32. Consider a two-dimensional infinite square well
0, 0  x  a, 0 ya
V  x, y   
 , otherwise

2  n x x   n y  y 
Its normalized Eigenfunctions are nx ,ny  x, y   sin   sin  ,
a  a   a 
where nx , n y  1, 2, 3, ..

 a a
V 0 x , 0 y
If a perturbation H '   0 2 2 is applied, then the correction to the
 0 otherwise

energy of the first excited state to order V0 is

V0 V0  64 
(a)
4
(b)
4 1  9 2 

V0  16  V0  32 
(c)
4 1  9 2  (d)
4 1  9 2 
Ans: (b)
Solution: For first excited state, which is doubly degenerate
2 x 2 y 2  2 x    y 
1  sin sin , 2  sin   sin  
a a a a  a   a 
2 a / 2 2   x  2 a / 2 2  2 y  1 1 V
H11  1 H 1  V0 
a 0
sin   dx  sin 
 a  a 0  a 
 dy  V0    0
2 2 4
2 a/2  x 2 x 2 a / 2 2 y y
H12  1 H 2 
a 0
 V0
sin
a
sin
a
dx  sin
a 0 a
sin
a
dy

 4  4  16 16 V
H 12  V0     V0 , H 21  2 H  1  V0 2 and H 22  2 H  2  0 .
 3  3  9 9
2
4

 V0 16V0 
   2 2

Thus  4 9 2   0   V0      16V0   0
 16V0 V0
    4   9 
2

 9 2 4 
V  16V0 V  64 
  0       0 1  2 
 4  9 2
4  9 

H.No. 40‐D, Ground Floor, Jia Sarai, Near IIT, Hauz Khas, New Delhi‐110016 
Phone: 011‐26865455/+91‐9871145498
Website: www.physicsbyfiziks.com  | Email: fiziks.physics@gmail.com  
18 
fiziks
Institute for NET/JRF, GATE, IIT‐JAM, M.Sc. Entrance, JEST, TIFR and GRE in Physics 
 
Q33. The bound on the ground state energy of the Hamiltonian with an attractive delta-
function potential, namely
2 d 2
H   a  x 
2m dx 2
using the variational principle with the trial wavefunction   x   A exp  bx 2 is  
 

 Note :  e t dt  a  1
t a

 0 

(a)  ma 2 / 4  2 (b)  ma 2 / 2  2 (c)  ma 2 /   2 (d)  ma 2 / 5  2


Ans: (c)
 2b 2b  2b 2b
Solution: For given wavefunction T  and V  a  E  a
2m  2m 
d E d E 2 2 1  12 2m 2 a 2
For variation of parameter 0  a  b 0 b .
db db 2m  2  4
ma 2
 E  .
min
 2
Q34. If the operators A and B satisfy the commutation relation  A, B  I , where I is the
identity operator, then
 
(a) e A , B  e A   
(b) e A , B  e B , A 
(c) e , B  e
A B
,A  (d) e , B  I
A

Ans: (a)
 A A2 
Solution: A, B  I and e A  1    .......
 1 2 
 A2 , B   A3 , B 
  A A2
e , B  1  
A 
 ......., B  = 1, B    A, B    ....
 1 2  2 3

A  A, B    A, B  A A  A2 , B    A2 , B  A
e , B  0  I 
A

2!

3!
 ....

e A , B   1  A 
A2
2!
 
 ....  e A where A, B  I , A 2 , B  2 A and A3 , B  3 A 2 .  

H.No. 40‐D, Ground Floor, Jia Sarai, Near IIT, Hauz Khas, New Delhi‐110016 
Phone: 011‐26865455/+91‐9871145498
Website: www.physicsbyfiziks.com  | Email: fiziks.physics@gmail.com  
19 
fiziks
Institute for NET/JRF, GATE, IIT‐JAM, M.Sc. Entrance, JEST, TIFR and GRE in Physics 
 
Q35. Two identical bosons of mass m are placed in a one-dimensional potential

V x  
1
m 2 x 2 . The bosons interact via a weak potential,
2

V12  V0 exp  m x1  x 2  / 4
2

where x1 and x 2 denote coordinates of the particles. Given that the ground state
1
m x 2
 m  4 
wavefunction of the harmonic oscillator is  0  x     e
2
. The ground state
  
energy of the two-boson system, to the first order in V0 , is

V0 
(a)   2V0 (b)  

1

   2  
(c)   V0 1   (d)   V0 1  
 2   
Ans: (c)
Solution: There are two bosons trapped in harmonic oscillator.

So, energy for ground state without perturbation is, 2    .
2
If perturbation is introduced, we have to calculate V1,2 

where V1,2  V0 exp  m  x1  x2  / 4  .


2

 
 1
m x12 m x22 
  m  2  
 is very tedious task.
But calculating V1,2  on state  0  x    2
e 2
    
e

 
So lets use a trick i.e perturbation is nothing but approximation used in Taylor series. So

just expand V1,2  V0 exp   m  x1  x2  / 4  and take average value of first term
2

 
 m  x1  x2 2 
V1,2  V0 exp m  x1  x2  / 4  V0  1 
   ... 
2

   4 
 

 V0  1 

 m x12  x22  2 x1.x2 
 ... 

 4 
 

H.No. 40‐D, Ground Floor, Jia Sarai, Near IIT, Hauz Khas, New Delhi‐110016 
Phone: 011‐26865455/+91‐9871145498
Website: www.physicsbyfiziks.com  | Email: fiziks.physics@gmail.com  
20 
fiziks
Institute for NET/JRF, GATE, IIT‐JAM, M.Sc. Entrance, JEST, TIFR and GRE in Physics 
 
    
 
 m  x12    x22   2 x1 . x2    m    0 
V1,2   V0 1   ...   Vo  1   2m 2m   ...
 4   4 
   
 
1 1
   2   2
 V12   Vo (1  )  V0 1   , so E    V0 1   .
4  2   2 

NET/JRF (DEC-2013)
1 1 1  i 
Q36. A spin - particle is in the state     in the eigenbasis of S 2 and S z . If we
2 11  3 
h h
measure S z , the probabilities of getting  and  , respectively are
2 2
1 1 2 9 1 3
(a) and (b) and (c) 0 and 1 (d) and
2 2 11 11 11 11
Ans: (b)
2
  1 1  i 1 2
Solution: P    10     2     1
 2 11  3  11 11
2
  1 1  i 9
P  
 2
 01   
11  3  11

   
i.e. probability of S z getting   and   
 2  2

Q37. The motion of a particle of mass m in one dimension is described by the


p2 1
Hamiltonian H   m 2 x 2  x . What is the difference between the (quantized)
2m 2
energies of the first two levels? (In the following, x is the expectation value of x in the
ground state)
2
(a)    x (b)    x (c)   (d) 
2m 2
Ans: (d)
p2 1 1
Solution: H   m 2 x 2   x  V  x   m 2 x 2   x
2m 2 2

H.No. 40‐D, Ground Floor, Jia Sarai, Near IIT, Hauz Khas, New Delhi‐110016 
Phone: 011‐26865455/+91‐9871145498
Website: www.physicsbyfiziks.com  | Email: fiziks.physics@gmail.com  
21 
fiziks
Institute for NET/JRF, GATE, IIT‐JAM, M.Sc. Entrance, JEST, TIFR and GRE in Physics 
 
1  2  1 2 2  2 2 
V  x   m 2  x 2   x  m  x  2  x    
2  m 2  2  m 2 m 2 4 m 2 4 
  2
2

V  x   m 2  x 
1
 
2  m 2  2m 2
 1 2 3 1
 En   n       E1  E0      
 2 2m 2
2 2
Q38. Let  nlm denote the eigenfunctions of a Hamiltonian for a spherically symmetric

potential V r  . The expectation value of L z in the state

 
1
6

 200  5 210  10 211  20 211 is 
5 5 5
(a)   (b)  (c)  (d) 
18 6 18
Ans: (d)
1 5 10 20 10 5
Solution: Lz   L z  =  0   0   (1)  (1)      1
36 36 36 36 36 18
Q39.  
If   x   A exp  x 4 is the eigenfunction of a one dimensional Hamiltonian with eigen
value E  0 , the potential V  x  (in units where   2m  1 ) is

(a) 12x 2 (b) 16x 6 (c) 16 x 6  12 x 2 (d) 16 x 6  12 x 2


Ans: (d)
Solution: Schrodinger equation
  2  V  0 (where   2m  1 and E  0 )


2
x 2
 4
 4 
Ae  x  VAe  x  0   e  x  4 x 3   Ve  x  0
x 
4


4

 4

4 3x 2 e  x  x 3  4 x 3 e  x
4
 Ve  x4
 0  12 x 2 e  x  16 x 6 e  x  Ve  x  0
4 4 4

 V  16 x 6  12 x 2
Q40. A particle is in the ground state of an infinite square well potential is given by,
0 for  a  x  a
V x   
 otherwise
a a
The probability to find the particle in the interval between  and is
2 2
1 1 1 1 1 1
(a)  (b) (c)  (d)
2 2  2  
H.No. 40‐D, Ground Floor, Jia Sarai, Near IIT, Hauz Khas, New Delhi‐110016 
Phone: 011‐26865455/+91‐9871145498
Website: www.physicsbyfiziks.com  | Email: fiziks.physics@gmail.com  
22 
fiziks
Institute for NET/JRF, GATE, IIT‐JAM, M.Sc. Entrance, JEST, TIFR and GRE in Physics 
 
Ans: (b)
a a
Solution: The probability to find the particle in the interval between  and is
2 2

x x x 1 1  2x  
a/2 a/2 a/2
2 2 1
   cos  cos dx   cos 2 dx     1  cos dx 
a / 2
2a 2a 2a 2a a / 2
a 2 a a 2 a / 2  2 a  

x 
a/2
1  1 a a a  1  2a   1 1 
  1  1 
a
  x  sin     a    
2a   a   a / 2 2a  2 2   2a    2  
Q41. The expectation value of the x - component of the orbital angular momentum L x in the

state  
1
5

3 2,1, 1  5 2,1,0  11 2,1, 1 
(where  nlm are the eigenfunctions in usual notation), is

(a) 
 10
25
 11  3 (b) 0 (c)
 10
25
 11  3 (d)  2

Ans: (a)

Solution: L l , m  l  l  1  m  m  1 l , m  1 and L l , m  l  l  1  m  m  1 l , m  1

L  L L  L
Lx   Lx  
2 2
1
L  3 2 210  5 2 211 
5
1 1 1
 L   .3 10  110  10(3  11)
25 25 25
1
L   2 5 211  2 11 210 
5
1 1
 L   .3 10  10 11
25 25
L  L 1
Lx  = 10(3  11)
2 25

 Lx  
1
25
.3 10 
1
25
10 11 = 
 10
25
 11  3

H.No. 40‐D, Ground Floor, Jia Sarai, Near IIT, Hauz Khas, New Delhi‐110016 
Phone: 011‐26865455/+91‐9871145498
Website: www.physicsbyfiziks.com  | Email: fiziks.physics@gmail.com  
23 
fiziks
Institute for NET/JRF, GATE, IIT‐JAM, M.Sc. Entrance, JEST, TIFR and GRE in Physics 
 
Q42. A particle is prepared in a simultaneous eigenstate of L2 and Lz . If l  l  1  2 and m

are respectively the eigenvalues of L2 and Lz , then the expectation value L2x of the

particle in this state satisfies


(a) L2x  0 (b) 0  L2x   2  2
    1  2  2     1  2
(c) 0  L 2
x  (d)  Lx 
2

2 2 2
Ans: (d)
Solution: L2x 
1
2

l  l  1  2  m 2  2 
For max value m  0 and for min m  l
l 2 l l  1 2
 L2x 
2 2
A, B, C are Non zero Hermitian operator.
A, B  C  AB  BA  AB  Ab  0  C
but C0
if AB  BA i.e. A, B   C false (2)

NET/JRF (JUNE-2014)
Q43. Consider a system of two non-interacting identical fermions, each of mass m in an
infinite square well potential of width a . (Take the potential inside the well to be zero
and ignore spin). The composite wavefunction for the system with total energy
5 2  2
E is
2ma 2
2   x1   2x 2   2x1   x 2 
(a) sin  sin   sin  sin 
a  a   a   a   a 

2   x1   2x 2   2x1   x 2 


(b) sin  sin   sin  sin 
a  a   a   a   a 

2   x1   3x 2   3x1   x 2 


(c) sin  sin   sin  sin 
a   a   2a   2a   a  

2   x1   x 2   x   x 
(d) sin  cos   sin 2  cos 2 
a  a   a   a   a 

H.No. 40‐D, Ground Floor, Jia Sarai, Near IIT, Hauz Khas, New Delhi‐110016 
Phone: 011‐26865455/+91‐9871145498
Website: www.physicsbyfiziks.com  | Email: fiziks.physics@gmail.com  
24 
fiziks
Institute for NET/JRF, GATE, IIT‐JAM, M.Sc. Entrance, JEST, TIFR and GRE in Physics 
 
Ans: (a)
Solution: Fermions have antisymmetric wave function
2    x1   2 x2   2 x1    x2  
  x1 x2    sin   sin    sin    sin  
a  a   a   a   a 

5 2  2
 En   nx1  1, nx2  2
2ma 2

A particle of mass m in the potential V  x, y   m 2 4 x 2  y 2  , is in an eigenstate of


1
Q44.
2
5
energy E   . The corresponding un-normalized eigen function is
2
 m  m
(a) y exp  2 x 2  y 2 

(b) x exp  2 x 2  y 2 

 2   2 
 m 2  m 2
(c) y exp  x  y 2 

(d) xy exp  x  y 2 

 2   2 
Ans: (a)

Solution: V  x, y  
1
2
  5
m 2 4 x 2  y 2 , E   
2
1 1
 V  x, y   m  2  x 2  m 2 y 2
2

2 2
 1  1  1  1
Now, E n   n x   x   n y   y   n x  2   n y  
 2  2  2  2

 3
 En   2 n x  n y    
 2
5
 En   when n x  0 and n y  1 .
2
Q45. A particle of mass m in three dimensions is in the potential
0, ra
V r   
 , ra
Its ground state energy is
 2 2  2 2 3 2  2 9 2  2
(a) (b) (c) (d)
2ma 2 ma 2 2ma 2 2ma 2

Ans: (a)
H.No. 40‐D, Ground Floor, Jia Sarai, Near IIT, Hauz Khas, New Delhi‐110016 
Phone: 011‐26865455/+91‐9871145498
Website: www.physicsbyfiziks.com  | Email: fiziks.physics@gmail.com  
25 
fiziks
Institute for NET/JRF, GATE, IIT‐JAM, M.Sc. Entrance, JEST, TIFR and GRE in Physics 
 
   d u  r  l  l  1
2 2

Solution:    2
 2
 V  r  u  r   Eu  r 
 2 m  dr 2 mr

d 2u  r  2mE
  K 2u  r  K  , l  0, V  r   0
dr 2
2
u  r   A sin Kr  B cos Kr

Using boundary condition, B  0,

 22
u  r   A sin Kr , r  a, u  r   0  sin Ka  0  Ka  n  E  n 1
2ma 2
  1
Q46. Given that pˆ r  i   , the uncertainty p r in the ground state
 r r 
1
 0 r   e  r / a0 of the hydrogen atom is
a 3
0

 2  2
(a) (b) (c) (d)
a0 a0 2a 0 a0

Ans: (a)
  1 1
Solution: pˆ r  i    ,  0  r   e  r / a0 , Pr  Pr2  Pr
2

 r r   a0
3


1  r / a
  1  e 0 
Now Pr   e  r / a 0    i     4 r dr
2

0 a 03   r r  a 03 

4 i    r / a0   r / a0  1  1  r / a0  2 

 r dr 
a 03  0
  e     e
e 
  a0  r  

4 i   1 
 

 
 2 r / a0 2
 3 
 e r dr  re  2 r / a0 dr 
a 0  a 0 0 0 

4 i   1  2!   1!  
    
 a03  a0   2 / a0 3    2 / a0 2  
 

4 i   a02 a02 
   0
 a03  4 4 

H.No. 40‐D, Ground Floor, Jia Sarai, Near IIT, Hauz Khas, New Delhi‐110016 
Phone: 011‐26865455/+91‐9871145498
Website: www.physicsbyfiziks.com  | Email: fiziks.physics@gmail.com  
26 
fiziks
Institute for NET/JRF, GATE, IIT‐JAM, M.Sc. Entrance, JEST, TIFR and GRE in Physics 
 
1

  2
2    r / a0 
Pr2  3  e  r / a0   2  2  e 4 r dr
2

a 0 0   r r r  

4  2   r / a0   r / a0  1  2  1   r / a0   2

 a03  0
  e e  a 2   r    a  e   r dr
 0 0  

4 2  1 2 2 r / a0

2

 4 2 1 2 ! 2 1 ! 
a03 0 a02 a0 0
2 r / a0
  r e dr  re dr     2  
 a0  2 / a0  a0  2 / a0  
3 2
 a03

4 2  2 ! a03 2 a02  4 2  a0 a0  4 2  a0   2
        4      
a03  a02 8 a0 4 a03  2  a03  4  a02

2 
 P  Pr2  Pr  0 
2
2
a0 a0

Q47. The ground state eigenfunction for the potential V  x     x  where   x  is the delta

function, is given by   x   Ae
 x
, where A and   0 are constants. If a perturbation

H   bx 2 is applied, the first order correction to the energy of the ground state will be
b b 2b b
(a) (b) (c) (d)
2 2
 2
 2
2 2
Ans: (d)

Solution: V  x     x  ,  x   Ae
 x

   1    x    e  x

E  1 H  1    e  x bx 2  e  x dx
1
1


 
0 
  

 e bx dx  b   e x dx  b   x e dx   x e dx   b 2   x e dx
 2 x 2 2 x 2 2 2 x 2 2 x 2  2x

    0   0 

 2!  2! b

 2 x
 e bx 2
dx  2b  3
 2  b 3 
 2   8 2 2

H.No. 40‐D, Ground Floor, Jia Sarai, Near IIT, Hauz Khas, New Delhi‐110016 
Phone: 011‐26865455/+91‐9871145498
Website: www.physicsbyfiziks.com  | Email: fiziks.physics@gmail.com  
27 
fiziks
Institute for NET/JRF, GATE, IIT‐JAM, M.Sc. Entrance, JEST, TIFR and GRE in Physics 
 
Q48. An electron is in the ground state of a hydrogen atom. The probability that it is within the
Bohr radius is approximately equal to
(a) 0.60 (b) 0.90 (c) 0.16 (d) 0.32
Ans: (d)
2
4
a0 a0
1
 4 r dr  3 r e
 r / a0 2 2 2 r / a0
Solution: Probability: e dr
0 a 3
0
 a0 0

4   2 2 r / a0  a0    2 r / a0  a0  a0  a0   
a0 a0 a0
 2 r / a0  a0  a0  
 
a03  
r e


 2 
0 
  2 r e 


 
 2  2  
0 
  2 e






 2  2  2   
  0 

4  2  2aa0  a0   a02  2 a0 / a0 a03 2 a0 / a0 0  a0


3

 3  a0 e 0  2   2 a0   e  e  2 e  
a0     4  4  8  

4  a03 1 a03 1 a03 a03   5 1  1 


        4   2     5  2  1
a03  4e 4  
2 2 2
 2 e 2 e 4e 4 e

  5  0.137  1   0.685  1  0.32

Q49. A particle in the infinite square well potential


0 , 0 xa
V  x  
 , otherwise
is prepared in a state with the wavefunction
 3x 
 A sin 
  x   , 0  x  a
 a 
0
 , otherwise
The expectation value of the energy of the particle is
5 2 2 9 2  2 9 2  2  2 2
(a) (b) (c) (d)
2ma 2 2ma 2 10ma 2 2ma 2

Ans: (c)
 3x 
0, 0 xa  A sin   , 0 xa 
Solution: V  x      x    a  
 , otherwise 0 , otherwise 

x  x 3 x
  x   A sin 3 
 a 
3
  A sin
4 a
1
 A sin
4 a
sin 3 A  3sin A  4sin A 3

H.No. 40‐D, Ground Floor, Jia Sarai, Near IIT, Hauz Khas, New Delhi‐110016 
Phone: 011‐26865455/+91‐9871145498
Website: www.physicsbyfiziks.com  | Email: fiziks.physics@gmail.com  
28 
fiziks
Institute for NET/JRF, GATE, IIT‐JAM, M.Sc. Entrance, JEST, TIFR and GRE in Physics 
 
A a 2 x a 2 3 x  A a a 
   3sin  sin     x   3 1  x   3  x  
4 2 a a 2 a a  4 2 2 

a 2 a 2 10a 2 32
  1 9 A  A 1  A 1  A 
32 32 32 10a

1 a 32 a 32  3 1
  x    3. 1  x   3  x    1  x   3  x 
4 2 10a 2 10a  10 10

 22 9 2  2
Now, E1  2
, E3   E  an P  an 
2ma 2ma 2
2 2
1  9 2  1
Probability P  E1    , P  E3   
 10  10

9  2  2 1 9 2  2 9 2  2
E      E 
10 2ma 2 10 2ma 2 10ma 2

NET/JRF (DEC-2014)
Q50. Suppose Hamiltonian of a conservative system in classical mechanics is H  xp , where
 is a constant and x and p are the position and momentum respectively. The
corresponding Hamiltonian in quantum mechanics, in the coordinate representation, is
  1   1
(a)  i  x   (b)  i  x  
 x 2   x 2 
 i 
(c)  ix (d)  x
x 2 x
Ans: (b)
Solution: Classically H   xp , quantum mechanically H must be Hermitian,
 
So, H   xp  px  and H   xp  px 
2 2
  i  x       
 H   x  i      i    x x  
2 x x  2  x x 
i      1
 H   2x    i  x  
2  x   x 2 

H.No. 40‐D, Ground Floor, Jia Sarai, Near IIT, Hauz Khas, New Delhi‐110016 
Phone: 011‐26865455/+91‐9871145498
Website: www.physicsbyfiziks.com  | Email: fiziks.physics@gmail.com  
29 
fiziks
Institute for NET/JRF, GATE, IIT‐JAM, M.Sc. Entrance, JEST, TIFR and GRE in Physics 
 
Q51. Let  1 and  2 denote the normalized eigenstates of a particle with energy eigenvalues
E1 and E2 respectively, with E 2  E1 . At time t  0 the particle is prepared in a state

 t  0 
 1   2 
1
2
The shortest time T at which  t  T  will be orthogonal to  t  0  is
2   
(a) (b) (c) (d)
E 2  E1  E 2  E1  2E 2  E1  4E 2  E1 
Ans: (b)
1  iE1T 1  iE2T
 1   2 
1
Solution:   t  0  and   t  T   e 1  e 2
2 2 2
1  iE1 T 1  iE2 T iE
 1T
iE
 2T i  E2  E1 
T

  0  T  dx  0  e  e  0  e   e   e   1
*

2 2
T   
Equate real part  cos   E2  E1   1  T  cos 1  1 
   E2  E1   E2  E1 
Q52. Consider the normalized wavefunction
  a1 11  a 2 10  a3 11
where  lm is a simultaneous normalized eigenfunction of the angular momentum

operators L2 and Lz , with eigenvalues l l  1 2 and m respectively. If  is an


eigenfunction of the operator L x with eigenvalue  , then

1 1 1 1
(a) a1  a3  , a2  (b) a1  a3  , a2 
2 2 2 2
1 1 1
(c) a1  a3  , a 2   (d) a1  a 2  a3 
2 2 3
Ans: (b)
L  L
Solution: Lx        
2
For L , L  a1 11  a2 10  a3 11   a1 0 12  a2 2 11  a3 2 10

 a2 2 11  a3 2 10

For L , L  a1 11  a2 10  a3 11   a1 2 10  a2 2 11

L  L
Given   
2

H.No. 40‐D, Ground Floor, Jia Sarai, Near IIT, Hauz Khas, New Delhi‐110016 
Phone: 011‐26865455/+91‐9871145498
Website: www.physicsbyfiziks.com  | Email: fiziks.physics@gmail.com  
30 
fiziks
Institute for NET/JRF, GATE, IIT‐JAM, M.Sc. Entrance, JEST, TIFR and GRE in Physics 
 
L  L 1
     a2 2 11   a1  a3  2 10  a2 2 11 
2 2
L  L
     a1 11  a2 10  a3 11  (Given)
2
a2
Thus  a1  a2  2a1
2
a1  a3 a1  a3 a22
 a2   2a1  a1  a3  a12  a22  1
2 2 2
1 1
a1  a3  , a2 
2 2
Q53. Let x and p denote, respectively, the coordinate and momentum operators satisfying the
canonical commutation relation x, p   i in natural units   1 . Then the commutator

x, pe  is
p

(a) i 1  p e  p (b) i 1  p 2 e  p (c) i 1  e  p  (d) ipe  p


Ans: (a)
Solution:   x, p   i
 p 2 p3 
 x, pe  p    x, p  e  p  p  x, e  p   ie  p  p  x,1  p   ....
 2 3 
  p2    2ip 3ip 2 
 ie p  p  x,1   x, p    x,  ....  ie  p  p 0  i   ......
  2   2 3 
 p 3

  x, pe p   ie  p  i  p  p 2  .....  ie  p  ipe  p  i 1  p  e  p
 2 
  
Q54. Let    1 ,  2 ,  3  , where  1 ,  2 ,  3 are the Pauli matrices. If a and b are two
   

arbitrary constant vectors in three dimensions, the commutator a   , b   is equal to (in 
the following I is the identity matrix)
  
 
 
(a) a  b  1   2   3  
(b) 2i a  b  
 
 
(c) a  b I
 
(d) a b I

Ans: (b)
 
Solution: a  a1iˆ  a2 ˆj  a3 kˆ , b  b1iˆ  b2 ˆj  b3 kˆ ,    x iˆ   y ˆj   z kˆ

H.No. 40‐D, Ground Floor, Jia Sarai, Near IIT, Hauz Khas, New Delhi‐110016 
Phone: 011‐26865455/+91‐9871145498
Website: www.physicsbyfiziks.com  | Email: fiziks.physics@gmail.com  
31 
fiziks
Institute for NET/JRF, GATE, IIT‐JAM, M.Sc. Entrance, JEST, TIFR and GRE in Physics 
 
   
 a   , b      a1 x  a2 y  a3 z , b1 x  b2 y  b3 z 
   
   
 a   , b     a1b1  x ,  x   a1b2  x ,  y   a1b3  x ,  z   a2b1  y ,  x   a2b2  y ,  y 
       

 a2b3  y ,  z   a3b1  z ,  x   a3b2  z ,  y   a3b3  z ,  z 

 a1b1  0  a1b2  2i z  2ia1b3 y  a2b1  2i z  0  a2b3  2i x  a3b1  2i y  a3b2  2i x  0
      

  a   , b      2i a  b   
Q55. The ground state energy of the attractive delta function potential
V  x   b  x  ,
where b  0 , is calculated with the variational trial function
 x 
 A cos , for  a  x  a, 
 x    2a  is
 0, otherwise, 

mb 2 2mb 2 mb 2 mb 2
(a)  (b)  (c)  (d) 
 2 2  2 2 2 2  2 4 2  2
Ans: (b)
 x
Solution: V  x   b  x  ; b  0 and   x    A cos ; a  x  a
 2a

2 x
Normalized   cos
2a 2a
a  2   2  22
T   *  2  dx 
a
 2m  x 8ma 2

2 b
V    *  b  x  dx   b   
a

a 2a a
 2 2 b  E 2 2  2 b  2  2  2 2
E       0   b  0  a 
8ma 2 a a 8ma 3 a2 4ma 4mb

b    4mb  b  4mb 
2
 22
2 2
2mb 2
Put the value of a in equation: E      
8ma 2 a 8m  2  2
2
 2 2  
 2 2 

H.No. 40‐D, Ground Floor, Jia Sarai, Near IIT, Hauz Khas, New Delhi‐110016 
Phone: 011‐26865455/+91‐9871145498
Website: www.physicsbyfiziks.com  | Email: fiziks.physics@gmail.com  
32 
fiziks
Institute for NET/JRF, GATE, IIT‐JAM, M.Sc. Entrance, JEST, TIFR and GRE in Physics 
 
Q56. Let   c0 0  c1 1 (where c0 and c1 are constants with c02  c12  1 ) be a linear

combination of the wavefunctions of the ground and first excited states of the one-
dimensional harmonic oscillator. For what value of c0 is the expectation value x a

maximum?
 1  1
(a) x  , c0  (b) x  , c0 
m 2 2m 2

 1  1
(c) x  , c0  (d) x  , c0 
2m 2 m 2
Ans: (c)
Solution:   c0 0  c1 1

X   X

  
 X  2c0 c1 0 X 1   c02  c12   c0  c1   0 X 1  1   c0  c1   0 X 1
2

 
2


1
For max X  c0  c1  c02  c12  1  c0 
2
1 1
 X 2 0 X 1  0 X 1
2 2

 
2m
 0 aa 1
X 
2m
Q57. Consider a particle of mass m in the potential V  x   a x , a  0 . The energy eigen-

values E n n  0, 1, 2, .... , in the WKB approximation, are


1/ 3 2/3
 3a  1   3a  1 
(a)   n   (b)   n  
 4 2m  2   4 2m  2 
4/3
3a  1  3a  1 
(c) n   (d)   n  
4 2m  2  4 2m  2 
Ans: (b)
Solution: V  x   a x , a0

H.No. 40‐D, Ground Floor, Jia Sarai, Near IIT, Hauz Khas, New Delhi‐110016 
Phone: 011‐26865455/+91‐9871145498
Website: www.physicsbyfiziks.com  | Email: fiziks.physics@gmail.com  
33 
fiziks
Institute for NET/JRF, GATE, IIT‐JAM, M.Sc. Entrance, JEST, TIFR and GRE in Physics 
 
2  1
According to W.K.B.,  pdq   n    where a1 and a2 are positive mid point
1  2

P2
E  a x  P  2m  E  a x 
2m E /  E /

 1
2m  E  a x dx   n   
E/ a
 E/ a
 2

 1
2m  E  ax dx   2m  E  ax dx   n   
0 E/a
E / a 0
 2

 1
2m  E  ax dx   n   
E/a
2
0
 2

2m  E  ax   t , At x  0, t  2mE; x  E / a, t  0

 2madx  dt
2 mE
2 mE  1 2  1
 2ma  t dt   n     2ma t 3/ 2
1/ 2
 n  
0
 2 3 0  2

4  1 4  1
  n     ma  2mE    n   
2 mE

3/ 2
ma t 3/2
3 0
 2 3  2
2/3
4  1  3a  1 
  23/ 2 am5 / 2 E 3/ 2   n     E    n  
3  2  4 2m  2 

Q58. The Hamiltonian H 0 for a three-state quantum system is given by the matrix

1 0 0  0 1 0
   
H 0   0 2 0  . When perturbed by H    1 0 1  where  1 , the resulting shift
0 0 2  0 1 0
   
in the energy eigenvalue E 0  2 is

(a) ,  2  (b)  , 2  (c)   (d)  2 


Ans: None of the answer is correct.
 1 0 0  0 1 0
Solution: H 0   0 2 0 , H  0  1 0 1
   
 0 0 2  0 1 0

H.No. 40‐D, Ground Floor, Jia Sarai, Near IIT, Hauz Khas, New Delhi‐110016 
Phone: 011‐26865455/+91‐9871145498
Website: www.physicsbyfiziks.com  | Email: fiziks.physics@gmail.com  
34 
fiziks
Institute for NET/JRF, GATE, IIT‐JAM, M.Sc. Entrance, JEST, TIFR and GRE in Physics 
 
 2 0  0 1
 0 2 in H 0 is not 0  1 0 in H  because H  is not in block diagonal form. So we

must diagonalised whole H  . The Eigen value at H   0,  2 0 ,  2 0 .

0 0 0 
 
After diagonalisation H  0  0 2 0  ,   0 is correction for Eigenvalue at H 0 .
 0 0  2 

So  2 0 is the correction for eigenvalue of H 0  2


None of the answer is correct.

NET/JRF (JUNE-2015)
Q59. The ratio of the energy of the first excited state E1 , to that of the ground state E0 , to that

L
of a particle in a three-dimensional rectangular box of side L, L and , is
2
(a) 3 : 2 (b) 2 :1 (c) 4 :1 (d) 4 : 3
Ans. (a)
 22 6 2  2
Solution: E   n 2
 n 2
 4 n 2
 , for ground state n  1, n  1, n  1  E 
2mL2 
x y z  x y z 0
2mL2
 22 9 2  2
For first excited state nx  1, n y  2, nz  1  E  E1  1  4  4  
2mL2 2mL2
E1 9 3
  
E0 6 2

Q60. If Li are the components of the angular momentum operator L , then the operator

 i 1,2,3  L, Li  equals
   
(a) L (b) 2L (c) 3L (d)  L
Ans. (b)

Solution: Let L  Lx iˆ  Ly ˆj  Lz kˆ

x  1, y  2, z  3

 L, Lx    Ly , Lx  j   Lz , Lx  kˆ  iLz ˆj  Ly kˆ i
 

H.No. 40‐D, Ground Floor, Jia Sarai, Near IIT, Hauz Khas, New Delhi‐110016 
Phone: 011‐26865455/+91‐9871145498
Website: www.physicsbyfiziks.com  | Email: fiziks.physics@gmail.com  
35 
fiziks
Institute for NET/JRF, GATE, IIT‐JAM, M.Sc. Entrance, JEST, TIFR and GRE in Physics 
 

  L, Lx  , Lx   i    Lz , Lx  ˆj   Ly , Lx  i   i .i Ly ˆj   i   Lz  i   Lz  i   .kˆ   2  Ly ˆj  Lz kˆ 
      

similarly,   L, Ly  Ly    2  Lx iˆ  Lz kˆ 
   

  L, Lz  Lz    2  Lx iˆ  Ly ˆj 
    

  L, Li  Li   2 2  Lx iˆ  Ly ˆj  Lz kˆ   2 L put   1
i 1,2,3
 

Q61. The wavefunction of a particle in one-dimension is denoted by   x  in the coordinate


 ipx
representation and by   p     x  e 
dx in the momentum representation. If the

action of an operator T̂ on   x  is given by Tˆ  x     x  a  , where a is a constant

then Tˆ  p  is given by


 iap
i
(a)  ap  p  (b) e 
  p

 iap
 i 
(c) e 
  p (d) 1  ap    p 
  
Ans. (c)
 ipx
Solution:   p     x e 
dx

T  x     x  a 
 ipx  ipx ipa  ip  x  a 

T   p    T  x  e 
dx    x  a  e 
dx  e 
  x  a e 
dx
ipa
 T  p   e    p 

Q62. The differential cross-section for scattering by a target is given by


d
 ,    a 2  b2 cos 2 
d
If N is the flux of the incoming particles, the number of particles scattered per unit time
is
4  1 
(a)
3

N a2  b2  (b) 4 N  a 2  b 2 
 6 

1 1   1 
(c) 4 N  a 2  b 2  (d) 4 N  a 2  b 2 
2 3   3 
H.No. 40‐D, Ground Floor, Jia Sarai, Near IIT, Hauz Khas, New Delhi‐110016 
Phone: 011‐26865455/+91‐9871145498
Website: www.physicsbyfiziks.com  | Email: fiziks.physics@gmail.com  
36 
fiziks
Institute for NET/JRF, GATE, IIT‐JAM, M.Sc. Entrance, JEST, TIFR and GRE in Physics 
 
Ans. (d)
d
Solution:  a 2  b 2 cos 2 
d
 2  2
2  b2 
  a 2   sin  d d  b 2  cos 2  sin  d  d  a 2 .4  b 2 .2   4  a 2  
0 0 0 0
3  3

 b2 
Number of particle scattered per unit time,  .N  4 N  a 2  
 3

1
Q63. A particle of mass m is in a potential V  m 2 x 2 , where  is a constant. Let
2
m  ipˆ  daˆ
aˆ   xˆ   . In the Heisenberg picture is given by
2  m  dt

(a)  â (b) i aˆ (c)  â † (d) i aˆ †


Ans. (b)
1
Solution: V  m 2 x 2
2
m  ipˆ 
aˆ   xˆ  
2  m 
daˆ 1 a a
  a, H   , 0
dt i t t

daˆ 1 m   p 2  im 2  1 m  2 p i 
   x,   pˆ , x 2     i   2 x  i  
dt i 2   2m  2m  i  2  2m 2 

m p  m  ip 
   i x   i x   i aˆ
2 m  2  m 

H.No. 40‐D, Ground Floor, Jia Sarai, Near IIT, Hauz Khas, New Delhi‐110016 
Phone: 011‐26865455/+91‐9871145498
Website: www.physicsbyfiziks.com  | Email: fiziks.physics@gmail.com  
37 
fiziks
Institute for NET/JRF, GATE, IIT‐JAM, M.Sc. Entrance, JEST, TIFR and GRE in Physics 
 
Q64. Two different sets of orthogonal basis vectors
 1   0    1 1 1  1  
  ,    and   ,    are given for a two dimensional real vector space.
 0   1    2  1  2  1  
The matrix representation of a linear operator  in these basis are related by a unitary
transformation. The unitary matrix may be chosen to be
 0 1  0 1
(a)   (b)  
1 0  1 0
1 1 1  1 1 0 
(c)   (d)  
2 1 1 2 1 1 
Ans. (c)
1 0 1 1 1  1 1 1 
Solution: u1    , u2     u  u1  u2   
0 1 2  1 1  2  1 1 
 
Q65. The Dirac Hamiltonian H  c . p   mc 2 for a free electron corresponds to the classical
relation E 2  p 2 c 2  m 2 c 4 . The classical energy-momentum relation of a piratical of
 q 
2
2  
 
charge q in a electromagnetic potential  , A is  E  q   c  p  A   m 2 c 4 .
2

 c 
Therefore, the Dirac Hamiltonian for an electron in an electromagnetic field is
 e      e 
(a) c . p  A. A   mc 2  e (b) c .  p  A    mc 2  e
c  c 

  e   e 
(c) c   . p  e  A    mc 2 (d) c .  p  A    mc 2  e
 c   c 
Ans. (d)
Solution: Electromagnetic interaction of Dirac particle
1
  qA  2 2
H   P   c  m c   q
2 2 4

 c  
Quantum mechanical Hamiltonian

    qA  
i   c  P     mc  q 
2

t   c  
put q  e
  e  
H   c .  P  A    mc 2  e 
  c  

H.No. 40‐D, Ground Floor, Jia Sarai, Near IIT, Hauz Khas, New Delhi‐110016 
Phone: 011‐26865455/+91‐9871145498
Website: www.physicsbyfiziks.com  | Email: fiziks.physics@gmail.com  
38 
fiziks
Institute for NET/JRF, GATE, IIT‐JAM, M.Sc. Entrance, JEST, TIFR and GRE in Physics 
 
Q66. A particle of energy E scatters off a repulsive spherical potential
V for r  a
V r    0
 0 for r  a
where V0 and a are positive constants. In the low energy limit, the total scattering cross-
2
 1  2m
section is   4 a  tanh ka  1 , where k 2  2 V0  E   0 . In the limit V0  
2

 ka  h
the ratio of  to the classical scattering cross-section off a sphere of radius a is
1
(a) 4 (b) 3 (c) 1 (d)
2
Ans. (a)
2
1 
Solution:   4 a  tanh ka  1
2

 ka 
2
 1 
ka   , tanh ka  1    4 a   1 2

 ka 
and ka   , lim  H  4 a 2
ka 

H
classically  c   a 2  4
c
NET/JRF (DEC-2015)

Q67. A Hermitian operator O has two normalized eigenstates 1 and 2 with eigenvalues 1

and 2 , respectively. The two states u  cos  1  sin  2 and v  cos  1  sin  2

are such that v O   7 / 4 and u v  0 . Which of the following are possible values of

 and  ?
   
(a)    and   (b)   and  
6 3 6 3
   
(c)    and   (d)   and   
4 4 3 6
Ans.: (a)
Solution: u  cos  1  sin  2 , v  cos  1  sin  2

H.No. 40‐D, Ground Floor, Jia Sarai, Near IIT, Hauz Khas, New Delhi‐110016 
Phone: 011‐26865455/+91‐9871145498
Website: www.physicsbyfiziks.com  | Email: fiziks.physics@gmail.com  
39 
fiziks
Institute for NET/JRF, GATE, IIT‐JAM, M.Sc. Entrance, JEST, TIFR and GRE in Physics 
 
7
it is given Oˆ 1  1 , Oˆ 2  2 2  v Oˆ v 
4
7 7
cos 2   2sin 2    cos 2   sin 2   1  sin 2    1
4 4
3 
sin    
2 3
u v  0  cos  cos   sin  sin   0  cos      0

      5 
    or        or      or   
2 2 2 3 3 2 6 6
x
Q68. The ground state energy of a particle of mass m in the potential V  x   V0 cosh   ,
L
2
where L and V0 are constants (with V0  ) is approximately
2mL2

 2V0  V0
(a) V0  (b) V0 
L m L m

 V0  V0
(c) V0  (d) V0 
4L m 2L m
Ans.: (d)
x V
Solution: 
V0  cosh    0 e x / L  e  x / L
L 2

V0  x 1  x   V0  x 1  x  
2 2

 1     ....  1      ....
2  L 2!  L   2  L 2!  L  
2
V V V x 1 V 
 0  0  0    V0   02  x 2
2 2 2 L 2 L 

V0 V0
K , 
L2 mL2
So, ground state energy is

  V0  V0
V0   V0  2
 V0 
2 2 mL 2L m

H.No. 40‐D, Ground Floor, Jia Sarai, Near IIT, Hauz Khas, New Delhi‐110016 
Phone: 011‐26865455/+91‐9871145498
Website: www.physicsbyfiziks.com  | Email: fiziks.physics@gmail.com  
40 
fiziks
Institute for NET/JRF, GATE, IIT‐JAM, M.Sc. Entrance, JEST, TIFR and GRE in Physics 
 
Q69. Let  nlm denote the eigenstates of a hydrogen atom in the usual notation. The state

1
2 200  3 211  7 210  5 211 
5
is an eigenstate of
(a) L2 , but not of the Hamiltonian or Lz (b) the Hamiltonian, but not of L2 or Lz

(c) the Hamiltonian, L2 and Lz (d) L2 and Lz , but not of the Hamiltonian
Ans.: (b)
1
Solution:    2 200  3 211  7 2 10  5 2 1 1 
5
13.6
H   
4
So  is eigen state of H

But L2     and Lz    

So  is not eigen state of L2 and Lz

1
Q70. The Hamiltonian for a spin- particle at rest is given by H  E0  z   x  , where  x
2
and  z are Pauli spin matrices and E0 and  are constants. The eigenvalues of this
Hamiltonian are

(a)  E0 1   2 (b)  E0 1   2

 1 
(c) E0 (doubly degenerate) (d) E0 1   2 
 2 
Ans.: (a)
 1 0   0 1  1 
Solution: H  E0  z   x   E0        H  E0  
  0 1   1 0   1 

if  is eigen value, then


 1     
H   I  0  E0  0,    E0 1   2
   1    

H.No. 40‐D, Ground Floor, Jia Sarai, Near IIT, Hauz Khas, New Delhi‐110016 
Phone: 011‐26865455/+91‐9871145498
Website: www.physicsbyfiziks.com  | Email: fiziks.physics@gmail.com  
41 
fiziks
Institute for NET/JRF, GATE, IIT‐JAM, M.Sc. Entrance, JEST, TIFR and GRE in Physics 
 
Q71. A hydrogen atom is subjected to the perturbation
2r
V pert  r   cos
a0

where a0 is the Bohr radius. The change in the ground state energy to first order in 

   
(a) (b) (c) (d)
4 2 2 4
Ans.: (d)
Solution: For First order perturbation
r
1  2r 
E11  100 V p 100 , 100  e a , V p  cos  
 a03  a0 
 2 r  2 r
1  2r  4  2r 
E   3 e a0  cos   4 r 2 dr  3  e a0 cos   r 2 dr
1
1
0
 a0  a0  a0 0  a0 

 ia2 r i 2 r

2   a0 1i  2 
 2 r  2 r  2 r 1 i 
4 e  e 0
0 a
 2
 3 e
a03  0 0
a0
  r dr   e r dr  e a0
r 2
dr 
a0 0

2
 

 
 
2 2! 2!   1 1 
     
a03   2 
3
 
3
2  1  i 3 1  i 3 
 1  i   1  i  
2  
  a0   0
a  

 
   
 1 1    1  1 
 
2 3
3  1 i  
3
4 2 4
 i 3 i 3

   1 i 
 
3
 2   2     e e 4

  2  2  

  i 34  i 3
    3  
  e  e 4
  2 cos  4  
4 2   4 2   

   1   
 2      E11 
4 2   2  4 4

H.No. 40‐D, Ground Floor, Jia Sarai, Near IIT, Hauz Khas, New Delhi‐110016 
Phone: 011‐26865455/+91‐9871145498
Website: www.physicsbyfiziks.com  | Email: fiziks.physics@gmail.com  
42 
fiziks
Institute for NET/JRF, GATE, IIT‐JAM, M.Sc. Entrance, JEST, TIFR and GRE in Physics 
 
Q72. The product of the uncertainties  Lx   Ly  for a particle in the state a 1,1  b 1, 1

where l , m denotes an eigenstate of L2 and Lz will be a minimum for

(a) a  ib (b) a  0 and b  1

3 1
(c) a  and b  (d) a  b
2 2
Ans.: (d)
Solution:   a 1,1  b 1, 1 , L   2b 1, 0 , L2   2 2b 1,1

L   2a 1, 0 , L2   2 2 a 1, 1

 L2   a 2 2  b 2 2  a  b 2 2 2
2 2
 2

 L2z   a  b 2  2 2

Lx  0, Ly  0

L2x 
1  2
4 
L  L2
  2 L2
 L2
2  4 
 
  1  a*b  b*a 2 2  2 2 2   2     a 2
b
2

2  *
L2x 
2 

a b  b* a  a  b 
2 2
 
L2y 
 
2 L2  L22  L2  L2
4
  2
2
L2y 
2 
a  b  a*b  b*a 
2
  
2  2
    a b  b a  
2 2
Lx Ly   a b
2 2 2
* *
 a  b 1
2 
2
 
2
Lx Ly  1  a*b  b*a (i)
2
1 i 2
Now check option (a) a  ib  a  ,b   Lx Ly 
2 2 2
 2
Option (b) a  0, b  1  Lx Ly 
2
3 1 2
Option (c) a  ,b   Lx Ly 
2 2 4
1 1
Option (d) a  b  a  ,b   Lx Ly  0 option (d) is correct
2 2
H.No. 40‐D, Ground Floor, Jia Sarai, Near IIT, Hauz Khas, New Delhi‐110016 
Phone: 011‐26865455/+91‐9871145498
Website: www.physicsbyfiziks.com  | Email: fiziks.physics@gmail.com  
43 
fiziks
Institute for NET/JRF, GATE, IIT‐JAM, M.Sc. Entrance, JEST, TIFR and GRE in Physics 
 
Q73. The ground state energy of a particle in potential V  x   g x , estimated using the trail
wavefunction
 c 2

  x    a5
a  x2 ,   x a
0, x a

(where g and c are constants) is
1/ 3 1/ 3 1/ 3 1/ 3
15   2 g 2  5  2 g 2  3  2 g 2  7  2 g 2 
(a)   (b)   (c)   (d)  
16  m  6 m  4 m  8 m 
Ans.: (a)
a
15
   dx  1  c  16
*
Solution:
a

 2  15  2 2
a
10 2
T 
2m  16a 2  a
  a 2
 
x 2

x 2
a  
x 2
dx  T
4ma 2

15  2 g
a
V 
16a 05   5

x a 2  x 2 dx  V  ga
16

E T  V (i)

10 2 5 ga
E 
4ma 2 16
1
dE 8  2  3
 0  a3   a  2 
da mg  mg 
put the value of a in equation (i)
1
15   2 g 2  3
E  
16  m 

H.No. 40‐D, Ground Floor, Jia Sarai, Near IIT, Hauz Khas, New Delhi‐110016 
Phone: 011‐26865455/+91‐9871145498
Website: www.physicsbyfiziks.com  | Email: fiziks.physics@gmail.com  
44 
fiziks
Institute for NET/JRF, GATE, IIT‐JAM, M.Sc. Entrance, JEST, TIFR and GRE in Physics 
 
NET/JRF (JUNE-2016)
Q74. The state of a particle of mass m in a one dimensional rigid box in the interval 0 to L is

2  3  2 x  4  4 x  
given by the normalized wavefunction   x    sin    sin    . If its
L  5  L  5  L 
energy is measured the possible outcomes and the average value of energy are,
respectively
h 2 2h 2 73 h 2 h2 h2 19 h 2
(a) , and (b) , and
2mL2 mL2 50 mL2 8mL2 2mL2 40 mL2
h 2 2h 2 19 h 2 h 2 2h 2 73 h 2
(c) , and (d) , and
2mL2 mL2 10 mL2 8mL2 mL2 200 mL2
Ans: (a)
2  3  2 x  4  4 x  
Solution:   x    sin    sin  
L  5  L  5  L 
n 2 2  2
Measurement E 
2mL2
h2 2h 2
 n  2  E2  and n  4  E4 
2mL2 mL2
9 16
Probability p  E2   and p  E4  
25 25
Now, average value of energy is
9 h2 16 2h 2 73h 2
E   an p  an      
25 2mL2 25 mL2 50mL2
Q75. If Lˆ x , Lˆ y , Lˆ z are the components of the angular momentum operator in three dimensions

the commutator  Lˆ x , Lˆ x Lˆ y Lˆ z  may be simplified to

 
(a) iLx Lˆ2z  Lˆ2y (b) iLˆ z Lˆ y Lˆ x

(c) iL  2 Lˆ  Lˆ 
x
2
z
2
y (d) 0

Ans: (a)
Solution:  Lx , Lx Ly Lz   Lx  Lx , Ly Lz    Lx , Lx  Ly Lz

 Lx  Lx , Ly  Lz  Lx Ly  Lx , Lz   0  Lx iLz  Lz  Lx Ly  iLy 


 iLx L2z  iLx L2y  iLx L2z  L2y 
H.No. 40‐D, Ground Floor, Jia Sarai, Near IIT, Hauz Khas, New Delhi‐110016 
Phone: 011‐26865455/+91‐9871145498
Website: www.physicsbyfiziks.com  | Email: fiziks.physics@gmail.com  
45 
fiziks
Institute for NET/JRF, GATE, IIT‐JAM, M.Sc. Entrance, JEST, TIFR and GRE in Physics 
 
Q76. Suppose that the Coulomb potential of the hydrogen atom is changed by adding an
 Ze 2 g
inverse-square term such that the total potential is V  r     2 , where g is a
r r
constant. The energy eigenvalues Enlm in the modified potential

(a) depend on n and l , but not on m


(b) depend on n but not on l and m
(c) depend on n and m , but not on l
(d) depend explicitly on all three quantum numbers n , l and m
Ans: (b)
ze 2 g
Solution: V  r     2 is central potential
r r
So angular momentum is conserve then eigen value En ,l ,m will depend only on n , which

is principal quantum number.


Q77. The eigenstates corresponding to eigenvalues E1 and E2 of a time independent

Hamiltonian are 1 and 2 respectively. If at t  0 , the system is in a state

  t  0   sin  1  cos  2 , then the value of   t    t  at time t will be

(a) 1 (b)
 E sin 1
2
  E2 cos 2  
E12  E22

(c) eiE1t /  sin   eiE2t /  cos  (d) e iE1t /  sin 2   eiE2t /  cos 2 
Ans: (a)
Solution:   t  0   sin  1  cos  2
 iE1t  iE2t
  t   sin  1 e 
 cos  2 e 

 i  E1  E2 t

  t    t   sin  1 1  cos  2 2  2 Re e
2 2 
sin   cos  1 2

 sin 2   cos 2   0  1  1 2  0

H.No. 40‐D, Ground Floor, Jia Sarai, Near IIT, Hauz Khas, New Delhi‐110016 
Phone: 011‐26865455/+91‐9871145498
Website: www.physicsbyfiziks.com  | Email: fiziks.physics@gmail.com  
46 
fiziks
Institute for NET/JRF, GATE, IIT‐JAM, M.Sc. Entrance, JEST, TIFR and GRE in Physics 
 
1
Q78. Consider a particle of mass m in a potential V  x   m 2 x 2  g cos kx . The change in
2
1
the ground state energy, compared to the simple harmonic potential m 2 x 2 , to first
2
order in g is

 k 2   k 2 
(a) g exp    (b) g exp  
 2m   2m 

 2k 2    k 2 
(c) g exp    (d) g exp   
 m   4m 
Ans: (d)
Solution: Ground state wavefunction
1
m x
 m  4  2 
2

 0  x    e
  
The perturbation term is H p  g cos kx

First order correction E01    0*  x  H P 0  x  dx


1 1
g  m  2   m x ikx 
 m x 2    m x
 m  2  eikx  e  ikx 
2 2

  ge  dx     e .e dx   e  . e ikx dx 


      2  2        
1 1
m x   m x 
g  m  2 g  m  2
2 2
  ikx ikx
   e dx    e
 
dx
2      2     
From 1st term, we have
m  2 2 ikx  ik    ik   
1 2 2 1
m 
2
  ik  
g  m  2  x      
2 m  2 m   2 m   g  m  2   x  
k 2

   e
    2 m 
  e dx   e 4 m
dx
2      2     
1
m 
2
 ik  
g  k   m  2
2
k 2
 x  
 e
  2 m 
 e 4 m  dx  e 4 m
2     
1
m x 
g  m  2
2
 ikx
 e

Similarly, from term (ii),  dx
2     

H.No. 40‐D, Ground Floor, Jia Sarai, Near IIT, Hauz Khas, New Delhi‐110016 
Phone: 011‐26865455/+91‐9871145498
Website: www.physicsbyfiziks.com  | Email: fiziks.physics@gmail.com  
47 
fiziks
Institute for NET/JRF, GATE, IIT‐JAM, M.Sc. Entrance, JEST, TIFR and GRE in Physics 
 
1
m 
2
 ik  
g  4kmh  m  2
2
 k 2
 x  
 
  2 m 
 e  e dx  e 4 m
2     

g  k  k  
2 2 2
k 
 
Hence, E   e 4 m  e 4 m   ge 4 m
1
0
2  
Q79. The energy levels for a particle of mass m in the potential V  x    x , determined in

the WKB approximation

 1
b
2m  E  V  x dx   n   
a  2

(where a, b are the turning points and n  0,1, 2... ), are


2 2
 h  1  3  3h  1  3
(a) En    n   (b) En    n  
4 m  2   4 2m  2 
2 2
 3h  1  3  h  1  3
(c) En    n   (d) En    n  
4 m  2   4 2m  2 
Ans: (b)
Solution: V  x    x
V  x
  x, x0
 V  x  
   x, x0
E E
b
 1 
2m  E  V  x dx   n      
a  2 x
E

 E E  1
From figure, a     , b     2m  E  V  x dx   n    
    
E  2

E E
 
 1  1
0
 2m  E   xdx   E   xdx   n      2 2m  E   x  dx    n    

E 0  2 0  2

dt
put E   x  t , dx  

E
limit x  0  t  E , x t 0

H.No. 40‐D, Ground Floor, Jia Sarai, Near IIT, Hauz Khas, New Delhi‐110016 
Phone: 011‐26865455/+91‐9871145498
Website: www.physicsbyfiziks.com  | Email: fiziks.physics@gmail.com  
48 
fiziks
Institute for NET/JRF, GATE, IIT‐JAM, M.Sc. Entrance, JEST, TIFR and GRE in Physics 
 
  dt   1
0
2 2m  t     n   
E     2
0
2 2m  2 32   1 2 2m 2 32  1
  t    n      .E   n    h
  3 E  2  3  2
2
 1  3 
3
 3  1  3
 E  n 2
 En    n  
 2  4 2m  4 2m  2 
Q80. A particle of mass m moves in one dimension under the influence of the potential
V  x     x  , where  is a positive constant. The uncertainty in the product

 x  p  in its ground state is

 
(a) 2 (b) (c) (d) 2
2 2
Ans: (c)
Solution: V  x     x 

For this potential wavefunction


  x
  e x , x0
  x  
 x
  e , x0

which evenfunction about x  0


so x  0, p  0 x

1 1
now x 2  2  x 2 e 2 x dx   x  x2  x 
2

0
2 2
2
 0 
d2 x d
2
 x d
2
p 2  2   * 2
 dx    2
  e 2
 ex
dx   2
  e 2
 e x dx

dx 
dx 0
dx

 2 3  2 3
0

e e
2 x 2 x
   2 3
dx    2 3
dx       2 2 , which is not possible
 0
2 2
 2
d
  dx   2 2 , p  p2  p  
2 2 2
so, we will use the formula p

dx
1 
now, x.p  . 
2 2
H.No. 40‐D, Ground Floor, Jia Sarai, Near IIT, Hauz Khas, New Delhi‐110016 
Phone: 011‐26865455/+91‐9871145498
Website: www.physicsbyfiziks.com  | Email: fiziks.physics@gmail.com  
49 
fiziks
Institute for NET/JRF, GATE, IIT‐JAM, M.Sc. Entrance, JEST, TIFR and GRE in Physics 
 
2  4
Q81. The ground state energy of a particle of mass m in the potential V  x   x ,
6m
1
 x 2
  4
estimated using the normalized trial wavefunction   x     e 2
, is
 

  1   3
 dxx 2 e  x   dx x 4 e  x 
2 2
[use and ]
  2   4 2
3 2 13 8 2 13 2 2 13 3 2 13
(a)   (b)  (c)  (d) 
2m 3m 3m 8m
Ans: (d)
1
x
  4   2
2

Solution: E  T  V , for   x     e 2 , T 
  4m
1 1
 
   2   4  x   2   2  3 2 
2 2
V    x e dx     x 4 e  x dx  . 2 
2 2

    6m    6m 
6m 4 8m 2

 2 2 
E   (i)
4m 8m 2
dE  2 2 2  2    1
   0   1    0       3
d 4m 8m 3 4m   3 
Putting the value of  in equation (i),
 1

2 1
2  2  1
   3  3 2 13
E    3     
3  
2
4m  2  8m
8m    3
4m
 

H.No. 40‐D, Ground Floor, Jia Sarai, Near IIT, Hauz Khas, New Delhi‐110016 
Phone: 011‐26865455/+91‐9871145498
Website: www.physicsbyfiziks.com  | Email: fiziks.physics@gmail.com  
50 
fiziks
Institute for NET/JRF, GATE, IIT‐JAM, M.Sc. Entrance, JEST, TIFR and GRE in Physics 
 
NET/JRF (DEC-2016)
Q82. Consider the two lowest normalized energy eigenfunctions  0  x  and  1  x  of a one

d 0
dimensional system. They satisfy  0  x    0*  x  and  1  x    , where  is a real
dx
constant. The expectation value of the momentum operator in the state  1 is

  2
(a)  (b) 0 (c) (d)
 2
 2
2
Ans. : (b)
d 0
Solution:  1  x   
dx
  
   d 0 d 2
px    px dx    1  i 1  dx    *  i  20 dx
*
*

x 
1
   
dx dx

d 0 d 2 0
 i   
2
dx

dx dx 2
Integrate by parts

2  d d 0 d 2 0 d 0 
  
d 0 d 2 0
I  i   0   dx 2 dx dx   0   i    dx dx 2 dx
2

 dx dx
  

d 0 d 2 0
I  0   i    dx dx 2 dx
2



d 0 
 0 , 0  0, x  
dx 
I  0  I  2I  0  I  0  px  0

d
Q83. Consider the operator, a  x  acting on smooth function of x . Then commutator
dx
 , cos x  is
(a)  sin x (b) cos x (c)  cos x (d) 0
Ans. : (a)
d
Solution: a  x 
dx

H.No. 40‐D, Ground Floor, Jia Sarai, Near IIT, Hauz Khas, New Delhi‐110016 
Phone: 011‐26865455/+91‐9871145498
Website: www.physicsbyfiziks.com  | Email: fiziks.physics@gmail.com  
51 
fiziks
Institute for NET/JRF, GATE, IIT‐JAM, M.Sc. Entrance, JEST, TIFR and GRE in Physics 
 

 a, cos x    x  , cos x    x, cos x    , cos x   0   , cos x 


d d d
 dx   dx   dx 
d  d d
 dx , cos x   x   dx cos x  x   cos x dx

d cos xd
 cos x    sin x     sin x
dx dx
 a, cos x   x    sin x
 a, cos x    sin x
   
Q84. Consider the operator   p  qA , where p is the momentum operator,

A   Ax , Ay , Az  is the vector potential and q denotes the electric charge. If

B   Bx , By , Bz  denotes the magnetic field, the z -component of the vector operator
 
   is
(a) iqBz  q  Ax p y  Ay px  (b) iqBz  q  Ax p y  Ay px 

(c) iqBz (d) iqBz


Ans. : (d)
  
Solution:   p  qA
       
 
    
 
   

 p  qA  p  qA   p  p  qp  A  qA  p  q 2 A  A
 
p  p  0
    
 
 qp  A   q i  A   qiB
   
 
qA  p  q A i   0 
 
q 2 A  A  0
  
    qiB
So, z component is given by qiBz

H.No. 40‐D, Ground Floor, Jia Sarai, Near IIT, Hauz Khas, New Delhi‐110016 
Phone: 011‐26865455/+91‐9871145498
Website: www.physicsbyfiziks.com  | Email: fiziks.physics@gmail.com  
52 
fiziks
Institute for NET/JRF, GATE, IIT‐JAM, M.Sc. Entrance, JEST, TIFR and GRE in Physics 
 

Q85. The 2  2 identity matrix I and the Pauli matrices  x ,  y ,  z do not form a group under matrix multiplication. The minimum number of 2  2
matrices, which includes these four matrices, and form a group (under matrix multiplication) is
(a) 20 (b) 8 (c) 12 (d) 16
Ans. (d)
 I I i I i  I x  x i x i x y  y i y i y z  z i z i z

I I I i I i  I x  x i x i x y  y i y i y z  z i z i z

I I I i  I i I  x x i x i x  y y i y i y  z z i z i z

i I i I i  I I I i x i x  x x i y i y  y y i z i z  z z


i  I i  I i I I I i x i x x  x i y i y y  y i z i z z  z

x x  x i x i x I I i I i  I i z i z  z z i y i y y  y

 x  x x i x i x I I i  I i I i z i z z  z i y i y  y y

i x i x i x  x x i I i  I I I  z z i z i z y  y i y i y

i x i x i x x  x i  I i I I I z  z i z i z  y y i y i y

y y  y i y i y i z i z z  z I I i I i  I i x i x  x x

 y  y y i y i y i z i z  z z I I i  I i I i x i x x  x

i y i y i y  y y z  z i z i z i I i  I I I  x x i x i x

i y i y i y y  y  z z i z i z i  I i I I I x  x i x i x

z z  z i z i z i y i y  y y i x i x x  x I I i I i  I

H.No. 40‐D, Ground Floor, Jia Sarai, Near IIT, Hauz Khas, New Delhi‐110016 
Phone: 011‐26865455/+91‐9871145498
Website: www.physicsbyfiziks.com  | Email: fiziks.physics@gmail.com  
53 
fiziks
Institute for NET/JRF, GATE, IIT‐JAM, M.Sc. Entrance, JEST, TIFR and GRE in Physics 
 

 z  z z i z i z i y i y y  y i x i x  x x I I i  I i I

i z i z i z  z z  y y i y i y x x i x i x i I i  I I I

i z i z i z z  z y  y i y i y x x i x i x i  I i I I I

H.No. 40‐D, Ground Floor, Jia Sarai, Near IIT, Hauz Khas, New Delhi‐110016 
Phone: 011‐26865455/+91‐9871145498
Website: www.physicsbyfiziks.com  | Email: fiziks.physics@gmail.com  
54 
fiziks
Institute for NET/JRF, GATE, IIT‐JAM, M.Sc. Entrance, JEST, TIFR and GRE in Physics 
 
Q86. The dynamics of a free relativistic particle of mass m is governed by the Dirac
   
Hamiltonian H  c . p   mc 2 , where p is the momentum operator and    x ,  y ,  z 

and  are four 4  4 Dirac matrices. The acceleration operator can be expressed as
2ic   
(a)  cp   H  (b) 2ic 2

ic  2ic  
(c) H (d)   cp   H 
 
Ans. : (a)
 
Solution: H  c . p   mc 2
If vx velocity of x direction
From the Ehrenfest theorem
dx 1 x 1
vx    x, H     x, c x px  c y p y  c z p z   mc 2   0
dt i t i 
c
  x,  x px   c x
i
Similarly, acceleration is given by
dvx 1 c
ax    c x , H    x , c x px  c y p y  c z pz   mc 2 
dt i i
Using relation  i j   j i  0 ,  i    i  0 and  i , p j   0

 x , c x px   0
 
 x , c y p y   c  x y   y x  p y   y c  x , p y   c x y  c x y  p y  0  2c x y p y
 
 x , c z pz   c x z  c z x  pz   z c x , pz   c x z   c x z  pz  0  2c x z pz
 x ,  mc 2    x    x  mc 2  2mc 2 x 
 
c
ax   2c x y p y  2c x z p z  2 x  mc 2 
i  

2 x c
ax  c y p y  c z pz   mc 2  c x p x  c x px 
i  

2 x c
ax  c x p x  c y p y  c z pz   mc 2  c x px 
i  

H.No. 40‐D, Ground Floor, Jia Sarai, Near IIT, Hauz Khas, New Delhi‐110016 
Phone: 011‐26865455/+91‐9871145498
Website: www.physicsbyfiziks.com  | Email: fiziks.physics@gmail.com  
55 
fiziks
Institute for NET/JRF, GATE, IIT‐JAM, M.Sc. Entrance, JEST, TIFR and GRE in Physics 
 
2c
ax   x .H  c x x px  
i
2ic

c x x px   x .H  ,  x2  x  
  2ic    
a  ax iˆ  a y ˆj  az kˆ  
 
cp   .H 

 
Q87. A particle of charge q in one dimension is in a simple harmonic potential with angular
2
t
 
frequency  . It is subjected to a time- dependent electric field E  t   Ae  
, where A

and  are positive constants and   1 . If in the distant past t   the particle was
in its ground state, the probability that it will be in the first excited state as t   is
proportional to
1 1
  2  2 1
2
(a) e (b) e 2 (c) 0 (d)
 
2

Ans. : (a)
2
t2
  i fi t
Solution: Transition probability is proportional to Pif   e  2
e where


3 1
  
 fi  2 2 

2
 t2
Pif   exp  it dt
 2
1  2 2 2 
i 2  
2
 t2   
2  i 
Now calculate  exp   2  it  dt   exp  2 t  it     
      
  2   2  

  2 2   1  it 
2
 exp     exp 2 
t   dt
 4     2 
2
 t2
Pif   exp  it dt
 2
2
  2 2   1  it 
2
Pif  exp     exp 2  t   dt
 4     2 
 2 2
Pif  exp 
2

H.No. 40‐D, Ground Floor, Jia Sarai, Near IIT, Hauz Khas, New Delhi‐110016 
Phone: 011‐26865455/+91‐9871145498
Website: www.physicsbyfiziks.com  | Email: fiziks.physics@gmail.com  
56 
fiziks
Institute for NET/JRF, GATE, IIT‐JAM, M.Sc. Entrance, JEST, TIFR and GRE in Physics 
 
Q88. A particle is scattered by a central potential V  r   V0 re   r , where V0 and  are positive
 
constants. If the momentum transfer q is such that q  q   , the scattering cross-

section in the Born approximation, as q   , depends on q as

dn
 x e dx  n 
n ax
[You may use e ax dx ]
da
(a) q 8 (b) q 2 (c) q 2 (d) q 6
Ans. : (a)
Solution: The form factor is given for high energy as q  
 
2 m 2m
f  ,     rV  r  sin qr dr  2  r 2V0 e   v sin qr dr
q 0
2
q 0

mV0  2  r   iq  
  
2 m 2  r e
iqr
 e  iqr
 2 V0  r e dr  2 i   r e dr   r 2 e   dr 
 r   iq

q 0 2i  q 0 0 

mV0i 
 2 
2

2
 2 0 

 2mV i     iq      iq 
3 3
 
 q     iq 3    iq 3   q     iq 3    iq 3 
   

2mV0 i    iq  3 iq  3 q      iq  3 iq  3 q  
3 3 2 2 3 3 2 2

 2
q   2  q2 
3

   3 2 
2mV0i  6  2iq  2iq 3  2mV0  2q  6  q 
 2  2
 q    2  q 2 3 
  
q   2  q2 3 
 

q3  6 2  1 1 1  2 
  2  2  3
 q2  6
 4  2  1
q q   2  q q  q 
q  2  1
6

q 
    f     q 4   q 8
2 2

Q89. A particle in one dimension is in a potential V  x   A  x  a  . Its wavefunction   x  is


d
continuous everywhere. The discontinuity in at x  a is
dx
(b) A   a    a  
2m
(a) A  a 
2
2
(c) A (d) 0
2m
H.No. 40‐D, Ground Floor, Jia Sarai, Near IIT, Hauz Khas, New Delhi‐110016 
Phone: 011‐26865455/+91‐9871145498
Website: www.physicsbyfiziks.com  | Email: fiziks.physics@gmail.com  
57 
fiziks
Institute for NET/JRF, GATE, IIT‐JAM, M.Sc. Entrance, JEST, TIFR and GRE in Physics 
 
Ans. : (a)

2 d   x 
2

Solution:   A  x  a   x   E  x 
2m dx 2
Integrates both side within limit
a   to a 
a  a  a 
2 d 2
dx   A   x  a  dx  E    x  dx
2m a dx 2

a  a 

 2  d II d I 
     A  a   0
2m  dx dx 
d II d I 2mA
  2  a
dx dx 
d 2mA
so discontinues in at x  a is 2   a  .
dx 

NET/JRF (JUNE-2017)
Q90. If the root-mean-squared momentum of a particle in the ground state of a one-
dimensional simple harmonic potential is p0 , then its root-mean-squared momentum in
the first excited state is
(a) p0 2 (b) p0 3 (c) p0 2 / 3 (d) p0 3 / 2
Ans. : (b)

Solution: P  m  Pˆ  m 
 a  a† 
2i
m  2
P2  
2
 a  a†2   2 N  1 
m 
P2  
2
a 2  a †2  2 N  1 
For any state n ,

a 2  0, a †2  0 and 2 N  1  2n  1

m 
P 2   2n  1 and P  0
2

H.No. 40‐D, Ground Floor, Jia Sarai, Near IIT, Hauz Khas, New Delhi‐110016 
Phone: 011‐26865455/+91‐9871145498
Website: www.physicsbyfiziks.com  | Email: fiziks.physics@gmail.com  
58 
fiziks
Institute for NET/JRF, GATE, IIT‐JAM, M.Sc. Entrance, JEST, TIFR and GRE in Physics 
 
m 
Prms  P 2  P  Prms  2n  1
2

m 
For ground stat n  0, Prms   P0
2

m 
So, for n  1, Prms  3
2

Prms  3P0

Q91. Consider a potential barrier A of height V0 and width b , and another potential barrier B

of height 2V0 and the same width b . The ratio TA / TB of tunnelling probabilities TA and

TB , through barriers A and B respectively, for a particle of energy V0 /100 is best


approximated by

 
(a) exp  1.99  0.99  8 mV0b 2 /  2 
 
(b) exp  1.98  0.98
  8 mV0b 2 /  2 

(c) exp 
  2.99  0.99  8 mV0b 2 /  2 

(d) exp 
  2.98  0.98  8 mV0b 2 /  2 

Ans. : (a)
 2 m V  E  V0
Solution: T  e , where E 
100
For potential A, V  V0
2m  V0 
  V0  
TA e 2  100 

2 m  99 
  V0 
 2 m 0.99V0 
TA e  2  100 
e
V0
For Potential B, V  2V0 and E =
100
2m  V0 
  2V0  
TB e 2  100 

2 m  199V0 
    2 m 1.99V0 
TB e  2  100 
 e
 0.99V0
TA e

TB e  1.99V0

H.No. 40‐D, Ground Floor, Jia Sarai, Near IIT, Hauz Khas, New Delhi‐110016 
Phone: 011‐26865455/+91‐9871145498
Website: www.physicsbyfiziks.com  | Email: fiziks.physics@gmail.com  
59 
fiziks
Institute for NET/JRF, GATE, IIT‐JAM, M.Sc. Entrance, JEST, TIFR and GRE in Physics 
 
TA
TB
 e  1.99V0
e
 0.99V 0

Q92. A constant perturbation H  is applied to a system for time  t (where H  t   )
leading to a transition from a state with energy Ei to another with energy E f . If the time

of application is doubled, the probability of transition will be


(a) unchanged (b) doubled (c) quadrupled (d) halved
Ans. : (c)
Solution: For constant potential transition probability
2
 f v i  2  fi ti 
pif  4  sin 
h 2 2
fi  2 

at ti  2ti ,
2
4  f v i  fi ti
pif  sin 2
h 2 2
fi 2

at ti  2ti ,
2 2
4  f v i   fi 2ti  4  f v  i
p ff  sin 2   sin  fi ti 
h 2 2fi  2  h 2 2fi

sin 2  fi ti 
 2fi ti2
pif sin  fi ti 
2
 t 2 2

  t1  0
fi i

p ff  t    t   t 
2
sin  fi i 
2
sin 2  fi i   fi i 
 2   2  2 
 2fi ti2
2
4 2fi ti2
 4
 2fi ti2

pif (2)
 4  pif (2)  4 pif (1)
pif (1)

H.No. 40‐D, Ground Floor, Jia Sarai, Near IIT, Hauz Khas, New Delhi‐110016 
Phone: 011‐26865455/+91‐9871145498
Website: www.physicsbyfiziks.com  | Email: fiziks.physics@gmail.com  
60 
fiziks
Institute for NET/JRF, GATE, IIT‐JAM, M.Sc. Entrance, JEST, TIFR and GRE in Physics 
 
a b
Q93. The two vectors   and   are orthonormal if
0 c
(a) a  1, b  1/ 2, c  1/ 2 (b) a  1, b  1, c  0
(c) a  1, b  0, c  1 (d) a  1, b  1/ 2, c  1/ 2
Ans. : (c)
a b
Solution: 1    , 2   
0 c
1 1  1  a  1

2 2  1  b  c 1
2 2

1 2  0   a 0   bc   0

a.b  0  c  0  a  b  0
so b  0

c  1, c  1
2

a  1, b  0, c  1
Q94. Consider the potential
  
V  r    i V0 a 3    r  ri 
3


where ri are the position vectors of the vertices of a cube of length a centered at the

2
origin and V0 is a constant. If V0 a 2  , the total scattering cross-section, in the low-
m
energy limit, is
2
 mV0 a 2 
2 16a 2  mV0 a 2 
(a) 16a   (b)  
  
2
 2  2 
2
64a 2  mV0 a 2  64a 2  mV0 a 2 
(c)   (d)  
  2   2  2 
Ans. : (c)
 
Solution: V  r    V0 a 3 3  r  ri 
i

H.No. 40‐D, Ground Floor, Jia Sarai, Near IIT, Hauz Khas, New Delhi‐110016 
Phone: 011‐26865455/+91‐9871145498
Website: www.physicsbyfiziks.com  | Email: fiziks.physics@gmail.com  
61 
fiziks
Institute for NET/JRF, GATE, IIT‐JAM, M.Sc. Entrance, JEST, TIFR and GRE in Physics 
 
  V0 a   x  xi    y  yi    z  zi 
3

where xi , yi , zi are co-ordinate at 8 corner cube whose center is at origin.

m
f     2 
V  r d 3 r
2 

m 8
       x  xi    y  yi   z  zi dxdydz
3
V a
2  2
0
i 1

m
 V a 3 1  1  1  1  1  1  1  1
2  2 0

8mV0 a 3 4mV0 a 3
 
2  2  2

total scattering cross section    f   sin  d d .


2

16m 2V02 a 6
Differential scattering cross section D    f   
2

 24
16m 2V02 a 6 64a 2  m 2V02 a 4 
 4   
 24   h4 
2
64a 2  mV0 a 2 
  
  2 

Q95. The Coulomb potential V  r   e 2 / r of a hydrogen atom is perturbed by adding

H   bx 2 (where b is a constant) to the Hamiltonian. The first order correction to the


ground state energy is
1
(The ground state wavefunction is  0  e  r / a0 )
a 3
0

(a) 2ba02 (b) ba02 (c) ba02 / 2 (d) 2ba02

Ans. : (b)
Solution: H '  bx 2 put x  r sin  cos 

H '  br 2 sin 2  cos 2  .


1
E11   1 H   1 ,  1  e  r / a0
a 3
0

H.No. 40‐D, Ground Floor, Jia Sarai, Near IIT, Hauz Khas, New Delhi‐110016 
Phone: 011‐26865455/+91‐9871145498
Website: www.physicsbyfiziks.com  | Email: fiziks.physics@gmail.com  
62 
fiziks
Institute for NET/JRF, GATE, IIT‐JAM, M.Sc. Entrance, JEST, TIFR and GRE in Physics 
 
  1 H  1r sin  dr d d
* 2

 
2r  2
b
 3  r 2 e a0 r 2 dr  sin 3  d  cos 2  d  ba02
 a0 0 0 0

Q96. Using the trial function


 A  a 2  x 2  , a  x  a
  x  
 0 otherwise

the ground state energy of a one-dimensional harmonic oscillator is

5 1 5
(a)  (b)  (c)  (d) 
14 2 7
Ans. : (b)
 A  a 2  x 2  , a  x  a
Solution:   x   
0 , otherwise

For normalization

  dx  1
*

15 15
A2  5
 A
16a 16a 5
 2 2   2 15
a a
T    *  dx   2  2    a 2  x 2 dx
5 
2m  a x 2
2m 16a 0

5 2
T 
4ma 2
a a
2 x 2  a 2  x 2  dx.
1 1 15
V    *V dx , where V  x   5 
2
m 2 x 2  m 2
a
2 2 16a 0

m 2 a 2
V 
14
5 2 m 2 a 2
E  T V  
4ma 2 14

dE 5   2   2 m 2 a 35   2 
0   0  a 4
  .
da 4ma 3 7 2  m 2 2 

H.No. 40‐D, Ground Floor, Jia Sarai, Near IIT, Hauz Khas, New Delhi‐110016 
Phone: 011‐26865455/+91‐9871145498
Website: www.physicsbyfiziks.com  | Email: fiziks.physics@gmail.com  
63 
fiziks
Institute for NET/JRF, GATE, IIT‐JAM, M.Sc. Entrance, JEST, TIFR and GRE in Physics 
 
1/ 2
 35    
a  
2
 .
 2   m 

5  2 m 2 m 2 35 
E  .  .
4 m  35 14 2 m

  5 2 1 35    5 5  5
         
2  2 35 7 2  2  14 14  14

H.No. 40‐D, Ground Floor, Jia Sarai, Near IIT, Hauz Khas, New Delhi‐110016 
Phone: 011‐26865455/+91‐9871145498
Website: www.physicsbyfiziks.com  | Email: fiziks.physics@gmail.com  
64 
fiziks
Institute for NET/JRF, GATE, IIT‐JAM, M.Sc. Entrance, JEST, TIFR and GRE in Physics 
 
THERMODYNAMICS AND STATISTICAL PHYSICS
NET/JRF (JUNE-2011)
Q1. Consider the transition of liquid water to steam as water boils at a temperature of 1000 C
under a pressure of 1 atmosphere. Which one of the following quantities does not change
discontinuously at the transition?
(a) The Gibbs free energy (b) The internal energy
(c) The entropy (d) The specific volume
Ans: (a)
Solution: In first order transition Gibbs free energy is continuous.
Q2. A particle is confined to the region x  0 by a potential which increases linearly as
u x   u 0 x . The mean position of the particle at temperature T is

k BT k BT
(b) k B T  / u 0
2
(a) (c) (d) u 0 k B T
u0 u0

Ans: (a)
p2 u x
1   0
Solution: Partition function Z   e 2 mkBT dp  e kBT dx and x   xp  x dxdpx
h
 0 x 2 
p2 u0 x   k BT 
 xe  te
k BT t
 
dx  u  dt
 xe dp  e
2 mk BT k BT
dx k BT
 x   0
 x  0 0



p2 u x
 0
  0  k BT  u0
e e
t
 e dt
k BT
2 mk BT
dp  e k BT
dx dx  u 
0 0 0

Q3. A cavity contains blackbody radiation in equilibrium at temperature T. The specific heat
per unit volume of the photon gas in the cavity is of the form CV  T 3 , where  is a
constant. The cavity is expanded to twice its original volume and then allowed to
equilibrate at the same temperature T. The new internal energy per unit volume is
T 4
(a) 4T 4
(b) 2T 4
(c) T 4
(d)
4
Ans: (d)
T 4
Solution: du  C v dT   T 3dT  u 
4

H.No. 40‐D, Ground Floor, Jia Sarai, Near IIT, Hauz Khas, New Delhi‐110016 
Phone: 011‐26865455/+91‐9871145498
Website: www.physicsbyfiziks.com  | Email: fiziks.physics@gmail.com  

fiziks
Institute for NET/JRF, GATE, IIT‐JAM, M.Sc. Entrance, JEST, TIFR and GRE in Physics 
 
Q4. Consider a system of N non-interacting spins, each of which has classical magnetic
moment of magnitude  . The Hamiltonian of this system in an external magnetic field
 N
  
H is   i .H , where  i is the magnetic moment of the i th spin. The magnetization per
i 1

spin at temperature T is

 2H   H  k B T 
(a) (b)  coth    
k BT   k B T  H 
 H   H 
(c)  sinh  (d)  tanh 
 k BT   k BT 
Ans: (b)
2 
 H cos 
   cos exp kT
sin  d d
Solution: For classical limit M  0 0
 H cos 
 exp kBT sin  d d
  H  k BT 
M   coth   
  k BT  H 
Q5. Consider an ideal Bose gas in three dimensions with the energy-momentum relation
  p s with s  0 . The range of s for which this system may undergo a Bose-Einstein
condensation at a non-zero temperature is
(a) 1  s  3 (b) 0  s  2 (c) 0  s  3 (d) 0  s  
Ans: (a)
NET/JRF (DEC-2011)
bS 3
Q6. The internal energy E of a system is given by E  , where b is a constant and other
VN
symbols have their usual meaning. The temperature of this system is equal to
2
bS 2 3bS 2 bS 3 S
(a) (b) (c) 2 (d)  
VN VN V N N
Ans: (b)
 E  3bS 2
Solution: TdS  dE  PdV  dE  TdS  PdV     T  T 
 S V VN

H.No. 40‐D, Ground Floor, Jia Sarai, Near IIT, Hauz Khas, New Delhi‐110016 
Phone: 011‐26865455/+91‐9871145498
Website: www.physicsbyfiziks.com  | Email: fiziks.physics@gmail.com  

fiziks
Institute for NET/JRF, GATE, IIT‐JAM, M.Sc. Entrance, JEST, TIFR and GRE in Physics 
 
Q7. Consider a Maxwellian distribution of the velocity of the molecules of an ideal gas. Let
Vmp and Vrms denote the most probable velocity and the root mean square velocity,

respectively. The magnitude of the ratio Vmp / Vrms is

(a) 1 (b) 2 / 3 (c) 2/3 (d) 3 / 2


Ans: (c)

2kT 3kT V 2
Solution: For Maxwellian distribution Vmp  , Vrms   mb 
m m Vrms 3
Q8. If the number density of a free electron gas in three dimensions is increased eight times,
its Fermi temperature will
(a) increase by a factor of 4 (b) decrease by a factor of 4
(c) increase by a factor of 8 (d) decrease by a factor of 8
Ans: (a)
2
 3N   2 3 N
Solution: Fermi energy E F    , where is number density and g is degeneracy
 4Vg  2m V
2 2

n n 
TF1 n1
EF  TF K  TF     TF  n  
3 2 3
3   1   4 since  8.
V  TF2  n2  n2
1
Q9. A system of N non-interacting spin - particles is placed in an external magnetic field H.
2
The behavior of the entropy of the system as a function of energy is given by
(a) S (b) S

E 
 BH BH  B H E  B H
S
(c) S (d)

 B H E BH  B H E

Ans: (a)

H.No. 40‐D, Ground Floor, Jia Sarai, Near IIT, Hauz Khas, New Delhi‐110016 
Phone: 011‐26865455/+91‐9871145498
Website: www.physicsbyfiziks.com  | Email: fiziks.physics@gmail.com  

fiziks
Institute for NET/JRF, GATE, IIT‐JAM, M.Sc. Entrance, JEST, TIFR and GRE in Physics 
 
S  N  U  N  U  N  U  N  U 
Solution:  ln   ln   , where   H . S is symmetrical
Nk 2N  2  2 N  2N 
about E .
Q10. A gas of N non-interacting particles is in thermal equilibrium at temperature T . Each
particle can be in any of the possible non-degenerate states of energy 0, 2 and 4 . The
average energy per particle of the gas, when   1 , is
(a) 2 (b) 3 (c) 2 / 3 (d) 
Ans: (a)
0  e  o  2 e2  4 e4
Solution: E1  0, E 2  2 , E 3  4 , Z  e 0   e 2   e 4   E 
e 0   e 2  e 4 1

2e 2   4e 4  2 1  2....  4 1   4  .... 2  4 6


 E      2
1  e  2   e  4  1  1  2....  1  4..... 111 3

where   1 .
Q11. A one-dimensional chain consists of a set of N rods each of length a . When stretched
by a load, each rod can align either parallel or perpendicular to the length of the chain.
The energy of a rod is  when perpendicular to it. When the chain is in thermal
equilibrium at temperature T , its average length is
(a) Na / 2 (b) Na

(c) Na / 1  e 2 / k BT  
(d) Na 1  e 2 / k BT 
Ans: (c)
Solution: Let n1 no. of rods are parallel and n2 no. of rods are perpendicular.
Energy of rod when it is perpendicular  
Energy of rod when it is parallel is  .
e      e e  
P     and P   
e      e   e  e  e   e  
n1ae   n2 ae  Nae  Na
Average length  n1aP     n2 aP       
   

e e e e 1  e 2 
Since P    P  so n2  N , n1  0 .

H.No. 40‐D, Ground Floor, Jia Sarai, Near IIT, Hauz Khas, New Delhi‐110016 
Phone: 011‐26865455/+91‐9871145498
Website: www.physicsbyfiziks.com  | Email: fiziks.physics@gmail.com  

fiziks
Institute for NET/JRF, GATE, IIT‐JAM, M.Sc. Entrance, JEST, TIFR and GRE in Physics 
 
Q12. The excitations of a three-dimensional solid are bosonic in nature with their frequency 
and wave-number k are related by   k 2 in the large wavelength limit. If the chemical
potential is zero, the behavior of the specific heat of the system at low temperature is
proportional to
(a) T 1/ 2 (b) T (c) T 3 / 2 (d) T 3
Ans: (c)
Solution: If dispersion relation is   k s ,
At low temperature specific heat  T 3/ s
Q13. Gas molecules of mass m are confined in a cylinder of radius R and height L
(with R  L ) kept vertically in the Earth’s gravitational field. The average energy of the
gas at low temperatures (such that mgL  k BT ) is given by

(a) Nk B T / 2 (b) 3Nk B T / 2 (c) 2 Nk B T (d) 5 Nk B T / 2


Ans: (d)
1 H
h3 
Solution: Z  e dpx dp y dpz dxdydz

  px2   p 2y   p z2 L mgz

Z e e e dpz  dx dy  e
2 mk BT 2 mk BT 2 mk BT k BT
dpx dp y dz
   0

 
mgL 
 
3 3
mgz
 mk T  1  e B
k T
 mk T  2 L  2
Z   R2  B 2   e k BT
dz  Z   R 2  B 2   
 2   0  2    mg 
 k T 
 B 
ZN  Z N ,

 ln z 5 Nk BT
 E  k BT 2  , since mgL  k BT
T 2

H.No. 40‐D, Ground Floor, Jia Sarai, Near IIT, Hauz Khas, New Delhi‐110016 
Phone: 011‐26865455/+91‐9871145498
Website: www.physicsbyfiziks.com  | Email: fiziks.physics@gmail.com  

fiziks
Institute for NET/JRF, GATE, IIT‐JAM, M.Sc. Entrance, JEST, TIFR and GRE in Physics 
 
NET/JRF (JUNE-2012)

Q14. Consider a system of non-interacting particles in d dimensional obeying the dispersion


relation   Ak s , where  is the energy, k is the wave vector; s is an integer and A is
constant. The density of states, N    , is proportional to
s d d s
1 1 1 1
(a)  d
(b)  s
(c)  s
(d)  d

Ans: (b)
Solution: We can solve this problem with intuition for example   Ak 2
1 3
1
Density of state in 3-dimensional N(ε)    
2 2

2
1
Density of state in 2-dimensional N(ε)    
0 2

1 1
1
Density of state in 1-dimensional N(ε)    
2 2

d
1
Density of state in d-dimensional, where   Ak  N     
s s

Q15. The number of ways in which N identical bosons can be distributed in two energy levels,
is
N  N  1 N  N  1
(a) N  1 (b) (c) (d) N
2 2
Ans: (a)
Solution: Number of boson  N , Number of energy level  g
N  g 1
So number of ways to distribute N boson into g level is, W  cN  N  1 since

g  2.
Q16. The free energy of the gas of N particles in a volume V and at a temperature T is

F  Nk B T ln a0V k B T 
5/ 2

/ N , where a 0 is a constant and k B denotes the Boltzmann
constant. The internal energy of the gas is
3 5
(a) Nk B T (b) Nk B T
2 2


(c) Nk B T ln a 0V k B T 
5/ 2

/N 
3
2
Nk B T 
(d) Nk B T ln a0V / k BT 
5/ 2

Ans: (b)

H.No. 40‐D, Ground Floor, Jia Sarai, Near IIT, Hauz Khas, New Delhi‐110016 
Phone: 011‐26865455/+91‐9871145498
Website: www.physicsbyfiziks.com  | Email: fiziks.physics@gmail.com  

fiziks
Institute for NET/JRF, GATE, IIT‐JAM, M.Sc. Entrance, JEST, TIFR and GRE in Physics 
 

Solution: F  Nk B T ln a0V k B T  / N , F  U  TS , U  F  TS
5/ 2

 F   F   F 
dF   SdT  PdV      S or S     U  F T 
 T V  T V  T V

F  Nk B T ln C T  a 0Vk B5 / 2
5/ 2
where C 
N
 F   F 
   Nk B ln CT
5/ 2

 Nk B T 
C 5 3/ 2
T  T   Nk B T ln CT
5/ 2 5
 Nk B T  
 T V CT  T V
5/ 2
2 2

 F  5  F  5
T   F  Nk B T  U  F  T     Nk B T .
 T V 2  T V 2

Q17. A system has two normal modes of vibration, with frequencies 1 and  2  21 . What

is the probability that at temperature T , the system has an energy less than 41 ?

[In the following x  e   1 and Z is the partition function of the system.]
(a) x 3 / 2 x  2 x 2  / Z (b) x 3 / 2 1  x  x 2  / Z

(c) x 3 / 2 1  2 x 2  / Z (d) x 3 / 2 1  x  2 x 2  / Z
Ans: (d)
Solution: There is two normal mode so there is two degree of freedom.
 1  1
Energy of harmonic oscillator is E   n1  1   n2   2 .
 2  2
 1  1
E   n1  1   n2   21 where n1  0,1,2,3.... and n 2  0,1,2,3....
 2  2
31 51
Ground state energy E  , first excited state energy E  . Second excited state
2 2
71
energy E  which is doubly degenerate state so g  2 , other state have more
2
energy than 41 .
3  1 5  1 7  1

PE  41  
e

2
e

2
 2e

2

x
3
2
1  x  2 x 
2
where x  e   1 .
Z Z
Q18. Bose condensation occurs in liquid He 4 kept at ambient pressure at 2.17 K . At which

temperature will Bose condensation occur in He 4 in gaseous state, the density of which
is 1000 times smaller than that of liquid He 4 ? (Assume that it is a perfect Bose gas.)
(a) 2.17 mK (b) 21.7 mK (c) 21.7  K (d) 2.17  K
H.No. 40‐D, Ground Floor, Jia Sarai, Near IIT, Hauz Khas, New Delhi‐110016 
Phone: 011‐26865455/+91‐9871145498
Website: www.physicsbyfiziks.com  | Email: fiziks.physics@gmail.com  

fiziks
Institute for NET/JRF, GATE, IIT‐JAM, M.Sc. Entrance, JEST, TIFR and GRE in Physics 
 
Ans: (b)
2
 N 3
Solution: For bosons T   
V 
Q19. Consider black body radiation contained in a cavity whose walls are at temperature T .
The radiation is in equilibrium with the walls of the cavity. If the temperature of the walls
is increased to 2T and the radiation is allowed to come to equilibrium at the new
temperature, the entropy of the radiation increases by a factor of
(a) 2 (b) 4 (c) 8 (d) 16
Ans: (c)

 8 5 k B4T 4  F   32 5 k B4  3
Solution: For Black Body, energy is given by F  V , S     VT .
45 2C 3  T V  45 C 
3 3

 S  T 3 , If temperate increase from T to 2T then entropy will incase S to 8S .

NET/JRF (DEC-2012)

Q20. The entropy of a system,  S  , is related to the accessible phase space volume  by

S  k B ln E , N , V  where E , N and V are the energy, number of particles and volume
respectively. From this one can conclude that 
(a) does not change during evolution to equilibrium
(b) oscillates during evolution to equilibrium
(c) is a maximum at equilibrium
(d) is a minimum at equilibrium
Ans: (c)
Solution: Entropy is maximum at equilibrium.
Q21. Let W be the work done in a quasistatic reversible thermodynamic process. Which of
the following statements about W is correct?
(a) W is a perfect differential if the process is isothermal
(b) W is a perfect differential if the process is adiabatic
(c) W is always a perfect differential
(d) W cannot be a perfect differential
Ans: (b)

H.No. 40‐D, Ground Floor, Jia Sarai, Near IIT, Hauz Khas, New Delhi‐110016 
Phone: 011‐26865455/+91‐9871145498
Website: www.physicsbyfiziks.com  | Email: fiziks.physics@gmail.com  

fiziks
Institute for NET/JRF, GATE, IIT‐JAM, M.Sc. Entrance, JEST, TIFR and GRE in Physics 
 
Solution: Work done is perfect differential in adiabatic process.
Q22. The free energy difference between the superconducting and the normal states of a

material is given by F  f S  f N      where  is an order parameter and
2 4

2
 and  are constants s.t.   0 in Normal and   0 in the super conducting state,
while   0 always, minimum value of F is

2 2 3 2 5 2
(a)  (b)  (c)  (d) 
 2 2 2
Ans: (b)
 F 4
Solution: F        2   
2 4 3

2  2

2   2   0   
3 2


 2
 2 2
Putting the value, F      F 
 2 2 min
2
Q23. A given quantity of gas is taken from the state A  C reversibly, by two P
A
paths, A  C directly and A  B  C as shown in the figure.
During the process A  C the work done by the gas is 100 J and the heat
absorbed is 150 J . If during the process A  B  C the work done by the B C
gas is 30 J , the heat absorbed is V
(a) 20 J (b) 80 J (c) 220 J (d) 280 J
Ans: (b)
Solution: During path AC , dU  dQ  dW  150  100  50 J
Since, internal energy is point function, so dU will same in all path
In path ABC , dQ  dU  dW  50  30  80 J .
Q24. Consider a one-dimensional Ising model with N spins, at very low temperatures when
almost all spins are aligned parallel to each other. There will be a few spin flips with each
flip costing an energy 2 J . In a configuration with r spin flips, the energy of the system
is E   NJ  2rJ and the number of configuration is N C r ; r varies from 0 to N . The
partition function is

H.No. 40‐D, Ground Floor, Jia Sarai, Near IIT, Hauz Khas, New Delhi‐110016 
Phone: 011‐26865455/+91‐9871145498
Website: www.physicsbyfiziks.com  | Email: fiziks.physics@gmail.com  

fiziks
Institute for NET/JRF, GATE, IIT‐JAM, M.Sc. Entrance, JEST, TIFR and GRE in Physics 
 
N N N
 J   J   J 
(a)   (b) e  NJ / k BT
(c)  sinh  (d)  cosh 
 k BT   k B T   k B T 

Ans: (d)
Solution: Let us consider only three energy levels, E r  2 J  2rJ i.e. E 0  2 J , E1  0 and

E2  2J .

Q2 
Ce
2
0
  E0
 2C1e  E1  2C 2 e  E2   e  2J
 2e 0  e  2 J  

e J  e J  2


4 4
2
Cr
r 0

2
 e J  e J 
Q2     cosh J 2  cosh J 2  Q N  cosh  J  N .
 2 
NET/JRF (JUNE-2013)

Q25. Ten grams of ice at 00 C is added to a beaker containing 30 grams of water at 250 C .
What is the final temperature of the system when it comes to thermal equilibrium? (The
specific heat of water is 1 cal / gm / 0 C and latent heat of melting of ice is 80 cal / gm )

(a) 00 C (b) 7.50 C (c) 12.50 C (d) 1.250 C


Ans: (a)
Solution: The amount of heat required to melt the ice of mass 10 gm at 00 C is
Q  m  L  10  80  800Cal , where L is the latent heat of melting of ice and m is the

mass of the ice. The amount of heat available in water of mass 30 gm at 250 C is

Q  m  Cv  T  30 1 25  750Cal
Since the heat available is less than the heat required to melt the ice therefore ice will not
melt as a result the temperature of the system will be at 00 C only.
Q26. A vessel has two compartments of volume V1 and V2 , containing an ideal gas at pressures
p1 and p 2 , and temperatures T1 and T2 respectively. If the wall separating the
compartments is removed, the resulting equilibrium temperature will be
p1T1  p 2T2 V1T1  V2T2 p1V1  p 2V2
(d) T1T2 
1/ 2
(a) (b) (c)
p1  p 2 V1  V2  p1V1 / T1    p 2V2 / T2 

H.No. 40‐D, Ground Floor, Jia Sarai, Near IIT, Hauz Khas, New Delhi‐110016 
Phone: 011‐26865455/+91‐9871145498
Website: www.physicsbyfiziks.com  | Email: fiziks.physics@gmail.com  
10 
fiziks
Institute for NET/JRF, GATE, IIT‐JAM, M.Sc. Entrance, JEST, TIFR and GRE in Physics 
 
Ans: (c)
p1V1 p2V2
Solution: V  V1  V2 , n  n1  n2   , U1  U 2  U , n1CvT1  n2CvT2  nCvT ,
T1 T2

p1V1  p2V2
n1T1  n2T2  nT  T 
p1V1 p2V2

T1 T2

Q27. For temperature T1  T2 , the qualitative temperature dependence of the probability


distribution F v  of the speed v of a molecule in three dimensions is correctly
represented by the following figure:

T2
T1
F(v)

(a) T1 (b) F(v)


T2

v v

T1
T2 T1
F(v)

F(v)

(c) (d)
T2

Ans: (a)
v v
Solution: Area under the F  v  is conserve and the mean velocity shift towards right for higher
temperature.
Q28. A system of non-interacting spin- 1/ 2 charged particles are placed in an external magnetic
field. At low temperature T , the leading behavior of the excess energy above the ground
state energy, depends on T as: ( c is a constant)
(a) cT (b) cT 3 (c) e  c / T (d) c (is independent of T )
Ans: (c)
 kTB H  H
 B 
 H e  e kT 
Solution: U    B H tanh B    B H   B H BH 
kT  e kT  e  kT 
 
Excess energy from the ground level

H.No. 40‐D, Ground Floor, Jia Sarai, Near IIT, Hauz Khas, New Delhi‐110016 
Phone: 011‐26865455/+91‐9871145498
Website: www.physicsbyfiziks.com  | Email: fiziks.physics@gmail.com  
11 
fiziks
Institute for NET/JRF, GATE, IIT‐JAM, M.Sc. Entrance, JEST, TIFR and GRE in Physics 
 
 kTB H

B H
   B H  H
 B    H
 B 
 e e kT
   e kT
e kT
   2e kT 
   B H B H  (  B H )   B H 1  B H B H  
  B H B H
 B H    B H 
 e kT  e kT    e kT  e kT  

 e kT  e kT 
   
C
At low temperature, the lower value, U  e T
, where C   B H .

Q29. Consider a system of two Ising spins S1 and S 2 taking values  1 with interaction energy
given by    JS1 S 2 , when it is in thermal equilibrium at temperature T . For large T , the
average energy of the system varies as C / k B T , with C given by

(a)  2J 2 (b)  J 2 (c) J 2 (d) 4 J


Ans: (b)
Solution: The interaction energy is given by E   J S1 S 2 where S1 and S 2 taking values  1 .
Possible values of the Energy of the system are
E1   J 11   J , E 2   J  1 1   J

E3   J 1  1   J , E 4   J  1  1   J

Er
 J  J 
 Er g r e kT
2 Je
J
 2 Je

J
 kTJ
e  e

J
kT
  1  kT  1 
 kT  

 J  J   J 
kT kT
U  r
  J 
E
 r
J

J
 e kT  e  kT   J   J 
g er
r
kT
2e kT
 2e kT
  1 
 kT
 1 


 kT 
J2 J
 U   C   J 2 (For large T ,  1 )
kT kT
Q30. Consider two different systems each with three identical non-interacting particles. Both
have single particle states with energies  0 ,3 0 and 5 0 ,  0  0 . One system is populated

1
by spin  fermions and the other by bosons. What is the value of E F  E B where E F
2
and EB are the ground state energies of the fermionic and bosonic systems respectively?

(a) 6 0 (b) 2 0 (c) 4 0 (d)  0


Ans: (b)
Solution: Energy of Fermion = 2 1 0  3 0  5 0

Energy of boson = 3 1 0  3 0

E F  E B = 5 0  3 0  2 0

H.No. 40‐D, Ground Floor, Jia Sarai, Near IIT, Hauz Khas, New Delhi‐110016 
Phone: 011‐26865455/+91‐9871145498
Website: www.physicsbyfiziks.com  | Email: fiziks.physics@gmail.com  
12 
fiziks
Institute for NET/JRF, GATE, IIT‐JAM, M.Sc. Entrance, JEST, TIFR and GRE in Physics 
 
NET/JRF (DEC-2013)
1
Q31. Three identical spin- fermions are to be distributed in two non-degenerate distinct
2
energy levels. The number of ways this can be done is
(a) 8 (b) 4 (c) 3 (d) 2
Ans: (b)
Solution: Total number of degeneracy
g  (Number of energy state (n))  (Number of degeneracy due to spin ( 2 s  1 ))
1 1
n  2, s  , g  2  (2.  1)  4
2 2
Number of particle, N  3 . So number of ways, g cN  4 c3  4
Q32. Consider the melting transition of ice into water at constant pressure. Which of the
following thermodynamic quantities does not exhibit a discontinuous change across the
phase transition?
(a) Internal energy (b) Helmholtz free energy (c) Gibbs free energy (d) entropy
Ans: (c)
Solution: Ice to water: 1st order phase transition.
So Gibbs free energy is continuous, so it doesn’t exhibit discontinuous change.
Q33. Two different thermodynamic systems are described by the following equations of state:
1 3RN 1 1 5RN 2 
 and  where T 1, 2  , N 1, 2  and U 1, 2  are respectively, the
T 1 2U 1 T 2  2U 2 
temperatures, the mole numbers and the internal energies of the two systems, and R is
the gas constant. Let U tot denote the total energy when these two systems are put in

U 1
contact and attain thermal equilibrium. The ratio is
U tot

5 N 2  3N 1 N 1 N 2 
(a) (b) (c) (d)
3N 1  5 N 2  3N 1  5 N 2  N 1  N 2  N 1  N 2 
Ans: (b)
1 3RN 1 1 5RN 2 
Solution:  and 
T 1 2U 1 T 2  2U 2 

H.No. 40‐D, Ground Floor, Jia Sarai, Near IIT, Hauz Khas, New Delhi‐110016 
Phone: 011‐26865455/+91‐9871145498
Website: www.physicsbyfiziks.com  | Email: fiziks.physics@gmail.com  
13 
fiziks
Institute for NET/JRF, GATE, IIT‐JAM, M.Sc. Entrance, JEST, TIFR and GRE in Physics 
 
3 5
Now U tot  U (1)  U  2  RN 1T 1  RN  2T  2
2 2
1
3RN 1T 1 
U 1 2  3 N 1T 1
  
U tot 1 3N 1T 1  5 RN  2T  2  3N 1T 1  5 N  2T  2
2 
3N 1
At thermal equilibrium T 1  T  2 , thus
3N 1  5 N 2 
Q34. The speed v of the molecules of mass m of an ideal gas obeys Maxwell’s velocity
distribution law at an equilibrium temperature T . Let  vx , v y , vz  denote the components

 
2
of the velocity and k B the Boltzmann constant. The average value of  vx   v y , where

 and  are constants, is


(a)  2   2 k B T / m (b)  2   2 k B T / m

(c)     k B T / m (d)     k B T / m
2 2

Ans: (b)
Solution: Ideal gas obeys Maxwell velocity distribution law at equilibrium temperature. Then

 
2
average value of  vx   v y

 v 
2
Now x  vy   2 v x2   2 v y2  2  v x v y

k BT
v x  0, v y  0 and vx2   v y2  vz2
m

 v 
2
Then x  v y   2 v x2   2 v y2  2 v x v y

 v x  v y 
2
 2
k BT
m
k T
 2 B   2   2
m
kT
m

Q35. The entropy S of a thermodynamic system as a function of energy S
C
E is given by the following graph. The temperatures of the phases B
A
A, B nd C denoted by T A , TB and TC respectively.
E
Satisfy the following inequalities:
(a) TC  TB  T A (b) T A  TC  TB (c) TB  TC  T A (d) TB  T A  TC
Ans: (c)
H.No. 40‐D, Ground Floor, Jia Sarai, Near IIT, Hauz Khas, New Delhi‐110016 
Phone: 011‐26865455/+91‐9871145498
Website: www.physicsbyfiziks.com  | Email: fiziks.physics@gmail.com  
14 
fiziks
Institute for NET/JRF, GATE, IIT‐JAM, M.Sc. Entrance, JEST, TIFR and GRE in Physics 
 
Solution: Temperatures of phase are: TA , TB , TC

 dS  1 S
Since,   C
 dE  T B
dS A
Hence, will be slope, then it will be zero for B - phase
dE E
So TB   and in C and A phases, internal energy of C phase is more, so TC  T A

Now TB  TC  T A

Q36. A system of N classical non-interacting particles, each of mass m , is at a temperature T


and is confined by the external potential V r   Ar 2 (where A is a constant) in three
1
2
dimensions. The internal energy of the system is
A  k BT 
(c) N 2mA k B T
3
ln 
3/ 2
(a) 3 Nk B T (b) Nk B T (d) N
2 m  m 
Ans: (a)

Solution: V r  
1 2 1
2
 
Ar  A x 2  y 2  z 2 it is harmonic oscillator.
2
3N
1  kT 
So its partition function will be z N   
N   
 ln Z N
Internal energy, U  kT 2  3 NkT
T
Q37. A Carnot cycle operates as a heat engine between two bodies of equal heat capacity until
their temperatures become equal. If the initial temperatures of the bodies are T1 and T2 ,
respectively and T1  T2 , then their common final temperature is

(a) T12 / T2 (b) T22 / T1 (c) T1T2 (d)


1
T1  T2 
2
Ans: (c)
Solution: For heat Carnot engine the change in entropy for source and sink
TF dT  T TF dT T 
dS1   and dS 2  
 log  F
  log  F 
T1 T   T1 T2 T
 T2 
T T
S  dS1  dS 2  log F  log F .
T1 T2
T 
2

Since, Carnot engine is reversible in nature, so log F  0  TF  T1T2


T1T2

H.No. 40‐D, Ground Floor, Jia Sarai, Near IIT, Hauz Khas, New Delhi‐110016 
Phone: 011‐26865455/+91‐9871145498
Website: www.physicsbyfiziks.com  | Email: fiziks.physics@gmail.com  
15 
fiziks
Institute for NET/JRF, GATE, IIT‐JAM, M.Sc. Entrance, JEST, TIFR and GRE in Physics 
 
NET/JRF (JUNE-2014)
Q38. Which of the graphs below gives the correct qualitative behaviour of the energy density
E r   of blackbody radiation of wavelength  at two temperatures T1 and T2 T1  T2  ?
(a) (b)
T2
T2
Er  
Er  
T1

 
(c) (d)
T2
T2
Er   Er  
T1
T1

Ans: (c)  
Q39. A system can have three energy levels: E  0,   . The level E  0 is doubly degenerate,
while the others are non-degenerate. The average energy at inverse temperature  is

 e   e      
(a)   tanh   (d)   tanh  
(b)
1  e 
e  
 (c) 0
 2 
Ans: (d)
Solution: E  0,   , E  0 doubly degenerate

z   gi e   Ei  2  e   0  e    e 


z  2  e   e   ln z  ln 2  e   e  
   
Now E  

ln  z   


ln 2  e   e     
1

2 e  e

 
   e    e    

      

   
  e 2
 e 2

 e e
E   

         tanh   
 2    
  
  2 
  e 2  e 2     e 2
 e 2
 
  
   
  

H.No. 40‐D, Ground Floor, Jia Sarai, Near IIT, Hauz Khas, New Delhi‐110016 
Phone: 011‐26865455/+91‐9871145498
Website: www.physicsbyfiziks.com  | Email: fiziks.physics@gmail.com  
16 
fiziks
Institute for NET/JRF, GATE, IIT‐JAM, M.Sc. Entrance, JEST, TIFR and GRE in Physics 
 
Q40. The free energy F of a system depends on a thermodynamic variable  as

F  a 2  b 6
with a, b  0 . The value of  , when the system is in thermodynamic equilibrium, is

(b)  a / 6b  (c)  a / 3b  (d)  a / b 


1/ 4 1/ 4 1/ 4
(a) zero
Ans: (c)
Solution: Frequency F  a 2  b 6 , a, b  0

2 F F
F is equilibrium i.e.  0 , now  2a  6b 5
 2


F
1/ 4
a  a 
 0  2a  6b 5   4    
 3b  3b 
Q41. For a particular thermodynamic system the entropy S is related to the internal energy U
and volume V by
S  cU 3 / 4V 1 / 4
where c is a constant. The Gibbs potential G  U  TS  PV for this system is
3PU cU US
(a) (b) (c) zero (d)
4T 3 4V
Ans: (c)
Solution: S  cU 3/ 4V 1/ 4 , dU  TdS  PdV

 S  1  S  1 c  3 1/ 4 1/ 4 4 U 1/ 4
        U V  T 
 U V T  U V T 4 3c V 1/ 4
1

 U  S V 5 / 4U 4 S V 5 / 4 1/ 4
    P    P  U
 V  S c 3 c 3

4 U 1/ 4 S V 5/ 4 1/4 4 1
G U   cU V 
3/4 1/4
U V  U  U  U  0
3c V 1/ 4
c 3 3 3
Q42. The pressure of a non-relativistic free Fermi gas in three-dimensions depends, at T  0 ,
on the density of fermions n as
(a) n 5 / 3 (b) n1 / 3 (c) n 2 / 3 (d) n 4 / 3
Ans: (a)

H.No. 40‐D, Ground Floor, Jia Sarai, Near IIT, Hauz Khas, New Delhi‐110016 
Phone: 011‐26865455/+91‐9871145498
Website: www.physicsbyfiziks.com  | Email: fiziks.physics@gmail.com  
17 
fiziks
Institute for NET/JRF, GATE, IIT‐JAM, M.Sc. Entrance, JEST, TIFR and GRE in Physics 
 
2
Solution: Pressure P  nEF , EF  n 2 / 3 , at T  0
3
2 2
P  n  n 2 / 3  n 5 / 3  P  n5 / 3
3 3
Q43. The vander Waals’ equation of state for a gas is given by
 a 
 P  2 V  b   RT
 V 
where P, V and T represent the pressure, volume and temperature respectively, and a
and b are constant parameters. At the critical point, where all the roots of the above
cubic equation are degenerate, the volume is given by
a a 8a
(a) (b) (c) (d) 3b
9b 27b 2 27bR
Ans: (d)

 a   P   2 P 
Solution:  P  2  V  b   RT , for critical volume    0,  2   0
 V   V   V 
a ab
PV   Pb  2  RT
V V
P a 2ab 2 P 2a 6ab 2a 6ab
 0  P  2  3  0,  0  3  4  0  3  4  Vc  3b
V V V V 2
V V V V

NET/JRF (DEC-2014)
Q44. The pressure P of a fluid is related to its number density  by the equation of state
P  a  b 2
where a and b are constants. If the initial volume of the fluid is V0 , the work done on
the system when it is compressed, so as to increase the number density from an initial
value of  0 to 2  0 is

(a) a 0V0 (b) a  b 0  0V0

 3a 7  b 
(c)   0   0V0 (d) a ln 2  b 0  0V0
 2 3 
Ans: (d)
n n2 n
Solution: P  a  b  P  a  b 2
2
 
V V V

H.No. 40‐D, Ground Floor, Jia Sarai, Near IIT, Hauz Khas, New Delhi‐110016 
Phone: 011‐26865455/+91‐9871145498
Website: www.physicsbyfiziks.com  | Email: fiziks.physics@gmail.com  
18 
fiziks
Institute for NET/JRF, GATE, IIT‐JAM, M.Sc. Entrance, JEST, TIFR and GRE in Physics 
 
V2 dV V2 dV n n
W   P  dV  an  bn 2  , where V1  , V2 
V1 V V1 V 2 0 2 0

 W   n  a ln 2  b 0    0V0  a ln 2  b0  ,  n  0V0

Work done on the system  W   a ln 2  b0  0V0

Q45. An ideal Bose gas is confined inside a container that is connected to a particle reservoir.
Each particle can occupy a discrete set of single-particle quantum states. If the probability
that a particular quantum state is unoccupied is 0.1 , then the average number of bosons
in that state is
(a) 8 (b) 9 (c) 10 (d) 11
Ans: (b)
Q46. In low density oxygen gas at low temperature, only the translational and rotational modes
of the molecules are excited. The specific heat per molecule of the gas is
1 3 5
(a) kB (b) k B (c) kB (d) kB
2 2 2
Ans: (d)
Solution: Total D.O.F. = 3 transition + 2 rotation i.e. f  5
k B T 5k B T U 5
U f   CV   kB
2 2 T 2
Q47. When a gas expands adiabatically from volume V1 to V2 by a quasi-static reversible
process, it cools from temperature T1 to T2 . If now the same process is carried out
adiabatically and irreversibly, and T2 is the temperature of the gas when it has
equilibrated, then
V V  T2V1
(a) T2  T2 (b) T2  T2 (c) T2  T2  2 1  (d) T2 
 V2  V2

Ans: (b)
Q48. A random walker takes a step of unit length in the positive direction with probability 2 / 3
and a step of unit length in the negative direction with probability 1 / 3 . The mean
displacement of the walker after n steps is
(a) n / 3 (b) n / 8 (c) 2n / 3 (d) 0
Ans: (a)
H.No. 40‐D, Ground Floor, Jia Sarai, Near IIT, Hauz Khas, New Delhi‐110016 
Phone: 011‐26865455/+91‐9871145498
Website: www.physicsbyfiziks.com  | Email: fiziks.physics@gmail.com  
19 
fiziks
Institute for NET/JRF, GATE, IIT‐JAM, M.Sc. Entrance, JEST, TIFR and GRE in Physics 
 
2 1
Solution: P  1   P  1 
3 3
2 1 1 n
For one step  1    , for n step 
3 3 3 3
Q49. A collection N of non-interacting spins S i , i  1, 2, ....., N , S i  1 is kept in an
external magnetic field B at a temperature T . The Hamiltonian of the system is
B
H   B i S i . What should be the minimum value of for which the mean value
k BT
1
Si  ?
3
1 1
(a) N ln 2 (b) 2 ln 2 (c) ln 2 (d) N ln 2
2 2
Ans: (c)
B B

e kT
e kT
Solution: P  Si  1  B B
, P  Si  1  B B
 
e kT
e kT e kT
 e kT
B B
 
1e kT
e kT
 B 
Si   Si    tanh 

B B
 kT 
e kT
 e kT
B
For N particle Si   N tanh
kT
Si 1  B  1 B 1
According to question,    tanh     ln 2
N 3  kT  3 kT 2
NET/JRF (JUNE-2015)
Q50. A system of N non-interacting classical particles, each of mass m is in a two
dimensional harmonic potential of the form V  r     x 2  y 2  where  is a positive

 1 
constant. The canonical partition function of the system at temperature T is    :
 k BT 
N N
   2    2m 
2N
  
N
 2m 2 
(a)    (b)   (c)   (d)  2 
 2m        2m     
Ans. (d)

H.No. 40‐D, Ground Floor, Jia Sarai, Near IIT, Hauz Khas, New Delhi‐110016 
Phone: 011‐26865455/+91‐9871145498
Website: www.physicsbyfiziks.com  | Email: fiziks.physics@gmail.com  
20 
fiziks
Institute for NET/JRF, GATE, IIT‐JAM, M.Sc. Entrance, JEST, TIFR and GRE in Physics 
 
Solution: V  r    x  y
2

2

 px2  p 2y  x 2  y 2
1    
z1  2
h 

e 2 mkT dpx  e 2 mkT dp y  e
 
kT
dx  e

kT
dy

2 mkT 2 mkT 1 1 1 
 z1  2  2
h2 h 2
2  2 
kT kT
N
 2 m 
2
 2 m  2
z1   2   kT   z N   2 2 
2

 h   h  
Q51. A system of N distinguishable particles, each of which can be in one of the two energy
levels 0 and  , has a total energy n  , where n is an integer. The entropy of the system
is proportional to

 N !  N! 
(a) N ln n (b) n ln N (d) ln 
 n ! N  n  ! 
(c) ln  
 n!   
Ans. (d)
Solution: No of ways for above configuration is  N Cn

N N
W   Entropy=k ln
n N n n N n
Q52. The condition for the liquid and vapour phases of a fluid to be in equilibrium is given by
dP Q
the approximate equation  1 (Clausius-Clayperon equation) where vvap is the
dT Tvvap

volume per particle in the vapour phase, and Q1 is the latent heat, which may be taken to
be a constant. If the vapour obeys ideal gas law, which of the following plots is correct?

(a) ln P (b) ln P (c) ln P (d) ln P

O O O T O T
T T

Ans. (c)
dP Q RT dP Ql P dP Ql dT C
Solution:  l ,
dT Tvap
vap 
P
 
dT RT 2

P

R T 2
 ln P  
T


H.No. 40‐D, Ground Floor, Jia Sarai, Near IIT, Hauz Khas, New Delhi‐110016 
Phone: 011‐26865455/+91‐9871145498
Website: www.physicsbyfiziks.com  | Email: fiziks.physics@gmail.com  
21 
fiziks
Institute for NET/JRF, GATE, IIT‐JAM, M.Sc. Entrance, JEST, TIFR and GRE in Physics 
 
Q53. Consider three Ising spins at the vertices of a triangle which interact with each other with
a ferromagnetic Ising interaction of strength J . The partition function of the system at
 1 
temperature T is given by    :
 k BT 
(a) 2e3 J  6e   J (b) 2e 3 J  6e  J

(c) 2e3 J  6e 3 J  3e  J  3e   J (d)  2 cosh  J 


3

Ans. (b)
Solution: H  J  S1S 2  S1S3  S 2 S3 

S1 S2 S3 E
1 1 1 3J

1 1
1

1 1 1   J
1 1 1 

1 11

1 1 1  J
1 1 1

1 1 1 3J

 z  2e 3 J  6e  J
Q54. A large number N of Brownian particles in one dimension start their diffusive motion
from the origin at time t  0 . The diffusion coefficient is D . The number of particles
crossing a point at a distance L from the origin, per unit time, depends on L and time t
as
 L2 4 Dt  L2 4 Dt
N NL N
(a) e  4 Dt  (b) e L2
(c) e  Dt 
4
(d) Ne L2
4 Dt 4 Dt 16 Dt 3
Ans. (a)
Solution: From Einstein Smoluchowski theory

dx   x2 
p  x  dx  exp  
4 Dt  4 Dt 

N   L2 
Number of particle passing from point L at origin  .exp  
4 Dt  4 Dt 
H.No. 40‐D, Ground Floor, Jia Sarai, Near IIT, Hauz Khas, New Delhi‐110016 
Phone: 011‐26865455/+91‐9871145498
Website: www.physicsbyfiziks.com  | Email: fiziks.physics@gmail.com  
22 
fiziks
Institute for NET/JRF, GATE, IIT‐JAM, M.Sc. Entrance, JEST, TIFR and GRE in Physics 
 

Q55. An ideal Bose gas in d -dimensions obeys the dispersion relation  k  Ak s , where A 
and s are constants. For Bose-Einstein condensation to occur, the occupancy of excited
states
d s

 s
Ne  c  d
0 e 
   
1 
where c is a constant, should remain finite even for   0 . This can happen if
d 1 1 d 1 d 1 d
(a)  (b)   (c) 1 (d)  1
s 4 4 s 2 s 2 s
Ans. (c)
d s
  s
Solution: Ne  c  d
0
e      1
B.E. condensation is possible in 3-D
1
d s 1 d 3
For materlistic particle g  2    
s 2 s 2
d s d
For massless particle g  2  2 3
s s
d
In both cases 1
s
NET/JRF (DEC-2015)
Q56. The heat capacity of the interior of a refrigerator is 4.2 kJ / K . The minimum work that

must be done to lower the internal temperature from 18o C to 17o C , when the outside

temperature is 27o C will be


(a) 2.20 kJ (b) 0.80 kJ (c) 0.30 kJ (d) 0.14 kJ
Ans.: (b)
Q57. For a system of independent non interacting one-dimensional oscillators, the value of the
free energy per oscillator, in the limit T  0 , is
1 3
(a)  (b)  (c)  (d) 0
2 2
Ans.: (a)

H.No. 40‐D, Ground Floor, Jia Sarai, Near IIT, Hauz Khas, New Delhi‐110016 
Phone: 011‐26865455/+91‐9871145498
Website: www.physicsbyfiziks.com  | Email: fiziks.physics@gmail.com  
23 
fiziks
Institute for NET/JRF, GATE, IIT‐JAM, M.Sc. Entrance, JEST, TIFR and GRE in Physics 
 
N
       
Solution: For the given system Z N   2sinh   F  kT ln Z N  NkT ln  2sinh  
 2kT    2kT  
   


 
2 e 2 kT
 e 2 kT

  NkT ln  e 2 kT 1  e  kT    NkT ln e 2 kT  NkT ln 1  e  kT 
   
 NkT ln   
       

2
      
 

F      
  kT ln 1  e kT   0   kT  0 
N 2   2 2

Q58. The partition function of a system of N Ising spins is Z  1N  2N where 1 and 2 are

functions of temperature, but are independent of N . If 1  2 , the free energy per spin

in the limit N   is
 
(a)  k BT ln  1  (b)  k BT ln 2 (c)  k BT ln  12  (d)  k BT ln 1
 2 
Ans.: (d)

 
Solution: Z  1N  2N , F   kT ln 1N  2N , it is given 1  2

     N  2
 F  kT ln 1N  1   2    , 0
   1    1
 
F
F   kT ln 1N   NkT ln 1    kT ln 1
N
1
Q59. The Hamiltonian of a system of N non interacting spin particles is H   0 B  Siz ,
2 i

where Siz  1 are components of i th spin along an external magnetic field B . At a


0 B

temperature T such that e kBT  2 . the specific heat per particle is


16 8 16
(c) k B  ln 2  k B  ln 2 
2 2
(a) kB (b) k B ln 2 (d)
25 25 25
Ans.: (d)
0 B
Solution: For the given system E   0 B tanh
kT

 B B
2

CV   0  Nk sec2 h 0
 kT  kT
H.No. 40‐D, Ground Floor, Jia Sarai, Near IIT, Hauz Khas, New Delhi‐110016 
Phone: 011‐26865455/+91‐9871145498
Website: www.physicsbyfiziks.com  | Email: fiziks.physics@gmail.com  
24 
fiziks
Institute for NET/JRF, GATE, IIT‐JAM, M.Sc. Entrance, JEST, TIFR and GRE in Physics 
 
0 B
CV  0 B  0 B
2
4
  k e kT  2   ln 2
N  kT   0 B  B
 0 
2
kT
 e kT
 e kT

 

CV  0 B    B  16
2 2
4 2 16k 16
  ln 2  k  ln 2 
2
  k  0  k 
N  kT   1
2
 kT  25 25 25
 2  
 2
1
Q60. An ensemble of non-interacting spin - particles is in contact with a heat bath at
2
temperature T and is subjected to an external magnetic field. Each particle can be in one
of the two quantum states of energies  0 . If the mean energy per particle is  0 / 2 ,
then the free energy per particle is

(a) 2 0

ln 4 / 3  (b)  0 ln  3 / 2  (c) 2 0 ln 2 (d)  0
ln 2
ln 3 ln 3
Ans.: (a)
0
Solution: For the given system, partition function, Z n  2 N cosh
kT
   
Mean energy per unit particle    0    0 tanh 0
 2 kT
 
 1 e e 1
put 0    tanh     

kT 2 e e 2
1  1 2 0
 e 2  3    ln 3  0  ln 3  kT 
2 kT 2 ln 3
F   
  kT ln  2 cosh 0    kT ln  2 cosh   b  4ac
2
It is given,
N  kT 
 e  e  
  kT ln  2  2
 
   kT ln e e  1   kT ln  e  3  1    kT ln e 4 
 

 2 
0 2
  kT ln e   ln 4   kT   kT ln 4  kT  kT ln 4  0  0 ln 4
kT ln 3
  3    16  
 ln 3  2 ln 4   ln  16    ln  3  
0   0       0   
 ln 3   ln 3   ln 3 
   

H.No. 40‐D, Ground Floor, Jia Sarai, Near IIT, Hauz Khas, New Delhi‐110016 
Phone: 011‐26865455/+91‐9871145498
Website: www.physicsbyfiziks.com  | Email: fiziks.physics@gmail.com  
25 
fiziks
Institute for NET/JRF, GATE, IIT‐JAM, M.Sc. Entrance, JEST, TIFR and GRE in Physics 
 
  4   2
 4   4 
 ln    ln   ln  
F
   0  
3 
 2 0 
3
 2 0 
3
N  ln 3  ln 3 ln 3
 
 
Q61. Which of the following graphs shows the qualitative dependence of the free energy
f  h, T  of a ferromagnet in an external magnetic field h , and at a fixed temperature

T  TC , where TC is the critical temperature?


f f f f
h
(a) (b) (c) (d)
h h h
Ans.: (c)
Solution: For super conductor state one will find two local minima
f
Option (c) is correct.
h

NET/JRF (JUNE-2016)
Q62. The specific heat per molecule of a gas of diatomic molecules at high temperatures is
(a) 8k B (b) 3.5k B (c) 4.5 k B (d) 3k B
Ans: (b)
Solution: For high temperature all number are excited so degree of freedom for diatomic
molecule is 7 .
fk BT 7k T  U 
Internal energy is , U  B , CV     3.5k B
2 2  T V
Q63. When an ideal monoatomic gas is expanded adiabatically from an initial volume V0 to
T
3V0 , its temperature changes from T0 to T . Then the ratio is
T0
2 1
1  1 3  1 3
(a) (b)   (c)   (d) 3
3 3 3
Ans: (b)
Solution: For adiabatic process PV   k , T0V0 1  k

H.No. 40‐D, Ground Floor, Jia Sarai, Near IIT, Hauz Khas, New Delhi‐110016 
Phone: 011‐26865455/+91‐9871145498
Website: www.physicsbyfiziks.com  | Email: fiziks.physics@gmail.com  
26 
fiziks
Institute for NET/JRF, GATE, IIT‐JAM, M.Sc. Entrance, JEST, TIFR and GRE in Physics 
 
 1 v 1
V  1
 T  3V0 
 1  1
T0V0  T  T0  0   T  T0  
 3V0  3
5
For monoatomic gas  
3
5 2
1 2/3
 1 3  1 3 T 1
T  T0    T0      
3 3 T0  3 
Q64. A box of volume V containing N molecules of an ideal gas, is divided by a wall with a
V
hole into two compartments. If the volume of the smaller compartment is , the
3
variance of the number of particles in it, is

N 2N N
(a) (b) (c) N (d)
3 9 3
Ans: (b)
V 1
Solution: Probability that one particle is in smaller compartment having volume , so p 
3 3
There are only two options either particle is in left half or right half, so for one particle
distribution is Bernoulli for Bernoulli’s distribution  2  p 1  p  . For N particle

distribution is
1  1 2N
 2  Np 1  p   N  1   ,  2 
3  3 9
Q65. A gas of non-relativistic classical particles in one dimension is subjected to a potential
 1 
V  x    x (where  is a constant). The partition function is    
 k BT 
4m 2m 8m 3m
(a) (b) (c) (d)
 h
3 2 2
 h
3 2 2
 h
3 2 2
 3 2 h 2
Ans: (c)
 p2  x  x
1  x  1 
Solution: z   e 2 mkT dpx  e kT dx   2 mkT   e kT dx
1/ 2

h  
h 

1/ 2  x
 2 mkT  
 z 
 h
2 

e

kT
dx

H.No. 40‐D, Ground Floor, Jia Sarai, Near IIT, Hauz Khas, New Delhi‐110016 
Phone: 011‐26865455/+91‐9871145498
Website: www.physicsbyfiziks.com  | Email: fiziks.physics@gmail.com  
27 
fiziks
Institute for NET/JRF, GATE, IIT‐JAM, M.Sc. Entrance, JEST, TIFR and GRE in Physics 
 
 x 0 x  x
   kT kT 2kT
e kT
dx  e kT
dx   e kT
dx   
  0
  
1
 2 mkT   2kT   8 m  2
1/ 2
1
z      2 3 2  , put  
 h     h    kT
2

Q66. The internal energy E T  of a system at a fixed volume is found to depend on the

temperature T as E T   aT 2  bT 4 . Then the entropy S T  , as a function of temperature,

is
1 2 1 4 4
(a) aT  bT (b) 2aT 2  4bT 4 (c) 2aT  bT 3 (d) 2aT  2bT 3
2 4 3
Ans: (c)
Solution: From first law of thermodynamics,
TdS  dE  PdV , dE  TdS  PdV , it is given dV  0
1
dE  TdS  dS  dE
T
E  aT 2  bT 4  dE  2aTdT  4bT 3 dT
4bT 3
dS 
1
T
 
2aTdT  4bT 3dT  2adT  4bT 2 dT  2aT 
3
Q67. Consider a gas of Cs atoms at a number density of 1012 atoms/cc. when the typical inter-
particle distance is equal to the thermal de Broglie wavelength of the particles, the
temperature of the gas is nearest to (Take the mass of a Cs atom to be 22.7 1026 kg )

(a) 1 109 K (b) 7  105 K (c) 1 103 K (d) 2 108 K


Ans: (d)
Solution: When de Broglie wavelength = thermal wavelength
g3 / 2  z   2.61
2/3
N h3 N h2
 2 mkT  3/ 2
  2 mkT   
V 2.61 V   2.612 / 3

   
2/3 2
1 h2 1 1012 6.6  1034
T n 2/3

2 mk  2.612 / 3 2  3.14  22.7 1026 1.38 1023   2.612 / 3

H.No. 40‐D, Ground Floor, Jia Sarai, Near IIT, Hauz Khas, New Delhi‐110016 
Phone: 011‐26865455/+91‐9871145498
Website: www.physicsbyfiziks.com  | Email: fiziks.physics@gmail.com  
28 
fiziks
Institute for NET/JRF, GATE, IIT‐JAM, M.Sc. Entrance, JEST, TIFR and GRE in Physics 
 
 6.6  108 1064  1049
2
 6.6 2  107
 
6.28  22.7  1.38   2.61 6.28  22.7  1.38   2.61
2/3 2/3

0.221 107 0.221


   107  0.116  107  1.16 108
 2.61 2/3
1.895

NET/JRF (DEC-2016)
Q68. The partition function of a two-level system governed by the Hamiltonian
  
H 
 
is
 

(a) 2sinh   2   2
(b) 2 cosh       2 2

(c) cosh        sinh    2   2  


1 2 2

2 
(d) cosh        sinh   
1 2 2
 2  2
2 
Ans. : (b)
  
Solution: H  
   

The eigen value is given by E1    2   2 and E2    2   2

Z  trace  e   H   e   E1  e   E2  e    2  2
e

    2  2   2 cosh   2  2

Q69. Consider a gas of N classical particles in a two-dimensional square box of side L . If the
total energy of the gas is E , the entropy (apart from an additive constant) is

 L2 E   LE  L E  E
(a) Nk B ln   (b) Nk B ln   (c) 2 Nk B ln   (d) L2 k B ln  
 N   N   N  N

Ans. : (c)
N
1  2 mkTL2  E
Solution: Z N    kT 
N  h2  N
N
1  L2 E  2 m
ZN    Assume 1
N N  h2

H.No. 40‐D, Ground Floor, Jia Sarai, Near IIT, Hauz Khas, New Delhi‐110016 
Phone: 011‐26865455/+91‐9871145498
Website: www.physicsbyfiziks.com  | Email: fiziks.physics@gmail.com  
29 
fiziks
Institute for NET/JRF, GATE, IIT‐JAM, M.Sc. Entrance, JEST, TIFR and GRE in Physics 
 
LE
2
 L2 E 
ln Z   ln N  N ln     N ln N  N  N ln  
 N   N 
 L2 E 
 F  kT ln Z  NkT ln N  NkT  NkT ln  
 N 
E  NkT

U F EF  L2 E 
S   Nk  Nk ln N  Nk  Nk ln  
T T  N 
2
 L2 E  L E L E 
 Nk ln  2   Nk ln    2 Nk ln  
 N   N   N 
Q70. Consider a random walk on an infinite two-dimensional triangular lattice, a part of which
is shown in the figure below.

If the probabilities of moving to any of the nearest neighbour sites are equal, what is the
probability that the walker returns to the starting position at the end of exactly three steps?
1 1 1 1
(a) (b) (c) (d)
36 216 18 12
Ans. : (c)
Solution: For walker to return to starting position it must move along an equivalent triangle in
three steps.
For steps one any movement can result in equilateral triangle.
For step two, two out of six options will form equilateral triangle.
For step three, only one out of six options will form equilateral triangle
6 2 1 1
Total probability    
6 6 6 18
Q71. An atom has a non-degenerate ground-state and a doubly-degenerate excited state. The
energy difference between the two states is  . The specific heat at very low temperatures
   1 is given by
(a) k B    (b) k B e   (c) 2k B    e  
2
(d) k B

Ans. : (c)
H.No. 40‐D, Ground Floor, Jia Sarai, Near IIT, Hauz Khas, New Delhi‐110016 
Phone: 011‐26865455/+91‐9871145498
Website: www.physicsbyfiziks.com  | Email: fiziks.physics@gmail.com  
30 
fiziks
Institute for NET/JRF, GATE, IIT‐JAM, M.Sc. Entrance, JEST, TIFR and GRE in Physics 
 
Solution: Assume energy at ground state is 0 and energy at first excited state is  . The partition
function is Z  1  2e   
2 e   
Energy 
1  2e  
 
1  2
2  e    2 2  e kT  2
kT
 U  kT  kT
Specific heat, CV    
 T V  
 2
 

1  2e 
kT
1  2e 
kT

   
 

2   1  2e
kT

2    1  2e 
 
   kT  
 2k   e  2k    e
 kT  1  2e    
2 2
 

 1  2e 
kT

 

CV  2k    e    ,   
2

Q72. The electrons in graphene can be thought of as a two-dimensional gas with a linear
 
energy-momentum relation E  p v , where p   px , p y  and v is a constant. If  is the

number of electrons per unit area, the energy per unit area is proportional to
(a)  3/ 2 (b)  (c)  1/ 3 (d)  2
Ans. : (a)
Solution: The number of k state in range k to k  dk :
2
 L 
In 2D , it is given by g  k  dk    2 kdk
 2 
Since, dispersion relation is E  P v  kv
2
 L  EdE L2
g  E  dE  2    2  2 2
 2   v    v
2

The number of electron at T  00 K is


EF EF
L2 L2 N
N  g Ed E   EdE  EF2  2  2 v 2 . 2  EF2
0
 2v2 0
2  v
2 2
L

 N
EF2  2  2 v 2    2 
 L 
The average energy at T  0 K is
H.No. 40‐D, Ground Floor, Jia Sarai, Near IIT, Hauz Khas, New Delhi‐110016 
Phone: 011‐26865455/+91‐9871145498
Website: www.physicsbyfiziks.com  | Email: fiziks.physics@gmail.com  
31 
fiziks
Institute for NET/JRF, GATE, IIT‐JAM, M.Sc. Entrance, JEST, TIFR and GRE in Physics 
 
EF

 E  g  E  dE L2
EF
L2 EF3
Eav  0

N

N 2v 2 
0
E 2 dE 
3N  2v 2
L 2
2 L2
Eav   2  v
2 2
 2  v 
2 2 1/ 2
 2 v  3/ 2
3N   v
2 2
3N
E NEa v 2 E
 2  2 v  3/ 2    3/ 2
L2
L 3 L2

NET/JRF (JUNE-2017)
Q73. A thermodynamic function
G T , P, N   U  TS  PV

is given in terms of the internal energy U , temperature T , entropy S , pressure P ,


volume V and the number of particles N . Which of the following relations is true? (In
the following  is the chemical potential.)

G G G G
(a) S   (b) S  (c) V   (d)   
T N ,P T N ,P P N ,T N P ,T

Ans. : (a)
Solution: G  U  TS  PV
dG  dU  Tds  sdT  PdV  VdP  TdS  PdV  TdS  SdT  PdV  VdP
dG   SdT  VdP
 G  G
    S and V
 T  N , P P N ,T

Q74. A box, separated by a movable wall, has two compartments filled by a monoatomic gas
CP
of   . Initially the volumes of the two compartments are equal, but the pressures are
CV

3P0 and P0 respectively. When the wall is allowed to move, the final pressures in the two
compartments become equal. The final pressure is
  
2 2  31/  
(c) 1  31/   P0
1 
(a)   P0 (b) 3   P0 (d)  1/  
P0
3 3 2  1 3 
Ans. : (c)
Solution: V1  V2  2V , V2  2V  V1 ,
H.No. 40‐D, Ground Floor, Jia Sarai, Near IIT, Hauz Khas, New Delhi‐110016 
Phone: 011‐26865455/+91‐9871145498
Website: www.physicsbyfiziks.com  | Email: fiziks.physics@gmail.com  
32 
fiziks
Institute for NET/JRF, GATE, IIT‐JAM, M.Sc. Entrance, JEST, TIFR and GRE in Physics 
 
   
3PV
0  PV1 , PV0  PV2

 P  2V  V1 
 
PV
0

From (i) and (ii)



 V1  1/  V1 1 2V  V1 2V 2V
3  3   1/     1  V1 
 2V  V1  2V  V1 3 V1 V1 1  1/ 31/  
put the value of V1 in (i)

 2V  3P P
 P  0 1  1/ 31/    0 1  31/  
  
3PV  P 1/  
 1  1/ 3 
0
2 2
Q75. A gas of photons inside a cavity of volume V is in equilibrium at temperature T . If the
temperature of the cavity is changed to 2T , the radiation pressure will change by a factor
of
(a) 2 (b) 16 (c) 8 (d) 4
Ans. : (b)
Solution: For 3 dimensional system P  T 4
4
P2  2T 
   P2  P1 2  p1 16  16 P1
4

P1  T 
Q76. In a thermodynamic system in equilibrium, each molecule can exist in three possible
states with probabilities 1/ 2, 1/ 3 and 1/ 6 respectively. The entropy per molecule is
1 2
(a) k B ln 3 (b) k B ln 2  k B ln 3
2 3
2 1 1 1
(c) k B ln 2  k B ln 3 (d) k B ln 2  k B ln 3
3 2 2 6
Ans. : (c)
Solution: S  k B  Pi ln Pi
i

1
P1  , P2  1/ 3 and P3  1/ 6 
2
1 
S  k B  ln1/ 2  1/ 3ln1/ 3  1/ 6 ln1/ 6  
2 
1 1 1
 kb   ln1  ln 2    ln1  ln 3   ln1  ln 6 
2 3 6

H.No. 40‐D, Ground Floor, Jia Sarai, Near IIT, Hauz Khas, New Delhi‐110016 
Phone: 011‐26865455/+91‐9871145498
Website: www.physicsbyfiziks.com  | Email: fiziks.physics@gmail.com  
33 
fiziks
Institute for NET/JRF, GATE, IIT‐JAM, M.Sc. Entrance, JEST, TIFR and GRE in Physics 
 
1 1 1 1  1 1 1 1 
 k B  ln 2  ln 3  ln 2  ln 3  k B  ln 2  ln 2  ln 3  ln 3
2 3 6 6  2 6 3 6 
 3ln 2  ln 2 2ln 3  ln 3   4 ln 2 3ln 3  2 1 
S  kB     kB     kB  3 ln 2  2 ln 3
 6 6   6 6 
Q77. The single particle energy levels of a non-interacting three-dimensional isotropic system,
labelled by momentum k , are proportional to k 3 . The ratio P /  of the average pressure
P to the energy density  at a fixed temperature, is
(a) 1/ 3 (b) 2 / 3 (c) 1 (d) 3
Ans. : (c)
Solution: E  p s , where p is momentum

sE
P    , where P is pressure
3V 
P s
 .
E 3
In problem, E  k 3 , so, s  3
3 E  E
pressure P     P    at fixed T .
3V  V 
Q78. The Hamiltonian for three Ising spins S0 , S1 and S2 , taking values 1 , is

H   JS0  S1  S 2 

If the system is in equilibrium at temperature T , the average energy of the system, in

terms of    k BT  , is
1

1  cosh  2 J 
(a)  (b) 2 J 1  cosh  2 J  
2  sinh  2 J 

sinh  2 J 
(c) 2 /  (d) 2 J
1  cosh  2  J 

Ans. : (d)
Solution: H   JS0  S1  S 2    J  S0 S1  S0 S 2  S0  1 S1   S 2  1

S0 S1 S2 E
1 1 1 2 J

H.No. 40‐D, Ground Floor, Jia Sarai, Near IIT, Hauz Khas, New Delhi‐110016 
Phone: 011‐26865455/+91‐9871145498
Website: www.physicsbyfiziks.com  | Email: fiziks.physics@gmail.com  
34 
fiziks
Institute for NET/JRF, GATE, IIT‐JAM, M.Sc. Entrance, JEST, TIFR and GRE in Physics 
 
1 1 1 2J
1 1 1 0
1 1 1 0
1 1 1 0
1 1 1 0
1 1 1 2J
1 1 1 2 J
E1  2 J g1  2

E2  2 J g2  2

E3  0 g3  4

E g e
  Ei

U i i

g e  i
 Ei

0  2 J 2e 2  J   2 J  2e 2  J 4 J  e 2  J  e 2  J  8 J   sinh 2 J 
U  
4  2 e e 
2  J 2 J
4  2e  2e 2  J 2 J
4  4 cosh 2  J

2 J sinh 2 J
U 
1  cosh 2 J

H.No. 40‐D, Ground Floor, Jia Sarai, Near IIT, Hauz Khas, New Delhi‐110016 
Phone: 011‐26865455/+91‐9871145498
Website: www.physicsbyfiziks.com  | Email: fiziks.physics@gmail.com  
35 
fiziks
Institute for NET/JRF, GATE, IIT‐JAM, M.Sc. Entrance, JEST, TIFR and GRE in Physics 
 
ELECTRONICS AND EXPERIMENTAL METHODS
NET/JRF (JUNE-2011)

Q1. A signal of frequency 10 k Hz is being digitalized by an A/D converter. A possible


sampling time which can be used is
(a) 100  s (b) 40  s (c) 60  s (d) 200  s
Ans. : (b)
1 1
Solution: f S  2 f  TS    50 s  TS  50 s
2 f 20  103
Q2. Consider the digital circuit shown below in which the input C is always high (1).
A
B Z

C
(high)

The truth table for the circuit can be written as


A B Z
0 0 1
0 1 0
1 0 1
1 1 1

The entries in the Z column (vertically) are


(a) 1010 (b) 0100 (c) 1111 (d) 1011
Ans. : (d)
Solution: Z  A.B  B  1

H.No. 40‐D, Ground Floor, Jia Sarai, Near IIT, Hauz Khas, New Delhi‐110016 
Phone: 011‐26865455/+91‐9871145498
Website: www.physicsbyfiziks.com  | Email: fiziks.physics@gmail.com  

fiziks
Institute for NET/JRF, GATE, IIT‐JAM, M.Sc. Entrance, JEST, TIFR and GRE in Physics 
 
Q3. A time varying signal Vin is fed to an op-amp circuit with output signal Vo as shown in
10 K
the figure below.
The circuit implements a
(a) high pass filter with cutoff frequency 16 Hz 1K

Vo
(b) high pass filter with cutoff frequency 100 Hz V in 
1K 10 K
10 K
(c) low pass filter with cutoff frequency 16 Hz 1 F

(d) low pass filter with cutoff frequency 100 Hz 


Ans. : (c)
Solution: Since circuit has R and C combination, its a Low Pass filter and cutoff frequency
1
  16 Hz.
2RC

NET/JRF (DEC-2011)
Q4. In the operational amplifier circuit below, the voltage at point A is
5V
1K A
1V 
1V 
1K
1K 5V

(a) 1.0 V (b) 0.5V (c) 0 V (d) 5.0V


Ans. : (b)
1
Solution: V A   1  0.5V .
11

H.No. 40‐D, Ground Floor, Jia Sarai, Near IIT, Hauz Khas, New Delhi‐110016 
Phone: 011‐26865455/+91‐9871145498
Website: www.physicsbyfiziks.com  | Email: fiziks.physics@gmail.com  

fiziks
Institute for NET/JRF, GATE, IIT‐JAM, M.Sc. Entrance, JEST, TIFR and GRE in Physics 
 
Q5. A counter consists of four flip-flops connected as shown in the figure:
A0 A1 A2 A3

J Q J Q J Q J Q
CLK
K Q K Q K Q K Q

If the counter is initialized as A0 A1 A2 A3  0110 , the state after the next clock pulse is

(a) 1000 (b) 0001 (c) 0011 (d) 1100


Ans: (b)
A 0 0 A1 1 A 2 1 A 3 0
Solution:

0 J Q J Q 0 J Q J Q
CLK
1 K Q K Q 0 1 K Q K Q
1 0 1

Q6. The pins 0, 1, 2 and 3 of part A of a microcontroller are connected with resistors to drive
an LED at various intensities as shown in the figure. For V CC
VCC = 4.2 V and a voltage drop of 1.2 V across the LED, 
the range (maximum current) and resolution (step size)
A3
of the drive current are, respectively, 0 . 75 k
A2
(a) 4.0 mA and 1.0 mA C 1 .5 k
A1
(b) 15.0 mA and 1.0 mA 3k
(c) 7.5 mA and 0.5 mA A0
6k
(d) 4.0 mA and 0.5 mA
Ans: (c)
A3 , A2 , A1 , A0
Solution: For Maximum current
0, 0, 0, 0
4.2  1.2 4.2  1.2 4.2  1.2 4.2  1.2
Thus, I max      7.5mA
0.75k 1.5k 3k 6k

H.No. 40‐D, Ground Floor, Jia Sarai, Near IIT, Hauz Khas, New Delhi‐110016 
Phone: 011‐26865455/+91‐9871145498
Website: www.physicsbyfiziks.com  | Email: fiziks.physics@gmail.com  

fiziks
Institute for NET/JRF, GATE, IIT‐JAM, M.Sc. Entrance, JEST, TIFR and GRE in Physics 
 
A ,A ,A,A 4.2  1.2
For Step size 3 2 1 0 . Thus I 0   0.5mA
0, 0, 0, 1 6k
Q7. The figure below shows a voltage regulator utilizing a Zener diode of breakdown voltage
5 V and a positive triangular wave input of amplitude 10 V.
500 12
Vi 10
i 8

i(mA)
6
1K 4
2
0
0 1 2 3 4 5 6 7 8
t (s)
For Vi > 5V, the Zener regulates the output voltage by channeling the excess current
through itself. Which of the following waveforms shows the current i passing through the
Zener diode?
(a) 12 (b) 12
10
10
8
i(mA)

8
6
i(mA)

6
4
4
2
2
0
0 1 2 3 4 5 6 7 8 0
0 1 2 3 4 5 6 7 8
t (s) t (s)
(c) 12 (d) 12
10 10
8 8
i(mA)
i(mA)

6 6
4 4
2 2
0 0
0 1 2 3 4 5 6 7 8 0 1 2 3 4 5 6 7 8
t (s) t (s)
Ans: (a)
Solution: When zener is OFF zener current is zero when zener is ON zener current will flow.

H.No. 40‐D, Ground Floor, Jia Sarai, Near IIT, Hauz Khas, New Delhi‐110016 
Phone: 011‐26865455/+91‐9871145498
Website: www.physicsbyfiziks.com  | Email: fiziks.physics@gmail.com  

fiziks
Institute for NET/JRF, GATE, IIT‐JAM, M.Sc. Entrance, JEST, TIFR and GRE in Physics 
 
NET/JRF (JUNE-2012)
Q8. In the op-amp circuit shown in the figure below, the input voltage is 1V. The value of the
1K
output V0 is

1K
 1K Vo
Vi  1V

1K

(a) -0.33 V (b) -0.50 V (c) -1.00 V (d) -0.25 V


Ans. : (b)
R F Vin 1 11 1
Solution: V0     V  0.05 where R F   K and R1  1K .
R1 2 11 2
Q9. An LED operates at 1.5 V and 5 mA in forward bias. Assuming an 80% external
efficiency of the LED, how many photons are emitted per second?
(a) 5.0 x 1016 (b) 1.5 x 1016 (c) 0.8 x 1016 (d) 2.5 x 1016
Ans. : (d)
i Pin i 5  103
Solution: Pin  ext hf , number of photon   ext  .8  19
 2.5  1016
e hf e 1.6  10

Q10. The transistor in the given circuit has hfe = 35Ω and hie = 1000Ω. If the load resistance
RL = 1000Ω, the voltage and current gain are, respectively.
(a) -35 and + 35 VO
RL
(b) 35 and - 35

(c) 35 and – 0.97
VI 

(d) 0.98 and - 35

Ans. : (a)

H.No. 40‐D, Ground Floor, Jia Sarai, Near IIT, Hauz Khas, New Delhi‐110016 
Phone: 011‐26865455/+91‐9871145498
Website: www.physicsbyfiziks.com  | Email: fiziks.physics@gmail.com  

fiziks
Institute for NET/JRF, GATE, IIT‐JAM, M.Sc. Entrance, JEST, TIFR and GRE in Physics 
 
Q11. The output, O, of the given circuit in cases I and II, where
Case I: A, B = 1; C, D = 0; E, F = 1 and G = 0
Case II: A, B = 0; C, D = 0: E, F = 0 and G = 1
are respectively 

(a) 1, 0
(b) 0, 1 C
D
(c) 0, 0
(d) 1, 1
Ans. : (d) O

 
Solution: O  AB  CD E  F G  E F
G

NET/JRF (DEC-2012)
Q12. A live music broadcast consists of a radio-wave of frequency 7 MHz, amplitude-
modulated by a microphone output consisting of signals with a maximum frequency of
10 kHz. The spectrum of modulated output will be zero outside the frequency band
(a) 7.00 MHz to 7.01 MHz (b) 6.99 MHz to 7.01 MHz
(c) 6.99 MHz to 7.00 MHz (d) 6.995 MHz to 7.005 MHz
Ans: (b)
Solution: Spectrum consists of f c  f m and f c  f m .
Q13. In the op-amp circuit shown in the figure, Vi is a sinusoidal input signal of frequency 10
Hz and V0 is the output signal. The magnitude of the gain and the phase shift, respectively,
0.01F
close to the values
(a) 5 2 and  / 2
10 K
(b) 5 2 and   / 2 1K
Vi 
(c) 10 and zero Vo

(d) 10 and π
Ans: (d)
v0 X C RF v
Solution:   0  10
vin R1 R1  R F  vin

H.No. 40‐D, Ground Floor, Jia Sarai, Near IIT, Hauz Khas, New Delhi‐110016 
Phone: 011‐26865455/+91‐9871145498
Website: www.physicsbyfiziks.com  | Email: fiziks.physics@gmail.com  

fiziks
Institute for NET/JRF, GATE, IIT‐JAM, M.Sc. Entrance, JEST, TIFR and GRE in Physics 
 
Q14. The logic circuit shown in the figure below Implements the Boolean expression
A

HIGH y

(a) y  A  B (b) y  A  B (c) y  A  B (d) y  A  B


Ans: (a)

Solution: Output of each Ex-OR gate is A and B . Thus y  A  B  A  B


Q15. A diode D as shown in the circuit has an i-v relation that can be approximated by
v 2  2v D , for v D  0
iD   D
0, for v D  0 1

The value of v D in the circuit is iD




(a)  1  11 V  (b) 8 V 10 V

D vD

(c) 5 V (d) 2 V
Ans: (d)
Solution:  10  v D2  2v D  1  v D  0  v D  2V
Q16. Band-pass and band-reject filters can be implemented by combining a low pass and a
high pass filter in series and in parallel, respectively. If the cut-off frequencies of the low
pass and high pass filters are  0LP and  0HP , respectively, the condition required to
implement the band-pass and band-reject filters are, respectively,
(a)  0HP   0LP and  0HP   0LP (b) 0HP  0LP and  0HP   0LP

(c)  0HP   0LP and 0HP  0LP (d)  0HP   0LP and 0HP  0LP
Ans: (b)

H.No. 40‐D, Ground Floor, Jia Sarai, Near IIT, Hauz Khas, New Delhi‐110016 
Phone: 011‐26865455/+91‐9871145498
Website: www.physicsbyfiziks.com  | Email: fiziks.physics@gmail.com  

fiziks
Institute for NET/JRF, GATE, IIT‐JAM, M.Sc. Entrance, JEST, TIFR and GRE in Physics 
 
NET/JRF (JUNE-2013)

Q17. A silicon transistor with built-in voltage 0.7 V is used in the circuit shown, with
V BB  9.7V , R B  300k, VCC  12V and RC  2k . Which of the following figures

correctly represents the load line and quiescent Q point?

RC

RB 
 
V CC
V BB

iC iC
    mA  
32 35 6 35
(a) 32 (b) 32
Q
30 Q
30
0 9 .7 VCE V  0 12 VCE V 

iC iC
mA   A  
6 35 32 Q 35
(c) Q
32 (d) 32
30 30
0 12 VCE V  0 9 .7 VCE V 

Ans: (b)
VBB  VBE 9.7  0.7 V 12
Solution: I B    30 A and I C , sat  CC   6mA
RB 300  10 3
RC 2  10 3
Q18. If the analog input to an 8-bit successive approximation ADC is increased from 1.0 V to
2.0 V, then the conversion time will
(a) remain unchanged (b) double
(c) decrease to half its original value (d) increase four times
Ans: (a)

H.No. 40‐D, Ground Floor, Jia Sarai, Near IIT, Hauz Khas, New Delhi‐110016 
Phone: 011‐26865455/+91‐9871145498
Website: www.physicsbyfiziks.com  | Email: fiziks.physics@gmail.com  

fiziks
Institute for NET/JRF, GATE, IIT‐JAM, M.Sc. Entrance, JEST, TIFR and GRE in Physics 
 
Q19. The input to a lock-in amplifier has the form Vi t   Vi sin  t   i  where Vi ,  ,  i are the
amplitude, frequency and phase of the input signal respectively. This signal is multiplied
by a reference signal of the same frequency  , amplitude Vr and phase  r . If the
multiplied signal is fed to a low pass filter of cut-off frequency  , then the final output
signal is
1  1 
(a) ViVr cos i   r  (b) ViVr cos i   r   cos  t   i   r 
2  2 

 1 
(c) ViVr sin  i   r  (d) ViVr cos i   r   cos  t   i   r 
 2 
Ans: (a)
Vi V r
Solution: V  Vr sin  t   r   Vi sin  t   i   cos i   r   cos2 t   i   r 
2
Vi V r
Output of low pass filter= cos i   r 
2
Q20. Four digital outputs V , P, T and H monitor the speed v , tyre pressure p , temperature t
and relative humidity h of a car. These outputs switch from 0 to 1 when the values of the
parameters exceed 85 km/hr, 2 bar, 40 0 C and 50%, respectively. A logic circuit that is
used to switch ON a lamp at the output E is shown below.
Which of the following condition will not switch the lamp ON?
(a) v  85km / hr , p  2 bar , t  40 0 C , h  50%
V
(b) v  85km / hr , p  2 bar , t  40 C , h  50%
0

P
(c) v  85km / hr , p  2 bar , t  40 0 C , h  50% E

(d) v  85km / hr , p  2 bar , t  400 C , h  50% T

H
Ans: (d)

H.No. 40‐D, Ground Floor, Jia Sarai, Near IIT, Hauz Khas, New Delhi‐110016 
Phone: 011‐26865455/+91‐9871145498
Website: www.physicsbyfiziks.com  | Email: fiziks.physics@gmail.com  

fiziks
Institute for NET/JRF, GATE, IIT‐JAM, M.Sc. Entrance, JEST, TIFR and GRE in Physics 
 
JRF/NET-(DEC-2013)
Q21. Consider the op-amp circuit shown in the figure.
If the input is a sinusoidal wave Vi  5 sin 1000t  , then 1 F

the amplitude of the output V0 is


1K
5 1K
(a) (b) 5 Vi  Vo
2 
5 2
(c) (d) 5 2
2
Ans: (c)
vo X RF X C 10 3 1
Solution:   F , XF   where R F  1  10 3 , X C 
vin R1 RF  X C 1  j  j  10  10 6
3

vo 10 3 1 1 5 5 2
  3   vo  sin t  sin t
vin 2 10 2 2 2

Q22. If one of the inputs of a J-K flip flop is high and the other is low, then the outputs Q and

Q
(a) oscillate between low and high in race around condition
(b) toggle and the circuit acts like a T flip flop
(c) are opposite to the inputs
(d) follow the inputs and the circuit acts like an R  S flip flop
Ans: (d)
Q23. A sample of Si has electron and hole mobilities of 0.13 and 0.05 m 2 /V- s respectively at
300 K. It is doped with P and Al with doping densities of 1.5  10 21 / m 3 and
2.5  10 21 / m 3 respectively. The conductivity of the doped Si sample at 300 K is
(a) 8  1 m 1 (b) 32  1 m 1 (c) 20.8  1 m 1 (d) 83.2  1 m 1
Ans: (a)
Solution: Resulting doped crystal is p-type and p p  2.5  1.5  10 21 / m 3  1  10 21 / m 3

  en p  n  p p  p   ep p  p  1.6  10 19  1  10 21  0.05  8  1 m 1

H.No. 40‐D, Ground Floor, Jia Sarai, Near IIT, Hauz Khas, New Delhi‐110016 
Phone: 011‐26865455/+91‐9871145498
Website: www.physicsbyfiziks.com  | Email: fiziks.physics@gmail.com  
10 
fiziks
Institute for NET/JRF, GATE, IIT‐JAM, M.Sc. Entrance, JEST, TIFR and GRE in Physics 
 
Q24. Two identical Zener diodes are placed back to back in series and are connected to a
variable DC power supply. The best representation of the I-V characteristics of the circuit
is
(a) I (b) I

V V

(c) I (d) I

V V

Ans: (d)
Q25. A 4-variable switching function is given by f   5, 7, 8, 10, 13, 15  d 0, 1, 2  , where

d is the do-not-care-condition. The minimized form of f in sum of products (SOP) form


is
(a) A C  B D (b) AB  CD (c) AD  BC (d) B D  BD

Ans: (d) CD CD CD CD
AB    BD

AB 1 1 BD
AB 1 1

AB 1 1

H.No. 40‐D, Ground Floor, Jia Sarai, Near IIT, Hauz Khas, New Delhi‐110016 
Phone: 011‐26865455/+91‐9871145498
Website: www.physicsbyfiziks.com  | Email: fiziks.physics@gmail.com  
11 
fiziks
Institute for NET/JRF, GATE, IIT‐JAM, M.Sc. Entrance, JEST, TIFR and GRE in Physics 
 
NET/JRF (JUNE-2014)
Q26. The inner shield of a triaxial conductor is driven by an (ideal) op-amp follower circuit as
shown. The effective capacitance between the signal-carrying conductor and ground is

Signal


(a) unaffected (b) doubled (c) halved (d) made zero


Ans: (a)
Q27. An op-amp based voltage follower
(a) is useful for converting a low impedance source into a high impedance source.
(b) is useful for converting a high impedance source into a low impedance source.
(c) has infinitely high closed loop output impedance
(d) has infinitely high closed loop gain
Ans: (b)
Q28. An RC network produces a phase-shift of 30 o . How many such RC networks should be
cascaded together and connected to a Common Emitter amplifier so that the final circuit
behaves as an oscillator?
(a) 6 (b) 12 (c) 9 (d) 3
Ans: (a)
Solution: Total phase shift must be 0 or 3600. Common Emitter amplifier has phase change of
1800 so we need 6 RC network for next 1800 phase shift.

H.No. 40‐D, Ground Floor, Jia Sarai, Near IIT, Hauz Khas, New Delhi‐110016 
Phone: 011‐26865455/+91‐9871145498
Website: www.physicsbyfiziks.com  | Email: fiziks.physics@gmail.com  
12 
fiziks
Institute for NET/JRF, GATE, IIT‐JAM, M.Sc. Entrance, JEST, TIFR and GRE in Physics 
 
Q29. For the logic circuit shown in the below

X
B

A simplified equivalent circuit is A


(a) A (b) B
B X X
C
C

A A
(c) (d)
B B
X X
C C
Ans: (d)
Solution:

A A
A.B  A  B  AC
B X
B

C
C
ABC

X   A  B  AC  ABC  AC  ABC  ABC  AC  AB  A  B  C 

H.No. 40‐D, Ground Floor, Jia Sarai, Near IIT, Hauz Khas, New Delhi‐110016 
Phone: 011‐26865455/+91‐9871145498
Website: www.physicsbyfiziks.com  | Email: fiziks.physics@gmail.com  
13 
fiziks
Institute for NET/JRF, GATE, IIT‐JAM, M.Sc. Entrance, JEST, TIFR and GRE in Physics 
 
NET/JRF (DEC-2014)
Q30. Consider the amplifier circuit comprising of the two op-amps A1 and A2 as shown in the
figure. 1M
R
 10 K

 r
A1  V0
A2

If the input ac signal source has an impedance of 50 k  , which of the following


statements is true?
(a) A1 is required in the circuit because the source impedance is much greater than r
(b) A1 is required in the circuit because the source impedance is much less than R
(c) A1 can be eliminated from the circuit without affecting the overall gain
(d) A1 is required in the circuit if the output has to follow the phase of the input signal
Ans: (a)
Solution: A1 is required in the circuit because the source impedance is much greater than r
Q31. The I  V characteristics of the diode in the circuit below is given by
V  0.7  / 500 for V  0.7
I  
 0 for V  0.7
where V is measured in volts and I is measured in amperes.
1K I

10 V

The current I in the circuit is


(a) 10.0 mA (b) 9.3 mA (c) 6.2 mA (d) 6.7 mA
Ans: (c)
Solution: Applying K.V.L. 10  1000  I  V  0  10  1000  V  0.7  / 500  V  0

 10  2 V  0.7   V  0  3V  11.4  V  3.8 Volts

Thus I  V  0.7  / 500   3.8  0.7  / 500  3.1/ 500  6.2 mA


H.No. 40‐D, Ground Floor, Jia Sarai, Near IIT, Hauz Khas, New Delhi‐110016 
Phone: 011‐26865455/+91‐9871145498
Website: www.physicsbyfiziks.com  | Email: fiziks.physics@gmail.com  
14 
fiziks
Institute for NET/JRF, GATE, IIT‐JAM, M.Sc. Entrance, JEST, TIFR and GRE in Physics 
 
Q32. In a measurement of the viscous drag force experienced by spherical particles in a liquid,
the force is found to be proportional to V 1 / 3 where V is the measured volume of each
particle. If V is measured to be 30 mm 3 , with an uncertainty of 2.7 mm 3 , the resulting
relative percentage uncertainty in the measured force is
(a) 2.08 (b) 0.09 (c) 6 (d) 3
Ans: (b)
2
 F  2
Solution: The relative percentage uncertainty in the measure force is    2

 V  V
F

 F 
F     V where  V is the uncertainty in the measurement of volume.
 V 
F 1 2 / 3
 F  V 1/3   V
V 3
1 1 1 1
F   V   2.7   2.7   2.7   F  0.09
3  30 3   900 3  9.7
2/3 2/3 1/ 3
3V

Q33. Consider a Low Pass (LP) and a High Pass (HP) filter with cut-off frequencies f LP and
f HP , respectively, connected in series or in parallel configurations as shown in the
Figures A and B below. 
fHP
(A) Input  L Output (B) Input Output
fHP fLP
L
Which of the following statements is correct? fLP
(a) For f HP  f LP , A acts as a Band Pass filter and B acts as a band Reject filter
(b) For f HP  f LP , A stops the signal from passing through and B passes the signal
without filtering
(c) For f HP  f LP , A acts as a Band Pass filter and B passes the signal without filtering
(d) For f HP  f LP , A passes the signal without filtering and B acts as a Band Reject filter
Ans: (c)
Q34. The power density of sunlight incident on a solar cell is 100 mW / cm 2 . Its short circuit

current density is 30 mA / cm 2 and the open circuit voltage is 0.7 V . If the fill factor of
the solar cell decreases from 0.8 to 0.5 then the percentage efficiency will decrease from
(a) 42.0 to 26.2 (b) 24.0 to 16.8 (c) 21.0 to 10.5 (d) 16.8 to 10.5

H.No. 40‐D, Ground Floor, Jia Sarai, Near IIT, Hauz Khas, New Delhi‐110016 
Phone: 011‐26865455/+91‐9871145498
Website: www.physicsbyfiziks.com  | Email: fiziks.physics@gmail.com  
15 
fiziks
Institute for NET/JRF, GATE, IIT‐JAM, M.Sc. Entrance, JEST, TIFR and GRE in Physics 
 
Ans: (d)
Solution: The efficiency of a solar cell is determined as the fraction of incident power which is
converted to electricity and is defined as
Voc I sc FF
 and Pmax  Voc I sc FF
Pin

where Voc is the open circuit voltage, I sc is the short circuit current density , FF is the

Fill factor, Pin is the input power and  is the efficiency of the solar cell.

Given Pin  100 mW / cm2 , I sc  30 mA / cm 2 , Voc  0.7 V

Let 1 is the efficiency of solar cell when FF  0.8

 0.7 V    30  103 A / cm2   0.8 16.8


 1  3
  1  0.168
100  10 W / cm 2
100
Let 2 is the efficiency of solar cell when FF  0.5

 0.7 V    30  103 A / cm2   0.5 10.5


 2  3
  2  0.105
100  10 W / cm 2
100
Thus efficiency decreases from 1  16.8% to 2  10.5%

NET/JRF (JUNE-2015)
Q35. The concentration of electrons, n and holes p , for an intrinsic semiconductor at a
3
 E 
temperature T can be expressed as n  p  AT 2 exp   g  , where Eg is the band
 2 k BT 
3
gap and A is a constant. If the mobility of both types of carrier is proportional to T 2
,
then the log of the conductivity is a linear function of T 1 , with slope
Eg Eg  Eg  Eg
(a) (b) (c) (d)
 2k B  kB  2k B  kB

Ans. (c)
3
  Eg  23   Eg 
Solution:  i  ni e  e   p   T 2 exp    T   i  C exp  
 2 k BT   2k B T 
Eg  Eg
ln  i    ln C  slope is
2 k BT 2k B

H.No. 40‐D, Ground Floor, Jia Sarai, Near IIT, Hauz Khas, New Delhi‐110016 
Phone: 011‐26865455/+91‐9871145498
Website: www.physicsbyfiziks.com  | Email: fiziks.physics@gmail.com  
16 
fiziks
Institute for NET/JRF, GATE, IIT‐JAM, M.Sc. Entrance, JEST, TIFR and GRE in Physics 
 
 Pa 4
Q36. The viscosity  of a liquid is given by Poiseuille’s formula   . Assume that
8lV
l and V can be measured very accurately, but the pressure P has an rms error of 1% and
the radius a has an independent rms error of 3% . The rms error of the viscosity is
closest to
(a) 2% (b) 4% (c) 12% (d) 13%
Ans. (c)
Solution:   k pa 4
2
     
     p2     a2   a 4   p2   4 Pa 3   a2
2 2 2
n
 P   a 
2
 p
2 2
n   a 
 n 100    p  100   16  a 100   1  16  3  1  144  145
2 2

     

n 
 n 100   12%
 
Q37. Consider the circuits shown in figures (a) and (b) below
2K 1K

10 K 10 K
10V 10V

10.7V 5V

(a) (b)
If the transistors in Figures (a) and (b) have current gain   dc  of 100 and 10 respectively,

then they operate in the


(a) active region and saturation region respectively
(b) saturation region and active region respectively
(c) saturation region in both cases
(d) active region in both cases
Ans. (b)
Solution: In both case input section is F.B.
10.7  0.7
For figure (a) I B   1 mA  I C  BI B  100 mA
10

H.No. 40‐D, Ground Floor, Jia Sarai, Near IIT, Hauz Khas, New Delhi‐110016 
Phone: 011‐26865455/+91‐9871145498
Website: www.physicsbyfiziks.com  | Email: fiziks.physics@gmail.com  
17 
fiziks
Institute for NET/JRF, GATE, IIT‐JAM, M.Sc. Entrance, JEST, TIFR and GRE in Physics 
 
Thus VCB  VC  VB  10  2  100   0.7  ve

 output section is F.B.


since both section are F.B. so it is in saturation region.
5  0.7
For Figure (b) I B   0.43 mA  I C  BI B  4.3 mA
10
Thus VCB  VC  VB  10  4.3  0.7)   ve

 out put section is R.B.


Thus it is in active region
Q38. In the circuit given below, the thermistor has a resistance 3 k at 250 C . Its resistance

decreases by 150 per 0 C upon heating. The output voltage of the circuit at 300 C is
T

1V

1K
 Vout

(a) 3.75 V (b) 2.25 V (c) 2.25 V (d) 3.75 V


Ans. (c)
Solution: At 300 C Resistance
 3000  150  5  2250 
RF 2250
 V0   vi   1  V0  2.25 volts
R1 1000

H.No. 40‐D, Ground Floor, Jia Sarai, Near IIT, Hauz Khas, New Delhi‐110016 
Phone: 011‐26865455/+91‐9871145498
Website: www.physicsbyfiziks.com  | Email: fiziks.physics@gmail.com  
18 
fiziks
Institute for NET/JRF, GATE, IIT‐JAM, M.Sc. Entrance, JEST, TIFR and GRE in Physics 
 
NET/JRF (DEC-2015)
Q39. If the reverse bias voltage of a silicon varactor is increased by a factor of 2 , the
corresponding transition capacitance
(a) increases by a factor of 2 (b) increases by a factor of 2
(c) decreases by a factor of 2 (d) decreases by a factor of 2
Ans.: (c)

1 CT V C V 1
Solution: CT     T   CT  CT
V CT V CT 2V 2

Q40. If the parameters y and x are related by y  log  x  , then the circuit that can be used to

produce an output voltage V0 varying linearly with x is

(a) y (b) y
 
Vo Vo
 

(c) (d)
y  y 
Vo Vo
 

Ans.: (c)
Solution: (1) Integrator
(2) Logarithmic Ampere V0  log y 

(3) Anti-log V0  e y  x 

(4) Differentiator

H.No. 40‐D, Ground Floor, Jia Sarai, Near IIT, Hauz Khas, New Delhi‐110016 
Phone: 011‐26865455/+91‐9871145498
Website: www.physicsbyfiziks.com  | Email: fiziks.physics@gmail.com  
19 
fiziks
Institute for NET/JRF, GATE, IIT‐JAM, M.Sc. Entrance, JEST, TIFR and GRE in Physics 
 
Q41. The state diagram corresponding to the following circuit is

x D A
y
CLOCK

Flip Flop
00, 01,10 01,11
(a) 11 00 (b) 00,10 00,10

0 1 0 1

01,10,11 01,11

00,11 00, 01,10


(c) 01,10 00,10 (d) 11 11

0 1 0 1

00,11 00, 01,10


Ans.: (d)
Solution: DA  xy  A

Input Present Flip-Flop Next State


x y State A Input DA A

0 0 0 1 1
0 0 1 0 0
0 1 0 1 1
0 1 1 0 0
1 0 0 1 1
1 0 1 0 0
1 1 0 0 0
1 1 1 1 1

H.No. 40‐D, Ground Floor, Jia Sarai, Near IIT, Hauz Khas, New Delhi‐110016 
Phone: 011‐26865455/+91‐9871145498
Website: www.physicsbyfiziks.com  | Email: fiziks.physics@gmail.com  
20 
fiziks
Institute for NET/JRF, GATE, IIT‐JAM, M.Sc. Entrance, JEST, TIFR and GRE in Physics 
 
Q42. A sinusoidal signal of peak to peak amplitude 1V and unknown time period is input to

the following circuit for 5 second’s duration. If the counter measures a value  3E 8  H in

hexadecimal, then the time period of the input signal is


0.1  F
 10 bit
Vi  counter
1K 10 K

(a) 2.5 ms (b) 4 ms (c) 10 ms (d) 5 ms


Ans.: (d)
Solution:  3E 8  H  3 162  14 16  8 1  1000 10

In 5 sec, number of counts is 1000


Then count per sec is  200 count/sec
1
So, T  sec  5ms
200

NET/JRF (JUNE-2016)
Q43. The dependence of current I on the voltage V of a certain device is given by
2
 V 
I  I 0 1  
 V0 
where I 0 and V0 are constants. In an experiment the current I is measured as the voltage

V applied across the device is increased. The parameters V0 and I 0 can be graphically

determined as
(a) the slope and the y -intercept of the I  V 2 graph
(b) the negative of the ratio of the y -intercept and the slope, and the y -intercept of the

I  V 2 graph
(c) the slope and the y -intercept of the I  V graph
(d) the negative of the ratio of the y -intercept and the slope, and the y -intercept of the

I  V graph
Ans: (d)

H.No. 40‐D, Ground Floor, Jia Sarai, Near IIT, Hauz Khas, New Delhi‐110016 
Phone: 011‐26865455/+91‐9871145498
Website: www.physicsbyfiziks.com  | Email: fiziks.physics@gmail.com  
21 
fiziks
Institute for NET/JRF, GATE, IIT‐JAM, M.Sc. Entrance, JEST, TIFR and GRE in Physics 
 
 V 
2
 V   I0 I
Solution: I  I 0 1    I  I 0 1    I  V  I0
 V0   V0  V0
 I0  I0
Slope    V0
V0  I0 V
V0
Intercept on y -axis  I 0
Q44. In the schematic figure given below, assume that the propagation delay of each logic gate
is t . 5 V
gate

The propagation delay of the circuit will be maximum when the logic inputs A and B
make the transition
(a)  0,1  1,1 (b) 1,1   0,1
(c)  0, 0   1,1 (d)  0, 0    0,1
Ans: (d)
Solution:
Input Output
A B NOT OR AND OR
0 1 0 0 0 0
    3t
1 1 0 1 1 1
1 1 0 1 1 1
    3t
0 1 0 0 0 0

0 0 1 1 1 1
    t
1 1 0 1 1 1
0 0 1 1 1 1
    4t
0 1 0 0 0 0

H.No. 40‐D, Ground Floor, Jia Sarai, Near IIT, Hauz Khas, New Delhi‐110016 
Phone: 011‐26865455/+91‐9871145498
Website: www.physicsbyfiziks.com  | Email: fiziks.physics@gmail.com  
22 
fiziks
Institute for NET/JRF, GATE, IIT‐JAM, M.Sc. Entrance, JEST, TIFR and GRE in Physics 
 
Q45. Given the input voltage Vi , which of the following waveforms correctly represents the

output voltage V0 in the circuit shown below?

0.5 10 K

5K
Vi 0 Vi 
t
V0
0.5V 
5K 10 K
0.5
0

3.0 3.0
(a) 2.5 (b) 2.5
2.0 t V0 2.0
V0
1.5 1.5
1.0 1.0 t
0.5 0.5
0.0 0.0
0 0

3.0 3.0
(c) 2.5 (d) 2.5
2.0 2.0 t
V0 1.5 V0 1.5
1.0 t 1.0
0.5 0.5
0.0 0.00
0
Ans: (b)
 10  10 10
Solution: V0   1     0.5   Vi  V0  1  2Vi
 5  15 5
When Vi  0  V0  1V , when Vi  0.1V  V0  0.8 V , when Vi  0.5V  V0  0V

Q46. The decay constants f p of the heavy pseudo-scalar mesons, in the heavy quark limit, are

a
related to their masses m p by the relation f p  , where a is an empirical parameter
mp

to be determined. The values m p   6400  160  MeV and f p  180  15  MeV

correspond to uncorrelated measurements of a meson. The error on the estimate of a is


3 3 3 3
(a) 175  MeV  2 (b) 900  MeV  2 (c) 1200  MeV  2 (d) 2400  MeV  2

Ans. : (c)
H.No. 40‐D, Ground Floor, Jia Sarai, Near IIT, Hauz Khas, New Delhi‐110016 
Phone: 011‐26865455/+91‐9871145498
Website: www.physicsbyfiziks.com  | Email: fiziks.physics@gmail.com  
23 
fiziks
Institute for NET/JRF, GATE, IIT‐JAM, M.Sc. Entrance, JEST, TIFR and GRE in Physics 
 
Solution: a  f p m p
1/ 2

2 2
 a  2  a  2 a a f
 
2
  f p     m p   m1/p 2 and  p1
a f p
   m p  f p m p
2m p2
1

f 2
  f
2 
2
 m 
2
  f 2
   mp 
2
2
  a2  m p 2f p   m2    p    a  a  p 
p
 p
a
    
4m p p
a  fp

2

 2m p 
   f p   2m p  
 

 a  f p m1/p 2  180 MeV  6400 MeV   180  80  MeV 


1/ 2 3/ 2

2 2
  fp   15  2 3
  mp   160  2 4
      6.9  10 and       1.56  10
 fp   180   2m p   2  6400 

   MeV 
1/ 2
 a  180  80  MeV 
3/ 2 1/ 2 3/ 2
6.9  103  1.56  104   180  80  7  103

  a  1204  MeV 
3/ 2

NET/JRF (DEC-2016)
Q47. Which of the following circuits implements the Boolean function
F  A, B, C    1, 2, 4, 6  ?

C I0 C I0
(a) I1 4  1 (b) I1 4  1

I MUX
2
F I MUX
2
F

I 3 S1 S0 I 3 S1 S0

A B A B

C I0 0 I0
(c) I1 4  1 (d) I1 4  1
1
I MUX
2
F I MUX
2
F

I 3 S1 S0 C I 3 S1 S0

A B A B
Ans. : (b)
H.No. 40‐D, Ground Floor, Jia Sarai, Near IIT, Hauz Khas, New Delhi‐110016 
Phone: 011‐26865455/+91‐9871145498
Website: www.physicsbyfiziks.com  | Email: fiziks.physics@gmail.com  
24 
fiziks
Institute for NET/JRF, GATE, IIT‐JAM, M.Sc. Entrance, JEST, TIFR and GRE in Physics 
 
Solution:
A B C F
0 0 0 0
0 0 1 1 F C
0 1 0 1
0 1 1 0 F C
1 0 0 1
1 0 1 0 F C
1 1 0 1
1 1 1 0 F C
  aV  
Q48. The I  V characteristics of a device can be expressed as I  I s exp    1 , where T
  T  
is the temperature and a and I s are constants independent of T and V . Which one of

the following plots is correct for a fixed applied voltage V ?


3 2
(a) (b) 1
2
0
log I
log I

1 1
2
0 3 3
1 2 3 0 1 2
aV / T aV / T

2 4
1 3
(c) (d)
2
0 1
log I

log I

1 0
1
2
2
3 3 3
0 1 2 0 1 2 3
aV / T aV / T
Ans. : (d)
av
Solution: Let  x For large x ; I  I s e x  log e I  log e Is  x  log e I  x
T

H.No. 40‐D, Ground Floor, Jia Sarai, Near IIT, Hauz Khas, New Delhi‐110016 
Phone: 011‐26865455/+91‐9871145498
Website: www.physicsbyfiziks.com  | Email: fiziks.physics@gmail.com  
25 
fiziks
Institute for NET/JRF, GATE, IIT‐JAM, M.Sc. Entrance, JEST, TIFR and GRE in Physics 
 
Q49. The active medium in a blue LED (light emitting diode) is a Gax In1 x N alloy. The band

gaps of GaN and InN are 3.5 eV and 1.5 eV respectively. If the band gap of Gax In1 x N

varies approximately linearly with x , the value of x required for the emission of blue
light of wavelength 400 nm is (take hc  1200 eV -nm )
(a) 0.95 (b) 0.75 (c) 0.50 (d) 0.33
Ans. : (b)
Solution: EgGaN  3.5eV and EgInN  1.5eV E  eV 

Band Gap energy of Gax In1 x N is E  x . 3.5


3.0
For blue light of wavelength 400nm , the band gap
1.5
hc 1200 eV .nm
energy is    3.0 eV .
 400nm x
0 x? 1
 3.5 1.5   3.0 1.5 
Thus equating slopes we get;     2x  1.5  x  0.75
 1 0   x  0 
Q50. In the circuit below, the input voltage Vi is 2V ,Vcc  16 V , R2  2k  and RL  10 k 
VCC


Vi  R1 RL
R2

The value of R1 required to deliver 10 mW of power across RL is

(a) 12k (b) 4k (c) 8k (d) 14 k


Ans. : (c)
0  vi vi  v0 v0  0 
Solution: Apply kCL ; I 2  I1  I L    Vo
R2 R1 RL 
Vi I1
2
Vi
v R1 RL
pL  0
 10mW  v0  10V I2 R2
RL IL
0  2 2  10 10V 8
    1   R1  8 k 
2 R1 10k R1

H.No. 40‐D, Ground Floor, Jia Sarai, Near IIT, Hauz Khas, New Delhi‐110016 
Phone: 011‐26865455/+91‐9871145498
Website: www.physicsbyfiziks.com  | Email: fiziks.physics@gmail.com  
26 
fiziks
Institute for NET/JRF, GATE, IIT‐JAM, M.Sc. Entrance, JEST, TIFR and GRE in Physics 
 
Q51. Two sinusoidal signals are sent to an analog multiplier of scale factor 1V 1 followed by a
low pass filter (LPF).

V1  5cos 100t 

LPF
Multiplier fC  5Hz
Vout

V2  20 cos 100t   / 3

If the roll-off frequency of the LPF is f c  5 Hz , the output voltage Vout is

(a) 5V (b) 25V (c) 100 V (d) 50V


Ans. : (b)
Solution: After multiplying
  1     
5cos 100t   20 cos 100t    1V 1  100  cos  200t    cos  
 3 2  3  3 

    1
 50 cos  200t    
  3  2
1
After pass L.P.F. v0  50   25V
2
Q52. The resistance of a sample is measured as a function of temperature, and the data are
shown below.
T  0C 2 4 6 8

R  90 105 110 115

The slope of R vs T graph, using a linear least-squares fit to the data, will be
6 4 2 8
(a) 0
(b) 0
(c) 0
(d) 0
C C C C
Ans. : (b)

H.No. 40‐D, Ground Floor, Jia Sarai, Near IIT, Hauz Khas, New Delhi‐110016 
Phone: 011‐26865455/+91‐9871145498
Website: www.physicsbyfiziks.com  | Email: fiziks.physics@gmail.com  
27 
fiziks
Institute for NET/JRF, GATE, IIT‐JAM, M.Sc. Entrance, JEST, TIFR and GRE in Physics 
 
NET/JRF (JUNE-2017)
Q53. In the n -channel JFET shown in figure below, Vi  2V , C  10 pF , VDD  16 V and

RD  2k  . VDD
RD
D C
VO

Vi
G
S

If the drain D - source S saturation current I DSS is 10 mA and the pinch-off voltage VP is

8V , then the voltage across points D and S is


(a) 11.125 V (b) 10.375 V (c) 5.75 V (d) 4.75 V
Ans. : (d)
Solution: VGSQ  VGG  2V
z z
 V   2 
I DQ  I DSS 1  GS   10mA 1    5.63mA
 VP   8 

VDS  VDD  I D RD  16  5.63  z  4.8V

1
Q54. The gain of the circuit given below is  .
 RC
C

R V
Vin 
a Vout
 b
V

ground
The modification in the circuit required to introduce a dc feedback is to add a resistor
(a) between a and b
(b) between positive terminal of the op-amp and ground
(c) in series with C
(d) parallel to C
H.No. 40‐D, Ground Floor, Jia Sarai, Near IIT, Hauz Khas, New Delhi‐110016 
Phone: 011‐26865455/+91‐9871145498
Website: www.physicsbyfiziks.com  | Email: fiziks.physics@gmail.com  
28 
fiziks
Institute for NET/JRF, GATE, IIT‐JAM, M.Sc. Entrance, JEST, TIFR and GRE in Physics 
 
Ans. : (d)
Q55. A 2  4 decoder with an enable input can function as a
(a) 4 1 multiplexer (b) 1 4 demultiplexer
(c) 4  2 encoder (d) 4  2 priority encoder
Ans. : (b)
Q56. The experimentally measured values of the variables x and y are 2.00  0.05 and
3.00  0.02 respectively. What is the error in the calculated value of z  3 y  2 x from the
measurements?
(a) 0.12 (b) 0.05 (c) 0.03 (d) 0.07
Ans. : (a)
Solution: z  3 y  2 x
2 2
 z   z 
     y2     x2  9 y2  4 x2  0.12
2
z
 y   x 
Q57. Let I 0 be the saturation current,  the ideality factor and vF and vR the forward and

reverse potentials respectively, for a diode. The ratio RR / RF of its reverse and forward

resistances RR and RF , respectively, varies as (In the following k B is the Boltzmann

constant, T is the absolute temperature and q is the charge.)

vR  qv  vF  qv 
(a) exp  F  (b) exp  F 
vF   k BT  vR   k BT 

vR  qv  vF  qv 
(c) exp   F  (d) exp   F 
vF   k BT  vR   k BT 
Ans. : (a)
KT
Solution: I  I 0  eV /VT  1 , VT 
q
RR VR / I R VR I F
  
RF VF / I F VF I R

RR VR I 0 eVF /VT VR  qV 
   exp  F 
RF VF I0 VF  KT 

H.No. 40‐D, Ground Floor, Jia Sarai, Near IIT, Hauz Khas, New Delhi‐110016 
Phone: 011‐26865455/+91‐9871145498
Website: www.physicsbyfiziks.com  | Email: fiziks.physics@gmail.com  
29 
fiziks
Institute for NET/JRF, GATE, IIT‐JAM, M.Sc. Entrance, JEST, TIFR and GRE in Physics 
 
Q58. In the figures below, X and Y are one bit inputs. The circuit which corresponds to a one
bit comparator is

X
X Y
(a) Y
X Y

X Y

X
X Y
(b)
X Y

Y X Y

X
(c) X Y

X Y

Y X Y

X
X Y
(d)
Y
X Y

Y X Y

Ans. : (c)
Solution: (a) 01  XY , 02  XY , 03  0

(b). 01  XY , 02  XY , 03  Y

(c) 01  XY , 02  XY , 03  XY  XY  X  Y (equality comparator )

(d) 01  XY , 02  X  Y , 03  XY

H.No. 40‐D, Ground Floor, Jia Sarai, Near IIT, Hauz Khas, New Delhi‐110016 
Phone: 011‐26865455/+91‐9871145498
Website: www.physicsbyfiziks.com  | Email: fiziks.physics@gmail.com  
30 
fiziks
Institute for NET/JRF, GATE, IIT‐JAM, M.Sc. Entrance, JEST, TIFR and GRE in Physics 
 
Q59. Both the data points and a linear fit to the current vs voltage of a resistor are shown in the
graph below. 1

I (amps)
0 V  volts  25

If the error in the slope is 1.255  103  1 , then the value of resistance estimated from the
graph is
(a)  0.04  0.8   (b)  25.0  0.8  

(c)  25  1.25   (d)  25  0.0125  

Ans. : (b)
I max  I min 1 0 1
Solution: Slope     m (let)
Vmax  Vmin 25  0 25
V 1 R 1
I   mV  R   25 where  2
R m m m
2
 R  2 1 2
Error in R is  R2    m  4 m  R m
4 2

 m  m

  R  R 2 m   25  1.255 103  0.8  R   25.0  0.8  


2

Q60. In the following operational amplifier circuit Cin  10 nF , Rin  20 k , RF  200 k  and
RF
CF  100 pF .
CF

Ri Ci

Vin
Vout

The magnitude of the gain at a input signal frequency of 16 kHz is


(a) 67 (b) 0.15 (c) 0.3 (d) 3.5
H.No. 40‐D, Ground Floor, Jia Sarai, Near IIT, Hauz Khas, New Delhi‐110016 
Phone: 011‐26865455/+91‐9871145498
Website: www.physicsbyfiziks.com  | Email: fiziks.physics@gmail.com  
31 
fiziks
Institute for NET/JRF, GATE, IIT‐JAM, M.Sc. Entrance, JEST, TIFR and GRE in Physics 
 
Ans. : (d)
1 1
RF  / RF 
Vo z RF  X CF J  cF J  cF
Solution:  F  
Vin zi Ri  X Ci  1 
 Ri  
 J  ci 

Vo  RF /  J  cF RF  1  RF j ci
  
Vin  jci Ri  1 / jci  jcF RF  1 1  j Ri ci 
Vo  ci RF
  ,   2 f
Vin 1   cF RF  1   Ri ci 
2 2


V0

 2  16 103 10 109  200  103 
Vln 1  4 2 16 103   200 103  100  1012  1  4 2 16  103   20  103  10  109 
2 2 2 2 2 2

64
  4.96
20.12  20.12

H.No. 40‐D, Ground Floor, Jia Sarai, Near IIT, Hauz Khas, New Delhi‐110016 
Phone: 011‐26865455/+91‐9871145498
Website: www.physicsbyfiziks.com  | Email: fiziks.physics@gmail.com  
32 
fiziks
Institute for NET/JRF, GATE, IIT‐JAM, M.Sc. Entrance, JEST, TIFR and GRE in Physics 
 
ATOMIC AND MOLECULAR PHYSICS
NET/JRF (JUNE-2011)

Q1. Consider the energy level diagram (as shown in the figure below) of a typical three level
ruby laser system with 1.6  1019 Chromium ions per cubic centimeter. All the atoms
excited by the 0.4 μm radiation decay rapidly to level E2 , which has a lifetime  = 3 ms.
E3
0.4  m

E2
0.7  m
E1

A. Assuming that there is no radiation of wavelength 0.7 μm present in the pumping


cycle and that the pumping rate is R atoms per cm3, the population density in the level N2
builds up as:
(a) N2 (t) = R(et/ –1) (b) N2 (t) = R(1 – e-t/)

(c) N 2 t  
Rt 2

1  e 
t / 
(d) N2 (t) = R t

Ans: (b)
Solution: N 2  t   R 1  et / 

B. The minimum pump power required (per cubic centimeter) to bring the system to
transparency, i.e. zero gain, is
(a) 1.52 kW (b) 2.64 kW (c) 0.76 kW (d) 1.32 kW
Ans: (c)
Solution: The Minimum Power required to achieve zero gain is
N hv N hc 1.6  1019 6.6  10 34  3  10 8
P     754 W cm 3
2  2  2 6
0.7  10  3  10 3

P  0.76kW per cubic centimeter

H.No. 40‐D, Ground Floor, Jia Sarai, Near IIT, Hauz Khas, New Delhi‐110016 
Phone: 011‐26865455/+91‐9871145498
Website: www.physicsbyfiziks.com  | Email: fiziks.physics@gmail.com  

fiziks
Institute for NET/JRF, GATE, IIT‐JAM, M.Sc. Entrance, JEST, TIFR and GRE in Physics 
 
NET/JRF (DEC-2011)
Q2. Given that the ground state energy of the hydrogen atom is –13.6 eV, the ground state
energy of positronium (which is a bound state of an electron and a positron) is
(a) + 6.8 eV (b) – 6.8 eV (c) – 13.6 eV (d) – 27.2 eV
Ans: (b)
13.6
Solution: The energy expression for Positronium atom is E n   eV 
2n 2
 13.6
For n = 1, E1  eV   6.8eV ,  E1  6.8 eV
2
Q3. A laser operating at 500 nm is used to excite a molecule. If the Stokes line is observed at
770 cm-1, the approximate positions of the Stokes and the anti-Stokes lines are
(a) 481.5 nm and 520 nm (b) 481.5 nm and 500 nm
(c) 500 nm and 520 nm (d) 500 nm and 600 nm
Ans:
Solution: Given 0  500 nm  5  105 cm , v stoke  770 cm 1   0  20,000cm 1

Raman shift v  v 0  v stoke  19230 cm 1

Wave number of anti-stokes line is vanti -stoke  v  v0 = 39,230 cm-1

In wavelength term  anti  stoke  2.549  10 7  254.9 nm and  stoke  12987 nm

Q4. If the hyperfine interaction in an atom is given by H  a S e  S p where S e and S p denote

the electron and proton spins, respectively, the splitting between the 3 S1 and 1 S 0 state is

(a) a 2 / 2 (b) a 2 (c) a 2 / 2 (d) 2a 2


Ans: (b)
       1

Solution: Total spin is S  S e  S p  S 2  S e2  S p2  2 S e  S p  S e  S p  S 2  S e2  S p2
2

 
 
 H  aS e  S p  S  S e2  S p2 where S e2  S p2  S S  1 2   2
a 2
2
3
4
a  2 3 2 3 2  a  2 3 2
H   S       S   
2 4 4  2 2 

For 3 S1 : S  1  S 2  S S  1 2  2 2 , For 1 S 0 : S  0  S 2  S S  1 2  0 2

a 3 a a 3 3 2
 H1   2   2   2 for 3 S1 and H 2   0   2  a for 1 S 0
2 2 4 2 2 4

H.No. 40‐D, Ground Floor, Jia Sarai, Near IIT, Hauz Khas, New Delhi‐110016 
Phone: 011‐26865455/+91‐9871145498
Website: www.physicsbyfiziks.com  | Email: fiziks.physics@gmail.com  

fiziks
Institute for NET/JRF, GATE, IIT‐JAM, M.Sc. Entrance, JEST, TIFR and GRE in Physics 
 
1 3
 The splitting between 3 S1 and 1 S 0 is H  H 1  H 2    a 2  a 2
4 4
Q5. The ratio of intensities of the D1 and D2 lines of sodium at high temperature is
(a) 1:1 (b) 2:3 (c) 1:3 (d) 1:2
Ans: (d)
Solution: The electronic transition for D2 and D1 line is
3
2 1
I  D2  2 J 2  1 2 4 2
D2 : 2P3/ 2  2S1/ 2 , D1 : 2P1/ 2  2S1/ 2     
I D1  2 J 1  1 1 2 1
2 1
2
Q6. An atom of mass M can be excited to a state of mass M    by photon capture. The
frequency of a photon which can cause this transition is
c 2 c 2 2 c 2 c 2
(a) (b) (c) (d)   2M 
2h h 2Mh 2Mh
Ans: (d)
Solution: The conservation law of energy and momentum give


Mc 2  h  M    c 4  p 2 c 2
2
1/ 2
and
h
c
 p

M 2 c 4  h 2 2  2 Mc 2 h  M 2 c 4  2 c 4  2 Mc 4  h 2 2  2Mc 2 h  2 c 4  2Mc 4

   c 2    c 2
 2Mc h  2Mc 1 
2
  
4
1       2M  .
 2M  h  2M  2Mh

NET/JRF (JUNE-2012)
12
Q7. The first absorption spectrum of C16O is at 3.842 cm-1 while that of 13
C16O is at
3.673 cm-1. The ratio of their moments of inertia is
(a) 1.851 (b) 1.286 (c) 1.046 (d) 1.038
Ans: (c)
Solution: For 12 C 16O : 2 B1  3.842 cm 1  B1  1.921 cm 1

For 13
2 B 2  3.673 cm 1
C 16 O :  B 2  1.8365 cm 1
h I 2 B1 1.921
Where, B  2     1.046
8 IC I1 B2 1.8365

H.No. 40‐D, Ground Floor, Jia Sarai, Near IIT, Hauz Khas, New Delhi‐110016 
Phone: 011‐26865455/+91‐9871145498
Website: www.physicsbyfiziks.com  | Email: fiziks.physics@gmail.com  

fiziks
Institute for NET/JRF, GATE, IIT‐JAM, M.Sc. Entrance, JEST, TIFR and GRE in Physics 
 
Q8. The spin-orbit interaction in an atom is given by H  aL  S , where L and S denote the
orbital and spin angular momenta, respectively, of the electron. The splitting between the
levels 2P3/2 and 2P1/2 is
3 2 1 2 5 2
(a) a (b) a (c) 3a 2 2 (d) a
2 2 2
Ans. : (a)
    
Solution: Given H  aL  S where J  L  S .
  1
 
 a

 J 2  L2  S 2  2 L  S  L  S  J 2  L2  S 2  H  J 2  L2  S 2
2 2
 
1 3
For 2P3/ 2 : S  which gives S 2  S  S  1  2   2
2 4

L = 1 which gives L2  LL  1 2  2 2


3 15 a 15 3
which gives J 2  J  J  1 2   2  H1    2    2   2
a
J
2 4 2 4 4 2

1 3
For 2P1/ 2 : S  which gives S 2  S S  1 2   2
2 4
L = 1 which gives L2  LL  1 2  2 2

1 3 a 3 3 
J which gives J 2  J  J  1 2   2  H 2    2   2    a 2
2 4 2 4 4 
a 2 3
H  H1  H 2    a 2  a 2
2 2
Q9. The spectral line corresponding to an atomic transition from J = 1 to J = 0 states splits in
a magnetic field of 0.1 Tesla into three components separated by 1.6  10-3 Ǻ. If the zero
field spectral line corresponds to 1849 Ǻ, what is the g-factor corresponding to the J = 1
hc
state? (You may use  2  10 4 cm)
0
(a) 2 (b) 3/2 (c) 1 (d) 1/2
Ans: (c)
Solution: The Zeeman splitting is E   gM J  B B   g B B for MJ = +1

2
Given, Zeeman splitting separations,     1.6  10 3 Å
c

H.No. 40‐D, Ground Floor, Jia Sarai, Near IIT, Hauz Khas, New Delhi‐110016 
Phone: 011‐26865455/+91‐9871145498
Website: www.physicsbyfiziks.com  | Email: fiziks.physics@gmail.com  

fiziks
Institute for NET/JRF, GATE, IIT‐JAM, M.Sc. Entrance, JEST, TIFR and GRE in Physics 
 
c 3  10 8
   2    1.6  10 1  0.1404  1010
 
1849  10 10 2

E 6.625  10 34  0.1404  1010
g    1.00
B B 9.27  10  24  0.1

NET/JRF (DEC-2012)
Q10. Consider the energy level diagram shown below, which corresponds to the molecular
nitrogen laser. 2
R  21

1
1
0
20 -3 -1
If the pump rate R is 10 atoms cm s and the decay routes are as shown with
 21  20 ns and  1  1s , the equilibrium populations of states 2 and 1 are, respectively,
(a) 1014 cm-3 and 2  1012 cm-3 (b) 21012 cm-3 and 1014 cm-3.
(c) 21012 cm-3 and 2  10 6 cm-3 (d) zero and 1020 cm-3
Ans: (b)
dN 2 N dN1 N 2 N 1
Solution:  R  2 and   .
dt  21 dt  21  1
dN 2 dN1
Under equilibrium condition  0
dt dt
 N 2   21 R  1020  20  109  2  1012 cm3

 1 N 2 10 6  2  1012 cm 3
 N1    1014 cm 3
 21 20  10 9

Q11. Consider a hydrogen atom undergoing a 2 P  1S transition. The lifetime tsp of the 2P
state for spontaneous emission is 1.6 ns and the energy difference between the levels is
10.2 eV. Assuming that the refractive index of the medium n0 = 1, the ratio of Einstein
coefficients for stimulated and spontaneous emission B21   / A21   is given by
(a) 0.683 × 1012 m3J-1s-1 (b) 0.146 × 10-12 Jsm-3 .
(c) 6.83 × 1012 m3J-1s-1 (d) 1.463 × 10-12 Jsm-3 .

H.No. 40‐D, Ground Floor, Jia Sarai, Near IIT, Hauz Khas, New Delhi‐110016 
Phone: 011‐26865455/+91‐9871145498
Website: www.physicsbyfiziks.com  | Email: fiziks.physics@gmail.com  

fiziks
Institute for NET/JRF, GATE, IIT‐JAM, M.Sc. Entrance, JEST, TIFR and GRE in Physics 
 
Ans: (a)
B21  2 c 3  2 2 c 3
Solution: n 0  1 , E  10.2 eV and    0.67  1012 .
A21  3 n03  E 3 n03

Q12. Consider a He-Ne laser cavity consisting of two mirrors of reflectivities R1 = 1 and
R2 = 0.98. The mirrors are separated by a distance d = 20 cm and the medium in between
has a refractive index n0 = 1 and absorption coefficient α = 0. The values of the
separation between the modes  and the width  p of each mode of the laser cavity are:

(a)   75kHz,  p  24kHz (b)   100kHz,  p  100kHz

(c)   750MHz,  p  2.4 MHz (d)   2.4MHz,  p  750MHz

Ans: (c)
c
Solution: Mode separation    750 MHz
2dn0

where c  3  10 8 m / sec , d  20  102 m and n0  1 .

1 2n 0 d
Width of each mode  p  where t c  .
2t c  1 
c ln  2d

 R1 R2 e 
Note: In this question, there is no need to calculate  p . Since in the given options there

is only one option with   750 MHz and i.e., option (c). You can calculate  p without

calculator so use your common sense.

NET/JRF (JUNE-2013)
Q13. A muon  
 from cosmic rays is trapped by a proton to form a hydrogen-like atom.
Given that a muon is approximately 200 times heavier than an electron, the longest
wavelength of the spectral line (in the analogue of the Lyman series) of such an atom will
be
o o o o
(a) 5.62 A (b) 6.67 A (c) 3.75 A (d) 13.3 A
Ans: (b)
m m p
Solution: In case of muonic atom, the reduce mass is m '   180me
m  m p

H.No. 40‐D, Ground Floor, Jia Sarai, Near IIT, Hauz Khas, New Delhi‐110016 
Phone: 011‐26865455/+91‐9871145498
Website: www.physicsbyfiziks.com  | Email: fiziks.physics@gmail.com  

fiziks
Institute for NET/JRF, GATE, IIT‐JAM, M.Sc. Entrance, JEST, TIFR and GRE in Physics 
 
m E
'
E
E n'    21  180 21 where, E1  13.6eV
 me  n n

For ground state of muonic atom n=1, E1'  180 E1

For first excited state of muonic atom n=2, E 2'  45 E1


The longest wavelength of the photon corresponds to the transition between first and ground state
of muonic atom.
The energy difference between first excited and ground state is
E  E 2'  E1'  135E1  1836eV  2938  10 19 J
In term of wavelength
hc hc 6.63  10 34  3  10 8 o
 
10
E  hv    6. 67  10 m  6.67 A
 E 2938  10 19
Q14. Consider the hydrogen-deuterium molecule HD. If the mean distance between the two
atoms is 0.08 nm and the mass of the hydrogen atom is 938 MeV / c 2 , then the energy
difference E between the two lowest rotational states is approximately
(a) 10 1 eV (b) 10 2 eV (c) 2  10 2 eV (d) 10 3 eV
Ans: (b)
h2
Solution: Rotational energy expression E  J  J  1  AJ  J  1
8 2 I
Difference between two lowest energy levels is E  2 A
h2 2
where A   and I  r 2
8 I
2
2I
MHMD M  2M H 2 2
Here,    H  MH ,    938 MeV/c2
M H  M D M H  2M H 3 3
1
  1.01  1034  J -sec   1.01  1034   1019  eV -sec   6.3  1016  eV -sec 
1.6

 E 
2


6.3  10 16 eV  sec 2

 9.2  103 eV  102 eV


I 2
3

 938  10 6 eV / c 2  0.08  10 9 m 2

Q15. The electronic energy levels in a hydrogen atom are given by E n  13.6 / n 2 eV. If a

selective excitation to the n  100 level is to be made using a laser, the maximum
allowed frequency line-width of the laser is
(a) 6.5 MHz (b) 6.5 GHz (c) 6.5 Hz (d) 6.5 kHz
H.No. 40‐D, Ground Floor, Jia Sarai, Near IIT, Hauz Khas, New Delhi‐110016 
Phone: 011‐26865455/+91‐9871145498
Website: www.physicsbyfiziks.com  | Email: fiziks.physics@gmail.com  

fiziks
Institute for NET/JRF, GATE, IIT‐JAM, M.Sc. Entrance, JEST, TIFR and GRE in Physics 
 
Ans: (b)
2  13.6 2  13.6
Solution: E n  13.6 / n 2  E n  3
eV  h   1.6  10 19    6.5 GHz .
n n3
Q16. Consider the laser resonator cavity shown in the figure.
If I1 is the intensity of the radiation at R1  1 R2  R
l
mirror M 1 and  is the gain coefficient of the
medium between the mirrors, then the energy
x
density of photons in the plane P at a
P
distance x from M 1 is M1 M2

(a)  I1 / c  e  x (b)  I1 / c  e x

(c)  I1 / c   e x  e x  (d)  I1 / c  e2 x


Ans: (c)
NET/JRF (DEC-2013)
Q17. A perturbation V pert  aL2 is added to the Hydrogen atom potential. The shift in the

energy level of the 2 P state, when the effects of spin are neglected up to second order in
a , is
3 2 4
(a) 0 (b) 2a 2  a 2  4 (c) 2a 2 (d) a 2  a 
2
Ans: (c)
Solution: For 2 P state, L=1
The shift in the energy due to perturbation V pert  aL2 is

E  aL( L  1) 2  a1(1  1) 2  2a 2


Q18. A gas laser cavity has been designed to operate at   0.5 m with a cavity length of 1 m .
With this set-up, the frequency is found to be larger than the desired frequency by 100 Hz.
The change in the effective length of the cavity required to retune the laser is
(a)  0.334  10 12 m (b) 0.334  10 12 m (c) 0.167  10 12 m (d)  0.167  10 12 m
Ans: (d)
c c  c  c
Solution:   m    m 2 L   2 L  2 L   L  L
2L 2L c 2L L L
 L 0.5  106  1  100
 L     0.167  1012 m
c 3  108
H.No. 40‐D, Ground Floor, Jia Sarai, Near IIT, Hauz Khas, New Delhi‐110016 
Phone: 011‐26865455/+91‐9871145498
Website: www.physicsbyfiziks.com  | Email: fiziks.physics@gmail.com  

fiziks
Institute for NET/JRF, GATE, IIT‐JAM, M.Sc. Entrance, JEST, TIFR and GRE in Physics 
 
Q19. The spectroscopic symbol for the ground state of 13 Al is 2 P1/ 2 . Under the action of a

strong magnetic field (when L  S coupling can be neglected) the ground state energy
level will split into
(a) 3 levels (b) 4 levels (c) 5 levels (d) 6 levels
Ans: (c)
Solution: In extremely strong magnetic field coupling between L-S breaks down. J is no longer a
valid quantum number.
The ground state energy level will have  2S  1 2 L  1  6 states, but 2 states are

degenerate. Therefore, total energy levels will be 5 .

NET/JRF (JUNE-2014)
Q20. A spectral line due to a transition from an electronic state p to an s state splits into three
Zeeman lines in the presence of a strong magnetic field. At intermediate field strengths
the number of spectral lines is
(a) 10 (b) 3 (c) 6 (d) 9
Ans: (a)
Solution: For p state: l  1, s  1/ 2 : j  1/ 2 & 3 / 2 .
This gives two spectral terms 2P3/2 & 2P1/2
For s state: l= 0, s = 1/2: j = 1/2 : This gives spectral terms 2S1/2
The terms 2P3/2 and 2S1/2 corresponding to J = 3/2 & J = 1/2 will break into 2J+1 Zeeman
levels, which is 4 and 2 respectively.
For 2P3/2 Mj = - 3/2 -1/2 +1/2 +3/2
For 2S1/2 Mj = -1/2 +1/2
The selection rule is ∆MJ = 0, ± 1 (MJ = 0 → MJ = 0 If ∆J = 0)
∆MJ = 0 gives π component, ∆MJ = ±1 gives σ component
Number of π component = 2, Numbers of + components = 2
Number of - components = 2
The terms 2P1/2 and 2S1/2 corresponding to J = 1/2 & J = 1/2 will break into 2J+1 Zeeman
levels, which is 2 & 2 respectively.
For 2P1/2 Mj = -1/2 +1/2, For 2S1/2 Mj = -1/2 +1/2

H.No. 40‐D, Ground Floor, Jia Sarai, Near IIT, Hauz Khas, New Delhi‐110016 
Phone: 011‐26865455/+91‐9871145498
Website: www.physicsbyfiziks.com  | Email: fiziks.physics@gmail.com  

fiziks
Institute for NET/JRF, GATE, IIT‐JAM, M.Sc. Entrance, JEST, TIFR and GRE in Physics 
 
The selection rule is ∆MJ = 0, ± 1 (MJ = 0 → MJ = 0 If ∆J = 0)
∆MJ = 0 gives π component, ∆MJ = ±1 gives σ component
Number of π component = 2, Numbers of + components = 1
Number of - components = 1
Thus, total number of Zeeman component = 10
Q21. A double slit interference experiment uses a laser emitting light of two adjacent
frequencies v1 and v 2 v1  v 2  . The minimum path difference between the interfering
beams for which the interference pattern disappears is
c c c c
(a) (b) (c) (d)
v 2  v1 v 2  v1 2v 2  v1  2v 2  v1 
Ans: (c)
Solution: The condition of maximum intensity for interfering laser beam is: d sin   n
 1
The condition of dark intensity for interfering laser beam is: d sin    n  
 2
For interference pattern to vanish, the minimum path difference should be /2
c c 2
The spectral bandwidth of laser is defined as, v  2  
 v 
For two closely spaced line of wavelength 1 and 2
c 2  1 c c
  1 2   
v  1  2 1 1

c c  2  1

2 1 2 1
Since, for interference pattern to vanish for two closely spaced line of wavelength 1 and
 c c
2, the minimum path difference should be =  
2 2v 2 2   1 

Q22. How much does the total angular momentum quantum number J change in the transition
of Cr 3d 6  atom as it ionize to Cr 2 3d 4 ?
(a) Increases by 2 (b) Decreases by 2 (c) Decreases by 4 (d) Does not change
Ans: (c)

 
Solution: In Cr 3d 6 state

M L  2  1 0  1  2

H.No. 40‐D, Ground Floor, Jia Sarai, Near IIT, Hauz Khas, New Delhi‐110016 
Phone: 011‐26865455/+91‐9871145498
Website: www.physicsbyfiziks.com  | Email: fiziks.physics@gmail.com  
10 
fiziks
Institute for NET/JRF, GATE, IIT‐JAM, M.Sc. Entrance, JEST, TIFR and GRE in Physics 
 
1
In this configuration, S  4   2 and L  2
2
This is the case of more than half filled subshell, thus state with highest J value will
have the lowest energy.
The highest J - value is J  L  S  y

Thus, the ground state spectral term for this configuration is 5 D4 .

 
Now in Cr 2 3d 4
M L  2  1 0  1  2

1
In this configuration, S  4   2 and L  2
2
Since this is the case of less than half filled subshell, thus, state with lowest J value will
have the lowest energy.
The lowest J - value is J  L  5  2  2  0 .
Thus the ground state spectral term for this configuration is the J - value decreases from
J  4 to J  0 .
Thus correct answer is option (c).
NET/JRF (DEC-2014)
Q23. An atomic transition 1 P  1S in a magnetic field 1 Tesla shows Zeeman splitting. Given
that the Bohr magneton  B  9.27  10 24 J / T , and the wavelength corresponding to the
transition is 250 nm, the separation in the Zeeman spectral lines is approximately
(a) 0.01 nm (b) 0.1 nm (c) 1.0 nm (d) 10 nm
Ans: (a)
Solution: This is the case of Normal Zeeman effect. The Zeeman separation in terms of
 B
frequency,   B , where  B is Bohr magneton
h
2  2 B B
In terms of wavelength,     
c c h

 250 10 m    9.27 10 J / T  1T   3 10


2
9 24

  12
m  0.003 nm
 3 10 m / s  6.625 10 Js 
8 34

None of the answer is matching correctly. But best suitable answer is option (a)
H.No. 40‐D, Ground Floor, Jia Sarai, Near IIT, Hauz Khas, New Delhi‐110016 
Phone: 011‐26865455/+91‐9871145498
Website: www.physicsbyfiziks.com  | Email: fiziks.physics@gmail.com  
11 
fiziks
Institute for NET/JRF, GATE, IIT‐JAM, M.Sc. Entrance, JEST, TIFR and GRE in Physics 
 
Q24. If the leading anharmonic correction to the energy of n th vibrational level of a diatomic
2
 1
molecule is  xe  n    with xe  0.001 , the total number of energy levels possible
 2
is approximately
(a) 500 (b) 1000 (c) 250 (d) 750
Ans: (a)
2
 1  1
Solution: The energy of anharmonic oscillator is Ev   v     xe  v   
 2  2
where v  0, 1, 2, .....vmax is vibrational quantum number v  vmax
dEv  1 
Now,  0    2 xe  vmax     0
dv v  vmax
 2 E

 1 1 1 1 1
 1  2 xe  vmax    vmax      500
 2  2 xe 2 2 xe 2  0.001 
 
Q25. The effective spin-spin interaction between the electron spin S e and the proton spin S p
 
in the ground state of the Hydrogen atom is given by H   aS e  S p . As a result of this

interaction, the energy levels split by an amount


1 2 3 2
(a) a (b) 2a 2 (c) a 2 (d) a
2 2
Ans: (c)
 
Solution: The Hamiltonian is given as H  aSe  S p where Se and S p are electron and proton spin.

The coupling between Se and S p gives net resultant spin angular momentum
       1
S  Se  S p , S 2  Se2  S p2  2Se  S p  Se  S p   S 2  Se2  S p2 
2 Sp
S
 H
2

a 2
S  Se2  S p2 
Se
where S 2  S  S  1  2 , Se2  Se  Se  1  2 , S p2  S p  S p  1  2 F 1
1 2
a
1 1 4
Since Se  and S p   S  0, 1 12 s1/2
2 2 3
 a 2
4
F 0
H.No. 40‐D, Ground Floor, Jia Sarai, Near IIT, Hauz Khas, New Delhi‐110016 
Phone: 011‐26865455/+91‐9871145498
Website: www.physicsbyfiziks.com  | Email: fiziks.physics@gmail.com  
12 
fiziks
Institute for NET/JRF, GATE, IIT‐JAM, M.Sc. Entrance, JEST, TIFR and GRE in Physics 
 
a 3 3 3
For S  0 (singlet state), H1   0     2   a 2
2 4 4 4
a 3 2 3 2 1 2
For S  1 (Triplet state), H 2   2       a
2 4 4  4
1 2 3 2
 H  H 2  H1  a  a  H  a 2
4 4

NET/JRF (JUNE-2015)
Q26. Of the following term symbols of the np 2 atomic configurations, 1 S0 ,3 P0 ,3 P1 ,3 P2 and
1
D2 which is the ground state?
(a) 3 P0 (b) 1 S0 (c) 3 P2 (d) 3 P1
Ans. (a)
Solution: According to Hund’s rules
(i) State with highest multiplicity has lowest energy
(ii) State with same multiplicity, the state with highest L will have lowest energy
(iii) State with same multiplicity and L value. The state with lowest J has lowest energy
(only if subshell is less than half filled) from the given states 1 S0 ,3 P0 ,3 P1 ,3 P21 D2

The 3 P0 will have the lowest energy


 1
Q27. A diatomic molecule has vibrational states with energies Ev    v   and rotational
 2

states with energies E j  Bj  j  1 , where v and j are non-negative integers . consider

the transitions in which both the initial and final states are restricted to v  1 and j  2
and subject to the selection rules v  1 and j  1 . Then the largest allowed energy
of transition is
(a)   3B (b)   B (c)   4 B (d) 2  B
Ans. (c)
 1
Solution: E    v    BJ  J  1
 2
For vibrational transition with v  1 and rotational transition with J   we get
E  Einitial  Efinal    2 B  J  1

H.No. 40‐D, Ground Floor, Jia Sarai, Near IIT, Hauz Khas, New Delhi‐110016 
Phone: 011‐26865455/+91‐9871145498
Website: www.physicsbyfiziks.com  | Email: fiziks.physics@gmail.com  
13 
fiziks
Institute for NET/JRF, GATE, IIT‐JAM, M.Sc. Entrance, JEST, TIFR and GRE in Physics 
 
where J is lowest quantum number
According to question in rotational states, two transition is possible, one for
J  0  J  1 and second for J  1  J  2
But, second transition will have photon of higher energy
 For J  1  J  2
E    2 B 1  1    4 B

NET/JRF (DEC-2015)
Q28. The LS configurations of the ground state of 12Mg , 13
Al , 17Cl and 18
Ar are respectively,

(a) 3 S1 , 2 P1/ 2 , 2 P1/ 2 and 1 S0 (b) 3 S1 , 2 P3 / 2 , 2 P3 / 2 and 3 S1

(c) 1 S0 , 2 P1/ 2 , 2 P3 / 2 and 1 S0 (d) 1 S0 , 2 P3 / 2 , 2 P1/ 2 and 3 S1


Ans.: (c)
Solution: 12 Mg :1s 2 2s 2 2 p 6 3s 2

Ground state: 1 S0
13
Al :1s 2 2s 2 2 p 6 3s 2 3 p1
the terms are 2 p1/ 2 and 2 p3/ 2 . Since its less than half filled. Thus

Ground state: 2 p1/ 2


17
Cl :1s 2 2s 2 2 p 6 3s 2 3 p 5
The terms are 2 p1/ 2 and 2 p3/ 2 . Since it shell is more than half filled.

Thus Ground state: 2 p3/ 2


18
Ar :1s 2 2s 2 2 p 6 3s 2 3 p 6

Ground state: 1 S0

Q29. For a two level system, the population of atoms in the upper and lower levels are 3 1018
and 0.7 1018 , respectively. If the coefficient of stimulated emission is
3.0  105 m3 / W -s 3 and the energy density is 9.0 J / m3 -Hz , the rate of stimulated
emission will be
(a) 6.3 1016 s 1 (b) 4.1  1016 s 1 (c) 2.7  1016 s 1 (d) 1.8  1016 s 1
Ans.: None of the answer is matching.

H.No. 40‐D, Ground Floor, Jia Sarai, Near IIT, Hauz Khas, New Delhi‐110016 
Phone: 011‐26865455/+91‐9871145498
Website: www.physicsbyfiziks.com  | Email: fiziks.physics@gmail.com  
14 
fiziks
Institute for NET/JRF, GATE, IIT‐JAM, M.Sc. Entrance, JEST, TIFR and GRE in Physics 
 
Solution: The rate of stimulated emission is
dN 2
 N 2 B21 u  w 
dt
m3 m3
where N 2  3 1018 , B21  3  105  3  10 5

W  s3 J  s2
J s
and u  w   9.0 J / m3  Hz  9.0
m3
dN 2 m3 J s
 3 1018  3 105  9 3  8.11024 s 1
dt J s 2
m
Q30. The first ionization potential of K is 4.34 eV , the electron affinity of Cl is 3.82 eV
and the equilibrium separation of KCl is 0.3 nm . Then energy required to dissociate a
KCl molecule into a K and a Cl atom is
(a) 8.62 eV (b) 8.16 eV (c) 4.28 eV (d) 4.14 eV
Ans.: (c)
Solution: Energy required to dissociate KCl is  KCl  K   Cl  

 
2
1 q1q2  9 Nm 
2 1.6  1019 c
V   9  10 2 
 7.7  1019 J  4.79 eV
4 0 r12  c  0.3 10 m9

The band dissociation energy is the energy required to dissociate a molecule into its
component atom KCl  K  Cl
To find the energy required to dissociate KCl into K and Cl , we must add an electron
to the K  ion, which releases the atomic potassium ionization energy. Remove one
electron from Cl  ion which requires the atomic chlorine electron affinity energy
Given ionization energy of K  Eie  4.34 eV

Electron affinity of Cl  Eai  3.82 eV


Thus the energy gained in the transformation from ion to atom is
 Eie  Eai  4.34  3.82  0.52 eV

Thus dissociation energy  4.79  0.52  4.27 eV

H.No. 40‐D, Ground Floor, Jia Sarai, Near IIT, Hauz Khas, New Delhi‐110016 
Phone: 011‐26865455/+91‐9871145498
Website: www.physicsbyfiziks.com  | Email: fiziks.physics@gmail.com  
15 
fiziks
Institute for NET/JRF, GATE, IIT‐JAM, M.Sc. Entrance, JEST, TIFR and GRE in Physics 
 
NET/JRF (JUNE-2016)
Q31. The ground state electronic configuration of 22 Ti is  Ar  3d 2 4s 2 . Which state, in the

standard spectroscopic notations, is not possible in this configuration?


(a) 1 F3 (b) 1 S0 (c) 1 D2 (d) 3 P0
Ans: (a)
Solution: The spectroscopic term for d 2 are obtained as
l1  2, l2  2  L  4,3, 2,1, 0

1 1
s1  , s2   S  0,1
2 2
Now, S  0, L  4  J  4, 1G4

S  0, L  2  J  2, 1 D2

S  0, L  0  J  0, 1S0

S  1, L  3  J  4,3, 2, 3
F4 , 3F3 , 3F2

S  1, L  1  J  2,1, 0, 3 P2 , 3 P1 ,3 P0
Thus 1 F3 is not possible spectroscopic term of  Ar  3d 2 4 S 2

Q32. In a normal Zeeman Effect experiment using a magnetic field of strength 0.3 T , the
splitting between the components of a 660 nm spectral line is
(a) 12 pm (b) 10 pm (c) 8 pm (d) 6 pm
Ans: (d)
 2 eB  660 10 
2
9
1.6  1019  0.3
Solution:      6.09  1012 m  6 pm
c 4 m 3  108 4  9.110 31

Q33. The separation between the energy levels of a two-level atom is 2 eV . Suppose that

4  1020 atoms are in the ground state and 7  1020 atoms are pumped into the excited state
just before lasing starts. How much energy will be released in a single laser pulse?
(a) 24.6 J (b) 22.4 J (c) 98 J (d) 48 J
Ans: (d)
Solution: N 2  N1  3  1020

 N  N1  3  1020
Energy of laser pulse, E   2  h   2 1.6 1019 J  E  48 J
 2  2
H.No. 40‐D, Ground Floor, Jia Sarai, Near IIT, Hauz Khas, New Delhi‐110016 
Phone: 011‐26865455/+91‐9871145498
Website: www.physicsbyfiziks.com  | Email: fiziks.physics@gmail.com  
16 
fiziks
Institute for NET/JRF, GATE, IIT‐JAM, M.Sc. Entrance, JEST, TIFR and GRE in Physics 
 
NET/JRF (DEC-2016)
Q34. In the L  S coupling scheme, the terms arising from two non-equivalent p -electrons are

(a) 3S , 1P, 3P, 1D, 3D (b) 1S , 3S , 1P, 1D

(c) 1S , 3S , 3P, 3D (d) 1S , 3S , 1P, 3P, 1D, 3D


Ans. : (d)
Solution: For pp configuration
1 1
s1  , s2   S  0,1
2 2
l1  1, l2  1  L  0,1, 2
Thus
S  0, L  0  J  0 ; 1S0 or 1S

S  0, L  1  J  1 ; 1 P1 or 1 P

S  0, L  2  J  2 ; 1 D2 or 1 D

S  1, L  0  J  1 ; 3 S1 or 3 S

S  1, L  1  J  0,1, 2 ; 3 P0,1,2 or 3 P

S  1, L  2  J  1, 2,3 ; 3 D1,2,3 or 3 D

Thus spectroscopic terms are 1 S ,3 S ,1 P,3 P 1 D, 3 D


Q35. The total spin of a hydrogen atom is due to the contribution of the spins of the electron
and the proton. In the high temperature limit, the ratio of the number of atoms in the spin-
1 state to the number in the spin- 0 state is
1 1
(a) 2 (b) 3 (c) (d)
2 3
Ans. : (b)
Solution: F  1 (when spins are parallel)
2
S1/ 2
F  0 (when spins are anti-parallel)

The degeneracy of quantum level is 2 F  1 .


2 F1  1 3
Thus,  3
2 F0  1 1

H.No. 40‐D, Ground Floor, Jia Sarai, Near IIT, Hauz Khas, New Delhi‐110016 
Phone: 011‐26865455/+91‐9871145498
Website: www.physicsbyfiziks.com  | Email: fiziks.physics@gmail.com  
17 
fiziks
Institute for NET/JRF, GATE, IIT‐JAM, M.Sc. Entrance, JEST, TIFR and GRE in Physics 
 
Q36. A two level system in a thermal (black body) environment can decay from the excited
state by both spontaneous and thermally stimulated emission. At room temperature
 300 K  , the frequency below which thermal emission dominates over spontaneous

emission is nearest to
(a) 1013 Hz (b) 108 Hz (c) 105 Hz (d) 1011 Hz
Ans. : (d)
Solution: At thermal equilibrium, the ratio of the number of spontaneous to stimulated emission
is given by

A21  1.054  1034 J .S
 e kT  1 ; where   2.551014 sec
B21 u   23
kT 1.38  10 J / K  300 K

Now, for v  1013 Hz    6.3 1013 rad / sec ,


A21
 e1.6  1  5  1  4
B21u  
For v  1011 Hz    6.3 1011 rad / sec ,
A21 3
  e1.610  1  1.03  1  0.03
B21u  
This ratio is less than one, thus stimulated thermal emission dominates over spontaneous
emission below all frequency of 1011 Hz
NET/JRF (JUNE-2017)
Q37. An atomic spectral line is observed to split into nine components due to Zeeman shift. If
the upper state of the atom is 3D2 then the lower state will be

(a) 3F2 (b) 3F1 (c) 3P1 (d) 3P2


Ans. : (c)
Solution: The possible zeeman component for different transitions are:
(i) 3D2  3 F2 : Total zeeman components are 12

(ii) 3D2  3P1 : Total zeeman components are 9

(iii) 3D2  3P2 : Total zeeman components are 12


3
F1 state is not allowed because for 3F , possible ‘ J ’ values are 2, 3 and 4.

Only 3 D2  3 P1 transition gives 9 components. Therefore, ground state is 3P1 .


Thus, correct option is (c).
H.No. 40‐D, Ground Floor, Jia Sarai, Near IIT, Hauz Khas, New Delhi‐110016 
Phone: 011‐26865455/+91‐9871145498
Website: www.physicsbyfiziks.com  | Email: fiziks.physics@gmail.com  
18 
fiziks
Institute for NET/JRF, GATE, IIT‐JAM, M.Sc. Entrance, JEST, TIFR and GRE in Physics 
 
Q38. If the coefficient of stimulated emission for a particular transition is 2.1 1019 m3W 1s 3
0
and the emitted photon is at wavelength 3000 A , then the lifetime of the excited state is
approximately
(a) 20 ns (b) 40 ns (c) 80 ns (d) 100 ns
Ans. : (c)
A21 8 h 3 1 c3 3
Solution:  . Life time     
B21 c3 A21 8 h 3 B21 8 hB21

 
 3000 10  10 3

8  6.6 10 J .s. 2.1 10


34 19
m3 w1s 3 

2.7  1020
  7.7 108 sec  80n sec
3.5  1013
Q39. If the binding energies of the electron in the K and L shells of silver atom are 25.4 keV
and 3.34 keV , respectively, then the kinetic energy of the Auger electron will be
approximately
(a) 22 keV (b) 9.3keV (c) 10.5 keV (d) 18.7 keV
Ans. : (d)
Solution: K .E. Auger electron is K .E   EK  EL   EL

 EK  2 EL Auger electorn
e
  25.4  2  3.34  keV
Vacuum level
 18.7keV
L

h

H.No. 40‐D, Ground Floor, Jia Sarai, Near IIT, Hauz Khas, New Delhi‐110016 
Phone: 011‐26865455/+91‐9871145498
Website: www.physicsbyfiziks.com  | Email: fiziks.physics@gmail.com  
19 
fiziks
Institute for NET/JRF, GATE, IIT‐JAM, M.Sc. Entrance, JEST, TIFR and GRE in Physics 
 
CONDENSED MATTER PHYSICS
NET/JRF (JUNE-2011)
Q1. A narrow beam of X-rays with wavelength 1.5 Ǻ is reflected from an ionic crystal with an
fcc lattice structure with a density of 3.32 gcm-3. The molecular weight is 108 AMU
(1AMU = 1.66 × 10-24 g)
A. The lattice constant is
(a) 6.00 Ǻ (b) 4.56 Ǻ (c) 4.00 Ǻ (d) 2.56 Ǻ
Ans: (a)
Solution: Given ne f f  4 , M = 108 kg,   3.32gm cm -3 = 3320 kgm-3,

N A  6.023  10 26 atoms kmd

ne f f  M 4  108
a3    6.00  10 30 m 3  6.00  10 10  6.00 A 0
NA   6.023  10 26  3320
B. The sine of the angle corresponding to (111) reflection is
3 3 1 1
(a) (b) (c) (d)
4 8 4 8
Ans: (b)
Solution: According to Bragg’s law
 a a
2d sin    , sin   where d   for (111) plane
2d h2  k 2  l 2 3

3 3  1.5 A0 3 3 3
 sin      .
2a 2 6A 0
2 6 2 8
Q2. A flux quantum (fluxoid) is approximately equal to 210-7 gauss-cm2. A type II
superconductor is placed in a small magnetic field, which is then slowly increased till the
field starts penetrating the superconductor. The strength of the field at this point is
2
 105 gauss.

A. The penetrating depth of this superconductor is
(a) 100 Ǻ (b) 10Ǻ (c) 1000Ǻ (d) 314Ǻ
Ans: (a)
Solution: Given Fluxoid  0  2  10 7 gauss –cm2

H.No. 40‐D, Ground Floor, Jia Sarai, Near IIT, Hauz Khas, New Delhi‐110016 
Phone: 011‐26865455/+91‐9871145498
Website: www.physicsbyfiziks.com  | Email: fiziks.physics@gmail.com  

fiziks
Institute for NET/JRF, GATE, IIT‐JAM, M.Sc. Entrance, JEST, TIFR and GRE in Physics 
 
2
First Critical field H c1    10 5 gauss

The relation between first critical field and penetration depth is
0 0 2.10 7
H c1    2
   10 12 cm 2    10 6 cm  100 A 0
2
H c1   2  10 5

B. The applied field is further increased till superconductivity is completely destroyed.
8
The strength of the field is now  105 gauss. The correlation length of the

superconductor is
(a) 20 Ǻ (b) 200 Ǻ (c) 628 Ǻ (d) 2000 Ǻ
Ans: None of the options is matched.
8
Solution: Given second critical field H c 2    10 5 gauss. The relation between second critical

0
field and correlation length is H c 2  2.

0 2  10 7 1 1 100
  2
   10 12 cm 2     106 cm   1010 m  50 A 0
H c 2 8
   10 5 4
2 2

Q3. The two dimensional lattice of graphene is an arrangement of Carbon atoms forming a
honeycomb lattice of lattice spacing a, as shown below. The Carbon atoms occupy the
vertices.
1

c1
1

(A). The Wigner-Seitz cell has an area of d2


1

c2
1

3 2
1

a1
1 1

(a) 2a 2 (b) a d1 b1
2
1

a2 b2
a
2 3 3 2
(c) 6 3a (d) a
2
Ans: (d)
 
Solution: Primitive lattice vectors are b1 and b2

b1  3a cos 300 iˆ  3a cos 600 ˆj 
2
3
a  3iˆ  ˆj 
   3 3 2
b2 
2
3
a  
3i  ˆj ,  A  b2  b1 
2
a

H.No. 40‐D, Ground Floor, Jia Sarai, Near IIT, Hauz Khas, New Delhi‐110016 
Phone: 011‐26865455/+91‐9871145498
Website: www.physicsbyfiziks.com  | Email: fiziks.physics@gmail.com  

fiziks
Institute for NET/JRF, GATE, IIT‐JAM, M.Sc. Entrance, JEST, TIFR and GRE in Physics 
 
(B). The Bravais lattice for this array is a
 
(a) rectangular lattice with basis vectors d1 and d 2
 
(b) rectangular lattice with basis vectors c1 and c2
 
(c) hexagonal lattice with basis vectors a1 and a2
 
(d) hexagonal lattice with basis vectors b1 and b2

Ans: (c)
 
Solution: The Bravaiss lattice for this array is the Hexagonal lattice with basis vectors a1 and a 2

NET/JRF (DEC-2011)
Q4. The potential of a diatomic molecule as a function of the distance r between the atoms is
a b
given by V r    6
 12 . The value of the potential at equilibrium separation between
r r
the atoms is:
(a)  4a 2 / b (b)  2a 2 / b (c)  a 2 / 2b (d)  a 2 / 4b
Ans: (d)
a b dV r 
Solution: Given V r    6
 12 . At equilibrium radius, 0
r r dr r r0

dV r  6a 12b r 13 12b 2b 2b
   7  13  0  07    r06 
dr r0 r0 r0 6a a a

a b a2 a2  a2
 The value of potential at equilibrium is V r0         .
r06 r012 2b 4b 4b
Q5. If the number density of a free electron gas in three dimensions is increased eight times,
its Fermi temperature will
(a) increase by a factor of 4 (b) decrease by a factor of 4
(c) increase by a factor of 8 (d) decrease by a factor of 8
Ans: (a)
Solution: The relation between Fermi energy and electron density is E F 
2
2m

3 2 n 
2/3
.

 E F' 
2
2m

3 2  8n 2/3
 4EF .

H.No. 40‐D, Ground Floor, Jia Sarai, Near IIT, Hauz Khas, New Delhi‐110016 
Phone: 011‐26865455/+91‐9871145498
Website: www.physicsbyfiziks.com  | Email: fiziks.physics@gmail.com  

fiziks
Institute for NET/JRF, GATE, IIT‐JAM, M.Sc. Entrance, JEST, TIFR and GRE in Physics 
 
Q6. The excitations of a three-dimensional solid are bosonic in nature with their frequency ω
and wave-number k are related by ω  k2 in the large wavelength limit. If the chemical
potential is zero, the behaviour of the specific heat of the system at low temperature is
proportional to
(a) T1/2 (b) T (c) T3/2 (d) T3
Ans: (c)
Solution: If the dispersion relation is   k s in large wavelength. Then the specific heat is
C v  T 3 / s . Given   k 2  C v  T 3 / 2

NET/JRF (JUNE-2012)
Q7. Consider a system of non-interacting particles in d dimensional obeying the dispersion
relation   Ak s , where ε is the energy, k is the wavevector, s is an integer and A is
constant. The density of states, N(ε), is proportional to
s d d s
1 1 1 1
(a)  d
(b)  s
(c)  s
(d)  d

Ans: (b)
Q8. The experimentally measured transmission spectra of metal, insulator and semiconductor
thin films are shown in the figure. It can be inferred that I, II and III correspond,
respectively, to

 
Transmission (%)

(a) insulator, semiconductor and metal


(b) semiconductor, metal and insulator
(c) metal, semiconductor and insulator
(d) insulator, metal and semiconductor 

Ans: (a) 300 1000


Wavelength (nm)

Q9. The energy required to create a lattice vacancy in a crystal is equal to 1 eV. The ratio of
the number densities of vacancies n(1200 K)/n(300 K) when the crystal is at equilibrium
at 1200 K and 300 K, respectively, is approximately
(a) exp 30  (b) exp 15 (c) exp15 (d) exp30
Ans: (d)

H.No. 40‐D, Ground Floor, Jia Sarai, Near IIT, Hauz Khas, New Delhi‐110016 
Phone: 011‐26865455/+91‐9871145498
Website: www.physicsbyfiziks.com  | Email: fiziks.physics@gmail.com  

fiziks
Institute for NET/JRF, GATE, IIT‐JAM, M.Sc. Entrance, JEST, TIFR and GRE in Physics 
 
Solution: The equation for number density of vacancies n  Ne  E / 2 k BT where E: Energy required
to form vacancies, N: density of lattice sites
E  1 1
n1200K 
E  1 1  E  1 
n e  E / 2 k BT1   
2k T T     
 1   E / 2 k BT2  e B  2 1  ,  e 2 k B  300 1200   e 2 k B  400   e 30
n2 e n300 K 
Q10. The dispersion relation of phonons in a solid is given by
 2 k    02 3  cos k x a  cos k y a  cos k z a 
The velocity of the phonons at large wavelength is
(a)  0 a / 3 (b)  0 a (c) 3 0 a (d)  0 a / 2
Ans: (d)
Solution: For large  , k x a, k y a, k z a  are small.
    kya
2 2
k x2 a 2   k z2 a 2   02 a 2 2
 2 k    02 3  1 
  1 
  2
2
  1 
  2
 
2

k x  k y2  k z2 
      
 2a2 a d  0 a
 2 k   0 k 2    0 k  v g   .
2 2 dk 2

NET/JRF (DEC-2012)
Q11. A magnetic field sensor based on the Hall Effect is to be fabricated by implanting As into
a Si film of thickness 1 μm. The specifications require a magnetic field sensitivity of
500 mV/Tesla at an excitation current of 1 mA. The implantation dose is to be adjusted
such that the average carrier density, after activation, is
(a) 1.25 × 1026 m-3 (b) 1.25 × 1022 m-3
(c) 4.1 × 1021 m-3 (d) 4.1 × 1020 m-3
Ans: (b)
IB 10 3 1 V
Solution: n   6 19
 3
 1.25  10 22 m 3 where H  500  10 3 V / T .
teVH 10  1.6  10 500  10 B
Q12. In a band structure calculation, the dispersion relation for electrons is found to be
 k   cos k x a  cos k y a  cos k z a ,
where β is a constant and a is the lattice constant. The effective mass at the boundary of
the first Brillouin zone is
2 2 4 2 2 2
(a) (b) (c) (d)
5 a 2 5 a 2 2 a 2 3 a 2

H.No. 40‐D, Ground Floor, Jia Sarai, Near IIT, Hauz Khas, New Delhi‐110016 
Phone: 011‐26865455/+91‐9871145498
Website: www.physicsbyfiziks.com  | Email: fiziks.physics@gmail.com  

fiziks
Institute for NET/JRF, GATE, IIT‐JAM, M.Sc. Entrance, JEST, TIFR and GRE in Physics 
 
Ans: (d)
2
Solution:  k   cos k x a  cos k y a  cos k z a , Effective mass m  
 d 2 k 
 2 
 d k 
 
  
Brilliouin zone boundary is at k x   ,ky   , kz   .
a a a
 d 2  2
Hence  2 k   3 a 2  m   .
 d k   , , 3 a 2
a a a

Q13. The radius of the Fermi sphere of free electrons in a monovalent metal with an fcc
structure, in which the volume of the unit cell is a3, is
1/ 3 1/ 3 1/ 3
 12 2   3 2   2  1
(a)  3  (b)  3  (c)  3  (d)
 a   a  a  a

Ans: (a)
1/ 3
 3 2 N   2    2 k F2 
 
2/3
Solution: Radius of Fermi sphere is k F    , EF    3 2
n   
 V   2m   2m 
1/ 3
N 4  12 2 
For fcc solid   k F   3  .
V a3  a 

NET/JRF (JUNE-2013)

Q14. Using the frequency-dependent Drude formula, what is the effective kinetic inductance of
a metallic wire that is to be used as a transmission line? [In the following, the electron
mass is m , density of electrons is n , and the length and cross-sectional area of the wire
 and A respectively.]
(a) mA / ne 2   (b) zero (c) m / ne 2 A 
(d) m A / ne 2  2 
Ans: (c)
Q15. The phonon dispersion for the following one-dimensional diatomic lattice with masses
M 1 and M 2 (as shown in the figure)
K

M1 M2 M1 M2
H.No. 40‐D, Ground Floor, Jia Sarai, Near IIT, Hauz Khas, New Delhi‐110016 
Phone: 011‐26865455/+91‐9871145498
Website: www.physicsbyfiziks.com  | Email: fiziks.physics@gmail.com  

fiziks
Institute for NET/JRF, GATE, IIT‐JAM, M.Sc. Entrance, JEST, TIFR and GRE in Physics 
 
is given by

 1 1  4M 1 M 2  qa  
 2 q   K    1  1  sin 2   
 M 1 M 2   M 1  M 2  2
 2  

where a is the lattice parameter and K is the spring constant. The velocity of sound is

K M 1  M 2  K
(a) a (b) a
2M 1 M 2 2M 1  M 2 

K M 1  M 2  KM 1 M 2
(c) a (d) a
2M 1  M 2 
3
M 1M 2

Ans: (b)
Solution: For small value of q (i.e. long wavelength approximation limit).

 qa  qa
We have sin   
 2  2

 1 1  4M 1M 2  qa  
2 q      1  1  sin 2   
 M1  M 2 
2
 M1 M 2    2  

1  4M 1 M 2  qa  
2
 1
  q    
2
 
 1  1  2   
M
 1 M 2  

 M 1  M 2   2  

 1 1    1 4 M 1 M 2 q 2 a 2 
  2 q       1  1  
 2 M  M 2 4 

M
 1 M 2    1 2 

 1 1    M 1 M 2 q 2 a 2 
  q    
2
  1  1 
 M  M 2 2  

M
 1 M 2    1 2 

 1 1    M 1 M 2 q 2 a 2 
For Acoustical branch:  2 q       1  1 
 M  M 2 2 

M
 1 M 2    1 2 

 M  M2  M 1 M 2 q 2 a 2  a 2
  2 q     1   
 M  M 2 2  2M  M  q
2

 M 1M 2  1 2  1 2


  q   aq
2M 1  M 2 

 
Velocity of sound is v g   a
q 2M 1  M 2 
H.No. 40‐D, Ground Floor, Jia Sarai, Near IIT, Hauz Khas, New Delhi‐110016 
Phone: 011‐26865455/+91‐9871145498
Website: www.physicsbyfiziks.com  | Email: fiziks.physics@gmail.com  

fiziks
Institute for NET/JRF, GATE, IIT‐JAM, M.Sc. Entrance, JEST, TIFR and GRE in Physics 
 
Q16. The electron dispersion relation for a one-dimensional metal is given by
 ka 1 2 
 k  2 0 sin 2  sin ka 
 2 6 
where k is the momentum, a is the lattice constant,  0 is a constant having dimensions of

energy and ka   . If the average number of electrons per atom in the conduction band

is 1/3, then the Fermi energy is


(a)  0 / 4 (b)  0 (c) 2 0 / 3 (d) 5 0 / 3
Ans: (a)
Q17. If the energy dispersion of a two-dimensional electron system is E  uk where u is the
velocity and k is the momentum, then the density of states DE  depends on the energy
as
(a) 1 / E (b) E (c) E (d) constant
Ans: (c)
Solution: In two dimensional system, the number of allowed k-states in range k and k  dk is
2
 L 
g k dk    2kdk .
 2 
E dE
Given dispersion relation is E  uk  k   dk 
u u
2 2
 L  E dE  L  2
 g E dE    2     EdE
 2   2  u 
2
u u

g E dE 1 L2
  E    E.
dE u 2 2
NET/JRF (DEC-2013)
Q18. The physical phenomenon that cannot be used for memory storage applications is
(a) large variation in magnetoresistance as a function of applied magnetic field
(b) variation in magnetization of a ferromagnet as a function of applied magnetic field
(c) variation in polarization of a ferroelectric as a function of applied electric field
(d) variation in resistance of a metal as a function of applied electric field

Ans: (d)

H.No. 40‐D, Ground Floor, Jia Sarai, Near IIT, Hauz Khas, New Delhi‐110016 
Phone: 011‐26865455/+91‐9871145498
Website: www.physicsbyfiziks.com  | Email: fiziks.physics@gmail.com  

fiziks
Institute for NET/JRF, GATE, IIT‐JAM, M.Sc. Entrance, JEST, TIFR and GRE in Physics 
 
Q19. The energy of an electron in a band as a function of its wave vector k is given
by E k   E 0  B cos k x a  cos k y a  cos k z a  , where E0 , B and a are constants. The

effective mass of the electron near the bottom of the band is


2 2 2 2 2
(a) (b) (c) (d)
3Ba 2 3Ba 2 2Ba 2 Ba 2
Ans: (d)
Solution: Near the bottom of the band the k  0
1 1 1
 k x a  , cos k y a  1   k y a  , cos k z a  1   kz a 
2 2 2
cos k x a  1 
2 2 2
 1 1 1 2
E  k   E0  B  cos k x a  cos k y a  cos k z a   E0  B 1   k x a   1   k y a   1   k z a  
2 2

 2 2 2 
 1 2 1
 E0  B  3  a 2  k x  k x  k x    E0  3B  Ba 2 k 2
 2  2
2
 2
Effective mass of the electron is m *  2 
d E / dk 2 Ba 2
Q20. A DC voltage V is applied across a Josephson junction between two superconductors
with a phase difference 0 . If I 0 and k are constants that depend on the properties of the
junction, the current flowing through it has the form
 2eVt   2eVt 
(a) I 0 sin   0  (b) kV sin   0 
     
(c) kV sin  0 (d) I 0 sin  0  kV
Ans: (a)
Q21. A uniform linear monoatomic chain is modeled by a spring-mass system of masses m
separated by nearest neighbour distance a and spring constant m 02 . The dispersion
relation for this system is
  ka    ka 
(a)  k   2 0 1  cos   (b)  k   2 0 sin 2  
  2   2 
 ka   ka 
(c)  k   2 0 sin   (d)  k   2 0 tan  
 2   2
Ans: (c)
Solution: The dispersion relation for uniform linear mono-atomic chain of atoms is
 ka 
 k   2 0 sin  
 2 
H.No. 40‐D, Ground Floor, Jia Sarai, Near IIT, Hauz Khas, New Delhi‐110016 
Phone: 011‐26865455/+91‐9871145498
Website: www.physicsbyfiziks.com  | Email: fiziks.physics@gmail.com  

fiziks
Institute for NET/JRF, GATE, IIT‐JAM, M.Sc. Entrance, JEST, TIFR and GRE in Physics 
 
NET/JRF (JUNE-2014)
Q22. The pressure of a nonrelativistic free Fermi gas in three-dimensions depends, at T  0 ,
on the density of fermions n as
(a) n 5 / 3 (b) n1 / 3 (c) n 2 / 3 (d) n 4 / 3
Ans: (a)
Solution: The Fermi energy in three dimension is defined as
2/3
 2  3 2 N 
EF  
2m  V 
 
2
2m

3 2 n 2/3

Where, n is the electron concentration or density of free Fermi gas.


The total energy of free Fermi gas in 3D is
2/3
3 3  2  3 2 N 
E  NE F  N   
5 5 2m  V 

The pressure of a nonrelativistic free Fermi gas is defined as


 E 
p F  
3
  N
2
3 2 N     V 5 / 3
2/3  2
 V  N 5 2m  3

2
5
2
 nEF  n 
5
2
2m
3 n  
2 2/3 2 2
5 2m
3 2  n 5 / 3
2/3

Q23. Consider an electron in b.c.c. lattice with lattice constant a . A single particle


wavefunction that satisfies the Bloch theorem will have the form f r  exp ik .r , with  

f r  being

 2   2   2 
(a) 1  cos   x  y  z   cos   x  y  z   cos   x  y  z 
 a   a   a 
 2   2   2 
(b) 1  cos   x  y   cos   y  z   cos   z  x 
 a   a   a 
     
(c) 1  cos   x  y   cos   y  z   cos   z  x 
a  a  a 
     
(d) 1  cos   x  y  z   cos   x  y  z   cos   x  y  z 
a  a  a 
Ans: (b)
Solution: The primitive translational vector for BCC is
H.No. 40‐D, Ground Floor, Jia Sarai, Near IIT, Hauz Khas, New Delhi‐110016 
Phone: 011‐26865455/+91‐9871145498
Website: www.physicsbyfiziks.com  | Email: fiziks.physics@gmail.com  
10 
fiziks
Institute for NET/JRF, GATE, IIT‐JAM, M.Sc. Entrance, JEST, TIFR and GRE in Physics 
 

 a
  a
  a
 
a '  iˆ  ˆj  kˆ , b '  iˆ  ˆj  kˆ , c '  iˆ  ˆj  kˆ
2 2 2

Bloch function defined as
  
 
 k r   u k r e ik .r  f r e ik .r

Here f r  is atomic wavefunction, which has the periodicity of the lattice i.e.
 
u k r  a   u k r 
Given Bloch function
  2   2   2 
f (r )  1  cos   x  y    cos   y  z    cos   z  x  
 a   a   a 
   2  a a   2  a a   2  a a 
f (r  a ' )  1  cos   x  y     cos   y  z     cos   z  x   
 a  2 2   a  2 2   a  2 2 

   2   2   2 
f (r  a ' )  1  cos   x  y   cos   y  z   2   cos   z  x 
 a   a   a 

   2   2   2  
f (r  a ' )  1  cos   x  y   cos   y  z   cos   z  x   f (r )
 a   a   a 
  
f (r  a ' )  f (r )
Similarly,
     
f (r  b ' )  f (r ) and f ( r  c ' )  f ( r )
Other functions do not satisfy the periodicity
Q24. The dispersion relation for electrons in an f.c.c. crystal is given, in the tight binding
approximation, by
 kxa kya kya k a k a k a
 k   4 0 cos cos  cos cos z  cos z cos x 
 2 2 2 2 2 2 

where a is the lattice constant and  0 is a constant with the dimension of energy. The x -

 
component of the velocity of the electron at  , 0, 0  is
a 
(a)  2 0 a /  (b) 2 0 a /  (c)  4 0 a /  (d) 4 0 a / 

H.No. 40‐D, Ground Floor, Jia Sarai, Near IIT, Hauz Khas, New Delhi‐110016 
Phone: 011‐26865455/+91‐9871145498
Website: www.physicsbyfiziks.com  | Email: fiziks.physics@gmail.com  
11 
fiziks
Institute for NET/JRF, GATE, IIT‐JAM, M.Sc. Entrance, JEST, TIFR and GRE in Physics 
 
Ans: (d)
Solution: Group velocity of electron in dispersive medium is expressed as

 1 d 1  d ˆ d ˆ d    
v   i j kˆ   v x iˆ  v y ˆj  v z kˆ
 dk   dk x dk y dk z 
 kx a kya ka k a  ka kya kya k a 
 sin cos  cos z sin x  iˆ   cos x sin  sin cos z  ˆj  
 2 a  2 2 2 2   2 2 2 2  
v 0  

  sin k z a cos y  cos k x a sin k z a  kˆ
k a

  2 2 2 2  

 
At  , 0, 0 
a 
 2 a         
v  0  sin cos 0  cos 0sin  iˆ   cos sin 0  sin 0cos 0  ˆj   cos 0sin 0  sin 0cos  kˆ 
  2 2  2   2 

 4 a   
v  0 iˆ  0 ˆj  0kˆ    0iˆ  0 ˆj  0kˆ   vx iˆ  v y ˆj  vz kˆ
    

 4 a  
vx  0 , v y  0, vz  0

4 0 a
The x - component of velocity is vx 

NET/JRF (DEC-2014)
Q25. When laser light of wavelength  falls on a metal scale with 1 mm engravings at a
grazing angle of incidence, it is diffracted to form a vertical chain of diffraction spots on
a screen kept perpendicular to the scale. If the wavelength of the laser is increased by 200
nm, the angle of the first-order diffraction spot changes from 5 0 to
(a) 6.60 0 (b) 5.14 0 (c) 5.018 0 (d) 5.210
Ans: (c)
Solution: The condition of maxima peak in grating is
b sin   m ; m  0,1, 2,3,....
where b is the width of slit or width of engraving, whereas ‘ m ’ is the order of
diffraction and  is the angle of diffraction
For 1st order diffraction: b sin    (i)

H.No. 40‐D, Ground Floor, Jia Sarai, Near IIT, Hauz Khas, New Delhi‐110016 
Phone: 011‐26865455/+91‐9871145498
Website: www.physicsbyfiziks.com  | Email: fiziks.physics@gmail.com  
12 
fiziks
Institute for NET/JRF, GATE, IIT‐JAM, M.Sc. Entrance, JEST, TIFR and GRE in Physics 
 
When wavelength of incident light increased to    200 nm , let’s assume the 1st order

peak appears at    b sin      200 (ii)


Subtracting equation (i) from equation (ii), we get
b sin   b sin    200
200 200  109
 sin    sin    sin     sin 
b 10 3
 sin    2  10 4  sin   2  10 4  sin 50  2  10 4  0.087196  0.08736
    sin 1  0.08736     5.010
Q26. Consider the crystal structure of sodium chloride which is modeled as a set of touching
spheres. Each sodium atom has a radius r1 and each chlorine atom has a radius r2 . The
centres of the spheres from a simple cubic lattice. The packing fraction of this system is
 r 3
  r2 
3
 2 r13  r23
(a)   1      (b)
 r1  r2   r1  r2   3 r1  r2 3

r13  r23 r13  r23


(c) (d) 
r1  r2 3 2r1  r2 
3

Ans: (b)
Solution: This question can only be solved by solving each option by assuming r1  r2 and

comparing result with the packing fraction of simple cubic which is .
6
  r 3  r 3    1 3  1 3  
Option (a):   1
 
2
          
 r1  r2   r1  r2    2   2   4
2 r13  r23 2 2r 3 2 1 
Option (b):     
3  r1  r2 3 3 8r 3 3 4 6
r13  r23 2r 3 1
Option (c):  3 
 r1  r2 
3
8r 4
 r13  r23 2r 3 
Option (d):  
2  r1  r2 3 2  8r 3
8

Thus, correct option is (b)

H.No. 40‐D, Ground Floor, Jia Sarai, Near IIT, Hauz Khas, New Delhi‐110016 
Phone: 011‐26865455/+91‐9871145498
Website: www.physicsbyfiziks.com  | Email: fiziks.physics@gmail.com  
13 
fiziks
Institute for NET/JRF, GATE, IIT‐JAM, M.Sc. Entrance, JEST, TIFR and GRE in Physics 
 
Q27. Consider two crystalline solids, one of which has a simple cubic structure, and the other
has a tetragonal structure. The effective spring constant between atoms in the c -direction
is half the effective spring constant between atoms in the a and b directions. At low
temperatures, the behaviour of the lattice contribution to the specific heat will depend as
a function of temperature T as
(a) T 2 for the tetragonal solid, but as T 3 for the simple cubic solid
(b) T for the tetragonal solid, and as T 3 for the simple cubic solid
(c) T for both solids
(d) T 3 for both solids
Ans: (d)
Solution: The specific heat of solid in three dimensions is proportional to T 3 and it is
independent of crystal structure.
In 3D : CV  T 3

In 2D : CV  T 2

In 1D : CV  T

Q28. A superconducting ring carries a steady current in the presence of a magnetic field B
normal to the plane of the ring. Identify the incorrect statement.
(a) The flux passing through the superconductor is quantized in units of hc / e
(b) The current and the magnetic field in the superconductor are time independent.
   
(c) The current density j and B are related by the equation   j  2 B  0 , where 
is a constant
(d) The superconductor shows an energy gap which is proportional to the transition
temperature of the superconductor
Ans: (a)
Solution: The flux quantization in superconducting ring is   no

hc h
where o  in CGS units and o  in MKS units.
2e 2e

H.No. 40‐D, Ground Floor, Jia Sarai, Near IIT, Hauz Khas, New Delhi‐110016 
Phone: 011‐26865455/+91‐9871145498
Website: www.physicsbyfiziks.com  | Email: fiziks.physics@gmail.com  
14 
fiziks
Institute for NET/JRF, GATE, IIT‐JAM, M.Sc. Entrance, JEST, TIFR and GRE in Physics 
 
NET/JRF (JUNE-2015)
Q29. X -ray of wavelength   a is reflected from the 111 plane of a simple cubic lattice. If

the lattice constant is a , the corresponding Bragg angle (in radian) is


   
(a) (b) (c) (d)
6 4 3 8
Ans. (c)
Solution: According to Bragg’s Law 2d sin   
a a a
where d    for 111 plane
h k l
2 2 2
111 3

 a 3  3 
 sin        sin 1   
2d a 2  2  3
2
3
Q30. The critical magnetic fields of a superconductor at temperatures 4 K and 8 K are
11 mA / m and 5.5 mA / m respectively. The transition temperature is approximately
(a) 8.4 K (b) 10.6 K (c) 12.9 K (d) 15.0 K
Ans. (b)
Solution: The relation between critical field and critical temperature is
  T 2 
H C T   H 0 1    
  TC  

Let at T  T1 , H C T1  , T  T2 , H C T   H C T2 

  T 2    T 2 
Thus we get H C T1   H 0 1     , H C T2   H 0 1   2  
1

  TC     TC  

H C T1  2
2
T 
1  1  T2  T12
H C T1  C  2 2 8   4 
2 2
T H T
  C  T   TC   10.6
H C T2  T 
2 C
H C T1  2 1
1  2  1
H C T2 
 TC
where T1  4 k , T2  8 k , H C T1   11 mA / m and H C T2   5.5 mA / m

H.No. 40‐D, Ground Floor, Jia Sarai, Near IIT, Hauz Khas, New Delhi‐110016 
Phone: 011‐26865455/+91‐9871145498
Website: www.physicsbyfiziks.com  | Email: fiziks.physics@gmail.com  
15 
fiziks
Institute for NET/JRF, GATE, IIT‐JAM, M.Sc. Entrance, JEST, TIFR and GRE in Physics 
 
Q31. The low-energy electronic excitations in a two-dimensional sheet of grapheme is given

 
by E k  vk , where v is the velocity of the excitations. The density of states is

proportional to
3 1
(a) E (b) E 2 (c) E 2 (d) E 2
Ans. (a)
Solution: The number of k - states in range k and k  dk in two dimension is
2
 L 
g  k  dk    2 kdk
 2 
2 2
 L  E dE  L  2
 E   k  dE   dk  g  E  dE    2     EdE
 2     2    
2

The density of state is


g  E  dE
2
 L  2
 E    E   E  E
 2    
2
dE
Q32. A He  Ne laser operates by using two energy levels of Ne separated by 2.26 eV .
Under steady state conditions of optical pumping, the equivalent temperature of the
system at which the ratio of the number of atoms in the upper state to that in the lower
1
state will be , is approximately (the Boltzmann constant k B  8.6 105 eV / K )
20
(a) 1010 K (b) 108 K (c) 106 K (d) 104 K
Ans. (d)
Solution: According to Boltzmann relation
E2 N2
N2  E  N1  kT  E E
 exp     exp  T 
N1  kT  N2  E  N  E1 N1
k ln  2 
 N1 
N1 2.26
E  2.26 eV , k B  8.6 105 eV / K ,  20  T   0.877  104 K
N2 5  1 
8.6  10 ln  
20
 

 T  104 K

H.No. 40‐D, Ground Floor, Jia Sarai, Near IIT, Hauz Khas, New Delhi‐110016 
Phone: 011‐26865455/+91‐9871145498
Website: www.physicsbyfiziks.com  | Email: fiziks.physics@gmail.com  
16 
fiziks
Institute for NET/JRF, GATE, IIT‐JAM, M.Sc. Entrance, JEST, TIFR and GRE in Physics 
 
NET/JRF (DEC-2015)
Q33. The first order diffraction peak of a crystalline solid occurs at a scattering angle of 300
when the diffraction pattern is recorded using an x-ray beam of wavelength 0.15 nm . If

the error in measurements of the wavelength and the angle are 0.01 nm and 10
respectively, then the error in calculating the inter-planar spacing will approximately be
(a) 1.1102 nm (b) 1.3  104 nm (c) 2.5  102 nm (d) 2.0  103 nm
Ans.: (a)
 d 1 d  cos 
Solution: Bragg’s Law for n  1,   2d sin   d    , 
2sin   2sin   2sin 2 
Error in d can be calculated as
2 2 2
 d   d  2  1  2   cos   2
     2  
2
        
      2sin    2sin  
d 2

2 2 2
 d2 1  2sin   2   cos    2sin   2
 2           
d  4sin     
2
 2sin  sin     
1

 2
2
 2        2 2 2
 d
  2   d  d        
d 2
 2
tan      tan   

where   30o ,   1.5  1010 m,    0.11010 m,    1o

 1.5 1010
d   1.5  1010 m
2sin  2sin 30o
1
    
2 2 1
 0.11010   2
    3  
2 2

  180    1.5  1010  0.067   


2
Thus,  d  1.5 1010  10   
 1.5 10   tan 30     180  
     
  
1 1
 1.5  10  0.067    0.03  2  1.5 1010  0.005389 2
102 2

 
 d  1.5 10  0.0734  0.111010  1.11011 m  1.1102 nm
10

H.No. 40‐D, Ground Floor, Jia Sarai, Near IIT, Hauz Khas, New Delhi‐110016 
Phone: 011‐26865455/+91‐9871145498
Website: www.physicsbyfiziks.com  | Email: fiziks.physics@gmail.com  
17 
fiziks
Institute for NET/JRF, GATE, IIT‐JAM, M.Sc. Entrance, JEST, TIFR and GRE in Physics 
 
Q34. The dispersion relation of electrons in a 3-dimensional lattice in the tight binding
approximation is given by,
 k   cos k x a   cos k y a   cos k z a
where a is the lattice constant and  ,  ,  are constants with dimension of energy. The

   
effective mass tensor at the corner of the first Brillouin zone  , ,  is
a a a

 1/  0 0   1/  0 0 
2   2  
(a) 2  0 1/  0  (b) 2  0 1/  0 
a  a 
 0 0 1/    0 0 1/  

 1/  0 0  1/  0 0 
2   2  
(c) 2  0 1/  0  (d) 2  0 1/  0 
a  a 
 0 0 1/    0 0 1/  

Ans.: (c)
Solution: The effective mass as a tensor quantity can be written as
 m*xx m*xy m*xz 
 *  2
mij*   m*yx m*yy m yz  where mij 
*

 2E 
 m*zx m*zy m*zz   

 ki k j 
since k   cos k x a   cos k y a   cos k z a

2  2 2 2
 m*xx   , m*yy  
 2    a 2 cos k x a  2    a 2 cos k y a
   2 
 k x k x   k y 

2 2
m  2
*
 , other terms are zero
     a 2 cos k z a
zz

 2 
 k z 

1/  0 0 
    * 2 2 2 2 
Now, at  , ,  ; mxx  , m yy 
*
, mzz  2  mij  2  0 1/ 
* *
0 
a a a a 2
a 2
a a
 0 0 1/  

H.No. 40‐D, Ground Floor, Jia Sarai, Near IIT, Hauz Khas, New Delhi‐110016 
Phone: 011‐26865455/+91‐9871145498
Website: www.physicsbyfiziks.com  | Email: fiziks.physics@gmail.com  
18 
fiziks
Institute for NET/JRF, GATE, IIT‐JAM, M.Sc. Entrance, JEST, TIFR and GRE in Physics 
 
Q35. A thin metal film of dimension 2 mm  2 mm contains 4  1012 electrons. The magnitude
of the Fermi wavevector of the system, in the free electron approximation, is
(a) 2   107 cm 1 (b) 2  107 cm 1 (c)   107 cm 1 (d) 2  107 cm 1
Ans.: (b)
Solution: This is the case of two dimensional metal box. The Fermi wave vector of electron in
2  D is
1
1
 N 2
k F   2 n  2   2 2  ; L2  2mm  2mm  4  102 cm 2 ,
 L 
1
 4  10  12 1
 2 10 cm   2  107 cm 1
2
 k F  2  2 2 
14 2 2

 4 10 cm 
Q36. For an electron moving through a one-dimensional periodic lattice of periodicity a ,
which of the following corresponds to an energy eigenfunction consistent with Bloch’s
theorem?
  x   x     x  2 x   
(a)   x   A exp  i   cos    (b)   x   A exp  i   cos   
  a  2a      a  a  

  2 x  2 x      x  x 
(c)   x   A exp  i   i cosh     (d)   x   A exp  i  i 
  a  a     a 2a  

Ans.: (b)
Solution: According to block theorem,   x  a     x 

   2    x 2 x   
  x  a   A exp i   x  a   cos   x  a      A exp i      cos   2   
  a  a      a   a   

  2 x   x 2 x  
 A exp i   x  a   cos   A exp i   cos 
 a a    a a  

H.No. 40‐D, Ground Floor, Jia Sarai, Near IIT, Hauz Khas, New Delhi‐110016 
Phone: 011‐26865455/+91‐9871145498
Website: www.physicsbyfiziks.com  | Email: fiziks.physics@gmail.com  
19 
fiziks
Institute for NET/JRF, GATE, IIT‐JAM, M.Sc. Entrance, JEST, TIFR and GRE in Physics 
 
NET/JRF (JUNE-2016)
Q37. Consider electrons in graphene, which is a planar monoatomic layer of carbon atoms. If
the dispersion relation of the electrons is taken to be   k   ck (where c is constant)

over the entire k -space, then the Fermi energy  F depends on the number density of

electrons  as
1 2 1
(a)  F   2 (b)  F   (c)  F   3 (d)  F   3
Ans: (a)
Solution: In 2 D , density of state is
 L   d
g  k  dk    2 kdk , where   ck  k  and dk 
 2  c c
2
 L   d L2
 g   d     2  .   d
 2  c c 2 c 2
Now, number electrons at T  0 K is
F L2 F L2 2 N
N   g   d     d     F2  4 c 2 2  4 c 2 
0 2 c 2 0 4 c 2 F
L

  F  4 c 2  1/ 2   F   1/ 2
Q38. Suppose the frequency of phonons in a one-dimensional chain of atoms is proportional to
the wave vector. If n is the number density of atoms and c is the speed of the phonons,
then the Debye frequency is
 cn
(a) 2 cn (b) 2 cn (c) 3 cn (d)
2
Ans: (d)
Solution: Given   k    ck ( c is velocity of phonon)
L d L
Now g   d   d
 d / dk c
D L D L
Also N   g   d    d  N  D
0 c 0 c
N  N cn
 D  c  c n,  n    f D  . Best answer is (d).
L  L 2

H.No. 40‐D, Ground Floor, Jia Sarai, Near IIT, Hauz Khas, New Delhi‐110016 
Phone: 011‐26865455/+91‐9871145498
Website: www.physicsbyfiziks.com  | Email: fiziks.physics@gmail.com  
20 
fiziks
Institute for NET/JRF, GATE, IIT‐JAM, M.Sc. Entrance, JEST, TIFR and GRE in Physics 
 
Q39. The band energy of an electron in a crystal for a particular k -direction has the form
  k   A  B cos 2ka , where A and B are positive constants and 0  ka   . The
electron has a hole-like behaviour over the following range of k :
 3    3
(a)  ka  (b)  ka   (c) 0  ka  (d)  ka 
4 4 2 4 2 4
Ans: (a)
d d 2
Solution:   k   A  B cos 2ka ,  2 Ba sin 2ka , 2
 4 Ba 2 cos 2ka
dk dk
2 2
 
Effective mass m*  2 
d  / dk 2 2
4 Ba cos 2ka

   
Effective mass of electron me* and effective mass of holes mh* are opposite in sign i.e.,
m *
h  m . *
e 

Now, in the range 0  ka  , m* is positive
4
3 
While in the range  ka  , m* is negative
4 4
 3
Thus, electron has hole like behaviour in the region  ka 
4 4
NET/JRF (DEC-2016)
Q40. Consider a hexagonal lattice with basis vectors as shown in the figure below.
y

x

a2 a
1

a
If the lattice spacing is a  1 , the reciprocal lattice vectors are

 4   2 2   4   2 2 
(a)  ,0,   ,  (b)  ,0,  , 
 3   3 3  3   3 3

 4   2   2 2   2 
(c)  0,  ,  ,  (d)  ,  ,  2 , 
 3   3  3 3  3
Ans. : (a)
H.No. 40‐D, Ground Floor, Jia Sarai, Near IIT, Hauz Khas, New Delhi‐110016 
Phone: 011‐26865455/+91‐9871145498
Website: www.physicsbyfiziks.com  | Email: fiziks.physics@gmail.com  
21 
fiziks
Institute for NET/JRF, GATE, IIT‐JAM, M.Sc. Entrance, JEST, TIFR and GRE in Physics 
 
Solution: From the figure, we can write


a1 
3a
2
   
3 xˆ  yˆ , a2  3a yˆ , a3  azˆ (let us assume)

  
Now V  a1.  a2  a3  
3a
2
 3 xˆ  yˆ . 3ayˆ  azˆ 

3a
2
 
3 xˆ  yˆ . 3a 2 xˆ  3 3 3
2
a

 
Also, a3  a1  azˆ 
3a
2
 3x  y 
ˆ ˆ
2

3a 2

3 yˆ  xˆ 
Reciprocal lattice vectors are
 
* a2  a3 3a 2 xˆ 4 4
a1  2  2  xˆ  0 yˆ  xˆ  0 yˆ
V 3 3 3 3a 3
a
2

 
a3  a1
3 2
a 
3 yˆ  xˆ
2   2 2
*
a2  2
V
 2 2
3 3 3

3a

 xˆ  3 yˆ  
 3
xˆ 
3
 
yˆ 

a
2
* 4  2 2
for a  1 : a1  xˆ  0 yˆ , a2*   xˆ  yˆ
3 3 3
Q41. Consider a one-dimensional chain of atoms with lattice constant a . The energy of an
electron with wave-vector k is   k      cos  ka  , where  and  are constants. If

an electric field E is applied in the positive x -direction, the time dependent velocity of
an electron is
(In the following B is the constant)
 eE 
(a) Proportional to cos  B  at  (b) proportional to E
  

 eE 
(c) independent of E (d) proportional to sin  B  at 
  
Ans. : (d)
Solution: In the presence of electric field E , we can write
   
dp dk
F  eE   eE    eE
dt dt

H.No. 40‐D, Ground Floor, Jia Sarai, Near IIT, Hauz Khas, New Delhi‐110016 
Phone: 011‐26865455/+91‐9871145498
Website: www.physicsbyfiziks.com  | Email: fiziks.physics@gmail.com  
22 
fiziks
Institute for NET/JRF, GATE, IIT‐JAM, M.Sc. Entrance, JEST, TIFR and GRE in Physics 
 
eE
Integration gives, k  t   k  0   t

d  1   k 
The group velocity v  
dk  dk
  k 
Since,   k      cos  ka  ,    a sin ka
k
a
Thus, v  sin  ka 

Time dependent velocity of electron is
a a  eE  
v t   sin  k  t  a   sin  k  0   ta
     

a  eE  a  eE 
 sin  k  0  a  at   v  t   sin  B  at
       
Q42. A thin rectangular conducting plate of length a and width b is placed in the xy -plane in
two different orientations as shown in the figures below. In both cases a magnetic field B
is applied in the z -direction and a current flows in the x direction due to the applied
voltage V . b
y B

B
x a
V1 a
V1 b

   
V V
If the Hall voltage across the y -direction in the two cases satisfy V2  2V1 the ratio a : b
must be
(a) 1: 2 (b) 1: 2 (c) 2 :1 (d) 2 :1
Ans. : (d)
IB
Solution: Since, Hall voltage is given by VH  , where w is width of conducting plate.
w

H.No. 40‐D, Ground Floor, Jia Sarai, Near IIT, Hauz Khas, New Delhi‐110016 
Phone: 011‐26865455/+91‐9871145498
Website: www.physicsbyfiziks.com  | Email: fiziks.physics@gmail.com  
23 
fiziks
Institute for NET/JRF, GATE, IIT‐JAM, M.Sc. Entrance, JEST, TIFR and GRE in Physics 
 
l a 
Since, in case (I), V  I1 R1 and R1  1  
A1 ab b

I1  bV
V  I1 
b 
I1 B bVB bVB
Then, VH  V1     w  a 
w 2w 2a
 l2 b 
And also in case (II), R2   
A2 ab a

V Va
V  I 2 R2  I 2  
R2 
I 2 B VaB
Then, VH  V2  
 w  2b

a2 2
Since, V2  2V1    a : b  2 :1
b2 1

NET/JRF (JUNE-2017)
Q43. The energy gap and lattice constant of an indirect band gap semiconductor are 1.875 eV
and 0.52 nm , respectively. For simplicity take the dielectric constant of the material to be
unity. When it is excited by broadband radiation, an electron initially in the valence band
at k  0 makes a transition to the conduction band. The wavevector of the electron in the
conduction band, in terms of the wavevector kmax at the edge of the Brillouin zone, after
the transition is closest to
(a) kmax /10 (b) kmax /100 (c) kmax /1000 (d) 0
Ans. : (a)
Solution: The K  value of electron in C.B. is

2mE  2  9.1 10 kg 1.875 1.6 10 J  


1/ 2
31 19

K   K  7 109 m 1
 1.05  1034 J .S .
2 2  3.14 K
K max at the Brillouin Zero is K max   9
 1.2 1010 m 1  K  max
a 0.52  10 m 10

H.No. 40‐D, Ground Floor, Jia Sarai, Near IIT, Hauz Khas, New Delhi‐110016 
Phone: 011‐26865455/+91‐9871145498
Website: www.physicsbyfiziks.com  | Email: fiziks.physics@gmail.com  
24 
fiziks
Institute for NET/JRF, GATE, IIT‐JAM, M.Sc. Entrance, JEST, TIFR and GRE in Physics 
 
Q44. The electrical conductivity of copper is approximately 95% of the electrical conductivity
of silver, while the electron density in silver is approximately 70 % of the electron
density in copper. In Drude’s model, the approximate ratio  Cu /  Ag of the mean collision

time in copper  Cu  to the mean collision time in silver  Ag  is

(a) 0.44 (b) 1.50 (c) 0.33 (d) 0.66


Ans. : (d)
ne2  n    n
Solution:    cu  cu cu  cu  cu  Ag
m  Ag nAg  Ag  Ag  Ag ncu

 cu 0.95 Ag 0.7ncu
    0.66
 Ag  Ag ncu

H.No. 40‐D, Ground Floor, Jia Sarai, Near IIT, Hauz Khas, New Delhi‐110016 
Phone: 011‐26865455/+91‐9871145498
Website: www.physicsbyfiziks.com  | Email: fiziks.physics@gmail.com  
25 
fiziks
Institute for NET/JRF, GATE, IIT‐JAM, M.Sc. Entrance, JEST, TIFR and GRE in Physics 
 
NUCLEAR AND PARTICLE PHYSICS
NET/JRF (JUNE-2011)

Q1. The radius of a 64


29 Cu nucleus is measured to be 4.8  10-13 cm.
(A). The radius of a 27
12 Mg nucleus can be estimated to be
(a) 2.86  10-13 cm (b) 5.2 10-13 cm (c) 3.6 10-13 cm (d) 8.6 10-13 cm
Ans: (c)
1/ 3
RMg  AMg 
1/ 3
 27 
Solution: Since R  R0  A      
1/ 3

RCu  ACu   64 
RMg 3 3
  RMg   4.8  10 13  3.6  10 13 cm.

RCu 4 4
(B). The root-mean-square (r.m.s) energy of a nucleon in a nucleus of atomic number A
in its ground state varies as:
(a) A4 / 3 (b) A1 / 3 (c) A1/ 3 (d) A2 / 3
Ans: (c)
Q2. A beam of pions (π+) is incident on a proton target, giving rise to the process
  p  n  
(A). Assuming that the decay proceeds through strong interactions, the total isospin I and
its third component I3 for the decay products, are
3 3 5 5
(a) I  , I 3  (b) I  , I 3 
2 2 2 2
5 3 1 1
(c) I  , I 3  (d) I  , I 3  
2 2 2 2
Ans: (c)
1 5 1 3
Solution:    p  n       ; I : 1 1  , I3 :  11 
2 2 2 2
(B). Using isospin symmetry, the cross-section for the above process can be related to
that of the process
(a)  n  p  (b)  p  n  
     

(c)  n  p  (d)  p  n 
     

Ans: (c)

H.No. 40‐D, Ground Floor, Jia Sarai, Near IIT, Hauz Khas, New Delhi‐110016 
Phone: 011‐26865455/+91‐9871145498
Website: www.physicsbyfiziks.com  | Email: fiziks.physics@gmail.com  

fiziks
Institute for NET/JRF, GATE, IIT‐JAM, M.Sc. Entrance, JEST, TIFR and GRE in Physics 
 
NET/JRF (DEC-2011)
Q3. According to the shell model the spin and parity of the two nuclei 125
51 Sb and 89
38 Sr are,
respectively,
   
5 5 5 7
(a)   and   (b)   and  
2 2 2 2
   
7 5 7 7
(c)   and   (d)   and  
2 2 2 2
Ans: (d)
Solution: 125
51 Sb ; Z  51 and N  74
Z  51

 s1/ 2   p3/ 2   p1/ 2   d5 / 2   s1/ 2   d3/ 2   f 7 / 2   p3/ 2   f5 / 2   p1/ 2   g9 / 2   g7 / 2 


2 4 2 6 2 4 8 4 6 2 10 1


7 7
 j  and l  4 . Thus spin and parity   
2 2
89
38 Sr ; Z  38 and N  51
N  51:

 s1/ 2   p3/ 2   p1/ 2   d5 / 2   s1/ 2   d3/ 2   f 7 / 2   p3/ 2   f5 / 2   p1/ 2   g9 / 2   g 7 / 2 


2 4 2 6 2 4 8 4 6 2 10 1


7 7
 j and l  4 . Thus spin and parity   
2 2
Q4. The difference in the Coulomb energy between the mirror nuclei 49
24 Cr and 49
25 Mn is

6.0 MeV . Assuming that the nuclei have a spherically symmetric charge distribution and

that e2 is approximately 1.0 MeV-fm, the radius of the 49


25 Mn nucleus is

(a) 4.9  10-13 m (b) 4.9  10-15 m


(c) 5.1  10-13 m (d) 5.1  10-15 m
3  1  10 15
Ans: (b) R
3e 2
5  W
Z1  Z 2  
2 2

5 6
25 2  24 2   4.9  10 15 m .

H.No. 40‐D, Ground Floor, Jia Sarai, Near IIT, Hauz Khas, New Delhi‐110016 
Phone: 011‐26865455/+91‐9871145498
Website: www.physicsbyfiziks.com  | Email: fiziks.physics@gmail.com  

fiziks
Institute for NET/JRF, GATE, IIT‐JAM, M.Sc. Entrance, JEST, TIFR and GRE in Physics 
 
NET/JRF (JUNE-2012)
1
Q5. The ground state of 12 Pb nucleus has spin-parity J  , while the first excited state
207 p

2
5
has J  p
.The electromagnetic radiation emitted when the nucleus makes a transition
2
from the first excited state to ground state are
(a) E2 and E3 (b) M2 or E3 (c) E2 or M3 (d) M2 or M3
Ans: (c)
Solution: No parity change; J  2,3

For El type,    1 , (for no parity change l  2 )


l

For M l type,    1 , (for no parity change l  3 )


l 1

J  2 , No parity change  E 2 ; J  3 , No parity change  M 3


Q6. The dominant interactions underlying the following processes
A. K   p      , B.       K   K  , C.    p   0 are

(a) A: strong, B: electromagnetic and; C: weak


(b) A: strong, B: weak and; C: weak
(c) A: weak, B: electromagnetic and; C: strong
(d) A: weak, B: electromagnetic and; C: weak
Ans: (a)
(A) K   p      (Strong interaction)

1 1
I 3 :    1  1 (Conserved)
2 2
(B)       K   K  (Electromagnetic interaction)

(C)    p   0 (Weak interaction)


1
I3 :1  0 (Not conserved)
2

H.No. 40‐D, Ground Floor, Jia Sarai, Near IIT, Hauz Khas, New Delhi‐110016 
Phone: 011‐26865455/+91‐9871145498
Website: www.physicsbyfiziks.com  | Email: fiziks.physics@gmail.com  

fiziks
Institute for NET/JRF, GATE, IIT‐JAM, M.Sc. Entrance, JEST, TIFR and GRE in Physics 
 
NET/JRF (JUNE-2013)
Q7. The binding energy of a light nucleus Z , A in MeV is given by the approximate formula

B A, Z   16 A  20 A 2/3 3
 Z 2 A 1 / 3  30
N  Z  2

4 A
where N  A  Z is the neutron number. The value of Z of the most stable isobar for a
given A is
1 1 1
A A2 / 3  A A A2 / 3  A A4 / 3 
(a) 1   (b) (c) 1   (d) 1  
2 160  2 2 120  2 64 
Ans: (a)
1
B A A2 / 3 
Solution:  0  Z   1  
Z Z Z  2 160 

Q8. A spin-1/2 particle A undergoes the delay A  B  C  D , where it is known


that B and C are also spin-1/2 particles. The complete set of allowed values of the spin of
the particle D is
1 3 5 1 1 3 5 7
(a) ,1, , 2, , 3, ... (b) 0, 1 (c) only (d) , , , ,....
2 2 2 2 2 2 2 2
Ans: (c)
Solution: Spin of the left side and combined spin of the products must be same to conserve the
spin angular momentum conservation law.
Q9. Muons are produced through the annihilation of particle a and its anti-particle, namely
the process a  a       . A muon has a rest mass of 105 MeV/c2 and its proper life
time is 2 s . If the center of mass energy of the collision is 2.1 GeV in the laboratory
frame that coincides with the center-of-mass frame, then the fraction of muons that will
decay before they reach a detector placed 6 km away from the interaction point is
(a) e 1 (b) 1  e 1 (c) 1  e 2 (d) e 10
Ans: (b)
t
 t N  2.1
Solution: N  N 0 e   e t  e  , where   2  10 6 s ,    10 3  20 and
N0 105

6  10 3
1
5 t 1 N 
t  2  10 sec . Thus    e 2
 1  e 1 .
3  10 8
 2 N0

H.No. 40‐D, Ground Floor, Jia Sarai, Near IIT, Hauz Khas, New Delhi‐110016 
Phone: 011‐26865455/+91‐9871145498
Website: www.physicsbyfiziks.com  | Email: fiziks.physics@gmail.com  

fiziks
Institute for NET/JRF, GATE, IIT‐JAM, M.Sc. Entrance, JEST, TIFR and GRE in Physics 
 
NET/JRF -(DEC-2013)
A
Q10. The intrinsic electric dipole moment of a nucleus Z X
(a) increases with Z , but independent of A
(b) decreases with Z , but independent of A
(c) is always zero
(d) increases with Z and A
Ans: (d)
Q11. According to the shell model, the total angular momentum (in units of  ) and the parity
of the ground state of the 37 Li nucleus is

3 3
(a) with negative parity (b) with positive parity
2 2
1 7
(c) with positive parity (d) with negative parity
2 2
Ans: (a)
Solution: Z  3, N  4

For odd Z  3; s12/ 2  p31 / 2   j  3 / 2, l  1 and parity   11  1 .

NET/JRF (JUNE-2014)
Q12. The recently-discovered Higgs boson at the LHC experiment has a decay mode into a
photon and a Z boson. If the rest masses of the Higgs and Z boson are 125 GeV/c 2 and

90 GeV/c 2 respectively, and the decaying Higgs particle is at rest, the energy of the
photon will approximately be
(a) 35 3 GeV (b) 35 GeV (c) 30 GeV (d) 15 GeV
Ans: (c)
Solution: Assume H is symbol of Higgs boson, H  Z 

E H2  E Z2 1252  902
E    30GeV
2EH 2  125

H.No. 40‐D, Ground Floor, Jia Sarai, Near IIT, Hauz Khas, New Delhi‐110016 
Phone: 011‐26865455/+91‐9871145498
Website: www.physicsbyfiziks.com  | Email: fiziks.physics@gmail.com  

fiziks
Institute for NET/JRF, GATE, IIT‐JAM, M.Sc. Entrance, JEST, TIFR and GRE in Physics 
 
Q13. In a classical model, a scalar (spin-0) meson consists of a quark and an antiquark bound
b
by a potential V r   ar  , where a  200 MeV fm -1 and b  100 MeV fm . If the
r
masses of the quark and antiquark are negligible, the mass of the meson can be estimated
as approximately
(a) 141 MeV/c 2 (b) 283 MeV/c 2 (c) 353 MeV/c 2 (d) 425 MeV/c 2
Ans: (b)
Solution: At equilibrium separation the potential is minimum, thus the equilibrium separation
can be determined as
dV  r  b b 100MeVfm 1
a  0  r0   1
 fm
dr r  r0
r02
a 200MeVfm 2

The equilibrium separation between particles is also estimated by uncertainty principle



r0  ct  r0  c ( where, Et  )
E
Where, c is the velocity of the virtual meson
 200 MeV . fm
r0  c 
E E MeV 
200MeV . fm 1
Using above two relation  fm
E MeV  2

E  200 2  283MeV  E  m  c 2
E
the mass of the meson m   283MeV / c 2
c 2

NET/JRF (DEC-2014)
Q14. Consider the four processes
(i) p   n  e   ve (ii) 0  p   e   v e

(iii)    e   ve (iv)  0    
which of the above is/are forbidden for free particles?
(a) only (ii) (b) (ii) and (iv) (c) (i) and (iv) (d) (i) and (ii)
Ans: (d)

H.No. 40‐D, Ground Floor, Jia Sarai, Near IIT, Hauz Khas, New Delhi‐110016 
Phone: 011‐26865455/+91‐9871145498
Website: www.physicsbyfiziks.com  | Email: fiziks.physics@gmail.com  

fiziks
Institute for NET/JRF, GATE, IIT‐JAM, M.Sc. Entrance, JEST, TIFR and GRE in Physics 
 
Solution: (i) p  n  e  e [Not allowed]
 

It violate energy conservation. The mass of proton is less than mass of neutron. Free
proton is stable and can not decay to neutron. Proton can decay to neutron only inside the
nucleus, where energy violation is taken care by Heisenberg uncertainty principle.
(ii)  0  p   e   e [Not allowed]. In this decay charge is not conserved

(iii)    e   e [allowed through Weak interaction]

(iv)  0     [allowed through Electromagnetic interaction]


Q15. In deep inelastic scattering electrons are scattered off protons to determine if a proton has
any internal structure. The energy of the electron for this must be at least
(a) 1.25  10 9 eV (b) 1.25  1012 eV (c) 1.25  10 6 eV (d) 1.25  10 8 eV
Ans: (b)
Solution: The internal structure of proton can only be determined if the wavelength of the
incoming electron is nearly equal to the size of the proton
i.e.   R  1.2 A1/ 3  fm   1.2 fm  1.2  1015 m

h h
According to de-Broglie relation,   
p 2mE

 0 150
This can be also written as    
  E  eV 

150 150
 E  eV     1.04  1012  E  1.04 1012 eV
1.2  10 
2 5 2
  0

    
 
The bet suitable answer is option (b).
Q16. If the binding energy B of a nucleus (mass number A and charge Z ) is given by

 2Z  A
2
aC Z 2
B  aV A  aS A 2/3
 asym  1/ 3
A A
where aV  16 MeV , a S  16 MeV , a sym  24 MeV and aC  0.75 MeV , then for the most

stable isobar for a nucleus with A  216 is


(a) 68 (b) 72 (c) 84 (d) 92
Ans: (c)

H.No. 40‐D, Ground Floor, Jia Sarai, Near IIT, Hauz Khas, New Delhi‐110016 
Phone: 011‐26865455/+91‐9871145498
Website: www.physicsbyfiziks.com  | Email: fiziks.physics@gmail.com  

fiziks
Institute for NET/JRF, GATE, IIT‐JAM, M.Sc. Entrance, JEST, TIFR and GRE in Physics 
 
dB 2  2Z  A   2 2aC Z
Solution: For the most stable isobar for a nucleus  0  asym  1/ 3  0
dZ A A
2  2Z  216   2 2Z 4  2Z  216  3 2Z
 24  0.75 0   0
 216 
1/ 3
216 9 4 6

4  2Z  216  Z
   0  16  2 Z  216   9 Z  0  41Z  216  16  Z  82.3
9 4

NET/JRF (JUNE-2015)
Q17. The reaction 2
1 D 12 D 42 He   0 cannot proceed via strong interactions because it
violates the conservation of
(a) angular momentum (b) electric charge
(c) baryon number (d) isospin
Ans. (d)
Solution: 1 D 2  1 D 2  2 He 4   0 (Not conserved)
I: 0 0  0 1
This isopin is not conserved in above reaction.
Q18. Let us approximate the nuclear potential in the shell model by a three dimensional
isotropic harmonic oscillator. Since the lowest two energy levels have angular momenta
l  0 and l  1 respectively, which of the following two nuclei have magic numbers of
protons and neutrons?
(a) 42 He and 16
8 O (b) 12 D and 84 Be (c) 42 He and 84 Be (d) 42 He and 12
6 C
Ans. (a)
Solution: 2 He 4 has Z  2, N  2

and 8 O16 has Z  8, N  8 magic numbers  2,8, 20, 28,50,82,126 

Q19. The charm quark S assigned a charm quantum number C  1 . How should the
Gellmann-Nishijima formula for electric charge be modified for four flavors of quarks?
1 1
(a) I 3  B  S C (b) I 3  B  S  C
2 2
1 1
(c) I 3  B  S C (d) I 3  B  S  C
2 2
H.No. 40‐D, Ground Floor, Jia Sarai, Near IIT, Hauz Khas, New Delhi‐110016 
Phone: 011‐26865455/+91‐9871145498
Website: www.physicsbyfiziks.com  | Email: fiziks.physics@gmail.com  

fiziks
Institute for NET/JRF, GATE, IIT‐JAM, M.Sc. Entrance, JEST, TIFR and GRE in Physics 
 
Ans. (d)
1
Solution: From Gell-Mann-Nishijima formula Q  I 3  B  S
2
1
For Quark it is generalized as Q  I 3  B  S  C
2
NET/JRF (DEC-2015)
Q20. Consider the following processes involving free particles
(i) n  p  e   ve (ii) p  n   

(iii) p  n      0   0 (iv) p  ve  n  e 
Which of the following statements is true?
(a) Process (i) obeys all conservation laws
(b) Process (ii) conserves baryon number, but violates energy-momentum conservation
(c) process (iii) is not allowed by strong interaction but is allowed by weak interactions
(d) Process (iv) conserves baryon number, but violates lepton number conservation
Ans.: (b)
Solution: (i) n  p  e   ve

q 0 1 1 0 (conserved)
1 1 1 1
spin     (not conserved)
2 2 2 2
Le 0 0  1  1 (not conserved)
(ii) Baryon number is conserved but energy and momentum conservation violated.
(iii) spin is not conserved
(iv) obeys all conservation laws.
Q21. Of the nuclei of mass number A  125 , the binding energy calculated from the liquid
drop model (given that the coefficients for the Coulomb and the asymmetry energy are
ac  0.7 MeV and asym  22.5 MeV respectively) is a maximum for

(a) 125
54 Xe (b) 124
53 I (c) 125
52 Te (d) 125
51 Sb
Ans.: (c)

H.No. 40‐D, Ground Floor, Jia Sarai, Near IIT, Hauz Khas, New Delhi‐110016 
Phone: 011‐26865455/+91‐9871145498
Website: www.physicsbyfiziks.com  | Email: fiziks.physics@gmail.com  

fiziks
Institute for NET/JRF, GATE, IIT‐JAM, M.Sc. Entrance, JEST, TIFR and GRE in Physics 
 
4  22.5 125  0.7  53 
2/3
4aa  ac A1/ 3 4aa A  ac A2 / 3
Solution: Z 0    Z0 
2ac A1/ 3  8aa A1 8aa  2ac A2 / 3 8  22.5  2  0.7  53 
2/3

11250  17.5 11267.5


 Z0    52.4  Z 0  52
180  35 215
NET/JRF (JUNE-2016)
Q22. A radioactive element X decays to Y , which in turn decays to a stable element Z . The
decay constant from X to Y is 1 , and that from Y to Z is 2 . If, to begin with, there are

1 1
only N 0 atoms of X , at short times ( t  as well as ) the number of atoms of Z
1 2
will be
1 12
(a) 12 N 0t 2 (b) N 0t
2 2  1  2 

(c)  1  2  N 0t 2 (d)  1  2  N 0t
2

Ans: (a)
1 2
X   Y  Z
Solution: t  0 N0 0 0
t N1 N2 N3
dN dN
Rate equations N1  N 0 e  1t , 2  1 N1  2 N 2 , 3  2 N 2
dt dt
  e 2t  e  1 t 
N 3  N 0 1  1  2 
  2  1   2  1  
 1  22t 2  2  12t 2  
 N 0 1   1   t     1   t  
  2  1   2   2  1  
2 1
2  
 1  t 1 22t 2 2 2 1t 2 12t 2 
 N 0 1   1 2     
  2  1   2  1   2  1  2  2  1   2  1   2  1  2 
 1  2t 2 2  2t 2    t 2   1  1
 N0   2   1   1 2 N0  2    12 N 0t
2

  2 1 
   2  2 1
   2  2  
 2 1 2  1 2

H.No. 40‐D, Ground Floor, Jia Sarai, Near IIT, Hauz Khas, New Delhi‐110016 
Phone: 011‐26865455/+91‐9871145498
Website: www.physicsbyfiziks.com  | Email: fiziks.physics@gmail.com  
10 
fiziks
Institute for NET/JRF, GATE, IIT‐JAM, M.Sc. Entrance, JEST, TIFR and GRE in Physics 
 
Q23. In the large hadron collider  LHC  , two equal energy proton beams traverse in opposite

directions along a circular path of length 27 km . If the total centre of mass energy of a
proton-proton pair is 14 TeV , which of the following is the best approximation for the
proper time taken by a proton to traverse the entire path?
(a) 12 ns (b) 1.2  s (c) 1.2 ns (d) 0.12  s
Ans: (a)
Solution: The proton travel at nearly speed of light in LHC , therefore
d 27 103
t   9 105 sec
c 3 108
v2 t
Since, proton is relativistic, t0  t 1  2 
c 
1 m0 c 2 938 MeV 938  106 eV
 E   m0 c 2      1.34 104
 E 7 TeV 7 1012 eV
t
Thus, t0   9  105  1.34 104  1.2 108 sec  12 ns

Q24. Let ES denotes the contribution of the surface energy per nucleon in the liquid drop

model. The ratio ES  27


13 
Al : ES  64
30 
Zn is

(a) 2 : 3 (b) 4 : 3 (c) 5 : 3 (d) 3 : 2


Ans: (b)
2 1 1
E  Al   27  3  64  3 4
1 
B A3 
Solution: ES   A3 S   
A A ES  Z n  1 1
 64  3  27  3 3

Q25. According to the shell model, the nuclear magnetic moment of the 27
13 Al nucleus is (Given

that for a proton gl  1, g s  5.586 , and for a neutron gl  0, g s  3.826 )

(a) 1.913  N (b) 14.414  N (c) 4.793  N (d) 0


Ans: (c)
5
Solution: 13 Al 27 : Z  13, N  14 for Z  13, S1/2 2 , P3/4 2 , P1/22 , d55/ 2  j  , l  2
2

 2 j  1  g S   N   2   1  5.586  N    4.793  N
1 1 5
Magnetic moment,  
2 2 2 

H.No. 40‐D, Ground Floor, Jia Sarai, Near IIT, Hauz Khas, New Delhi‐110016 
Phone: 011‐26865455/+91‐9871145498
Website: www.physicsbyfiziks.com  | Email: fiziks.physics@gmail.com  
11 
fiziks
Institute for NET/JRF, GATE, IIT‐JAM, M.Sc. Entrance, JEST, TIFR and GRE in Physics 
 
NET/JRF (DEC-2016)
Q26. What should be the minimum energy of a photon for it to split an  -particle at rest into a
tritium and a proton?
(The masses of 4
2 He, 13 H and 1
1 H are 4.0026 amu,3.0161 amu and 1.0073 amu

respectively, and 1 amu  938 MeV )


(a) 32.2 MeV (b) 3MeV (c) 19.3 MeV (d) 931.5 MeV
Ans. : (c)
Solution: From conservation of energy
E  m c 2  m1H 3 c 2  m1H 1 c 2

or E   m1H 3  m1H 1  m   938 MeV  19.5 MeV

Q27. Which of the following reaction(s) is/are allowed by the conservation laws?
(i)    n   0  K 
(ii)    p   0  K 0
(a) both (i) and (ii) (b) only (i)
(c) only (ii) (d) neither (i) nor (ii)
Ans. : (a)
Solution: (i)    n   0  K 
q :1  0  0  1
B : 0 1  1 0
S : 0  0  1  1
Reaction is allowed
(ii)    p   0  K 0
q : 1  1  0  0
B : 0 1  1 0
S : 0  0  1  1
Reaction is allowed

H.No. 40‐D, Ground Floor, Jia Sarai, Near IIT, Hauz Khas, New Delhi‐110016 
Phone: 011‐26865455/+91‐9871145498
Website: www.physicsbyfiziks.com  | Email: fiziks.physics@gmail.com  
12 
fiziks
Institute for NET/JRF, GATE, IIT‐JAM, M.Sc. Entrance, JEST, TIFR and GRE in Physics 
 
Q28. A particle, which is a composite state of three quarks u , d and s , has electric charge,
spin and strangeness respectively, equal to
1 1 1
(a) 1, , 1 (b) 0, 0, 1 (c) 0, , 1 (d) 1,   1
2 2 2
Ans. : (c)
Solution: charge, spin and strangers of Quarks u , d & s are given as
U D S Total
Charge 2 1 1 0
3 3 3
Spin 1 1 1 1 3
or
2 2 2 2 2
Strangeness 0 0 1 1
If a particle x is a composite of u, d & s , then net charge, spin and strangeness on x is
net charge  0
1 3
net spin  or and net strangeness  1
2 2
NET/JRF (JUNE-2017)
Q29. If in a spontaneous  - decay of 232
92 U at rest, the total energy released in the reaction is

Q , then the energy carried by the  - particle is


(a) 57Q / 58 (b) Q / 57 (c) Q / 58 (d) 23Q / 58
Ans. : (a)
Solution: Energy carried by the   particle is
 A4 228 57
KE   Q  Q Q
 A  232 58
Q30. The range of the nuclear force between two nucleons due to the exchange of pions is
1.40 fm . If the mass of pion is 140 MeV / c 2 and the mass of the rho-meson is

770 MeV / c 2 , then the range of the force due to exchange of rho-mesons is
(a) 1.40 fm (b) 7.70 fm (c) 0.25 fm (d) 0.18 fm
Ans. : (c)

H.No. 40‐D, Ground Floor, Jia Sarai, Near IIT, Hauz Khas, New Delhi‐110016 
Phone: 011‐26865455/+91‐9871145498
Website: www.physicsbyfiziks.com  | Email: fiziks.physics@gmail.com  
13 
fiziks
Institute for NET/JRF, GATE, IIT‐JAM, M.Sc. Entrance, JEST, TIFR and GRE in Physics 
 
c
Solution: Range for nuclear force between nucleon will be R  ct  and c  199MeVfm
mc 2
199 MeVfm
R  0.25 fm
MeV 2
770 2  c
c
Q31. A baryon X decays by strong interaction as X         0 , where   is a member
of the isotriplet    ,  0 ,    . The third component I 3 of the isospin of X is

(a) 0 (b) 1/ 2 (c) 1 (d) 3 / 2


Ans. : (a)
Solution: X        0

I 3 :1
1 0

 I 3 for X is 0 .

H.No. 40‐D, Ground Floor, Jia Sarai, Near IIT, Hauz Khas, New Delhi‐110016 
Phone: 011‐26865455/+91‐9871145498
Website: www.physicsbyfiziks.com  | Email: fiziks.physics@gmail.com  
14 
fiziks
Forum for CSIR-UGC JRF/NET, GATE, IIT-JAM, GRE in PHYSICAL SCIENCES

JEST SYLLABUS
The JEST screening test for entrance to the PhD programme in Physics focusses on the
following broad areas:
• Classical Mechanics
• Electromagnetic Theory and Optics
• Quantum Mechanics
• Thermodynamics and Statistical Mechanics
• Mathematical Methods
• Electronics and Experimental Methods
• Advanced topics (like Atomic, Molecular and Nuclear Physics and very elementary
Solid State Physics)
In each of these areas, familiarity with the basics (including the necessary mathematics)
is assumed. The test will contain only multiple choice questions. Half the questions
would carry ONE POINT and the candidate may have to do a short calculation to get the
answer. The rest would be THREE POINT QUESTIONS which will require more
detailed problem solving. A sample questionnaire is attached.
Some textbooks which may help the candidate prepare for the test are listed below. It is
not essential to read every chapter of all the books. There is no specified syllabus for the
test; rather the objective is to check whether the student understands the basic principles
of physics and is able to apply them in solving problems.
• Classical Mechanics by H. Goldstein
• An Introduction to Mechanics by Daniel Kleppner & Robert J. Kolenkow
• Introduction to Electrodynamics by David J. Griffiths
• Principles of Quantum Mechanics by R. Shankar
• Introduction to Quantum Mechanics by David J. Griffiths
• Fundamentals of Statistical and Thermal Physics by Frederick Reif
• Heat and Thermodynamics by M. W. Zemansky & Richard H. Dittman
• Mathematical Methods for Physicists by George B. Arfken & Hans J. Weber
• Concepts of Modern Physics by Arthur Beiser

fiziks c/o Anand Institute of mathematics, 28-B/6 Jia Sarai


Near IIT, Hauz Khas, New Delhi, PIN- 110016 (INDIA)
Phone: 011-32718565, +91-9871145498
Website: http://www.physicsbyfiziks.com 1
Email: fiziks.physics@gmail.com
fiziks
Forum for CSIR-UGC JRF/NET, GATE, IIT-JAM, GRE in PHYSICAL SCIENCES

Sample Questions
THREE POINT QUESTIONS
Q.1 In quantum mechanics, one may picture a wave function in either momentum space or
configuration space. If the wave function in momentum space is (p) = N / (p2 + α2), then
calculate the wave function in configuration space (aside from a multiplicative constant).
(a) exp ( - α2 x2 / ħ2) (b) exp( - α | x | / ħ)
(c) sin(px / ħ) (d) cos(px / ħ)
Q.2 Consider the two-body decay of a particle X which is at rest in the laboratory and of mass
M in its rest-frame into a particle A of mass mA and another particle which is mass less.
The energy of the particle A in the laboratory frame is (c = 1):
(a) M / 2 (b) (M 2 + m2A) / (2M)
(c) (M 2 – m2A) / (2M) (d) (M 2 – 4m2A) / (2M)
Q.3 The electric potential of a grounded conducting sphere of radius a in an uniform electric
field is given as (r, θ) = - E0 r [1 – (a/r)3] cosθ. Find the surface charge distribution on
the sphere.
(a)  0 E0 sin  (b)  0 E0 cos

(c) 3 0 E 0 cos (d) 2 0 E 0 cos


Q.4 Compute
Re( z 2 )  Im( z 2 )
lim
z 0 z2
(a) the limit does not exist (b) 1
(c) – i (d) – 1
Q.5 Consider a two state system with an observable represented by

A 1 1 
 1 1 

(a) 0  Â   (b) 0  Aˆ  1

(c) 0  Aˆ  2 (d) Aˆ  1

fiziks c/o Anand Institute of mathematics, 28-B/6 Jia Sarai


Near IIT, Hauz Khas, New Delhi, PIN- 110016 (INDIA)
Phone: 011-32718565, +91-9871145498
Website: http://www.physicsbyfiziks.com 2
Email: fiziks.physics@gmail.com
fiziks
Forum for CSIR-UGC JRF/NET, GATE, IIT-JAM, GRE in PHYSICAL SCIENCES

ONE POINT QUESTIONS


Q.1 The states n , n  0,1,2 form a complete orthonormal set of states in Fock space.

Annihilation operator â is defined by

aˆ n  n n  1 , aˆ 0  0

The matrix elements of hermitian adjoint â  are given by n aˆ  n' =

(a) n nn ' (b) n n n '1 (c) n n 1 n' (d) n n'

Q.2 A hard ball dropped from a height of 1 m in earth’s gravitational field bounces to a height
of 95 cm. What will be the total distance traversed by the ball?
(a) 2000 cm (b) 1010 cm (c) 1810 cm (d) 3900 cm
Q.3 The force experienced by a mirror when it reflects all the light from a laser with a power
of 10m W is about
(a) 6.7 x 10-11 N (b) 0.7 x 10 -13 N
(c) 3.3 x 10-11 N (d) 7.0 x 10-13 N
Q.4 The coordinate transformation x' = 0.8x + 0.6y, y' = 0.6 x – 0.8y represents
(a) a translation (b) a rotation in the x – y plane.
(c) a reflection in the x – y plane (d) none of the above
Q.5 The equivalent capacitance of a system of capacitors shown in the adjacent figure
between A and B is

A B

C1 C2 C3

1
 1 1 1 
(a)     (b) C1 – C 2 + C3
 C1 C 2 C 3 
C1C 2
(c)  C3 (d) C 1 + C2 + C3
C1  C 2

fiziks c/o Anand Institute of mathematics, 28-B/6 Jia Sarai


Near IIT, Hauz Khas, New Delhi, PIN- 110016 (INDIA)
Phone: 011-32718565, +91-9871145498
Website: http://www.physicsbyfiziks.com 3
Email: fiziks.physics@gmail.com
fiziks
Institute for NET/JRF, GATE, IIT‐JAM, JEST, TIFR and GRE in PHYSICAL SCIENCES 
 
Mathematical Physics
JEST-2016
Q1. Given the condition ∇ 2φ = 0 , the solution of the equation ∇ 2ψ = k ∇φ .∇φ is given by

kφ 2 kφ ln φ kφ ln φ
(a) ψ = (b) ψ = kφ 2 (c) ψ = (d)ψ =
2 2 2
Ans: (a)

( ) ( )
Solution: ∇ 2φ = 0 ⇒ ∇. ∇φ = 0 ⇒ ∇. ∇φ = 0 ⇒ ∇φ = α xˆ + β yˆ + γ zˆ ⇒ φ = α x + β y + γ z

(
k ∇φ .∇φ = k α 2 + β 2 + γ 2 )
kφ 2 k
= (α x + β y + γ z )
2
If ψ =
2 2
∂ 2ψ ∂ 2ψ ∂ 2ψ
⇒ ∇ 2ψ = + +
∂x 2 ∂y 2 ∂z 2
(
= k α 2 + β 2 + γ 2 ⇒ ∇ 2ψ = k ∇φ .∇φ )
Q2. The mean value of random variable x with probability density

p ( x) =
1 ⎡ x2 + μ x
.exp ⎢ −
( ) ⎤⎥ is:
σ 2π ⎢⎣ 2σ 2 ( ) ⎥⎦

μ −μ
(a) 0 (b) (c) (d) σ
2 2
Ans. : (a)

1 ∞ x2 ∞ μx
Solution: x = ∫ x exp− 2 dx ∫ x exp − 2 dx = 0
σ 2π −∞ 2σ −∞ 2σ

                                                                                
Head office  Branch office 
 
fiziks, H.No. 40 D, G.F, Jia Sarai,  Anand Institute of Mathematics, 
 
Near IIT, Hauz Khas, New Delhi‐16  28‐B/6, Jia Sarai, Near IIT 
 
Phone: 011‐26865455/+91‐9871145498 Hauz Khas, New Delhi‐16 
                                                   
                                             Website: www.physicsbyfiziks.com                                                                                          
                                                           Email: fiziks.physics@gmail.com                                                                   1 
fiziks
Institute for NET/JRF, GATE, IIT‐JAM, JEST, TIFR and GRE in PHYSICAL SCIENCES 
 
⎛2 1⎞ ⎛πM ⎞
Q3. Given a matrix M = ⎜ ⎟ , which of the following represents cos ⎜ ⎟
⎝1 2⎠ ⎝ 6 ⎠

1 ⎛1 2⎞ 3 ⎛ 1 −1 ⎞ 3 ⎛ 1 1⎞ 1⎛ 1 3⎞
(a) ⎜ ⎟ (b) ⎜ ⎟ (c) ⎜ ⎟ (d) ⎜ ⎟
2 ⎝2 1⎠ 4 ⎝ −1 1 ⎠ 4 ⎝ 1 1⎠ 2 ⎜⎝ 3 1 ⎟⎠

Ans. : (b)
Solution: We have
2−λ 1
=0
1 2−λ

⇒ λ 2 − 4λ + 3 = 0 ⇒ λ = 1 or λ = 3
For λ = 1
⎛ 2 −1 1 ⎞ ⎛ x ⎞ ⎛ 0 ⎞
⎜ ⎟ ⎜ ⎟ = ⎜ ⎟ gives
⎝ 1 2 − 1⎠ ⎝ y ⎠ ⎝ 0 ⎠
Thus x + y = 0 ⇒ x = − y . Taking x = 1 , the eigenvector associated with λ = 1 is

⎡ 1⎤
x1 = ⎢ ⎥
⎣ −1⎦
For λ = 3
⎛ −1 1 ⎞ ⎛ x ⎞ ⎛ 0 ⎞
⎜ ⎟⎜ ⎟ = ⎜ ⎟ ⇒ x = y
⎝ 1 −1 ⎠ ⎝ y ⎠ ⎝ 0 ⎠
⎡1⎤
Taking x = 1 , the eigenvectors associated with λ = 3 is x2 = ⎢ ⎥
⎣1⎦
Thus
⎡ 1 1⎤ ⎡1 0 ⎤ ⎡1/ 2 −1/ 2 ⎤
M =⎢ ⎥⎢ ⎥⎢ ⎥
⎣ −1 1⎦ ⎣0 3⎦ ⎣1/ 2 1/ 2 ⎦
⎡ π ⎤
i 0⎥
π ⎡ 1 1⎤ ⎢ 6 ⎡1/ 2 −1/ 2 ⎤
i M =⎢ ⎥⎢ ⎥⎢
6 ⎣ −1 1⎦ ⎢ 0 π 1/ 2 1/ 2 ⎥⎦
i ⎥⎣
⎣⎢ 6 ⎦⎥

                                                                                
Head office  Branch office 
 
fiziks, H.No. 40 D, G.F, Jia Sarai,  Anand Institute of Mathematics, 
 
Near IIT, Hauz Khas, New Delhi‐16  28‐B/6, Jia Sarai, Near IIT 
 
Phone: 011‐26865455/+91‐9871145498 Hauz Khas, New Delhi‐16 
                                                   
                                             Website: www.physicsbyfiziks.com                                                                                          
                                                           Email: fiziks.physics@gmail.com                                                                   2 
fiziks
Institute for NET/JRF, GATE, IIT‐JAM, JEST, TIFR and GRE in PHYSICAL SCIENCES 
 
π
π
⎡ i6 ⎤
i M ⎡ 1 1 ⎤ ⎢ e 0 ⎥ ⎡1/ 2 −1/ 2 ⎤
e6 =⎢ ⎥⎢ π ⎥⎢
⎣ −1 1⎦ ⎢ 1/ 2 1/ 2 ⎥⎦
6 ⎥⎣
i
⎣ 0 e ⎦
⎡ iπ i
π
i
π π
i ⎤
⎡ 1 i π6 π
1 i6 ⎤ ⎢ e 6 + e 2 e 6 +e 2 ⎥
− e ⎥ −
⎡ 1 1⎤ ⎢ 2
e
2 ⎥=⎢ 2 2 ⎥
=⎢ ⎥⎢ π ⎢ π π ⎥
⎣ −1 1⎦ ⎢ 1 i 2
π π π
1 i2 ⎥ ⎢ i6
e 6 +e 2 ⎥
i i i
⎢ e e ⎥ e +e 2
⎣2 2 ⎦ ⎢⎢ − ⎥
⎥⎦
⎣ 2 2
⎡ 3 3i 3 i⎤
⎢ + − + ⎥
⎛πM ⎞ ⎛πM ⎞ ⎢ 4 4 4 4⎥
⇒ cos ⎜ ⎟ + i sin ⎜ ⎟=
⎝ 6 ⎠ ⎝ 6 ⎠ ⎢ 3 i 3 3i ⎥
⎢− + + ⎥
⎣ 4 4 4 4 ⎦
⎡ 3 3 ⎤ ⎡ 3i i⎤
⎢ − ⎥
⎛πM ⎞
⇒ cos ⎜
⎛πM ⎞ ⎢ 4 4 ⎥+⎢4 4⎥
⎟ + i sin ⎜ ⎟= ⎢ ⎥
⎝ 6 ⎠ ⎝ 6 ⎠ ⎢ 3 3 ⎥ ⎢i 3i ⎥
⎢− ⎥
⎣ 4 4 ⎦ ⎢⎣ 4 4 ⎥⎦
Thus

⎛πM ⎞ 3 ⎛ 1 −1 ⎞
cos ⎜ ⎟= ⎜ ⎟
⎝ 6 ⎠ 4 ⎝ −1 1 ⎠
1 1 1
Q4. The sum of the infinite series 1 − + − + ... is
3 5 7
π π
(a) 2π (b) π (c) (d)
2 4
Ans. : (d)
Solution: The series for tan −1 x for x > ,1 is given by
π 1 1 1 1
tan − x = − + 3 − 5 + 7 + ⋅⋅⋅⋅
2 x 3x 5x 7 x
Putting x = 1 , we obtain
π ⎛ 1 1 1 ⎞ π π ⎛ 1 1 1 ⎞
tan −1 1 = − ⎜1 − + − + ⋅⋅⋅ ⎟ ⇒ = − ⎜ 1 − + − ⋅⋅⋅⋅ ⎟
2 ⎝ 3 5 7 ⎠ 4 2 ⎝ 3 5 7 ⎠

                                                                                
Head office  Branch office 
 
fiziks, H.No. 40 D, G.F, Jia Sarai,  Anand Institute of Mathematics, 
 
Near IIT, Hauz Khas, New Delhi‐16  28‐B/6, Jia Sarai, Near IIT 
 
Phone: 011‐26865455/+91‐9871145498 Hauz Khas, New Delhi‐16 
                                                   
                                             Website: www.physicsbyfiziks.com                                                                                          
                                                           Email: fiziks.physics@gmail.com                                                                   3 
fiziks
Institute for NET/JRF, GATE, IIT‐JAM, JEST, TIFR and GRE in PHYSICAL SCIENCES 
 
Q5. A semicircular piece of paper is folded to make a cone with the centre of the semicircle
as the apex. The half-angle of the resulting cone would be:
(a) 90o (b) 60o (c) 45o (d) 30o
Ans. : (d)
Solution: When the semicircular piece of paper is folded to make a cone, the circumference of
base is equal to the circumference of the original semicircle. Let r be the radius of the
base of the core and R be the radius of the semicircle.
R
Hence, 2π r = π R ⇒ r = .
2
The stay height of the come will also be R . R

R/2 1 α R
Hence, sin α = =
R 2
Thus, α = 300 r

JEST-2015
Q6. Given an analytic function f ( z ) = φ ( x. y ) + iψ ( x, y ) , where φ ( x, y ) = x 2 + 4 x − y 2 + 2 y .

If C is a constant, which of the following relations is true?


(a) ψ ( x, y ) = x 2 y + 4 y + C (b) ψ ( x, y ) = 2 xy − 2 x + C

(c) ψ ( x. y ) = 2 xy + 4 y − 2 x + C (d) ψ ( x, y ) = x 2 y − 2 x + C

Ans. : (c)
Solution: u = Q ( x, y ) = x 2 + 4 x − y 2 + 2 y

∂u ∂v
From C.R. equation =
∂x ∂y
∂u ∂v
=−
∂y ∂x
∂u
= 2x + 4
∂x

                                                                                
Head office  Branch office 
 
fiziks, H.No. 40 D, G.F, Jia Sarai,  Anand Institute of Mathematics, 
 
Near IIT, Hauz Khas, New Delhi‐16  28‐B/6, Jia Sarai, Near IIT 
 
Phone: 011‐26865455/+91‐9871145498 Hauz Khas, New Delhi‐16 
                                                   
                                             Website: www.physicsbyfiziks.com                                                                                          
                                                           Email: fiziks.physics@gmail.com                                                                   4 
fiziks
Institute for NET/JRF, GATE, IIT‐JAM, JEST, TIFR and GRE in PHYSICAL SCIENCES 
 
∂v
= 2x + 4
∂y

v = 2 xy + 4 y + f ( x ) (i)

∂u
= −2 y + 2
∂y
∂v
= +2 y − 2
∂x
v = 2 xy + 2 x + f ( y )

2 xy + 4 y + f ( x ) = 2 xy − 2 x + f ( y ) (ii)

f ( x ) = 2 x, f ( y) = 4 y
v = 2 xy + 4 y − 2 x + c
Q7. If two ideal dice are rolled once, what is the probability of getting at least one ‘6’?
11 1 10 5
(a) (b) (c) (d)
36 36 36 36
Ans: (a)
Solution: Number of point in sample space n ( S ) = 11

⎡⎣(1, 6 ) , ( 2, 6 ) , ( 3, 6 ) , ( 4, 6 ) , ( 5, 6 ) , ( 6,1) , ( 6, 2 ) , ( 6,3) , ( 6, 4 ) , ( 6,5 ) , ( 6, 6 ) ⎤⎦

Number of point in population n ( P ) = 62 = 36

n(S ) 11
Probability that at least one six on face of dice = =
n ( P) 36

                                                                                
Head office  Branch office 
 
fiziks, H.No. 40 D, G.F, Jia Sarai,  Anand Institute of Mathematics, 
 
Near IIT, Hauz Khas, New Delhi‐16  28‐B/6, Jia Sarai, Near IIT 
 
Phone: 011‐26865455/+91‐9871145498 Hauz Khas, New Delhi‐16 
                                                   
                                             Website: www.physicsbyfiziks.com                                                                                          
                                                           Email: fiziks.physics@gmail.com                                                                   5 
fiziks
Institute for NET/JRF, GATE, IIT‐JAM, JEST, TIFR and GRE in PHYSICAL SCIENCES 
 
⎛ x4 x2 ⎞
−⎜ + ⎟
What is the maximum number of extrema of the function f ( x ) = Pk (x )e
⎜ 4 2 ⎟
⎝ ⎠
Q8. where

x ∈ (− ∞, ∞ ) and Pk ( x ) is an arbitrary polynomial of degree k ?

(a) k + 2 (b) k + 6 (c) k + 3 (d) k


Ans: (c)
⎛ x4 x2 ⎞
−⎜ + ⎟
Solution: f ( x ) = Px ( x ) e ⎝ 4 2⎠

⎛ x4 x2 ⎞
−⎜ + ⎟
(
f ′ ( x ) = ⎡⎣ Px′ ( x ) + Px ( x ) − x3 + x ⎤⎦ e ) ⎝ 2 2⎠
⇒ f ′( x) = 0

( )
= ⎣⎡ Px ( x ) x3 + x − P ′ ( x ) ⎦⎤ = 0 is polynomial if order k + 3
From the sign scheme maximum number of extrema = k + 3
xe xs xn
Q9. The Bernoulli polynominals Bn (s ) are defined by, = B (s )
∑ n n! . Which one of the
ex −1
following relations is true?
xe x (1− s ) xn xe ( )
x 1− s
xn
(a) = B ( s )
∑ n (n + 1)! (b) = ∑ n
B ( s )( −1)
n

ex −1 ex −1 ( n + 1)!
xe x (1− s ) n
xe x (1− s ) n
(c) = ∑ n
B (− s )(− 1) n x
(d) = ∑ n
B (s )(− 1) n x

ex −1 n! ex −1 n!
Ans: (d)
xe xS xn
Solution: = B (
∑ n nS )
ex − 1

xe x( S −1) xn
Put S = ( S − 1) , x = ∑ Bn ( S − 1)
e −1 n

Bn ( S − 1) = ( −1) B ( S )
n

xe x( S −1) n x
n
= ∑ Bn ( S )( −1)
ex − 1 n

                                                                                
Head office  Branch office 
 
fiziks, H.No. 40 D, G.F, Jia Sarai,  Anand Institute of Mathematics, 
 
Near IIT, Hauz Khas, New Delhi‐16  28‐B/6, Jia Sarai, Near IIT 
 
Phone: 011‐26865455/+91‐9871145498 Hauz Khas, New Delhi‐16 
                                                   
                                             Website: www.physicsbyfiziks.com                                                                                          
                                                           Email: fiziks.physics@gmail.com                                                                   6 
fiziks
Institute for NET/JRF, GATE, IIT‐JAM, JEST, TIFR and GRE in PHYSICAL SCIENCES 
 
Q10. Consider the differential equation G ′(x ) + kG ( x ) = δ ( x ) ; where k is a constant. Which
following statements is true?
(a) Both G ( x ) and G′(x ) are continuous at x = 0 .
(b) G (x ) is continuous at x = 0 but G′(x ) is not.
(c) G ( x ) is discontinuous at x = 0 .
(d) The continuity properties of G ( x ) and G′(x ) at x = 0 depends on the value of k .
Ans: (c)
1

99
Q11. The sum m =1
is equal to
m +1 + m
1
99 − 1
(a) 9 (b) (c)
( 99 − 1 ) (d) 11

Ans: (a)
1

99
Solution: m =1
m +1 + m
m +1 − m
∑ ( m + 1) − = ∑ m =1 m + 1 − m
99 99
m =1
m
= 2 − 1 + 3 − 2...... + 100 − 99

= 100 − 1 = 10 − 1 = 9
JEST-2014
Q12. What are the solutions to f ′′ ( x ) − 2 f ′ ( x ) + f ( x ) = 0 ?

(a) c1e x / x (b) c1 x + c 2 / x (c) c1 xe x + c 2 (d) c1e x + c 2 xe x


Ans.: (d)

Solution: Auxilary equation D 2 − 2 D + 1 = 0 ⇒ ( D − 1) = 0 ⇒ D = +1, +1


2

∵ Roots are equal then f ( x ) = ( c1 + c2 x ) e x ⇒ f ( x ) = c1e x + c2 xe x


2. 2
Q13. The value of ∫ 0. 2
xe x dx by using the one-segment trapezoidal rule is close to
(a) 11.672 (b) 11.807 (c) 20.099 (d) 24.119
Ans.: (c)
h
Solution: h = 2.2 − 0.2 = 2 ⇒ I = ⎡⎣ y ( 2.2 ) + y ( 0.2 ) ⎤⎦ = 20.099 ∵ y = xe x
2
                                                                                
Head office  Branch office 
 
fiziks, H.No. 40 D, G.F, Jia Sarai,  Anand Institute of Mathematics, 
 
Near IIT, Hauz Khas, New Delhi‐16  28‐B/6, Jia Sarai, Near IIT 
 
Phone: 011‐26865455/+91‐9871145498 Hauz Khas, New Delhi‐16 
                                                   
                                             Website: www.physicsbyfiziks.com                                                                                          
                                                           Email: fiziks.physics@gmail.com                                                                   7 
fiziks
Institute for NET/JRF, GATE, IIT‐JAM, JEST, TIFR and GRE in PHYSICAL SCIENCES 
 
Q14. Given the fundamental constants (Planck’s constant), G (universal gravitation
constant) and c (speed of light), which of the following has dimension of length?

G G G c
(a) (b) (c) (d)
c3 c5 c3 8πG
Ans.: (a)

[ ][ ]
1
⎡ ML2T −1 M −1 L3T − 2 ⎤ 2
[ ]
1

⎥ = L =L
2
Solution: ⎢ 3 −3
2

⎣ LT ⎦

= ⎡⎣ ML2T −1 ⎤⎦ , G =
gr 2
m
[
= M −1 L3T − 2 ]
Q15. The Laplace transformation of e −2t sin 4t is
4 4
(a) (b)
s + 4s + 25
2
s + 4s + 20
2

4s 4s
(c) (d)
s + 4s + 20
2
2s + 4 s + 20
2

Ans.: (b)
b
Solution: ∵ L ⎡⎣e − at sin bt ⎤⎦ =
(s + a)
2
+ b2

4 4
⇒ L ⎡⎣e−2t sin 4t ⎤⎦ = =
( s + 2) s + 4s + 20
2 2
+4 2

Q16. Let us write down the Lagrangian of a system as L( x, x, x ) = mxx + kx 2 + cxx . What is the
dimension of c ?
(a) MLT −3 (b) MT −2 (c) MT (d) ML2T −1
Ans.: (c)
Solution: According to dimension rule same dimension will be added or subtracted then
dimension of Mxx = dimension of Cxx
⎡⎣ ML2T −2 ⎤⎦ = [C ] [ L ] ⎡⎣ LT −3 ⎤⎦

[C ] = [ML2 T−3 ] = [MT ]


2 −2

[L M ]
                                                                                
Head office  Branch office 
 
fiziks, H.No. 40 D, G.F, Jia Sarai,  Anand Institute of Mathematics, 
 
Near IIT, Hauz Khas, New Delhi‐16  28‐B/6, Jia Sarai, Near IIT 
 
Phone: 011‐26865455/+91‐9871145498 Hauz Khas, New Delhi‐16 
                                                   
                                             Website: www.physicsbyfiziks.com                                                                                          
                                                           Email: fiziks.physics@gmail.com                                                                   8 
fiziks
Institute for NET/JRF, GATE, IIT‐JAM, JEST, TIFR and GRE in PHYSICAL SCIENCES 
 

Q17. The Dirac delta function δ ( x ) satisfies the relation ∫ f ( x )δ ( x )dx = f (0 ) for a well
−∞

behaved function f (x ) . If x has the dimension of momentum then


(a) δ ( x ) has the dimension of momentum

(b) δ ( x ) has the dimension of (momentum)


2

(c) δ ( x ) is dimensionless

(d) δ ( x ) has the dimension of (momentum)


−1

Ans.: (d)

Solution: ∫ f ( x )δ ( x ) dx = f (0 )
−∞

f ( x ) δ ( x ) dx = f (0) ⇒ [ f ( x )]δ (x ) ⋅ P = [ f (0)] ⇒ δ ( x ) = P −1 [ ]


Since, [ f (x )] = [ f (0)]
If F ( x ) = αx + β is force M L T −2 [ ]
F (0 ) = β is also M L T 2 [ ]
Q18. The value of limit
z 10 + 1
lim 6
z →i z + 1

is equal to
(a) 1 (b) 0 (c) -10/3 (d) 5/3
Ans.: (d)
z 10 + 1 10 z 9 10 z 4 10 5
Solution: lim ⇒ lim ⇒ lim ⇒ =
z →i z +1
6 z →i 6 z 5 z →i 6 6 3

Q19. The value of integral


sin z
I =∫ dz
c 2z − π
with c a circle z = 2 , is

(a) 0 (b) 2π i (c) π i (d) − π i


                                                                                
Head office  Branch office 
 
fiziks, H.No. 40 D, G.F, Jia Sarai,  Anand Institute of Mathematics, 
 
Near IIT, Hauz Khas, New Delhi‐16  28‐B/6, Jia Sarai, Near IIT 
 
Phone: 011‐26865455/+91‐9871145498 Hauz Khas, New Delhi‐16 
                                                   
                                             Website: www.physicsbyfiziks.com                                                                                          
                                                           Email: fiziks.physics@gmail.com                                                                   9 
fiziks
Institute for NET/JRF, GATE, IIT‐JAM, JEST, TIFR and GRE in PHYSICAL SCIENCES 
 
Ans.: (c)
sin z π
Solution: I = ∫ pole ⇒ 2 z − π = 0 ⇒ z =
C 2z − π 2
π
Residue at z = ∵ z = 2 so it will be lies within the contour
2
eiz
I ( emg ) = ∫ = ∑ R × 2πi
C ⎛ π⎞
2⎜ z − ⎟
⎝ 2⎠

⎛ π ⎞ iz
⎜z− ⎟ e eiπ / 2 i 1
=⎝
2⎠
Res = = (taking imaginary part) ; Residue =
⎛ π⎞ 2 2 2
2⎜ z − ⎟
z=
π
⎝ 2
2⎠

1
Now I = × 2π i = π i
2

JEST-2013
Q20. A box contains 100 coins out of which 99 are fair coins and 1 is a double-headed coin.
Suppose you choose a coin at random and toss it 3 times. It turns out that the results of all
3 tosses are heads. What is the probability that the coin you have drawn is the double-
headed one?
(a) 0.99 (b) 0.925 (c) 0.75 (d) 0.01
Ans.: (c)
Re(z 2 ) + Im(z 2 )
Q21. Compute lim
z →0 z2
(a) The limit does not exist. (b) 1
(c) –i (d) -1
Ans.: (a)

Solution: lim
( )
Re z 2 + Im z 2 ( ) = lim x 2
− y 2 + 2 xy
⇒ lim
x 2 − y 2 + 2 xy
=1
z →0 z2 z →0 x 2 − y 2 + 2ixy y = 0 x 2 − y 2 + 2ixy
x →0

                                                                                
Head office  Branch office 
 
fiziks, H.No. 40 D, G.F, Jia Sarai,  Anand Institute of Mathematics, 
 
Near IIT, Hauz Khas, New Delhi‐16  28‐B/6, Jia Sarai, Near IIT 
 
Phone: 011‐26865455/+91‐9871145498 Hauz Khas, New Delhi‐16 
                                                   
                                             Website: www.physicsbyfiziks.com                                                                                          
                                                           Email: fiziks.physics@gmail.com                                                                   10 
fiziks
Institute for NET/JRF, GATE, IIT‐JAM, JEST, TIFR and GRE in PHYSICAL SCIENCES 
 
x − y + 2 xy
2 2
x − y + 2 xy
2 2
lim 2 = 1 and lim 2 = −i
x = 0 x − y + 2ixy
2 y = x x − y 2 + 2ixy
y →0 x →0

Q22. The vector field xziˆ + yˆj in cylindrical polar coordinates is

(a) ρ (z cos 2 φ + sin 2 φ )eˆ ρ + ρ sin φ cos φ (1 − z )eˆφ

(b) ρ (z cos 2 φ + sin 2 φ )eˆ ρ + ρ sin φ cos φ (1 + z )eˆφ

(c) ρ (z sin 2 φ + cos 2 φ )eˆ ρ + ρ sin φ cos φ (1 + z )eˆφ

(d) ρ (z sin 2 φ + cos 2 φ )eˆ ρ + ρ sin φ cos φ (1 − z )eˆφ

Ans.: (a)
Solution: A = xziˆ + yˆj ⇒ Ax = xz , Ay = y, Az = 0

Aρ = A ⋅ eˆρ = Ax ( xˆ ⋅ eˆρ ) + Ay ( yˆ ⋅ eˆρ ) + Az ( zˆ ⋅ eˆρ )

⇒ Aρ = ρ cos φ z ( cos φ ) + ρ sin φ ( sin φ ) + 0 ⇒ Aρ = ρ cos φ 2 z + ρ sin 2 φ eˆρ ( )


Aφ = A ⋅ eˆφ = Ax ( xˆ ⋅ eˆφ ) + Ay ( yˆ ⋅ eˆφ ) + Az ( zˆ ⋅ eˆφ )

⇒ Aφ = ρ cos φ ( − sin φ ) z + ρ sin φ ⋅ cos φ ⇒ Aφ = ρ cos φ ⋅ sin φ (1 − z ) eˆφ

( )
A = Aρ eˆρ + Aφ eˆφ + Az eˆz = ρ cos 2 φ z + sin 2 φ eˆρ + ρ cos φ sin φ (1 − z ) eˆφ

Q23. There are on average 20 buses per hour at a point, but at random times. The probability
that there are no buses in five minutes is closest to
(a) 0.07 (b) 0.60 (c) 0.36 (d) 0.19
Ans.: (d)
Q24. Two drunks start out together at the origin, each having equal probability of making a
step simultaneously to the left or right along the x axis. The probability that they meet
after n steps is
1 2n! 1 2n! 1 1
(a) (b) (c) 2n! (d) n!
4 n n!2 2 n n!2 2n 4n
Ans.: (a)

                                                                                
Head office  Branch office 
 
fiziks, H.No. 40 D, G.F, Jia Sarai,  Anand Institute of Mathematics, 
 
Near IIT, Hauz Khas, New Delhi‐16  28‐B/6, Jia Sarai, Near IIT 
 
Phone: 011‐26865455/+91‐9871145498 Hauz Khas, New Delhi‐16 
                                                   
                                             Website: www.physicsbyfiziks.com                                                                                          
                                                           Email: fiziks.physics@gmail.com                                                                   11 
fiziks
Institute for NET/JRF, GATE, IIT‐JAM, JEST, TIFR and GRE in PHYSICAL SCIENCES 
 
Solution: Into probability of taking ' r ' steps out of N steps
r N −r
⎛1⎞ ⎛1⎞
= N Cr ⎜ ⎟ ⎜ ⎟
⎝2⎠ ⎝2⎠
total steps = N = n + n = 2n
for taking probability of n steps out of N
n N −n n 2n−n 2n
⎛1⎞ ⎛1⎞ N! ⎛1⎞ ⎛1⎞ 2n ! ⎛ 1 ⎞ 2n !
P = N Cn ⎜ ⎟ ⎜ ⎟ = = =
⎝2⎠ ⎝2⎠ ( N − n ) !n ! ⎜⎝ 2 ⎟⎠ ⎜⎝ 2 ⎟⎠ ⎜ ⎟
n !n ! ⎝ 2 ⎠ ( n !)
2
4n

Q25. What is the value of the following series?


2 2
⎛ 1 1 ⎞ ⎛ 1 1 ⎞
⎜1 − + − ....⎟ + ⎜1 − + − ... ⎟
⎝ 2! 4! ⎠ ⎝ 3! 5! ⎠
(a) 0 (b) e (c) e 2 (d) 1
Ans.: (d)
θ2 θ4 θ3 θ5
Solution: ⇒ cos θ = 1 − + ..... , sin θ = θ − + .....
2! 4! 3! 5!
2 2
⎛ 1 1 ⎞ ⎛ 1 1⎞
⇒ ⎜1 − + ...⎟ + ⎜1 − + ⎟ ⇒ cos12 + sin 12 = 1 ∵ sin 2 θ + cos 2 θ = 1
⎝ 2! 4! ⎠ ⎝ 3! 5! ⎠

Q26. If the distribution function of x is f ( x ) = xe − x / λ over the interval 0 < x < ∞, the mean
value of x is
λ
(a) λ (b) 2λ (c) (d) 0
2
Ans.: (b)
x −x
∞ ∞ − ∞
∫ xf ( x )dx ∫ x.xe dx λ

2
x e λ dx
Solution: ∵ it is distribution function so x = −∞

= 0
⇒ 0
−x
= 2λ
f ( x ) dx
x

−∞ ∫

0

xe λ dx ∫ 0

xe λ dx

                                                                                
Head office  Branch office 
 
fiziks, H.No. 40 D, G.F, Jia Sarai,  Anand Institute of Mathematics, 
 
Near IIT, Hauz Khas, New Delhi‐16  28‐B/6, Jia Sarai, Near IIT 
 
Phone: 011‐26865455/+91‐9871145498 Hauz Khas, New Delhi‐16 
                                                   
                                             Website: www.physicsbyfiziks.com                                                                                          
                                                           Email: fiziks.physics@gmail.com                                                                   12 
fiziks
Institute for NET/JRF, GATE, IIT‐JAM, JEST, TIFR and GRE in PHYSICAL SCIENCES 
 
JEST-2012
∞ ln x
Q27. The value of the integral ∫ (x
0 2
+ 1)
2
dx is

−π −π π
(a) 0 (b) (c) (d)
4 2 2
Ans. : (b)
∞ ∞
ln x ln z
Solution: ∫ dx = ∫ dz
(x + 1) (z + 1)
2 2 2 2
0 0

2 ∞ 2
⎛ ln z ⎞ ⎛ ln z ⎞
Let us consider new function f ( z ) = ⎜ 2 ⎟ , then I = ∫ ⎜ 2 ⎟ dz
⎝ z +1⎠ 0⎝
z +1⎠
R
Pole at z = ±i is simple pole of second order.

Residue at z = i is
A r
( ln z ) d ( ln z )
2 2
d
= ( z − i)
2
=
( z − i ) ( z + i ) dz ( z + i )
2 2 2
dz B

1 1 •
( z + i ) 2 ( ln z ) . − ( ln z ) .2 ( z + i ) ( z + i ) 2 ln ( z ) − ( ln z ) .2
2 2 2

= z = z
( z + i) ( z + i)
4 3

2
1 iπ ⎛ iπ ⎞ π2
( 2i ) 2 × ln i − ( ln i ) ⋅ 2 4 2 − ⎜⎝ 2 ⎟⎠ × 2 2π i +
2

= i = = 2
( 2i ) −8 i −8 i
3

−π π 2
⇒ Res z =i = + i
4 16
−π π 2
Similarly at z = −i ; Res z =− i = − i
4 16
∞ 2
⎛ ln z ⎞ ⎛ −π π 2 π π 2 ⎞
I = ∫ ⎜ 2 ⎟ dz = 2π i ⎜ + i − − i ⎟ = −π 2i
0⎝
z + 1 ⎠ ⎝ 4 16 4 16 ⎠

⎛ ⎞ ⎛ ⎞
−π 2i = ⎜ ∫ ∫ ∫ ∫ ⎟ f ( z ) dz = ⎜ ∫ ∫ ⎟ f ( z ) dz ; ∫∫ vanish
⎝R AB r ⎠ ⎝AB ⎠ AB

                                                                                
Head office  Branch office 
 
fiziks, H.No. 40 D, G.F, Jia Sarai,  Anand Institute of Mathematics, 
 
Near IIT, Hauz Khas, New Delhi‐16  28‐B/6, Jia Sarai, Near IIT 
 
Phone: 011‐26865455/+91‐9871145498 Hauz Khas, New Delhi‐16 
                                                   
                                             Website: www.physicsbyfiziks.com                                                                                          
                                                           Email: fiziks.physics@gmail.com                                                                   13 
fiziks
Institute for NET/JRF, GATE, IIT‐JAM, JEST, TIFR and GRE in PHYSICAL SCIENCES 
 
Along path A; z = − x + iε and along path B; z = − x − iε

⎛ ⎞ 0 ⎡
ln ( − x + iε ) ⎤ ∞⎡
ln ( − x − iε ) ⎤
Thus −π 2i = ⎜ ∫ ∫ ⎟ f ( z ) dz = − ∫ ⎢ ⎥dx − ∫0 ⎢ ( − x − iε )2 + 1⎥⎥dx

( )
2
⎝AB ⎠ ∞⎢⎣ − x + iε + 1 ⎥
⎦ ⎣ ⎦
2 2
⎡ ln ( − x + iε ) ⎤
∞ ∞⎡
ln ( − x − iε ) ⎤
⇒ −π 2i = ∫ ⎢ ⎥ dx − ∫0 ⎢ ( − x − iε )2 + 1⎥⎥ dx

( )
2
0 ⎢
⎣ − x + iε + 1 ⎥
⎦ ⎣ ⎦

⎡ ln ( x ) + iπ ⎤ ⎡ ln ( x ) − iπ ⎤
∞ ∞ 2 2

⇒ −π i = ∫ ⎢ 2
⎥ dx − ∫ ⎢ ⎥ dx; ε →0
0 ⎣
1 + x 2
⎦ 0 ⎣
1 + x 2

( ln ( x ) + iπ ) − ( ln ( x ) − iπ ) dx = 4π i
2 2
∞ ∞
ln x
⇒ −π i = ∫ 2

(1 + x ) 2 2
(x + 1)
2 2
0 0


ln x −iπ 2 −π
⇒∫ = =
(x + 1) 4π i
2
0
2 4

∫ [x]e

−x
Q28. If [x] denotes the greatest integer not exceeding x, then dx
0

1 e −1 e
(a) (b) 1 (c) (d)
e −1 e e −1
2

Ans.: (a)
Solution: [x ]
0 ≤ x < 1 = [x ] = 0 , 1 ≤ x < 2 = [x ] = 1 , 2 ≤ x < 3 = [x ] = 2
∞ 1 2 3 4

∫ [x]e dx = ∫ [x ]e dx + ∫ [x ]e dx + ∫ [x ]e dx + ∫ [x ]e − x dx
−x −x −x −x
now
0 0 1 2 3

2 3 4
⇒ 0 + ∫ 1.e − x dx + ∫ 2.e − x dx + ∫ 3.e − x dx
1 2 3

[
⇒ − e−x ]2
1 (
+ 2 − e −x )
3
2 (
+ 3 − e −x )
4
3 + ....

⇒ e −1 − e −2 + 2e −2 − 2e −3 + 3e −3 − 3e −4 + 4e −4 − 4e −5 +
⇒ e −1 + e −2 + −e −3 + e −4 + .....∞

                                                                                
Head office  Branch office 
 
fiziks, H.No. 40 D, G.F, Jia Sarai,  Anand Institute of Mathematics, 
 
Near IIT, Hauz Khas, New Delhi‐16  28‐B/6, Jia Sarai, Near IIT 
 
Phone: 011‐26865455/+91‐9871145498 Hauz Khas, New Delhi‐16 
                                                   
                                             Website: www.physicsbyfiziks.com                                                                                          
                                                           Email: fiziks.physics@gmail.com                                                                   14 
fiziks
Institute for NET/JRF, GATE, IIT‐JAM, JEST, TIFR and GRE in PHYSICAL SCIENCES 
 
e −1
1 ⎛ e −2
−2 +1 −1 ⎞
⇒ = ∵ ⎜ r = −
= e = e ⎟
1 − e −1 e − 1 ⎝ e 1

1 1 1
Q29. As x → 1, the infinite series x − x 3 + x 5 − x 7 + .......
3 5 7
(a) diverges (b) converges to unity
(c) converges to π / 4 (d) none of the above
Ans.: (c)
x3 x5 x7 π
Solution: tan −1 x = x − + − + ....... ⇒ tan −1 1 =
3 5 7 4
Q30. What is the value of the following series?
2 2
⎛ 1 1 ⎞ ⎛ 1 1 ⎞
⎜1 + + + ....⎟ − ⎜1 + + + .....⎟
⎝ 2! 4! ⎠ ⎝ 3! 5! ⎠
(a) 0 (b) e (c) e2 (d) 1
Ans.: (d)
12 13
Solution: e1 = 1 + 1 + + +−,
2! 3!
12 13
e −1 = 1 − 1 + − .....
2! 3!
e1 + e −1 1 1
cosh 1 = = 1 + + + ....
2 2! 4!

sinh 1 =
(e 1
)
− e −1 1 1
= 1 + + + ...
2 3! 5!
i.e cos 2 h1 − sin 2 h1 = 1
Q31. An unbiased die is cast twice. The probability that the positive difference (bigger -
smaller) between the two numbers is 2 is
(a) 1 / 9 (b) 2 / 9 (c) 1 / 6 (d) 1 / 3
Ans.: (a)
n( E )
Solution: p(2 ) =
n (S )

                                                                                
Head office  Branch office 
 
fiziks, H.No. 40 D, G.F, Jia Sarai,  Anand Institute of Mathematics, 
 
Near IIT, Hauz Khas, New Delhi‐16  28‐B/6, Jia Sarai, Near IIT 
 
Phone: 011‐26865455/+91‐9871145498 Hauz Khas, New Delhi‐16 
                                                   
                                             Website: www.physicsbyfiziks.com                                                                                          
                                                           Email: fiziks.physics@gmail.com                                                                   15 
fiziks
Institute for NET/JRF, GATE, IIT‐JAM, JEST, TIFR and GRE in PHYSICAL SCIENCES 
 
The number of ways to come positive difference [(3,1), (4, 2 ), (5, 3), (6, 4 )]
4 1
p(2 ) = =
36 9
Q32. For an N x N matrix consisting of all ones,
(a) all eigenvalues = 1 (b) all eigenvalues = 0
(c) the eigenvalues are 1, 2, …., N (d) one eigenvalue = N, the others = 0
Ans.: (d)
⎡1 1⎤
Solution: ⎢ ⎥ = 0, 2
⎣1 1⎦
⎡1 1 1⎤
⎢1 1 1⎥ = 0, 0, 3
⎢ ⎥
⎢⎣1 1 1⎥⎦

so far N × N matrix one eigen value is N and another’s eigen value is zero

                                                                                
Head office  Branch office 
 
fiziks, H.No. 40 D, G.F, Jia Sarai,  Anand Institute of Mathematics, 
 
Near IIT, Hauz Khas, New Delhi‐16  28‐B/6, Jia Sarai, Near IIT 
 
Phone: 011‐26865455/+91‐9871145498 Hauz Khas, New Delhi‐16 
                                                   
                                             Website: www.physicsbyfiziks.com                                                                                          
                                                           Email: fiziks.physics@gmail.com                                                                   16 
fiziks
Institute for NET/JRF, GATE, IIT‐JAM, JEST, TIFR and GRE in PHYSICAL SCIENCES 
 
Classical Mechanics
JEST-2016

Q1. A hoop of radius a rotates with constant angular velocity ω about


the vertical axis as shown in the figure. A bead of mass m can slide
on the hoop without friction. If g < ω 2 a at what angle θ apart from
a
dθ d θ 2
0 and π is the bead stationary (i.e., = 2 = 0 )?
dt dt
θ
πg g
(a) tan θ = 2 (b) sin θ =
ω a ω 2a
g g
(c) cos θ = (d) tan θ =
ω a2
πω 2 a
Ans: (c)

Solution: L =
1 2 2
2
(
ma θ + sin 2 θφ 2 + mga cos θ )
d ⎛ ∂L ⎞ ⎛ ∂L ⎞
⎜ ⎟−⎜ ⎟ = 0 ⇒ ma θ − ma sin θ cos θφ + mga sin θ = 0
dt ⎝ ∂θ ⎠ ⎝ ∂θ ⎠
2 2 2
( )
dθ d 2θ
dt
(
= 2 = 0 ⇒ − ma 2 sin θ cos θφ 2 + mga sin θ = 0 ,
dt
)
g
φ = ω and g < ω 2 a then cos θ =
ω 2a

                                                                                
Head office  Branch office 
 
fiziks, H.No. 40 D, G.F, Jia Sarai,  Anand Institute of Mathematics, 
 
Near IIT, Hauz Khas, New Delhi‐16  28‐B/6, Jia Sarai, Near IIT 
 
Phone: 011‐26865455/+91‐9871145498 Hauz Khas, New Delhi‐16 
                                                   
                                             Website: www.physicsbyfiziks.com                                                                                          
                                                           Email: fiziks.physics@gmail.com                                                                   1 
fiziks
Institute for NET/JRF, GATE, IIT‐JAM, JEST, TIFR and GRE in PHYSICAL SCIENCES 
 
Q2. The central force which results in the orbit r = a (1 + cos θ ) for a particle is proportional

to:
(a) r (b) r 2 (c) r −2 (d) None o the above
Ans: (c)
1 1
Solution: r = a (1 + cos θ ) ⇒ u = = =
r a (1 + cos θ )

If J is angular momentum and m is mass of particle

J 2 ⎛ d 2u ⎞ ⎛1⎞ du sin θ
− ⎜ 2 +u⎟ = f ⎜ ⎟ ⇒ =
m ⎝ dθ ⎝ u ⎠ dθ a (1 + cos θ )
2

d 2u sin 2 θ cos θ
= −2 +
dθ a (1 + cos θ ) a (1 + cos θ )
2 3 2

J 2 ⎛ d 2u ⎞ J2 ⎛ sin 2 θ cos θ 1 ⎞ ⎛1⎞


− ⎜ 2 + u ⎟ = − ⎜ − 2 + + ⎟= f ⎜ ⎟
m ⎝ dθ ⎠ m ⎜ a (1 + cos θ ) a (1 + cos θ )
3 2
a (1 + cos θ ) ⎟ ⎝u⎠
⎝ ⎠

J2 ⎛ 1 − cos 2 θ cos θ 1 ⎞ ⎛1⎞


− ⎜ −2 + + ⎟= f ⎜ ⎟
m ⎜ a (1 + cos θ ) a (1 + cos θ )
3 2
a (1 + cos θ ) ⎟ ⎝u⎠
⎝ ⎠
1 1
Put u = , cos θ = and solving
a (1 + cos θ ) au − 1

⎛1⎞
f ⎜ ⎟ ∝ u 2 so f ( r ) ∝ r −2
⎝u⎠

                                                                                
Head office  Branch office 
 
fiziks, H.No. 40 D, G.F, Jia Sarai,  Anand Institute of Mathematics, 
 
Near IIT, Hauz Khas, New Delhi‐16  28‐B/6, Jia Sarai, Near IIT 
 
Phone: 011‐26865455/+91‐9871145498 Hauz Khas, New Delhi‐16 
                                                   
                                             Website: www.physicsbyfiziks.com                                                                                          
                                                           Email: fiziks.physics@gmail.com                                                                   2 
fiziks
Institute for NET/JRF, GATE, IIT‐JAM, JEST, TIFR and GRE in PHYSICAL SCIENCES 
 
Q3. Light takes approximately 8 minutes to travel from the Sun to the Earth. Suppose in the
frame of the Sun an event occurs at t = 0 at the Sun and another event occurs on Earth at
t = 1 minute. The velocity of the inertial frame in which both these events are
simultaneous is:
c
(a) with the velocity vector pointing from Earth to Sun
8
c
(b) with the velocity vector pointing from Sun to Earth : -
8
(c) The events can never be simultaneous - no such frame exists
2
⎛1⎞
(d) c 1 − ⎜ ⎟ with velocity vector Pointing from to Earth
⎝8⎠
Ans: (a)
Solution: x2' − x1' = c × 8 × 60

t2' − t1' = 60

vx2' vx1'
tt2' +
'
+
( )
1
t2 − t1 = 0 ⇒ c2 − c 2 = 0 ⇒ t ' − t ' + v x' − x' = 0
2 1 2 1
v2 v2 c2
1− 2 1− 2
c c

t2' − t1' +
c
v '
2 ( ) v
x2 − x1' ⇒ 60 + 2 c × 8 × 60 ⇒ v = −
c
c
8
c
Negative sign indicate frame is moving with the velocity vector pointing from Earth to
8
Sun.

                                                                                
Head office  Branch office 
 
fiziks, H.No. 40 D, G.F, Jia Sarai,  Anand Institute of Mathematics, 
 
Near IIT, Hauz Khas, New Delhi‐16  28‐B/6, Jia Sarai, Near IIT 
 
Phone: 011‐26865455/+91‐9871145498 Hauz Khas, New Delhi‐16 
                                                   
                                             Website: www.physicsbyfiziks.com                                                                                          
                                                           Email: fiziks.physics@gmail.com                                                                   3 
fiziks
Institute for NET/JRF, GATE, IIT‐JAM, JEST, TIFR and GRE in PHYSICAL SCIENCES 
 
Q4. For the coupled system shown in the figure, the normal coordinates are x1 + x2 and

x1 − x2 corresponding to the normal frequencies ω0 and 3ω0 respectively.


x1 x2
k k k

m m

At t = 0 , the displacements are x1 = A , x2 = 0 , and the velocities are v1 = v2 = 0 . The

displacement of the second particle at time t is given by:

(a) x2 ( t ) =
A
2
(
cos (ω0t ) + cos ( 3ω0t )) (b) x2 ( t ) =
A
2
(
cos (ω0t ) − cos ( 3ω0t ))
(c) x2 ( t ) =
A
2
(
sin (ω0t ) − sin ( 3ω0t )) (d) x2 ( t ) =
A⎛

2⎝
sin (ω0t ) −
1
3
sin ( ⎞
3ω0t ⎟

)
Ans: (b)
Solution: Using boundary condition at t = 0 x2 = 0 and v2 = 0

Only x2 ( t ) =
A
2
(
cos (ω0t ) − cos ( 3ω0t )) will satisfied
Q5. A cylindrical shell of mass in has an outer radius b and an inner radius a . The moment
of inertia of the shell about the axis of the cylinder is:
1 1
(a)
2
(
m b2 − a 2 ) (b)
2
(
m b2 + a 2 ) (
(c) m b 2 + a 2 ) (
(d) m b 2 − a 2 )
Ans: (b)

( )
b m b m 2
∫ x 2π xdx ⇒
2 2
∫ x dm = b + a2
a (
π b −a2 2
) a 2

                                                                                
Head office  Branch office 
 
fiziks, H.No. 40 D, G.F, Jia Sarai,  Anand Institute of Mathematics, 
 
Near IIT, Hauz Khas, New Delhi‐16  28‐B/6, Jia Sarai, Near IIT 
 
Phone: 011‐26865455/+91‐9871145498 Hauz Khas, New Delhi‐16 
                                                   
                                             Website: www.physicsbyfiziks.com                                                                                          
                                                           Email: fiziks.physics@gmail.com                                                                   4 
fiziks
Institute for NET/JRF, GATE, IIT‐JAM, JEST, TIFR and GRE in PHYSICAL SCIENCES 
 
JEST-2015
Q6. The distance of a star from the Earth is 4.25 light years, as measured from the Earth. A
space ship travels from Earth to the star at a constant velocity in 4.25 years, according to
the clock on the ship. The speed of the space ship in units of the speed of light is,
1 1 2 1
(a) (b) (c) (d)
2 2 3 3
Ans: (b)
4.25 4.25
Solution: Proper life-time Δt0 = , Δt =
c v
Δt0 1
Δt = ⇒v= c
1− v / c2 2
2
Q7. A classical particle with total energy E moves under the influence of a potential
V ( x, y ) = 3x 3 + 2 x 2 y + 2 xy 2 + y 3 . The average potential energy, calculated over a long
time is equal to,
2E E E 2E
(a) (b) (c) (d)
3 3 5 5
Ans: None of the above is correct.
n
Solution: If one will use virial theorem then T = V if V ∝ r n according to problem n = 3
2
2 2
So E = T + V E = V + V V = E
3 5
But virial theorem is used only for conservative forces.
Force conservative ∇ × F = 0 where F = −∇V

(
∵V ( x, y ) = 3 x3 + 2 x 2 y + 2 y 2 x + y 3 ⇒ ∇V = 9 x 2 + 2 xy + 2 y 2 iˆ + 2 x 2 + 4 yx + 3 y 2 ˆj ) ( )
⇒ ∇ × F ≠ 0 i.e. non conservative in nature.
So we cannot use viral theorem. Therefore, none of the answer is correct

                                                                                
Head office  Branch office 
 
fiziks, H.No. 40 D, G.F, Jia Sarai,  Anand Institute of Mathematics, 
 
Near IIT, Hauz Khas, New Delhi‐16  28‐B/6, Jia Sarai, Near IIT 
 
Phone: 011‐26865455/+91‐9871145498 Hauz Khas, New Delhi‐16 
                                                   
                                             Website: www.physicsbyfiziks.com                                                                                          
                                                           Email: fiziks.physics@gmail.com                                                                   5 
fiziks
Institute for NET/JRF, GATE, IIT‐JAM, JEST, TIFR and GRE in PHYSICAL SCIENCES 
 
Q8, A chain of mass M and length L is suspended vertically with its lower end touching a
weighing scale. The chain is released and falls freely onto the scale. Neglecting the size
of the individual links, what is the reading of the scale when a length x of the chain has
fallen?
Mgx 2Mgx 3Mgx 4Mgx
(a) (b) (c) (d)
L L L L
Ans: (c)
dp Mgx d ( Δmv ) Mgx
Solution: Reading of scale = impulse + actual weight = + = +
dt L dt L
M ⎛ dx ⎞ Mgx Mv 2 Mgx 2 Mgx Mgx 3Mgx M
⇒ ⎜ ⎟v + = + = + = ∵ v 2 = 2 gx and Δm = dx
L ⎝ dt ⎠ L L L L L L L

Q9. A bike stuntman rides inside a well of frictionless surface given by z = a(x 2 + y 2 ) , under
the action of gravity acting in the negative z direction. g = − gzˆ What speed should he
maintain to be able to ride at a constant height z 0 without falling down?

(a) gz 0

(b) 3gz 0

(c) 2gz 0

(d) The biker will not be able to maintain a constant height, irrespective of speed.
Ans: (c)
Solution: z = a x 2 + y 2 ( )
Using equation of constrain, we must solve the given system in cylindrical co-ordinate.
z = ar 2 z = 2arr

L=
1
2
(
m r 2 + r 2θ + z 2 − mgz )
⇒L=
1
2
( )
1
m r 2 + r 2 θ + 4a 2 r 2 r 2 − mgar 2 = m r 2 1 + 4a 2 r 2 + r 2 θ 2
2
( ( ) )

                                                                                
Head office  Branch office 
 
fiziks, H.No. 40 D, G.F, Jia Sarai,  Anand Institute of Mathematics, 
 
Near IIT, Hauz Khas, New Delhi‐16  28‐B/6, Jia Sarai, Near IIT 
 
Phone: 011‐26865455/+91‐9871145498 Hauz Khas, New Delhi‐16 
                                                   
                                             Website: www.physicsbyfiziks.com                                                                                          
                                                           Email: fiziks.physics@gmail.com                                                                   6 
fiziks
Institute for NET/JRF, GATE, IIT‐JAM, JEST, TIFR and GRE in PHYSICAL SCIENCES 
 
Equation of motion
d ⎛ ∂L ⎞ ∂L
⎜ ⎟− =0
dt ⎝ ∂r ⎠ ∂r

( )
mr 1 + ra 2 r 2 + mr 2 4arr − mrθ 2 + 2mgar = 0

At z = z0 , r = 0, r = r0

So − mr0 θ 2 = 2mgar0

θ 2 = 2 ga ⇒ θ = 2 ga ,
v
r0
= 2 ga , v = 2 ga ⋅ r0 (z 0 = ar02 )
1/ 2
⎛z ⎞
= 2 ga ⋅ ⎜ 0 ⎟ ⇒ v = 2 gz0
⎝ a⎠

Q10. The Lagrangian of a particle is given by L = q 2 − qq . Which of the following statements


is true?
(a) This is a free particle
(b) The particle is experiencing velocity dependent damping
(c) The particle is executing simple harmonic motion
(d) The particle is under constant acceleration.
Ans: (a)
∂L d ⎛ ∂L ⎞
Solution: ∵ L = q 2 − qq ⇒ = 2q − q ⇒ ⎜ ⎟ = 2q − q
∂q dt ⎝ ∂q ⎠

d ⎛ ∂L ⎞ ∂L
∵ ⎜ ⎟− =0
dt ⎝ ∂q ⎠ ∂q

d 2q dq
⇒ 2q − q + q = 0 ⇒ 2q = 0 ⇒ 2 = 0 ⇒ = C ⇒ q = Ct + α
dt dt
Q11. How is your weight affected if the Earth suddenly doubles in radius, mass remaining the
same?
(a) Increases by a factor of 4 (b) Increases by a factor of 2
(c) Decreases by a factor of 4 (d) Decreases by a factor of 2
Ans: (a)
                                                                                
Head office  Branch office 
 
fiziks, H.No. 40 D, G.F, Jia Sarai,  Anand Institute of Mathematics, 
 
Near IIT, Hauz Khas, New Delhi‐16  28‐B/6, Jia Sarai, Near IIT 
 
Phone: 011‐26865455/+91‐9871145498 Hauz Khas, New Delhi‐16 
                                                   
                                             Website: www.physicsbyfiziks.com                                                                                          
                                                           Email: fiziks.physics@gmail.com                                                                   7 
fiziks
Institute for NET/JRF, GATE, IIT‐JAM, JEST, TIFR and GRE in PHYSICAL SCIENCES 
 
GM GM W
Solution: W = m ⋅ 2 and W ′ = m ⋅ ⇒W′ =
( 2R )
2
R 4

Q12. A spring of force constant k is stretched by x . It takes twice as much work to stretch a
x
second spring by . The force constant of the second spring is,
2
(a) k (b) 2k (c) 4k (d) 8k
Ans: (d)
Solution:
1
The relation between energy and maximum displacement is E = k1 A2
2
2
1 x 1 ⎛ x⎞ 1
For A = x ; E1 = k2 x 2 and For A = ; E1 = k2 ⎜ ⎟ = k2 x 2
2 2 2 ⎝ 2⎠ 8
1 1
∵ E2 = 2 E1 ∴ k2 x 2 = 2 × k1 x 2 ⇒ k2 = 8k1 ⇒ k2 = 8k
8 2

JEST-2014
Q13. A dynamical system with two generalized coordinates q1 and q2 has Lagrangian

L = q12 + q 22 . If p1 and p2 are the corresponding generalized momenta, the Hamiltonian


is given by
(a) ( p12 + p 22 ) / 4 (b) (q12 + q 22 ) / 4 (c) ( p12 + p 22 ) / 2 (d) ( p1 q1 + p 2 q 2 ) / 4
Ans.: (a)
Solution: H = ∑ qi pi − L = q1 p1 + q2 p2 − L

∂L p ∂L p
= p1 = 2q1 ⇒ q1 = 1 and = p 2 = 2q 2 ⇒ q 2 = 2
∂q1 2 ∂q 2 2

H=
p1
⋅ p1 +
p2
⋅ p2 −
p12 p 22
− ⇒H =
( p12 + p22 )
2 2 4 4 4

                                                                                
Head office  Branch office 
 
fiziks, H.No. 40 D, G.F, Jia Sarai,  Anand Institute of Mathematics, 
 
Near IIT, Hauz Khas, New Delhi‐16  28‐B/6, Jia Sarai, Near IIT 
 
Phone: 011‐26865455/+91‐9871145498 Hauz Khas, New Delhi‐16 
                                                   
                                             Website: www.physicsbyfiziks.com                                                                                          
                                                           Email: fiziks.physics@gmail.com                                                                   8 
fiziks
Institute for NET/JRF, GATE, IIT‐JAM, JEST, TIFR and GRE in PHYSICAL SCIENCES 
 
Q14. In a certain inertial frame two light pulses are emitted, a distance 5 km apart and
separated by 5μ s . An observer who is traveling, parallel to the line joining the points
where the pulses are emitted, at a velocity V with respect to this frame notes that the
pulses are simultaneous. Therefore V is
(a) 0.7 c (b) 0.8 c (c) 0.3 c (d) 0.9 c
Ans.: (c)
Solution: x2′ − x1′ = 5 × 103 m, t2′ − t1′ = 5 × 10−6 sec

⎛ −v ⎞ ⎛ −v ⎞ ⎡ v ⎤
t2′ + ⎜ 2 x2′ ⎟ t1′ + ⎜ 2 x1′ ⎟ ⎢( t2′ − t1′ ) − 2 ( x2′ − x1′ ) ⎥
t2 − t1 = ⎝c ⎠− ⎝c ⎠=⎣ c ⎦
2 2 2
v v v
1− 2 1− 2 1− 2
c c c
v
∵t2 = t1 ⇒ 5 × 10−6 − 5 × 103 = 0 ⇒ v = 0.3c
c2
Q15. A double pendulum consists of two equal masses m suspended by two strings of length
l . What is the Lagrangian of this system for oscillations in a plane? Assume the angles
θ1 , θ 2 made by the two strings are small (you can use cos θ = 1 − θ 2 / 2 ).

Note: ω 0 = g / l .

⎛ 1 1 ⎞
(a) L ≈ ml 2 ⎜θ12 + θ 22 − ω 02θ12 − ω 02θ 22 ⎟
⎝ 2 2 ⎠
⎛ 1 1 ⎞
(b) L ≈ ml 2 ⎜θ12 + θ 22 + θ1θ 2 − ω 02θ12 − ω 02θ 22 ⎟
⎝ 2 2 ⎠
⎛ 1 1 ⎞
(c) L ≈ ml 2 ⎜θ12 + θ 22 − θ1θ 2 − ω 02θ12 − ω 02θ 22 ⎟
⎝ 2 2 ⎠
⎛1 1 ⎞
(d) L ≈ ml 2 ⎜ θ12 + θ 22 + θ1θ 2 − ω 02θ12 − ω 02θ 22 ⎟
⎝2 2 ⎠

                                                                                
Head office  Branch office 
 
fiziks, H.No. 40 D, G.F, Jia Sarai,  Anand Institute of Mathematics, 
 
Near IIT, Hauz Khas, New Delhi‐16  28‐B/6, Jia Sarai, Near IIT 
 
Phone: 011‐26865455/+91‐9871145498 Hauz Khas, New Delhi‐16 
                                                   
                                             Website: www.physicsbyfiziks.com                                                                                          
                                                           Email: fiziks.physics@gmail.com                                                                   9 
fiziks
Institute for NET/JRF, GATE, IIT‐JAM, JEST, TIFR and GRE in PHYSICAL SCIENCES 
 
Ans.: (b)
Solution: x1 = l sin θ1 , y1 = l cos θ1
θ1 l
x2 = x1 + l sin θ 2 y2 = y1 + l cos θ 2

x2 = l sin θ1 + l sin θ 2 , y2 = l cos θ1 + l cos θ 2

x2 = l cos θ1θ1 + l cos θ 2θ 2 , y2 = −l sin θ1θ1 − l sin θ 2θ 2 θ2 l

x 22 + y 22 = l 2 cos 2 θ1θ12 + l 2 cos 2 θ 2θ 22 + 2l 2 cos θ1θ1 cos θ 2θ 2 + l 2 sin θ12θ12 m


+ l 2 sin θ 22θ 22 + 2l 2 sin θ1 sin θ 2θ1θ 2

⇒ x22 + y22 = l 2θ12 + l 2θ 22 + 2l 2 cos (θ1 − θ 2 )θ1θ 2 also x12 + y12 = l 2θ12

L = T −V =
1
2
( )
m x12 + y12 + x 22 + y 22 − mgy1 − mgy 2

1
⇒L=
2
( )
m l 2θ12 + l 2θ12 + l 2θ 22 + 2l 2 cos (θ1 − θ 2 )θ1θ 2 + 2mgl cos θ1 + mgl cos θ 2

⎡ 1 2g ⎡ θ12 ⎤ 1 g ⎡ θ 22 ⎤ ⎤
⇒ L = ml 2 ⎢θ12 + θ 22 + θ1θ 2 + ⎢1 − ⎥ + ⎢1 − ⎥ ⎥ ∵cos (θ1 − θ 2 ) ≈ 1
⎣ 2 2l ⎣ 2⎦ 2 l ⎣ 2 ⎦⎦

⎡ 2 1 2 g g θ12 g g θ 22 ⎤
⇒ L = ml ⎢θ1 + θ 2 + θ1θ 2 + −
2
+ − ⎥
⎣ 2 l l 2 2l 2l 2 ⎦

comparing given options, option (b) is correct i.e.


⎛ 1 ω 2θ 2 1 ⎞
L = ml 2 ⎜ θ12 + θ 22 + θ1θ 2 − 0 1 − ω0θ 22 ⎟
⎝ 2 2 4 ⎠
Q16. A monochromatic wave propagates in a direction making an angle 60 o with the x -axis
4c
in the reference frame of source. The source moves at speed v = towards the
5
observer. The direction of the (cosine of angle) wave as seen by the observer is
13 3 13 1
(a) cos θ ′ = (b) cos θ ′ = (c) cos θ ′ = (d) cos θ ′ =
14 14 6 2
Ans.: (a)

4c c 3
Solution: v = , u x′ = c cos 60o = , u ′y = c sin 60o = c
5 2 2
                                                                                
Head office  Branch office 
 
fiziks, H.No. 40 D, G.F, Jia Sarai,  Anand Institute of Mathematics, 
 
Near IIT, Hauz Khas, New Delhi‐16  28‐B/6, Jia Sarai, Near IIT 
 
Phone: 011‐26865455/+91‐9871145498 Hauz Khas, New Delhi‐16 
                                                   
                                             Website: www.physicsbyfiziks.com                                                                                          
                                                           Email: fiziks.physics@gmail.com                                                                   10 
fiziks
Institute for NET/JRF, GATE, IIT‐JAM, JEST, TIFR and GRE in PHYSICAL SCIENCES 
 
c 4
+ c
13c 13
Now u x = 2 5 = ⇒ cos θ =
c 4c 14 14
1+ ⋅ 2
2 5c
Q17. The acceleration experienced by the bob of a simple pendulum is
(a) maximum at the extreme positions
(b) maximum at the lowest (central) positions
(c) maximum at a point between the above two positions
(d) same at all positions
Ans.: (a)
Solution: T sin θ = ma , T cos θ = mg l
θ T T cosθ
a = g tan θ at θ = 90 o

T sin θ
a is maximum at extreme position.
mg
Q18. Consider a Hamiltonian system with a potential energy function given by V ( x ) = x 2 − x 4 .
Which of the following is correct?
(a) The system has one stable point (b) The system has two stable points
(c) The system has three stable points (d) The system has four stable points
Ans.: (a)
Solution: V ( x ) = x 2 − x 4
∂V
∂x
= 2x − 4x3 = 0 ⇒ 2x 1 − 2x 2 = 0 [ ]
1
x=± ,0
2
∂ 2V ∂ 2V 1
2
= 2 − 12 x 2
⇒ 2
= 2 − 12 × = −4 < 0
dx dx x =± 1 2
2

∂V ∂ 2V
For stable point = 0 and >0
∂x ∂x

∂ 2V
=2>0
∂x 2 x =0

                                                                                
Head office  Branch office 
 
fiziks, H.No. 40 D, G.F, Jia Sarai,  Anand Institute of Mathematics, 
 
Near IIT, Hauz Khas, New Delhi‐16  28‐B/6, Jia Sarai, Near IIT 
 
Phone: 011‐26865455/+91‐9871145498 Hauz Khas, New Delhi‐16 
                                                   
                                             Website: www.physicsbyfiziks.com                                                                                          
                                                           Email: fiziks.physics@gmail.com                                                                   11 
fiziks
Institute for NET/JRF, GATE, IIT‐JAM, JEST, TIFR and GRE in PHYSICAL SCIENCES 
 
Q19. Two point objects A and B have masses 1000 Kg and 3000 Kg respectively. They are
initially at rest with a separation equal to 1 m. Their mutual gravitational attraction then
draws them together. How far from A’s original position will they collide?
(a) 1/3 m (b) 1/2 m (c) 2/3 m (d) 3/4 m
Ans.: (d)
Solution: Since gravitational force is conservative, therefore they collide at their centre of mass
m1 x = (1 − x )m2 m1 x 1− x m2
A B
3 1m
x = 3(1 − x ) ⇒ x =
4

JEST-2013
Q20. In an observer’s rest frame, a particle is moving towards the observer with an energy E
and momentum P . If c denotes the velocity of light in vacuum, the energy of the
particle in another frame moving in the same direction as particle with a constant velocity
v is

(a)
(E + vp ) (b)
(E − vp ) (c)
(E + vp ) (d)
(E − vp )
1 − (v / c )
2
1 − (v / c )
2
[1 − (v / c )2 ]2 [1 − (v / c )2 ]2
Ans.: (a)
vx x v v
2
t+
′ + 2x x+ x
x
c ⇒ = c c ⇒ x′ = c ∵ x = ct , x′ = ct ′
Solution: t ′ =
2 2
v c v v2
1− 2 1− 2 1− 2
c c c
E
v E+
Now x′ = E ′, x = E ⇒ E ′ = c ⇒ E = mc 2 , E = Pc ⇒ P = E ⇒ E ′ = E + Pv
v2 c v2
1− 2 1− 2
c c
Q21. The free fall time of a test mass on an object of mass M from a height 2R to R is

R3 R3 R3 2R 3
(a) (π / 2 + 1) (b) (c) (π / 2) (d) π
GM GM GM GM
Ans.: (a)
                                                                                
Head office  Branch office 
 
fiziks, H.No. 40 D, G.F, Jia Sarai,  Anand Institute of Mathematics, 
 
Near IIT, Hauz Khas, New Delhi‐16  28‐B/6, Jia Sarai, Near IIT 
 
Phone: 011‐26865455/+91‐9871145498 Hauz Khas, New Delhi‐16 
                                                   
                                             Website: www.physicsbyfiziks.com                                                                                          
                                                           Email: fiziks.physics@gmail.com                                                                   12 
fiziks
Institute for NET/JRF, GATE, IIT‐JAM, JEST, TIFR and GRE in PHYSICAL SCIENCES 
 
2 2
md r GMm d r GM d 2r A
Solution: Equation of motion 2
= − 2
⇒ 2
= − 2
⇒ 2
=− 2 ∵ GM = A
dt r dt r dt r
dv A dr d ⎛ v2 ⎞ d ⎛ A⎞ v2 A
v =− 2 ⇒ ⎜⎜ ⎟⎟ = ⎜ ⎟ ⇒ = +C
dt r dt dt ⎝ 2 ⎠ dt ⎝ r ⎠ 2 r

when r = 2 R, v = 0

0 A A v2 A A 2A 2A dr 2A 2R − r
= +C ⇒ C = − ⇒ = − ⇒v= − ⇒ =
2 2R 2R 2 r 2R r 2R dt 2R r

R r A t
∫2R
2R − r
dr = −
R ∫0
dt

put r = u 2 , dr = 2udu when r = 2 R, r = R, u = 2 R , u = R

R u A t A R u2
∫ 2R
2R − u 2
× 2udu = − ∫
R 0
dt ⇒ −
R
t = 2∫
2R
2R − u 2
du

R
A ⎡ u 2 R −1 u ⎤
⇒− t = 2 ⎢− 2R − u 2 + sin ⎥
R ⎣ 2 2 2R ⎦ 2R

A ⎡− R 2 R −1 R 2R 2R ⎤
⇒− t = 2⎢ 2R − R + sin + 2 R − 2 R − R sin −1 ⎥
R ⎣ 2 2 2R 2 2R ⎦

⎡ − R Rπ Rπ ⎤ R R ⎛π ⎛π
3
A ⎞ ⎞ R
⇒− t = 2⎢ + − ⇒t = ⎜ + 1 ⎟ ⇒ t = ⎜ + 1⎟ ∵ A = GM
R ⎣ 2 4 2 ⎥⎦ A ⎝2 ⎠ ⎝ 2 ⎠ GM
Q22. Under a Galilean transformation, the coordinates and momenta of any particle or system

transform as: t ' = t , r ' = r + v t and p ' = p + mv where v is the velocity of the boosted
frame with respect to the original frame. A unitary operator carrying out these
transformations for a system having total mass M , total momentum P and centre of
mass coordinate X is
2
t / (2 )
(a) e i M v . X / e i t v .P / (b) e i M v . X / e − i t v .P / e − i M v
2
t / (2 ) 2
t / (2 )
(c) e i M v . X / e i t v .P / e i M v (d) e i t v . P / e − i M v
Ans.: (b)

                                                                                
Head office  Branch office 
 
fiziks, H.No. 40 D, G.F, Jia Sarai,  Anand Institute of Mathematics, 
 
Near IIT, Hauz Khas, New Delhi‐16  28‐B/6, Jia Sarai, Near IIT 
 
Phone: 011‐26865455/+91‐9871145498 Hauz Khas, New Delhi‐16 
                                                   
                                             Website: www.physicsbyfiziks.com                                                                                          
                                                           Email: fiziks.physics@gmail.com                                                                   13 
fiziks
Institute for NET/JRF, GATE, IIT‐JAM, JEST, TIFR and GRE in PHYSICAL SCIENCES 
 
Q23. A spherical planet of radius R has a uniform density ρ and does not rotate. If the planet
is made up of some liquid, the pressure at point r from the center is
4πρ 2 G 2 4πρG 2
(a)
3
R − r2 ( ) (b)
3
R − r2 ( )
2πρ 2 G 2 ρG
(c)
3
R − r2 ( ) (d)
2
(R 2
− r2 )
Ans.: (c)
r
ρ ⋅ 4πr 2 drGM
dm ⋅ g dm ⋅ g R3
Solution: Pressure dp = ⇒ dp = ⇒ dp =
A 4πr 2 4πr 2
dr

r dm (mass of elementary part )


R

4π 3 r
ρ ⋅ 4πr 2 drG ⋅ ρ ⋅ R 3
⇒ dp = 3 R ⇒ dp = 4π ρ 2 Grdr
4πr 2
3
R
R 4π 2 4π 2 ⎛ r 2 ⎞ 4π 2 ⎛ R 2 r 2 ⎞
∫ dp = ∫ r 3
ρ Grdr ⇒ p =
3
ρ G⎜⎜
⎝ 2
⎟⎟ ⇒ p =
⎠r 3
ρ G⎜⎜
⎝ 2
− ⎟⎟
2⎠
4π ρ 2 G 2 2π 2
⇒ p=
3 2
(R − r2 )⇒ p =
3
ρ G (R 2 − r 2 )

Q24. A particle of mass m is thrown upward with velocity v and there is retarding air
resistance proportional to the square of the velocity with proportionality constant k . If
the particle attains a maximum height after time t , and g is the gravitational
acceleration, what is the velocity?

⎛ g ⎞ ⎛ g ⎞
k
tan⎜⎜ t ⎟⎟ gk tan⎜⎜ t ⎟⎟
(a) (b)
g ⎝ k ⎠ ⎝ k ⎠

(c)
g
k
tan ( gk t ) (d) gk tan ( gk t )
Ans.: (c)
                                                                                
Head office  Branch office 
 
fiziks, H.No. 40 D, G.F, Jia Sarai,  Anand Institute of Mathematics, 
 
Near IIT, Hauz Khas, New Delhi‐16  28‐B/6, Jia Sarai, Near IIT 
 
Phone: 011‐26865455/+91‐9871145498 Hauz Khas, New Delhi‐16 
                                                   
                                             Website: www.physicsbyfiziks.com                                                                                          
                                                           Email: fiziks.physics@gmail.com                                                                   14 
fiziks
Institute for NET/JRF, GATE, IIT‐JAM, JEST, TIFR and GRE in PHYSICAL SCIENCES 
 
mdv dv k dv
Solution: Equation of motion = mg + kv 2 ⇒ = g + v2 ⇒ = dt
dt dt m k
g + v2
m
dv dv m 1 v
⇒∫ = ∫ dt ⇒ ∫ = ∫ dt ⇒ × tan −1 =t
k k ⎛ gm ⎞ k gm gm
g + v2 ⎜ + v2 ⎟
m m⎝ k ⎠ k k
Q25. Consider a uniform distribution of particles with volume density n in a box. The particles
have an isotropic velocity distribution with constant magnitude v . The rate at which the
particles will be emitted from a hole of area A on one side of this box is
A A
(a) nvA (b) nv (c) nv (d) none of the above
2 4
Ans.: (c)
Q26. If, in a Kepler potential, the pericentre distance of particle in a parabolic orbit is rp while

the radius of the circular orbit with the same angular momentum is rc , then

(a) rc = 2rp (b) rc = rp (c) 2rc = rp (d) rc = 2rp

Ans.: (a)
l
Solution: Ionic equation = 1 + e cos θ for parabola e = 1 for circle, e = 0 , θ = 0
r
l l
= 1+ e , = 1, l = 2rp , l = rC ⇒ 2rp = rC
rp rC

Q27. A K meson (with a rest mass of 494 MeV) at rest decays into a muon (with a rest mass of
106 MeV) and a neutrino. The energy of the neutrino, which can be massless, is
approximately
(a) 120 MeV (b) 236 MeV (c) 300 MeV (d) 388 MeV
Ans.: (b)
⎛ 494 494 106 106 ⎞ 2
( m − mμ ) c ⎜ 2 × 2 − 2 × 2 ⎟c
2 2 2

⇒⎝
c c c c ⎠
Solution: k → μ + ν , Eν =
k

2mk 494
2× 2
c
11236
⇒ 244036 − = 235.6275 ≈ 236 MeV
988
                                                                                
Head office  Branch office 
 
fiziks, H.No. 40 D, G.F, Jia Sarai,  Anand Institute of Mathematics, 
 
Near IIT, Hauz Khas, New Delhi‐16  28‐B/6, Jia Sarai, Near IIT 
 
Phone: 011‐26865455/+91‐9871145498 Hauz Khas, New Delhi‐16 
                                                   
                                             Website: www.physicsbyfiziks.com                                                                                          
                                                           Email: fiziks.physics@gmail.com                                                                   15 
fiziks
Institute for NET/JRF, GATE, IIT‐JAM, JEST, TIFR and GRE in PHYSICAL SCIENCES 
 
Q28. A light beam is propagating through a block of glass with index of refraction n . If the
glass is moving at constant velocity v in the same direction as the beam, the velocity of
the light in the glass block as measured by an observer in the laboratory is approximately
c ⎛ 1 ⎞ c ⎛ 1 ⎞
(a) u = + v ⎜1 − 2 ⎟ (b) u = − v⎜ 1 − 2 ⎟
n ⎝ n ⎠ n ⎝ n ⎠
c ⎛ 1 ⎞ c
(c) u = + v⎜1 + 2 ⎟ (d) u =
n ⎝ n ⎠ n
Ans.: (a)
c
v+ −1
n = ⎛v + c ⎞⎛ v ⎞ ⎛ c ⎞⎛ v v2 ⎞
Solution: now u = ⎜ ⎟⎜1 + ⎟ = ⎜ v + ⎟ ⎜ 1 − + 2 2 ⎟
v⋅c ⎝ n ⎠⎝ cn ⎠ ⎝ n ⎠ ⎝ cn c n ⎠
1+ 2
c ⋅n
v2 v3 c v cv 2 c ⎛ 1 ⎞
⇒v− + 2 2 + − 2 + 3 ⇒ u = + v ⎜1 − 2 ⎟
cn c n n cn cn n ⎝ n ⎠
Q29. The period of a simple pendulum inside a stationary lift is T . If the lift accelerates
g
downwards with an acceleration , the period of the pendulum will be
4
(a) T (b) T / 4 (c) 2T / 3 (d) 2T / 5
Ans.: (c)

l
Solution: T = 2π ⇒ lift accelerates down wards then
g

l l 4l l
T = 2π ⇒ T = 2π = 2π ⇒ 2π × 2
g − g′ g 3g 3g
g−
4
2T
T′ =
3

                                                                                
Head office  Branch office 
 
fiziks, H.No. 40 D, G.F, Jia Sarai,  Anand Institute of Mathematics, 
 
Near IIT, Hauz Khas, New Delhi‐16  28‐B/6, Jia Sarai, Near IIT 
 
Phone: 011‐26865455/+91‐9871145498 Hauz Khas, New Delhi‐16 
                                                   
                                             Website: www.physicsbyfiziks.com                                                                                          
                                                           Email: fiziks.physics@gmail.com                                                                   16 
fiziks
Institute for NET/JRF, GATE, IIT‐JAM, JEST, TIFR and GRE in PHYSICAL SCIENCES 
 
Q30. The velocity of a particle at which the kinetic energy is equal to its rest energy is (in
terms of c , the speed of light in vacuum)
(a) 3c / 2 (b) 3c / 4 (c) 3 / 5c (d) c / 2
Ans.: (a)
Solution: K .E = mc 2 − m0 c 2 , rest mass energy = m0 c 2

K .E. = rest mass energy


mc 2 − m0 c 2 = m0 c 2

mc 2 = 2m0 c 2

m0 1 ⎛ v2 ⎞ v2 3
c 2 = 2m0 c 2 ⇒ = 2 ⇒ 4⎜⎜1 − 2 ⎟⎟ = 1 ⇒ 4 2 = 3 ⇒ v = c
v2 v2 ⎝ c ⎠ c 2
1− 1−
c2 c2

Q31. If the Poisson bracket { x, p} = −1 , then the Poisson bracket x 2 + p, p is ? { }


(a) −2x (b) 2x (c) 1 (d) −1
Ans.: (a)
Solution: {x 2 + p, p} = {x 2 , p}+ {p, p} ⇒ x{x, p} + {x, p}x + 0 ⇒ x ( −1) + ( −1) x ⇒ −2 x

Q32. The coordinate transformation


x ′ = 0.8 x + 0.6 y, y ′ = 0.6 x − 0.8 y
represents
(a) a translation (b) a proper rotation
(c) a reflection (d) none of the above
Ans.: (b)

                                                                                
Head office  Branch office 
 
fiziks, H.No. 40 D, G.F, Jia Sarai,  Anand Institute of Mathematics, 
 
Near IIT, Hauz Khas, New Delhi‐16  28‐B/6, Jia Sarai, Near IIT 
 
Phone: 011‐26865455/+91‐9871145498 Hauz Khas, New Delhi‐16 
                                                   
                                             Website: www.physicsbyfiziks.com                                                                                          
                                                           Email: fiziks.physics@gmail.com                                                                   17 
fiziks
Institute for NET/JRF, GATE, IIT‐JAM, JEST, TIFR and GRE in PHYSICAL SCIENCES 
 
Q33. A small mass M hangs from a thin string and can swing like a pendulum. It is attached
above the window of a car. When the car is at rest, the string hangs vertically. The angle
made by the string with the vertical when the car has a constant acceleration a = 1.2 m / s 2
is approximately
(a) 10 (b) 70 (c) 150 (d) 900
Ans.: (b)
a a ⎛ 1.2 ⎞
Solution: T sin θ = ma , T cos θ = mg , tan θ = ⇒ θ = tan −1 = tan −1 ⎜ ⎟ = 6.98 ≈ 7
0 0

g g ⎝ 9.8 ⎠

JEST-2012
Q34. For small angular displacement (i.e., sinθ ≈ θ), a simple pendulum oscillates
harmonically. For larger displacements, the motion
(a) becomes a periodic
(b) remains periodic with the same period
(c) remains periodic with a higher period
(d) remains periodic with a lower period
Ans. : (c)
Q35. A planet orbits a massive star in a highly elliptical orbit, i.e., the total orbital energy E is
close to zero. The initial distance of closest approach is R0. Energy is dissipated through
tidal motions until the orbit is circularized with a final radius of Rf. Assume that orbital
angular momentum is conserved during the circularization process. then
(a) Rf = R0/2 (b) Rf = R0 (c) R f = 2 R0 (d) Rf = 2R0

Ans. : (d)
1 2 J2 GMm
Solution: For elliptically motion E = mr + 2

2 2mr r
E = 0 and closest approach is R0 at R0 ⇒ r = 0

J2 GMm
0 =0+ 2

2mR0 R0

                                                                                
Head office  Branch office 
 
fiziks, H.No. 40 D, G.F, Jia Sarai,  Anand Institute of Mathematics, 
 
Near IIT, Hauz Khas, New Delhi‐16  28‐B/6, Jia Sarai, Near IIT 
 
Phone: 011‐26865455/+91‐9871145498 Hauz Khas, New Delhi‐16 
                                                   
                                             Website: www.physicsbyfiziks.com                                                                                          
                                                           Email: fiziks.physics@gmail.com                                                                   18 
fiziks
Institute for NET/JRF, GATE, IIT‐JAM, JEST, TIFR and GRE in PHYSICAL SCIENCES 
 
2
J GMm
2
=
2MR0 R0

J 2 = 2GMm 2 R0
from condition of circular orbit

J2 ∂V
= f (r ) = −
mR f3
∂r

J2 GMm
3
=
mR f R 2f

2GMm 2 R0 GMm
=
mR 3f R 2f

R f = 2 R0

Q36. A binary system consists of two stars of equal mass m orbiting each other in a circular
orbit under the influence of gravitational forces. The period of the orbit is т. At t = 0, the
motion is stopped and the stars are allowed to fall towards each other. After what time t,
expressed in terms of т, do they collide?
x 2 dx x α ⎛ x ⎞
∫ α − x2
=
2
α − x 2 + sin −1 ⎜⎜
2 ⎝ α⎠
⎟⎟

τ τ τ
(a) 2τ (b) (c) (d)
2 2 2 4 2
Ans. : (d)
d 2x GMm
Solution: M 2
=− 2
dt x
d 2x GM
2
=− 2
dt x
d 2x A
2
=− 2
dt x
dv − A dx
v = 2
dt x dt

                                                                                
Head office  Branch office 
 
fiziks, H.No. 40 D, G.F, Jia Sarai,  Anand Institute of Mathematics, 
 
Near IIT, Hauz Khas, New Delhi‐16  28‐B/6, Jia Sarai, Near IIT 
 
Phone: 011‐26865455/+91‐9871145498 Hauz Khas, New Delhi‐16 
                                                   
                                             Website: www.physicsbyfiziks.com                                                                                          
                                                           Email: fiziks.physics@gmail.com                                                                   19 
fiziks
Institute for NET/JRF, GATE, IIT‐JAM, JEST, TIFR and GRE in PHYSICAL SCIENCES 
 
d ⎛v ⎞ d ⎛ A⎞
2
⎜ ⎟= ⎜ ⎟
dt ⎜⎝ 2 ⎟⎠ dt ⎝ x ⎠

v2 A
= +C
2 x
where x = R v=0
v2 A A
= −
2 x R
1 1
v = 2A −
x R

dx 2A R − x
=
dt R x
0 t
x 2A
∫ R−x
dx = ∫
0
R
dt
R

put x = u 2
dx = 2udu , x = 0, u = 0

x = R, u = R
0
2u 2
t
2A
∫ R − u2
du = ∫
0
R
dt
R

0
⎡u R u ⎤ 2A
− 2⎢ R − u 2 + sin −1 ⎥ = t
⎣2 2 R⎦ R
R

⎡ R R −1 R⎤ 2A
+ 2⎢ R−R + sin ⎥= t
⎣ 2 2 R⎦ R

R −1 2A
2× sin 1 = t
2 R

2A R π
t = 2× ×
R 2 2

                                                                                
Head office  Branch office 
 
fiziks, H.No. 40 D, G.F, Jia Sarai,  Anand Institute of Mathematics, 
 
Near IIT, Hauz Khas, New Delhi‐16  28‐B/6, Jia Sarai, Near IIT 
 
Phone: 011‐26865455/+91‐9871145498 Hauz Khas, New Delhi‐16 
                                                   
                                             Website: www.physicsbyfiziks.com                                                                                          
                                                           Email: fiziks.physics@gmail.com                                                                   20 
fiziks
Institute for NET/JRF, GATE, IIT‐JAM, JEST, TIFR and GRE in PHYSICAL SCIENCES 
 
Rπ R
t= ×
2 2A

1 R 3π 2
t= 2 (1)
2 GM
and
mv 2 GMm
=
R R2
GM
v2 =
R
2πR
v=
τ
4π 2 R 2 GM
=
τ 2
R
4π 2 R 3
=τ 2
GM

R 3π 2 R 3π 2 τ
τ =2 ⇒ =
GM GM 2
1 τ τ
t= =
2 22 4 2
Q37. In a certain intertial frame two light pulses are emitted at point 5 km apart and separated
in time by 5 μs. An observer moving at a speed V along the line joining these points
notes that the pulses are simultaneous. Therefore V is
(a) 0.7c (b) 0.8c (c) 0.3c (d) 0.9c
Ans. : (c)
Solution: Δt = 0 , t 2′ − t1′ = 5μs , x 2′ − x1′ = 5km v = −V

⎛ −V ⎞ ⎛ −V ⎞
t 2′ + ⎜ 2 ⎟ x ′2 t1′ + ⎜ 2 ⎟ x1′
t 2 − t1 = ⎝C ⎠ − ⎝C ⎠
V2 V2
1− 2 1− 2
C C

                                                                                
Head office  Branch office 
 
fiziks, H.No. 40 D, G.F, Jia Sarai,  Anand Institute of Mathematics, 
 
Near IIT, Hauz Khas, New Delhi‐16  28‐B/6, Jia Sarai, Near IIT 
 
Phone: 011‐26865455/+91‐9871145498 Hauz Khas, New Delhi‐16 
                                                   
                                             Website: www.physicsbyfiziks.com                                                                                          
                                                           Email: fiziks.physics@gmail.com                                                                   21 
fiziks
Institute for NET/JRF, GATE, IIT‐JAM, JEST, TIFR and GRE in PHYSICAL SCIENCES 
 
⎡ ′ ′ ⎤
⎢⎣(t 2 − t1 ) − C 2 ( x 2′ − x1′ )⎥⎦
V
V
⇒ = 0 ⇒ 5 × 10 −6 − 2 × 5 × 10 3 = 0
V2 C
1− 2
C

V 5 × 10 −6
⇒ = = 10 −9 ⇒ V = 3 × 10 8 × C × 10 −9 = 0.3C
C 2
5 × 10 3

Q38. A jet of gas consists of molecules of mass m, speed v and number density n all moving
co-linearly. This jet hits a wall at an angle θ to the normal. The pressure exerted on the
wall by the jet assuming elastic collision will be
(a) p = 2mnv 2 cos 2 θ (b) p = 2mnv 2 cos θ

(c) p = (3 / 2)mnv cos 2 θ (d) p = mnv2


Ans.: (a)
Solution: change in momentum along y − direction will be cancelled out
∵ change in momentum along x − direction
Δp = 2mv cos θ
Δp
Force Δt Δp Δp Δpv cos θ
Pressure = = = = =
Area A A.Δ t L A. L
A⋅
v cos θ
2mv cos θ ⋅ v cos θN ⎛ N⎞
Pressure p ′ = , ∵ ⎜ n = ⎟ , (V = Area × L = A × L ) ,
v ⎝ V ⎠

p ′ = 2mnv 2 cos 2 θ
Q39. If the coordinate q and the momentum p from a canonical pair (q, p), which one of the
sets given below also forms a canonical?
(a) (q - p) (b) (q2 , p2) (c) (p, - q) (d) (q2, - p2)
Ans.: (c)
Solution: for canonical pair ( p, − q )
∂p ∂ (− q ) ∂ ( p ) ∂ (− q )
= ⋅ − ⋅ = 0 − (− 1) = 1
∂q ∂p ∂p ∂q
                                                                                
Head office  Branch office 
 
fiziks, H.No. 40 D, G.F, Jia Sarai,  Anand Institute of Mathematics, 
 
Near IIT, Hauz Khas, New Delhi‐16  28‐B/6, Jia Sarai, Near IIT 
 
Phone: 011‐26865455/+91‐9871145498 Hauz Khas, New Delhi‐16 
                                                   
                                             Website: www.physicsbyfiziks.com                                                                                          
                                                           Email: fiziks.physics@gmail.com                                                                   22 
fiziks
Institute for NET/JRF, GATE, IIT‐JAM, JEST, TIFR and GRE in PHYSICAL SCIENCES 
 
Q40. A girl measures the period of a simple pendulum inside a stationary lift and finds it to be
T seconds. If the lift accelerates upward with an acceleration g / 4, then the pendulum
will be
(a) T (b) T / 4 (c) 2T 5 (d) 2T 5
Ans.: (c)

l
Solution: T = 2π
g
Since lift accelerated upward then

l
T ′ = 2π
g + g′

l
T ′ = 2π
g
g+
4

l
T ′ = 2π ×4
5g

l 2 2T
T ′ = 2π × ⇒ T′ =
g 5 5

                                                                                
Head office  Branch office 
 
fiziks, H.No. 40 D, G.F, Jia Sarai,  Anand Institute of Mathematics, 
 
Near IIT, Hauz Khas, New Delhi‐16  28‐B/6, Jia Sarai, Near IIT 
 
Phone: 011‐26865455/+91‐9871145498 Hauz Khas, New Delhi‐16 
                                                   
                                             Website: www.physicsbyfiziks.com                                                                                          
                                                           Email: fiziks.physics@gmail.com                                                                   23 
fiziks
Institute for NET/JRF, GATE, IIT‐JAM, JEST, TIFR and GRE in PHYSICAL SCIENCES 
 
Electromagnetic Theory
JEST-2016
Q1. The maximum relativistic kinetic energy of β particles from a radioactive nucleus is
equal to the rest mass energy of the particle. A magnetic field is applied perpendicular to
the beam of β particles, which bends it to a circle of radius R . The field is given by:

3m0 c 2m0 c 3m0 c 3m0 c


(a) (b) (c) (d)
eR eR eR 2eR
Ans: (c)
Solution: KEmax = mc 2 − m0 c 2 = m0 c 2 ⇒ m = 2m0

m0 m0 3
∵m = ⇒ 2m0 = ⇒v= c
v 2
v 2 2
1− 1−
c2 c2

mv mv 2m0 3 3m0 c
∵R = ⇒B= = c=
eB eR eR 2 eR
Q2. The strength of magnetic field at the center of a regular hexagon with sides of length a
carrying a steady current I is:
μ0 I 6 μ0 I 3μ0 I 3μ 0 I
(a) (b) (c) (d)
3π a πa πa πa
Ans14: (d)
C
3 I a
d = a cos 30 = a 0

2 a d C
μI 600
∵ B = 0 ( sin θ 2 − sin θ1 )
4π d I
μ0 I μ0 I μ0 I
⇒ B1 = 2sin 300 = 2sin 300 =
4π d 3 2 3π a
4π a
2
μ0 I 3μ0 I 3μ0 I
⇒ B = 6 B1 = 6 × = =
2 3π a 3π a πa

                                                                                
Head office  Branch office 
 
fiziks, H.No. 40 D, G.F, Jia Sarai,  Anand Institute of Mathematics, 
 
Near IIT, Hauz Khas, New Delhi‐16  28‐B/6, Jia Sarai, Near IIT 
 
Phone: 011‐26865455/+91‐9871145498 Hauz Khas, New Delhi‐16 
                                                   
                                             Website: www.physicsbyfiziks.com                                                                                          
                                                           Email: fiziks.physics@gmail.com                                                                   1 
fiziks
Institute for NET/JRF, GATE, IIT‐JAM, JEST, TIFR and GRE in PHYSICAL SCIENCES 
 
Q3. A spherical shell of radius R carries a constant surface charge density σ and is rotating
about one of its diameters with an angular velocity ω . The magnitude of the magnetic
moment of the shell is:
4πσω R 4 4πσω R 4 4πσω R 4
(a) 4πσω R 4 (b) (c) (d)
3 15 9
Ans : (b)
Solution: The total charge on the shaded ring is z
dq = σ (2π R sin θ ) Rdθ ω R sin θ
2π Rdθ
Time for one revolution is dt =
ω
θ
dq
⇒Current in the ring I = = σω R 2 sin θ dθ dθ
dt
R
Area of the ring = π(R sin θ)2 , so the magnetic moment of the
ring is

dm = (σω R 2 sin θ dθ ) × π R 2 sin 2 θ

π 4 4π
m = σω R 4 ∫0 sin 3 θ dθ = π × σω R 4 ⇒ m = σω R 4 zˆ
3 3
⎛ π⎞
Q4. The electric field E = E0 sin (ωt − kz ) xˆ + 2 E0 sin ⎜ ωt − kz + ⎟ yˆ represents:
⎝ 2⎠
(a) a linwearly polarized wave
(b) a right-hand circularly polarized wave
(c) a left-hand circularly polarized wave
(d) an elliptically polarized wave
Ans: (d)

                                                                                
Head office  Branch office 
 
fiziks, H.No. 40 D, G.F, Jia Sarai,  Anand Institute of Mathematics, 
 
Near IIT, Hauz Khas, New Delhi‐16  28‐B/6, Jia Sarai, Near IIT 
 
Phone: 011‐26865455/+91‐9871145498 Hauz Khas, New Delhi‐16 
                                                   
                                             Website: www.physicsbyfiziks.com                                                                                          
                                                           Email: fiziks.physics@gmail.com                                                                   2 
fiziks
Institute for NET/JRF, GATE, IIT‐JAM, JEST, TIFR and GRE in PHYSICAL SCIENCES 
 
Q5. Suppose yz plane forms the boundary between two linear dielectric media I and II
with dielectric constant ∈I = 3 and ∈II = 4 , respectively. If the electric field in region I at

the interface is given by EI = 4 xˆ + 3 yˆ + 5 zˆ , then the electric field EII at the interface in
region II is:
(a) 4 xˆ + 3 yˆ + 5 zˆ (b) 4 xˆ + 0.75 yˆ − 1.25 zˆ
(c) −3xˆ + 3 yˆ + 5 zˆ (d) 3xˆ + 3 yˆ + 5 zˆ
Ans: (d)
E ⊥ II ε I ε 3
Solution: ∵ E I = E ⇒E = 3 yˆ + 5 zˆ and = ⇒ E ⊥ II = I E ⊥ I = 4 xˆ = 3 xˆ
E I ε II ε II
II II ⊥
4

⇒ EII = 3 xˆ + 3 yˆ + 5 zˆ

Q6. How much force does light from a 1.8 W laser exert when it is totally absorbed by an
object?
(a) 6.0 ×10−9 N (b) 0.6 ×10−9 N (c) 0.6 × 10−8 N (d) 4.8 ×10−9 N
Ans: (a)
F I P P 1.8
Solution: Radiation Pressure = = ⇒F= ⇒F= = 6.0 ×10−9 N
A c Ac c 3 ×10 8

Q7. Self inductance per unit length of a long solenoid of radius R with n turns per unit
length is:
(a) μ0π R 2 n 2 (b) 2μ0π R 2 n

(c) 2μ0π R 2 n 2 (d) μ0π R 2 n


Ans: (a)

                                                                                
Head office  Branch office 
 
fiziks, H.No. 40 D, G.F, Jia Sarai,  Anand Institute of Mathematics, 
 
Near IIT, Hauz Khas, New Delhi‐16  28‐B/6, Jia Sarai, Near IIT 
 
Phone: 011‐26865455/+91‐9871145498 Hauz Khas, New Delhi‐16 
                                                   
                                             Website: www.physicsbyfiziks.com                                                                                          
                                                           Email: fiziks.physics@gmail.com                                                                   3 
fiziks
Institute for NET/JRF, GATE, IIT‐JAM, JEST, TIFR and GRE in PHYSICAL SCIENCES 
 
JEST-2015
Q8. A circular loop of radius R , carries a uniform line charge density λ . The electric field,
calculated at a distance z directly above the center of the loop, is maximum if z is equal
to,
R R R
(a) (b) (c) (d) 2 R
3 2 2
Ans: (b)

Solution: E =
1 ( λ × 2π R ) z
4πε 0 ( R 2 + z 2 )3/ 2

λ × 2π R ⎡ ( R 2 + z 2 ) − z × 3 / 2 R 2 + z 2 × 2 z ⎤
3/ 2
dE
For maximum E , =0⇒ ⎢ ⎥=0
dz 4πε 0 ⎢ ( R 2
+ z )
2 3
⎥⎦

R
⇒ ( R2 + z 2 )
3/ 2
= 3z 2 R 2 + z 2 ⇒ R 2 + z 2 = 3z 2 ⇒ R 2 = 2 z 2 ⇒ z =
2
Q9. Consider two point charges q and λq located at the points, x = a and x = μa ,
respectively. Assuming that the sum of the two charges is constant, what is the value of
λ for which the magnitude of the electrostatic force is maximum?
1
(a) μ (b) 1 (c) (d) 1 + μ
μ
Ans: (b)

Solution: F =
1 (λq × q ) =
1 λq2
=
1 λc2
∵ q + λq = c
4πε 0 ( μ a − a )2 4πε 0 a 2 ( μ − 1)2 4πε 0 a 2 ( μ − 1)2 (1 + λ )2

dF 1 ⎡ (1 + λ )2 c 2 − λ c 2 × 2 (1 + λ ) ⎤
For maximum F , =0⇒ ⎢ ⎥=0
4πε 0 a ( μ − 1) (1 + λ )4
2
dz 2
⎣⎢ ⎦⎥

⇒ (1 + λ ) c 2 = λ c 2 × 2 (1 + λ ) ⇒ 1 + λ = 2λ ⇒ λ = 1
2

                                                                                
Head office  Branch office 
 
fiziks, H.No. 40 D, G.F, Jia Sarai,  Anand Institute of Mathematics, 
 
Near IIT, Hauz Khas, New Delhi‐16  28‐B/6, Jia Sarai, Near IIT 
 
Phone: 011‐26865455/+91‐9871145498 Hauz Khas, New Delhi‐16 
                                                   
                                             Website: www.physicsbyfiziks.com                                                                                          
                                                           Email: fiziks.physics@gmail.com                                                                   4 
fiziks
Institute for NET/JRF, GATE, IIT‐JAM, JEST, TIFR and GRE in PHYSICAL SCIENCES 
 
Q10. A spherical shell of inner and outer radii a and b , respectively, is made of a dielectric
k
material with frozen polarization P(r ) = rˆ ,where k is a constant and r is the distance
r
from the its centre. The electric field in the region a < r < b is,
k k
(a) E = rˆ (b) E = − rˆ
∈0 r ∈0 r
k
(c) E = 0 (d) E = rˆ
∈0 r 2
Ans: (b)
⎧ ˆ k ⎫
⎪ + P.r = (at r = b) ⎪
1 ∂ ⎛ 2 k ⎞ −k ⎪ b ⎪
Solution: ρb = −∇.P = − r ⎟ = 2 and σb = P.nˆ = ⎨ ⎬
2 ∂r ⎜⎝
r r⎠ r ⎪−P.rˆ = −k (at r = a) ⎪
⎪⎩ a ⎭⎪
⎛ −k ⎞ r ⎛ −k ⎞
⎟ × 4πa + ∫a ⎜ 2 ⎟ 4πr dr = −4πka − 4πk ( r − a ) = −4πkr
2 2
For a< r <b ; Qenc = ⎜
⎝ a ⎠ ⎝r ⎠
1 Qenc −k
E= ⇒E= rˆ
4πε 0 r 2 ε0 r

e −λ r
Q11. The electrostatic potential due to a charge distribution is given by V (r ) = A where
r
1
A and λ are constants The total charge enclosed within a sphere of radius , with its
λ
origin at r = 0 is given by,
8π ∈0 A 4π ∈0 A π ∈0 A
(a) (b) (c) (d) 0
e e e
Ans: (a)
e− λr
Solution: ∵V ( r ) = A
r
⎡ re − λ r × ( −λ ) − e − λ r ⎤ Ae− λ r
E = −∇V = − A ⎢ 2 ⎥ ˆ
r = 2 (1 + λ r ) rˆ
⎣ r ⎦ r
π 2π
Ae − λ r
Qenc = ε 0 ∫ E.d a = ε 0 ∫ ∫ 2 (1 + λ r ) rˆ.r 2 sin θ dθ dφ rˆ = 4πε 0 Ae− λr (1 + λ r )
0 0
r

                                                                                
Head office  Branch office 
 
fiziks, H.No. 40 D, G.F, Jia Sarai,  Anand Institute of Mathematics, 
 
Near IIT, Hauz Khas, New Delhi‐16  28‐B/6, Jia Sarai, Near IIT 
 
Phone: 011‐26865455/+91‐9871145498 Hauz Khas, New Delhi‐16 
                                                   
                                             Website: www.physicsbyfiziks.com                                                                                          
                                                           Email: fiziks.physics@gmail.com                                                                   5 
fiziks
Institute for NET/JRF, GATE, IIT‐JAM, JEST, TIFR and GRE in PHYSICAL SCIENCES 
 
1
Thus total charge enclosed within a sphere of radius r = is
λ
−λ
1
⎛ 1 ⎞ 8πε 0 A
Qenc = 4πε 0 Ae λ
⎜1 + λ ⎟ =
⎝ λ⎠ e
Q12. The skin depth of a metal is dependent on the conductivity (σ ) of the metal and the
angular frequency ω of the incident field. For a metal of high conductivity, which of the
following relations is correct? (Assume that σ >>∈ ω , where ∈ is the electrical
permittivity of the medium.)

σ 1
(a) d ∝ (b) d ∝
ω σω
ω
(c) d ∝ σω (d) d ∝
σ
Ans: (b)
2
Solution: Skin depth d =
σμω
o
Q13. The wavelength of red helium-neon laser in air is 6328 A . What happens to its frequency
in glass that has a refractive index of 1.50 ?
(a) Increases by a factor of 3
(b) Decreases by a factor of 1.5
(c) Remains the same
(d) Decreases by a factor of 0.5
Ans: (c)
Solution: Frequency of electromagnetic wave does not change when it enter in medium of any
refractive index.

                                                                                
Head office  Branch office 
 
fiziks, H.No. 40 D, G.F, Jia Sarai,  Anand Institute of Mathematics, 
 
Near IIT, Hauz Khas, New Delhi‐16  28‐B/6, Jia Sarai, Near IIT 
 
Phone: 011‐26865455/+91‐9871145498 Hauz Khas, New Delhi‐16 
                                                   
                                             Website: www.physicsbyfiziks.com                                                                                          
                                                           Email: fiziks.physics@gmail.com                                                                   6 
fiziks
Institute for NET/JRF, GATE, IIT‐JAM, JEST, TIFR and GRE in PHYSICAL SCIENCES 
 
Q14. The approximate force exerted on a perfectly reflecting mirror by an incident laser beam
of power 10 mW at normal incidence is

(a) 10 −13 N (b) 10 −11 N (c) 10 −9 N (d) 10 −15 N


Ans: (b)
Solution: When electromagnetic wave is reflected by mirror the momentum transferred to the
mirror per unit area per second is twice the momentum of the light striking the mirror per
unit area per second
dp 2 × Power 10 × 10 −3
=
i.e. = 2× = 6.6 × 10−11 kg m / s 2
dt c 3 × 10−8
dp
The force exerted on the reflecting mirror is F = = 6.6 × 10−11 N
dt
Thus best suitable answer is option (b).
Q15. Which of the following expressions represents an electric field due to a time varying
magnetic field?
(a) K ( xxˆ + yyˆ + zzˆ ) (b) K ( xxˆ + yyˆ − zzˆ )
(c) K ( xxˆ − yyˆ ) (d) K ( yyˆ − xyˆ + 2 zzˆ )
Ans: (d)
Solution: For time varying fields ∇ × E ≠ 0
xˆ yˆ zˆ
⎛ ∂z ∂y ⎞ ⎛ ∂x ∂z ⎞ ⎛ ∂y ∂x ⎞
(a) ∇ × E = K ∂ / ∂x ∂ / ∂y ∂ / ∂z = xˆ ⎜ − ⎟ + yˆ ⎜ − ⎟ + zˆ ⎜ − ⎟ = 0
x y z
⎝ ∂y ∂z ⎠ ⎝ ∂z ∂x ⎠ ⎝ ∂x ∂y ⎠

xˆ yˆ zˆ
⎛ ∂z ∂y ⎞ ⎛ ∂x ∂z ⎞ ⎛ ∂y ∂x ⎞
(b) ∇ × E = K ∂ / ∂x ∂ / ∂y ∂ / ∂z = xˆ ⎜ − − ⎟ + yˆ ⎜ + ⎟ + zˆ ⎜ − ⎟ = 0
x y −z
⎝ ∂y ∂z ⎠ ⎝ ∂z ∂x ⎠ ⎝ ∂x ∂y ⎠

xˆ yˆ zˆ
⎛ ∂y ⎞ ⎛ ∂x ⎞ ⎛ ∂y ∂x ⎞
(c) ∇ × E = K ∂ / ∂x ∂ / ∂y ∂ / ∂z = xˆ ⎜ 0 + ⎟ + yˆ ⎜ − 0 ⎟ + zˆ ⎜ − − ⎟ = 0
⎝ ∂z ⎠ ⎝ ∂z ⎠ ⎝ ∂x ∂y ⎠
x −y 0

                                                                                
Head office  Branch office 
 
fiziks, H.No. 40 D, G.F, Jia Sarai,  Anand Institute of Mathematics, 
 
Near IIT, Hauz Khas, New Delhi‐16  28‐B/6, Jia Sarai, Near IIT 
 
Phone: 011‐26865455/+91‐9871145498 Hauz Khas, New Delhi‐16 
                                                   
                                             Website: www.physicsbyfiziks.com                                                                                          
                                                           Email: fiziks.physics@gmail.com                                                                   7 
fiziks
Institute for NET/JRF, GATE, IIT‐JAM, JEST, TIFR and GRE in PHYSICAL SCIENCES 
 
xˆ yˆ zˆ
⎛ ∂ ( 2 z ) ∂x ⎞ ⎛ ∂x ∂ ( 2 z ) ⎞ ⎛ ∂y ∂y ⎞
(d) ∇ × E = K ∂ / ∂x ∂ / ∂y ∂ / ∂z = xˆ ⎜ + ⎟ + yˆ ⎜ − − ⎟ + zˆ ⎜ − ⎟
y −x 2z ⎝ ∂y ∂z ⎠ ⎝ ∂z ∂x ⎠ ⎝ ∂x ∂y ⎠

= − zˆ ≠ 0
Q16. A charged particle is released at time t = 0 , from the origin in the presence of uniform
static electric and magnetic fields given by E = E 0 yˆ and B = B0 zˆ respectively. Which of

the following statements is true for t > 0 ?


(a)The particle moves along the x -axis.
(b) The particle moves in a circular orbit.
(c) The particle moves in the ( x, y ) plane.
(d) Particle moves in the ( y, z ) plane
Ans: (c)
Solution: In a cycloid charged particle will always confine in a plane perpendicular to B.

JEST-2014
Q17. For an optical fiber with core and cladding index of n1 = 1.45 and n2 = 1.44 , respectively,
what is the approximate cut-off angle of incidence? Cut-off angle of incidence is defined
as the incidence angle below which light will be guided.
(a) 7 o (b) 22 o (c) 5 o (d) 0 o
Ans.: (a)
1/ 2
⎡ ⎛ n ⎞2 ⎤
Solution: θ = sin ⎢1 − ⎜⎜ 2 ⎟⎟ ⎥
−1
where n 2 = 1.44, n1 = 1.45
⎢⎣ ⎝ n1 ⎠ ⎥⎦

1/ 2
⎛ 1.44 × 1.44 ⎞
⇒ θ = sin −1 ( 0.11726 ) ⇒ θ = 6.670 ≈ 70
1
θ = sin −1 ⎜1 − ⎟
⎝ 1.45 × 1.45 ⎠

                                                                                
Head office  Branch office 
 
fiziks, H.No. 40 D, G.F, Jia Sarai,  Anand Institute of Mathematics, 
 
Near IIT, Hauz Khas, New Delhi‐16  28‐B/6, Jia Sarai, Near IIT 
 
Phone: 011‐26865455/+91‐9871145498 Hauz Khas, New Delhi‐16 
                                                   
                                             Website: www.physicsbyfiziks.com                                                                                          
                                                           Email: fiziks.physics@gmail.com                                                                   8 
fiziks
Institute for NET/JRF, GATE, IIT‐JAM, JEST, TIFR and GRE in PHYSICAL SCIENCES 
 
Q18. Two large nonconducting sheets one with a fixed uniform positive charge and another
with a fixed uniform negative charge are placed at a distance of 1 meter from each other.
The magnitude of the surface charge densities are σ + = 6.8μ C / m 2 for the positively

charged sheet and σ − = 4.3μ C / m 2 for the negatively charged sheet. What is the electric
field in the region between the sheets?
(a) 6.30 × 10 5 N / C (b) 3.84 × 10 5 N / C
(c) 1.40 × 10 5 N / C (d) 1.16 × 10 5 N / C
Ans.: (a)
σ+ σ− 6.8 × 10−6 4.3 × 10−6
Solution: Electric field between the sheet is = + = +
2 ∈0 2 ∈0 2 ∈0 2 ∈0

11.2 × 10−6
⇒ −12
= 0.626 × 106 ⇒ 6.3 × 105 N / C
2 × 8.86 × 10
Q19. A system of two circular co-axial coils carrying equal currents I along same direction
having equal radius R and separated by a distance R (as shown in the figure below).
The magnitude of magnetic field at the midpoint P is given by
μ0 I 4μ 0 I I I
(a) (b) Y
2 2R 5 5R
8μ 0 I
(c) (d) 0 X
5 5R P
R
Ans.: (c)
R
μ0 IR 2 μ0 IR 2 μ0 IR 2 R
Solution: ∵ B = 3
⇒ B1 = 3
, B2 = 3
∵d =
2
(
2 R2 + d )
2 2

2 ⎜ R2 +
R2 ⎞

2 ⎛
2 ⎜ R2 +
R2 ⎞

2

⎝ 4 ⎠ ⎝ 4 ⎠
3
μ0 I × 2 μ0 I 4 2 8μ0 I
B = B1 + B2 = 3
⇒B= 3
=
5 5R
⎛5⎞ 2
R 5 2
2 R⎜ ⎟
⎝4⎠

                                                                                
Head office  Branch office 
 
fiziks, H.No. 40 D, G.F, Jia Sarai,  Anand Institute of Mathematics, 
 
Near IIT, Hauz Khas, New Delhi‐16  28‐B/6, Jia Sarai, Near IIT 
 
Phone: 011‐26865455/+91‐9871145498 Hauz Khas, New Delhi‐16 
                                                   
                                             Website: www.physicsbyfiziks.com                                                                                          
                                                           Email: fiziks.physics@gmail.com                                                                   9 
fiziks
Institute for NET/JRF, GATE, IIT‐JAM, JEST, TIFR and GRE in PHYSICAL SCIENCES 
 
Q20. Find the resonance frequency (rad/sec) of the circuit shown in the figure below


V 0.25 F

2H

(a) 1.41 (b) 1.0 (c) 2.0 (d) 1.73


Ans.: (b)
1 R2
Solution: ω = − = 1.0 (where R = 2Ω, L = 2 H , C = 0.25 F )
LC L2
Q21. An electron is executing simple harmonic motion along the y-axis in right handed
coordinate system. Which of the following statements is true for emitted radiation?
(a) The radiation will be most intense in xz plane
(b) The radiation will be most intense in xy plane
(c) The radiation will violate causality
(d) The electron’s rest mass energy will reduce due to radiation loss
Ans.: (a)
Solution: Oscillating electron does not emit radiation in the direction of oscillation.
In the perpendicular direction of oscillation intensity is maximum.
So in this case the intensity will be maximum along x and z - axis or xz - plane
(intensity is also en xy -plane but less).
Q22. A conducting sphere of radius r has charge Q on its surface. If the charge on the sphere
is doubled and its radius is halved, the energy associated with the electric field will
(a) increase four times (b) increase eight times
(c) remain the same (d) decrease four times
Ans.: (b)
Q ∈ R ∈ ∞ Q2
Solution: E = rˆ W = 0 ∫ E 4π r dr + 0 ∫ E 4π r dr ⇒ W =
2 2 2 2

4π ∈0 r 8π ∈0 R
2 1 2
2 0 2 R

( 2Q )
2
8Q 2
⇒W′ = = = 8W
8π ∈0
R 8π ∈0 R
2
                                                                                
Head office  Branch office 
 
fiziks, H.No. 40 D, G.F, Jia Sarai,  Anand Institute of Mathematics, 
 
Near IIT, Hauz Khas, New Delhi‐16  28‐B/6, Jia Sarai, Near IIT 
 
Phone: 011‐26865455/+91‐9871145498 Hauz Khas, New Delhi‐16 
                                                   
                                             Website: www.physicsbyfiziks.com                                                                                          
                                                           Email: fiziks.physics@gmail.com                                                                   10 
fiziks
Institute for NET/JRF, GATE, IIT‐JAM, JEST, TIFR and GRE in PHYSICAL SCIENCES 
 
JEST-2013
Q23. At equilibrium, there can not be any free charge inside a metal. However, if you forcibly
put charge in the interior then it takes some finite time to ‘disappear’ i.e. move to the

surface. If the conductivity σ of a metal is 106 ( Ωm )


−1
and the dielectric constant

ε 0 = 8.85 × 10 −12 Farad/m, this time will be approximately:

(a) 10−5 sec (b) 10−11 sec (c) 10−9 sec (d) 10−17 sec
Ans.: (d)
∈ 8.85 × 10 −12
Solution: Characteristic time: τ = = = 8.85 × 10 −18
σ 10 6

Q24. The electric fields outside ( r > R ) and inside ( r < R ) a solid sphere with a uniform

1 q 1 q
volume charge density are given by Er>R = rˆ and Er<R = rrˆ
4πε 0 r 2 4πε 0 R 3
respectively, while the electric field outside a spherical shell with a uniform surface
1 q
charge density is given by Er < R = rˆ , q being the total charge. The correct ratio
4πε 0 r 2
of the electrostatic energies for the second case to the first case is
(a) 1: 3 (b) 9 :16 (c) 3 : 8 (d) 5 : 6
Ans.: (d)
∈0 R 2 ∈0 ∞ 2
Solution: Electrostatic energy in spherical shell wsp =
2 ∫0
E1 4π r 2 dr +
2 ∫R
E 2 4π r 2 dr


∈0 ∞ q2 q2 ⎛ 1⎞ q2 1

2 ∫ (4π ∈ )
R 2
r4
4π r 2 dr =
8π ∈0
⎜ − ⎟ =
⎝ r ⎠ R 8π ∈0 R
0

∈0 R ∈0 ∞
∫ ∫
2 2
Electrostatic energy in solid sphere ws = E1 4π r 2 dr + E 2 4π r 2 dr
2 0 2 R

R ∞
q2 1 ⎡r5 ⎤ q2 ⎡ 1⎤
⇒ × 6⎢ ⎥ + ⎢⎣− r ⎥⎦
8π ∈0 R ⎣ 5 ⎦ 0 8π ∈0 R

q2 1 q2 6q 2
ws = ⋅ + =
5 × 8π ∈0 R 8π ∈0 R 40π ∈0 R
                                                                                
Head office  Branch office 
 
fiziks, H.No. 40 D, G.F, Jia Sarai,  Anand Institute of Mathematics, 
 
Near IIT, Hauz Khas, New Delhi‐16  28‐B/6, Jia Sarai, Near IIT 
 
Phone: 011‐26865455/+91‐9871145498 Hauz Khas, New Delhi‐16 
                                                   
                                             Website: www.physicsbyfiziks.com                                                                                          
                                                           Email: fiziks.physics@gmail.com                                                                   11 
fiziks
Institute for NET/JRF, GATE, IIT‐JAM, JEST, TIFR and GRE in PHYSICAL SCIENCES 
 
2
q
W 8π ∈0 5
Now spherical = 2
=
Wsphere 6q 6
40π ∈0 R
Q25. A thin uniform ring carrying charge Q and mass M rotates about its axis. What is the
gyromagnetic ratio (defined as ratio of magnetic dipole moment to the angular
momentum) of this ring?
(a) Q / (2πM ) (b) Q/M (c) Q/(2M) (d) Q / (πM )
Ans.: (c)
Q 2 Q Qω r 2
Solution: Magnetic dipole moment M ′ = IA = πr ⇒ × 2π × π r 2 =
T 2π T 2
M′ Q
Angular momentum J = Mr 2ω ⇒ =
J 2M
Q26. The electric and magnetic field caused by an accelerated charged particle are found to
scale as E ∝ r − n and B ∝ r − m at large distances. What are the value of n and m ?
(a) n = 1, m = 2 (b) n = 2, m = 1 (c) n = 1, m = 1 (d) n = 2, m = 2
Ans.: (c)
qa sin θ qa sin θ 1 1
Solution: For large distance F = , B= ⇒E∝ , B∝
r r r r
So m = n = 1

Q27. If E1 = xyiˆ + 2 yzˆj + 3xzkˆ and E 2 = y 2 iˆ + (2 xy + z 2 ) ˆj + 2 yzkˆ then


(a) Both are impossible electrostatic fields
(b) Both are possible electrostatic fields
(c) Only E1 is a possible electrostatic field

(d) Only E 2 is a possible electrostatic field


Ans.: (d)
Solution: For electrostatic field ∇ × E = 0

                                                                                
Head office  Branch office 
 
fiziks, H.No. 40 D, G.F, Jia Sarai,  Anand Institute of Mathematics, 
 
Near IIT, Hauz Khas, New Delhi‐16  28‐B/6, Jia Sarai, Near IIT 
 
Phone: 011‐26865455/+91‐9871145498 Hauz Khas, New Delhi‐16 
                                                   
                                             Website: www.physicsbyfiziks.com                                                                                          
                                                           Email: fiziks.physics@gmail.com                                                                   12 
fiziks
Institute for NET/JRF, GATE, IIT‐JAM, JEST, TIFR and GRE in PHYSICAL SCIENCES 
 
iˆ ˆj kˆ
∂ ∂ ∂
∇ × E2 =
∂x ∂y ∂z
y2 2 xy + z 2 2 yz

(2 z − 2 z )iˆ + 0 + (2 y − 2 y )zˆ = 0
iˆ ˆj kˆ
∂ ∂ ∂
∇ × E1 = = ( 0 − 2 y ) iˆ + 0 + xjˆ ≠ 0
∂x ∂y ∂z
xy ∂yz yxz
Q28. A charge q is placed at the centre of an otherwise neutral dielectric sphere of radius a and
relative permittivity ε r . We denote the expression q / 4πε 0 r 2 by E ( r ) . Which of the

following statements is false?


(a) The electric field inside the sphere, r < a , is given by E (r ) / ε r

(b) The field outside the sphere, r > a , is given by E ( r )

(c) The total charge inside a sphere of radius r > a is given by q .


(d) The total charge inside a sphere of radius r < a is given by q .
Ans.: (d)
Q29. An electromagnetic wave of frequency ω travels in the x - direction through vacuum. It
ω
is polarized in the y - direction and the amplitude of the electric field is E0 . With k =
c
where c is the speed of light in vacuum, the electric and the magnetic fields are then
conventionally given by
E0
(a) E = E 0 cos(ky − ω t )xˆ and B = cos(ky − ω t ) zˆ
c
E
(b) E = E 0 cos(kx − ω t ) yˆ and B = 0 cos(kx − ω t ) zˆ
c
E
(c) E = E 0 cos(kx − ω t )zˆ and B = 0 cos(ky − ω t ) yˆ
c
E
(d) E = E 0 cos(kx − ω t )xˆ and B = 0 cos(ky − ω t ) yˆ
c
                                                                                
Head office  Branch office 
 
fiziks, H.No. 40 D, G.F, Jia Sarai,  Anand Institute of Mathematics, 
 
Near IIT, Hauz Khas, New Delhi‐16  28‐B/6, Jia Sarai, Near IIT 
 
Phone: 011‐26865455/+91‐9871145498 Hauz Khas, New Delhi‐16 
                                                   
                                             Website: www.physicsbyfiziks.com                                                                                          
                                                           Email: fiziks.physics@gmail.com                                                                   13 
fiziks
Institute for NET/JRF, GATE, IIT‐JAM, JEST, TIFR and GRE in PHYSICAL SCIENCES 
 
Ans.: (b)
Solution: E = E0 cos ( kx − ω t ) yˆ

B=
1 ˆ
c
( 1
)
k × E ⇒ B = ⎡⎣ xˆ × E0 cos ( kx − ωt ) yˆ ⎤⎦
c
E0 E
⇒B= cos ( kx − ω t )( xˆ × yˆ ) ⇒ B = 0 cos ( kx − ω t )( zˆ )
c c
JEST-2012
Q30. ( )
A magnetic field B = B0 iˆ + 2 ˆj − 4kˆ exists at point. If a test charge moving with a

( )
velocity, v = v 0 3iˆ − ˆj + 2kˆ experiences no force at a certain point, the electric field at
that point in SI units is

(
(a) E = −v 0 B0 3iˆ − 2 ˆj − 4kˆ ) ( )
(b) E = −v 0 B0 iˆ + ˆj + 7kˆ

(
(c) E = v0 B0 14 ˆj + 7kˆ ) (d) E = −v B (14 ˆj + 7kˆ )
0 0

Ans. : (d)
[
Solution: F = q E + v × B = 0 ⇒ E = − v × B ] ( )
⇒ E = −v0 B0 {( 4 − 4) iˆ + ( 2 + 12) ˆj + ( 6 + 1) kˆ} (
= −v 0 B0 14 ˆj + 7 kˆ )
Q31. An observer in an inertial frame finds that at a point P the electric field vanishes but the
magnetic field does not. This implies that in any other inertial frame the electric field E
and the magnetic field B satisfy
2 2
(a) E = B (b) E ⋅ B = 0 (c) E × B = 0 (d) E = 0
Ans.: (b)
Q32. A circular conducting ring of radius R rotates with constant angular velocity ω about its
diameter placed along the x-axis. A uniform magnetic field B is applied along the y-axis.
If at time t = 0 the ring is entirely in the xy-plane, the emf induced in the ring at time
t > 0 is
(a) Bω 2πR 2 t (b) Bωπ R 2 tan (ωt )

(c) Bωπ R 2 sin (ωt ) (d) Bωπ R 2 cos (ωt )

                                                                                
Head office  Branch office 
 
fiziks, H.No. 40 D, G.F, Jia Sarai,  Anand Institute of Mathematics, 
 
Near IIT, Hauz Khas, New Delhi‐16  28‐B/6, Jia Sarai, Near IIT 
 
Phone: 011‐26865455/+91‐9871145498 Hauz Khas, New Delhi‐16 
                                                   
                                             Website: www.physicsbyfiziks.com                                                                                          
                                                           Email: fiziks.physics@gmail.com                                                                   14 
fiziks
Institute for NET/JRF, GATE, IIT‐JAM, JEST, TIFR and GRE in PHYSICAL SCIENCES 
 
Ans. : (c)
Solution: φ m = B ⋅ A = BA cos θ = BA cos ω t

dφm
ε =−
dt
=−
d
dt
d
dt
( )
B ⋅ A = − [ BA cos ωt ] = − BA(− sin ω t )ω

⇒ ε = Bπ R 2ω sin ωt ⇒ ε = Bωπ R 2 sin ωt


Q33. An electric field in a region is given by E ( x, y, z ) = ax iˆ + czˆj + 6bykˆ . For which values of
a, b, c does this represent an electrostatic field?
(a) 13, 1, 12 (b) 17, 6, 1 (c) 13, 1, 6 (d) 45, 6, 1
Ans.: (c)
Solution: For electrostatic field ∇ × E = 0
⎡ iˆ ˆj kˆ ⎤
⎢ ⎥
∂ ∂ ∂ ⎥
∇× E = ⎢ = 0 ⇒ ∇ × E = ( 6b − c ) iˆ + ˆj [ 0 − 0] + kˆ [ 0] = 0
⎢ ∂x ∂y ∂z ⎥
⎢ ⎥
⎣⎢ ax cz 6by ⎦⎥

⇒ ( 6b − c ) iˆ = 0 ⇒ c = 6b

Q34. A capacitor C is connected to a battery V0 through three equal resistors R and a switch S
as shown below:
S R R

R C
V0

The capacitor is initially uncharged. At time t = 0 , the switch S is closed. The voltage
across the capacitor as a function of time t for t > 0 is given by
(a) (V0 / 2 ) (1 − exp ( −t / 2 RC ) ) (b) (V0 / 3) (1 − exp ( −t / 3RC ) )

(c) (V0 / 3) (1 − exp ( −3t / 2 RC ) ) (d) (V0 / 2 ) (1 − exp ( −2t / 3RC ) )

Ans.: (d)

                                                                                
Head office  Branch office 
 
fiziks, H.No. 40 D, G.F, Jia Sarai,  Anand Institute of Mathematics, 
 
Near IIT, Hauz Khas, New Delhi‐16  28‐B/6, Jia Sarai, Near IIT 
 
Phone: 011‐26865455/+91‐9871145498 Hauz Khas, New Delhi‐16 
                                                   
                                             Website: www.physicsbyfiziks.com                                                                                          
                                                           Email: fiziks.physics@gmail.com                                                                   15 
fiziks
Institute for NET/JRF, GATE, IIT‐JAM, JEST, TIFR and GRE in PHYSICAL SCIENCES 
 
Solution: S +R − + R −

i1 + i2
R i1 − i2
V0 C

Apply KVL in loop 1: −V0 + i1 ( t ) R + ( i1 − i2 ) R = 0 ⇒ 2i1 ( t ) R − i2 ( t ) R = V0 ….. (i)

Apply KVL in loop 2:


t t
1 1 ⎛ V0 + i2 R ⎞
( ) ∫
C ∫0
i2 R + i 2 dt − i1 − i 2 R = 0 ⇒ i2 dt + 2i2 R − ⎜ ⎟=0
C0 ⎝ 2 ⎠
t
1 3 −V

C0∫ i2 dt + i2 R = 0
2 2
……………(ii)

−2
1 3 di 3 di 1 di −2 t
⇒ i2 + R 2 = 0 ⇒ R 2 = − i2 ⇒ 2 = i2 ⇒ i2 ( t ) = Ke 3 RC
C 2 dt 2 dt C dt 3RC
Initial Conditions

( )
i1 0 + =
V0
R× R
=
V0
=
2 V0
R 3 R
and i2 ( 0+ ) =
R 2V V
× 0 = 0 , vC ( 0+ ) = 0
R + R 3R 3R
R+ R+
R+R 2
V0 V V −2 t
∵ i2 ( 0+ ) = ⇒ K = 0 ⇒ i2 ( t ) = 0 e 3 RC
3R 3R 3R
t
⎡ −2t ⎤
−2 t
V ⎢ e 3 RC ⎥
t t
1 V
vC ( t ) = ∫ i2 dt = 0 ∫ e 3 RC
dt = 0 ⎢ ⎥
C0 3RC 0 3RC ⎢ −2 RC ⎥
⎣ 3 ⎦0

V0 −3RC ⎡ 3−RC
2t
⎤ V ⎡ −2 t

⇒ vC ( t ) = × ⎢ e − 1⎥ ⇒ vC ( t ) = 0 ⎢1 − e 3 RC

3RC 2 ⎣ ⎦ 2 ⎣ ⎦

                                                                                
Head office  Branch office 
 
fiziks, H.No. 40 D, G.F, Jia Sarai,  Anand Institute of Mathematics, 
 
Near IIT, Hauz Khas, New Delhi‐16  28‐B/6, Jia Sarai, Near IIT 
 
Phone: 011‐26865455/+91‐9871145498 Hauz Khas, New Delhi‐16 
                                                   
                                             Website: www.physicsbyfiziks.com                                                                                          
                                                           Email: fiziks.physics@gmail.com                                                                   16 
fiziks
Institute for NET/JRF, GATE, IIT‐JAM, JEST, TIFR and GRE in PHYSICAL SCIENCES 
 
Q35. A small magnet is dropped down a long vertical copper tube in a uniform gravitational
field. After a long time, the magnet
(a) attains a constant velocity (b) moves with a constant acceleration
(c) moves with a constant deceleration (d) executes simple harmonic motion
Ans. : (a)
Q36. Consider a particle of electric charge e and mass m moving under the influence of a
constant horizontal electric field E and constant vertical gravitational field described by
acceleration due to gravity g. If the particle starts from rest, what will be its trajectory?
(a) parabolic (b) elliptic (c) straight line (d) circular
Ans.: (c)
md 2 x dx
Solution: Equation of motion qE = 2
⇒ = α 1t + c1
dt dt
qE dx α t2
at t = 0, v = 0 , α 1 = = α1t ⇒ x = 1
m dt 2
md 2 y
similarly, mg =
dt 2
α 2t 2 α2x
y= ⇒y= , α2 = g
2 α1
Q37. A point charge +q is placed at (0, 0, d) above a grounded infinite conducting plane
defined by z = 0. There are no charges present anywhere else. What is the magnitude of
electric field at (0, 0, - d)?
(a) q / (8π ∈0 d 2 ) (b) - ∞ (c) 0 (d) q / (16π ∈0 d 2 )
Ans.: (d)
Solution: Electric field at Q
−q −q
E= (zˆ ) = zˆ
4π ∈0 (2d ) 16π ∈0 d 2
2

q
E =
16π ∈0 d 2

                                                                                
Head office  Branch office 
 
fiziks, H.No. 40 D, G.F, Jia Sarai,  Anand Institute of Mathematics, 
 
Near IIT, Hauz Khas, New Delhi‐16  28‐B/6, Jia Sarai, Near IIT 
 
Phone: 011‐26865455/+91‐9871145498 Hauz Khas, New Delhi‐16 
                                                   
                                             Website: www.physicsbyfiziks.com                                                                                          
                                                           Email: fiziks.physics@gmail.com                                                                   17 
fiziks
Institute for NET/JRF, GATE, IIT‐JAM, JEST, TIFR and GRE in PHYSICAL SCIENCES 
 
Q38. A time-dependent magnetic field B(t ) is produced in a circular region of space, infinitely

long and of radius R. The magnetic field is given as B = B0 t zˆ and is zero for r > R,
where B0 is a positive constant. The electric field at point r = 2 R is
B0 R B0 R ˆ B0 R B0 R ˆ
(a) rˆ (b) − θ (c) − rˆ (d) θ
2 4 2 4
Ans.: (b)
⎛ ∂B ⎞
∫line ⋅ = ∫ ⎜⎝ − ∂t ⎟⎠ ⋅ da ⇒ E × 2π r = − B0π R
2
Solution: Solution: E dl

R2 B R2
⇒ E = − B0 ⇒ E = − 0 θˆ
2r 2r
B0 R ˆ
The electric field at point r = 2 R is E = − θ
4
Q39. When unpolarised light is incident on a glass plate at a particular angle, it is observed that
the reflected beam is linearly polarized. What is the angle of the refracted beam with
respect to the surface normal?
(a) 56.7°
(b) 33.4°
(c) 23.3°
(d) The light is completely reflected and there is no refracted beam.
Ans.: (b)
Solution: Since n1 = 1 , n 2 = 1.52

⎛n ⎞ ⎛ 1.52 ⎞
Brewster angle θ B = tan −1 ⎜⎜ 2 ⎟⎟ = tan −1 ⎜ ⎟ = 56.7
0

⎝ n1 ⎠ ⎝ 1 ⎠

Now θ R = 180 − 90 − 56.7 = 33.40


Q40. A cube has a constant electric potential V on its surface. If there are no charges inside the
cube, the potential at the center of the cube is
(a) V (b) V / 8 (c) 0 (d) V / 6
Ans.: (a)
                                                                                
Head office  Branch office 
 
fiziks, H.No. 40 D, G.F, Jia Sarai,  Anand Institute of Mathematics, 
 
Near IIT, Hauz Khas, New Delhi‐16  28‐B/6, Jia Sarai, Near IIT 
 
Phone: 011‐26865455/+91‐9871145498 Hauz Khas, New Delhi‐16 
                                                   
                                             Website: www.physicsbyfiziks.com                                                                                          
                                                           Email: fiziks.physics@gmail.com                                                                   18 
fiziks
Institute for NET/JRF, GATE, IIT‐JAM, JEST, TIFR and GRE in PHYSICAL SCIENCES 
 
Quantum Mechanics
JEST-2016
Q1. The wavefunction of a hydrogen atom is given by the following superposition of energy
eigen functions υ/ nlm ( r ) ( n, l , m are the usual quantum numbers):

2 3 1
υ/ ( r ) = υ/ 100 ( r ) − υ/ 210 ( r ) + υ/ 322 ( r )
7 14 14
The ratio of expectation value of the energy to the ground state energy and the
expectation value of L2 are, respectively:
229 12 2 101 12 2
(a) and (b) and
504 7 504 7
101 2 229 2
(c) and (d) and
504 504
Ans: (a)
2 E 9 E 1 E 229
Solution: E = × 0 + × 0 + × 0 = E0
7 1 14 4 14 9 504
2 9 1 24 12
L2 = × 0 2
+ ×2 2
+ ×6 2
= 2
= 2
7 14 14 14 7
1
Q2. A spin- particle in a uniform external magnetic field has energy eigenstates 1 and
2

2 . The system is prepared in ket-state


( 1 + 2 ) at time t = 0. It evolves to the state
2

described by the ket


(1 − 2 ) in time T . The minimum energy difference between two
2
levels is:
h h h h
(a) (b) (c) (d)
6T 4T 2T T
Ans: (c)

                                                                                
Head office  Branch office 
 
fiziks, H.No. 40 D, G.F, Jia Sarai,  Anand Institute of Mathematics, 
 
Near IIT, Hauz Khas, New Delhi‐16  28‐B/6, Jia Sarai, Near IIT 
 
Phone: 011‐26865455/+91‐9871145498 Hauz Khas, New Delhi‐16 
                                                   
                                             Website: www.physicsbyfiziks.com                                                                                          
                                                           Email: fiziks.physics@gmail.com                                                                   1 
fiziks
Institute for NET/JRF, GATE, IIT‐JAM, JEST, TIFR and GRE in PHYSICAL SCIENCES 
 
⎛ ⎛ E1t ⎞ ⎛ E t ⎞⎞
1 ⎜ −i + 2 exp ⎜ −i 2 ⎟ ⎟
Solution: ψ ( t = 0 ) =
(1 + 2 )⇒ ψ t =t = ⎝ ⎝⎜ ⎟
⎠ ⎝ ⎠⎠
( )
2 2

⎛ ⎛ ( E2 − E1 ) t ⎞ ⎞
⎜⎜ 1 + 2 exp ⎜ −i ⎟ ⎟⎟
⎛ E1t ⎞ ⎝ ⎝ ⎠⎠
ψ (t = t ) = ⎜ −i ⎟
⎝ ⎠ 2
⎛ ( E − E1 ) t ⎞
exp ⎜ −i 2 ⎟ = −1
⎝ ⎠
( E2 − E1 ) T = π ⇒ E − E = π h
( 2 1) =
T 2T
Q3. The energy of a particle is given by E = p + q where p and q are the generalized

momentum and coordinate, respectively. All the states with E ≤ E0 are equally probable

and states with E > E0 are inaccessible. The probability density of finding the particle at

coordinate q , with q > 0 is:

(a)
( E0 + q ) (b)
q
(c)
( E0 − q ) (d)
1
2
E 0 E02 E 2
0 E0
Ans: (c)
Solution: For condition E = p + q total no of accessible state upto energy E0 for q > 0 is area

1
under the curve × 2 × E02 = E02
2
The probability density of finding the particle at coordinate q , with q > 0

dpdq pdq ( E − q ) dq
= 2 ⇒ 0 2
2
E0 E0 E0

For probability at point point q dq is insignificant so ( p ( q ) ) =


( E0 − q )
E02

                                                                                
Head office  Branch office 
 
fiziks, H.No. 40 D, G.F, Jia Sarai,  Anand Institute of Mathematics, 
 
Near IIT, Hauz Khas, New Delhi‐16  28‐B/6, Jia Sarai, Near IIT 
 
Phone: 011‐26865455/+91‐9871145498 Hauz Khas, New Delhi‐16 
                                                   
                                             Website: www.physicsbyfiziks.com                                                                                          
                                                           Email: fiziks.physics@gmail.com                                                                   2 
fiziks
Institute for NET/JRF, GATE, IIT‐JAM, JEST, TIFR and GRE in PHYSICAL SCIENCES 
 
Q4. Consider a quantum particle of mass m in one dimension in an infinite potential well,
−a a a
i.e., V ( x ) = 0 for <x< and V ( x ) = ∞ for x≥ . A small perturbation,
2 2 2
2∈ x
V ′( x) = is added. The change in the ground state energy to O (∈) is:
a
∈ ∈
(a)
2π 2 (
π2 −4 ) (b)
2π 2 (
π2 +4 )
∈π 2 2 ∈π 2 2
(c)
2
(π + 4 ) (d)
2
(π − 4 )
Ans: (a)
a a
2∈ 2 2 2 πx
2
Solution: E11 = ∫ φ1*V ' ( x )φ1dx ⇒ ∫ x cos dx

a a −a a a
2 2

a a a
2∈ 2 2 πx 4∈ 2 1 ⎛ 2π x ⎞ 2∈ 2 ⎛ 2π x ⎞
= .2. ∫ x cos 2 dx ⇒ 2 ∫ x ⎜ cos + 1⎟dx ⇒ 2 ∫ x ⎜ cos + 1⎟dx ⇒
a 0 a a a 0 2⎝ a ⎠ a 0 ⎝ a ⎠
a
2∈ 2 ⎛ 2π x ⎞ ∈
x cos
2 ∫ ⎜
a 0 ⎝ a
+ 1⎟dx =
⎠ 2π 2
π2 −4 ( )
1
Q5. If Yxy =
2
(Y22 − Y2,−2 ) where Yl ,m are spherical harmonics then which of the following is
true?
(a) Yxy is an eigenfunction of both L2 and Lz

(b) Yxy is an eigenfunction of L2 but not Lz

(c) Yxy is an eigenfunction both of Lz but not L2

(d) Yxy is not an eigenfunction of either L2 and Lz

Ans: (b)
Solution: The L2Yxy = l ( l + 1) 2Yxy where l = 2 and LzYxy ≠ mYxy

So Yxy is an eigenfunction of L2 but not Lz


                                                                                
Head office  Branch office 
 
fiziks, H.No. 40 D, G.F, Jia Sarai,  Anand Institute of Mathematics, 
 
Near IIT, Hauz Khas, New Delhi‐16  28‐B/6, Jia Sarai, Near IIT 
 
Phone: 011‐26865455/+91‐9871145498 Hauz Khas, New Delhi‐16 
                                                   
                                             Website: www.physicsbyfiziks.com                                                                                          
                                                           Email: fiziks.physics@gmail.com                                                                   3 
fiziks
Institute for NET/JRF, GATE, IIT‐JAM, JEST, TIFR and GRE in PHYSICAL SCIENCES 
 

Q6.
⎛ 1 ⎞
A spin- 1 particle is in a state υ/ described by the colunm matrix ⎜ ⎟ 2,
⎝ 10 ⎠
2, 2i { }
in the S z basis. What is the probability that a measurement of operator S z will yield the

result h for the state S x υ/ ?

1 1 1 1
(a) (b) (c) (d)
2 3 4 6
Ans: (c)

⎛0 1 0⎞ ⎛ 2 ⎞
⎜ ⎟ ⎜ ⎟
Solution: S x = 1 0 1 ψ = ⎜ 2⎟
2 ⎜⎜ ⎟
⎟ ⎜ 2i ⎟
⎝0 1 0⎠ ⎝ ⎠
⎛ 2 ⎞
⎜ ⎟
S x ψ = ⎜ 2 + 2i ⎟
⎜ ⎟
⎝ 2 ⎠
⎛1 0 0 ⎞ ⎛1⎞
⎜ ⎟ ⎜ ⎟
S z = ⎜ 0 0 0 ⎟ the eigen state for eigen value of sz is ⎜ 0 ⎟
⎜ 0 0 −1 ⎟ ⎜0⎟
⎝ ⎠ ⎝ ⎠
2 2 1
p( )= = =
2+4+2 8 4
Q7. The Hamiltonian of a quantum particle of mass in confined to a ring of unit radius is:
2
⎛ ∂
2

H= ⎜ −i −α ⎟
2m ⎝ ∂θ ⎠
where θ is the angular coordinate, α is a constant. The energy eigenvalues and
eigenfunctions of the particle are ( n is an integer):
einθ 2
sin ( nθ ) 2
(a) υ/ n (θ ) = (n −α ) (b) υ/ n (θ ) = (n −α )
2 2
and En = and En =
2π 2m 2π 2m
cos ( nθ ) 2
einθ 2
(c) υ/ n (θ ) = (n −α ) (d) υ/ n (θ ) = (n +α )
2 2
and En = and En =
2π 2m 2π 2m
Ans: (a)
                                                                                
Head office  Branch office 
 
fiziks, H.No. 40 D, G.F, Jia Sarai,  Anand Institute of Mathematics, 
 
Near IIT, Hauz Khas, New Delhi‐16  28‐B/6, Jia Sarai, Near IIT 
 
Phone: 011‐26865455/+91‐9871145498 Hauz Khas, New Delhi‐16 
                                                   
                                             Website: www.physicsbyfiziks.com                                                                                          
                                                           Email: fiziks.physics@gmail.com                                                                   4 
fiziks
Institute for NET/JRF, GATE, IIT‐JAM, JEST, TIFR and GRE in PHYSICAL SCIENCES 
 
2
2
⎛ ∂ ⎞
2
⎡ ∂ψ 2
∂ψ ⎤
Solution: H = ⎜ −i −α ⎟ ⇒ ⎢ − 2 + 2iα + α 2ψ ⎥ = Eψ
2m ⎝ ∂θ ⎠ 2m ⎣ ∂θ ∂θ ⎦
einθ
By inspection ψ n (θ ) = wich will also satisfied boundary condition

ψ n (θ + 2π ) = ψ n (θ ) will satisfied the Hamiltonian and value of E =


2
( n − α )2
2m
d
Q8. The adjoint of a differential operator acting on a wavefunction υ/ ( x ) for a quantum
dx
mechanical system is:
d d d d
(a) (b) −i (c) − (d) i
dx dx dx dx
Ans: (c)
⎛ 1⎞
Q9. For a quantum mechanical harmonic oscillator with energies, En = ⎜ n + ⎟ ω , where
⎝ 2⎠
n = 0,1, 2... , the partition function is:
ω ω
k BT ω ω 2 k BT
e e
(a) ω
(b) e 2 kBT − 1 (c) e 2 kBT + 1 (d) ω

e 2 kBT −1 e kBT −1
Ans: (d)
ω 3 ω 5 ω 7 ω
Solution: z = exp− + exp− + exp− + exp− ......
2kT 2kT 2kT 2kT
ω ⎛ ω 2 ω ⎞
z = exp− ⎜1 + exp− + exp− ..... ⎟
2kT ⎝ kT kT ⎠
ω ω
exp− exp
2kT ⇒ 2kT 1
z= ⇒
ω ω ω ω
1 − exp exp − exp− exp −1
kT 2kT 2kT kT

                                                                                
Head office  Branch office 
 
fiziks, H.No. 40 D, G.F, Jia Sarai,  Anand Institute of Mathematics, 
 
Near IIT, Hauz Khas, New Delhi‐16  28‐B/6, Jia Sarai, Near IIT 
 
Phone: 011‐26865455/+91‐9871145498 Hauz Khas, New Delhi‐16 
                                                   
                                             Website: www.physicsbyfiziks.com                                                                                          
                                                           Email: fiziks.physics@gmail.com                                                                   5 
fiziks
Institute for NET/JRF, GATE, IIT‐JAM, JEST, TIFR and GRE in PHYSICAL SCIENCES 
 
Q10. In the ground state of hydrogen atom, the most probable distance of the electron from the
nucleus, in units of Bohr radius a0 is:

1 3
(a) (b) 1 (c) 2 (d)
2 2
Ans: (d)
3
Solution: r = a0 most probable distance is rp = a0
2
r 3
=
rp 2

Q11. For operators P and Q , the commutator ⎡⎣ P, Q −1 ⎤⎦ is

(a) Q −1 [ P, Q ] Q −1 (b) −Q −1 [ P, Q ] Q −1 (c) Q −1 [ P, Q ] Q (d) −Q [ P, Q ] Q −1

Ans (b)
⎡⎣ P, Q −1 ⎤⎦ = PQ −1 − Q −1 P

−Q −1 [ P, Q ] Q −1 ⇒ −Q −1 [ PQ − QP ] Q −1 = −Q −1 ⎡⎣ PQQ −1 − QPQ −1 ⎤⎦ = −Q −1 P + PQ −1 = ⎡⎣ P, Q −1 ⎤⎦

Q12. A spin
1
particle is in a state
(↑ + ↓ ) where ↑ and ↓ are the eigenstates of S z
2 2
operator. The expectation value of the spin angular momentum measured along x
direction is:

(a) (b) − (c) 0 (d)


2
Ans: (d)
⎛ 1 ⎞

Solution:
( ↑ + ↓ ) ⎜
⇒⎜
2 ⎟⎟
sx = ⎜
⎛0 1⎞

2 ⎜ 1 ⎟ 2 ⎝1 0⎠
⎜ ⎟
⎝ 2⎠

                                                                                
Head office  Branch office 
 
fiziks, H.No. 40 D, G.F, Jia Sarai,  Anand Institute of Mathematics, 
 
Near IIT, Hauz Khas, New Delhi‐16  28‐B/6, Jia Sarai, Near IIT 
 
Phone: 011‐26865455/+91‐9871145498 Hauz Khas, New Delhi‐16 
                                                   
                                             Website: www.physicsbyfiziks.com                                                                                          
                                                           Email: fiziks.physics@gmail.com                                                                   6 
fiziks
Institute for NET/JRF, GATE, IIT‐JAM, JEST, TIFR and GRE in PHYSICAL SCIENCES 
 
⎛ 1 ⎞
⎛ 1 1 ⎞ ⎛0 1⎞⎜ 2 ⎟
sx = ⎜ ⎟ ⎜ ⎟⎜ ⎟=
⎝ 2 2 ⎠ 2 ⎝1 0⎠⎜ 1 ⎟ 2
⎜ ⎟
⎝ 2⎠

JEST-2015
2
α
− +
Q13. Consider a harmonic oscillator in the state ψ = e 2
eα a 0 , where 0 is the ground

state, a + is the raising operator and α is a complex number. What is the probability that
the harmonic oscillator is in the n -th eigenstate n ?
2 n
2n a a
−α2 α −
2
(a) e (b) e n!
n!
n 2 2n
α
−α
2 α − α
(c) e (d) e 2
n! n!
Ans: (a)

(α a ) (a )
n n
α
2
α
2 + +

( )
− −

+ n
Solution: ψ = e 2
eα a 0 = e 2
0 and n = 0 ⇒ a+ 0 = n n
n n n

(α )
2 n
α
− n
ψ =e 2
∑ n n
n

(α α )
n n
*
n α
∑ ∑
2 2 2 2
−α −α
⇒ ψ ψ =e n n =e = e− α e α = 1
( n)
2
n n
2
nψ 2
Probability that ψ is in n state = nψ
ψ ψ
(α )
2 n 2
α α
− n − 1
ψ =e 2
∑ n n
n =e 2
∑α
n
n

n
n
2
α
α
2 − 2n
− 1 e 2 n α
∑α
2
2 −α
⇒ nψ =e 2 n
nn = α ⇒ nψ =e
n n n n

                                                                                
Head office  Branch office 
 
fiziks, H.No. 40 D, G.F, Jia Sarai,  Anand Institute of Mathematics, 
 
Near IIT, Hauz Khas, New Delhi‐16  28‐B/6, Jia Sarai, Near IIT 
 
Phone: 011‐26865455/+91‐9871145498 Hauz Khas, New Delhi‐16 
                                                   
                                             Website: www.physicsbyfiziks.com                                                                                          
                                                           Email: fiziks.physics@gmail.com                                                                   7 
fiziks
Institute for NET/JRF, GATE, IIT‐JAM, JEST, TIFR and GRE in PHYSICAL SCIENCES 
 
Q14. For non-interacting Fermions in d − dimensions, the density of states D (E ) varies as
⎛d ⎞
⎜ −1 ⎟
E ⎝2 ⎠
. The Fermi energy E F of an N particle system in 3−, 2 − and 1 − dimensions
will scale respectively as,
(a) N 2 , N 2 / 3 , N (b) N , N 2 / 3 , N 2

(c) N , N 2 , N 2 / 3 (d) N 2 / 3 , N , N 2
Ans: (d)
Q15. A particle of mass m moves in 1 − dimensional potential V ( x ) , which vanishes at infinity.

The exact ground state eigenfunction is υ/ ( x ) = A sec h ( λ x ) where A and λ are

constants. The ground state energy eigenvalue of this system is,


2
λ2 2
λ2
(a) E = (b) E = −
m m
2
λ2 2
λ2
(c) E = − (d) E =
2m 2m
Ans: (d)

Solution: ∵ψ ( x ) = A sec h ( λ x ) ⇒ = − Aλ sec h ( λ x ) tanh ( λ x )
dx
d 2ψ
⇒ 2
= − Aλ ⎡⎣ − sec h ( λ x ) tan 2 h ( λ x ) λ + λ sec h ( λ x ) sec 2 h ( λ x ) ⎤⎦
dx

= − Aλ 2 ⎡⎣sec h ( λ x ) ⎡⎣ − tan 2 h ( λ x ) + sec2 h ( λ x ) ⎤⎦ ⎤⎦

= − Aλ 2 ⎡⎣sec h ( λ x ) ⎡⎣sec 2 h ( λ x ) − tan 2 h ( λ x ) ⎤⎦ ⎤⎦

= − Aλ 2 ⎡sec h ( λ x ) ⎣⎡sec 2 h ( λ x ) − ⎡⎣1 − sec 2 h ( λ x ) ⎤⎦ ⎦⎤ ⎤


⎣ ⎦
∵ tan 2 h ( λ x ) = 1 − sec 2 h ( λ x )

= − Aλ 2 ⎡⎣sec h ( λ x ) ⎡⎣sec 2 h ( λ x ) − 1 + sec 2 h ( λ x ) ⎤⎦ ⎤⎦

d 2ψ
⇒ 2 = − Aλ 2 ⎡⎣ 2sec3 h ( λ x ) − sec h ( λ x ) ⎤⎦
dx

                                                                                
Head office  Branch office 
 
fiziks, H.No. 40 D, G.F, Jia Sarai,  Anand Institute of Mathematics, 
 
Near IIT, Hauz Khas, New Delhi‐16  28‐B/6, Jia Sarai, Near IIT 
 
Phone: 011‐26865455/+91‐9871145498 Hauz Khas, New Delhi‐16 
                                                   
                                             Website: www.physicsbyfiziks.com                                                                                          
                                                           Email: fiziks.physics@gmail.com                                                                   8 
fiziks
Institute for NET/JRF, GATE, IIT‐JAM, JEST, TIFR and GRE in PHYSICAL SCIENCES 
 

2 2
dψ2
Now put the value 2
in equation − + V ( x )ψ ( x ) = Eψ ( x )
dx 2m dx 2
2
− λ 2 A ⎡⎣ 2sec3 h ( λ x ) − sec h ( λ x ) ⎤⎦ + V ( x ) A sec h ( λ x ) = EA sec h ( λ x )
2m
∵V ( x ) → 0 as x → ∞
2 2
λ2
⇒+ λ 2 A sec h ( λ x ) − 2 A sec3 h ( λ x ) = EA sec h ( λ x )
2m 2m
Now we have to do approximation i.e. sec3 h ( λ x ) dacays very fastly as x → ∞ so second

term
2
λ2 2
λ2 2
λ2
2 A sec3 h ( λ x ) = 0 . Thus A sec h ( λ x ) = EA sec h ( λ x ) ⇒ E =
2m 2m 2m
1
Q16. Consider a spin − particle characterized by the Hamiltonian H = ωS z . Under a
2
perturbation H ′ = gS x , the second order correction to the ground state energy is given by,

g2 g2 g2 g2
(a) − (b) (c) − (d)
4ω 4ω 2ω 2ω
Ans: (a)
⎡1 0 ⎤
Solution: ∵ H = ω sz and sz =
2 ⎢⎣0 −1⎥⎦

ω ⎛1 0⎞ g ⎛1 0 ⎞
⇒H = ⎜ ⎟ and H ′ = gsx = ⎜ ⎟
2 ⎝ 0 −1⎠ 2 ⎝ 0 −1 ⎠

ω ⎛0⎞
Ground state energy is − with eigenvector φ1 = ⎜ ⎟
2 ⎝1⎠
ω ⎛1⎞
and first excited state energy is with eigenvector φ2 = ⎜ ⎟
2 ⎝ 0⎠
2 2
φm H ′ φ1 φm H ′ φ1
Second order correction in ground state E = ∑ 2
=
2
E10 − Em0 ω ω
m ≠1
− −
2 2

                                                                                
Head office  Branch office 
 
fiziks, H.No. 40 D, G.F, Jia Sarai,  Anand Institute of Mathematics, 
 
Near IIT, Hauz Khas, New Delhi‐16  28‐B/6, Jia Sarai, Near IIT 
 
Phone: 011‐26865455/+91‐9871145498 Hauz Khas, New Delhi‐16 
                                                   
                                             Website: www.physicsbyfiziks.com                                                                                          
                                                           Email: fiziks.physics@gmail.com                                                                   9 
fiziks
Institute for NET/JRF, GATE, IIT‐JAM, JEST, TIFR and GRE in PHYSICAL SCIENCES 
 
2
⎛ 0 1⎞⎛ 0⎞
(1 0 ) ⎜ ⎟⎜ ⎟
g2 ⎝1 0⎠⎝1⎠
2
g2 2 g2
⇒E = 2
=− =−
4
2
2ω 4ω 4ω

2
Q17. Given that υ/ 1 and υ/ 2 are eigenstates of a Hamiltonian with eigenvalues E1 and E 2
respectively, what is the energy uncertainty in the state (υ/ 1 + υ/ 2 ) ?
1
(a) − E1 E 2 (b) E1 − E 2
2
1 1
(c) (E1 + E 2 ) (d) E 2 − E1
2 2
Ans: (b)

1 2 1 2 ( E1 + E2 )
2 2
1 1
Solution: E 2
= E1 + E2 = and E = E1 + E2
2 2 2 2 2

∵ ΔE = E2 − E
2
=
(E 1
2
+ E22 ) − 1 (E + E ) 2
=
2 E12 + 2 E22 − E12 − E22 − 2 E1 E2
1 2
2 4 4

E12 + E22 − 2 E1 E2 1
⇒ ΔE = = E1 − E2
4 2
kr 2
Q18. A particle moving under the influence of a potential V (r ) = has a wavefunction
2
υ/ (r , t ) . If the wavefunction changes to υ/ (α r , t ) , the ratio of the average final kinetic
energy to the initial kinetic energy will be,
1 1
(a) (b) α (c) (d) α 2
α 2
α
Ans: (c)
⎛ 2
⎞ 2
( )

Solution: For ψ ( r , t ) the average kinetic energy T = ∫ ψ * ( r, t ) ⎜ − ⎟ ∇ ψ r dr ∇ is
2 2
0
⎝ 2m ⎠

written in spherical polar coordinate, which is dimension of ( length )


−2

For wave function ψ (α r , t )

                                                                                
Head office  Branch office 
 
fiziks, H.No. 40 D, G.F, Jia Sarai,  Anand Institute of Mathematics, 
 
Near IIT, Hauz Khas, New Delhi‐16  28‐B/6, Jia Sarai, Near IIT 
 
Phone: 011‐26865455/+91‐9871145498 Hauz Khas, New Delhi‐16 
                                                   
                                             Website: www.physicsbyfiziks.com                                                                                          
                                                           Email: fiziks.physics@gmail.com                                                                   10 
fiziks
Institute for NET/JRF, GATE, IIT‐JAM, JEST, TIFR and GRE in PHYSICAL SCIENCES 
 
⎛ 2
⎞ 2
⎟ ( ∇ ψ (α r , t ) ) r dr

Tα = ∫ ψ * (α r , t ) ⎜ − 2
0
⎝ 2m ⎠
r′ dr ′
Put α r = r ′ or r = ⇒ dr = and ∇ 2r = α 2 ∇ r2
α α
α2 ∞ ⎛ ⎞ 2 2
1 ∞ ⎛ ⎞ 2
2
Tα = ∫ ψ * ( r ′, t ) ⎜ − ⎟ ∇ ψ ( r ′, t ) r ′ dr ′ =
2
∫ ψ * ( r ′, t ) ⎜ − ⎟ ∇ ψ ( r ′, t ) r ′ dr ′
2

α3 0
⎝ 2m ⎠ α 0
⎝ 2 m ⎠
T Tα 1
⇒ Tα = ⇒ =
α T α

Q19. If a Hamiltonian H is given as H = 0 0 − 1 1 + i ( 0 1 − 1 0 ) , where 0 and 1 are

orthonormal states, the eigenvalues of H are


(a) ± 1 (b) ± i (c) ± 2 (d) ± i 2
Ans: (c)
Solution: H = 0 0 − 1 1 + i ( 0 1 − 1 0 )

H 0 = 0 −i 1 and H 1 = − 1 +i 0

0 H 0 0 H 1 ⎛1 i ⎞
The matrix representation of H is =⎜ ⎟
1H 0 1H 1 ⎝ −i −1⎠

⎛1 − λ i ⎞
Eigenvalue of H ⎜
−1 − λ ⎠
2
(
⎟ = 0 ⇒ − 1 − λ − 1 = −0 ⇒ λ = ± 2 )
⎝ −i

JEST-2014
Q20. Suppose a spin 1 / 2 particle is in the state
1 ⎡1 + i ⎤
υ/ = ⎢ ⎥
6⎣ 2 ⎦
If S x ( x component of the spin angular momentum operator) is measured what is the

probability of getting + / 2 ?
(a) 1 / 3 (b) 2 / 3 (c) 5 / 6 (d) 1 / 6
Ans.: (c)
                                                                                
Head office  Branch office 
 
fiziks, H.No. 40 D, G.F, Jia Sarai,  Anand Institute of Mathematics, 
 
Near IIT, Hauz Khas, New Delhi‐16  28‐B/6, Jia Sarai, Near IIT 
 
Phone: 011‐26865455/+91‐9871145498 Hauz Khas, New Delhi‐16 
                                                   
                                             Website: www.physicsbyfiziks.com                                                                                          
                                                           Email: fiziks.physics@gmail.com                                                                   11 
fiziks
Institute for NET/JRF, GATE, IIT‐JAM, JEST, TIFR and GRE in PHYSICAL SCIENCES 
 
⎡0 1 ⎤ 1 ⎛1⎞
Solution: S x = ⎢ ⎥ with eigenvalues ± and eigenvector corresponding to is ⎜ ⎟
2 ⎣1 0 ⎦ 2 2 2 ⎝1⎠

Now probability getting +


2
2
1 1 ⎡1 + i ⎤
φψ
⋅ [1 1] ⎢ 2 ⎥ 1
1+ i + 2
2

⎛ ⎞ 2 6 ⎣ ⎦ 5
p⎜ ⎟ = = = 12 =
⎝2⎠ ψ ψ 1 ⎡1 + i ⎤ 1 6
[1 − i 2] ⎢ 2 ⎥ 6×
6
6 ⎣ ⎦
Q21. The Hamiltonian operator for a two-state system is given by
H = α ( 1 1 − 2 2 + 1 2 + 2 1 ),

where α is a positive number with the dimension of energy. The energy eigenstates
corresponding to the larger and smaller eigenvalues respectively are:
(a) 1 − ( 2 + 1) 2 , 1 + ( 2 − 1) 2
(b) 1 + ( 2 − 1) 2 , 1 − ( 2 + 1) 2

(c) 1 + ( 2 − 1) 2 , ( 2 + 1) 1 − 2

(d) 1 − ( 2 + 1) 2 , ( 2 − 1) 1 + 2

Ans.: (b)
Solution: H = α ( 1 1 − 2 2 + 1 2 + 2 1 ) ⇒ H 1 = α ( 1 + 2 ) , H 2 = α ( 1 − 2 )
Lets check for option (b): 1 + ( )
2 −1 2 , 1 − ( 2 + 1) 2
Now H ψ = α ψ ⇒ H ⎡1 +( 2 − 1) 2 ⎤ = H 1 + H ( 2 + 1) 2
⎣ ⎦

H ⎡1 +
⎣ ( )
2 −1 2 ⎤ ⇒ H ( 1 ) +
⎦ ( )
2 −1 H 2 ⇒ α ( 1 + 2 ) + ( )
2 −1 α ( 1 − 2 )
⇒ α ⎡⎣1 + 2 − 1⎤⎦ 1 + α ⎡1 −
⎣ ( )
2 −1 ⎤ 2 ⇒ α 2 1 + α 2 − 2 2

( )
[ ( 2 − 1) 2 ]
⇒α 2 1 +

Now H ( 1 − 2 + 1) 2 ⇒ H [ 1 − ( ) ]
2 +1 2 ⇒ H 1 − H ( 2 +1 2 )
                                                                                
Head office  Branch office 
 
fiziks, H.No. 40 D, G.F, Jia Sarai,  Anand Institute of Mathematics, 
 
Near IIT, Hauz Khas, New Delhi‐16  28‐B/6, Jia Sarai, Near IIT 
 
Phone: 011‐26865455/+91‐9871145498 Hauz Khas, New Delhi‐16 
                                                   
                                             Website: www.physicsbyfiziks.com                                                                                          
                                                           Email: fiziks.physics@gmail.com                                                                   12 
fiziks
Institute for NET/JRF, GATE, IIT‐JAM, JEST, TIFR and GRE in PHYSICAL SCIENCES 
 
( )
⇒ α ( 1 + 2 ) − α ⎡ 2 + 1 ( 1 − 2 )⎤ ⇒ α 1 − 2 − 1 1 + α 1 + 2 + 1 2
⎣ ⎦ ( ) ( )
(
⇒ − 2α 1 + 2 + 2 α 2 ⇒ −α 2 1 − 1 + 2 2) [ ( ) ]
Q22. Consider an eigenstate of L2 and Lz operator denoted by l, m . Let A = nˆ ⋅ L denote an

operator, where n̂ is a unit vector parametrized in terms of two angles


as (n x , n y n z ) = (sin θ cos φ , sin φ , cos θ ) . The width Δ A in l, m state is:

l (l + 1) − m 2 l (l + 1) − m 2
(a) cos θ (b) sin θ
2 2

(c) l (l + 1) − m 2 sin θ (d) l (l + 1) − m 2 cos θ


Ans.: (b)
x y z
Solution: A = nˆ ⋅ L ⇒ A = Lx ⋅ + Ly ⋅ + Lz ⋅
r r r
r sin θ cos φ r sin θ sin φ r cos θ
⇒ A = Lx ⋅ + Ly ⋅ + Lz ⋅
r r r
⇒ A = Lx sin θ cos φ + Ly sin θ ⋅ sin φ + Lz cos θ

2
Now ΔA = A2 − A

A = Lx sin θ cos φ + Ly sin θ sin φ + Lz cos θ

A = ( m ) cos θ ∵ Lx = 0, Ly = 0

A 2 = L2x sin 2 θ cos 2 φ + L2y sin 2 θ sin 2 φ + L2z cos 2 θ

⎡l ( l + 1) − m 2 ⎦⎤
⎣ ⎛ l ( l + 1) − m 2 ⎞
⇒ A 2
= 2
sin θ cos φ + ⎜
2 2
⎟⎟
2
sin 2 θ sin 2 φ + m 2 2
cos 2 θ
2 ⎜ 2
⎝ ⎠
⎡⎣l ( l + 1) − m 2 ⎤⎦
⇒ A 2
= 2
sin 2 θ ⎡⎣sin 2 φ + cos 2 φ ⎤⎦ + m 2 2
cos 2 θ
2
⎡⎣l ( l + 1) − m 2 ⎤⎦
⇒ A 2
= 2
sin 2 θ + m 2 2
cos 2 θ
2

                                                                                
Head office  Branch office 
 
fiziks, H.No. 40 D, G.F, Jia Sarai,  Anand Institute of Mathematics, 
 
Near IIT, Hauz Khas, New Delhi‐16  28‐B/6, Jia Sarai, Near IIT 
 
Phone: 011‐26865455/+91‐9871145498 Hauz Khas, New Delhi‐16 
                                                   
                                             Website: www.physicsbyfiziks.com                                                                                          
                                                           Email: fiziks.physics@gmail.com                                                                   13 
fiziks
Institute for NET/JRF, GATE, IIT‐JAM, JEST, TIFR and GRE in PHYSICAL SCIENCES 
 

ΔA = A 2
− A
2
=
( l ( l + 1) − m ) 2
2
sin 2 θ + m 2 2
cos 2 θ − m 2 2
cos 2 θ
2

⎡⎣l ( l + 1) − m 2 ⎤⎦
ΔA = sin θ
2
Q23. Consider a three-state system with energies E, E and E − 3g (where g is a constant) and
respective eigenstates
⎛1⎞ ⎛ 1 ⎞ ⎛ 1⎞
1 ⎜ ⎟ 1 ⎜ ⎟ 1 ⎜ ⎟
υ/ 1 = ⎜ − 1⎟ υ/ 2 = ⎜ 1 ⎟ υ/ 3 = ⎜ 1⎟
2⎜ ⎟ 6⎜ ⎟ 3⎜ ⎟
⎝0⎠ ⎝ − 2⎠ ⎝ 1⎠

⎛1⎞
⎜ ⎟
If the system is initially (at t = 0 ), in state υ/ i = ⎜ 0 ⎟
⎜0⎟
⎝ ⎠

⎛0⎞
⎜ ⎟
what is the probability that at a later time t system will be in state υ/ f = ⎜0⎟
⎜1⎟
⎝ ⎠
4 2 ⎛ 3 gt ⎞
(a) 0 (b) sin ⎜ ⎟
9 ⎝2 ⎠
4 ⎛ 3 gt ⎞ 4 2 ⎛ E − 3gt ⎞
(c) cos 2 ⎜ ⎟ (d) sin ⎜ ⎟
9 ⎝2 ⎠ 9 ⎝ 2 ⎠
Ans.: (b)
Q24. A hydrogen atom in its ground state is collided with an electron of kinetic energy 13.377
eV. The maximum factor by which the radius of the atom would increase is
(a) 7 (b) 8 (c) 49 (d) 64
Ans.: (c)
−13.6
Solution: En = eV
n2
⇒ E1 = −13.6 eV , E2 = −3.4 eV , E3 = 1.5 eV , E4 = 0.85 eV , E5 = 0.54 eV

E6 = 0.3777 eV , E7 = 0.2775 eV

                                                                                
Head office  Branch office 
 
fiziks, H.No. 40 D, G.F, Jia Sarai,  Anand Institute of Mathematics, 
 
Near IIT, Hauz Khas, New Delhi‐16  28‐B/6, Jia Sarai, Near IIT 
 
Phone: 011‐26865455/+91‐9871145498 Hauz Khas, New Delhi‐16 
                                                   
                                             Website: www.physicsbyfiziks.com                                                                                          
                                                           Email: fiziks.physics@gmail.com                                                                   14 
fiziks
Institute for NET/JRF, GATE, IIT‐JAM, JEST, TIFR and GRE in PHYSICAL SCIENCES 
 
Since Electron have kinetic energy 13.377 eV = −13.6 + 0.2775 eV ⇒ n = 7

∵ rn = a0 n 2 ⇒ rn = 49a0
Q25. The lowest quantum mechanical energy of a particle confined in a one-dimensional box
of size L is 2 eV. The energy of the quantum mechanical ground state for a system of
1
three non-interacting spin particles is
2
(a) 6 eV (b) 10 eV (c) 12 eV (d) 16 eV
Ans.: (c)
π2 2
Solution: E1 = = 2eV , E2 = 4 E1 = 8 eV
2ml 2
1
∵ spin is
2
1
then degeneracy 2S + 1 = 2 × + 1 = 2
2
⇒ ground state 2 × 2 eV + 1 × 8 eV = 12 eV
Q26. A ball bounces off earth. You are asked to solve this quantum mechanically assuming the
earth is an infinitely hard sphere. Consider surface of earth as the origin implying
V (0 ) =∝ and a linear potential elsewhere (i.e. V ( x ) = −mgx for x > 0 ). Which of the
following wave functions is physically admissible for this problem (with k > 0 ):
2 2
(a) υ/ = e − kx / x (b) υ/ = xe − kx (c) υ/ = − Axe kx (d) υ/ = Ae − kx
Ans.: (b)
2
Solution: ψ = xe − kx
For given potential, at x = 0, and x = ∞ wave function must vanish.
Q27. The operator A and B share all the eigenstates. Then the least possible value of the
product of uncertainties ΔAΔB is
(a) (b) 0 (c) /2 (d) Determinant (AB)
Ans.: (b)

                                                                                
Head office  Branch office 
 
fiziks, H.No. 40 D, G.F, Jia Sarai,  Anand Institute of Mathematics, 
 
Near IIT, Hauz Khas, New Delhi‐16  28‐B/6, Jia Sarai, Near IIT 
 
Phone: 011‐26865455/+91‐9871145498 Hauz Khas, New Delhi‐16 
                                                   
                                             Website: www.physicsbyfiziks.com                                                                                          
                                                           Email: fiziks.physics@gmail.com                                                                   15 
fiziks
Institute for NET/JRF, GATE, IIT‐JAM, JEST, TIFR and GRE in PHYSICAL SCIENCES 
 
Solution: ΔA ⋅ ΔB ≥
[AB]
2
ΔA ⋅ ΔB ≥ 0 ∵ A and B have share their eigen values
so [AB ] = 0
Q28. Consider a square well of depth − V0 and width a with V0 a fixed. Let V0 → ∞ and

a → 0 . This potential well has


(a) No bound states (b) 1 bound state
(c) 2 bound states (d) Infinitely many bound states
Ans.: (b)
Solution: It forms delta potential so it has only one bound state.

                                                                                
Head office  Branch office 
 
fiziks, H.No. 40 D, G.F, Jia Sarai,  Anand Institute of Mathematics, 
 
Near IIT, Hauz Khas, New Delhi‐16  28‐B/6, Jia Sarai, Near IIT 
 
Phone: 011‐26865455/+91‐9871145498 Hauz Khas, New Delhi‐16 
                                                   
                                             Website: www.physicsbyfiziks.com                                                                                          
                                                           Email: fiziks.physics@gmail.com                                                                   16 
fiziks
Institute for NET/JRF, GATE, IIT‐JAM, JEST, TIFR and GRE in PHYSICAL SCIENCES 
 
JEST-2013
Q29. A particle of mass m is contained in a one-dimensional infinite well extending from
L L
x=− to x = . The particle is in its ground state given by ϕ 0 ( x ) = 2 / L cos(πx / L ) .
2 2
The walls of the box are moved suddenly to form a box extending from x = − L to x = L .
what is the probability that the particle will be in the ground state after this sudden
expansion?
(a) (8 / 3π ) (c) (16 / 3π ) (d) (4 / 3π )
2 2 2
(b) 0
Ans.: (a)
2 2 πx 2 πx
Solution: Probability φ 0 φ1 ,φ0 = cos , φ1 cos
L L 2L 2L
Since the wall of box are moved suddenly then
2 2
L/2 2 1 cos π x cos π x 2 1 L / 2 2 cos π x cos π x
Probability = ∫−L / 2 L

L L

2L
dx =
L 2 ∫− L / 2 L

2L
dx

2 L/2 2
2 1 L / 2 ⎡ ⎛ 3π x ⎞ ⎛ π x ⎞⎤ 2 1 ⎡ 2L 3π x 2 L π x⎤
⇒ ⋅ ∫ ⎢cos ⎜ ⎟ + cos ⎜ ⎟ ⎥ dx ⇒ ⋅ ⎢ sin + sin
L 2 − L / 2
⎣ ⎝ 2L ⎠ ⎝ 2L ⎠⎦ L 2 ⎣ 3π 2L π 2 L ⎦⎥ − L / 2
2
2 1 ⎡ 2 L ⎛ 3π 3π ⎞ 2L ⎛ π π ⎞⎤
⇒ ⋅ ⎢ ⎜ sin + sin ⎟+ ⎜ sin + sin ⎟ ⎥
L 2 ⎣ 3π ⎝ 4 4 ⎠ π ⎝ 4 4 ⎠⎦
2 2
2 2 8
⇒ + =
3π π 3π
Q30. A quantum mechanical particle in a harmonic oscillator potential has the initial wave
function ψ/ 0 ( x ) + ψ/ 1 ( x ), where ψ/ 0 and ψ/ 1 are the real wavefunctions in the ground and
first excited state of the harmonic oscillator Hamiltonian. For convenience we take
m= = ω = 1 for the oscillator. What is the probability density of finding the particle at
x at time t = π ?
(a) (ψ/ 1 ( x ) − ψ/ 0 ( x )) (b) (ψ 1 ( x )) − (ψ/ 0 ( x ))
2 2 2

(c) (ψ/ 1 ( x ) + ψ/ 0 ( x )) (d) (ψ/ 1 ( x )) + (ψ/ 0 ( x ))


2 2 2

                                                                                
Head office  Branch office 
 
fiziks, H.No. 40 D, G.F, Jia Sarai,  Anand Institute of Mathematics, 
 
Near IIT, Hauz Khas, New Delhi‐16  28‐B/6, Jia Sarai, Near IIT 
 
Phone: 011‐26865455/+91‐9871145498 Hauz Khas, New Delhi‐16 
                                                   
                                             Website: www.physicsbyfiziks.com                                                                                          
                                                           Email: fiziks.physics@gmail.com                                                                   17 
fiziks
Institute for NET/JRF, GATE, IIT‐JAM, JEST, TIFR and GRE in PHYSICAL SCIENCES 
 
Ans.: (a)
Solution: ψ (x ) = ψ 0 ( x ) + ψ 1 ( x )

E0t E1t
ψ ( x, t ) = ψ 0 ( x ) e − i +ψ 1 ( x ) e−i

Now probability density at time t


t
ψ ( x, t ) = ψ * ( x, t )ψ ( x, t ) = ψ 0 ( x ) + ψ 1 ( x ) + 2 Reψ 0* ( x )ψ 1 ( x ) cos ( E1 − E0 )
2 2 2

putting t = π

ψ ( x, t ) = ψ 0 ( x ) + ψ 1 ( x ) + 2 Reψ 0* ( x )ψ 1 ( x ) cos π
2 2 2
∵ E1 − E0 = ω = 1

ψ ( x, t ) = ψ 0 ( x ) + ψ 1 ( x ) − 2 Reψ 0* ( x )ψ 1 ( x ) = ⎡⎣ψ 1 ( x ) −ψ 0 ( x ) ⎤⎦
2 2 2 2

Q31. If J x , J y and J z are angular momentum operators, the eigenvalues of the operator

(J x + jy )
are:

(a) real and discrete with rational spacing


(b) real and discrete with irrational spacing
(c) real and continuous
(d) not all real
Ans.: (b)
1 i ⎡0 1 ⎤ ⎡0 0⎤
Solution: J x = ( J+ + J− ) , J y = ( J− − J+ ) ⇒ J+ = ⎢ ⎥ , J − = ⎢1 0 ⎥
2 2 ⎣0 0 ⎦ ⎣ ⎦
⎡0 1 ⎤ i ⎡0 −1⎤ J + J y 1 ⎡ 0 1− i⎤
Jx = ⎢ ⎥ , Jy = ⎢ ⎥ ⇒ x = ⎢
2 ⎣1 0 ⎦ 2 ⎣1 0 ⎦ 2 ⎣1 + i 0 ⎥⎦

1 ⎛ − λ 1− i⎞
eigen value ⎜ ⎟ ⇒ λ2 − 2 = 0 ⇒ λ = ± 2
2 ⎜⎝1 + i − λ ⎟⎠

                                                                                
Head office  Branch office 
 
fiziks, H.No. 40 D, G.F, Jia Sarai,  Anand Institute of Mathematics, 
 
Near IIT, Hauz Khas, New Delhi‐16  28‐B/6, Jia Sarai, Near IIT 
 
Phone: 011‐26865455/+91‐9871145498 Hauz Khas, New Delhi‐16 
                                                   
                                             Website: www.physicsbyfiziks.com                                                                                          
                                                           Email: fiziks.physics@gmail.com                                                                   18 
fiziks
Institute for NET/JRF, GATE, IIT‐JAM, JEST, TIFR and GRE in PHYSICAL SCIENCES 
 
Q32. A simple model of a helium-like atom with electron-electron interaction is replaced by
Hooke’s law force is described by Hamiltonian
− 2 2 λ
2m
(∇1 + ∇ 22 ) + mω 2 (r12 + r22 ) −
1
2 4
2
mω 2 r1 − r2 .

What is the exact ground state energy?

(a) E =
3
2
(
ω 1+ 1+ λ ) (b) E =
3
2
ω 1+ λ ( )
(c) E =
3
2
ω 1− λ (d) E =
3
2
(
ω 1+ 1− λ )
Ans.: (b)
⎛1 / 2 ⎞
Q33. Consider the state ⎜⎜1 / 2 ⎟⎟ corresponding to the angular momentum l = 1 in the L z basis
⎝1 / 2 ⎠
of states with m = +1, 0, − 1 . If L2z is measured in this state yielding a result 1, what is the
state after the measurement?

⎛1⎞ ⎛ 1/ 3 ⎞ ⎛ 0⎞ ⎛1 / 2 ⎞
⎜ ⎟ ⎜ ⎟ ⎜ ⎟ ⎜ ⎟
(a) ⎜ 0 ⎟ (b) ⎜ 0 ⎟ (c) ⎜ 0 ⎟ (d) ⎜ 0 ⎟
⎜ 0⎟ ⎜⎜ ⎟⎟ ⎜1⎟ ⎜⎜ ⎟⎟
⎝ ⎠ ⎝ 2/3⎠ ⎝ ⎠ ⎝1 / 2 ⎠
Ans.: (d)
⎛1 0 0 ⎞ ⎛1 0 0⎞ ⎛1⎞ ⎛0⎞ ⎛0⎞
⎜ ⎟ ⎜ ⎟ ⎜ ⎟ ⎜ ⎟ ⎜ ⎟
Solution: L z = ⎜ 0 0 0 ⎟ , L z = ⎜ 0 0 0 ⎟ , eigenvector
2
⎜ 0⎟ , ⎜1⎟, ⎜0⎟
⎜ 0 0 − 1⎟ ⎜0 0 1⎟ ⎜ 0⎟ ⎜0⎟ ⎜1⎟
⎝ ⎠ ⎝ ⎠ ⎝ ⎠ ⎝ ⎠ ⎝ ⎠
Corresponding eigenvalue 1, 0, 1

⎛1⎞ ⎛1⎞
⎜ ⎟ 1 ⎜ ⎟
Now state after measurement yielding 1⇒ φ1 + φ 3 = ⎜0⎟ = ⎜0⎟
⎜1⎟ 2⎜ ⎟
⎝ ⎠ ⎝1⎠

                                                                                
Head office  Branch office 
 
fiziks, H.No. 40 D, G.F, Jia Sarai,  Anand Institute of Mathematics, 
 
Near IIT, Hauz Khas, New Delhi‐16  28‐B/6, Jia Sarai, Near IIT 
 
Phone: 011‐26865455/+91‐9871145498 Hauz Khas, New Delhi‐16 
                                                   
                                             Website: www.physicsbyfiziks.com                                                                                          
                                                           Email: fiziks.physics@gmail.com                                                                   19 
fiziks
Institute for NET/JRF, GATE, IIT‐JAM, JEST, TIFR and GRE in PHYSICAL SCIENCES 
 
Q34. What are the eigenvalues of the operator H = σ ⋅ a , where σ are the three Pauli matrices
and a is a vector?
(a) a x + a y and a z (b) a x + a z ± ia y (c) ± (a x + a y + a z ) (d) ± a

Ans.: (d)
Solution: H = σ ⋅ a = (σ x .a x + σ y .a y + σ z .a z )

⎛0 1⎞ ⎛ 0 −i ⎞ ⎛1 0 ⎞
=⎜ ⎟ ax + ⎜ ⎟ ay + ⎜ ⎟ az
⎝1 0⎠ ⎝i 0 ⎠ ⎝ 0 −1⎠
⎛ az (a − ia y )⎞ ⎛ (a z − λ ) (a x − ia y ) ⎞
⇒ ⎜⎜ ⎟⇒⎜ ⎟
x

⎝ (a x + ia y ) − a z ⎟⎠ ⎜⎝ (a x + ia y ) − (a z + λ )⎟⎠

⇒ −(a z − λ )(a z + λ ) − (a x − ia y )(a x + ia y )

− az2 + λ 2 − ax2 − a y2 = 0

λ 2 = ax2 + a y2 + az2

⇒λ =± a

⎛−∂⎞
Q35. The hermitian conjugate of the operator ⎜ ⎟ is
⎝ ∂x ⎠
∂ ∂ ∂ ∂
(a) (b) − (c) i (d) − i
∂x ∂x ∂x ∂x
Ans.: (a)

⎞ ⎛ −∂ψ ( x ) ⎞
† *
⎛ * ∂
Solution: ⇒ ⎜ψ ( x ) − ψ ( x ) ⎟ = ⎜ ψ ( x) ⎟
⎝ ∂x ⎠ ⎜⎝ ∂x ⎟

∞ ⎡ ∂ ⎤ ∞ ∞ ∂ψ ( x ) *

⇒ ∫ ψ ( x ) ⎢ − ψ ( x ) ⎥ dx −ψ * ( x )ψ ( x ) − ∫ −
*
ψ ( x ) dx
−∞
⎣ ∂x ⎦ −∞ −∞ ∂x

∞ ∂ψ * ( x )
⇒∫ ψ ( x ) dx
−∞ ∂x

                                                                                
Head office  Branch office 
 
fiziks, H.No. 40 D, G.F, Jia Sarai,  Anand Institute of Mathematics, 
 
Near IIT, Hauz Khas, New Delhi‐16  28‐B/6, Jia Sarai, Near IIT 
 
Phone: 011‐26865455/+91‐9871145498 Hauz Khas, New Delhi‐16 
                                                   
                                             Website: www.physicsbyfiziks.com                                                                                          
                                                           Email: fiziks.physics@gmail.com                                                                   20 
fiziks
Institute for NET/JRF, GATE, IIT‐JAM, JEST, TIFR and GRE in PHYSICAL SCIENCES 
 
Q36. If the expectation value of the momentum is p for the wavefunction ψ ( x ) , then the

expectation value of momentum for the wavefunction ei k x / ψ ( x ) is

(a) k (b) p − k (c) p + k (d) p

Ans.: (c)
∞ ⎛ ∂ ⎞
Solution: ∫ ψ * ( x ) ⎜ −i ⎟ψ ( x ) dx = p
−∞
⎝ ∂x ⎠
Now
∞ −
ikx
⎛ ∂ ⎞ ikx ∞
− ikx
⎡ ikx
∂ ik ikx ⎤
∫ e ψ * ( x ) ⎜ −i ⎟ e ψ ( x ) dx ⇒ ∫ e ψ * ( x )( −i ) ⎢e ψ ( x ) + e ψ ( x )⎥
−∞
⎝ ∂x ⎠ −∞
⎣ ∂x ⎦
∞ −
ikx
⎛ ∂ ⎞
ikx
∞ ik − ikx
⇒∫ e ψ * ( x ) ⎜ −i ψ ( x ) ⎟ e + ∫ −i . e ψ * ( x )ψ ( x ) dx
−∞
⎝ ∂x ⎠ −∞

∞ ⎡ ∂ ⎤ ∞
⇒ ∫ ψ * ( x ) ⎢ −i ψ ( x )⎥ + k ∫ ψ * ( x )ψ ( x ) ⇒ P + K
−∞
⎣ ∂x ⎦ −∞

Q37. Two electrons are confined in a one dimensional box of length L . The one-electron states

2 ⎛ nπ x ⎞
are given byψ n ( x ) = sin ⎜ ⎟ . What would be the ground state wave function
L ⎝ L ⎠
ψ ( x1 , x2 ) if both electrons are arranged to have the same spin state?

1 ⎡ 2 ⎛ πx1 ⎞ ⎛ 2πx 2 ⎞ 2 ⎛ 2πx1 ⎞ ⎛ πx 2 ⎞⎤


(a) υ/ ( x1 , x 2 ) = ⎢ sin ⎜ ⎟ sin ⎜ ⎟ + sin ⎜ ⎟ sin ⎜ ⎟⎥
2 ⎣L ⎝ L ⎠ ⎝ L ⎠ L ⎝ L ⎠ ⎝ L ⎠⎦

1 ⎡ 2 ⎛ πx1 ⎞ ⎛ 2πx 2 ⎞ 2 ⎛ 2πx1 ⎞ ⎛ πx 2 ⎞⎤


(b) υ/ ( x1 , x 2 ) = ⎢ sin ⎜ ⎟ sin ⎜ ⎟ − sin ⎜ ⎟ sin ⎜ ⎟⎥
2 ⎣L ⎝ L ⎠ ⎝ L ⎠ L ⎝ L ⎠ ⎝ L ⎠⎦

2 ⎛ πx1 ⎞ ⎛ 2πx 2 ⎞
(c) υ/ ( x1 , x 2 ) = sin ⎜ ⎟ sin ⎜ ⎟
L ⎝ L ⎠ ⎝ L ⎠

2 ⎛ 2πx1 ⎞ ⎛ πx 2 ⎞
(d) υ/ ( x1 , x 2 ) = sin ⎜ ⎟ sin ⎜ ⎟
L ⎝ L ⎠ ⎝ L ⎠
Ans.: (b)

                                                                                
Head office  Branch office 
 
fiziks, H.No. 40 D, G.F, Jia Sarai,  Anand Institute of Mathematics, 
 
Near IIT, Hauz Khas, New Delhi‐16  28‐B/6, Jia Sarai, Near IIT 
 
Phone: 011‐26865455/+91‐9871145498 Hauz Khas, New Delhi‐16 
                                                   
                                             Website: www.physicsbyfiziks.com                                                                                          
                                                           Email: fiziks.physics@gmail.com                                                                   21 
fiziks
Institute for NET/JRF, GATE, IIT‐JAM, JEST, TIFR and GRE in PHYSICAL SCIENCES 
 
1
Solution: Electrons are Fermions of spin and it wave functions are anti symmetric
2
Spin part is symmetric and space part will be anti symmetric (since total wave function is
anti symmetric)
Then

1 ⎡ 2 ⎛ πx1 ⎞ ⎛ 2πx 2 ⎞ 2 ⎛ 2πx1 ⎞ ⎛ πx 2 ⎞⎤


= ⎢ L sin ⎜ L ⎟ . sin ⎜ L ⎟ − sin ⎜ ⎟. sin ⎜ ⎟⎥
2⎣ ⎝ ⎠ ⎝ ⎠ L ⎝ L ⎠ ⎝ L ⎠⎦

Q38. The operator


⎛d ⎞⎛ d ⎞
⎜ − x ⎟⎜ + x ⎟
⎝ dx ⎠⎝ dx ⎠
is equivalent to
d2 d2
(a) 2
− x2 (b) 2
− x2 +1
dx dx
d2 d d2 d
(c) 2
− x x2 +1 (d) 2
− 2x − x 2
dx dx dx dx
Ans.: (b)
⎛d ⎞⎛ d ⎞ ⎛d ⎞⎡ d ⎤
Solution: ⇒ ⎜ − x ⎟⎜ + x ⎟ f ( x ) ⇒ ⎜ − x ⎟ ⎢ f ( x ) + xf ( x )⎥
⎝ dx ⎠⎝ dx ⎠ ⎝ dx ⎠ ⎣ dx ⎦
d ⎡d ⎤ d
⇒ ⎢ f (x ) + xf (x )⎥ − x f (x ) − x 2 f (x )
dx ⎣ dx ⎦ dx

d2 df ( x ) d
⇒ 2
f ( x) + f ( x) + x −x f ( x ) − x2 f ( x )
dx dx dx
d2 ⎛ d2 ⎞
⇒ 2
f ( x ) − x 2
f ( x ) + f ( x ) = ⎜⎜ 2 − x 2 + 1⎟⎟ f (x )
dx ⎝ dx ⎠

                                                                                
Head office  Branch office 
 
fiziks, H.No. 40 D, G.F, Jia Sarai,  Anand Institute of Mathematics, 
 
Near IIT, Hauz Khas, New Delhi‐16  28‐B/6, Jia Sarai, Near IIT 
 
Phone: 011‐26865455/+91‐9871145498 Hauz Khas, New Delhi‐16 
                                                   
                                             Website: www.physicsbyfiziks.com                                                                                          
                                                           Email: fiziks.physics@gmail.com                                                                   22 
fiziks
Institute for NET/JRF, GATE, IIT‐JAM, JEST, TIFR and GRE in PHYSICAL SCIENCES 
 
Q39. If a proton were ten times, the ground state energy of the electron in a hydrogen atom
would be
(a) less
(b) more
(c) the same
(d) less, more or equal depending on the electron mass
Ans.: (b)
−13.6 0.99995 me
Solution: En = × ⇒ −13.59932 ∵ μ = 0.99995me
n2 me

JEST-2012
Q40. The ground state (apart from normalization) of a particle of unit mass moving in a one-
( )
dimensional potential V(x) is exp − x 2 / 2 cosh 2 x . The potential V(x), in suitable ( )
units so that h = 1, is (up to an addiative constant.)
(a) π2/2 (b) π 2 / 2 − 2 x tanh 2 x ( )
(c) π 2 / 2 − 2 x tan 2 x ( ) (d) π 2 / 2 − 2 x coth ( 2 x )
Ans. : (b)
Q41. Consider the Bohr model of the hydrogen atom. If α is the fine-structure constant, the
velocity of the electron in its lowest orbit is
c c
(a) (b) or (1 − α ) c (c) α2 c (d) α c
1+α 1+α 2
Ans. : (d)
Solution: mvr = n
mv 2 1 ze 2
=
r 4π ∈0 r 2

1 ze 2
r=
4π ∈0 mr 2

                                                                                
Head office  Branch office 
 
fiziks, H.No. 40 D, G.F, Jia Sarai,  Anand Institute of Mathematics, 
 
Near IIT, Hauz Khas, New Delhi‐16  28‐B/6, Jia Sarai, Near IIT 
 
Phone: 011‐26865455/+91‐9871145498 Hauz Khas, New Delhi‐16 
                                                   
                                             Website: www.physicsbyfiziks.com                                                                                          
                                                           Email: fiziks.physics@gmail.com                                                                   23 
fiziks
Institute for NET/JRF, GATE, IIT‐JAM, JEST, TIFR and GRE in PHYSICAL SCIENCES 
 
2
1 ze
mv ⋅ =n
4π ∈0 mv 2

ze 2 e2
v= and fine structure constant α =
4π ∈0 n 4π ∈0 c

ze 2
v=
4π ∈0 n

ze 2 ze 2 c
for lowest orbit v = ⇒v=
4π ∈0 4π ∈0 c
v = αc
Q42. ( )
Define σ x = f + + f , and σ y = −i ( f + − f ), where the σ’ are Pauli spin matrices and f,f†

obey anticommutation relations { f , f } = 0, {f , f † } = 1 . Then σ2 is given by


(a) ,f† f-1 (b) 2f† f-1 (c) 2f† f + 1 (d) f† f
Ans. : (c)
Solution: σ xσ y = iσ z

iσ z = σ x σ y

−i +
1
σ z = σ xσ y =
i i
( f + f )( f + − f )
[( )
2
= − f + − f + f + ff + − f 2 ]
[
= − − f + f + (1 − f + . f ) ]
= −[1 − 2 f f ] +

= 2 f + f −1
Q43. Consider a system of two spin-1/2 particles with total spin S = s1 +s2, where s1 and s2 are
in terms of Pauli matrices σi. The spin triplet projection operator is
1 3 3 1
(a) + s1 ⋅ s 2 (b) − s1 ⋅ s 2 (c) + s1 ⋅ s 2 (d) − s1 ⋅ s 2
4 4 4 4
Ans. : (c)

                                                                                
Head office  Branch office 
 
fiziks, H.No. 40 D, G.F, Jia Sarai,  Anand Institute of Mathematics, 
 
Near IIT, Hauz Khas, New Delhi‐16  28‐B/6, Jia Sarai, Near IIT 
 
Phone: 011‐26865455/+91‐9871145498 Hauz Khas, New Delhi‐16 
                                                   
                                             Website: www.physicsbyfiziks.com                                                                                          
                                                           Email: fiziks.physics@gmail.com                                                                   24 
fiziks
Institute for NET/JRF, GATE, IIT‐JAM, JEST, TIFR and GRE in PHYSICAL SCIENCES 
 
Solution: ⇒ S = S1 + S 2 S = S1 + S 2 + 2 S 1 ⋅ S 2
2 2 2

⎛3 3 ⎞
S 2 = ⎜ + + 2.S1 ⋅ S2 ⎟ 2
∵ S = 0,1
⎝4 4 ⎠
⎡3 ⎤
S 2 = 2 ⎢ + S1 ⋅ S2 ⎥ 2
for Triplet projection operator
⎣4 ⎦
⎡3 ⎤
s ( s + 1) 2
= 2 ⎢ + S1 ⋅ S 2 ⎥ 2
S =1
⎣4 ⎦
⎛3 ⎞ 3
1(1 + 1) = 2⎜ + S1 ⋅ S 2 ⎟ ⇒ + S1 ⋅ S 2 = I
⎝4 ⎠ 4
Q44. Consider a spin-1/2 particle in the homogeneous magnetic field of magnitude B along z-

axis which is prepared initially in a state ψ =


1
2
(↑ + ↓ ) at time t = 0. at what time t
will the particles be in the state − ψ (μB is Bohr magneton)?

π 2π π
(a) t = (b) t = (c) t = (d) Never
μB B μB B 2μ B B
Ans.: (a)
1 ⎛1⎞
Solution: E = μ B ⋅ Bzˆ ψ = ⎜⎜ ⎟⎟
2 ⎝1⎠
iEt
1 ⎛1⎞ −
ψ ( x, t ) = ⎜⎜ ⎟⎟e ⇒ ψ 1 (x, t ) = − ψ
2 ⎝1⎠
− iμ B Bt
1 ⎛1⎞ − 1 ⎛1⎞
⎜ ⎟e = ⎜ ⎟
2 ⎜⎝1⎟⎠ 2 ⎜⎝1⎟⎠
− iμ B Bt
e = −1
⎛ μ Bt ⎞
cos⎜ B ⎟ = cos π
⎝ ⎠
μB B t π
=π ⇒t =
μB B

                                                                                
Head office  Branch office 
 
fiziks, H.No. 40 D, G.F, Jia Sarai,  Anand Institute of Mathematics, 
 
Near IIT, Hauz Khas, New Delhi‐16  28‐B/6, Jia Sarai, Near IIT 
 
Phone: 011‐26865455/+91‐9871145498 Hauz Khas, New Delhi‐16 
                                                   
                                             Website: www.physicsbyfiziks.com                                                                                          
                                                           Email: fiziks.physics@gmail.com                                                                   25 
fiziks
Institute for NET/JRF, GATE, IIT‐JAM, JEST, TIFR and GRE in PHYSICAL SCIENCES 
 
Q45. The ground state energy of 5 identical spin-1/2 particles which are subject to a one-
dimensional simple harmonic oscillator potential of frequency ωis
(a) (15/2)ћω (b) (13/2)ћω (c) (1/2)ћω (d) 5ћω
Ans. : (b)
1
Solution: ⇒ degeneracy 2s + 1 = 2 × +1 = 2
2
1 3 5 13
Eground = 2 × ω + 2 × ω + 1× ω = ω
2 2 2 2
Q46. The spatial part of a two-electron state is symmetric under exchange. If ↑ and ↓

represent the spin-up and spin-down states respectively of each particle, the spin-part of
the two-particle state is
(a) ↑ ↓ (b) ↓ ↑

(
(c) ↓ ↑ − ↑ ↓ / 2 ) (
(d) ↓ ↑ + ↑ ↓ / 2 )
Ans. : (c)
Solution: Since electron are Fermion and Fermions have antisymmetric wave function
∵ spatial part is symmetric then its spin part is antisymmetric to maintain antisymmtric
wave function

ψ (x ) =
1
2
(↓ ↑ − ↑ ↓ )
Q47. The wave function of a free particle in one dimension is given by
υ/ ( x ) = A sin x + B sin 3x . Then υ/ ( x ) is an eigenstate of
(a) the position operator (b) the Hamiltonian
(c) the momentum operator (d) the parity operator
Ans. : (d) ψ (− x ) = ψ ( x )
= −ψ ( x ) {parity (even and odd)
ψ (− x ) = A sin (− x ) + B sin (− 3x ) = −[A sin x + B sin 3x ]
ψ (− x ) = −ψ ( x ) ⇒ parity i.e. parity operator

                                                                                
Head office  Branch office 
 
fiziks, H.No. 40 D, G.F, Jia Sarai,  Anand Institute of Mathematics, 
 
Near IIT, Hauz Khas, New Delhi‐16  28‐B/6, Jia Sarai, Near IIT 
 
Phone: 011‐26865455/+91‐9871145498 Hauz Khas, New Delhi‐16 
                                                   
                                             Website: www.physicsbyfiziks.com                                                                                          
                                                           Email: fiziks.physics@gmail.com                                                                   26 
fiziks
Institute for NET/JRF, GATE, IIT‐JAM, JEST, TIFR and GRE in PHYSICAL SCIENCES 
 
Q48. The quantum state sin x ↑ + exp(iφ ) cos x ↓ , where ↑ ↓ = 0 and x, ф are,real, is

orthogonal to:
(a) sin x ↑ (b) cos x ↑ + exp(iφ )sin x ↓

(c) − cos x ↑ − exp(iφ )sin x ↓ (d) − exp(− iφ ) cos x ↑ + sin x ↓

Ans.: (d)
Solution: ↑ ↓ = 0 , ψ = sin x ↑ + exp(iφ ) cos x ↓

ψ ′ ψ = − exp ( iφ ) cos x sin x ↑ ↑ − exp ( iφ ) exp ( iφ ) cos x ↓ ↑ + sin 2 x ↓ ↑ + exp ( iφ ) cos x sin x ↓ ↓
= − exp(iφ ) cos x sin x + exp(iφ ) cos x sin x = 0
Q49. The binding energy of the hydrogen atom (electron bound to proton) is 13.6 eV. The
binding energy of peritoneum (electron bound to positron) is
(a) 13.6 / 2 eV (b) 13.6 / 1810 eV
(c) 13.6 × 1810 eV (d) 13.6 × 2 eV
Ans.: (a)
13.6 μ
Solution: E n′ = −
n 2 me
me ⋅ me m
μ= =
me + me 2
13.6 me
E n′ = − ⋅
n 2 2me
13.6 1
E n′ = − × 2
2 n
13.6
Thus binding energy will be eV
2

                                                                                
Head office  Branch office 
 
fiziks, H.No. 40 D, G.F, Jia Sarai,  Anand Institute of Mathematics, 
 
Near IIT, Hauz Khas, New Delhi‐16  28‐B/6, Jia Sarai, Near IIT 
 
Phone: 011‐26865455/+91‐9871145498 Hauz Khas, New Delhi‐16 
                                                   
                                             Website: www.physicsbyfiziks.com                                                                                          
                                                           Email: fiziks.physics@gmail.com                                                                   27 
fiziks
Institute for NET/JRF, GATE, IIT‐JAM, JEST, TIFR and GRE in PHYSICAL SCIENCES 
 
Thermodynamics & Statistical Mechanics
JEST-2016
Q1. An ideal gas with adiabatic exponent γ undergoes a process in which its pressure P is
related to its volume V by the relation P = P0 − αV , where P0 and α are positive
constants. The volume starts from being very close to zero and increases monotonically
P0
to . At what value of the volume during the process does the gas have maximum
α
entropy?
P0 γ P0 γ P0 P0
(a) (b) (c) (d)
α (1 + γ ) α (1 − γ ) α (1 + γ ) α (1 − γ )
Ans: (c)
nRdT
Solution: TdS = nCV dT + PdV ⇒ TdS = + PdV
γ −1
For maximum entropy dS = 0
For Ideal gas PV = nRT ⇒ PdV + VdP = nRdT
PdV + VdP pV γ VdP
TdS = nCV dT + PdV ⇒ TdS = + PdV ⇒ dS = PdV +
γ −1 nR γ −1 γ −1
dP = −α dV
PV γ αVdV dS γ nRP nR
dS = PdV − ⇒ = − αV
nR γ −1 γ −1 dV ( γ − 1) PV ( γ − 1) PV

dS
For maximum entropy = 0 ⇒ γ P − αV = 0 ⇒ γ ( P0 − αV ) = αV
dV
γ P0
V=
α (1 + γ )

                                                                                
Head office  Branch office 
 
fiziks, H.No. 40 D, G.F, Jia Sarai,  Anand Institute of Mathematics, 
 
Near IIT, Hauz Khas, New Delhi‐16  28‐B/6, Jia Sarai, Near IIT 
 
Phone: 011‐26865455/+91‐9871145498 Hauz Khas, New Delhi‐16 
                                                   
                                             Website: www.physicsbyfiziks.com                                                                                          
                                                           Email: fiziks.physics@gmail.com                                                                   1 
fiziks
Institute for NET/JRF, GATE, IIT‐JAM, JEST, TIFR and GRE in PHYSICAL SCIENCES 
 
Q2. A point charge q of mass m is released from rest at a distance d from an infinite
grounded conducting plane (ignore gravity). How long does it take for the charge to hit
the plane?

2π 3ε 0 md 3 2π 3ε 0 md
(a) (b)
q q

π 3ε 0 md 3 π 3ε 0 md
(c) (d)
q q
Ans: (a)
d 2x 1 q2 d 2x A q2 x
Solution: F = ma = m = − ⇒ = − where A = .
dt 2 4πε 0 4d 2 dt 2 x2 16π mε 0
P +q
1 d 2 d ⎛ A⎞

dv
dt
A dv
=− 2 v ⇒v
x dt
dv
dt
A dx
=− 2
x dt

2 dt
v = ⎜ ⎟
dt ⎝ x ⎠
( ) d

0
v2 A A ⎛1 1⎞
⇒ = + C at ⇒ x = d , v = 0 ⇒ C = − ⇒ v = 2 A ⎜ − ⎟ .
2 x d ⎝x d⎠ d

dx ⎛1 1⎞ xd
0 t −q
⇒− = 2A ⎜ − ⎟ ⇒ ∫ dx = − 2 A ∫ dt
dt ⎝x d⎠ d
d−x 0

Put x = d sin 2 θ ⇒ dx = x = 2d sin θ cos θ dθ


0
( d sin θ ) d 2d sin θ cos θ dθ = −
2


π

/2
d cos 2 θ
2 At

sin θ
0 0
⇒ − 2 At = ∫
π /2
d
cos θ
2d sin θ cos θ dθ = 2d 3/ 2 ∫ sin 2 θ dθ
π /2

0
(1 − cos 2θ ) dθ = d 3/ 2 ⎡θ − sin 2θ ⎤ 0 π
⇒ − 2 At = 2d 3/ 2 ∫ ⎥⎦ = − d 2
3/ 2

2 ⎢⎣ 2
π /2 π /2

π q2 π
⇒ − 2 At = −d 3/ 2 ⇒− 2 × t = − d 3/ 2
2 16π mε 0 2

π 8π mε 0 2π 3ε 0 md 3
⇒t =d 3/ 2
× =
2 q2 q
                                                                                
Head office  Branch office 
 
fiziks, H.No. 40 D, G.F, Jia Sarai,  Anand Institute of Mathematics, 
 
Near IIT, Hauz Khas, New Delhi‐16  28‐B/6, Jia Sarai, Near IIT 
 
Phone: 011‐26865455/+91‐9871145498 Hauz Khas, New Delhi‐16 
                                                   
                                             Website: www.physicsbyfiziks.com                                                                                          
                                                           Email: fiziks.physics@gmail.com                                                                   2 
fiziks
Institute for NET/JRF, GATE, IIT‐JAM, JEST, TIFR and GRE in PHYSICAL SCIENCES 
 
N 1
Q3. A two dimensional box in a uniform magnetic field B contains localised spin-
2 2
N
particles with magnetic moment μ , and free spinless particles which do not interact
2
with each other. The average energy of the system at a temperature T is:
1 ⎛ μB ⎞ 1 ⎛ μB ⎞
(a) 3 NkT − N μ B sinh ⎜ ⎟ (b) NkT − N μ B tanh ⎜ ⎟
2 ⎝ k BT ⎠ 2 ⎝ k BT ⎠

1 1 ⎛ μB ⎞ 3 1 ⎛ μB ⎞
(c) NkT − N μ B tanh ⎜ ⎟ (d) NkT + N μ B cosh ⎜ ⎟
2 2 ⎝ k BT ⎠ 2 2 ⎝ k BT ⎠
Ans: (c)
N N N 1
Solution: For free particle in two dimension energy is kT , for localized spin-
2 2 2 2

1 ⎛ μB ⎞
particle the energy is − N μ B tanh ⎜ ⎟
2 ⎝ k BT ⎠

NkT 1 ⎛ μB ⎞
− N μ B tanh ⎜ ⎟
2 2 ⎝ k BT ⎠
CP
Q4. An ideal gas has a specific heat ratio = 2 . Starting at a temperature T1 the gas under
CV
goes an isothermal compression to increase its density by a factor of two. After this an
adiabatic compression increases its pressure by a factor of two. The temperature of the
gas at the end of the second process would be:
T1 T1
(a) (b) 2T1 (c) 2T1 (d)
2 2
Ans (b)
During the isothermal process T = T1 is constant

Let us assume the adiabatic process started at point A ( P1 , T1 ) and at point B the
1−γ 1− 2
⎛P⎞ γ ⎛ P ⎞ 2
coordinate is ( P2 , T2 ) it is given P11−γ T1γ = P21−γ T2γ ⇒ T2 = ⎜ 1 ⎟ T1 ⇒ T2 = ⎜ 1 ⎟ T1
⎝ P2 ⎠ ⎝ 2 P1 ⎠

T2 = 2T1

                                                                                
Head office  Branch office 
 
fiziks, H.No. 40 D, G.F, Jia Sarai,  Anand Institute of Mathematics, 
 
Near IIT, Hauz Khas, New Delhi‐16  28‐B/6, Jia Sarai, Near IIT 
 
Phone: 011‐26865455/+91‐9871145498 Hauz Khas, New Delhi‐16 
                                                   
                                             Website: www.physicsbyfiziks.com                                                                                          
                                                           Email: fiziks.physics@gmail.com                                                                   3 
fiziks
Institute for NET/JRF, GATE, IIT‐JAM, JEST, TIFR and GRE in PHYSICAL SCIENCES 
 
Q5. A gas of N molecules of mass m is confined in a cube of volume V = L3 at temperature
T . The box is in a uniform gravitational field − gzˆ . Assume that the potential energy of a

molecule is U = mgz where z ∈ [ 0, L ] is the vertical coordinate inside the box. The

pressure P ( z ) at height z is:

⎛ ⎛ L⎞⎞ ⎛ ⎛ L⎞⎞
⎜ mg ⎜ z − 2 ⎟ ⎟ ⎜ mg ⎜ z − 2 ⎟ ⎟
exp ⎜ − ⎝ ⎠⎟ exp ⎜ − ⎝ ⎠⎟
⎜ k BT ⎟ ⎜ k BT ⎟
⎜ ⎟ ⎜ ⎟
(a) P ( z ) =
N mgL ⎝ ⎠ (b) P ( z ) =
N mgL ⎝ ⎠
V 2 ⎛ mgL ⎞ V 2 ⎛ mgL ⎞
sinh ⎜ ⎟ cosh ⎜ ⎟
⎝ 2 k BT ⎠ ⎝ 2 k BT ⎠
k BTN N
(c) P ( z ) = (d) P ( z ) = mgz
V V
Ans: (c)
The partition function of a system is given by
3N N
⎛ k BTV ⎞ ⎛ ⎛ mgl ⎞ ⎞
N
⎛ 2π mk BT ⎞ 2
ZN = ⎜ 2 ⎟ ⎜ ⎟ ⎜⎜ 1 − exp ⎜ − ⎟ ⎟⎟
⎝ h ⎠ ⎝ mgL ⎠ ⎝ ⎝ k BT ⎠ ⎠
Helmohtz free energy is given by F = −kBT ln Z N

⎛ ∂F ⎞ k TN
Pressure is given by P = − ⎜ ⎟ = B
⎝ ∂V ⎠T , N V

                                                                                
Head office  Branch office 
 
fiziks, H.No. 40 D, G.F, Jia Sarai,  Anand Institute of Mathematics, 
 
Near IIT, Hauz Khas, New Delhi‐16  28‐B/6, Jia Sarai, Near IIT 
 
Phone: 011‐26865455/+91‐9871145498 Hauz Khas, New Delhi‐16 
                                                   
                                             Website: www.physicsbyfiziks.com                                                                                          
                                                           Email: fiziks.physics@gmail.com                                                                   4 
fiziks
Institute for NET/JRF, GATE, IIT‐JAM, JEST, TIFR and GRE in PHYSICAL SCIENCES 
 
JEST-2015
Q6. For a system in thermal equilibrium with a heat bath at temperature T , which one of the
⎛ 1 ⎞
following equalities is correct? ⎜ β = ⎟
⎝ k BT ⎠

∂ 2 ∂ 2
(a) E = E − E2 (b) E = E2 − E
∂β ∂β
(c)

∂β
E = E2 + E
2
(d)

∂β
E = − E2 + E ( 2
)
Ans: (a)

∑Ee β i
− Ei

Solution: ∵ E = i

∑e β i
− Ei

∂ E ∑E e β i
2 − Ei
∑E e βi
2 − Ei
⋅ e− β Ei ∑E e β i
2 − Ei
∑E e i
2 −2 β Ei

=− i
+ i
=− i
+ i
∂β ∑e β − Ei
⎛ − β Ei ⎞
2
∑e β − Ei
⎛ − β Ei ⎞
2

i ⎜∑e ⎟ i ⎜∑e ⎟
⎝ i ⎠ ⎝ i ⎠
∂ E 2
⇒ = E − E2
∂β
Q7. An ideal gas is compressed adiabatically from an initial volume V to a final volume αV
and a work W is done on the system in doing so. The final pressure of the gas will be
⎛ C ⎞
⎜⎜ γ = P ⎟⎟
⎝ CV ⎠
W 1− γ W γ −1
(a) (b)
V γ α −α γ V γ α −α γ
W 1− γ W γ −1
(c) (d)
V α −αγ V α −αγ
Ans: (c)
Solution: Work done in adiabatic process

2 2 − PV
PV
W= 1 1
1− γ

P2V2γ = PV 1 1
γ

                                                                                
Head office  Branch office 
 
fiziks, H.No. 40 D, G.F, Jia Sarai,  Anand Institute of Mathematics, 
 
Near IIT, Hauz Khas, New Delhi‐16  28‐B/6, Jia Sarai, Near IIT 
 
Phone: 011‐26865455/+91‐9871145498 Hauz Khas, New Delhi‐16 
                                                   
                                             Website: www.physicsbyfiziks.com                                                                                          
                                                           Email: fiziks.physics@gmail.com                                                                   5 
fiziks
Institute for NET/JRF, GATE, IIT‐JAM, JEST, TIFR and GRE in PHYSICAL SCIENCES 
 
γ
⎛V ⎞
P1 = P2 ⎜ 2 ⎟ ⇒ P1 = P2 (α )
γ

⎝ V1 ⎠
P2αV − P2α γ V
W =
(1 − γ )
W (1 − γ )
P2 =
V (α − α γ )

Q8. What is the area of the irreducible Brillouin zone of the crystal structure as given in the
figure?
2π 2
(a)
3 A2

3π 2
(b)
2 A2
2π 2 o A
(c) 60
A2 B
π 2
A = B =A
(d)
3A 2
Ans: (a)
Solution: Area of the Brillouin zone can be related to the area of normal cell as
π2 π2
Area of B.Z. = =
Area of cell A× B B
600
3 2
A × B = A B sin θ = A2 sin 600 = ( ) 2
A A
A= B =A
2π 2
∴ Area of Brillouin zone =
3A2

                                                                                
Head office  Branch office 
 
fiziks, H.No. 40 D, G.F, Jia Sarai,  Anand Institute of Mathematics, 
 
Near IIT, Hauz Khas, New Delhi‐16  28‐B/6, Jia Sarai, Near IIT 
 
Phone: 011‐26865455/+91‐9871145498 Hauz Khas, New Delhi‐16 
                                                   
                                             Website: www.physicsbyfiziks.com                                                                                          
                                                           Email: fiziks.physics@gmail.com                                                                   6 
fiziks
Institute for NET/JRF, GATE, IIT‐JAM, JEST, TIFR and GRE in PHYSICAL SCIENCES 
 
Q9. A particle in thermal equilibrium has only 3 possible states with energies − ∈, 0, ∈ . If

the system is maintained at a temperature T >> , then the average energy of the particle
kB
can be approximated to,
2 ∈2 −2 ∈2
(a) (b)
3k BT 3k BT

− ∈2
(c) (d) 0
k BT
Ans: (b)
ε ε
+ − ⎛ − kTε ε

−ε e + 0 +εe −
=ε⎜ ⎟
kT kT kT
e e
Solution: E = ε ε ⎜ ε ε ⎟
+1+ e

⎜ 1 + e − kT + e kT ⎟
⎝ ⎠
kT kT
e

⎡⎛ ε ⎞ ⎛ ε ⎞⎤
⎢⎜1 − kT ⎟ − ⎜1 + kT ⎟ ⎥ −2ε 2
⎝ ⎠ ⎝ ⎠⎦
⇒ E = ⎣ =
⎛ ε ⎞ ⎛ ε ⎞ 3kT
1 + ⎜1 − ⎟ + ⎜1 + ⎟
⎝ kT ⎠ ⎝ kT ⎠
Q10. The blackbody at a temperature of 6000 K emits a radiation whose intensity spectrum
peaks at 600 nm . If the temperature is reduced to 300 K , the spectrum will peak at,
(a) 120μm (b) 12 μ m (c) 12 mm (d) 120 mm
Ans: (b)
λ1T1 600 × 20
Solution: λ1T1 = λ2T2 ⇒ λ2 = = = 12000 nm = 12 μ m
T2 300

                                                                                
Head office  Branch office 
 
fiziks, H.No. 40 D, G.F, Jia Sarai,  Anand Institute of Mathematics, 
 
Near IIT, Hauz Khas, New Delhi‐16  28‐B/6, Jia Sarai, Near IIT 
 
Phone: 011‐26865455/+91‐9871145498 Hauz Khas, New Delhi‐16 
                                                   
                                             Website: www.physicsbyfiziks.com                                                                                          
                                                           Email: fiziks.physics@gmail.com                                                                   7 
fiziks
Institute for NET/JRF, GATE, IIT‐JAM, JEST, TIFR and GRE in PHYSICAL SCIENCES 
 
Q11. The entropy-temperature diagram of two Carnot engines, A and B , are shown in the
figure 4. The efficiencies of the engines are η A and η B respectively. Which one of the
following equalities is correct? S
ηB
(a) η A =
2 A B
(b) η A = η B
(c) η A = 3η B
(d) η A = 2η B
T
Ans: (d)
ΔW
Solution: η = where ΔW = area under the curve , Q1 = area under high temperature
Q1

( 2T − T ) T 1 ( 4T − 3T ) ( 4S − S ) T 1
ηA = = = and η B = = =
2T 2T 2 4T ( 4S − S ) 4T 4

η A 1/ 2
⇒ = = 2 ⇒ η A = 2ηB
η B 1/ 4

Q12. Electrons of mass m in a thin, long wire at a temperature T follow a one-dimensional


Maxwellian velocity distribution. The most probable speed of these electrons is,

⎛ kT ⎞ ⎛ 2kT ⎞ ⎛ 8kT ⎞
(a) ⎜ ⎟ (b) ⎜ ⎟ (c) 0 (d) ⎜ ⎟ .
⎝ 2πm ⎠ ⎝ m ⎠ ⎝ πm ⎠
Ans: (c) f ( vx )
1/ 2 mv 2
⎛ m ⎞ − x
Solution: f ( vx ) = ⎜ ⎟ e 2 kT
dvx ; −∞ < vx < ∞
⎝ 2π kT ⎠
Most probable speed vx = 0 vx

                                                                                
Head office  Branch office 
 
fiziks, H.No. 40 D, G.F, Jia Sarai,  Anand Institute of Mathematics, 
 
Near IIT, Hauz Khas, New Delhi‐16  28‐B/6, Jia Sarai, Near IIT 
 
Phone: 011‐26865455/+91‐9871145498 Hauz Khas, New Delhi‐16 
                                                   
                                             Website: www.physicsbyfiziks.com                                                                                          
                                                           Email: fiziks.physics@gmail.com                                                                   8 
fiziks
Institute for NET/JRF, GATE, IIT‐JAM, JEST, TIFR and GRE in PHYSICAL SCIENCES 
 
JEST-2014
Q13. A monoatomic gas consists of atoms with two internal energy levels, ground state E 0 = 0

and an excited state E1 = E . The specific heat of the gas is given by

3 E 2 e E / kT
(a) k (b)
2 (
kT 2 1 + e E / kT )
2

3 E 2 e E / kT 3 E 2 e E / kT
(c) k+ (d) k−
2 (
kT 2 1 + e E / kT )2
2 (
kT 2 1 + e E / kT )2

Ans.: (c)
Solution: E0 = 0, E1 = E

z = ∑ e − β Ei ⇒ z = e − β ×0 + e− β E

(
ln z = ln 1 + e − β E1 )
−∂ ∂ 1
U= E = ln z = − ln 1 + e − β ( E
)=− ( − E ) e− β E
∂β ∂β ( 1 + e− β E )
Ee− β E
U= ∵ β = k BT
1 + e− β E
⎛ −
E
⎞ − kET ⎛ E ⎞ −
E

k BT ⎛
E
E ⎞
⎜1 + e k BT
⎟ E.e . ⎜
B
⎟ − Ee k BT
⋅e ⎜ 2 ⎟
∂ ⎜ ⎟ 2
⎝ k BT ⎠ ⎝ k BT ⎠
⎛ U ⎞ ⎝ ⎠
⎜ ⎟ = C =
⎝ ∂T ⎠v
V 2
⎛ −
E

⎜ 1 + e k BT ⎟
⎜ ⎟
⎝ ⎠
E 2E 2E
E 2 − k BT E 2 − k BT E 2 − k BT
+ −
E E
e e e −
k BT 2 k BT 2 k BT 2 E 2e k BT
E 2 e k BT
CV = 2
= 2
= 2
⎛ −
E
⎞ ⎛ −
E
⎞ ⎛ E

⎜1 + e k BT
⎟ k BT ⎜ 1 + e
2 k BT
⎟ k BT ⎜ 1 + e
2 k BT

⎜ ⎟ ⎜ ⎟ ⎜ ⎟
⎝ ⎠ ⎝ ⎠ ⎝ ⎠
3
If gas will classically allowed then CV = kB
2

                                                                                
Head office  Branch office 
 
fiziks, H.No. 40 D, G.F, Jia Sarai,  Anand Institute of Mathematics, 
 
Near IIT, Hauz Khas, New Delhi‐16  28‐B/6, Jia Sarai, Near IIT 
 
Phone: 011‐26865455/+91‐9871145498 Hauz Khas, New Delhi‐16 
                                                   
                                             Website: www.physicsbyfiziks.com                                                                                          
                                                           Email: fiziks.physics@gmail.com                                                                   9 
fiziks
Institute for NET/JRF, GATE, IIT‐JAM, JEST, TIFR and GRE in PHYSICAL SCIENCES 
 
E
2 k BT
E e
and due to quantum mechanically CV = 2
⎛ E

k BT ⎜ 1 + e k B T
2

⎜ ⎟
⎝ ⎠
3 E 2 e E / kT
∴ CV = kB +
( )
2
2 kT 2 1 + e E / kT

Q14. The temperature of a thin bulb filament (assuming that the resistance of the filament is
nearly constant) of radius r and length L is proportional to
(a) r 1 / 4 L−1 / 2 (b) L2 r (c) L1 / 4 r −1 (d) r 2 L−1
Ans.: (a)
Q15. Ice of density ρ1 melts at pressure P and absolute temperature T to form water of
density ρ 2 . The latent heat of melting of 1 gram of ice is L . What is the change in the
internal energy ΔU resulting from the melting of 1 gram of ice?
⎛ 1 1 ⎞ ⎛ 1 1 ⎞
(a) L + P⎜⎜ − ⎟⎟ (b) L − P⎜⎜ − ⎟⎟
⎝ ρ 2 ρ1 ⎠ ⎝ ρ 2 ρ1 ⎠

⎛ 1 1 ⎞ ⎛ 1 1 ⎞
(c) L − P⎜⎜ − ⎟⎟ (d) L + P⎜⎜ − ⎟⎟
⎝ ρ1 ρ 2 ⎠ ⎝ ρ1 ρ 2 ⎠
Ans.: (c)
Solution: dU = dQ − δ W = dQ − pdV
ρ
2
⎛ 1 ⎞ ⎡1 1 ⎤
dU = mL − pdV ⇒ dU = L − P ∫ ⎜ − 2 ⎟dρ = L − P⎢ − ⎥
ρ1 ⎝ ρ ⎠ ⎣ ρ1 ρ 2 ⎦
1 1
∵V = ⇒ dV = − 2 d ρ
ρ ρ
Q16. What is the contribution of the conduction electrons in the molar entropy of a metal with
electronic coefficient of specific heat?
(a) γT (b) γT 2 (c) γT 3 (d) γT 4
Ans.: (a)
Solution: CV = BT 3 + AT
                                                                                
Head office  Branch office 
 
fiziks, H.No. 40 D, G.F, Jia Sarai,  Anand Institute of Mathematics, 
 
Near IIT, Hauz Khas, New Delhi‐16  28‐B/6, Jia Sarai, Near IIT 
 
Phone: 011‐26865455/+91‐9871145498 Hauz Khas, New Delhi‐16 
                                                   
                                             Website: www.physicsbyfiziks.com                                                                                          
                                                           Email: fiziks.physics@gmail.com                                                                   10 
fiziks
Institute for NET/JRF, GATE, IIT‐JAM, JEST, TIFR and GRE in PHYSICAL SCIENCES 
 
Q17. Consider a system of 2N non-interacting spin 1 / 2 particles each fixed in position and
carrying a magnetic moment μ . The system is immersed in a uniform magnetic field B.
The number of spin up particle for which the entropy of the system will be maximum is
(a) 0 (b) N (c) 2 N (d) N / 2
Ans.: (b)
Solution: Let us consider n number of spin out of 2 N particle have spin up remaining 2 N − n is
down.
1
Number of ways ω = 2 N C n for spin (up)
2
1
ω = 2 NC for spin down
2 N −n
2
entropy S = k ln ω ⇒ S = k ln 2N
C2 N − n + k ln 2N
Cn

⎧⎪ ⎡ 2N ! ⎤ ⎡ 2N ! ⎤ ⎫⎪
S = K ⎨ ⎢ ln ⎥ + ⎢ ln ⎥⎬
⎩⎪ ⎢⎣ ( n !)( 2 N − n ) !⎥⎦ ⎢⎣ ( n !)( 2 N − n ) !⎥⎦ ⎭⎪
S = 2k [(ln 2 N !− ln n!− ln (2 N − n )!)]

S = 2 K ⎡⎣ 2 N ln 2 N − 2 N − n ln n + n − {( 2 N − n ) ln ( 2 N − n ) − ( 2 N − n )}⎤⎦
∵ ln N != N ln N − N !
S = 2 K ⎡⎣ 2 N ln 2 N − 2 N − n ln n + n − 2 N ln ( 2 N − n ) + n ln ( 2 N − n ) + ( 2 N − n ) ⎤⎦

S = 2 K ⎡⎣ 2 N ln 2 N − n ln n − 2 N ln ( 2 N − n ) + n ln ( 2 N − n ) ⎤⎦

1
now for entropy maximum at equilibrium for spin up particle
2
dS
=0
dn
⎡ n
( −1) + ln ( 2 N − n )⎤⎥
2N n
= 2 K ⎢ − ⋅1 − ln n − ( −1) +
⎣ n 2N − n 2N − n ⎦
⎡ 2N ⎤
+ ln(2 N − n )⎥
n
= 2 K ⎢− 1 − ln n + −
⎣ 2N − n 2N − n ⎦

                                                                                
Head office  Branch office 
 
fiziks, H.No. 40 D, G.F, Jia Sarai,  Anand Institute of Mathematics, 
 
Near IIT, Hauz Khas, New Delhi‐16  28‐B/6, Jia Sarai, Near IIT 
 
Phone: 011‐26865455/+91‐9871145498 Hauz Khas, New Delhi‐16 
                                                   
                                             Website: www.physicsbyfiziks.com                                                                                          
                                                           Email: fiziks.physics@gmail.com                                                                   11 
fiziks
Institute for NET/JRF, GATE, IIT‐JAM, JEST, TIFR and GRE in PHYSICAL SCIENCES 
 
⎡ 2N − n ⎤
= 2 K ⎢− 1 + + ln(2 N − n ) − ln n⎥
⎣ 2N − n ⎦

= 2k ⎢− 1 + 1 + ln
(2 N − n )⎤ = 0
⎣ n ⎥⎦

∵ 2k ≠ 0
⎛ 2N − n ⎞ 2N − n
∴ ln⎜ ⎟=0 ⇒ =1 ⇒ 2 N = 2n ⇒ n = N
⎝ n ⎠ n
Q18. For which gas the ratio of specific heats (C p / C v ) will be the largest?

(a) mono-atomic (b) di-atomic (c) tri-atomic (d) hexa-atomic


Ans.: (a)
CP ⎛ 2⎞
Solution: = γ = ⎜1 + ⎟ where f is degree of freedom.
CV ⎝ f ⎠
For monoatomic: f = 3 , For diatomic: f = 6 , For Triatomic: f = 9
For hexaatomic: f = 18

JEST-2013
Q19. Consider a system of two particles A and B . Each particle can occupy one of three
possible quantum states 1 , 2 and 3 . The ratio of the probability that the two particles

are in the same state to the probability that the two particles are in different states is
calculated for bosons and classical (Maxwell-Boltzmann) particles. They are respectively
1 1 1
(a) 1, 0 (b) ,1 (c) 1, (d) 0,
2 2 2
Ans.: (c)
Solution: For two particle in same state:
AB 3 AB
3 3 3 3 3
AB 2 AB
2 2 2 2 2
AB AB
1 1 1 1 1 1
Boson Classical (Maxwell - Boltzman)
                                                                                
Head office  Branch office 
 
fiziks, H.No. 40 D, G.F, Jia Sarai,  Anand Institute of Mathematics, 
 
Near IIT, Hauz Khas, New Delhi‐16  28‐B/6, Jia Sarai, Near IIT 
 
Phone: 011‐26865455/+91‐9871145498 Hauz Khas, New Delhi‐16 
                                                   
                                             Website: www.physicsbyfiziks.com                                                                                          
                                                           Email: fiziks.physics@gmail.com                                                                   12 
fiziks
Institute for NET/JRF, GATE, IIT‐JAM, JEST, TIFR and GRE in PHYSICAL SCIENCES 
 
1/ 3
Probability ratio: =1
1/ 3
For two particle in different states
B B B A B A
3 3 3 3 3 3 3 3 3
B A B A A B
2 2 2 2 2 2 2 2 2
A A 1 A B A B
1 1 1 1 1 1 1 1
Boson Classical (Maxwell-Boltzmann)
1/ 3 1
Probability ratio: =
2/3 2

Q20. For a diatomic ideal gas near room temperature, what fraction of the heat supplied is
available for external work if the gas is expanded at constant pressure?
1 5 3 2
(a) (b) (c) (d)
7 7 4 7
Ans.: (d)
Solution: It is isobaric process (constant pressure)
Then δθ = nC p ΔT ⇒ ΔW = nRΔT

In this process δθ is heat exchange during process.


Function of heat supplied
δW nRΔT R γ −1 1
= = = = = 1−
ΔQ nC p ΔT R γ γ γ
γ −1
1 Cp γR
⇒ 1− γ= ⇒ Cp =
⎛ 2⎞ CV γ −1
⎜1 + f ⎟
⎝ ⎠
f
⇒ 1− f = degree of freedom, for diatomic molecule f = 5
f +2
5 2
⇒ 1− ⇒
5+2 7
                                                                                
Head office  Branch office 
 
fiziks, H.No. 40 D, G.F, Jia Sarai,  Anand Institute of Mathematics, 
 
Near IIT, Hauz Khas, New Delhi‐16  28‐B/6, Jia Sarai, Near IIT 
 
Phone: 011‐26865455/+91‐9871145498 Hauz Khas, New Delhi‐16 
                                                   
                                             Website: www.physicsbyfiziks.com                                                                                          
                                                           Email: fiziks.physics@gmail.com                                                                   13 
fiziks
Institute for NET/JRF, GATE, IIT‐JAM, JEST, TIFR and GRE in PHYSICAL SCIENCES 
 
Q21. Consider the differential equation
dG ( x )
+ kG( x ) = δ ( x ) ,
dx
where k is a constant. Which of the following statements is true?
(a) Both G ( x ) and G ′ ( x ) are continuous at x = 0

(b) G ( x ) is continuous at x = 0 but G ′ ( x ) is not.

(c) G ( x ) is discontinuous at x = 0

(d) The continuity properties of G ( x ) and G ′ ( x ) at x = 0 depend on the value of k .

Ans.: (c)
Q22. A metal bullet comes to rest after hitting its target with a velocity of 80 m/s. If 50% of the
heat generated remains in the bullet, what is the increase in its temperature? (The specific
heat of the bullet = 160 Joule / kg / 0 C )

(a) 140 C (b) 12.50 C (c) 100 C (d) 8.20 C


Ans.: (c)
1 2 1
Solution: Conservation of momentum mv × 50 % = mcΔT ⇒ 80 × 80 = 160 ΔT
2 2
80 × 80 1
⇒ ΔT = × = 100 C
4 160
Q23. Consider a particle with three possible spin states: s = 0 and ±1 . There is a magnetic
field h present and the energy for a spin state s is − hs . The system is at a
temperatureT . Which of the following statements is true about the entropy S (T ) ?

(a) S (T ) = ln 3 at T = 0, and 3 at high T (b) S (T ) = ln 3 at T = 0, and 0 at high T

(c) S (T ) = 0 at T = 0, and 3 at high T (d) S (T ) = 0 at T = 0, and ln 3 at high T

Ans.: (d)
Solution: S = k ln ω where ω = number of microstates
S = k ln 3 at high T ω =3
and at T = 0 it is perfect ordered i.e. S = 0
                                                                                
Head office  Branch office 
 
fiziks, H.No. 40 D, G.F, Jia Sarai,  Anand Institute of Mathematics, 
 
Near IIT, Hauz Khas, New Delhi‐16  28‐B/6, Jia Sarai, Near IIT 
 
Phone: 011‐26865455/+91‐9871145498 Hauz Khas, New Delhi‐16 
                                                   
                                             Website: www.physicsbyfiziks.com                                                                                          
                                                           Email: fiziks.physics@gmail.com                                                                   14 
fiziks
Institute for NET/JRF, GATE, IIT‐JAM, JEST, TIFR and GRE in PHYSICAL SCIENCES 
 
Q24. Consider three situations of 4 particles in one dimensional box of width L with hard
walls. In case (i), the particles are fermions, in case (ii) they are bosons, and in case (iii)
they are classical. If the total ground state energy of the four particles in these three cases
are EF , EB and Ecl respectively, which of the following is true?

(a) EF = EB = Ecl (b) EF > EB = Ecl

(c) EF < EB < Ecl (d) EF > EB > Ecl


Ans.: (b)
π2 2
Solution: For fermions =∈0
2ml 2
1×∈0 +1× 4 ∈0 +1× 9 ∈0 +1× 16 ∈0 = 30 ∈0

For Boson = 4×∈0 , For Maxwell = 4×∈0

EF > EB = Ecl

JEST-2012
Q25. A monoatomic ideal gas at 170C is adiabatically compressed to 1/8 of its original
volume. The temperature after compression is
(a) 2.1 oC (b) 17oC (c) -200.5oC (d) 887oC
Ans. : (d)
Solution: ⇒ PV γ = costant , PV = RT
TV γ
= costant
V
⇒ TV γ −1 = costant
r −1
γ −1 γ −1 ⎛V ⎞
⇒ TV
1 1 = T2V2 ⇒ T2 = T1 ⎜ 1 ⎟
⎝ V2 ⎠
= 443 > (θ ) .6 = 443 × 3.40 = 1541.64

T2 = 1541.64 − 273 = 126θ 0 C

⎛ 2⎞ 2
∵ r = ⎜1 + ⎟ = 1 + = 1.66
⎝ 1⎠ 3

                                                                                
Head office  Branch office 
 
fiziks, H.No. 40 D, G.F, Jia Sarai,  Anand Institute of Mathematics, 
 
Near IIT, Hauz Khas, New Delhi‐16  28‐B/6, Jia Sarai, Near IIT 
 
Phone: 011‐26865455/+91‐9871145498 Hauz Khas, New Delhi‐16 
                                                   
                                             Website: www.physicsbyfiziks.com                                                                                          
                                                           Email: fiziks.physics@gmail.com                                                                   15 
fiziks
Institute for NET/JRF, GATE, IIT‐JAM, JEST, TIFR and GRE in PHYSICAL SCIENCES 
 
Q26. Consider a system of particles in three dimensions with momentum p and energy

E = c p , c being a constant. The system is maintained at inverse temperature β, volume

V and chemical potential μ. What is the grand partition function of the system?
[
(a) exp e βμ 8πV / (β ch ) ] 3
(b) e βμ 6πV / (β ch )
2

(c) exp[e βμ
6πV / (βch ) ] (d) e βμ 8πV / (β ch )
3 2

Ans. : (a)
Solution: canonical partition function
1 −β H
h3 ∫
zN = e dpx dp y dpz dxdydz E = pc

∞ ∞
V V 4π V 3 8π V
z N = 3 ∫ 4π p 2 e − Eβ dp = 3 ∫ 4π p 2 e − βpc dp = 3 ⋅ =
h 0 h 0 h (βc ) (βhc )3
3

⎡ μ ⎤ ⎡ μ 8πV ⎤
grand canonical partition function z u = exp ⎢e kT z N ⎥ = exp ⎢e kT ⋅ ⎥
⎣ ⎦ ⎣ (βhc )3 ⎦
⎡ 8πV ⎤
⇒ exp ⎢e βμ ⋅
⎣ (βhc )3 ⎥⎦
Q27. Consider a system maintained at temperature T, with two available energy states E1 and
E2 each with degeneracies g1 and g2. If p1 and p2 are probabilities of occupancy of the two
energy states, what is the entropy of the system?
(a) S = −k B [ p1 ln ( p1 / g1 ) + p 2 ln ( p 2 / g 2 )]
(b) S = −k B [ p1 ln ( p1 g1 ) + p 2 ln ( p 2 g 2 )]

(c) S = −k B p1 ln p1[ ( ) + p ln(p )]


g1
2 2
g2

(d) S = −k B [(1 / p1 ) ln ( p1 / g1 ) + (1 / p 2 ) ln ( p 2 / g 2 )]
Ans. : (a)
Σgi e− β Ei
Solution: pi = where z is partition function
z
ln pi = ln g i − β Ei − ln z

                                                                                
Head office  Branch office 
 
fiziks, H.No. 40 D, G.F, Jia Sarai,  Anand Institute of Mathematics, 
 
Near IIT, Hauz Khas, New Delhi‐16  28‐B/6, Jia Sarai, Near IIT 
 
Phone: 011‐26865455/+91‐9871145498 Hauz Khas, New Delhi‐16 
                                                   
                                             Website: www.physicsbyfiziks.com                                                                                          
                                                           Email: fiziks.physics@gmail.com                                                                   16 
fiziks
Institute for NET/JRF, GATE, IIT‐JAM, JEST, TIFR and GRE in PHYSICAL SCIENCES 
 
ln pi − ln g i = − β Ei − ln z ∵ F = − kT ln z

pi F
ln = − βEi +
gi kT

pi
ln = − βEi + βF
gi

pi
ln = − β Ei + β F
gi

pi
ln = β [F −U ] ∵ F = U − TS
gi

pi 1
ln = − β × TS β=
gi kT

pi ⎛ p ⎞ ⎡ p p ⎤
S = − k ln = − k ⎜ ∑ pi ln i ⎟ = − k ⎢ p1 ln 1 + p2 ln 2 ⎥
gi ⎝ gi ⎠ ⎣ g1 g2 ⎦

Q28. Consider an ideal gas of mass m at temperature T1 which is mixed isobarically (i.e. at

constant pressure) with an equal mass of same gas at temperature T2 in a thermally


insulated container. What is the change of entropy of the universe?
⎛ T + T2 ⎞ ⎛ T − T2 ⎞
(a) 2mC p ln⎜ 1 ⎟ (b) 2mC p ln⎜ 1 ⎟
⎜2 TT ⎟ ⎜2 TT ⎟
⎝ 1 2 ⎠ ⎝ 1 2 ⎠

⎛ T + T2 ⎞ ⎛ T − T2 ⎞
(c) 2mC p ln⎜⎜ 1 ⎟⎟ (d) 2mC p ln⎜ 1 ⎟
⎝ 2T1T2 ⎠ ⎜2 TT ⎟
⎝ 1 2 ⎠
Ans. : (a)
Solution: Let us consider final temperature will be T
mc(T1 − T ) = mc(T − T2 )

T1 + T2
T=
2
ΔT
ΔS1 = mc p
T
                                                                                
Head office  Branch office 
 
fiziks, H.No. 40 D, G.F, Jia Sarai,  Anand Institute of Mathematics, 
 
Near IIT, Hauz Khas, New Delhi‐16  28‐B/6, Jia Sarai, Near IIT 
 
Phone: 011‐26865455/+91‐9871145498 Hauz Khas, New Delhi‐16 
                                                   
                                             Website: www.physicsbyfiziks.com                                                                                          
                                                           Email: fiziks.physics@gmail.com                                                                   17 
fiziks
Institute for NET/JRF, GATE, IIT‐JAM, JEST, TIFR and GRE in PHYSICAL SCIENCES 
 
T T
dT dT
now ΔS = ΔS1 + ΔS 2 ⇒ ΔS = mc p ∫ + mc p ∫
T1
T T2
T

⎛T ⎞ ⎛T ⎞
ΔS = mc p ln ⎜ ⎟ + mc p ln ⎜ ⎟
⎝ T1 ⎠ ⎝ T2 ⎠
2
T ⎛ T +T ⎞ ⎛ T +T ⎞
ΔS = 2mc p ln = mc p ln ⎜ 1 2 ⎟ ⇒ ΔS = 2mc p ln ⎜ 1 2 ⎟
T1T2 ⎜ 2 TT ⎟ ⎜ 2 TT ⎟
⎝ 1 2 ⎠ ⎝ 1 2 ⎠
Q29. A collection of N two-level systems with energies 0 and E > 0 is in thermal equilibrium
at temperature T. For T → ∞, the specific heat approaches
(a) 0 (b) NkB (c) 3NkB/2 (d) ∞
Ans.: (a)
Solution: Z = ∑ e − βEi = e − β ×0 + e − βEi ⇒ Z = 1 + e − βE ⇒ ln z = ln (1 + e − βE )

∂ ∂ 1 Ee − β E
U = E =−

ln z = −
∂β
(
ln 1 + e − β E = −
1 + e− β E
×)e −β E
, ( − E ) =
1 + e− β E

⎛ −
E

⎛ ∂U ⎞ ∂ ⎜ Ee kT ⎟
now ⎜ ⎟ = CV =
⎝ ∂T ⎠V ∂T ⎜⎜ −
E ⎟

⎝ 1 + e kT

⎛ E 2 −kTE 2 −2 E 2 −2 E
⎞ −E
⎜ 2 e + E 2 e kT − E 2 e kT ⎟ E 2 kT
⎜ kT ⎟ e
CV = ⎝ ⎠ ⇒ C = kT 2
kT kT
2
⇒ CV T →∞
=0
−E 2
V
⎛ −E
⎞ ⎛ ⎞
⎜1 + e kT ⎟ ⎜1 + e kT ⎟
⎜ ⎟ ⎜ ⎟
⎝ ⎠ ⎝ ⎠
Q30. Efficiency of a perfectly reversible (Carnot) heat engine operating between absolute
temperature T and zero is equal to
(a) 0 (b) 0.5 (c) 0.75 (d) 1
Ans. : (d)
T2 T
Solution: η = 1 − = 1− = 1
T1 0

                                                                                
Head office  Branch office 
 
fiziks, H.No. 40 D, G.F, Jia Sarai,  Anand Institute of Mathematics, 
 
Near IIT, Hauz Khas, New Delhi‐16  28‐B/6, Jia Sarai, Near IIT 
 
Phone: 011‐26865455/+91‐9871145498 Hauz Khas, New Delhi‐16 
                                                   
                                             Website: www.physicsbyfiziks.com                                                                                          
                                                           Email: fiziks.physics@gmail.com                                                                   18 
fiziks
Institute for NET/JRF, GATE, IIT‐JAM, JEST, TIFR and GRE in PHYSICAL SCIENCES 
 
Q31. A thermally insulated ideal gas of volume V1 and temperature T expands to another
enclosure of volume V2 through a prous plug. What is the change in the temperature of
the gas?
(a) 0 (b) T ln(V1 / V2) (c) T ln(V2 / V1) (d) T ln(V2 – V1) / V2)
Ans. : ()
Solution: dH = TdS + VdP , for porous plug Joul Thomshon dH = 0 and TdS = 0 since it is
thermally insulated ideal gas
VdP = 0
nRTdV
∵ VdP = 0 ⇒ nRdT = pdV ⇒ nRdT =
V
dV V2 dV V
dT = T ⇒ dT = T ∫ ⇒ dT = T ln 2
V V1 V V1

                                                                                
Head office  Branch office 
 
fiziks, H.No. 40 D, G.F, Jia Sarai,  Anand Institute of Mathematics, 
 
Near IIT, Hauz Khas, New Delhi‐16  28‐B/6, Jia Sarai, Near IIT 
 
Phone: 011‐26865455/+91‐9871145498 Hauz Khas, New Delhi‐16 
                                                   
                                             Website: www.physicsbyfiziks.com                                                                                          
                                                           Email: fiziks.physics@gmail.com                                                                   19 
fiziks
Institute for NET/JRF, GATE, IIT‐JAM, JEST, TIFR and GRE in PHYSICAL SCIENCES 
 
Electronics
JEST-2016
Q1. It is found that when the resistance R indicated in the figure below is changed from
1 kΩ to 10 kΩ the current flowing through the resistance R′ does not change. What is
10 kΩ
the value of the resistor R′ ?

1k Ω R 1kΩ

R′

10 kΩ

5V
(a) 5 kΩ (b) 100 kΩ (c) 10 kΩ (d) 1 kΩ

R3 = 10 kΩ
Ans: (b)
Solution: Apply Wheatstone bridge condition R4 =
R1 = 1k Ω R 1 kΩ
R1 R3 1 10 • •
= ⇒ = A B
R2 R4 R′ 1
R2 =R ′

10 kΩ

5V
Q2. A transistor in common base configuration has ratio of collector current to emitter current
β and ratio of collector to base current α . Which of the following is true?

(a) β =
α
(b) β =
(α + 1)
(α + 1) α

(c) β =
α
(d) β =
(α − 1)
(α − 1) α
Ans: (a)
IE I 1 1 α
Solution: ∵ I E = I C + I B ⇒ = 1+ B ⇒ = 1+ ⇒ β =
IC IC β α 1+ α
                                                                                
Head office  Branch office 
 
fiziks, H.No. 40 D, G.F, Jia Sarai,  Anand Institute of Mathematics, 
 
Near IIT, Hauz Khas, New Delhi‐16  28‐B/6, Jia Sarai, Near IIT 
 
Phone: 011‐26865455/+91‐9871145498 Hauz Khas, New Delhi‐16 
                                                   
                                             Website: www.physicsbyfiziks.com                                                                                          
                                                           Email: fiziks.physics@gmail.com                                                                   1 
fiziks
Institute for NET/JRF, GATE, IIT‐JAM, JEST, TIFR and GRE in PHYSICAL SCIENCES 
 
JEST-2015
Q3. What is the voltage at the output of the following operational amplifier circuit. [See in the
10 k Ω
figure]?
(a) 1V −
1nA
+
(b) 1 mV +

RL
(c) 1 μV Vout

(d) 1 nV 99 kΩ
1 kΩ
Ans: (b)
Solution: Output of first Op-Amp v01 = − (10 × 103 )(1× 10−9 ) = −10−5 volt

⎛ 99 ⎞
Output of first Op-Amp vout = ⎜1 + ⎟ × 10−5 = 10−3 volts = 1 mV
⎝ 1 ⎠
Q4. The reference voltage of an analog to digital converter is 1 V . The smallest voltage step
that the converter can record using a 12 -bit converter is,
(a) 0.24V (b) 0.24 mV (c) 0.24μV (d) 0.24 nV
Ans: (b)
1
Smallest voltage step = ≈ 0.24 mV
2 −1
12

Q5. In Millikan’s oil drop experiment the electronic charge e could be written as kη 1.5 ,where
κ is a function of all experimental parameters with negligible error. If the viscosity of air
η is taken to be 0.4% lower than the actual value, what would be the error in the
calculated value of e ?
(a) 1.5% (b) 0.7% (c) 0.6% (d) 0.4%
Ans: (d)
Solution: Electronic charge is proportional to the viscosity i.e. e = Kη 1.5 = Kη 3/ 2
2
⎛ ∂e ⎞ 2
Now error in the measurement of charge is σ = ⎜ ⎟ ση
2

⎝ ∂η ⎠
e

                                                                                
Head office  Branch office 
 
fiziks, H.No. 40 D, G.F, Jia Sarai,  Anand Institute of Mathematics, 
 
Near IIT, Hauz Khas, New Delhi‐16  28‐B/6, Jia Sarai, Near IIT 
 
Phone: 011‐26865455/+91‐9871145498 Hauz Khas, New Delhi‐16 
                                                   
                                             Website: www.physicsbyfiziks.com                                                                                          
                                                           Email: fiziks.physics@gmail.com                                                                   2 
fiziks
Institute for NET/JRF, GATE, IIT‐JAM, JEST, TIFR and GRE in PHYSICAL SCIENCES 
 
⎛ ∂e ⎞ ∂e 3
⇒ σe = ⎜ ⎟ σ η where = Kη 1/ 2
⎝ ∂η ⎠ ∂η 2

⎛3 ⎞ 3 ση 3 ση
∴ σ e = ⎜ Kη 1/ 2 ⎟ σ η = Kη 3 / 2 = e
⎝2 ⎠ 2 η 2 η

σe 3 ση
⇒ =
e 2 η
ση
Given = 0.4%
η
σe 3
∴ = × 0.4% = 0.6% . Thus correct answer is option (c).
e 2

Q6. For the logic circuit shown in figure 5, the required input condition ( A, B, C ) to
make the output ( X ) = 1 is, A
U1
B
(a) 1,0,1 XOR U3 X
(b) 0,0,1
AND
U2
(c) 1,1,1 C
XNOR
(d) 0,1,1
Ans: (d)
Solution: XOR is inequality comparator and XNOR is equality comparator. In AND gate output
will be high when all the input is 1.

                                                                                
Head office  Branch office 
 
fiziks, H.No. 40 D, G.F, Jia Sarai,  Anand Institute of Mathematics, 
 
Near IIT, Hauz Khas, New Delhi‐16  28‐B/6, Jia Sarai, Near IIT 
 
Phone: 011‐26865455/+91‐9871145498 Hauz Khas, New Delhi‐16 
                                                   
                                             Website: www.physicsbyfiziks.com                                                                                          
                                                           Email: fiziks.physics@gmail.com                                                                   3 
fiziks
Institute for NET/JRF, GATE, IIT‐JAM, JEST, TIFR and GRE in PHYSICAL SCIENCES 
 
JEST-2014
Q7. Which of the following circuits will act like a 4-input NAND gate?
(a) (b)

(c) (d)

Ans.: (d)
Solution:
A AB AB
B ABC
C ABCD
D

A AB
B ABC
C
D ABCD

A AB
B
C ABCD
D CD

AB
A AB
B
ABCD
C
D CD
CD

                                                                                
Head office  Branch office 
 
fiziks, H.No. 40 D, G.F, Jia Sarai,  Anand Institute of Mathematics, 
 
Near IIT, Hauz Khas, New Delhi‐16  28‐B/6, Jia Sarai, Near IIT 
 
Phone: 011‐26865455/+91‐9871145498 Hauz Khas, New Delhi‐16 
                                                   
                                             Website: www.physicsbyfiziks.com                                                                                          
                                                           Email: fiziks.physics@gmail.com                                                                   4 
fiziks
Institute for NET/JRF, GATE, IIT‐JAM, JEST, TIFR and GRE in PHYSICAL SCIENCES 
 
F
Q8. The formula for normal strain in a longitudinal bar is given by ε = , where F is
AE
normal force applied, A is cross-sectional area of the bar and E is Young’s modulus. If
F = 50 ± 0.5 N , A = 0.2 ± 0.002 m 2 and E = 210 × 10 9 ± 1 × 10 9 Pa, the maximum error in
the measurement of strain is
(a) 1.0 × 10 −12 (b) 2.95 × 10 −11 (c) 1.22 × 10 −9 (d) 1.19 × 10 −9
Ans.: (b)
F
Solution: ∈=
AE
Δ ∈ ΔF ΔA ΔE 0.5 .002 1 × 10 9
= + + = + +
∈ F A E 50 0.2 210 × 10 9
Δ∈ 0.2476 × 50
= .02476 ⇒ Δ ∈= 0.2476×∈ = = 2.95 ×10−11
∈ 0.2 × 210 ×109
Q9. A 100 ohms resistor carrying current of 1 Amp is maintained at a constant temperature of
30 o C by a heat bath. What is the rate of entropy increase of the resistor?
(a) 3.3 Joules/K/sec (b) 6.6 Joules/K/sec
(c) 0.33 Joules/K/sec (d) None of the above
Ans.: (c)
Solution: ω = q.V ⇒ W = i ⋅ t ⋅ R

W = i2 R t

∂W i 2 R t 1× 100
now = = = 0.33
∂T T 30 + 273

JEST-2012
Q10. The ratio of maximum to minimum resistance that can be obtained with N 1-Ω resistors is
(a) N (b) N2 (c) 1 (d) ∞
Ans.: (b)
Solution: resistance in series is maximum and minimum in parallel
Rs = 1 + 1 + 1 + 1 + .....N = N
                                                                                
Head office  Branch office 
 
fiziks, H.No. 40 D, G.F, Jia Sarai,  Anand Institute of Mathematics, 
 
Near IIT, Hauz Khas, New Delhi‐16  28‐B/6, Jia Sarai, Near IIT 
 
Phone: 011‐26865455/+91‐9871145498 Hauz Khas, New Delhi‐16 
                                                   
                                             Website: www.physicsbyfiziks.com                                                                                          
                                                           Email: fiziks.physics@gmail.com                                                                   5 
fiziks
Institute for NET/JRF, GATE, IIT‐JAM, JEST, TIFR and GRE in PHYSICAL SCIENCES 
 
1 1 1 1 1 1 1
= + + + + ..... = N
Rp 1 1 1 1 1 N
Rs 1
= N × N = N 2 ⇒ Rp =
Rp N
Q11. The net charge of an n-type semiconductor is
(a) positive (b) zero (c) negative (d) dependent
Ans.: (b)

                                                                                
Head office  Branch office 
 
fiziks, H.No. 40 D, G.F, Jia Sarai,  Anand Institute of Mathematics, 
 
Near IIT, Hauz Khas, New Delhi‐16  28‐B/6, Jia Sarai, Near IIT 
 
Phone: 011‐26865455/+91‐9871145498 Hauz Khas, New Delhi‐16 
                                                   
                                             Website: www.physicsbyfiziks.com                                                                                          
                                                           Email: fiziks.physics@gmail.com                                                                   6 
fiziks
Institute for NET/JRF, GATE, IIT‐JAM, JEST, TIFR and GRE in PHYSICAL SCIENCES 
 
Solid State Physics
JEST-2016

Q1. If k is the wavevector of incident light ( k = , λ is the wavelength of light) and G is
λ
a reciprocal lattice vector, then the Bragg’s law can be written as:
(a) k + G = 0 (b) 2k .G + G 2 = 0
(c) 2k .G + k 2 = 0 (d) k .G = 0
Ans. : (b)
Solution: By means of Eward construction we can write the Bragg’s law in B
vector form K
A G
G = OB, K ′ = AO
K
For diffraction it is necessary that vector K ′ + G , that is vector AB O

be equal in magnitude to the vector K or

(K + G) = K 2 ⇒ 2K ⋅ G + G 2 = 0
2

Q2. The number of different Bravais lattices possible in two dimensions is:
(a) 2 (b) 3 (c) 5 (d) 6
Ans. : (c)
Solution: Five Bravais lattices in 2D are:
(i) square lattice
(ii) Rectangular ( P ) lattice

(iii) Rectangular ( C ) lattice

(iv) Hexagonal lattice


(v) Oblique lattice

                                                                                
Head office  Branch office 
 
fiziks, H.No. 40 D, G.F, Jia Sarai,  Anand Institute of Mathematics, 
 
Near IIT, Hauz Khas, New Delhi‐16  28‐B/6, Jia Sarai, Near IIT 
 
Phone: 011‐26865455/+91‐9871145498 Hauz Khas, New Delhi‐16 
                                                   
                                             Website: www.physicsbyfiziks.com                                                                                          
                                                           Email: fiziks.physics@gmail.com                                                                   1 
fiziks
Institute for NET/JRF, GATE, IIT‐JAM, JEST, TIFR and GRE in PHYSICAL SCIENCES 
 
JEST-2015
Q3. For a 2 - dimensional honeycomb lattice as shown in the figure 3, the first Bragg spot
occurs for the grazing angle θ 1 while sweeping the angle from 0 o . The next Bragg spot is
obtained at θ 2 given by A

⎛3 ⎞ a 120 o
(a) sin −1 (3 sin θ1 ) (b) sin −1 ⎜ sin θ1 ⎟ 120 o
⎝2 ⎠ B
120 o
⎛ 3
(c) sin −1 ⎜⎜

sin θ1 ⎟⎟ (
(d) sin −1 3 sin θ1 )
⎝ 2 ⎠
Ans: (c)
Solution: According to Bragg’s law, the condition for first Bragg spot and second spot is
2d1 sin θ1 = nλ and 2d 2 sin θ 2 = nλ
⎛d ⎞
∴2d1 sin θ1 = 2d 2 sin θ 2
⇒ θ 2 = sin −1 ⎜ 1 sin θ1 ⎟
⎝ d2 ⎠
For 2 - dimensional honeycomb lattice, the lattice constant ‘ a ’ and interplanar spacing
‘ d ’ is linked as
2
⎛a⎞ a2 3 a
d = a − ⎜ ⎟ ⇒ d1 = a −
1
2 2 2
= a and d 2 = a d1
⎝2⎠ 4 2
0
60
⎛ 3 ⎞
∴θ 2 = sin ⎜⎜ sin θ1 ⎟⎟
−1 a
⎝ 2 ⎠
−L L
Q4. A particle of mass m is confined in a potential well given by V ( x ) = 0 for <x<
2 2
L/2 and V ( x ) = ∞ elsewhere. A perturbing potential H ′( x ) = ax has been applied to the

system. Let the first and second order corrections to the ground state be E 0(1) and E0(2 ) ,
respectively. Which one of the following statements is correct?
(a) E 0(1) < 0 and E 0(2 ) > 0 (b) E 0(1) = 0 and E 0(2 ) > 0

(c) E 0(1) > 0 and E (02 ) < 0 (d) E0(1) = 0 and E (02) < 0
Ans: (d)

                                                                                
Head office  Branch office 
 
fiziks, H.No. 40 D, G.F, Jia Sarai,  Anand Institute of Mathematics, 
 
Near IIT, Hauz Khas, New Delhi‐16  28‐B/6, Jia Sarai, Near IIT 
 
Phone: 011‐26865455/+91‐9871145498 Hauz Khas, New Delhi‐16 
                                                   
                                             Website: www.physicsbyfiziks.com                                                                                          
                                                           Email: fiziks.physics@gmail.com                                                                   2 
fiziks
Institute for NET/JRF, GATE, IIT‐JAM, JEST, TIFR and GRE in PHYSICAL SCIENCES 
 
⎧0 − L / 2 < x < +L / 2
Solution: V ( x ) = ⎨ and H ′ ( x ) = α x
⎩∞ elsewhere

2 πx
For ground state φ0 = cos
L L
φ0 H ′ φ0 2 L/2 πx
E0(1) = = α ∫ x cos 2 =0
φ0 φ0 L − L/2 L
2
φm H ′ φ0
E0 = ∑
( 2)
⇒ E0( ) < 0 ∵ E00 < Em0
2

m≠0 E −E
0
0
0
m

⎛ ka ⎞
Q5. Given the tight binding dispersion relation E (k ) = E 0 + A sin 2 ⎜ ⎟ , where E 0 and A are
⎝ 2⎠
constants and a is the lattice parameter. What is the group velocity of an electron at the
second Brillouin zone boundary?
a 2a a
(a) 0 (b) (c) (d)
h h 2h
Ans: (a)
1 dE
Solution: Group velocity is defined as vg =
dk
⎛ ka ⎞ dE ⎛ ka ⎞ ⎛ ka ⎞ aA
since E = E0 + A sin 2 ⎜ ⎟ ⇒ = aA sin ⎜ ⎟ cos ⎜ ⎟ = sin ka
⎝ 2⎠ dk ⎝ 2 ⎠ ⎝ 2 ⎠ 2
In one dimension, the Brillouin zone boundary is
π
The 1st Brillouin zone boundaries lie at ±
π 2π
− 2aπ − πa
a 0
2 π K a a
The 2nd Brillouin zone boundaries lie at ±
a
Thus, the group velocity at the second Brillouin zone boundary is
aA ⎛ 2π ⎞ aA
vg 2π = sin ⎜ × a⎟ = sin 2π ⇒ vg = 0
±
a 2 ⎝ a ⎠ 2

                                                                                
Head office  Branch office 
 
fiziks, H.No. 40 D, G.F, Jia Sarai,  Anand Institute of Mathematics, 
 
Near IIT, Hauz Khas, New Delhi‐16  28‐B/6, Jia Sarai, Near IIT 
 
Phone: 011‐26865455/+91‐9871145498 Hauz Khas, New Delhi‐16 
                                                   
                                             Website: www.physicsbyfiziks.com                                                                                          
                                                           Email: fiziks.physics@gmail.com                                                                   3 
fiziks
Institute for NET/JRF, GATE, IIT‐JAM, JEST, TIFR and GRE in PHYSICAL SCIENCES 
 
+ −
Q6. The total number of Na and Cl ions per unit cell of NaCl is,
(a) 2 (b) 4 (c) 6 (d) 8
Ans: (d)
Solution: Total number of Na + and Cl − ions per unit ( d ) is

1 1 1
N Cl− = nc + n f , N Na
+
= ne + 1 × ni
8 2 4
where nc = number of ions at corner
n f = number of ions at face
ne = number of ions at edges
ni = number of ions inside

1 1 1 Cl −
N =N +N −
Cl
+
Na = × 8 + × 6 + × 12 + 1× 1 = 1 + 3 + 3 + 1 = 8 Na +
8 2 4
JEST-2014
Q7. Circular discs of radius 1 m each are placed on a plane so as to form a closely packed
triangular lattice. The number of discs per unit area is approximately equal to
(a) 0.86 m −2 (b) 0.43 m −2 (c) 0.29 m −2 (d) 0.14 m −2
Ans.: (c)
Solution: For closely packed hexagonal
1 1
a = 2r , r = 1 neff = × nC + × n f + 1 × nl
6 2
1 1
⇒ neff = × 3 + 0 × + 1 × 0 ⇒ neff = 0.5
6 2
neff × π r 2
Occupancy = (∵ a = 2r )
A
Closely packed hexagonal
0.5 × π r 2 0.5 × π
⇒ = = 0.9064
3 3
× ( 2r )
2

4
0.9064
Now number of disc per unit area will be = .302 ≈ 0.29
3

                                                                                
Head office  Branch office 
 
fiziks, H.No. 40 D, G.F, Jia Sarai,  Anand Institute of Mathematics, 
 
Near IIT, Hauz Khas, New Delhi‐16  28‐B/6, Jia Sarai, Near IIT 
 
Phone: 011‐26865455/+91‐9871145498 Hauz Khas, New Delhi‐16 
                                                   
                                             Website: www.physicsbyfiziks.com                                                                                          
                                                           Email: fiziks.physics@gmail.com                                                                   4 
fiziks
Institute for NET/JRF, GATE, IIT‐JAM, JEST, TIFR and GRE in PHYSICAL SCIENCES 
 
Q8. An ideal gas of non-relativistic fermions in 3-dimensions is at 0K. When both the number
density and mass of the particles are doubled, then the energy per particle is multiplied by
a factor
(a) 21 / 2 (b) 1 (c) 21 / 3 (d) 2 −1 / 3
Ans.: (d)
2
h 2 ⎛ 3n ⎞ 3
Solution: E F = ⎜ ⎟ at T = 0 K
2m ⎝ 4π ⎠
2 2 2
−1
h2 ⎛ 3 ⎞ 3 h ⎛ 3n ⎞ 3 1 3 h ⎛ 3n ⎞ 3
2 2 2
∵ n ′ = 2n and m′ = 2m ⇒ EF′ = ⎜ × 2 n ⎟ = ⎜ ⎟ × × 2 = ⎜ ⎟ × 2 3
4m ⎝ 4π ⎠ 2 m ⎝ 4π ⎠ 2 2 m ⎝ 4π ⎠
Q9. When two different solids are brought in contact with each other, which one of the
following is true?
(a) Their Fermi energies become equal
(b) Their band gaps become equal
(c) Their chemical potentials become equal
(d) Their work functions become equal
Ans.: (c)

JEST-2013
Q10. A flat surface is covered with non-overlapping disks of same size. What is the largest
fraction of the area that can be covered?
3 5π 6 π
(a) (b) (c) (d)
π 6 7 2 3
Ans.: (d)
1 1 1
Solution: neff = nC + n f + 1× ni = × 6 + 1 = 3 a = 2r
3 2 3
neff × A 3 × πr 2 π
Now largest fraction of area i.e. packing fraction = = =
× (2r )
3 3 2 3
6× × a2 6×
2

4 4

                                                                                
Head office  Branch office 
 
fiziks, H.No. 40 D, G.F, Jia Sarai,  Anand Institute of Mathematics, 
 
Near IIT, Hauz Khas, New Delhi‐16  28‐B/6, Jia Sarai, Near IIT 
 
Phone: 011‐26865455/+91‐9871145498 Hauz Khas, New Delhi‐16 
                                                   
                                             Website: www.physicsbyfiziks.com                                                                                          
                                                           Email: fiziks.physics@gmail.com                                                                   5 
fiziks
Institute for NET/JRF, GATE, IIT‐JAM, JEST, TIFR and GRE in PHYSICAL SCIENCES 
 
Q11. A metal suffers a structural phase transition from face-centered cubic ( FCC ) to the

simple cubic ( SC ) structure. It is observed that this phase transition does not involve any

change of volume. The nearest neighbor distances d fc and d SC for the FCC and the SC

⎛ d fc ⎞ 1
structures respectively are in the ratio ⎜ ⎟ [Given 2 3
= 1.26 ]
⎝ SC ⎠
d

(a) 1.029 (b) 1.122 (c) 1.374 (d) 1.130


Ans. 1: ()
Solution: Nearest neighbour in SC → a → C.N = 6
a
Nearest neighbour in FCC → → C.N = 12
2
a
dFCC 1 1
= 2 = = = 0.707
dSC a 2 1.414
JEST-2012
Q12. A beam of X-rays is incident on a BCC crystal. If the difference between the incident and
scatered wavevectors is K = xˆ + kyˆ + lzˆ where xˆ , yˆ , zˆ are the unit vectors of the
associated cubic lattice, the necessary condition for the scattered beam to give a Laue
maximum is
(a) h + k + l = even (b) h + k + l = even
(c) h, k, l are all distinct (d) h + k + l = odd
Ans.: (a)
⎛1 1⎞
Solution: In BCC basis (0, 0, 0), ⎜ ,
1
, ⎟
⎝2 2 2⎠
Crystal structure factor F
neff

F = f (S ) ⇒ S = ∑e
2π i ( hun + kVn + lωn )

n =1

⎡ ⎛ 1 ⎞⎤
F = f ⎢e 2π i (0) + e 2π i ⎜ ⎟⎥[h + k + l ]
⎣ ⎝ 2 ⎠⎦
                                                                                
Head office  Branch office 
 
fiziks, H.No. 40 D, G.F, Jia Sarai,  Anand Institute of Mathematics, 
 
Near IIT, Hauz Khas, New Delhi‐16  28‐B/6, Jia Sarai, Near IIT 
 
Phone: 011‐26865455/+91‐9871145498 Hauz Khas, New Delhi‐16 
                                                   
                                             Website: www.physicsbyfiziks.com                                                                                          
                                                           Email: fiziks.physics@gmail.com                                                                   6 
fiziks
Institute for NET/JRF, GATE, IIT‐JAM, JEST, TIFR and GRE in PHYSICAL SCIENCES 
 
π i( h+ k +l )
F = f ⎡⎣1 + e ⎤

now for plane (1 1 0 )

F110 = f (2) ⇒ I = 4 f 2

F111 = 0

F200 = 2 f ⇒ I = f 2

i.e h + k + l = even then plane will be present and if h + k + l = odd then plane will be
absent.
Q13. The second order maximum in the diffraction of X-rays of 0.20 nanometer wavelength
from a simple cubic crystal is found to occur at an angle of thirty degrees to the crystal
plane. The distance between the lattice planes is
(a) 1 Angstrom (b) 2 Angstrom (c) 4 Angstrom (d) 8 Angstrom
Ans.: (c)
Solution: 2d sin θ = nλ
2d sin θ = 2λ
2 × d × sin 30 o = 2 × 0.2 × 10 −9 m
d = 2 × 0.2 × 10 −9 m
d = 0.4 × 10 −9 m ⇒ 4 × 10 −10 m = 4 A o
Q14. The Dulong –Petit law fails near room temperature (300 K) for many light elements (such
as boron and beryllium) because their Debye temperature is
(a) >> 300 K (b) ~ 300 K (c) << 300 K (d) 0 K
Ans.: (a)

                                                                                
Head office  Branch office 
 
fiziks, H.No. 40 D, G.F, Jia Sarai,  Anand Institute of Mathematics, 
 
Near IIT, Hauz Khas, New Delhi‐16  28‐B/6, Jia Sarai, Near IIT 
 
Phone: 011‐26865455/+91‐9871145498 Hauz Khas, New Delhi‐16 
                                                   
                                             Website: www.physicsbyfiziks.com                                                                                          
                                                           Email: fiziks.physics@gmail.com                                                                   7 
fiziks
Institute for NET/JRF, GATE, IIT‐JAM, JEST, TIFR and GRE in PHYSICAL SCIENCES 
 
Atomic and Molecular Physics
JEST 2016
Q1. The H 2 molecule has a reduced mass M = 8.35 × 10−28 kg and an equilibrium

internuclear distance R = 0.742 × 10−10 m . The rotational energy in terms of the rotational
quantum number J is
5
(a) Erot ( J ) = 7 J ( J − 1) meV (b) Erot ( J ) = J ( J + 1) meV
2
5
(c) Erot ( J ) = 7 J ( J + 1) meV (d) Erot ( J ) = J ( J − 1) meV
2
Ans. : (c)
2
Solution: E = J ( J + 1)
2I
where

I = μ r 2 = 8.35 × 10−28 kg × ( 0.742 × 10−10 m ) = 4.597 × 10−48 kgm 2


2

(1.05 ×10 J ⋅ s ) = 1.112 ×10


−34 2
2 −68
∴ =
2 I 2 × ( 4.597 ×10 kgm ) 9.18 × 10 −48 2 −48

1
= 1.21 × 10−21 J = 1.21 × 10−21 × eV
1.6 × 10−19
= 7.57 × 10−3 eV = 7.57 meV

∴ E ≅ 7 J ( J + 1) meV

Q2. If the Rydberg constant of an atom of finite nuclear mass is α R∞ , where R∞ the Rydberg
con stant corresponding to an infinite nuclear mass, the ratio of the electronic to nuclear
mass of the atom is:-

(a)
(1 − α ) (b)
(α − 1) (c) (1 − α ) (d)
1
α α α
Ans. : (a)

                                                                                
Head office  Branch office 
 
fiziks, H.No. 40 D, G.F, Jia Sarai,  Anand Institute of Mathematics, 
 
Near IIT, Hauz Khas, New Delhi‐16  28‐B/6, Jia Sarai, Near IIT 
 
Phone: 011‐26865455/+91‐9871145498 Hauz Khas, New Delhi‐16 
                                                   
                                             Website: www.physicsbyfiziks.com                                                                                          
                                                           Email: fiziks.physics@gmail.com                                                                   1 
fiziks
Institute for NET/JRF, GATE, IIT‐JAM, JEST, TIFR and GRE in PHYSICAL SCIENCES 
 
R∞
Solution: RM =
m
1+ c
M
1 m 1 m 1 1−α
∴α = ⇒ 1+ c = ⇒ c = −1 =
m
1+ c M α M α α
M
JEST 2015
Q3. The energy difference between the 3 p and 3s levels in Na is 2.1 eV . Spin-orbit
o
coupling splits the 3 p level, resulting in two emission lines differing by 6 A . The
splitting of the 3p level is approximately,
(a) 2 eV (b) 0.2 eV (c) 0.02 eV (d) 2 meV
Ans: (d)
Solution: The fine structure splitting of Na in ground and excited state is
32 p3 / 2
3p
32 p1/ 2
2.1eV
λ1 λ2
3s 32 s1/ 2
The transition 3 p3/ 2 → 3 s1/ 2 produces photon of wavelength λ2 and corresponding
2 2

12400
photon energy is E2 = eV
⎛0⎞
λ2 ⎜ Α ⎟
⎝ ⎠
The transition 3 p1/ 2 → 32 s1/ 2 produces photon of wavelength λ1 and corresponding
2

12400
photon energy is E1 = eV .The separation between 2 p3/ 2 and 2 p1/ 2 is
λ1 Λ( )
0

12400 12400 ⎛1 1 ⎞ ⎛ λ −λ ⎞
ΔE = E2 − E1 =− = 12400 ⎜ − ⎟ = 12400 ⎜ 1 2 ⎟
λ2 λ1 ⎝ λ1 λ2 ⎠ ⎝ λ1λ2 ⎠
Given Δλ = λ1 − λ2 = 6A also fine structure splitting is of the order of 10−3 eV .
0

                                                                                
Head office  Branch office 
 
fiziks, H.No. 40 D, G.F, Jia Sarai,  Anand Institute of Mathematics, 
 
Near IIT, Hauz Khas, New Delhi‐16  28‐B/6, Jia Sarai, Near IIT 
 
Phone: 011‐26865455/+91‐9871145498 Hauz Khas, New Delhi‐16 
                                                   
                                             Website: www.physicsbyfiziks.com                                                                                          
                                                           Email: fiziks.physics@gmail.com                                                                   2 
fiziks
Institute for NET/JRF, GATE, IIT‐JAM, JEST, TIFR and GRE in PHYSICAL SCIENCES 
 
Thus λ1 and λ2 are approximately same as λ corresponds to 3 p → 3s , whereas

12400 0
wavelength ( λ ) corresponding to 3 p → 3s transition is λ= A
2.1
2
⎛ 12400 ⎞
Thus, λ1λ2 ≅ λ 2 = ⎜ ⎟
⎝ 2.1 ⎠
6 2.1× 2.1× 6
∴ ΔE = 12400 × 2
= ≅ 2 × 10−3 eV ⇒ ΔE = 2 meV
⎛ 12400 ⎞ 12400
⎜ ⎟
⎝ 2.1 ⎠

Q4. Which of the following excited states of a hydrogen atom has the highest lifetime?
(a) 2 p (b) 2 s (c) 3s (d) 3 p
Ans: (b)
Solution: 2 p and 3 p are normal states. Electron from 2 p and 3 p make transition to ground

state within 10−9 sec . Electron in 3s state although can not come directly to ground state
but it can come to 1s through 2 p as 3s → 2 p → 1s , while 2s is metastable state and
electron in 2s state can make transition to 1s slowly. Thus 2s has long life time.

Q5. Which of the following statements is true for the energies of the terms of the carbon atom
in the ground state electronic configuration 1s 2 2s 2 2 p 2 ?

(a) 3 P <1 D <1 S (b) 3 P <1 S <1 D


(c) 3 P <1 F <1 S (d) 3 P <1 F <1 D
Ans: (a)
Solution: The spectroscopy terms for p 2 are 1 S0 , 1 D2 , 3 P1, 2,3 . According to Hund’s rule, state with

highest multiplicity lies lowest. Then, out of same multiplicity, state with highest L lies
lowest. Thus these terms can be arranged as 3P < 1D < 1S

                                                                                
Head office  Branch office 
 
fiziks, H.No. 40 D, G.F, Jia Sarai,  Anand Institute of Mathematics, 
 
Near IIT, Hauz Khas, New Delhi‐16  28‐B/6, Jia Sarai, Near IIT 
 
Phone: 011‐26865455/+91‐9871145498 Hauz Khas, New Delhi‐16 
                                                   
                                             Website: www.physicsbyfiziks.com                                                                                          
                                                           Email: fiziks.physics@gmail.com                                                                   3 
fiziks
Institute for NET/JRF, GATE, IIT‐JAM, JEST, TIFR and GRE in PHYSICAL SCIENCES 
 
JEST 2014
Q6. The value of elastic constant for copper is about 100 Nm −1 and the atomic spacing is
0.256 nm . What is the amplitude of the vibration of the Cu atoms at 300 K as a
percentage of the equilibrium separation?
(a) 4.55 % (b) 3.55 % (c) 2.55 % (d) 1.55 %
Ans.: (b)
Solution: we know that
1
E= β A2 a = 0.256 × 10 −9 m
2
for one dimension
1 1
K .E = K B T and P.E = KT
2 2
1 2 k BT
E = kT now K B T = β A2 ⇒ A =
2 β

2k β T 2 × 1.30 × 10−23 × 300


A= ⇒ A= = 8.28 × 10−23 = 9.09 × 10−12 = 0.0090 nm
β 100
Now x of 0.256nm = 0.009nm
0.00909nm
x= = 0.03551 = 3.5%
0.256nm
Q7. Which functional form of potential best describes the interaction between a neutral atom
and an ion at large distances (i.e. much larger than their diameters)
(a) V ∝ −1 / r 2 (b) V ∝ −1 / r (c) V ∝ −e − r / a / r (d) V ∝ −1 / r 3
Ans.: (a)

                                                                                
Head office  Branch office 
 
fiziks, H.No. 40 D, G.F, Jia Sarai,  Anand Institute of Mathematics, 
 
Near IIT, Hauz Khas, New Delhi‐16  28‐B/6, Jia Sarai, Near IIT 
 
Phone: 011‐26865455/+91‐9871145498 Hauz Khas, New Delhi‐16 
                                                   
                                             Website: www.physicsbyfiziks.com                                                                                          
                                                           Email: fiziks.physics@gmail.com                                                                   4 
fiziks
Institute for NET/JRF, GATE, IIT‐JAM, JEST, TIFR and GRE in PHYSICAL SCIENCES 
 
Q8. If a proton were ten times lighter, then the ground state energy of the electron in a
hydrogen atom would have been
(a) Less (b) More
(c) The same (d) Depends on the electron mass
Ans.: (b)
− 13.6 μ − 13.6 1.00545me
Solution: E n = 2
= × = −13.526
n me n2 me

Mp me ⋅ m p
∵M p = μ= = 1.00545 me
10 me + m p

Q9. If hydrogen atom is bombarded by energetic electrons, it will emit


(a) K α X - rays (b) β -rays
(c) Neutrons (d) none of the above
Ans.: (d)
JEST 2013
Q10. A sodium atom in the first excited 3P states has a lifetime of 16ns for decaying to the
ground 3S state. The wavelength of the emitted photon is 589 nm. The corresponding line
width of the transition (in frequency units) is about
(a) 1.7 x 106 Hz (b) 1 x 107 Hz
(c) 6.3 x 107 Hz (d) 5 x 1014 Hz
Ans.: (c)
Solution: ΔE ⋅ Δt =
Δf ⋅ Δt =
1
Δf =
Δt
1
= Hz
16 × 109
6.25 × 107 Hz = 6.3 × 107 Hz

                                                                                
Head office  Branch office 
 
fiziks, H.No. 40 D, G.F, Jia Sarai,  Anand Institute of Mathematics, 
 
Near IIT, Hauz Khas, New Delhi‐16  28‐B/6, Jia Sarai, Near IIT 
 
Phone: 011‐26865455/+91‐9871145498 Hauz Khas, New Delhi‐16 
                                                   
                                             Website: www.physicsbyfiziks.com                                                                                          
                                                           Email: fiziks.physics@gmail.com                                                                   5 
fiziks
Institute for NET/JRF, GATE, IIT‐JAM, JEST, TIFR and GRE in PHYSICAL SCIENCES 
 
Nuclear & Particle Physics
JEST-2016
Q1. The half-life of a radioactive nuclear source is 9 days. The fraction of nuclei which are
left under cayed after 3 days is:
7 1 5 1
(a) (b) (c) (d) 1
8 3 6
23
Ans: (d)
n 3/ 9
⎛1⎞ ⎛1⎞ N 1
Solution: N = N 0 ⎜ ⎟ = N 0 ⎜ ⎟ ⇒ = 1/ 3
⎝2⎠ ⎝2⎠ N0 2

JEST-2015
1 189
Q2. The stable nucleus that has the radius of Os nucleus is,
3
16
(a) Li (b) O (c) 4 He (d) 14
N
Ans: (a)
1 1
Solution: R = ROs ⇒ R0 ( A ) = R0 (189 ) ⇒ A = 7
1/ 3 1/ 3

3 3
Q3. The reaction e + + e − → γ is forbidden because,
(a) lepton number is not conserved
(b) linear momentum is not conserved
(c) angular momentum is not conserved
(d) charge is not conserved
Ans: (b)
Solution: In order to conserve linear momentum two photons are required that move in opposite
direction.

                                                                                
Head office  Branch office 
 
fiziks, H.No. 40 D, G.F, Jia Sarai,  Anand Institute of Mathematics, 
 
Near IIT, Hauz Khas, New Delhi‐16  28‐B/6, Jia Sarai, Near IIT 
 
Phone: 011‐26865455/+91‐9871145498 Hauz Khas, New Delhi‐16 
                                                   
                                             Website: www.physicsbyfiziks.com                                                                                          
                                                           Email: fiziks.physics@gmail.com                                                                   1 
fiziks
Institute for NET/JRF, GATE, IIT‐JAM, JEST, TIFR and GRE in PHYSICAL SCIENCES 
 
JEST-2014
238
Q4. In the mixture of isotopes normally found on the earth at the present time, 92 U has an
235
abundance of 99.3% and 92 U has an abundance of 0.7%. The measured lifetimes of

these isotopes are 6.52 × 10 9 years and 1.02 × 10 9 years, respectively. Assuming that they
were equally abundant when the earth was formed, the estimated age of the earth, in
years is
(a) 6.0 × 10 9 (b) 1.0 × 10 9 (c) 6.0 × 10 8 (d) 1.0 × 10 8
Ans.: (a)
92
Solution: If the number of U 238 nuclei originally formed is N , the number present now is
N 238 = Ne−t / T = Ne−t / 6.52

where t is elapsed time in units of 109 year and T is life time of U . Since the number of
92
U 235 nuclei originally formed is. The number now present is
N 235 = Ne− t /1.02
92
The present abundance of U 235 is
N 235 N Ne − t /1.02 1 4.96
7 ×10−3 = ≈ 235 = − t / 6.52
= e0.827 t ≈ −3
= 143 = t = = 6.0
N 238 + N 235 N 238 Ne 7 × 10 0.827

That is, the elapsed time is t = 6.0 × 109 yr.


JEST-2013
Q5. 238
U decays with a half life of 4.51×1019 years, the decay series eventually ending at
206
Pb , which is stable. A rock sample analysis shows that the ratio of the numbers of
206 238 206
atoms of Pb to U is 0.0058 . Assuming that all the Pb has been produced by the
238
decay of U and that all other half-lives in the chain are negligible, the age of the rock
sample is
(a) 38 × 106 years (b) 48 × 106 years (c) 38 ×107 years (d) 48 × 107 years
Ans.: (a)

1 ⎛ N pb + N u ⎞
Solution: t = ln ⎜ ⎟
λu ⎝ N u ⎠
                                                                                
Head office  Branch office 
 
fiziks, H.No. 40 D, G.F, Jia Sarai,  Anand Institute of Mathematics, 
 
Near IIT, Hauz Khas, New Delhi‐16  28‐B/6, Jia Sarai, Near IIT 
 
Phone: 011‐26865455/+91‐9871145498 Hauz Khas, New Delhi‐16 
                                                   
                                             Website: www.physicsbyfiziks.com                                                                                          
                                                           Email: fiziks.physics@gmail.com                                                                   2 
fiziks
Institute for NET/JRF, GATE, IIT‐JAM, JEST, TIFR and GRE in PHYSICAL SCIENCES 
 
Q6. The binding energy of the k -shell electron in a Uranium atom ( Z = 92, A = 238 ) will be

modified due to (i) screening caused by other electrons and (ii) the finite extent of the
nucleus as follows:
(a) increases due to (i), remains unchanged due to (ii)
(b) decreases due to (i), decreases due to (ii)
(c) increases due to (i), increases due to (ii)
(d) decreases due to (i), remains unchanged due to (ii)
Ans.: (b)

                                                                                
Head office  Branch office 
 
fiziks, H.No. 40 D, G.F, Jia Sarai,  Anand Institute of Mathematics, 
 
Near IIT, Hauz Khas, New Delhi‐16  28‐B/6, Jia Sarai, Near IIT 
 
Phone: 011‐26865455/+91‐9871145498 Hauz Khas, New Delhi‐16 
                                                   
                                             Website: www.physicsbyfiziks.com                                                                                          
                                                           Email: fiziks.physics@gmail.com                                                                   3 
fiziks
Institute for NET/JRF, GATE, IIT‐JAM, JEST, TIFR and GRE in PHYSICAL SCIENCES 
 
Optics
JEST 2015
Q1. Let λ be the wavelength of incident light. The diffraction pattern of a circular aperture of
dimension r0 will transform from Fresnel to Fraunhofer regime if the screen distance z

is,
r02 λ2 λ2 r02
(a) z >> (b) z >> (c) z << (d) z <<
λ r0 r0 λ
Ans: (a)
Solution: Fraunhofer made an approximation on the quadratic phase function: Y

i
(
k x02 + y02 ) i
kr02
e 2z
=e 2z
≈1
X
kr02 π r02 r02
If z >> ⇒ z >> ⇒ z >> r0 = aperture
2 λ λ
For this reason Fraunhofer diffraction is also called Far-field diffraction, whereas for
Fresnel diffraction, the condition is
z >> λ called near-field diffraction.

                                                                                
Head office  Branch office 
 
fiziks, H.No. 40 D, G.F, Jia Sarai,  Anand Institute of Mathematics, 
 
Near IIT, Hauz Khas, New Delhi‐16  28‐B/6, Jia Sarai, Near IIT 
 
Phone: 011‐26865455/+91‐9871145498 Hauz Khas, New Delhi‐16 
                                                   
                                             Website: www.physicsbyfiziks.com                                                                                          
                                                           Email: fiziks.physics@gmail.com                                                                   1 
fiziks
Institute for NET/JRF, GATE, IIT‐JAM, JEST, TIFR and GRE in PHYSICAL SCIENCES 
 
JEST 2014

Q2. A spherical air bubble is embedded in a glass slab. It will behave like a
(a) Cylindrical lens (b) Achromatic lens (c) Converging lens (d) Diverging lens
Ans.: (c)
Q3. The resolving power of a grating spectrograph can be improved by
(a) recording the spectrum in the lowest order
(b) using a grating with longer grating period
(c) masking a part of the grating surface
(d) illuminating the grating to the maximum possible extent
Ans.: (d)
Δλ
Solution: ⇒ R ⋅ P = = nN where N - Number of slit and n - order of diffraction.
λ
Q4. Three sinusoidal waves have the same frequency with amplitude A, A / 2 and A / 3 while
their phase angles are 0, π / 2 and π respectively. The amplitude of the resultant wave is
11A 2A 5A 7A
(a) (b) (c) (d)
6 3 6 6
Ans.: (c)
A ⎛ π⎞
Solution: y1 = A sin (ωt + 0), sin ⎜ ωt + ⎟ , y3 = sin (ωt + π )
A
y2 =
2 ⎝ 2⎠ 3
A A
y = y1 + y 2 + y 3 = A sin ωt + cos ωt − sin ωt
2 3
2A A
⇒ sin ωt + cos ωt
3 2
2 2
⎛ 2A ⎞ ⎛ A ⎞ 4 A2 A2 25 A 2 5 A
A′ = ⎜ ⎟ +⎜ ⎟ = + = =
⎝ 3 ⎠ ⎝2⎠ 9 4 36 6

                                                                                
Head office  Branch office 
 
fiziks, H.No. 40 D, G.F, Jia Sarai,  Anand Institute of Mathematics, 
 
Near IIT, Hauz Khas, New Delhi‐16  28‐B/6, Jia Sarai, Near IIT 
 
Phone: 011‐26865455/+91‐9871145498 Hauz Khas, New Delhi‐16 
                                                   
                                             Website: www.physicsbyfiziks.com                                                                                          
                                                           Email: fiziks.physics@gmail.com                                                                   2 
fiziks
Institute for NET/JRF, GATE, IIT‐JAM, JEST, TIFR and GRE in PHYSICAL SCIENCES 
 
JEST 2013
Q5. The equation describing the shape of curved mirror with the property that the light from a
point source at the origin will be reflected in a beam of rays parallel to the x -axis is (with
a as some constant)
(a) y2 = ax + a2 (b) 2y = x2 + a2 (c) y2 = 2ax + a2 (d) y2 = ax3 + 2a2
Ans.: (c)

                                                                                
Head office  Branch office 
 
fiziks, H.No. 40 D, G.F, Jia Sarai,  Anand Institute of Mathematics, 
 
Near IIT, Hauz Khas, New Delhi‐16  28‐B/6, Jia Sarai, Near IIT 
 
Phone: 011‐26865455/+91‐9871145498 Hauz Khas, New Delhi‐16 
                                                   
                                             Website: www.physicsbyfiziks.com                                                                                          
                                                           Email: fiziks.physics@gmail.com                                                                   3 
fiziks
Forum for CSIR-UGC JRF/NET, GATE, IIT-JAM, GRE in PHYSICAL SCIENCES

IIT-JAM Physics Syllabus


Mathematical Methods
Calculus of single and multiple variables, partial derivatives, Jacobian, imperfect and
perfect differentials, Taylor expansion, Fourier series. Vector algebra, Vector Calculus,
Multiple integrals, Divergence theorem, Green's theorem, Stokes' theorem. First and
linear second order differential equations. Matrices and determinants, Algebra of
complex numbers.
Mechanics and General Properties of Matter
Newton's laws of motion and applications, Velocity and acceleration in Cartesian, polar
and cylindrical coordinate systems, uniformly rotating frame, centrifugal and Coriolis
forces, Motion under a central force, Kepler's laws, Gravitational Law and field,
Conservative and non-conservative forces. System of particles, Centre of mass, equation
of motion of the CM, conservation of linear and angular momentum, conservation of
energy, variable mass systems. Elastic and inelastic collisions. Rigid body motion, fixed
axis rotations, rotation and translation, moments of Inertia and products of Inertia.
Principal moments and axes. Elasticity, Hooke's law and elastic constants of isotropic
solid, stress energy. Kinematics of moving fluids, equation of continuity, Euler's
equation, Bernoulli's theorem, viscous fluids, surface tension and surface energy,
capillarity.
Oscillations, Waves and Optics: Differential equation for simple harmonic oscillator
and its general solution. Superposition of two or more simple harmonic oscillators.
Lissajous figures. Damped and forced oscillators, resonance. Wave equation, traveling
and standing waves in one-dimension. Energy density and energy transmission in waves.
Group velocity and phase velocity. Sound waves in media. Doppler Effect. Fermat's
Principle. General theory of image formation. Thick lens, thin lens and lens
combinations. Interference of light, optical path retardation. Fraunhofer diffraction.
Rayleigh criterion and resolving power. Diffraction gratings. Polarization: linear, circular
and elliptic polarization. Double refraction and optical rotation.

fiziks c/o Anand Institute of mathematics, 28-B/6 Jia Sarai


Near IIT, Hauz Khas, New Delhi, PIN- 110016 (INDIA)
Phone: 011-32718565, +91-9871145498
Website: http://www.physicsbyfiziks.com 1
Email: fiziks.physics@gmail.com
fiziks
Forum for CSIR-UGC JRF/NET, GATE, IIT-JAM, GRE in PHYSICAL SCIENCES

Electricity and Magnetism


Coulomb's law, Gauss's law. Electric field and potential. Electrostatic boundary
conditions, Solution of Laplace's equation for simple cases. Conductors, capacitors,
dielectrics, dielectric polarization, volume and surface charges, electrostaticenergy. Biot-
Savart law, Ampere's law, Faraday's law of electromagnetic induction, Self and mutual
inductance. Alternating currents. Simple DC and AC circuits with R, L and C
components. Displacement current, Maxwell's equations and plane electromagnetic
waves, Poynting's theorem, reflection and refraction at a dielectric interface, transmission
and reflection coefficients (normal incidence only). Lorentz Force and motion of charged
particles in electric and magnetic fields.
Kinetic theory, Thermodynamics
Elements of Kinetic theory of gases. Velocity distribution and Equipartition of energy.
Specific heat of Mono-, di- and tri-atomic gases. Ideal gas, van-der-Waals gas and
equation of state. Mean free path. Laws of thermodynamics. Zeroeth law and concept of
thermal equilibrium. First law and its consequences. Isothermal and adiabatic processes.
Reversible, irreversible and quasi-static processes. Second law and entropy. Carnot cycle.
Maxwell's thermodynamic relations and simple applications. Thermodynamic potentials
and their applications. Phase transitions and Clausius-Clapeyron equation.
Modern Physics
Inertial frames and Galilean invariance. Postulates of special relativity. Lorentz
transformations. Length contraction, time dilation. Relativistic velocity addition theorem,
mass energy equivalence. Blackbody radiation, photoelectric effect, Compton effect,
Bohr's atomic model, X-rays. Wave-particle duality, Uncertainty principle, Schrodinger
equation and its solution for one, two and three dimensional boxes. Reflection and
transmission at a step potential, tunneling through a barrier. Pauli exclusion principle.
Distinguishable and indistinguishable particles. Maxwell-Boltzmann, Fermi-Dirac and
Bose-Einstein statistics. Structure of atomic nucleus, mass and binding energy.
Radioactivity and its applications. Laws of radioactive decay. Fission and fusion.

fiziks c/o Anand Institute of mathematics, 28-B/6 Jia Sarai


Near IIT, Hauz Khas, New Delhi, PIN- 110016 (INDIA)
Phone: 011-32718565, +91-9871145498
Website: http://www.physicsbyfiziks.com 2
Email: fiziks.physics@gmail.com
fiziks
Forum for CSIR-UGC JRF/NET, GATE, IIT-JAM, GRE in PHYSICAL SCIENCES

Solid State Physics, Devices and Electronics


Crystal structure, Bravais lattices and basis. Miller indices. X-ray diffraction and Bragg's
law, Einstein and Debye theory of specific heat. Free electron theory of metals. Fermi
energy and density of states. Origin of energy bands. Concept of holes and effective
mass. Elementary ideas about dia-, para- and ferromagnetism, Langevin's theory of
paramagnetism, Curie's law. Intrinsic and extrinsic semiconductors. Fermi level. p-n
junctions, transistors. Transistor circuits in CB, CE, CC modes. Amplifier circuits with
transistors. Operational amplifiers. OR, AND, NOR and NAND gates.

fiziks c/o Anand Institute of mathematics, 28-B/6 Jia Sarai


Near IIT, Hauz Khas, New Delhi, PIN- 110016 (INDIA)
Phone: 011-32718565, +91-9871145498
Website: http://www.physicsbyfiziks.com 3
Email: fiziks.physics@gmail.com
fiziks
Institute for NET/JRF, GATE, IIT‐JAM, JEST, TIFR and GRE in PHYSICAL SCIENCES 
 
Mathematical Methods
OBJECTIVE QUESTIONS
IIT-JAM-2005
0 1
Q1. Which of the following is INCORRECT for the matrix M   
1 0 
(a) It is its own inverse (b) It is its own transpose
(c) It is non-orthogonal (d) It has eigen values ± 1
Ans. : (c)
1  0 1
Solution: The inverse of the given matrix is M 1   1 0 
M  

1  0 1 0 1 
  M
 1  1 0  1 0 
Thus the given matrix is its own inverse.
The transpose of M is
0 1 
MT   M
1 0 
The given matrix is orthogonal as each row vector is a unit vector and the two rows are
orthogonal.
The eigenvalues of orthogonal matrix are 1 or 1 . For the given matrix
1  2  0  1  1 and 2  1
Thus option (c) is correct option.
Q2. A periodic function can be expressed in a Fourier series of the form,

f  x    a n cosnx   bn sin nx  . The functions f1(x) = cos2x and f2(x) = sin2x are
n 0

expanded in their respective Fourier series. If the coefficients for the first series are a n1

and bn1 , and the coefficients for the second series are a n2  and bn2  , respectively, then
which of the following is correct?

                                                                                
Head office  Branch office 
 
fiziks, H.No. 40 D, G.F, Jia Sarai,  Anand Institute of Mathematics, 
 
Near IIT, Hauz Khas, New Delhi‐16  28‐B/6, Jia Sarai, Near IIT 
 
Phone: 011‐26865455/+91‐9871145498 Hauz Khas, New Delhi‐16 
                                                   
                                             Website: www.physicsbyfiziks.com                                                                                          
                                                            Email: fiziks.physics@gmail.com                                                                   1 
fiziks
Institute for NET/JRF, GATE, IIT‐JAM, JEST, TIFR and GRE in PHYSICAL SCIENCES 
 
1 1 1 1
(a) a 21  and b22   (b) b21  and a 22  
2 2 2 2
1 1 1 1
(c) a 21  and a 22   (d) b21  and b22  
2 2 2 2
Ans. : (c)
1 1 1 1
Solution: f1  x    cos 2 x and f 2  x    cos 2 x
2 2 2 2
1 1
Hence, a21  and a2 2  
2 2
All the bn ’s of each of the series are zero. As there is no sine terms in any of the two
given functions.
Thus the correct option is (c).
IIT-JAM 2006
a
Q3. The symmertric part of P   a  2 b  is
b
 2  a a  2 
(a)  a  2 ba2  1  (b) 
b 
 ba  1 b  2   b b 2 

a a  1 ba  1 a a  2  ba  1


(c)   (d)  
 a  1
b b2   a  1
b b2 
Ans. : (d)

a  a  a  2 a b 
Solution: The given matrix can be written as p     a  2 b    2 
b   b  a  2 b 
The transpose of p is

 a  a  2 b  a  2 
PT   
 ab b2 

Hence the symmetric part of p is

p  pT 1  2a  a  2  2ab  2b   a  a  2  b  a  1 
    
2 2  2ab  2b 2b 2   b  a  1 b2 

Hence the correct option is (d).

                                                                                
Head office  Branch office 
 
fiziks, H.No. 40 D, G.F, Jia Sarai,  Anand Institute of Mathematics, 
 
Near IIT, Hauz Khas, New Delhi‐16  28‐B/6, Jia Sarai, Near IIT 
 
Phone: 011‐26865455/+91‐9871145498 Hauz Khas, New Delhi‐16 
                                                   
                                             Website: www.physicsbyfiziks.com                                                                                          
                                                            Email: fiziks.physics@gmail.com                                                                   2 
fiziks
Institute for NET/JRF, GATE, IIT‐JAM, JEST, TIFR and GRE in PHYSICAL SCIENCES 
 
IIT-JAM 2007
x y  7
5  7    15
x
3  y 
Q4.

The matrix equation above represents
(a) a circle of radius 15 (b) an ellipse of semi major axis 5
(c) an ellipse of semi major axis 5 (d) a hyperbola
Ans. : (c)
 5 7   x   5x  7 y 
Solution: x y     15   x y   15
7 3  y   7x  3y 
x2 y2
 5 x 2  7 xy  7 xy  3 y 2  15  5 x 2  3 y 2  15   1
3 5
Thus the given equation represents an ellipse with semi-major axis 5 .
Q5. f(x) is a periodic function of x with a period of 2π. In the interval –π < x < π, f(x) is
given by
0,   x  0
f x   
sin x, 0  x  
In the expansion of f(x) as a Fourier series of sine and cosine functions, the coefficient of
cos  2x  is

2 1 2
(a) (b) (c) 0 (d) 
3  3
Ans. : (d)
Solution: The coefficients of cos 2x is a2 .

1 
0
Thus, a2    0  cos 2 xdx   sin x  cos 2 xdx 
   0 
 
1 1  cos 3 x 
 a2 
2   sin 3x  sin x  dx 
0
2   3  cos x 
0

1  cos 3x   1  1 1 1  2

2   3  cos      3  1   2  3  1   1
3 
 
3
   

                                                                                
Head office  Branch office 
 
fiziks, H.No. 40 D, G.F, Jia Sarai,  Anand Institute of Mathematics, 
 
Near IIT, Hauz Khas, New Delhi‐16  28‐B/6, Jia Sarai, Near IIT 
 
Phone: 011‐26865455/+91‐9871145498 Hauz Khas, New Delhi‐16 
                                                   
                                             Website: www.physicsbyfiziks.com                                                                                          
                                                            Email: fiziks.physics@gmail.com                                                                   3 
fiziks
Institute for NET/JRF, GATE, IIT‐JAM, JEST, TIFR and GRE in PHYSICAL SCIENCES 
 
IIT-JAM 2008
Q6. The product PQ of any two real, symmetric matrices P and Q is
(a) symmetric for all P and Q (b) never symmetric
(c) symmetric if PQ = QP (d) antisymmtric for all P and Q
Ans. : (c)
Solution: A matrix is symmetric if its transpose is equal to the matrix itself.

 PQ   QT PT (since  AB   BT AT )
T T
Hence for the matrix PQ ,

Since Q and P are symmetric matrices; QT  Q , PT  P

Hence,  PQ   QP
T

It is easily seen that  PQ  will be equal to PQ only if QP  PQ . Hence (c) is correct


T

option.
Q7. The work done by a force in moving a particle of mass m from any point (x, y) to a
neighboring point (x + dx, y + dy) is given by dW = 2xy dx + x2 dy. The work done for a
complete cycle around a unit circle is
(a) 0 (b) 1 (c) 3 (d) 2π
Ans. : (a)
Solution: Let us write the co-ordinates x and y as
 x, y 
x  0 1 cos  , y  1 sin   x  cos  and y  sin  . 1

Thus dx   sin  d and dy  cos  d
Thus the given work dw can be written as unit circle
dw  2  cos   sin    sin   d   cos   cos  d  2sin 2   cos  d  cos3  d
2

Thus the total work done along the complete circle is


2 2
W  2  sin 2   cos  d   cos  d
3

0 0

It can be easily checked that the value of each of these integrals is 0 , hence the correct
option is (a).

                                                                                
Head office  Branch office 
 
fiziks, H.No. 40 D, G.F, Jia Sarai,  Anand Institute of Mathematics, 
 
Near IIT, Hauz Khas, New Delhi‐16  28‐B/6, Jia Sarai, Near IIT 
 
Phone: 011‐26865455/+91‐9871145498 Hauz Khas, New Delhi‐16 
                                                   
                                             Website: www.physicsbyfiziks.com                                                                                          
                                                            Email: fiziks.physics@gmail.com                                                                   4 
fiziks
Institute for NET/JRF, GATE, IIT‐JAM, JEST, TIFR and GRE in PHYSICAL SCIENCES 
 
IIT-JAM 2009
Q8. In the Fourier series of the periodic function (shown in the figure)
f  x   sin x

   n cos nx   n sin nx 
n 0

 2
Which of the following coefficients are nonzero?

(a)  n for odd n (b)  n for even n

(c)  n for odd n (d)  n for even n


Ans. : (b)
Solution: The given function is an even function (assuming the basic interval of definition to be
symmetric about the origin)
Hence, all the Bn1 are 0 .
 
2 2
n 
  sin x cos nx dx 
0
  sin x  cos nx dx
0

1   cos  n  1 x cos  n  1 x 

2
   
2 0
 
 sin n  1 x  sin n  1 x 
 dx    
   n  1  n  1  0
For odd n .

1 1 1 1 1 
n         n  0 , for odd n .
   n  1  n  1  n  1  n  1 

For even n

1 1 1 1 1  2 1 1 
n       n    
   n  1  n  1  n  1  n  1     n  1  n  1 

2  n  1  n  1 4
   
   n  1 

2
   n 2  1

Thus for even n ,  n is nonzero. Hence the correct option is (b).

                                                                                
Head office  Branch office 
 
fiziks, H.No. 40 D, G.F, Jia Sarai,  Anand Institute of Mathematics, 
 
Near IIT, Hauz Khas, New Delhi‐16  28‐B/6, Jia Sarai, Near IIT 
 
Phone: 011‐26865455/+91‐9871145498 Hauz Khas, New Delhi‐16 
                                                   
                                             Website: www.physicsbyfiziks.com                                                                                          
                                                            Email: fiziks.physics@gmail.com                                                                   5 
fiziks
Institute for NET/JRF, GATE, IIT‐JAM, JEST, TIFR and GRE in PHYSICAL SCIENCES 
 
IIT-JAM 2010
1  i 1
Q9. A matrix is given by M    . The eigenvalues of M are
2 1 i 
(a) real and positive (b) purely imaginary with modulus 1
(c) complex with modulus 1 (d) real and negative
Ans. : (c)
Solution: We know that if  is an eigenvalue of matrix A , then k  is the eigenvalue of matrix
kA . Hence Let as evaluate the eigenvalue of matrix
 i 1
M   
1 i 
For the calculation of eigenvalues
i 1
 0   i     1  0  i 2  2i   2  1  0
2

1 i

  1  2i   2  1  0   2  2i  2  0

2i   2i   4 1  2 
2
2i  4  8 2i  2
    i 1
2 2 2
Thus the eigenvalues of the given matrix M are
1 1 1 1 1 1
1  1  i    i and 2   i  1   i
2 2 2 2 2 2
We see that 1  2  1 . Thus the correct option is (c).

3 2 3 2
Q10. The equation of a surface of revolution is z   x  y . The unit normal to the
2 2

 2 
surface at the point A ,0,1 is
 3 

3ˆ 2 ˆ 3ˆ 2 ˆ 3ˆ 2 ˆ 3ˆ 2 ˆ
(a) i k (b) i k (c) i k (d) i k
5 10 5 10 5 5 10 10

                                                                                
Head office  Branch office 
 
fiziks, H.No. 40 D, G.F, Jia Sarai,  Anand Institute of Mathematics, 
 
Near IIT, Hauz Khas, New Delhi‐16  28‐B/6, Jia Sarai, Near IIT 
 
Phone: 011‐26865455/+91‐9871145498 Hauz Khas, New Delhi‐16 
                                                   
                                             Website: www.physicsbyfiziks.com                                                                                          
                                                            Email: fiziks.physics@gmail.com                                                                   6 
fiziks
Institute for NET/JRF, GATE, IIT‐JAM, JEST, TIFR and GRE in PHYSICAL SCIENCES 
 
Ans. : (b)
3 2 3 2 3 3
Solution: z   x  y  z 2  x 2  y 2  3x 2  3 y 2  2 z 2  0
2 2 2 2

Let V  3x 2  3 y 2  2 z 2 , Taking gradient  V  6 xxˆ  6 yyˆ  4 zzˆ .

 2  V
The unit normal to the surface at the point A ,0,1 is nˆ   . Thus
 3  V

2 2
6 xˆ  6  0 yˆ  4 1zˆ 6 xˆ  4 zˆ
3 3 3 2
nˆ    xˆ  zˆ
2 40 5 10
36   16
3
IIT-JAM 2011
B
   x y
Q11. The line integral  F  dl , where F  xˆ  yˆ , along the semi-circular path as
A x y
2 2
x y
2 2

shown in the figure below is y

x
 1,0 1,0
(a) -2 (b) 0 (c) 2 (d) 4
Ans. : (b)

Solution: x 2  y 2  1  xdx   ydy and dl  dxxˆ  dyyˆ
  xdx ydy
B
 
 F .dl   0  xdx   ydy    F  dl  0
x2  y2 x2  y2 A

                                                                                
Head office  Branch office 
 
fiziks, H.No. 40 D, G.F, Jia Sarai,  Anand Institute of Mathematics, 
 
Near IIT, Hauz Khas, New Delhi‐16  28‐B/6, Jia Sarai, Near IIT 
 
Phone: 011‐26865455/+91‐9871145498 Hauz Khas, New Delhi‐16 
                                                   
                                             Website: www.physicsbyfiziks.com                                                                                          
                                                            Email: fiziks.physics@gmail.com                                                                   7 
fiziks
Institute for NET/JRF, GATE, IIT‐JAM, JEST, TIFR and GRE in PHYSICAL SCIENCES 
 
Q12. Given two (nxn) matrices P̂ and Q̂ such that P̂ is Hermitian and Q̂ is skew (anti)-

Hermitian. Which one of the following combinations of P̂ and Q̂ is necessarily a


Hermitian matrix?
(a) Pˆ Qˆ (b) iPˆ Qˆ (c) Pˆ  iQˆ (d) Pˆ  Qˆ
Ans.: (c)
Solution: Any matrix is hermitian if its conjugate transpose is equal to the matrix itself.

     Pˆ    Qˆ   Pˆ   QP
* * *
ˆ ˆ  Qˆ
For, P̂Qˆ , we have PQ ˆˆ

Thus P̂Qˆ is not hermitian.

ˆ ˆ   2  PQ  
ˆ ˆ   i  Qˆ Pˆ  iQP     
*
ˆ ˆ , we have iPQ
For matrix iPQ ˆˆ

ˆ ˆ is not hermitian.
Thus iPQ

For matrix P̂  iQˆ , we have

 Pˆ  iQˆ    Pˆ   iQˆ     
 Pˆ   i  Qˆ  Pˆ   i  Qˆ  Pˆ  iQˆ
* * * *

Thus P̂  iQˆ is hermitian.

       
* * *
For P̂  Qˆ , we have Pˆ  Qˆ  Pˆ  Qˆ  Pˆ  Qˆ

 
Thus P̂  Qˆ is not hermitian.

Note: In this question “ * ” symbol has been used to denote the conjugate transpose of a
matrix.

                                                                                
Head office  Branch office 
 
fiziks, H.No. 40 D, G.F, Jia Sarai,  Anand Institute of Mathematics, 
 
Near IIT, Hauz Khas, New Delhi‐16  28‐B/6, Jia Sarai, Near IIT 
 
Phone: 011‐26865455/+91‐9871145498 Hauz Khas, New Delhi‐16 
                                                   
                                             Website: www.physicsbyfiziks.com                                                                                          
                                                            Email: fiziks.physics@gmail.com                                                                   8 
fiziks
Institute for NET/JRF, GATE, IIT‐JAM, JEST, TIFR and GRE in PHYSICAL SCIENCES 
 
IIT-JAM-2012
   
Q13.

If F is a constant vector and r is the position vector then  F  r would be  
        

(a)   r F  (b) F (c)   F r (d) r F  
Ans.: (b)
   
Solution: Let F  F0  xˆ  yˆ  zˆ  and r  xxˆ  yyˆ  zzˆ  F .r  F0  x  y  z  .
   
 
Thus  F  r  F0  xˆ  yˆ  zˆ   F

IIT-JAM-2013
Q14. The inverse of the matrix
0 1 1
 
M   0 0 1  is
1 0 0
 
(a) M  I (b) M 2  I (c) I  M 2 (d) I  M
where I is the identity matrix.
Ans.: (b)
0 1 1 
Solution: Given M  0 0 1 
1 0 0 

The characteristics equation is


 1 1
0  1 0
1 0 

   2  0    1  1    0    3    1  0   3    1  0

Thus the cayley-Hamilton theorem gives


M3 M I 0
Multiply both sides by M 1 gives
M 2  I  M 1  0  M 1  M 2  I . Thus option (b) is correct option.
                                                                                
Head office  Branch office 
 
fiziks, H.No. 40 D, G.F, Jia Sarai,  Anand Institute of Mathematics, 
 
Near IIT, Hauz Khas, New Delhi‐16  28‐B/6, Jia Sarai, Near IIT 
 
Phone: 011‐26865455/+91‐9871145498 Hauz Khas, New Delhi‐16 
                                                   
                                             Website: www.physicsbyfiziks.com                                                                                          
                                                            Email: fiziks.physics@gmail.com                                                                   9 
fiziks
Institute for NET/JRF, GATE, IIT‐JAM, JEST, TIFR and GRE in PHYSICAL SCIENCES 
 
Q15. The value of i   i , where i   1, is
1
(a) 0 (b) (c) 2 (d)  2
2
Ans.: (a)

Solution: i  i 
i  i  i  i   i i 2  i 2  i 2
i  i i  i

i  i i i
  0
i  i i  i
Q16. The solution of the differential equation dz  x, y   xz  x, y dx  yz  x, y dy  0 is………

Ans.: Ce
 
 x2  y2 / 2

Given differential equation can be written as


dz
dz  x, y   z  x, y   xdx  ydy   0    xdx  ydy
z
Integrating both sides gives

z x  y 
2 2
x2 y2
ln z     ln c  ln 
2 2 c 2
z  x 2  y 2  / 2  x 2  y 2  / 2
 e  z  ce .
c
Q17. Given that f 1  1, f 1  1, and f " 1  1, the value of f 1 2  is
Ans.: 1.65
Solution: Let f  x   ke x

1
In order to satisfy each of the three given conditions k  .
e
ex
Thus f  x 
e
e1/ 2 1
Hence, f 1/ 2     1.65 .
e e

                                                                                
Head office  Branch office 
 
fiziks, H.No. 40 D, G.F, Jia Sarai,  Anand Institute of Mathematics, 
 
Near IIT, Hauz Khas, New Delhi‐16  28‐B/6, Jia Sarai, Near IIT 
 
Phone: 011‐26865455/+91‐9871145498 Hauz Khas, New Delhi‐16 
                                                   
                                             Website: www.physicsbyfiziks.com                                                                                          
                                                            Email: fiziks.physics@gmail.com                                                                   10 
fiziks
Institute for NET/JRF, GATE, IIT‐JAM, JEST, TIFR and GRE in PHYSICAL SCIENCES 
 
IIT-JAM-2014
   
Q18.
 
For vectors a  ˆj  kˆ, b  2iˆ  3 ˆj  5kˆ and c  ˆj  kˆ , the vector product a  b  c is  
 
(a) in the same direction as c (b) in the direction opposite to c
 
(c) in the same direction as b (d) in the direction opposite to b

iˆ ˆj kˆ
 
Ans.: (a) b  c  2 3 5  iˆ  3  5   ˆj  2  0   kˆ  2  0   2iˆ  2 ˆj  2kˆ
0 1 1

iˆ ˆj kˆ
   
 
a  b  c  0 1 1  iˆ  2  2   ˆj  0  2   kˆ  0  2   2iˆ  2kˆ  2c
2 2 2

Two Marks


Q19. The value of r n
sin n  for r = 0.5 and   is
n 0 3

1 2 3
(a) (b) (c) (d) 3
3 3 2
Ans.: (a)

Q20. If the surface integral of the field A x, y, z   2  x iˆ   y ˆj  3  z kˆ over the closed
surface of an arbitrary unit sphere is to be zero, then the relationship between  ,  and 
is
(a)    / 6    0 (b)  / 3   / 6   / 2  0
(c)  / 2     / 3  0 (d) 2 /   1 /   3 /   0
Ans.: (b)
   
Solution: It is given that  A .d a  0    
 .A d  0 (From Divergence Theorem)
S V

    

V

. A d  0  2     3   0     0
3 6 2

                                                                                
Head office  Branch office 
 
fiziks, H.No. 40 D, G.F, Jia Sarai,  Anand Institute of Mathematics, 
 
Near IIT, Hauz Khas, New Delhi‐16  28‐B/6, Jia Sarai, Near IIT 
 
Phone: 011‐26865455/+91‐9871145498 Hauz Khas, New Delhi‐16 
                                                   
                                             Website: www.physicsbyfiziks.com                                                                                          
                                                            Email: fiziks.physics@gmail.com                                                                   11 
fiziks
Institute for NET/JRF, GATE, IIT‐JAM, JEST, TIFR and GRE in PHYSICAL SCIENCES 
 
  
Q21.
r
1

The line integral  A  dl of a vector field A x, y   2  yiˆ  xˆj , where r 2  x 2  y 2 , is 
taken around a square (see figure) of side unit length and centered at x 0 , y 0  with

1 1
x0  and y 0  . If the value of the integral is L, then
2 2
y

 x0 , y0 

(a) L depends on x 0 , y 0 

(b) L is independent of x 0 , y 0  and its value is -1

(c) L is independent of x 0 , y 0  and its value is 0

(d) L is independent of x 0 , y 0  and its value is 2


Ans.: (c)
 
 
 xˆ yˆ zˆ 
      
Solution:   A   
 x y z 
 y x 
 2 0
 x y x  y2
2 2

   x    y 
=xˆ  0  0   yˆ  0  0   zˆ   2 2 
  2 2 
 x  x  y  y  x  y  

    x2  y 2   x  2x  x2  y 2   y  2 y    y 2  x2   x2  y 2 
  A  zˆ     zˆ  0
  x 2  y 2 2  x  y     x  y  
2 2 2   2 2 2

 
Thus  A  dl  0 .

                                                                                
Head office  Branch office 
 
fiziks, H.No. 40 D, G.F, Jia Sarai,  Anand Institute of Mathematics, 
 
Near IIT, Hauz Khas, New Delhi‐16  28‐B/6, Jia Sarai, Near IIT 
 
Phone: 011‐26865455/+91‐9871145498 Hauz Khas, New Delhi‐16 
                                                   
                                             Website: www.physicsbyfiziks.com                                                                                          
                                                            Email: fiziks.physics@gmail.com                                                                   12 
fiziks
Institute for NET/JRF, GATE, IIT‐JAM, JEST, TIFR and GRE in PHYSICAL SCIENCES 
 
IIT-JAM-2015
x y x y
Q22. Consider the coordinate transformation x   , y  . The relation between the
2 2
area elements dx dy  and dxdy is given by dx dy   jdxdy . The value of j is
(a) 2 (b) 1 (c)  1 (d)  2
Ans.: (c)
x y x y
Solution: x  , y 
2 2
 dxdy  J dxdy

 x x   1 1 
 x  
y
 2 2  1 1
 J      1
 y y   1 1  2 2
 x    
 y   2 2
Q23. The trace of a 2 2 matrix is 4 and its determinant is 8 . If one of the eigenvalues is
21  i  , the other eigenvalue is

(a) 21  i  (b) 21  i  (c) 1  2i  (d) 1  2i 

Ans.: (a)
Solution: 1  2  2i, 2  2 1  i   1  2  4 and 1   2  8

Two Marks:

Q24. Consider a vector field F  yiˆ  xz 3 ˆj  zykˆ . Let C be the circle x 2  y 2  4 on the
 
plane z  2 , oriented counter-clockwise. The value of the contour integral  F  d r is
C

(a) 28  (b) 4  (c)  4  (d)  28 


Ans.: (a)
    
Solution: 
  F .d r     F .d a 
C S

                                                                                
Head office  Branch office 
 
fiziks, H.No. 40 D, G.F, Jia Sarai,  Anand Institute of Mathematics, 
 
Near IIT, Hauz Khas, New Delhi‐16  28‐B/6, Jia Sarai, Near IIT 
 
Phone: 011‐26865455/+91‐9871145498 Hauz Khas, New Delhi‐16 
                                                   
                                             Website: www.physicsbyfiziks.com                                                                                          
                                                            Email: fiziks.physics@gmail.com                                                                   13 
fiziks
Institute for NET/JRF, GATE, IIT‐JAM, JEST, TIFR and GRE in PHYSICAL SCIENCES 
 
xˆ yˆ zˆ
 
  F   / x  / y  / z
y xz 3  zy

      yz    xz 3    y    zy      xz  y 
3

   F  xˆ     yˆ     zˆ   
 y z   z x   x y 
   
 
   F  xˆ   z  3xz   yˆ  0  0   zˆ  z  1
2 3

 
 z  2    F    2  12 x  xˆ  7 zˆ
   
 
 d a  rdrd zˆ    F .d a     2  12 x  xˆ  7 zˆ  .rdrd zˆ  7 rdrd

   2

 
2
   F .d a  7  rdr  d  28
S 0 0

dy y 2
Q25. Consider the equation  with the boundary condition y 1  1 . Out of the following
dx x
the range of x in which y is real and finite is
(a)    x  3 (b)  3  x  0
(c) 0  x  3 (d) 3  x  
Ans.: (d)
Solution:
dy y 2 dy dx 1
  2     ln x  C 
dx x y x y

y 1  1    ln1  C   C   1    ln x  1  y 
1 1 1
1 y 1  ln x
At x  0, y   and ln x is not defined for negative values of x .
Thus correct option is (d).

                                                                                
Head office  Branch office 
 
fiziks, H.No. 40 D, G.F, Jia Sarai,  Anand Institute of Mathematics, 
 
Near IIT, Hauz Khas, New Delhi‐16  28‐B/6, Jia Sarai, Near IIT 
 
Phone: 011‐26865455/+91‐9871145498 Hauz Khas, New Delhi‐16 
                                                   
                                             Website: www.physicsbyfiziks.com                                                                                          
                                                            Email: fiziks.physics@gmail.com                                                                   14 
fiziks
Institute for NET/JRF, GATE, IIT‐JAM, JEST, TIFR and GRE in PHYSICAL SCIENCES 
 
Q26. The Fourier series for an arbitrary periodic function with period 2 L , is given by
a0 n x n x
f x    n 1 a n cos  n 1 bn sin
 
. For the particular periodic function
2 L L
shown in the figure the value of a0 is f x 
1

1/ 2

x
2 1 0 1 2
(a) 0 (b) 0.5 (c) 1 (d) 2
Ans.: (c)
Solution: The wavefunction of the given function can be written as
x 0  x 1
f  x  
 x 1  x  0

Coefficient a0 is defined as

a0  1  x dx  1 x dx
0 1

1 0

 1   1

 x2   x2 
1
 2 2
 1 1
         0    0     1
 2  1  2  0  2   2  2 2

 a0  1

Q27. The phase of the complex number 1  i  i in the polar representation is

  3 5
(a) (b) (c) (d)
4 2 4 4
Ans.: (c)
Solution: z  1  i  i  z   1  i   z  x  iy

y 3
tan    1    tan 1 ( 1)   
x 4

                                                                                
Head office  Branch office 
 
fiziks, H.No. 40 D, G.F, Jia Sarai,  Anand Institute of Mathematics, 
 
Near IIT, Hauz Khas, New Delhi‐16  28‐B/6, Jia Sarai, Near IIT 
 
Phone: 011‐26865455/+91‐9871145498 Hauz Khas, New Delhi‐16 
                                                   
                                             Website: www.physicsbyfiziks.com                                                                                          
                                                            Email: fiziks.physics@gmail.com                                                                   15 
fiziks
Institute for NET/JRF, GATE, IIT‐JAM, JEST, TIFR and GRE in PHYSICAL SCIENCES 
 
IIT-JAM-2016
1
Q28. Which of the following points represent the complex number  ?
1 i
(a) y (b) y

1 1
0.5 0.5
x x
1 0.5 0.5 1 1 0.5 0.5 1
0.5 0.5
1 1

y y
(c) (d)
1 1
0.5 0.5
x x
1 0.5 0.5 1 1 0.5 0.5 1
0.5 0.5
1 1
Ans.: (a)
1 1  1 i  1 i 1 1
Solution:      i
1 i 1 i  1 i  11 2 2
Q29. The eigenvalues of the matrix representing the following pair of linear equations
x  iy  0
ix  y  0
are
(a) 1  i, 1  i (b) 1  i, 1  i (c) 1, i (d) 1  i, 1  i
Ans.: (d)
Solution: Characteristic equation is A   I  0

1  i
 0  1     i 2  1     1  0  1     i    1  i, 1  i
2 2

i 1 

                                                                                
Head office  Branch office 
 
fiziks, H.No. 40 D, G.F, Jia Sarai,  Anand Institute of Mathematics, 
 
Near IIT, Hauz Khas, New Delhi‐16  28‐B/6, Jia Sarai, Near IIT 
 
Phone: 011‐26865455/+91‐9871145498 Hauz Khas, New Delhi‐16 
                                                   
                                             Website: www.physicsbyfiziks.com                                                                                          
                                                            Email: fiziks.physics@gmail.com                                                                   16 
fiziks
Institute for NET/JRF, GATE, IIT‐JAM, JEST, TIFR and GRE in PHYSICAL SCIENCES 
 
Two Marks.
Q30. For the given set of equations
x  y  1, y  z  1 , x  z  1 ,
which one of the following statements is correct?
(a) Equations are inconsistent
(b) Equations are consistent and a single non-trivial solution exists
(c) Equations are consistent and many solutions exist
(d) Equations are consistent and only a trivial solution exists.
Ans.: (b)
1 1 0 1
Solution: The augmented matrix of the system can be written as M  0 1 1 1
1 0 1 1
1 1 0 1  1 1 0 1
Row reduction gives M   0 1 1 1    0 1 1 1
 0 1 1 0   0 0 2 1
Thus, x  y  1 , y  z  1 and 2 z  1
The last equation gives z  1/ 2 . Using first two equations we find x  y  1/ 2 . Thus the
system has a single non trivial solution. The correct option is (b)
Q31. The tangent line to the curve x 2  xy  5  0 at 1,1 is represented by

(a) y  3x  2 (b) y  3 x  4
(c) x  3 y  2 (d) x  3 y  4
Ans.: (b)
dy dy   2x  y 
Solution: Given x 2  xy  5  0  2 x  y  x 0  
dx dx x
dy 3
At 1,1 ,    3
dx 1
Hence the equation of tangent line is y  1  3  x  1  y  3 x  4

                                                                                
Head office  Branch office 
 
fiziks, H.No. 40 D, G.F, Jia Sarai,  Anand Institute of Mathematics, 
 
Near IIT, Hauz Khas, New Delhi‐16  28‐B/6, Jia Sarai, Near IIT 
 
Phone: 011‐26865455/+91‐9871145498 Hauz Khas, New Delhi‐16 
                                                   
                                             Website: www.physicsbyfiziks.com                                                                                          
                                                            Email: fiziks.physics@gmail.com                                                                   17 
fiziks
Institute for NET/JRF, GATE, IIT‐JAM, JEST, TIFR and GRE in PHYSICAL SCIENCES 
 
Q32. Fourier series of a given function f  x  in the interval 0 to L is

a0  2 nx   2 nx 
f  x    n 1 an cos     n 1 bn sin 
 
.
2  L   L 
If f  x   x in the region  0,   ,b2  ………………

Ans.: 0  5
Solution: Here 2l  2  l   / 2
  
1 2 x 2 2   x cos 4 x 1 
b2   x sin dx   x sin 4 xdx    sin 4 x 
 0  /2  0  4 16 0

2   cos  1   2    1
    sin 4  0         0.5 .
  4 16    4  2

Q33. Consider a function f  x, y   x 3  y 3 , where y represents a parabolic curve x 2  1 . The

total derivative of f with respect to x , at x  1 is………………….


Ans.: 27
Solution: f  x , y   x 3  y 3

Also given that y  x 2  1

Hence, f  x, y   f  x   x3   x 2  1
3

df  x , y  df  x 
 3x 2  3  x 2  1  2 x
2
 
dx dx
Hence the total derivative at x  1 is

3 1  3 12  1  2 1  3  6  4  27
2

                                                                                
Head office  Branch office 
 
fiziks, H.No. 40 D, G.F, Jia Sarai,  Anand Institute of Mathematics, 
 
Near IIT, Hauz Khas, New Delhi‐16  28‐B/6, Jia Sarai, Near IIT 
 
Phone: 011‐26865455/+91‐9871145498 Hauz Khas, New Delhi‐16 
                                                   
                                             Website: www.physicsbyfiziks.com                                                                                          
                                                            Email: fiziks.physics@gmail.com                                                                   18 
fiziks
Institute for NET/JRF, GATE, IIT‐JAM, JEST, TIFR and GRE in PHYSICAL SCIENCES 
 
Q34. Consider a closed triangular contour traversed in counter-clockwise
y
direction, as shown in the figure.
  Q
The value of the integral,  F  dl evaluated along this contour, for a

vector field, F  yeˆx  xeˆy , is………….. ( eˆx , eˆy and eˆz are unit vectors
 
4 4
in Cartesian-coordinate system). x
O P  2,0 
Ans.: 2
      
Solution:  F  yeˆx  xeˆy    F  2 zˆ and d a  dxdyzˆ    F .d a  2dxdy  
    
 F  dl      F  .d a     2 dxdy   2  2  2 1  2
1

Q35. A hemispherical shell is placed on the xy - plane centered at the origin. For a vector
  
  
field E    yeˆx  xeˆy  / x 2  y 2 , the value of the integral    E  da over the
S
 
hemispherical surface is………………  .

( da is the elemental surface area, eˆx , eˆy and eˆz are unit vectors in Cartesian-coordinate

system)
Ans.: 2
  
 
Solution: E    yeˆx  xeˆy  / x 2  y 2    E  0 except at origin.
    

    E .d a    E.dl
S line

We have to take line integral around circle x 2  y 2  r 2 in z  0 plane. Let use cylindrical
coordinate and use x  r cos  , y  r sin   dx  r sin  d , dy  r cos  d .
   r sin    r sin   d  r cos   r cos   d
E.dl    ydx  xdy  / x 2  y 2    r2
 d

  2
  E.dl   d  2
0
line

                                                                                
Head office  Branch office 
 
fiziks, H.No. 40 D, G.F, Jia Sarai,  Anand Institute of Mathematics, 
 
Near IIT, Hauz Khas, New Delhi‐16  28‐B/6, Jia Sarai, Near IIT 
 
Phone: 011‐26865455/+91‐9871145498 Hauz Khas, New Delhi‐16 
                                                   
                                             Website: www.physicsbyfiziks.com                                                                                          
                                                            Email: fiziks.physics@gmail.com                                                                   19 
fiziks
Institute for NET/JRF, GATE, IIT‐JAM, JEST, TIFR and GRE in PHYSICAL SCIENCES 
 
IIT-JAM-2017
Q36. For the three matrices given below, which one of the choices is correct?
0 1  0 i 1 0 
1    2    3   
1 0  i 0   0 1 
(a)  1 2  i 3 (b)  1 2  i 3

(c)  1 2   2 1  I (d)  3 2  i 1


Ans. : (b)
Solution: These are pauli spin matrix which will satisfied
 1 2  i 3 and  1 2   2 1  0
 40
Q37. The integral of the vector A   ,  , z   cos  ˆ (standard notation for cylindrical

coordinates is used) over the volume of a cylinder of height L and radius R0 is:

(a) 20 R0 L iˆ  ˆj   (b) 0

(c) 40 R0 L ˆj (d) 40 R0 L iˆ


Ans. : (d)
 R0 2 L

Solution: By seeing the options lets calculate  Ad 


40


   cos  cos iˆ  sin  ˆj  drd dz 
V 0 0 0

 2


  Ad  40 R0 L  cos  cos  iˆ  sin  ˆj d  40 R0 L iˆ 
V 0

df
Q38. Which one of the following graphs represents the derivative f   x   of the function
dx
1
f  x  most closely (graphs are schematic and not drawn to scale)?
1  x2

f  x f  x
(a) (b)
x x

                                                                                
Head office  Branch office 
 
fiziks, H.No. 40 D, G.F, Jia Sarai,  Anand Institute of Mathematics, 
 
Near IIT, Hauz Khas, New Delhi‐16  28‐B/6, Jia Sarai, Near IIT 
 
Phone: 011‐26865455/+91‐9871145498 Hauz Khas, New Delhi‐16 
                                                   
                                             Website: www.physicsbyfiziks.com                                                                                          
                                                            Email: fiziks.physics@gmail.com                                                                   20 
fiziks
Institute for NET/JRF, GATE, IIT‐JAM, JEST, TIFR and GRE in PHYSICAL SCIENCES 
 

f  x f  x

(c) (d)
x x

Ans. : (a)
f  x
1 df 2 x
Solution: f  x   and f  x   anti symmetric function but
1  x2 dx 1  x 2
x
f   x  is positive and f  x  is positive

Q39. For the Fourier series of the following function of period 2


0   x  0
f  x  
1 0  x  
the ratio (to the nearest integer) of the Fourier coefficients of the first and the third
harmonic is:
(a) 1 (b) 2 (c) 3 (d) 6
Ans. : (c)

1 1 1
Solution: a0 
2  1 dx  2    2
0


1 1
1 cos nx dx  sin nx 


an  0
0
n 0


1 1  1  2
1 sin nx dx   cos nx    1  1 


bn   
0
n 0
 n  n

b1 2 3
Hence,   3
b3  2

Q40. The volume integral of the function f  r ,  ,    r 2 cos  over the region

 0  r  2, 0     / 3 and 0    2  is………….

(Specify your answer to two digits after the decimal point)


Ans. : 15.07

                                                                                
Head office  Branch office 
 
fiziks, H.No. 40 D, G.F, Jia Sarai,  Anand Institute of Mathematics, 
 
Near IIT, Hauz Khas, New Delhi‐16  28‐B/6, Jia Sarai, Near IIT 
 
Phone: 011‐26865455/+91‐9871145498 Hauz Khas, New Delhi‐16 
                                                   
                                             Website: www.physicsbyfiziks.com                                                                                          
                                                            Email: fiziks.physics@gmail.com                                                                   21 
fiziks
Institute for NET/JRF, GATE, IIT‐JAM, JEST, TIFR and GRE in PHYSICAL SCIENCES 
 
2  / 3 2  /3
25 1  cos 2 
Solution: I   fd    0  r cos  r sin  drd d  5 2  2  0  2
2 2

V 0 0

32 1 24
  cos 2 / 3  cos 00  2    15.07
 /3
I
5 4 5
Q41. Consider two particles moving along the x - axis. In terms of their coordinates x1 and x2 ,

dx1 dx
their velocities are given as  x2  x1 and 2  x1  x2 , respectively. When they start
dt dt
moving from their initial locations of x1  0   1 and x2  0   1 , the time dependence of

both x1 and x2 contains a term of the form e a t , where a is a constant. The value of a
(an integer) is………………
Ans. : 2
Solution: From the given relations we can write
dx1 dx
 2
dt dt
Integrating both sides with respect to t gives
x1   x2  c1

c being a constant of integration


At t  0, x1  1 and x2  1

Hence c  0
Thus, x1   x2 (i)
Using equation (i) the first equation can be written as
dx1 dx
 2 x1  1  2dt
dt x1

 ln x1  2t  ln k1

x1
 ln  2t  x1  t   k1e 2t
k1

Using x1  0   1 , we obtain l  k1 , thus x1  e 2t

                                                                                
Head office  Branch office 
 
fiziks, H.No. 40 D, G.F, Jia Sarai,  Anand Institute of Mathematics, 
 
Near IIT, Hauz Khas, New Delhi‐16  28‐B/6, Jia Sarai, Near IIT 
 
Phone: 011‐26865455/+91‐9871145498 Hauz Khas, New Delhi‐16 
                                                   
                                             Website: www.physicsbyfiziks.com                                                                                          
                                                            Email: fiziks.physics@gmail.com                                                                   22 
fiziks
Institute for NET/JRF, GATE, IIT‐JAM, JEST, TIFR and GRE in PHYSICAL SCIENCES 
 
Using equation (ii) the second equation can be written as
dx2 dx
 2 x2  2  2dt
dt x2
Integrating gives
x2
ln x2  2t  ln k2  ln  e 2t
k2

Thus x2  k2 e 2t

Using x2  0   1 , we obtain k2  1

Thus x2  t   e 2t

Hence the value of a is 2 .


Q42. Consider the differential equation y  2 y  y  0 . If y  0   0 and y   0   1 , then the

value of y  2  is……………….

(Specify your answer to two digits after the decimal point)


Ans. : 0.27

Solution: The characteristic equation is m 2  2m  1  0   m  1  0


2

Thus m  1 is a repeated root


Thus the general solution is
y   c1  c2 x  e  x

since y  0   0  0  c1  c1  0

Thus we can write y  c2 xe  x  y  c2 e  x  xe  x  


since y  0   1

1  c2 1  0   c2  1

y  xe  x
2 2
y  2   2e 2    0.27
e  2.72 
2 2

                                                                                
Head office  Branch office 
 
fiziks, H.No. 40 D, G.F, Jia Sarai,  Anand Institute of Mathematics, 
 
Near IIT, Hauz Khas, New Delhi‐16  28‐B/6, Jia Sarai, Near IIT 
 
Phone: 011‐26865455/+91‐9871145498 Hauz Khas, New Delhi‐16 
                                                   
                                             Website: www.physicsbyfiziks.com                                                                                          
                                                            Email: fiziks.physics@gmail.com                                                                   23 
fiziks
Institute for NET/JRF, GATE, IIT‐JAM, JEST, TIFR and GRE in PHYSICAL SCIENCES 
 
Mechanics and General Properties of Matter
IIT-JAM-2005

Q1. A solid sphere of mass m and radius a is rolling with a linear speed v on a flat surface
without slipping. The magnitude of the angular momentum of the sphere on the surface is
2 7 3
(a) mav (b) mav (c) mav (d) mav
5 5 2
Ans.: (b)
2 2v 2 7 mva
Solution: L  I   mva  ma  mva  mva  mva 
5 a 5 5

IIT-JAM-2007
Q2. In terms of the basic units of mass (M), length (L), time (T) and charge (Q), the
dimensions of magnetic permeability of vacuum (μ0) are
(a) MLQ-2 (b) ML2T-1Q-2 (c) LTQ-1 (d) LT-1Q-1
Ans.: (a)
   0 I Q
2

Solution: F  I  dl  B, B   F  I 0  MLT    0  0  MLQ 2


2 2

d T 
Q3. A projectile is fired from the origin O at an angle of 45 0 from the horizontal. At the
highest point P of its trajectory the radial and transverse components of its acceleration in
y
terms of the gravitational acceleration g are
P
2g g  2g g
(a) a r  , a  (b) a r  , a 
5 5 5 5
g 2g g  2g
(c) a r  , a  (d) a r  , a 
5 5 5 5
Ans.: (d) O x
v 2 sin 2  v 2 sin 2 
Solution: Maximum hmax  and range R  where  
2g g 4

                                                                                
Head office  Branch office 
 
fiziks, H.No. 40 D, G.F, Jia Sarai,  Anand Institute of Mathematics, 
 
Near IIT, Hauz Khas, New Delhi‐16  28‐B/6, Jia Sarai, Near IIT 
 
Phone: 011‐26865455/+91‐9871145498 Hauz Khas, New Delhi‐16 
                                                   
                                             Website: www.physicsbyfiziks.com                                                                                          
                                                            Email: fiziks.physics@gmail.com                                                                   20 
fiziks
Institute for NET/JRF, GATE, IIT‐JAM, JEST, TIFR and GRE in PHYSICAL SCIENCES 
 
h 1
tan   max 
R 2
2
g
From the figure ar   g cos  90      g sin  
5
2 g
a   g cos  
5
Q4. A satellite moves around a planet in a circular orbit at a distance R from its centre. The
time period of revolution of the satellite is T. If the same satellite is taken to an orbit of
radius 4R around the same planet, the time period would be
(a) 8T (b) 4T (c) T/4 (d) T/8
Ans.: (a)
3
T  4R  2
Solution: T 2  R3  2     8  T2  8T
T  R 
IIT-JAM-2008
Q5. EFGH is a thin square plate of uniform density σ and side
y
4a. Four point masses, each of mass m, are placed on the  F
plate as shown in the figure. In the moment of intertia m m
matrix I of the composite system, (a, a,0) (a, a,0)
(a) only Ixy is zero x
m (a,a,0)
(b) only Ixz and Iyz are zero
(a,a,0) m
(c) all the product of inertia terms are zero
H G
(d) none of the product of inertia terms is zero
Ans.: (c)
Solution: I xy    mi xi yi  0, I xz    mi xi zi  0, I yz    mi yi zi  0
i i i

                                                                                
Head office  Branch office 
 
fiziks, H.No. 40 D, G.F, Jia Sarai,  Anand Institute of Mathematics, 
 
Near IIT, Hauz Khas, New Delhi‐16  28‐B/6, Jia Sarai, Near IIT 
 
Phone: 011‐26865455/+91‐9871145498 Hauz Khas, New Delhi‐16 
                                                   
                                             Website: www.physicsbyfiziks.com                                                                                          
                                                            Email: fiziks.physics@gmail.com                                                                   21 
fiziks
Institute for NET/JRF, GATE, IIT‐JAM, JEST, TIFR and GRE in PHYSICAL SCIENCES 
 
Q6. A circular disc (in the horizontal xy-plane) is spinning about a vertical axis through its

center O with a constant angular velocity  . As viewed from the reference frame of the
disc, a particle is observed to execute uniform circular motion, in the anticlockwise sense,
centered at P. when the particle is at the point Q, which of the following figures correctly
 
represents the directions of the Coriolis force Fcor and the centrifugal force Fcfg ?

a  F  r , F cfg P
b  Fr P
Q Q F cfg

O O

 

c  F cfg F cfg
Fr
P d  Fr P
Q Q

O O

 
Ans.: (c)
   
Solution: F centifugal  1  1  r   1 zˆ  1 zˆ  rrˆ   rˆ
   
F corr  2m   v   2m  zˆ  vˆ  rˆ  

IIT-JAM-2009
Q7. A space crew has a life support system that can last only for 1000 hours. What minimum
speed would be required for safe travel of the crew between two space stations separated
by a fixed distance of 1.08 x 1012 km?
c c c c
(a) (b) (c) (d)
3 2 2 5
Ans.: (b)
                                                                                
Head office  Branch office 
 
fiziks, H.No. 40 D, G.F, Jia Sarai,  Anand Institute of Mathematics, 
 
Near IIT, Hauz Khas, New Delhi‐16  28‐B/6, Jia Sarai, Near IIT 
 
Phone: 011‐26865455/+91‐9871145498 Hauz Khas, New Delhi‐16 
                                                   
                                             Website: www.physicsbyfiziks.com                                                                                          
                                                            Email: fiziks.physics@gmail.com                                                                   22 
fiziks
Institute for NET/JRF, GATE, IIT‐JAM, JEST, TIFR and GRE in PHYSICAL SCIENCES 
 
1000  3600  v
Solution:  1.08  1012  1000
v2
1 2
c

105  36  v v c
 1080  1012  cv
v2 v2 2
1 1
c2 c2
Q8. A particle is moving in space with O as the origin. Some possible expression for its
position, velocity and acceleration in cylindrical coordinates (ρ, φ, z) are given below.
Which one of these is correct?
  d d dz
(a) Position vector r  ˆ  ˆ  zzˆ and velocity v  ˆ   ˆ  zˆ
dt dt dt
 d d dz  d 2 d  d  d 2 z
(b) Velocity v  ˆ   ˆ  zˆ and acceleration a  2 ˆ    ˆ   zˆ
dt dt dt dt dt  dt  dt 2
  d d dz
(c) Position vector r  ˆ  zzˆ and velocity v  ˆ   ˆ  zˆ
dt dt dt
  d d dz
(d) Position vector r  ˆ  ˆ  zzˆ and velocity v  ˆ  ˆ   zˆ
dt dt dt
Ans.: (d)
Q9. A thin massless rod of length 2l has equal point masses m attached at its ends (see figure).
The rod is rotating about an axis passing through its centre and making angle θ with it.
 
dL
The magnitude of the rate of change of its angular momentum is
dt
m

(a) 2ml2ω2 sinθ cosθ  l


(b) 2ml2ω2 sinθ

(c) 2ml2ω2 sin2θ

(d) 2ml2ω2 cos2θ

                                                                                
Head office  Branch office 
 
fiziks, H.No. 40 D, G.F, Jia Sarai,  Anand Institute of Mathematics, 
 
Near IIT, Hauz Khas, New Delhi‐16  28‐B/6, Jia Sarai, Near IIT 
 
Phone: 011‐26865455/+91‐9871145498 Hauz Khas, New Delhi‐16 
                                                   
                                             Website: www.physicsbyfiziks.com                                                                                          
                                                            Email: fiziks.physics@gmail.com                                                                   23 
fiziks
Institute for NET/JRF, GATE, IIT‐JAM, JEST, TIFR and GRE in PHYSICAL SCIENCES 
 
Ans.: (a)
      
Solution: J  m  r    r    m  r    r    2m  r  l sin   2ml 2 sin 

dL  
Torque     J  2ml 2 sin  90    sin   2ml 2 cos  sin 
dt
Q10. Moment of inertia of a solid cylinder of mass M , height l and radius R about an axis
(shown in the figure by dashed line) passing through its centre of mass and perpendicular
to its symmetry axis is
1 1
(a) MR 2  Ml 2
4 12
1 1
(b) MR 2  Ml 2 R
2 8
1 1
(c) MR 2  Ml 2 l
2 12
1 1
(d) MR 2  Ml 2
2 4
Ans.: (a)
Solution: If R be the radius, l , the length and M , the mass of the solid cylinder, supposed to be
M
uniform and of a homogeneous composition, we have its mass per unit length = .
l
Now, imagining the cylinder to be made up of a number of discs each of radius R ,
placed adjacent to each other, and considering one such disc of thickness dx and at a
distance x from the centre O of the cylinder, (figure), we have
 M
 dx and radius = R
Mass of the disc = 
  l
M R2
And  M.I. of the disc about its diameter AB  dx  and its M.I. about the parallel
l 4
axis YOY  , passing through the centre O of the cylinder and perpendicular to its axis of
cylindrical symmetry (or its length), in accordance with the principle of parallel axes,
M R2 M
 dx  dx  x 2
l 4 l
                                                                                
Head office  Branch office 
 
fiziks, H.No. 40 D, G.F, Jia Sarai,  Anand Institute of Mathematics, 
 
Near IIT, Hauz Khas, New Delhi‐16  28‐B/6, Jia Sarai, Near IIT 
 
Phone: 011‐26865455/+91‐9871145498 Hauz Khas, New Delhi‐16 
                                                   
                                             Website: www.physicsbyfiziks.com                                                                                          
                                                            Email: fiziks.physics@gmail.com                                                                   24 
fiziks
Institute for NET/JRF, GATE, IIT‐JAM, JEST, TIFR and GRE in PHYSICAL SCIENCES 
 
Y
l /2
A
R x
X
X
O
dx
B
l
Y
Hence, M.I. of the whole cylinder about this axis, i.e. I  twice the integral of the above
l
expression between the limits x  0 and x  ,
2

l/2 M R M 2  2M l / 2  R 2 
2
i.e., I  2 0   dx  x dx   0 
dx  x 2 dx 
 l 4 l  l  4 
l/2
2M  R 2 x x3 
   
l  4 3 0

2M  R 2 l l 3  2 M  R 2l l 3   R2 l 2 
or I          M   
l  4 2 8 3 l  8 24   4 12 

IIT-JAM-2010
Q11. A circular platform is rotating with a uniform angular speed ω counterclockwise about an
axis passing through its centre and perpendicular to its plane as shown in the figure. A
person of mass m walks radially inward with a uniform speed v on the platform. The
magnitude and the direction of the Coriolis force (with respect to the direction along
which the person walks) is

(a) 2mωv towards his left (b) 2mωv towards his front
(c) 2mωv towards his right (d) 2mωv towards his back

                                                                                
Head office  Branch office 
 
fiziks, H.No. 40 D, G.F, Jia Sarai,  Anand Institute of Mathematics, 
 
Near IIT, Hauz Khas, New Delhi‐16  28‐B/6, Jia Sarai, Near IIT 
 
Phone: 011‐26865455/+91‐9871145498 Hauz Khas, New Delhi‐16 
                                                   
                                             Website: www.physicsbyfiziks.com                                                                                          
                                                            Email: fiziks.physics@gmail.com                                                                   25 
fiziks
Institute for NET/JRF, GATE, IIT‐JAM, JEST, TIFR and GRE in PHYSICAL SCIENCES 
 
Ans.: (c)
 
Solution: F  2m  v  2m v  zˆ  rˆ   2m vˆ

2mωv towards his right


Q12.

A particle of mass m, moving with a velocity v  v0 iˆ  ˆj , collides elastically with  
another particle of mass 2m which is at rest initially. Here, v 0 is a constant. Which of the
following statements is correct?
 iˆ  ˆj 
(a) The direction along which the centre of mass moves before collision is   
 2 
(b) The speed of the particle of mass m before collision in the center of mass frame
is 2v0 .
(c) After collision the speed of the particle with mass 2m in the centre of mass frame is

2
v0 .
3
(d) The speed of the particle of mass 2m before collision in the center of mass frame
is 2v0 .
Ans. : (c)

Solution: Velocity of center of mass is


 
mv0 iˆ  ˆj  2m  0


v0 iˆ  ˆj  so option a is wrong
m  3m 3
Velocity of mass m with respect to center of mass before collision

  v0 iˆ  ˆj 
2v0 iˆ  ˆj   
u1  vcm  v0 i  j 
ˆ ˆ  3

3
so speed is v0
4 4
  v0
9 9
8
9

Velocity of mass 2m with respect to center of mass before collision


u2  vcm  0 
v0 iˆ  ˆj

 
v0 iˆ  ˆj   so speed is v0
1 1
  v0
2

2v0
which is
3 3 9 9 9 3
also speed after the collision with respect to center of mass

                                                                                
Head office  Branch office 
 
fiziks, H.No. 40 D, G.F, Jia Sarai,  Anand Institute of Mathematics, 
 
Near IIT, Hauz Khas, New Delhi‐16  28‐B/6, Jia Sarai, Near IIT 
 
Phone: 011‐26865455/+91‐9871145498 Hauz Khas, New Delhi‐16 
                                                   
                                             Website: www.physicsbyfiziks.com                                                                                          
                                                            Email: fiziks.physics@gmail.com                                                                   26 
fiziks
Institute for NET/JRF, GATE, IIT‐JAM, JEST, TIFR and GRE in PHYSICAL SCIENCES 
 
IIT-JAM-2011
Q13. A rain drop falling vertically under gravity gathers moisture from the atmosphere at a rate
dm
given by  kt 2 , where m is the instantaneous mass, t is time and k is a constant. The
dt

dv dm
equation of motion of the rain drop is m v  mg
dt dt

If the drop starts falling at t = 0, with zero initial velocity and initial mass m0 (given m0 =
2gm, k = 12 gm/s3 and g = 1000 cm/s2), the velocity (v) of the drop after one second is
(a) 250 cm/s (b) 500 cm/s (c) 750 cm/s (d) 1000 cm/s
Ans.: (b)
dm kt 3
Solution:  kt 2  m  2
dt 3
dv dm
m v  mg
dt dt

dv
m  vkt 2  mg
dt

dv kt 2 dv 3kt 2
 vg  v  g which is linear differential equation
dt m dt kt 3  6

 
So I .F  exp  g kt 3  6 dt  c  
   
v kt 3  6   g kt 3  6 dt  c t  0, v  0  c  0

g  kt  24t 
4

v
4  kt  6  3

Put 2gm, k = 12 gm/s3 and g = 1000 cm/s2),and t  1


1000 12  24 
v  500cm / sec
4 12  6 

                                                                                
Head office  Branch office 
 
fiziks, H.No. 40 D, G.F, Jia Sarai,  Anand Institute of Mathematics, 
 
Near IIT, Hauz Khas, New Delhi‐16  28‐B/6, Jia Sarai, Near IIT 
 
Phone: 011‐26865455/+91‐9871145498 Hauz Khas, New Delhi‐16 
                                                   
                                             Website: www.physicsbyfiziks.com                                                                                          
                                                            Email: fiziks.physics@gmail.com                                                                   27 
fiziks
Institute for NET/JRF, GATE, IIT‐JAM, JEST, TIFR and GRE in PHYSICAL SCIENCES 
 
a
A particle of mass m is moving in a potential V  x   m 02 x 2 
1
Q14. where  0 and a
2 2mx 2
are positive constants. The angular frequency of small oscillations for the simple
harmonic motion of the particle about a stable minimum of the potential V(x) is
(a) 2 0 (b) 2  0 (c) 4  0 (d) 4 2 0
Ans.: (b)
a
Solution: V  x  
1
m 02 x 2 
2 2mx 2
dV a a
 m02 x  3  0  x 4  2 2
dx mx m 0

d 2V 3a 3m 202
 m02   m0
2
  4m02
dx 2 x  x0
mx 4 m

d 2V
dx 2 x  x0 4m02
   20
m m
IIT-JAM-2012

Q15. Three masses m , 2m and 3m are moving in x-y plane with speeds 3u, 2u and u,
respectively, as shown in the figure. The three masses collide at the same time at P and
stick together. The velocity of the resulting mass would be
y
(a)
u
12

xˆ  3 yˆ  2m

2u
(b)
u
12

xˆ  3 yˆ  m 60
x
3u P 60
(c)
u
12

 xˆ  3 yˆ  u

(d)
u
12

 xˆ  3 yˆ  3m

                                                                                
Head office  Branch office 
 
fiziks, H.No. 40 D, G.F, Jia Sarai,  Anand Institute of Mathematics, 
 
Near IIT, Hauz Khas, New Delhi‐16  28‐B/6, Jia Sarai, Near IIT 
 
Phone: 011‐26865455/+91‐9871145498 Hauz Khas, New Delhi‐16 
                                                   
                                             Website: www.physicsbyfiziks.com                                                                                          
                                                            Email: fiziks.physics@gmail.com                                                                   28 
fiziks
Institute for NET/JRF, GATE, IIT‐JAM, JEST, TIFR and GRE in PHYSICAL SCIENCES 
 
Ans.: (d)
m3u  2m .2u cos 60  3m u cos 60 u
Solution: ucmiˆ    xˆ
6 12
m0  2m .2u sin 60  3m u sin 60 3u
ucmiˆ   yˆ
6m 12
The combined mass will move with same velocity as initial center of mass is moving

v
u
12

 xˆ  3 yˆ 
Q16. The figure shows a thin square sheet of metal of uniform density along with possible
choices for a set of principal axes (indicated by dashed lines) of the moment of inertia,
lying in the plane of the sheet. The correct choice(s) for the principal axes would be

p q r
(a) p, q, and r (b) p and r (c) p and q (d) p only
Ans.: (c)
Solution: I z about center of mass due to p and q are same

So p and q are correct choice.

                                                                                
Head office  Branch office 
 
fiziks, H.No. 40 D, G.F, Jia Sarai,  Anand Institute of Mathematics, 
 
Near IIT, Hauz Khas, New Delhi‐16  28‐B/6, Jia Sarai, Near IIT 
 
Phone: 011‐26865455/+91‐9871145498 Hauz Khas, New Delhi‐16 
                                                   
                                             Website: www.physicsbyfiziks.com                                                                                          
                                                            Email: fiziks.physics@gmail.com                                                                   29 
fiziks
Institute for NET/JRF, GATE, IIT‐JAM, JEST, TIFR and GRE in PHYSICAL SCIENCES 
 
IIT-JAM-2013
  2 x2 
Q17. A particle is released at x  1 in a force field F  x    2  eˆ x , x  0 . Which one of
x 2 

the following statements is FALSE?



(a) F x  is conservative
(b) The angular momentum of the particle about the origin is constant
(c) The particle moves towards x  2
(d) The particle moves towards the origin
Ans.: (c)
  2 x2 
Solution: F  x    2  eˆ x , x  0
x 2 
 
  F  0 are zero so force is conservative
 
r  F  0 so angular momentum is conserve
 2 x2 
For equilibrium point F  0  F   2    0  x   2, 2 x  0 so equilibrium
x 2 

point is 2
dF 4 2 x
To check stable and unstable equilibrium point 0 3 
dx x 2
dF 4 2 x 4
At x  2 0 3    2  ve so it is unstable point so particle
dx x 2 2 2

moves away from point x  2 so it is obvious that it will moves towards origin
Q18. If the dimensions of mass, length, time and charge are M , L, T and C respectively, the

dimensions of the magnetic induction field B is
(a) ML2T 1C 1 (b) MT 1C 1 (c) L2T 1C (d) L1T 1C
Ans.: (b)
   C 
Solution: F  I  dl  B,  MLT 2    LB  B  MT 1C 1
T 

                                                                                
Head office  Branch office 
 
fiziks, H.No. 40 D, G.F, Jia Sarai,  Anand Institute of Mathematics, 
 
Near IIT, Hauz Khas, New Delhi‐16  28‐B/6, Jia Sarai, Near IIT 
 
Phone: 011‐26865455/+91‐9871145498 Hauz Khas, New Delhi‐16 
                                                   
                                             Website: www.physicsbyfiziks.com                                                                                          
                                                            Email: fiziks.physics@gmail.com                                                                   30 
fiziks
Institute for NET/JRF, GATE, IIT‐JAM, JEST, TIFR and GRE in PHYSICAL SCIENCES 
 
Q19. The path of a particle of mass m , moving under the influence of a central force, in plane
polar coordinates is given by r  r0 e k , where r0 and k are positive constants of
appropriate dimensions. The angular momentum of the particle is L and its total energy
is zero. The potential energy functions V r  , in terms of m, L and k is……………..
Ans.:

L2u 2  d 2u  1
Solution:   2 u  f  
m  d  u

1  k d 2u 1 k 2 e  k
r  r0 e k , u  e ,    k   k  e  k

r0 d 2 r0 r0

L2u 2  d 2u  1
  2 u  f  
m  d  u

 L2u 2  k 2 e  k k  k   L2 e 2 k  k 2 e  k k  k  L2 e 3k 2

m  r0
 e 
r0 m
. 2 
r0  r0
 e 
r0 mr0 3
1
k k = f  
u
 
 

L2 3
f 1/ u    u  k  k  1 
m
m 1
f r   
L k  k  1 r 3
2

dV m 1 3m 1
  2  V   c
dr L k  k  1 r 3 L2 k  k  1 r 2

1 2 L2 3m 1
E mr   2 c
2 2mr 2
L k  k  1 r 2

L L
r  r0 e k r  ro e  k .  r  ro e  k . 2 
mr mr

L2 L2 3m 1
E   2  c it is given E  0
2mr 2
2mr 2
L k  k  1 r 2

r 0; 0c

                                                                                
Head office  Branch office 
 
fiziks, H.No. 40 D, G.F, Jia Sarai,  Anand Institute of Mathematics, 
 
Near IIT, Hauz Khas, New Delhi‐16  28‐B/6, Jia Sarai, Near IIT 
 
Phone: 011‐26865455/+91‐9871145498 Hauz Khas, New Delhi‐16 
                                                   
                                             Website: www.physicsbyfiziks.com                                                                                          
                                                            Email: fiziks.physics@gmail.com                                                                   31 
fiziks
Institute for NET/JRF, GATE, IIT‐JAM, JEST, TIFR and GRE in PHYSICAL SCIENCES 
 
IIT-JAM-2014
Q20. A spherical ball of ice has radius R0 and is rotating with an angular speed  about an

axis passing through its centre. At time t  0, it starts acquiring mass because the
moisture (at rest) around it starts to freeze on it uniformly. As a result its radius increases
as Rt   R0   t , where  is a constant. The curve which best describes its angular
speed with time is

 
(a) (b)

t t

(c)  (d) 

t t

Ans.: (b)
Q21. The moment of inertia of a disc about one of its diameters is I M . The mass per unit area
of the disc is proportional to the distance from its centre. If the radius of the disc is R and
its mass is M, the value of I M is
1 2 3 3
(a) MR 2 (b) MR 2 (c) MR 2 (d) MR 2
2 5 10 5
Ans.: (c)
Q22. Two points N and S are located in the northern and southern hemisphere, respectively, on
the same longitude. Projectiles P and Q are fired from N and S, respectively, towards
each other. Which of the following options is correct for the projectiles as they approach
the equator?
(a) Both P and Q will move towards the east
(b) Both P and Q will move towards the west
(c) P will move towards the east and Q towards the west
(d) P will move towards the west and Q towards the east
Ans.: (b)
                                                                                
Head office  Branch office 
 
fiziks, H.No. 40 D, G.F, Jia Sarai,  Anand Institute of Mathematics, 
 
Near IIT, Hauz Khas, New Delhi‐16  28‐B/6, Jia Sarai, Near IIT 
 
Phone: 011‐26865455/+91‐9871145498 Hauz Khas, New Delhi‐16 
                                                   
                                             Website: www.physicsbyfiziks.com                                                                                          
                                                            Email: fiziks.physics@gmail.com                                                                   32 
fiziks
Institute for NET/JRF, GATE, IIT‐JAM, JEST, TIFR and GRE in PHYSICAL SCIENCES 
 
Q23. Two particles A and B of mass m and one particle C of mass M are kept on the x axis in
the order ABC. Particle A is given a velocity viˆ. Consequently there are two collisions,
both of which are completely inelastic. If the net energy loss because of these collisions is
7
of the initial energy, the value of M is (ignore frictional losses)
8
(a) 8 m (b) 6 m (c) 4 m (d) 2 m
Ans.: (b)
Q24. What is the maximum height above the
A
dashed line attained by the water stream
coming out at B from a thin tube of the h
L B
water tank assembly shown in the 
y0
figure? Assume h = 10 m, L = 2 m and
  30.
(a) 10 m (b) 2 m (c) 1.2 m (d) 3.2 m
Ans.: (d)
Q25. At an instant shown, three point masses m, 2m and 3m rest on a horizontal surface, and
are at the vertices of an equilateral triangle
3m
of unit side length. Assuming that G is the
gravitational constant, the magnitude and
direction of the torque on the mass 3m,
about the point O, at that instant is m 2m
O
3
(a) Zero (b) G 3m 2 , going into the paper
2
3
(c) 3G 3m 2 , coming out of the paper (d) G 3m 2 , going into the paper
4
Ans.: (d)

                                                                                
Head office  Branch office 
 
fiziks, H.No. 40 D, G.F, Jia Sarai,  Anand Institute of Mathematics, 
 
Near IIT, Hauz Khas, New Delhi‐16  28‐B/6, Jia Sarai, Near IIT 
 
Phone: 011‐26865455/+91‐9871145498 Hauz Khas, New Delhi‐16 
                                                   
                                             Website: www.physicsbyfiziks.com                                                                                          
                                                            Email: fiziks.physics@gmail.com                                                                   33 
fiziks
Institute for NET/JRF, GATE, IIT‐JAM, JEST, TIFR and GRE in PHYSICAL SCIENCES 
 
Q26. A stationary source (see figure below) emits sound
waves of frequency f towards a wall. If an observer Source
Wall
moving with speed u in a direction perpendicular to
30 

9 30
the wall, measures a frequency f   f at the instant
8
Observer u
shown, then u is related to the speed of sound Vs as

(a) Vs (b) Vs / 2 (c) Vs / 4 (d) Vs / 8


Ans.: (c)
Q27. Two frames, O and O , are in relative motion as shown. O  c/2
is moving with speed c/2, where c is the speed of light. In
O O
frame O, two separate events occur at x1 , t1  and  x 2 , t 2 . x
In frame O , these events occur simultaneously. The value
of  x 2  x1  / t 2  t1  is
(a) c / 4 (b) c / 2 (c) 2c (d) c
Ans.: (c)
IIT-JAM-2015
Q28. A mass m , lying on a horizontal, frictionless surface, is connected to one end of a spring.
The other end of the spring is connected to a wall, as shown in the figure. At t  0 , the
mass is given an impulse.

m Impulse

The time dependence of the displacement and the velocity of the mass (in terms of non-
zero constants A and B ) are given by
(a) xt   A sin t , vt   B cos t (b) xt   A sin t , vt   B sin t
(c) xt   A cos t , vt   B sin t (d) xt   A cos t , vt   B cos t

                                                                                
Head office  Branch office 
 
fiziks, H.No. 40 D, G.F, Jia Sarai,  Anand Institute of Mathematics, 
 
Near IIT, Hauz Khas, New Delhi‐16  28‐B/6, Jia Sarai, Near IIT 
 
Phone: 011‐26865455/+91‐9871145498 Hauz Khas, New Delhi‐16 
                                                   
                                             Website: www.physicsbyfiziks.com                                                                                          
                                                            Email: fiziks.physics@gmail.com                                                                   34 
fiziks
Institute for NET/JRF, GATE, IIT‐JAM, JEST, TIFR and GRE in PHYSICAL SCIENCES 
 
Ans.: (a)
Solution: At time t  0 , the mass ‘ m ’ is at rest. Thus, displacement will be zero at time t  0 .
 x  A sin t 

dx
Velocity is v   A cos t  B cos  t
dt
Thus, x  A sin  t and V  t   B cos t

Q29. A proton from outer space is moving towards earth with velocity 0.99 c as measured in
earth’s frame. A spaceship, traveling parallel to the proton, measures proton’s velocity to
be 0.97 c . The approximate velocity of the spaceship in the earth’s frame, is
(a) 0.2 c (b) 0.3 c (c) 0.4 c (d) 0.5 c
Ans.: (d)
Solution: Velocity of proton w.r.t. spaceship  0.97 c
s s
 u x  0.99 c, v  v, u x  0.97 c E p  0.99 c
v
u x  v 0.99 c  v
 ux   0.97 c   v  0.5 c
ux v 0.97v p  0.99 c
1 2 1
c c

Q30. A satellite moves around the earth in a circular orbit of radius R centered at the earth. A
second satellite moves in an elliptic orbit of major axis 8 R , with the earth at one of the
foci. If the former takes 1 day to complete a revolution, the latter would take
(a) 21.6 days (b) 8 days (c) 3 hours (d) 1.1 hour
Ans.: (a)
2
T   R 
3

  T2   8  T1  22 days
3/ 2
Solution:  1   
 T2   8 R 

                                                                                
Head office  Branch office 
 
fiziks, H.No. 40 D, G.F, Jia Sarai,  Anand Institute of Mathematics, 
 
Near IIT, Hauz Khas, New Delhi‐16  28‐B/6, Jia Sarai, Near IIT 
 
Phone: 011‐26865455/+91‐9871145498 Hauz Khas, New Delhi‐16 
                                                   
                                             Website: www.physicsbyfiziks.com                                                                                          
                                                            Email: fiziks.physics@gmail.com                                                                   35 
fiziks
Institute for NET/JRF, GATE, IIT‐JAM, JEST, TIFR and GRE in PHYSICAL SCIENCES 
 
Q31. An observer is located on a horizontal, circular turntable which rotates about a vertical
axis passing through its center, with a uniform angular speed of 2 rad/sec . A mass of
10 grams is sliding without friction on the turntable. At an instant when the mass is at a
distance of 8 cm from the axis it is observed to move towards the center with a speed of
6 cm/sec. The net force on the mass, as seen by the observer at that instant, is
(a) 0.0024 N (b) 0.0032 N (c) 0.004 N (d) 0.006 N
Ans.: (c)
Solution: Two forces will act on the particle
First coriolis force Fc  2m(  v)  240 105 N (in tangential direction)

Another force is centrifugal force Fr  m 2 r  320  105 N (in radial direction)

Total force F  Fc2  Fc2 r  0.04 N

Q32. Seven uniform disks, each of mass m and radius r , are inscribed
inside a regular hexagon as shown. The moment of inertia of this
system of seven disks, about an axis passing through the central disk
and perpendicular to the plane of the disks, is
7 2
(a) mr (b) 7mr 2
2
13 55
(c) mr2 (d) mr2
2 2
Ans.: (d)
mr 2  mr 2  mr 2 54mr 2 55mr 2
Solution: 6  4mr 2    
2  2  2 2 2

                                                                                
Head office  Branch office 
 
fiziks, H.No. 40 D, G.F, Jia Sarai,  Anand Institute of Mathematics, 
 
Near IIT, Hauz Khas, New Delhi‐16  28‐B/6, Jia Sarai, Near IIT 
 
Phone: 011‐26865455/+91‐9871145498 Hauz Khas, New Delhi‐16 
                                                   
                                             Website: www.physicsbyfiziks.com                                                                                          
                                                            Email: fiziks.physics@gmail.com                                                                   36 
fiziks
Institute for NET/JRF, GATE, IIT‐JAM, JEST, TIFR and GRE in PHYSICAL SCIENCES 
 
SECTION–B: MSQ
Q33. A particle of mass m is moving in x  y plane. At any given time t , its position vector is

given by r  t   A cos t i  B sin t ˆj where A, B and  are constants with A  B .

Which of the following statements are true?


(a) Orbit of the particle is an ellipse
(b) Speed of the particle is constant
(c) At any given time t the particle experiences a force towards origin
(d) The angular momentum of the particle is m ABkˆ
Ans.: (a), (c) and (d)

Solution: (a) r  t   A cos  t iˆ  B sin  t ˆj  x  A cos t , y  B sin t

x y x2 y2
  cos t ,  sin t  2  2  1 (Ellipse)
A B A B

dr
(b)   A sin t iˆ  B cos t ˆj
dt

dr
Speed   A2 2 sin 2 t  B 2 2 cos 2 t . Speed is function of time, so not constant.
dt
2
dr 
(c) 2   A 2 cos t iˆ  B 2 sin t ˆj   2 r . Force act towards origin.
dt
 i ˆj kˆ
   
(d) L   r  p   m  A cos  t B sin  t 0  L  m ABk
  A sin  t B cos  t 0

                                                                                
Head office  Branch office 
 
fiziks, H.No. 40 D, G.F, Jia Sarai,  Anand Institute of Mathematics, 
 
Near IIT, Hauz Khas, New Delhi‐16  28‐B/6, Jia Sarai, Near IIT 
 
Phone: 011‐26865455/+91‐9871145498 Hauz Khas, New Delhi‐16 
                                                   
                                             Website: www.physicsbyfiziks.com                                                                                          
                                                            Email: fiziks.physics@gmail.com                                                                   37 
fiziks
Institute for NET/JRF, GATE, IIT‐JAM, JEST, TIFR and GRE in PHYSICAL SCIENCES 
 
Q34. A rod is hanging vertically from a pivot. A partic1e traveling in horizontal
direction, collides with the rod as shown in the figure. For the rod-particle
system, consider the linear momentum and the angular momentum about
the pivot .Which of the following statements are NOT true?
(a) Both linear momentum and angular momentum are conserved
(b) Linear momentum is conserved but angular momentum is not
(c) Linear momentum is not conserved but angular momentum is conserved
(d) Neither linear momentum nor annular momentum are conserved
Ans.: (b), (c) and (d)
Q35. Muons are elementary particles produced in the upper atmosphere. They have a life time
of 2.2s . Consider muons which are traveling vertically towards the earth’s surface at a
speed of 0.998c . For an observer on earth, the height of the atmosphere above the
surface of the earth is 10.4 km . Which of the following statements are true?
(a) The muons can never reach earth’s surface
(b) The apparent thickness of earth’s atmosphere in muon’s frame of reference is 0.96 km
(c) The lifetime of muons in earth’s frame of reference is 34.8s
(d) Muons traveling at a speed greater than 0.998 c reach the earth’s surface
Ans.: (c) and (d)
t0 2.2 106
Solution: t   t   34.8 106 sec
v 1   0.998 
2 2
1
c2
Now distance will be  t  v  34.8 106  0.998  3 108  10.4192 km

Apparent thickness X  t  v  2.2 106  0.998  3 108  0.658 km

                                                                                
Head office  Branch office 
 
fiziks, H.No. 40 D, G.F, Jia Sarai,  Anand Institute of Mathematics, 
 
Near IIT, Hauz Khas, New Delhi‐16  28‐B/6, Jia Sarai, Near IIT 
 
Phone: 011‐26865455/+91‐9871145498 Hauz Khas, New Delhi‐16 
                                                   
                                             Website: www.physicsbyfiziks.com                                                                                          
                                                            Email: fiziks.physics@gmail.com                                                                   38 
fiziks
Institute for NET/JRF, GATE, IIT‐JAM, JEST, TIFR and GRE in PHYSICAL SCIENCES 
 
SECTION–C: NAT (Numerical Answer Type)
Q36. A rod is moving with a speed of 0.8c in a direction at 60 o to its own length. The
percentage contraction in the length of the rod is………….
Ans.: 9

v2 1 l 3
Solution: lx  l0 x 1  2  l0 cos  1   0.8  lx  l0   0.6  0.3l0 and l y  l0 sin   0
2

c 2 2
2
 3l0  3
 0.3l0 
2
New length l      l0 0.09   0.916 l0
 2  4

% change in length
1  0.91 l0  100  0.09  100  9%
l0
Q37. A uniform disk of mass m and radius R rolls, without slipping, down a fixed plane
inclined at an angle 30 o to the horizontal. The linear acceleration of the disk (in m / sec 2 )
is………………
Ans.: 3.266
Solution: Equation of Motion mg sin   f  ma
Torque  fR  I 

I  a mR 2 
mg sin    ma ,    , I  
R  R 2 

g
a  3.266
3
Q38. A nozzle is in the shape of a truncated cone, as shown in the figure.
The area at the wide end is 25cm 2 and the narrow end has an area

of 1 cm 2 . Water enters the wider end at a rate of 500 gm / sec . The 50 cm

height of the nozzle is 50 cm and it is kept vertical with the wider


end at the bottom. The magnitude of the pressure difference in kPa
( 1 kPa  10 3 N / m 2 ) between the two ends of the nozzle is………….

                                                                                
Head office  Branch office 
 
fiziks, H.No. 40 D, G.F, Jia Sarai,  Anand Institute of Mathematics, 
 
Near IIT, Hauz Khas, New Delhi‐16  28‐B/6, Jia Sarai, Near IIT 
 
Phone: 011‐26865455/+91‐9871145498 Hauz Khas, New Delhi‐16 
                                                   
                                             Website: www.physicsbyfiziks.com                                                                                          
                                                            Email: fiziks.physics@gmail.com                                                                   39 
fiziks
Institute for NET/JRF, GATE, IIT‐JAM, JEST, TIFR and GRE in PHYSICAL SCIENCES 
 
Ans.: 17.5
Solution: According to Bernoulli’s equation
1 1 Pt , Vt , At
Pb   ghb  Vb2  Pt   ght  Vt 2
2 2

 Pb  Pt   g  ht  hb    Vt 2  Vb2
1
 
2 ht  hb  50 cm
Now given  AV
t t  500 gm / sec

500  103 kg / sec 500  103 kg / sec


 Vt  
  At 1000 kg / m3  104 m 2 Pb , Vb , Ab
 Vt  5 m / sec
According to equation of continuity
At 1 cm 2
t t  AbVb  Vb 
AV Vt  Vb   5 m / sec  0.2 m / sec
Ab 25 cm 2

1
 P  Pb  Pt  1000  10  50  10 2   1000  52   0.2 
2

2  

 P  5000  500  25  0.04   5000  12480  17480 N / m 2  P  17.5 kPa

Q39. A block of mass 2 kg is at rest on a horizontal table The coefficient of friction between
the block and the table is 0.1. A horizontal force 3 N is applied to the block The speed of
the block (in m/s) after it has moved a distance 10 m is…………….
Ans.: 3.225
Solution: f r   N  0.1  2  10  2 N  m  2 kg
Applied force is more than friction
1 1
ma  F   N  3  2  1  a    0.5 m / s 2
m 2
 v  u 2  2as  v  2as  2  0.5 10  10  3.225 m / s
 u  0, s  10m

                                                                                
Head office  Branch office 
 
fiziks, H.No. 40 D, G.F, Jia Sarai,  Anand Institute of Mathematics, 
 
Near IIT, Hauz Khas, New Delhi‐16  28‐B/6, Jia Sarai, Near IIT 
 
Phone: 011‐26865455/+91‐9871145498 Hauz Khas, New Delhi‐16 
                                                   
                                             Website: www.physicsbyfiziks.com                                                                                          
                                                            Email: fiziks.physics@gmail.com                                                                   40 
fiziks
Institute for NET/JRF, GATE, IIT‐JAM, JEST, TIFR and GRE in PHYSICAL SCIENCES 
 
Q40. A homogeneous semi-circular plate of radius R  3m is shown in the figure. The
distance of the center of mass of the p1ate (in meter) from the point O is……….

0
Ans.: 1.3 3m

Solution: In problem R  3m
The area of the shaded part is  rdr . The area of the
r  dr r
plate is  R 2 / 2 . As the plate is uniform, the mass per
R
M
unit area is . Hence the mass of the
 R2 / 2
semicircular element
M 2Mrdr
 rdr   2
R /2
2
R
The y - coordinate of the centre of mass of this wire is 2r /  . The y - coordinate of the
centre of the plate is, therefore,
R
1  2r   2 Mr  1 4 M R 3 4 R 4
M 0     R 2
Y   dr       1.3
 M R 3 3  
2

The x - coordinate of the centre of mass is zero by symmetry.

                                                                                
Head office  Branch office 
 
fiziks, H.No. 40 D, G.F, Jia Sarai,  Anand Institute of Mathematics, 
 
Near IIT, Hauz Khas, New Delhi‐16  28‐B/6, Jia Sarai, Near IIT 
 
Phone: 011‐26865455/+91‐9871145498 Hauz Khas, New Delhi‐16 
                                                   
                                             Website: www.physicsbyfiziks.com                                                                                          
                                                            Email: fiziks.physics@gmail.com                                                                   41 
fiziks
Institute for NET/JRF, GATE, IIT‐JAM, JEST, TIFR and GRE in PHYSICAL SCIENCES 
 
IIT-JAM-2016
Q41. A particle is moving in a plane with a constant radial velocity of 12 m / s and constant
angular velocity of 2 rad / s . When the particle is at a distance r  8 m from the origin,
the magnitude of the instantaneous velocity of the particle in m / s is
(a) 8 15 (b) 20 (c) 2 37 (d) 10
Ans.: (b)
Solution: vr  12 m / s v   r  2  8  16m / sec

v  vr2  v2  144  256  400  20m / sec

Q42. A cylindrical rod of length L has a mass density distribution given by


 x
  x   0 1   , where x is measured from one end of the rod and 0 is a constant of
L 
appropriate dimensions. The centre of mass of the rod is
5 4 1 1
(a) L (b) L (c) L (d) L
9 9 9 2
Ans.: (a)
L L
 x L
L x2 x3 L2 L3 1 1
 xdm  x 0 1   dx  
 x  dx  L2   
Solution: xcm  0L  0L  0L  L   0 2 3L  2 3L   2 3 
 x L x2 L2  1
 dm   dx  0 1   dx  x  L  L 1  
0 0 0  L 0 2L 2L  2
5
L
6 5
 xcm   L
3 9
2

                                                                                
Head office  Branch office 
 
fiziks, H.No. 40 D, G.F, Jia Sarai,  Anand Institute of Mathematics, 
 
Near IIT, Hauz Khas, New Delhi‐16  28‐B/6, Jia Sarai, Near IIT 
 
Phone: 011‐26865455/+91‐9871145498 Hauz Khas, New Delhi‐16 
                                                   
                                             Website: www.physicsbyfiziks.com                                                                                          
                                                            Email: fiziks.physics@gmail.com                                                                   42 
fiziks
Institute for NET/JRF, GATE, IIT‐JAM, JEST, TIFR and GRE in PHYSICAL SCIENCES 
 
Q43. An incompressible, non-viscous fluid is injected into a conical pipe
at its orifice as schematically shown in the figure. The pressure at the
orifice of area A0 is P0 . Neglecting the effect of gravity and  /4
x
assuming streamline flow, which one of the following plots correctly
predicts the pressure along axis of the cone?

X 0  P0 ; v0 
(a) (b)
P P

P0 P0

X0 X X0 X
(c) (d)

P P

P0 P0

X0 X X0 X
Ans. : (a)
Solution: The area cross-section at B is Ax

2
 A0
Ax   r 2    x tan   /4
 4
A B
Ax   x 2

A0 v0 A0 v0 Av O x0 x
Velocity at B is A0 v0  Ax vx  vx    vx  0 20  0 , v0 
P  Px , vx 
Ax x 2
x
According to Bernauli equation,

1  2 A02 v02 
P0   v0  Px   vx  Px  P0    v0  vx   P0    v0  2 4 
1 2 1 2 1 2 2

2 2 2 2   x 

                                                                                
Head office  Branch office 
 
fiziks, H.No. 40 D, G.F, Jia Sarai,  Anand Institute of Mathematics, 
 
Near IIT, Hauz Khas, New Delhi‐16  28‐B/6, Jia Sarai, Near IIT 
 
Phone: 011‐26865455/+91‐9871145498 Hauz Khas, New Delhi‐16 
                                                   
                                             Website: www.physicsbyfiziks.com                                                                                          
                                                            Email: fiziks.physics@gmail.com                                                                   43 
fiziks
Institute for NET/JRF, GATE, IIT‐JAM, JEST, TIFR and GRE in PHYSICAL SCIENCES 
 
1  A 
2
Px  P0   v02 1  4 0 4 
2   x 
Graph (a) correctly represent the variation of Px w.r.t. x

Q44. A particle moves in a circular path in the xy -plane centered at the origin. If the speed of
the particle is constant, then its angular momentum
(a) about the origin is constant both in magnitude and direction
(b) about  0, 0,1 is constant in magnitude but not in direction

(c) about  0, 0,1 varies both in magnitude and direction

(d) about  0, 0,1 is constant in direction but not in magnitude

Ans.: (a) and (b)


Solution: Angular momentum will constant in x  y plane

IIT-JAM-2017
Q45. Consider a uniform thin circular disk of radius R and mass M . A concentric square of
side R / 2 is cut out from the disk (see figure). What is the moment of inertia of the
resultant disk about an axis passing through the centre of the disk and perpendicular to it?
MR 2  1  MR 2  1 
(a) I  1  48  (b) I  1  48 
4   2  
R
MR 2  1  MR 2  1 
(c) I  1  24  (d) I  1  24 
4   2   R/2
Ans. : (b)

Solution: I  I disc  I square 


MR 2 M a  a

' 2 2
 
2 12
M R R M R
M'     and a 
 R 2 2 4
2
2
MR 2  1 
I 1  48 
2  

                                                                                
Head office  Branch office 
 
fiziks, H.No. 40 D, G.F, Jia Sarai,  Anand Institute of Mathematics, 
 
Near IIT, Hauz Khas, New Delhi‐16  28‐B/6, Jia Sarai, Near IIT 
 
Phone: 011‐26865455/+91‐9871145498 Hauz Khas, New Delhi‐16 
                                                   
                                             Website: www.physicsbyfiziks.com                                                                                          
                                                            Email: fiziks.physics@gmail.com                                                                   44 
fiziks
Institute for NET/JRF, GATE, IIT‐JAM, JEST, TIFR and GRE in PHYSICAL SCIENCES 
 
Q46. The linear mass density of a rod of length L varies from one end to the other as
 x2 
0 1  2  , where x is the distance from one end with tensions T1 and T2 in them (see
 L 
figure), and 0 is a constant. The rod is suspended from a ceiling by two massless
strings. Then, which of the following statement(s) is (are) correct?
20 L
(a) The mass of the rod is
3
T1 T2
9L
(b) The centre of gravity of the rod is located at x 
16 0 L x
70 Lg
(c) The tension T1 in the left string is
12
30 Lg
(d) The tension T2 in the right string is
2
Ans. : (b), (c)
 x2  4 L
L
Solution: The mass of rod is m   0 1  2  dx  0 so (a) is wrong
0  L  3

 x2  L

0 x0 1  L2  dx 9L
The centre of gravity of the rod is located at xcm  L 
 x2  16
0 0 1  L2 dx
40 Lg
Force equation T1  T2  and torque equation
3
9L  9L  9L 7L 9
T1   T2   L    T1   T2   T2  T1 
16  16  16 16 7
9 4 Lg
put value of T2  T1 in equation T1  T2  0
7 3
16 4 Lg 7 Lg
T1  0  T1  0
7 3 12
9 9 7 Lg 90 Lg
T2   T1   0 
7 7 12 12
                                                                                
Head office  Branch office 
 
fiziks, H.No. 40 D, G.F, Jia Sarai,  Anand Institute of Mathematics, 
 
Near IIT, Hauz Khas, New Delhi‐16  28‐B/6, Jia Sarai, Near IIT 
 
Phone: 011‐26865455/+91‐9871145498 Hauz Khas, New Delhi‐16 
                                                   
                                             Website: www.physicsbyfiziks.com                                                                                          
                                                            Email: fiziks.physics@gmail.com                                                                   45 
fiziks
Institute for NET/JRF, GATE, IIT‐JAM, JEST, TIFR and GRE in PHYSICAL SCIENCES 
 
Q47. An object of mass m with non-zero angular momentum  is moving under the influence
of gravitational force of a much larger mass (ignore drag). Which of the following
statement(s) is (are) correct?
(a) If the total energy of the system is negative, then the orbit is always circular
(b) The motion of m always occurs in a two-dimensional plane
(c) If the total energy of the system is 0 , then the orbit is a parabola
(d) If the area of the particle’s bound orbit is S , then its time period is 2mS / 
Ans. : (b), (c) and (d)

2 El 2
Solution: The eccentricity of curve e  1  where k  Gm1m2 and E is energy .
mk 2
If total energy is negative then orbit can be either elliptical or circular so (a) is wrong
In two body central force problem motion is confine in plane so (b) is correct
If the total energy of the system is 0 , then the orbit is a parabola one can calculate
e  1 so (c) is correct
dA l S l 2mS
From second law of thermodynamics    T  so (d) is correct
dt 2m T 2m l
Q48. A particle of mass m fixed in space is observed from a frame rotating about its z - axis
with angular speed  . The particle is in the frame’s xy plane at a distance R from its
 
origin. If the Coriolis and centrifugal forces on the particle are FCOR and FCFG ,
respectively, then (all the symbols have their standard meaning and refer to the rotating
frame),
   
(a) FCOR  FCFG  0 (b) FCOR  FCFG  m 2 Rrˆ
 
(c) FCOR  2m 2 Rrˆ (d) FCFG  m 2 Rrˆ
Ans. : (b), (c)
 
Solution: Fcorr  Fcentipetal  mr 2 Rrˆ
 
Fcor  2m   v  , v   r  2m 2 R

                                                                                
Head office  Branch office 
 
fiziks, H.No. 40 D, G.F, Jia Sarai,  Anand Institute of Mathematics, 
 
Near IIT, Hauz Khas, New Delhi‐16  28‐B/6, Jia Sarai, Near IIT 
 
Phone: 011‐26865455/+91‐9871145498 Hauz Khas, New Delhi‐16 
                                                   
                                             Website: www.physicsbyfiziks.com                                                                                          
                                                            Email: fiziks.physics@gmail.com                                                                   46 
fiziks
Institute for NET/JRF, GATE, IIT‐JAM, JEST, TIFR and GRE in PHYSICAL SCIENCES 
 
Q49. A particle of unit mass is moving in a one-dimensional potential V  x   x 2  x 4 . The

minimum mechanical energy (in the same units as V  x  ) above which the motion of the

particle cannot be bounded for any given initial condition is……………


(Specify your answer to two digits after the decimal point)
Ans. : 0.25
Solution: V  x   x 2  x 4

1
The potential is mimima at x  0 and maxima at x   fig
2
The value V  x   0 x  0

1
The value V  x   0.25 x  
2
minimum mechanical energy above which the motion of the particle cannot be bounded
is E f  .25

Q50. Sand falls on a conveyor belt at the rate of 1.5 kg / s . If the belt is moving with a constant
speed of 7 m / s , the power needed to keep the conveyor belt running is…………….
(Specify your answer in Watts to two digits after the decimal point)
Ans. : 73.4
1 
d  mv 2 
dW   1 .v 2 . dm  2 mv dv if v is constant dv  0
 
2
Solution: P 
dt dt 2 dt 2 dt dt
1 dm 2 1
P1  . .v  1.5  49  36.7 watt
2 dt 2
1 2 v v
Work done due to friction W   mg .d ,d  at , v  at  t  
2 a g
2
1  v 
d  g  
2  g 

                                                                                
Head office  Branch office 
 
fiziks, H.No. 40 D, G.F, Jia Sarai,  Anand Institute of Mathematics, 
 
Near IIT, Hauz Khas, New Delhi‐16  28‐B/6, Jia Sarai, Near IIT 
 
Phone: 011‐26865455/+91‐9871145498 Hauz Khas, New Delhi‐16 
                                                   
                                             Website: www.physicsbyfiziks.com                                                                                          
                                                            Email: fiziks.physics@gmail.com                                                                   47 
fiziks
Institute for NET/JRF, GATE, IIT‐JAM, JEST, TIFR and GRE in PHYSICAL SCIENCES 
 
1  v   1 2
2
W   mg .   g    mv
2   g   2
 
1 dm 2 1
P2  . .v  1.5  49  36.7 watt
2 dt 2
P  P1  P2  36.7 watt  36.7 watt  73.4 watt.

Q51. In planar polar co-ordinates, an object’s position at time t is given

 
as  r ,    et m, 8 t rad . The magnitude of its acceleration in m / s 2 at t  0 (to the

nearest integer) is……………….


Ans. : 9
r   2 r  et  8  et at t  0 , 1  8  1  7 m / sec 2
Solution: ar  

a  r  2r  et  0  2et 8  2 8 a   7 2  4  8  49  32  81  9m / sec 2

Q52. At t  0 , a particle of mass m having velocity v0 starts moving through a liquid kept in a

 dx 
horizontal tube and experiences a drag force  Fd   k  . It covers a distance L before
 dt 
coming to rest. If the times taken to cover the distances L / 2 and L / 4 are t2 and t4

respectively, then the ratio t2 / t4 (ignoring gravity) is……….


(Specify your answer to two digits after the decimal point)
Ans. : 2.41
Solution: Using Newton’s second law we obtain
dx d 2x d 2x dx d 2 x k dx
k  m 2 m 2  k 0  2  0
dt dt dt dt dt m dt
The characteristic equation is
k  k k
2    0        0 ,   0 and   
m  m m
General solution:
k
 t
x  t   c1  c2 e m
(i)
                                                                                
Head office  Branch office 
 
fiziks, H.No. 40 D, G.F, Jia Sarai,  Anand Institute of Mathematics, 
 
Near IIT, Hauz Khas, New Delhi‐16  28‐B/6, Jia Sarai, Near IIT 
 
Phone: 011‐26865455/+91‐9871145498 Hauz Khas, New Delhi‐16 
                                                   
                                             Website: www.physicsbyfiziks.com                                                                                          
                                                            Email: fiziks.physics@gmail.com                                                                   48 
fiziks
Institute for NET/JRF, GATE, IIT‐JAM, JEST, TIFR and GRE in PHYSICAL SCIENCES 
 
Using the condition x  0 , we obtain
c1  c2  0 (ii)
k
k  t
v  t    c2 e m
(iii)
m
Using the condition v  0   v0

k m m
v0   c2  c2   v0 and c1  v0
m k k
mv0   t 
k
x t    1  e m
 (iv)
k  
mv0  k   mk t k
 t
k
 t v
v t       e  v0 e m
 e m
 (v)
k  m v0

mv0  v
Using equations (iv) and (v), we obtain x  t   1   ,
k  v0 
mv0 mv
L 1  L  0
k k
From the question

L mv0  k
t2 
k
 t2 kL
 1  e m
  e m
 1 (vi)
2 k   2mv0

L mv0   t4 
k k
 t4 kL
 1  e m
  e m
 1 (vii)
4 k   4mv0

Taking Logarithm on both sides of equations (vi) and (vii)

k  kL  k  kL 
 t2  ln  1   and  t4  ln  1  
m  2mv0  m  4mv0 

 k mv0 
ln  1   
t 2mv0 k  ln 1/ 2 
Thus, 2     2  41 .
t4  k mv0  ln  3 / 4 
ln  1   
 4mv0 k 

                                                                                
Head office  Branch office 
 
fiziks, H.No. 40 D, G.F, Jia Sarai,  Anand Institute of Mathematics, 
 
Near IIT, Hauz Khas, New Delhi‐16  28‐B/6, Jia Sarai, Near IIT 
 
Phone: 011‐26865455/+91‐9871145498 Hauz Khas, New Delhi‐16 
                                                   
                                             Website: www.physicsbyfiziks.com                                                                                          
                                                            Email: fiziks.physics@gmail.com                                                                   49 
fiziks
Institute for NET/JRF, GATE, IIT‐JAM, JEST, TIFR and GRE in PHYSICAL SCIENCES 
 
Oscillations, Waves and Optics
OBJECTIVE QUESTIONS
IIT-JAM-2005
Q1. Consider a beam of light of wavelength λ incident on a system of a polarizer and an
analyzer. The analyzer is oriented at 450 to the polarizer. When an optical component is
introduced between them, the output intensity becomes zero. (Light is incident normally
on all components). The optical component is
(a) a full-wave plate (b) a half-wave plate
(c) a quarter-wave plate (d) an ordinary glass plate
Ans.: (b)
Solution: Half wave plate introduce phase difference of π, if incidence wave is plane polarized
than after passing through HWP the wave is also plane polarized. If electric field of the
incidence wave makes angle 450 with optic axis of HWP than plane polarized at output
will be at 450, as a result it will incidence on polarizer at 900. According to malus law
intensity at output will be
I = I 0 cos 2 θ = I 0 cos 2 (π / 2) = 0
Q2. A combination of two thin convex lenses of equal focal lengths, is kept separated along
the optic axes by a distance of 20 cm between them. The combination behaves as a lens
system of infinite focal length. If an object is kept at 10 cm from the first lens, its image
will be formed on the other side at a distance x from the second lens. The value of x is
(a) 10 cm (b) 20 cm (c) 6.67 cm (d) infinite
Ans.: (b)
1 1 1
According to lens formula = +
v u F
Given u = 20 cm, F = ∞
Thus v = 20cm

                                                                                
Head office  Branch office 
 
fiziks, H.No. 40 D, G.F, Jia Sarai,  Anand Institute of Mathematics, 
 
Near IIT, Hauz Khas, New Delhi‐16  28‐B/6, Jia Sarai, Near IIT 
 
Phone: 011‐26865455/+91‐9871145498 Hauz Khas, New Delhi‐16 
                                                   
                                             Website: www.physicsbyfiziks.com                                                                                          
                                                            Email: fiziks.physics@gmail.com                                                                   45 
fiziks
Institute for NET/JRF, GATE, IIT‐JAM, JEST, TIFR and GRE in PHYSICAL SCIENCES 
 
IIT-JAM-2006
Q3. At a given point in space the total light wave is composed of three phasors P1 = a,
a iθ a
P2 = e and P3 = e −iθ . The intensity of light at this point is
2 2
⎛θ ⎞ ⎛θ ⎞
(a) 4a 2 cos 2 ⎜ ⎟ (b) 4a 2 cos 4 ⎜ ⎟
⎝2⎠ ⎝2⎠
(c) a 2 cos 2 (θ ) (d) 4a 2 cos 2 (2θ )
Ans.: (b)
a iθ a − iθ a
Solution: P = P1 + P2 + P3 = a + e + e = ( 2 + cos θ + i sin θ + cos θ − i sin θ )
2 2 2
θ
= a (1 + cos θ ) = 2a cos 2
2
⎛θ ⎞
I = P 2 = 4a 2 cos 4 ⎜ ⎟
⎝2⎠
Q4. A spring-mass system has undamped natural angular frequency ω 0 = 100 rad s-1. The

solution x(t) at critical damping is given by x(t ) = x0 (1 + ω 0 t ) exp(− ω 0 t ) , where x0 is a


constant. The system experiences the maximum damping force at time
(a) 0.01 s (b) 0.1 s (c) 0.01π s (d) 0.1π s
Ans.: (a)
dx
Damping force, Fd = −b
dt
dFd d 2x d 2x
For maximum damping force, = 0 ⇒ −b 2 = 0 ⇒ =0
dt dt dt 2
dx
= x0 (1 + ω 0 t )e −ω0t (− ω 0 ) + x0ω 0 e −ω0t = ( x0ω 0 + x0ω 0 (1 + ω 0 t ))e −ω0t
dt
d 2x
2
= ( x0ω 0 + x0ω 0 (1 + ω 0 t ))e −ω0t (− ω 0 ) + x0ω 0ω 0 e −ω0t = − x0ω 02 (1 + ω 0 t )e −ω0t = 0
dt
1 + ω0t = 0 ⇒ t = 1 ⇒ t = 0.01sec
ω0

                                                                                
Head office  Branch office 
 
fiziks, H.No. 40 D, G.F, Jia Sarai,  Anand Institute of Mathematics, 
 
Near IIT, Hauz Khas, New Delhi‐16  28‐B/6, Jia Sarai, Near IIT 
 
Phone: 011‐26865455/+91‐9871145498 Hauz Khas, New Delhi‐16 
                                                   
                                             Website: www.physicsbyfiziks.com                                                                                          
                                                            Email: fiziks.physics@gmail.com                                                                   46 
fiziks
Institute for NET/JRF, GATE, IIT‐JAM, JEST, TIFR and GRE in PHYSICAL SCIENCES 
 
G
Q5. ( )
E ( x, y, z, t ) = A 3iˆ + 4 ˆj exp[i (ωt − kz )] represents an electromagnetic wave. Possible
directions of the fast axis of a quarter wave plate which convert this wave into a
circularly polarized wave are

(a)
1
[7iˆ + ˆj ] and
1
[− iˆ + 7 ˆj ]
2 2

(b)
1
[3iˆ + 4 ˆj ] and
1
[4iˆ − 3 ˆj ]
2 2

(c)
1
[3iˆ − 4 ˆj ] and
1
[4iˆ + 3 ˆj ]
2 2

(d)
1
[7iˆ − ˆj ] and
1 ˆ
[
i + 7 ˆj ]
2 2
Ans.: (a)
Solution: The fast axis of the quarter wave plate must make angle of 450 with the direction of
vibration of electric field so that amplitude of ordinary ray and extra-ordinary ray is equal
to produce circularly polarized light.
G G
( )
E ( x, y, z , t ) = A 3iˆ + 4 ˆj exp[i (ωt − kz )] = E 0 A exp[i (ωt − kz )]
G
Where E 0 = 3iˆ + 4 ˆj ( )
G G G
Let us calculate the angle between E0 and A =
1 ˆ ˆ
7i + j [ ] and B=
1
[
− iˆ + 7 ˆj ]
2 2
1 1
G G (21 + 4) × 25
E0 ⋅ A 2 2 1
cos θ = G G = = = ⇒ θ = 45 0 and
E0 A 25 × 50 / 2 25 2

1 1
G G (− 3 + 28) × 25
E0 ⋅ B 2 2 1
cos θ = G G = = = ⇒ θ = 45 0
E0 B 25 × 50 / 2 25 2

                                                                                
Head office  Branch office 
 
fiziks, H.No. 40 D, G.F, Jia Sarai,  Anand Institute of Mathematics, 
 
Near IIT, Hauz Khas, New Delhi‐16  28‐B/6, Jia Sarai, Near IIT 
 
Phone: 011‐26865455/+91‐9871145498 Hauz Khas, New Delhi‐16 
                                                   
                                             Website: www.physicsbyfiziks.com                                                                                          
                                                            Email: fiziks.physics@gmail.com                                                                   47 
fiziks
Institute for NET/JRF, GATE, IIT‐JAM, JEST, TIFR and GRE in PHYSICAL SCIENCES 
 
IIT-JAM-2007
Q6. When two simple harmonic oscillations represented by x = A0 cos(ωt + α) and y=
B0 cos(ωt + β) are superposed at right angles, the resultant is an ellipse with its major axis
along the y-axis as shown in the figure. The conditions which correspond to this are
y

2
1
O x
1 2

π π
(a) β = α + ; A0 = 2 B0 (b) β = α − ; A0 = B0
2 4
π π
(c) β = α + ;2 A0 = B0 (d) β = α + ; A0 = B0
2 4
Ans.: (c)
Q7. Three polarizers P, Q and R are placed parallel to each other with their planes
perpendicular to the z-axis. Q is placed between P and R. Initially the polarizing
directions of P and Q are parallel, but that of R is perpendicular to them. In this
arrangement when unpolaized light of intensity I 0 is incident on P, the intensity coming
out of R is zero. The polarizer Q is now rotated about the z-axis. As a function of angle of
rotation, the intensity of light coming out of R is best represented by
(a)
Io
4

Io
8

π π 3π 2π
2 2

                                                                                
Head office  Branch office 
 
fiziks, H.No. 40 D, G.F, Jia Sarai,  Anand Institute of Mathematics, 
 
Near IIT, Hauz Khas, New Delhi‐16  28‐B/6, Jia Sarai, Near IIT 
 
Phone: 011‐26865455/+91‐9871145498 Hauz Khas, New Delhi‐16 
                                                   
                                             Website: www.physicsbyfiziks.com                                                                                          
                                                            Email: fiziks.physics@gmail.com                                                                   48 
fiziks
Institute for NET/JRF, GATE, IIT‐JAM, JEST, TIFR and GRE in PHYSICAL SCIENCES 
 
(b) I0
4

I0
8
π π 3π 2π
2 2
(c) I0
4

I0
8
π π 3π 2π
(d) 2 2
I0
4

I0
8
π π 3π 2π
2 2
Ans.: (c)
Solution: After Polarizer P Intensity of light will be I 0 / 2
If polarizer Q is aligned at 450 with respect to P. The intensity of light after P is
I0 I I 1 I
I= cos 2 θ = 0 cos 2 (π / 4) = 0 × = 0
2 2 2 2 4

                                                                                
Head office  Branch office 
 
fiziks, H.No. 40 D, G.F, Jia Sarai,  Anand Institute of Mathematics, 
 
Near IIT, Hauz Khas, New Delhi‐16  28‐B/6, Jia Sarai, Near IIT 
 
Phone: 011‐26865455/+91‐9871145498 Hauz Khas, New Delhi‐16 
                                                   
                                             Website: www.physicsbyfiziks.com                                                                                          
                                                            Email: fiziks.physics@gmail.com                                                                   49 
fiziks
Institute for NET/JRF, GATE, IIT‐JAM, JEST, TIFR and GRE in PHYSICAL SCIENCES 
 
IIT-JAM-2008
Q8. The instantaneous position x(t) of a small block performing one-dimensional damped
oscillations x(t) = Ae-rt cos(ωt + a). Here ω is the angular frequency, γ the damping
dx
coefficient, A the initial amplitude and α the initial phase. If x t =0 = 0 and = v,
dt t =0

the values of A and α (with n = 0, 1, 2, ….) are

(a) A =
v
,α =
(2n + 1) π (b) A =
v
, α = nπ
2ω 2 ω

(c) A =
v
,α =
(2n + 1)π (d) A =
2v
,α =
(2n + 1)π
ω 2 ω 2
Ans.: (c)
( 2n + 1)
Solution: x t =0 = A cos α = 0 ⇒ cos α = 0 ⇒ α =
2
dx
= Ae−γ t ( −γ ) cos (ω t + α ) + Ae−γ t sin (ωt + α ) ω
dt

= Ae −γ t ( −γ cos (ωt + α ) + sin (ωt + α ) ω )


dx v
=v ⇒ A ( −γ cos (α ) + ω sin (α ) ) = v ⇒ Aω = v ⇒ A=
dt t =0 ω

IIT-JAM-2009
Q9. Among the following displacement versus time plots, which ones may represent an over-
damped oscillator?
(P) (Q)

x(t ) x(t )

t t

                                                                                
Head office  Branch office 
 
fiziks, H.No. 40 D, G.F, Jia Sarai,  Anand Institute of Mathematics, 
 
Near IIT, Hauz Khas, New Delhi‐16  28‐B/6, Jia Sarai, Near IIT 
 
Phone: 011‐26865455/+91‐9871145498 Hauz Khas, New Delhi‐16 
                                                   
                                             Website: www.physicsbyfiziks.com                                                                                          
                                                            Email: fiziks.physics@gmail.com                                                                   50 
fiziks
Institute for NET/JRF, GATE, IIT‐JAM, JEST, TIFR and GRE in PHYSICAL SCIENCES 
 

(R) (S)

x(t )
x(t ) t

(a) only (P) and (Q) (b) only (P) and (R)
(c) only (P) and (S) (d) only (P), (R) and (S)
Ans.: (a)

IIT-JAM-2010
Q10. A quarter-wave plate is placed in between a polarizer and a photo-director. When the
optic axis of the quarter-wave plate is kept initially parallel to the pass axis of the
polarizer and perpendicular to the direction of light propagation. The intensity of light
passing through the quarter-wave plate is measured to be I0 (see figure). If the quarter
wave plate is now rotated by 45 0 about an axis parallel to the light propagation, what
would be the intensity of the emergent light measured by the photo-director?
Direction of
rotation of
quarter wave
plate

Polaroid Quarter wave plate Photo - detector


Io I0 I
(a) (b) (c) 0 (d) I 0
2 2 2 2
Ans.: (d)
Solution: After passing through QWP plane polarized light of intensity I0 will convert into
circularly polarized with intensity I0.
                                                                                
Head office  Branch office 
 
fiziks, H.No. 40 D, G.F, Jia Sarai,  Anand Institute of Mathematics, 
 
Near IIT, Hauz Khas, New Delhi‐16  28‐B/6, Jia Sarai, Near IIT 
 
Phone: 011‐26865455/+91‐9871145498 Hauz Khas, New Delhi‐16 
                                                   
                                             Website: www.physicsbyfiziks.com                                                                                          
                                                            Email: fiziks.physics@gmail.com                                                                   51 
fiziks
Institute for NET/JRF, GATE, IIT‐JAM, JEST, TIFR and GRE in PHYSICAL SCIENCES 
 
IIT-JAM-2011
Q11. Six simple harmonic oscillations each of same frequency and equal amplitude are
superposed. The phase difference between any two consecutive oscillations i.e.,
th
φ n − φ n −1 = Δφ is constant, where φ n is the phase of the n oscillation. If the resultant
amplitude of the superposition is zero, what is the phase difference Δφ ?
π π π
(a) (b) (c) (d) 2π
6 3 2

Ans.: (d)
Solution: Resultant amplitude of the superposition of n SHM is
a sin (nδ / 2) Δφ
R= , where, δ = ⇒ nδ = Δφ
sin (δ / 2) n
a sin (nδ / 2)
R= = 0 ⇒ sin (nδ / 2 ) = 0 ⇒ nδ / 2 = π
sin (δ / 2)
Δφ
=π ⇒ Δφ = 2π
2
Q12. Intensity of three different light beams after passing through an analyzer is found to vary
as shown in the following graphs. Identify the option giving the correct states of
polarization of the incident beams from graphs.
1 1

Graph1 Graph 2

0.5 0.5

0 0
0 1 2 3 4 5 6 0 1 2 3 4 5 6

Analyzer Orientation (Radians) Analyzer Orientation (Radians)

                                                                                
Head office  Branch office 
 
fiziks, H.No. 40 D, G.F, Jia Sarai,  Anand Institute of Mathematics, 
 
Near IIT, Hauz Khas, New Delhi‐16  28‐B/6, Jia Sarai, Near IIT 
 
Phone: 011‐26865455/+91‐9871145498 Hauz Khas, New Delhi‐16 
                                                   
                                             Website: www.physicsbyfiziks.com                                                                                          
                                                            Email: fiziks.physics@gmail.com                                                                   52 
fiziks
Institute for NET/JRF, GATE, IIT‐JAM, JEST, TIFR and GRE in PHYSICAL SCIENCES 
 
1

Graph 3

0.5

0
0 1 2 3 4 5 6
Analyzer Orientation (Radians)

(a) Graph1: Linear Polarization Graph2: Circular Polarization, Graph3: Elliptic


Polarization
(b) Graph 1: Circular Polarization, Graph 2: Linear Polarization, Graph 3: Elliptic
Polarization
(c) Graph 1: Unpolarized Graph 2: Circular Polarization, Graph 3: Linear Polarization
(d) Graph 1: Unpolarized Graph 2: Elliptic Polarization, Graph 3: Circular Polarization
Ans.: (b)

IIT-JAM-2012

Q13. A lightly damped harmonic oscillator loses energy at the rate of 1% per minute. The
decrease in amplitude of the oscillator per minute will be closest to
(a) 0.5% (b) 1% (c) 1.5% (d) 2%
Ans.: (d)
Solution: Decay of energy is governed by equation, E = E 0 e −2γt

Decay of amplitude is governed by equation, A = ae−γ t

                                                                                
Head office  Branch office 
 
fiziks, H.No. 40 D, G.F, Jia Sarai,  Anand Institute of Mathematics, 
 
Near IIT, Hauz Khas, New Delhi‐16  28‐B/6, Jia Sarai, Near IIT 
 
Phone: 011‐26865455/+91‐9871145498 Hauz Khas, New Delhi‐16 
                                                   
                                             Website: www.physicsbyfiziks.com                                                                                          
                                                            Email: fiziks.physics@gmail.com                                                                   53 
fiziks
Institute for NET/JRF, GATE, IIT‐JAM, JEST, TIFR and GRE in PHYSICAL SCIENCES 
 
Q14. Group I contains x- and y- components of the electric field and Group II contains the type
of polarization of light.
Group I Group II
E0
Ex = cos(ωt + kz )
P. 2 1. Linearly Polarized
E y = E 0 sin (ωt + kz )

Q. E x = E 0 sin (ωt + kz ) 2. Circularly Polarized


E y = E 0 cos(ωt + kz )

R. E x = E1 sin (ωt + kz ) 3. Unpolarized


E y = E 2 sin (ωt + kz )
E x = E 0 sin (ωt + kz )
S. ⎛ π⎞ 4. Elliptically Polarized
E y = E 0 sin ⎜ ωt + kz + ⎟
⎝ 4⎠
The correct set of matches is
(a) P → 4; Q → 2; R → 4; S → 1 (b) P → 1; Q → 3; R → 1; S → 4
(c) P → 4; Q → 2; R → 1; S → 4 (d) P → 3; Q → 1; R → 3; S → 2
Ans.: (c)
E0
Solution: P. E x = cos(ωt + kz ) and E y = E 0 sin (ωt + kz )
2
The phase difference between Ex and Ey is π/2 with different amplitude. Therefore the
resultant will be elliptically polarized.
Q. E x = E 0 sin (ωt + kz ) and E y = E 0 cos(ωt + kz )

The phase difference between Ex and Ey is π/2 with same amplitude. Therefore the
resultant will be circularly polarized.
R. E x = E1 sin (ωt + kz ) and E y = E 2 sin (ωt + kz )

The phase difference between Ex and Ey is 0 with different amplitude. Therefore the
resultant will be linarly polarized.
⎛ π⎞
S. E x = E 0 sin (ωt + kz ) and E y = E 0 sin ⎜ ωt + kz + ⎟
⎝ 4⎠

                                                                                
Head office  Branch office 
 
fiziks, H.No. 40 D, G.F, Jia Sarai,  Anand Institute of Mathematics, 
 
Near IIT, Hauz Khas, New Delhi‐16  28‐B/6, Jia Sarai, Near IIT 
 
Phone: 011‐26865455/+91‐9871145498 Hauz Khas, New Delhi‐16 
                                                   
                                             Website: www.physicsbyfiziks.com                                                                                          
                                                            Email: fiziks.physics@gmail.com                                                                   54 
fiziks
Institute for NET/JRF, GATE, IIT‐JAM, JEST, TIFR and GRE in PHYSICAL SCIENCES 
 
The phase difference between Ex and Ey is π/4 with same amplitude. Therefore the
resultant will be elliptically polarized.

IIT-JAM-2013
⎛ 2abxt − a 2 x 2 − b 2 t 2 ⎞
Q15. A traveling pulse is given by f ( x, t ) = A exp⎜⎜ ⎟⎟ where A, a, b and c
⎝ c2 ⎠
are positive constants of appropriate dimensions. The speed of the pulse is
b 2b cb b
(a) (b) (c) (d)
a a a 2a
Ans.: (a)

⎛ 2abxt − a 2 x 2 − b 2 t 2 ⎞ ⎡ − (ax − bt )2 ⎤
Solution: f ( x, t ) = A exp⎜⎜ ⎟⎟ = A exp ⎢ ⎥
⎝ c2 ⎠ ⎣ c2 ⎦
Phase factor is constant
− (ax − bt )
2
= const ⇒ − (ax − bt ) = const × c 2
2
2
c
Taking differentiation, we get
(ax − bt )(adx − bdt ) = 0 ⇒ adx − bdt = 0 ⇒ dx / dt = b / a
Velocity of the pulse is b/a

IIT-JAM-2014

Q16. A collimated beam of light of diameter 1 mm is propagating along the x-axis. The beam
is to be expanded to a collimated beam of diameter 10 mm using a combination of two
convex lenses. A lens of focal length of 50 mm and another lens with focal length F are
to be kept at a distance d between them. The values of F and d respectively, are
(a) 450 mm and 10 mm (b) 400 mm and 500 mm
(c) 550 mm and 600 mm (d) 500 mm and 550 mm
Ans.: (d)

                                                                                
Head office  Branch office 
 
fiziks, H.No. 40 D, G.F, Jia Sarai,  Anand Institute of Mathematics, 
 
Near IIT, Hauz Khas, New Delhi‐16  28‐B/6, Jia Sarai, Near IIT 
 
Phone: 011‐26865455/+91‐9871145498 Hauz Khas, New Delhi‐16 
                                                   
                                             Website: www.physicsbyfiziks.com                                                                                          
                                                            Email: fiziks.physics@gmail.com                                                                   55 
fiziks
Institute for NET/JRF, GATE, IIT‐JAM, JEST, TIFR and GRE in PHYSICAL SCIENCES 
 
Q17. The electric fields of two light sources with nearby frequencies ω1 and ω 2 , and wave
G G
vectors k1 and k 2 , are expressed as E1 = E10 iˆ e − i (k1z −ω1t ) and E 2 = E 20 iˆ e − i (k2 z −ω2t ) ,

respectively. The interference pattern on the screen is photographed at t = t 0 ; denote

(k1 − k 2 )z − (ω1 − ω 2 )t 0 by θ . For this pattern

(a) a bright fringe will be obtained for cos θ = −1


(b) a bright fringe intensity is given by (E10 ) + (E 20 )
2 2

(c) a dark fringe will be obtained for cos θ = 1


(d) a drak fringe intensity is given by (E10 − E 20 )
2

Ans.: (d)
Q18. White light is incident on a grating G1 with groove density 600 lines/mm and width
50 mm. A small portion of the diffracted light is incident on another grating G 2 with
groove density 1800 lines/mm and width 15 mm. The resolving power of the combined
system is
(a) 3× 10 3 (b) 57 × 10 3 (c) 81× 10 7 (d) 108 × 10 5
Ans.: (c)

                                                                                
Head office  Branch office 
 
fiziks, H.No. 40 D, G.F, Jia Sarai,  Anand Institute of Mathematics, 
 
Near IIT, Hauz Khas, New Delhi‐16  28‐B/6, Jia Sarai, Near IIT 
 
Phone: 011‐26865455/+91‐9871145498 Hauz Khas, New Delhi‐16 
                                                   
                                             Website: www.physicsbyfiziks.com                                                                                          
                                                            Email: fiziks.physics@gmail.com                                                                   56 
fiziks
Institute for NET/JRF, GATE, IIT‐JAM, JEST, TIFR and GRE in PHYSICAL SCIENCES 
 
IIT-JAM-2015
Q19. At room temperature, the speed of sound in air is 340 m/sec. An organ pipe with both
ends open has a length L = 29 cm . An extra hole is created at the position L / 2 . The
lowest frequency of sound produced is
(a) 293 Hz (b) 586 Hz (c) 1172 Hz (d) 2344 Hz
Ans.: (c)
v
Solution: The fundamental frequency in organ pipe with both end open is f =
2L
L/2

L
L
with additional rate at , the fundamental frequency becomes
2
v v v 340 m / sec
f′= = = = = 1172 Hz
2 L ′ 2 L L 29 × 10−2 m
2
Q20. Vibrations of diatomic molecules can be represented as those of harmonic oscillators.
Two halogen molecules X2 and Y2 have fundamental vibrational frequencies

vX = 16.7 ×1012 Hz and vY = 26.8 × 1012 Hz , respectively. The respective force constants

are K X = 325 N / m and K Y = 446 N / m . The atomic masses of F, Cl and Br are


19.0, 35.5 and 79.9 atomic mass unit respectively. The halogen molecules X 2 and Y2 are
(a) X 2 = F2 and Y2 = Cl2 (b) X 2 = Cl 2 and Y2 = F2
(c) X 2 = Br2 and Y2 = F2 (d) X 2 = F2 and Y2 = Br2
Ans.: (b)
Solution: The oscillation frequency of diatomic molecule with reduce mass ‘ μ ’ is

1 k 1 k
f = ⇒μ= where k is force constant.
2π μ 4π 2
f2

                                                                                
Head office  Branch office 
 
fiziks, H.No. 40 D, G.F, Jia Sarai,  Anand Institute of Mathematics, 
 
Near IIT, Hauz Khas, New Delhi‐16  28‐B/6, Jia Sarai, Near IIT 
 
Phone: 011‐26865455/+91‐9871145498 Hauz Khas, New Delhi‐16 
                                                   
                                             Website: www.physicsbyfiziks.com                                                                                          
                                                            Email: fiziks.physics@gmail.com                                                                   57 
fiziks
Institute for NET/JRF, GATE, IIT‐JAM, JEST, TIFR and GRE in PHYSICAL SCIENCES 
 
mx mx m
For X 2 molecule: μ = = x
m x + mx 2
1 kx 1 325 N / m
⇒ mx = × = ×
2π f x 2 × ( 3.14 ) ( )
2 2 2 2
16.7 × 1012 Hz

⇒ mx = 59.07 ×10−27 kg = 35.5 × 1.67 × 10−27 kg = 35.5 a.m.u.

This is the atomic mass of chlorine ( Cl ) .

my my my
For Y2 molecule: μ = =
my + my 2

1 ky 1 446 N / m
⇒ my = × = ×
2π (f ) 2 × ( 3.14 ) ( 26.8 ×10 )
2 2 2 2
12
y Hz

⇒ my = 31.73 × 10−27 kg = 19 × 1.67 × 10−27 kg = 19 a.m.u.

This is the atomic mass of F . Thus, correct answer is option (b)


Q21. Doppler effect can be used to measure the speed of blood through vessels. Sound of
frequency 1.0522 MHz is sent through the vessels along the direction of blood flow. The
reflected sound generates a beat signal of frequency 100 Hz. The speed of sound in blood
is 1545 m/sec. The speed of blood through the vessel, in m/sec, is
(a) 14.68 (b) 1.468 (c) 0.1468 (d) 0.01468
Ans.: (d)
Solution: Consider Vb , Vsound are velocities of blood cell and sound in blood. The sound of

frequency ( f0 ) is traveling towards blood cell where blood cell is moving away with

velocity Vb
f0
Vsound Vb
Frequency of sound observed on blood cell is
⎛V − Vb ⎞
f ′ = f 0 ⎜ sound (i)
⎝ Vsound ⎟⎠

                                                                                
Head office  Branch office 
 
fiziks, H.No. 40 D, G.F, Jia Sarai,  Anand Institute of Mathematics, 
 
Near IIT, Hauz Khas, New Delhi‐16  28‐B/6, Jia Sarai, Near IIT 
 
Phone: 011‐26865455/+91‐9871145498 Hauz Khas, New Delhi‐16 
                                                   
                                             Website: www.physicsbyfiziks.com                                                                                          
                                                            Email: fiziks.physics@gmail.com                                                                   58 
fiziks
Institute for NET/JRF, GATE, IIT‐JAM, JEST, TIFR and GRE in PHYSICAL SCIENCES 
 
Sound from blood cell of frequency f ′ reflect back.

f′
observer Vb

⎛ Vsound ⎞
The frequency observed by observer is f = f ′ ⎜ (ii)
⎝ Vsound + Vb ⎟⎠

⎛V − Vb ⎞ ⎛ Vsound ⎞
From equation (i) and (ii), we get f = f 0 ⎜ sound
⎝ Vsound ⎟⎠ ⎜⎝ Vsound + Vb ⎟⎠

⎛V − Vb ⎞
⇒ f = f 0 = ⎜ sound ⎟ (iii)
⎝ Vsound + Vb ⎠
⎛V − Vb ⎞ ⎛ 2Vb ⎞
Now, Δf = f 0 − f = f 0 − f 0 ⎜ sound ⎟ = f0 ⎜ ⎟
⎝ Vsound + Vb ⎠ ⎝ Vsound + Vb ⎠
2Vb Δf V + Vb 2 f 0 Vsound
⇒ = ⇒ sound = ⇒ Vb =
Vsound + Vb f0 Vb Δf ⎛ 2 f0 ⎞
⎜⎝ Δf − 1⎟⎠

Given Vsound = 1545 m / sec, f 0 = 1.0522 × 106 Hz , Δf = 100 Hz

1545 1545
∴Vb = = = 0.073 ⇒ Vb = 0.073 m / sec
⎛ 2 × 1.0522 × 106 ⎞ 21043
⎜⎝ − 1⎟
100 ⎠

Thus the best suitable answer is option (d).

                                                                                
Head office  Branch office 
 
fiziks, H.No. 40 D, G.F, Jia Sarai,  Anand Institute of Mathematics, 
 
Near IIT, Hauz Khas, New Delhi‐16  28‐B/6, Jia Sarai, Near IIT 
 
Phone: 011‐26865455/+91‐9871145498 Hauz Khas, New Delhi‐16 
                                                   
                                             Website: www.physicsbyfiziks.com                                                                                          
                                                            Email: fiziks.physics@gmail.com                                                                   59 
fiziks
Institute for NET/JRF, GATE, IIT‐JAM, JEST, TIFR and GRE in PHYSICAL SCIENCES 
 
SECTION–B: MSQ
Q22. The following figure shows a double slit Fraunhofer diffraction pattern produced by two
slits, each of width a separated by a distance b, a < b .
Secondary maxima

Primary maxima
Which of the following statements are correct?
(a) Reducing a increases the separation between consecutive primary maxima
(b) Reducing a increases the separation between consecutive secondary maxima
(c) Reducing b increases the separation between consecutive primary maxima
(d) Reducing b increases the separation between consecutive secondary maxima
Ans.: (a) and (d)
Solution: The minima condition for double slit Fraunhofer diffraction is

a sin θ = nλ ⇒ sin θ = where a is the width of slit.
a
Reducing ‘ a ’ increases the separation between diffraction minima i.e. increases the
separation between consecutive primary maxima.
The condition of interference maxima is

b sin θ = mλ ⇒ sin θ = where b is the separation between slits.
b
The position of interference maxima gives the separation between secondary maxima.
Reducing ‘ b ’ increases the separation between consecutive secondary maxima.
The correct answer is option (a) and (d).

                                                                                
Head office  Branch office 
 
fiziks, H.No. 40 D, G.F, Jia Sarai,  Anand Institute of Mathematics, 
 
Near IIT, Hauz Khas, New Delhi‐16  28‐B/6, Jia Sarai, Near IIT 
 
Phone: 011‐26865455/+91‐9871145498 Hauz Khas, New Delhi‐16 
                                                   
                                             Website: www.physicsbyfiziks.com                                                                                          
                                                            Email: fiziks.physics@gmail.com                                                                   60 
fiziks
Institute for NET/JRF, GATE, IIT‐JAM, JEST, TIFR and GRE in PHYSICAL SCIENCES 
 
Q23. Unpolarized light is incident on a calcite plate at an angle of incidence 50 o as shown in
the figure. Take n0 = 1.6584 and ne = 1.4864 for calcite. The angular separation
( in degrees) between the two emerging rays within the plate is
Air
50 0

Optic axis Calcite

Ans.: 3.51
Solution: Inside the crystal incident light split into two components, ordinary ray and extra-
ordinary ray
i = 500
sin i
According to Snell’s law =n
sin r
For ordinary ray i = 500 , no = 1.6584 ro

sin i ⎛ sin i ⎞ re
∴ sin ro = ⇒ ro = sin −1 ⎜
no ⎝ no ⎟⎠ e- ray
o- ray
⎡ sin 500 ⎤ −1 ⎡ 0.766 ⎤
⇒ ro = sin −1 ⎢ ⎥ = sin ⎢ ⎥ = sin −1 [ 0.462] ⇒ r0 = 27.510
⎣ no ⎦ ⎣ 1.6584 ⎦

For extra-ordinary ray i = 500 , ne = 1.4864

sin i ⎛ sin i ⎞
∴ sin re = ⇒ re = sin −1 ⎜
ne ⎝ ne ⎟⎠

⎡ sin 500 ⎤ −1 ⎡ 0.766 ⎤


⇒ re = sin −1 ⎢ ⎥ = sin ⎢ ⎥ = sin −1 [ 0.515] ⇒ re = 31.020
⎣ ne ⎦ ⎣ 1.4864 ⎦

Thus, the angular separation between the o - ray and e - ray is θ = re − ro = 3.510

                                                                                
Head office  Branch office 
 
fiziks, H.No. 40 D, G.F, Jia Sarai,  Anand Institute of Mathematics, 
 
Near IIT, Hauz Khas, New Delhi‐16  28‐B/6, Jia Sarai, Near IIT 
 
Phone: 011‐26865455/+91‐9871145498 Hauz Khas, New Delhi‐16 
                                                   
                                             Website: www.physicsbyfiziks.com                                                                                          
                                                            Email: fiziks.physics@gmail.com                                                                   61 
fiziks
Institute for NET/JRF, GATE, IIT‐JAM, JEST, TIFR and GRE in PHYSICAL SCIENCES 
 
Q24. For the arrangement given in the following figure, the coherent light sources A, B and C

have individual intensities of 2 mW / m 2, 2 mW / m 2 and 5 mW / m 2 respectively at point P .


The wavelength of each of the sources is 600 nm . The resultant intensity at point P
P
(in mW / m 2 ) is ___________.
15 mm
A
3.22 mm
B 1m
2.04 mm
C
2
Ans.: 9.23 mw / m
p
Solution: The electric field on the screen is the sum of the fields
produced by the slits individually. y
A
E = E1 + E2 + E3
d D O
iδ iaδ
= A + Ae + Be B
ad
2πd C
where δ = sin θ
λ
The total intensity at θ is
I = EE * = 2 A2 + B 2 + 2 A2 cos δ + 2 AB ⎡⎣ cos ( aδ ) + cos (1 − a ) δ ⎤⎦

2π d 2π d 2π d y 3.22 × 10−3 15 × 10−3


where δ = sin θ ≅ θ= × = 2π × × = 505.7
λ λ λ D 6 ×10−7 1
δ = 145.80
given, A2 = 2 mw / m , B 2 = 5 mw / m 2 , d = 3.22 mm, ad = 2.04 mm, a = 0.6335 mm

∴ I = 2 × 2 × 10−3 + 5 × 10−3 + 2 × 2 × 10−3 cos (δ ) + 2 2 5 × 10−3 ⎡⎣cos aδ + cos (1 − a ) δ ⎤⎦

= 9.23 × 10−3 w / m 2

I = 9.23 mw / m 2

                                                                                
Head office  Branch office 
 
fiziks, H.No. 40 D, G.F, Jia Sarai,  Anand Institute of Mathematics, 
 
Near IIT, Hauz Khas, New Delhi‐16  28‐B/6, Jia Sarai, Near IIT 
 
Phone: 011‐26865455/+91‐9871145498 Hauz Khas, New Delhi‐16 
                                                   
                                             Website: www.physicsbyfiziks.com                                                                                          
                                                            Email: fiziks.physics@gmail.com                                                                   62 
fiziks
Institute for NET/JRF, GATE, IIT‐JAM, JEST, TIFR and GRE in PHYSICAL SCIENCES 
 
IIT-JAM-2016
Q25. Consider a particle of mass m following a trajectory given by x = x0 cos ω1t and

y = y0 sin ω2t , where x0 , y0 , ω1 and ω2 are constants of appropriate dimensions. The force on
the particle is
(a) central only if ω1 = ω2 (b) central only if x0 = y0 and ω1 = ω2

(c) always central (d) central only if x0 = y0 and ω1 ≠ ω2


Ans.: (c)
x = − x0ω12 cos ω1t 
Solution:  y = − y0ω22 cos ω2t
G G
r = xiˆ + yiˆ ⇒  (
r = − y0ω22 cos ω2tiˆ + x0ω12 cos ω1tjˆ )
G
If ω1 = ω2 then  ( G
r = − y0ω22 cos ω2tiˆ + x0ω12 cos ω1tjˆ = −ω 2 r )

Q26. Two sinusoidal signals of frequency ω x and ω y having same y


1
amplitude are applied to x - and y - channels of a cathode ray
oscilloscope (CRO), respectively. The following stationary figure
x
will be observed when −1 1

(a) ω y = ω x (b) phase difference is 0


−1
(c) ω y = 2ω x (d) phase difference is π / 2

Ans.: (b)
ωx number of cuts on y -axis 4
Solution: = = ⇒ ω x = 2ω y
ω y number of cuts on x-axis 2
and this lissajous figure appears when phase difference is 0 . Thus correct option is (b)

                                                                                
Head office  Branch office 
 
fiziks, H.No. 40 D, G.F, Jia Sarai,  Anand Institute of Mathematics, 
 
Near IIT, Hauz Khas, New Delhi‐16  28‐B/6, Jia Sarai, Near IIT 
 
Phone: 011‐26865455/+91‐9871145498 Hauz Khas, New Delhi‐16 
                                                   
                                             Website: www.physicsbyfiziks.com                                                                                          
                                                            Email: fiziks.physics@gmail.com                                                                   63 
fiziks
Institute for NET/JRF, GATE, IIT‐JAM, JEST, TIFR and GRE in PHYSICAL SCIENCES 
 
Q27. Light traveling between two points takes a path for which
(a) time of flight is always minimum (b) distance is always minimum
(c) time of flight is extremum (d) distance is extremum
Ans.: (c)
Solution: According to Fermat’s principle, the ray will correspond to that path for which the time
taken is an extremum in comparison to nearby paths i.e. it is either a minimum or a
maximum or stationary. Thus correction option is (c).

Q28. A train passes through a station with a constant speed. A stationary observer at the station
platform measures the tone of the train whistle as 484 Hz when it approaches the station
and 442 Hz when it leaves the station. If the sound velocity in air is 330 m / s , then the
tone of the whistle and the speed of the rain are
(a) 462 Hz , 54 km / h (b) 463 Hz , 52 km / h
(c) 463 Hz , 56 km / h (d) 464 Hz , 52 km / h
Ans.: (a)
Solution: Let f o = original frequency of the whistle
f a = observed frequency when train approaches platform
f r = observed frequency when train recedes platform
vt = velocity of train
v = velocity of sound in air
⎛ v ⎞ ⎛ v ⎞
∴ fa = fo ⎜ ⎟ and f r = f o ⎜ ⎟
⎝ v − vt ⎠ ⎝ v + vt ⎠
f a v + vt ⎛ f + fr ⎞ 484 − 442
Now, = ⇒ vt = ⎜ a ⎟ v ⇒ vt = × 330 = 15 m / sec = 54 km / hr
f r v − vt ⎝ fa + fr ⎠ 484 + 442
f ( v − vt ) 330 − 15
and f o = a = × 484 = 462 Hz
v 330

                                                                                
Head office  Branch office 
 
fiziks, H.No. 40 D, G.F, Jia Sarai,  Anand Institute of Mathematics, 
 
Near IIT, Hauz Khas, New Delhi‐16  28‐B/6, Jia Sarai, Near IIT 
 
Phone: 011‐26865455/+91‐9871145498 Hauz Khas, New Delhi‐16 
                                                   
                                             Website: www.physicsbyfiziks.com                                                                                          
                                                            Email: fiziks.physics@gmail.com                                                                   64 
fiziks
Institute for NET/JRF, GATE, IIT‐JAM, JEST, TIFR and GRE in PHYSICAL SCIENCES 
 
Q29. The minimum length of a plane mirror to see the entire full-length image of an object is
half of the object’s height. Suppose δ is the distance between eye and top of the head of
a person of height h . The person will be able to see his entire full-length image with a
mirror of height h / 2 fixed on the wall
(a) when the bottom edge of mirror is kept h / 2 above the floor
(b) when the bottom edge of mirror is kept ( h + δ ) / 2 above the floor
(c) when the bottom edge of mirror is kept ( h − δ ) / 2 above the floor

(d) when the centre of the mirror is at the same height as centre of the person
Ans.: (c)
H
Solution: Let BH = h is the height of person
H′
HE = δ , where H represents top of head and E represents eye. δ
δ /2
δ E′
In the mirror, distance between eye and top of head will be . E
2 h/2
h
h
Since total height of mirror is ,
2
h δ B′
therefore, for diagram B′E ′ = − and BE′ = h − δ
2 2
δ δ δ h δ h −δ B
∴ BH ′ = h − δ + and BB′ = BH ′ − B′H ′ = h − − = − =
2 2 2 2 2 2

                                                                                
Head office  Branch office 
 
fiziks, H.No. 40 D, G.F, Jia Sarai,  Anand Institute of Mathematics, 
 
Near IIT, Hauz Khas, New Delhi‐16  28‐B/6, Jia Sarai, Near IIT 
 
Phone: 011‐26865455/+91‐9871145498 Hauz Khas, New Delhi‐16 
                                                   
                                             Website: www.physicsbyfiziks.com                                                                                          
                                                            Email: fiziks.physics@gmail.com                                                                   65 
fiziks
Institute for NET/JRF, GATE, IIT‐JAM, JEST, TIFR and GRE in PHYSICAL SCIENCES 
 
Q30. A particle travels in a medium along a horizontal linear path. The initial velocity of the
particle is v0 and the viscous force acting on it is proportional to its instantaneous
velocity. In the absence of any other forces, which one of the following figures correctly
represents the velocity of the particle as a function of time?
(a) (b)
v (t ) v (t )

t t

(c) v t (d)
() v (t )

t t
Ans.: (d)
Solution: Viscous force ∝ instantaneous velocity
mdv ( t ) dv ( t ) b
F = −bv ( t ) ⇒ = −bv ( t ) ⇒ = − dt
dt v (t ) m

Integrating on both sides


dv ( t ) b b
∫ v (t ) = −∫ dt ⇒ ln v ( t ) = − t + c
m m

where t = 0, v ( t ) = v0 ∴ c = ln v0

b ⎛ v (t ) ⎞ b v (t ) b
− t
⇒ ln v ( t ) = − t + ln v0 ⇒ ln ⎜ ⎟=− t⇒ =e m
m ⎝ v0 ⎠ m v0

Thus graph (d) correctly represent the variation of v ( t ) w.r.t. time.

                                                                                
Head office  Branch office 
 
fiziks, H.No. 40 D, G.F, Jia Sarai,  Anand Institute of Mathematics, 
 
Near IIT, Hauz Khas, New Delhi‐16  28‐B/6, Jia Sarai, Near IIT 
 
Phone: 011‐26865455/+91‐9871145498 Hauz Khas, New Delhi‐16 
                                                   
                                             Website: www.physicsbyfiziks.com                                                                                          
                                                            Email: fiziks.physics@gmail.com                                                                   66 
fiziks
Institute for NET/JRF, GATE, IIT‐JAM, JEST, TIFR and GRE in PHYSICAL SCIENCES 
 
Q31. A lightly damped harmonic oscillator with natural frequency ω0 is driven by a periodic

force of frequency ω . The amplitude of oscillation is maximum when


(a) ω is slightly lower than ω0

(b) ω = ω0

(c) ω is slightly higher than ω0


(d) The force is in phase with the displacement
Ans.: (a)
Solution: Amplitude in driven oscillator is
F0 / m
A=
(ω − ω 2 ) + 4b 2ω 2
2 2
0

To find the condition for maximum amplitude, differentiate above equation w.r.t. ω and
dA
put =0

dA ⎛ F0 ⎞ ⎛ 1 ⎞ 2 (ω0 − ω ) ( −2ω ) + 8b ω
2 2 2

i.e. = ⎜− ⎟ =0
d ω ⎜⎝ m ⎟⎠ ⎝ 2 ⎠ ⎡ 2 1/ 2
(ω − ω ) + 4b ω ⎦⎥
⎣⎢ 0
2 2 2 2⎤

⇒ 2b 2 − ω02 + ω 2 = 0 ⇒ ω = ω02 − 2b2

Thus ω is slightly lower than ω0 . Correct option is (a).

Q32. A block of mass 0.38 kg is kept at rest on a frictionless surface and attached to a wall
with a spring of negligible mass. A bullet weighing 0.02 kg moving with a speed of
200 m / s hits the block at time t = 0 and gets stuck to it. The displacement of the block
(in metre) with respect to the equilibrium position is given by

(Spring constant = 640 N / m )


(a) 2sin 5t (b) cos10t (c) 0.4 cos 25t (d) 0.25sin 40t
                                                                                
Head office  Branch office 
 
fiziks, H.No. 40 D, G.F, Jia Sarai,  Anand Institute of Mathematics, 
 
Near IIT, Hauz Khas, New Delhi‐16  28‐B/6, Jia Sarai, Near IIT 
 
Phone: 011‐26865455/+91‐9871145498 Hauz Khas, New Delhi‐16 
                                                   
                                             Website: www.physicsbyfiziks.com                                                                                          
                                                            Email: fiziks.physics@gmail.com                                                                   67 
fiziks
Institute for NET/JRF, GATE, IIT‐JAM, JEST, TIFR and GRE in PHYSICAL SCIENCES 
 
Ans.: (d)

k k 640
Solution: ω = = = = 1600
m m + m′ 0.38 + 0.02
ω = 40 rad/sec
Let v′ be the velocity acquired by the block m where bullet m strikes it and comes to
rest in it.
By conservation of momentum
m′ 0.02 0.02
( m + m′ ) v′ = m′v ⇒ v′ = v= × 200 = × 200 = 10 m / sec
m + m′ 0.38 + 0.02 0.4
The block is set in oscillation about it mean position with maximum amplitude A
dx
∴ x = A sin ωt ⇒ = Aω cos ωt
dt
In the mean position, the velocity is maximum
10 10
∴ Aω = 10 ⇒ A = = = 0.25
ω 40
∴ x = 0.25sin 40t

Q33. In the optical arrangement as shown below, the axes


of two polarizing sheets P and Q are oriented such detector
unpolarized
that no light is detected. Now when a third polarizing light
P Q
sheet ( R ) is placed in between P and Q , then light is

detected. Which of the following statement ( s ) is (are) true?

(a) Polarization axes of P and Q are perpendicular to each other.


(b) Polarization axis of R is not parallel to P
(c) Polarization axis of R is not parallel to Q
(d) Polarization axes of P and Q are parallel to each other.

                                                                                
Head office  Branch office 
 
fiziks, H.No. 40 D, G.F, Jia Sarai,  Anand Institute of Mathematics, 
 
Near IIT, Hauz Khas, New Delhi‐16  28‐B/6, Jia Sarai, Near IIT 
 
Phone: 011‐26865455/+91‐9871145498 Hauz Khas, New Delhi‐16 
                                                   
                                             Website: www.physicsbyfiziks.com                                                                                          
                                                            Email: fiziks.physics@gmail.com                                                                   68 
fiziks
Institute for NET/JRF, GATE, IIT‐JAM, JEST, TIFR and GRE in PHYSICAL SCIENCES 
 
Ans.: (a), (b) and (c)
Solution: According to Malu’s law
I0 I0
I= cos 2 θ where θ is angle between pass axis of P and Q I
2
unpolarized
where I = 0 , ⇒ θ = 900 light
P Q
i.e. P and Q are perpendicular to each other. Thus option (a) is correct.
If third polarizer R is introduced between P and Q making angle θ1 w.r.t. pass axis of

P and 900 − θ1 w.r.t. θ .

I0
∴I = cos 2 θ1 cos 2 ( 90 − θ1 )
2
If θ1 = 0 , then I = 0 thus R can’t be parallel to P . Now, If θ1 = 900 , then again I = 0 .

Thus R can’t be parallel to θ also.


Thus options (a), (b) and (c) are correct.

Q34. When sunlight is focused on a paper using a bi-convex lens, it starts to burn in the
shortest time if the lens is kept 0.5 m above it. If the radius of curvature of the lens is
0.75 m then, the refractive index of the material is…………….

Ans.: 1.75
Solution: For bi-convex lens
R1
⎛1 1 ⎞ 1
( μ − 1) ⎜ − ⎟ = R2
⎝ R1 R2 ⎠ f 0.5m = f
1 1 1 1
( μ − 1) = ⋅ = = 0.75 ⇒ μ = 1.75
f ⎛ 1 1 ⎞ 0.5 ⎛ 1 + 1 ⎞
⎜ − ⎟ ⎜ ⎟
⎝ 0.75 0.75 ⎠
⎝ R1 R2 ⎠

                                                                                
Head office  Branch office 
 
fiziks, H.No. 40 D, G.F, Jia Sarai,  Anand Institute of Mathematics, 
 
Near IIT, Hauz Khas, New Delhi‐16  28‐B/6, Jia Sarai, Near IIT 
 
Phone: 011‐26865455/+91‐9871145498 Hauz Khas, New Delhi‐16 
                                                   
                                             Website: www.physicsbyfiziks.com                                                                                          
                                                            Email: fiziks.physics@gmail.com                                                                   69 
fiziks
Institute for NET/JRF, GATE, IIT‐JAM, JEST, TIFR and GRE in PHYSICAL SCIENCES 
 
Electricity and Magnetism
OBJECTIVE QUESTIONS
IT-JAM-2005
Q1. A small loop of wire of area A = 0.01 m 2 , N = 40 turns and resistance R = 20 Ω is
initially kept in a uniform magnetic field B in such a way that the field is normal to the
loop. When it is pulled out of the magnetic field, a total charge of Q = 2 ×10−5 C flows
through the coil. The magnetic of the field B is
(a) 1× 10−3 T (b) 4 × 10−3 T
(c) zero (d) unobtainable, as the data is insufficient
Ans.: (a)
Solution: Magnetic flux through the loop φ = NBA
dφ 1 dφ dQ 1
Induced e.m.f ε = − and induced current i = − = ⇒ − dφ = dQ .
dt R dt dt R

× (40 × B × 0.01) = 2 × 10 −5 ⇒ B = 1 × 10 −3 T .
1
Thus
20
Q2. Two point charges + q1 and + q 2 are fixed with a finite distance d between them. It is
desired to put a third charge q3 in between these two charges on the line joining them so

that the charge q3 is in equilibrium. This is

(a) possible only if q3 is positive

(b) possible only if q3 is negative

(c) possible irrespective of the sign of q3


(d) not possible at all
Ans.: (c)

                                                                                
Head office  Branch office 
 
fiziks, H.No. 40 D, G.F, Jia Sarai,  Anand Institute of Mathematics, 
 
Near IIT, Hauz Khas, New Delhi‐16  28‐B/6, Jia Sarai, Near IIT 
 
Phone: 011‐26865455/+91‐9871145498 Hauz Khas, New Delhi‐16 
                                                   
                                             Website: www.physicsbyfiziks.com                                                                                          
                                                            Email: fiziks.physics@gmail.com                                                                   70 
fiziks
Institute for NET/JRF, GATE, IIT‐JAM, JEST, TIFR and GRE in PHYSICAL SCIENCES 
 
IIT-JAM-2006
Q3. Two electric dipoles P1 and P2 are placed at ( 0, 0, 0 ) and (1, 0, 0 ) respectively with both

of them pointing in the + z direction. Without changing the orientations of the dipoles P2

is moved to ( 0, 2, 0 ) . The ratio of the electrostatic potential energy of the dipoles after

moving to that before moving is


1 1 1 1
(a) (b) (c) (d)
16 2 4 8
Ans: (d)
1 U 2 r13 1
Solution: Electrostatic potential energy U ∝ ⇒ = =
r3 U 1 r23 8
Q4. A small magnetic dipole is kept at the origin in the x-y plane. One wire L1 is located at
z = −a in the x-z plane with a current I flowing in the positive x direction. Another wire
L2 is at z = + a in y-z plane with the same current I as in L1, flowing in the positive y-
direction. The angle φ made by the magnetic dipole with respect to the positive x-axis is
(a) 225o (b) 120o (c) 45o (d) 270o
Ans.: (a)

Solution: Magnetic field at z = 0 due to wire at z = −a is B = − Byˆ .

Magnetic field at z = 0 due to wire at z = + a is B = − Bxˆ .


Resultant magnetic field at z = 0 makes an angle of 45 0 with − x̂ and 225 0 with x̂ .

                                                                                
Head office  Branch office 
 
fiziks, H.No. 40 D, G.F, Jia Sarai,  Anand Institute of Mathematics, 
 
Near IIT, Hauz Khas, New Delhi‐16  28‐B/6, Jia Sarai, Near IIT 
 
Phone: 011‐26865455/+91‐9871145498 Hauz Khas, New Delhi‐16 
                                                   
                                             Website: www.physicsbyfiziks.com                                                                                          
                                                            Email: fiziks.physics@gmail.com                                                                   71 
fiziks
Institute for NET/JRF, GATE, IIT‐JAM, JEST, TIFR and GRE in PHYSICAL SCIENCES 
 
IIT-JAM-2007
Q5. A uniform and constant magnetic field B coming out of the
plane of the paper exists in a rectangular region as shown in B

the figure. A conducting rod PQ is rotated about O with a ω


uniform angular speed ω in the plane of the paper. The emf P
EPQ induced between P and Q is best represented by the
O
graph
Q
(a ) E PQ
(b ) E PQ

O O
t t

(c ) E PQ (d ) E PQ

O O
t t

Ans.: (a)

IIT-JAM-2008
Q6. If the electrostatic potential at a point ( x, y ) is given by V = ( 2 x + 4 y ) volts, the

electrostatic energy density at that point ( in J / m3 ) is

ε 0 (2 x + 4 y )2
1
(a) 5ε 0 (b) 10ε 0 (c) 20ε 0 (d)
2
Ans.: (a)

                                                                                
Head office  Branch office 
 
fiziks, H.No. 40 D, G.F, Jia Sarai,  Anand Institute of Mathematics, 
 
Near IIT, Hauz Khas, New Delhi‐16  28‐B/6, Jia Sarai, Near IIT 
 
Phone: 011‐26865455/+91‐9871145498 Hauz Khas, New Delhi‐16 
                                                   
                                             Website: www.physicsbyfiziks.com                                                                                          
                                                            Email: fiziks.physics@gmail.com                                                                   72 
fiziks
Institute for NET/JRF, GATE, IIT‐JAM, JEST, TIFR and GRE in PHYSICAL SCIENCES 
 
Solution: E = −∇V = −2 xˆ − 4 yˆ ⇒ E = 20 V / m

1 2 1
Electrostatic energy density = ε 0 E = ε 0 × 20= 10ε 0 J / m 3
2 2

IIT-JAM-2009
Q7. An oscillating voltage V ( t ) = V0 cos ωt is applied across a parallel plate capacitor having

a plate separation d. The displacement current density through the capacitor is

V (t ) = V0 cos ω t

ε 0ωV0 cos ωt ε 0 μ 0ωV0 cos ωt


(a) (b)
d d
ε 0 μ 0ωV0 cos ωt ε 0ωV0 sin ωt
(c) − (d) −
d d
Ans.: (d)
∂E ε 0 ∂V (t ) ε ωV sin ωt
Solution: Displacement current density J d = ε 0 = =− 0 0
∂t d ∂t d
Q8. ( )
An electric field E (r ) = αrˆ + β sin θ cos φφˆ exists in space. What will be the total charge
enclosed in a sphere of unit radius centered at the origin?
(a) 4πε0α (b) 4πε0 (α + β) (c) 4πε0 (α - β) (d) 4πε0β
Ans.: (a)
(
Solution: Qenc = ε 0 ∫ E ⋅ da = ε 0 ∫ αrˆ + β sin θ cos φφˆ ⋅ r 2 sin θdθdφrˆ = 4παε 0 )( )

                                                                                
Head office  Branch office 
 
fiziks, H.No. 40 D, G.F, Jia Sarai,  Anand Institute of Mathematics, 
 
Near IIT, Hauz Khas, New Delhi‐16  28‐B/6, Jia Sarai, Near IIT 
 
Phone: 011‐26865455/+91‐9871145498 Hauz Khas, New Delhi‐16 
                                                   
                                             Website: www.physicsbyfiziks.com                                                                                          
                                                            Email: fiziks.physics@gmail.com                                                                   73 
fiziks
Institute for NET/JRF, GATE, IIT‐JAM, JEST, TIFR and GRE in PHYSICAL SCIENCES 
 
IIT-JAM-2010
Q9. The magnetic field associated with the electric field vector E = E 0 sin (kz − ωt ) ˆj is given
by
E0 E0
(a) B = − sin (kz − ωt )iˆ (b) B = sin (kz − ωt )iˆ
c c
E0 E0
(c) B = sin (kz − ωt ) ˆj (d) B = sin (kz − ωt )kˆ
c c
Ans.: (a)

k ×E kzˆ × E0 sin ( kz − ωt ) yˆ kE0 E0


Solution: B = = =− sin ( kz − ωt ) xˆ = − sin ( kz − ωt ) xˆ
ω ω ω c
Q10. Assume that z = 0 plane is the interface between two linear and homogenous dielectrics
(see figure). The relative permittivities are ε r = 5 for z > 0 and ε r = 4 for z < 0 . The

( )
electric field in the region z > 0 is E 1 = 3iˆ − 5 ˆj + 4kˆ k V m . If there are no free charges
on the interface, the electric field in the region z < 0 is given by
z
εr = 5
z=0

εr = 4

⎛3 5 ⎞
(a) E 2 = ⎜ iˆ − ˆj + kˆ ⎟k V m
⎝4 4 ⎠
(
(b) E 2 = 3iˆ − 5 ˆj + kˆ k V m )
(
(c) E 2 = 3iˆ − 5 ˆj − 5kˆ k V m ) (
(d) E 2 = 3iˆ − 5 ˆj + 5kˆ k V m )
Ans.: (d)
Solution: ∵ E1 = E 2 ⇒ E 2 = 3iˆ − 5 ˆj

and σ f = 0 ⇒ D1⊥ = D2⊥ ⇒ E 2⊥ =


ε1 ⊥ 5
ε2
E1 = + 4kˆ = 5kˆ
4
( )
(
⇒ E 2 = 3iˆ − 5 ˆj + 5kˆ k V m )
                                                                                
Head office  Branch office 
 
fiziks, H.No. 40 D, G.F, Jia Sarai,  Anand Institute of Mathematics, 
 
Near IIT, Hauz Khas, New Delhi‐16  28‐B/6, Jia Sarai, Near IIT 
 
Phone: 011‐26865455/+91‐9871145498 Hauz Khas, New Delhi‐16 
                                                   
                                             Website: www.physicsbyfiziks.com                                                                                          
                                                            Email: fiziks.physics@gmail.com                                                                   74 
fiziks
Institute for NET/JRF, GATE, IIT‐JAM, JEST, TIFR and GRE in PHYSICAL SCIENCES 
 
Q11. A closed Gaussian surface consisting of a hemisphere and a circular disc of radius R , is
placed in a uniform electric field E , as shown in the figure. The circular disc makes an
angle θ = 300 with the vertical. The flux of the electric field vector coming out of the
curved surface of the hemisphere is
1
(a) π R2 E
2 E

3 θ
(b) π R2 E
2
(c) π R 2 E
(d) 2π R 2 E
Ans.: (b)

3 1
Solution: E = E cos 30 zˆ + E sin 30 xˆ = E zˆ + E xˆ E
2 2
⎛ 3 ⎞ 300
φ E = ∫ E ⋅ da = ∫ ∫ ⎜⎜
1
E zˆ + E xˆ ⎟⎟ ⋅ R 2 sin θdθdφrˆ ( ) ẑ
S ⎝ 2 2 ⎠
π / 2 2π
⎛ 3 ⎞
E cos θ + E sin θ cos φ ⎟⎟(sin θdθdφ )
1
φE = R ∫ ∫ ⎜⎜
2

θ φ ⎝
=0 =0
2 2 ⎠
π / 2 2π π / 2 2π
ER 2 ∫ ∫ (cos θ sin θ )dθdφ + ER 2 ∫ ∫ (sin 2 θ cos φ )dθdφ
3 1
φE =
2 θ =0 φ =0 2 θ =0 φ =0

3 1 3 2
φE = ER 2 × 2π × + 0 = πR E
2 2 2
OR

3
φE = ∫ E ⋅ da = E cos 300 × π R 2 = π R2 E
S
2

                                                                                
Head office  Branch office 
 
fiziks, H.No. 40 D, G.F, Jia Sarai,  Anand Institute of Mathematics, 
 
Near IIT, Hauz Khas, New Delhi‐16  28‐B/6, Jia Sarai, Near IIT 
 
Phone: 011‐26865455/+91‐9871145498 Hauz Khas, New Delhi‐16 
                                                   
                                             Website: www.physicsbyfiziks.com                                                                                          
                                                            Email: fiziks.physics@gmail.com                                                                   75 
fiziks
Institute for NET/JRF, GATE, IIT‐JAM, JEST, TIFR and GRE in PHYSICAL SCIENCES 
 
IIT-JAM-2011
Q12. Equipotential surface corresponding to a particular charge distribution are given by

4 x 2 + ( y − 2 ) + z 2 = Vi where the values of Vi are constants. The electric field E at the


2

origin is
(a) E = 0 (b) E = 2 xˆ (c) E = 4 yˆ (d) E = −4 yˆ
Ans.: (d)
Solution: E = −∇V = 8 xxˆ + 2 ( y − 2 ) yˆ + 2 zzˆ ⇒ E ( 0, 0 ) = −4 yˆ

IIT-JAM-2012
Q13. A parallel plate air-gap capacitor is made up of two plates of area 10 cm2 each kept at a
distance of 0.88 mm. A sine wave of amplitude 10 V and frequency 50 Hz is applied
across the capacitor as shown in the figure. The amplitude of the displacement current
density (in mA/m2) between the plates will be closest to

(a) 0.03 (b) 0.30 (c) 3.00 (d) 30.00


Ans.: (a)
∂E ε 0 ∂V (t ) ε ωV sin ωt
Solution: Displacement current density J d = ε 0 = =− 0 0
∂t d ∂t d
ε 0ωV0 2πε 0 fV0
Amplitude of the displacement current density (in mA/m2) J 0 d = =
d d
fV0 1 50 × 10
⇒ J 0 d = 4πε 0 = −5
= 0.03 mA / m 2
2d 9 × 10 2 × 88 ×10
9

                                                                                
Head office  Branch office 
 
fiziks, H.No. 40 D, G.F, Jia Sarai,  Anand Institute of Mathematics, 
 
Near IIT, Hauz Khas, New Delhi‐16  28‐B/6, Jia Sarai, Near IIT 
 
Phone: 011‐26865455/+91‐9871145498 Hauz Khas, New Delhi‐16 
                                                   
                                             Website: www.physicsbyfiziks.com                                                                                          
                                                            Email: fiziks.physics@gmail.com                                                                   76 
fiziks
Institute for NET/JRF, GATE, IIT‐JAM, JEST, TIFR and GRE in PHYSICAL SCIENCES 
 
Q14. A segment of a circular wire of radius R, extending from θ = 0 to π / 2 , carries a constant
linear charge density λ . The electric field at origin O is
λ y
(a) (− xˆ − yˆ )
4πε 0 R
λ ⎛ 1 1 ⎞
(b) ⎜− xˆ − yˆ ⎟
4πε 0 R ⎝ 2 2 ⎠
λ ⎛ 1 1 ⎞ R
(c) ⎜ − xˆ − yˆ ⎟
4πε 0 R ⎝ 2 2 ⎠ θ
O x
(d) 0
Ans.: (a) y
Solution: E = − Ex xˆ − E y yˆ

where Ex = ∫ dE cosθ , E
line
y = ∫ dE sin θ .
line
dl

1 λ dl R
and dE = .
4πε 0 R 2 O θ
x
1 λ dl λ π /2
Rdθ θ
Ex = ∫
line
4πε 0 R 2
cos θ =
4πε 0 ∫ cos θ
0
R2
dE
λ λ
[sin θ ]0 =
π /2
⇒ Ex =
4πε 0 R 4πε 0 R
π /2
1 λ dl λ Rdθ
Similarly E y = ∫ sin θ = ∫ sin θ
line
4πε 0 R 2
4πε 0 0
R2

λ λ
[ − cos θ ]0 =
π /2
⇒ Ey =
4πε 0 R 4πε 0 R
λ
Thus E = − Ex xˆ − E y yˆ = ( − xˆ − yˆ )
4πε 0 R

                                                                                
Head office  Branch office 
 
fiziks, H.No. 40 D, G.F, Jia Sarai,  Anand Institute of Mathematics, 
 
Near IIT, Hauz Khas, New Delhi‐16  28‐B/6, Jia Sarai, Near IIT 
 
Phone: 011‐26865455/+91‐9871145498 Hauz Khas, New Delhi‐16 
                                                   
                                             Website: www.physicsbyfiziks.com                                                                                          
                                                            Email: fiziks.physics@gmail.com                                                                   77 
fiziks
Institute for NET/JRF, GATE, IIT‐JAM, JEST, TIFR and GRE in PHYSICAL SCIENCES 
 
IIT-JAM-2014
Q15. A particle of mass m carrying charge q is moving in a circle in a magnetic field B.
According to Bohr’s model, the energy of the particle in the nth level is

1 ⎛ hqB ⎞ ⎛ hqB ⎞ ⎛ hqB ⎞ ⎛ hqB ⎞


(a) ⎜⎜ ⎟⎟ (b) n⎜⎜ ⎟⎟ (c) n⎜⎜ ⎟⎟ (d) n⎜⎜ ⎟⎟
n2 ⎝π m⎠ ⎝π m⎠ ⎝ 2π m ⎠ ⎝ 4π m ⎠
Ans.: (d)
q 2 B 2 rn2 mvn m n n
Solution: En = ∵ mvn rn = n and rn = ⇒ rn = ⇒ rn2 =
2m qB qB mrn qB

q 2 B 2 rn2 q 2 B 2 n ⎛ qBh ⎞
⇒ En = = × = n⎜ ⎟
2m 2m qB ⎝ 4π m ⎠
Q16. A conducting slab of copper PQRS is kept on the xy plane in a uniform magnetic field
along x-axis as indicted in the figure.
Z
A steady current I flows through the
S R
cross section of the slab along the
Q
y-axis. The direction of the electric P I
Y
field inside the slab, arising due to B
the applied magnetic field is along X
the
(a) negative Y direction (b) positive Y direction
(c) negative Z direction (d) positive Z direction
Ans.: (c)

                                                                                
Head office  Branch office 
 
fiziks, H.No. 40 D, G.F, Jia Sarai,  Anand Institute of Mathematics, 
 
Near IIT, Hauz Khas, New Delhi‐16  28‐B/6, Jia Sarai, Near IIT 
 
Phone: 011‐26865455/+91‐9871145498 Hauz Khas, New Delhi‐16 
                                                   
                                             Website: www.physicsbyfiziks.com                                                                                          
                                                            Email: fiziks.physics@gmail.com                                                                   78 
fiziks
Institute for NET/JRF, GATE, IIT‐JAM, JEST, TIFR and GRE in PHYSICAL SCIENCES 
 
Q17. In a parallel plate capacitor the distance between the plates is 10 cm. Two dielectric slabs
of thickness 5 cm each and dielectric constants K 1 = 2 and K 2 = 4 respectively, are
inserted between the plates. A potential of 100 V is applied across the capacitor as shown
in the figure. The value of the net bound surface charge density at the interface of the two
dielectrics is

K2 = 4
10 cm 100 V
K1 = 2

2000 1000 2000


(a) − ε0 (b) − ε0 (c) − 250ε 0 (d) ε0
3 3 3
Ans.: (a)
σ σ σ σ 3σ −σ
Solution: V = E1d + E2 d = d+ d= d+ d= d K2 = 4 +σ 2
ε1 ε2 2ε 0 4ε 0 4ε 0 −σ 2
K1 = 2 +σ 1
−σ 1
V = 100 volts, d = 5 × 10−2 cm +σ
4ε 0 4ε 0 4 ×104
⇒σ = V= ×100 = ε0
3d 3 × 5 ×10−2 15
σ σ
P1 = ε 0 χ e E1 = ε 0 ( K1 − 1) E1 ⇒ σ 1 = ε 0 × =
2ε 0 2
σ 3σ
P2 = ε 0 χ e E2 = ε 0 ( K 2 − 1) E1 ⇒ σ 2 = 3ε 0 × =
4ε 0 4
σ 3σ σ 1 4 × 104 2000
⇒ σ = σ1 − σ 2 = − =− =− × ε0 = − ε0
2 4 4 4 15 3
Q18. A rigid uniform horizontal wire PQ of mass M, pivoted at P, carries a constant current I.
It rotates with a constant angular speed in a
P Q
uniform vertical magnetic field B. If the current
were switched off, the angular acceleration of
the wire, in terms of B, M and I would be
2 BI 3BI BI
(a) 0 (b) (c) (d)
3M 2M M
Ans.: (c)

                                                                                
Head office  Branch office 
 
fiziks, H.No. 40 D, G.F, Jia Sarai,  Anand Institute of Mathematics, 
 
Near IIT, Hauz Khas, New Delhi‐16  28‐B/6, Jia Sarai, Near IIT 
 
Phone: 011‐26865455/+91‐9871145498 Hauz Khas, New Delhi‐16 
                                                   
                                             Website: www.physicsbyfiziks.com                                                                                          
                                                            Email: fiziks.physics@gmail.com                                                                   79 
fiziks
Institute for NET/JRF, GATE, IIT‐JAM, JEST, TIFR and GRE in PHYSICAL SCIENCES 
 
Q19. A steady current in a straight conducting wire produces a surface charge on it. Let E out

and Ein be the magnitudes of the electric fields just outside and just inside the wire,
respectively. Which of the following statements is true for these fields?
(a) E out is always greater than Ein

(b) E out is always smaller than Ein

(c) E out could be greater or smaller than Ein

(d) E out is equal to Ein


Ans.: (a)
Q20. A small charged spherical shell of radius 0.01 m is at a potential of 30 V. The
electrostatic energy of the shell is
(a) 10 −10 J (b) 5 × 10 −10 J (c) 5 × 10 −9 J (d) 10 −9 J
Ans.: (b)
q q2
Solution: V = and W = .
4πε 0 R 8πε 0 R

( 4πε 0VR )
2
4πε 0V 2 R 900 × 10−2
Thus W = = = = 0.5 × 10−9 = 5 × 10−10 Joules
8πε 0 R 2 9 ×109 × 2
Q21. A ring of radius R carries a linear charge density λ . It is rotating with angular speed ω.
The magnetic field at its center is
3μ 0 λω μ 0 λω μ 0 λω
(a) (b) (c) (d) μ 0 λω
2 2 π
Ans.: (b)
μ0 I μ0λω
Solution: B = where I = λ v = λ Rω . Thus B = .
2R 2

                                                                                
Head office  Branch office 
 
fiziks, H.No. 40 D, G.F, Jia Sarai,  Anand Institute of Mathematics, 
 
Near IIT, Hauz Khas, New Delhi‐16  28‐B/6, Jia Sarai, Near IIT 
 
Phone: 011‐26865455/+91‐9871145498 Hauz Khas, New Delhi‐16 
                                                   
                                             Website: www.physicsbyfiziks.com                                                                                          
                                                            Email: fiziks.physics@gmail.com                                                                   80 
fiziks
Institute for NET/JRF, GATE, IIT‐JAM, JEST, TIFR and GRE in PHYSICAL SCIENCES 
 
IIT-JAM-2015
⎡ ⎛ π ⎞⎤
Q22. The electric field of a light wave is given by E = E 0 ⎢iˆ sin (ωt − kz ) + ˆj sin ⎜ ωt − kz − ⎟⎥ .
⎣ ⎝ 4 ⎠⎦
The polarization state of the wave is
(a) Left handed circular (b) Right handed circular
(c) Left handed elliptical (d) Right handed elliptical
Ans.: (c)
⎛ π⎞
Solution: Ex = E0 sin (ωt − kz ) , E y = E0 sin ⎜ ωt − kz − ⎟ .
⎝ 4⎠
Thus resultant is elliptically polarized wave.
⎛ π⎞
At z = 0, Ex = E0 sin (ωt ) , E y = E0 sin ⎜ ωt − ⎟
⎝ 4⎠
E π E
When ωt = 0, Ex = 0, E y = − 0 and when ωt = , Ex = 0 , E y = 0
2 4 2
Q23. A charge q is at the center of two concentric spheres. The outward electric flux through
the inner sphere is φ while that through the outer sphere is 2φ . The amount of charge
contained in the region between the two spheres is
(a) 2q (b) q (c) − q (d) − 2q
Ans.: (b)
q q + q′
Solution: φ = , φ ′ = 2φ = ⇒ q′ = q
ε0 ε0
Q24. A positively charged particle, with a charge q , enters a region in which there is a uniform

electric field E and a uniform magnetic field B , both directed parallel to the positive
y -axis. At t = 0 , the particle is at the origin and has a speed v0 directed along the
positive x - axis. The orbit of the particle, projected on the x- z plane, is a circle. Let T
be the time taken to complete one revolution of this circle. The y -coordinate of the
particle at t = T is given by
π 2 mE 2π 2 mE π 2 mE v0π m 2πmv0
(a) (b) (c) + (d)
2qB 2
qB 2
qB 2
qB qB
                                                                                
Head office  Branch office 
 
fiziks, H.No. 40 D, G.F, Jia Sarai,  Anand Institute of Mathematics, 
 
Near IIT, Hauz Khas, New Delhi‐16  28‐B/6, Jia Sarai, Near IIT 
 
Phone: 011‐26865455/+91‐9871145498 Hauz Khas, New Delhi‐16 
                                                   
                                             Website: www.physicsbyfiziks.com                                                                                          
                                                            Email: fiziks.physics@gmail.com                                                                   81 
fiziks
Institute for NET/JRF, GATE, IIT‐JAM, JEST, TIFR and GRE in PHYSICAL SCIENCES 
 
Ans.: (b) z
2
1 1 qE ⎛ 2π m ⎞ 2π 2 mE
Solution: y = u y t + a y t 2 ⇒ y = ⎜ ⎟ =
2 2 m ⎝ qB ⎠ qB 2
E, B
y
v0
x
Q25. A hollow, conducting spherical shell of inner radius R1 and
outer radius R2 encloses a charge q inside, which is located at a
R1
distance d (< R1 ) from the centre of the spheres. The potential at
q d
the centre of the shell is R2

1 q
(a) Zero (b)
4πε 0 d
1 ⎛q q ⎞ 1 ⎛q q q ⎞
(c) ⎜ − ⎟ (d) ⎜ − + ⎟
4πε 0 ⎝ d R1 ⎠ 4πε 0 ⎝ d R1 R2 ⎠
Ans.: (d)
Solution: charge induced on inner surface is −q and charge induced on outer surface is + q .

1 ⎛q q q ⎞
Thus V = ⎜ − + ⎟.
4πε 0 ⎝ d R1 R2 ⎠
Q26. A conducting wire is in the shape of a regular hexagon, which is
inscribed inside an imaginary circle of radius R , as shown. A current I R
I flows through the wire The magnitude of the magnetic field at the C
center of the circle is

3μ 0 I μ0 I 3μ 0 I 3μ 0 I
(a) (b) (c) (d)
2πR 2 3πR πR 2πR
Ans.: (c) C
3
Solution: d = R cos 300 = R
2 R d
μI 600
∵ B = 0 ( sin θ 2 − sin θ1 )
4π d I
μ0 I μ0 I μ0 I
⇒ B1 = 2sin 300 = 2sin 300 =
4π d 3 2 3π R
4π R
2
                                                                                
Head office  Branch office 
 
fiziks, H.No. 40 D, G.F, Jia Sarai,  Anand Institute of Mathematics, 
 
Near IIT, Hauz Khas, New Delhi‐16  28‐B/6, Jia Sarai, Near IIT 
 
Phone: 011‐26865455/+91‐9871145498 Hauz Khas, New Delhi‐16 
                                                   
                                             Website: www.physicsbyfiziks.com                                                                                          
                                                            Email: fiziks.physics@gmail.com                                                                   82 
fiziks
Institute for NET/JRF, GATE, IIT‐JAM, JEST, TIFR and GRE in PHYSICAL SCIENCES 
 
μ0 I 3μ0 I 3μ0 I
⇒ B = 6 B1 = 6 × = =
2 3π R 3π R πR

SECTION–B: MSQ
Q27. For an electromagnetic wave traveling in free space, the electric field is given

( V
)
by E = 100 cos 10 8 t + kx ˆj . Which of the following statements are true?
m
(a) The wavelength of the wave in meter is 6π
(b) The corresponding magnetic field is directed along the positive z direction
(c) The Poynting vector is directed along the positive z direction
(d) The wave is linearly polarized
Ans.: (a) and (d)
Solution: E = 100 cos (108 t + kx ) ˆj V / m

2π c
2π × 3 × 108
ω = 10 ⇒ = 10 ⇒ λ =
8
= 6π . Option (a) is true
8

λ 108

( )
B ∝ kˆ × E ∝ ( − xˆ × yˆ ) ∝ − zˆ . Option (b) is wrong

S ∝ kˆ ∝ − xˆ . Option (c) is wrong. Option (d) is true.


Q28. Consider the circuit, consisting of an AC function generator V (t ) = V0 sin 2πvt with

V0 = 5V an inductor L = 8.0mH , resistor R = 5Ω and a capacitor C = 100μF . Which of


the following statements are true if we vary the frequency?
L

R C

(a) The current in the circuit would be maximum at ν = 178Hz


(b) The capacitive reactance increases with frequency
(c) At resonance, the impedance of the circuit is equal to the resistance in the circuit
(d) At resonance, the current in the circuit is out of phase with the source voltage

                                                                                
Head office  Branch office 
 
fiziks, H.No. 40 D, G.F, Jia Sarai,  Anand Institute of Mathematics, 
 
Near IIT, Hauz Khas, New Delhi‐16  28‐B/6, Jia Sarai, Near IIT 
 
Phone: 011‐26865455/+91‐9871145498 Hauz Khas, New Delhi‐16 
                                                   
                                             Website: www.physicsbyfiziks.com                                                                                          
                                                            Email: fiziks.physics@gmail.com                                                                   83 
fiziks
Institute for NET/JRF, GATE, IIT‐JAM, JEST, TIFR and GRE in PHYSICAL SCIENCES 
 
Ans.: (a) and (c)
1 1
Solution: ν = = = 178 Hz . Option (a) is true.
2π LC 2 × 3.14 (8 ×10 )(100 ×10 )
−3 −6

1
XC = ⇒ X C ↓ as ω ↑ . Option (b) is wrong
ωC
Option (c) is true
Option (d) is wrong
Q29. A unit cube made of a dielectric material has a polarization P = 3iˆ + 4 ˆj units. The edges
of the cube are parallel to the Cartesian axes. Which of the following statements are
true?
(a) The cube carries a volume bound charge of magnitude 5 units
(b) There is a charge of magnitude 3 units on both the surfaces parallel to the y − z plane
(c) There is a charge of magnitude 4 units on both the surfaces parallel to the x − z plane
(d) There is a net non-zero induced charge on the cube
Ans.: (b) and (c)
Solution: ∵ P = 3iˆ + 4 ˆj ⇒ ρb = −∇.P = 0 . Option (a) is wrong

( )( )
At x = 0 , σ b = P.nˆ = 3iˆ + 4 ˆj . −iˆ = −3 , At x = 1 , σ b = P.nˆ = 3iˆ + 4 ˆj . iˆ = 3 ( )( )
Option (b) is true

( )( )
At y = 0 , σ b = P.nˆ = 3iˆ + 4 ˆj . − ˆj = −4 , At y = 1 , σ b = P.nˆ = 3iˆ + 4 ˆj . ˆj = 4 ( )( )
Option (c) is true.
Option (d) is wrong
Q30. The power radiated by sun is 3.8 × 10 26 W and its radius is 7 × 10 5 km . The magnitude of
W
the Poynting vector (in ) at the surface of the sun is………………
cm 2
Ans.: 6174
P 3.8 × 1026
Solution: I = = W / cm2 = 6174 W / cm 2
A 4π × ( 7 ×10 )
10

                                                                                
Head office  Branch office 
 
fiziks, H.No. 40 D, G.F, Jia Sarai,  Anand Institute of Mathematics, 
 
Near IIT, Hauz Khas, New Delhi‐16  28‐B/6, Jia Sarai, Near IIT 
 
Phone: 011‐26865455/+91‐9871145498 Hauz Khas, New Delhi‐16 
                                                   
                                             Website: www.physicsbyfiziks.com                                                                                          
                                                            Email: fiziks.physics@gmail.com                                                                   84 
fiziks
Institute for NET/JRF, GATE, IIT‐JAM, JEST, TIFR and GRE in PHYSICAL SCIENCES 
 
Q31. In an experiment on charging of an initially uncharged capacitor, an RC circuit is made
with the resistance R = 10kΩ and the capacitor C = 1000μF along with a voltage source
of 6V . The magnitude of the displacement current through the capacitor (in μA ),
5 seconds after the charging has started, is…………………
Ans.: 364
V − t / RC 6 −6 6 6 6
Solution: I = = e −5/10×10 ×1000×10 = 4 e−5/10 = = = 364 μ A
3
e
R 10 ×10 3
10 e × 10 1.65 × 10
4 4

Q32. In a region of space, a time dependent magnetic field B (t ) = 0.4t tesla points vertically
upwards. Consider a horizontal, circular loop of radius 2 cm in this region. The
magnitude of the electric field (in mV / m ) induced in the loop is…………….
Ans.: 4
∂B r ∂B 2 ×10−2
Solution: E × 2π r = − × π r ⇒ E =
2
= 0.4 = 4 mV / m
∂t 2 ∂t 2
Q33. A plane electromagnetic wave of frequency 5 × 1014 Hz and amplitude 103 V / m traveling
in a homogeneous dielectric medium of dielectric constant 1.69 is incident normally at
the interface with a second dielectric medium of dielectric constant 2.25 . The ratio of the
amplitude of the transmitted wave to that of the incident wave is………………
Ans.: 0.93

⎛ 2n1 ⎞ E0T ⎛ 2 ε r1 ⎞ ⎛ 2 1.69 ⎞


Solution: E0T = ⎜ ⎟ E0 I ⇒ =⎜ ⎟ = ⎜⎜ ⎟⎟ = 0.93
+ ⎜ ⎟
n
⎝ 1 2⎠n E0I ⎝ ε r1 + ε r 2 ⎠ ⎝ 1.69 + 2.25 ⎠

                                                                                
Head office  Branch office 
 
fiziks, H.No. 40 D, G.F, Jia Sarai,  Anand Institute of Mathematics, 
 
Near IIT, Hauz Khas, New Delhi‐16  28‐B/6, Jia Sarai, Near IIT 
 
Phone: 011‐26865455/+91‐9871145498 Hauz Khas, New Delhi‐16 
                                                   
                                             Website: www.physicsbyfiziks.com                                                                                          
                                                            Email: fiziks.physics@gmail.com                                                                   85 
fiziks
Institute for NET/JRF, GATE, IIT‐JAM, JEST, TIFR and GRE in PHYSICAL SCIENCES 
 
IIT-JAM-2016
Q34. For an infinitely long wire with uniform line-charge density, λ along the z -axis, the
electric field at a point ( a, b, 0 ) away from the origin is

( eˆx , eˆy and eˆz are unit vectors in Cartesian – coordinate system)

λ λ
(a) ( eˆ + eˆy ) (b) ( aeˆ + beˆy )
2πε 0 a 2 + b 2
x
(
2πε 0 a 2 + b 2 ) x

λ λ
(c) eˆx (d) eˆz
2πε 0 a + b 2 2
2πε 0 a + b 2 2

Ans.: (b)
λ λ λ
Solution: E =
2πε 0 r
rˆ =
2πε 0 r 2
r=
2πε 0 ( a 2 + b 2 )
( aeˆx + beˆy ) ∵ r = a 2 + b2

Q35. A 1 W point source at origin emits light uniformly in all the directions. If the units for

both the axes are measured in centimeter, then the Poynting vector at the point (1,1, 0 ) in

W
is
cm 2

(a)

1
2
( eˆ x + eˆy ) (b)
1
16π
( eˆx + eˆy )

(c)
16π
1
2
( eˆ x + eˆy ) (d)
4π 2
1
( eˆ x + eˆy )

Ans.: (a)

P P r P 1 1
Solution: I =< S >= rˆ = = r= ( xˆ + yˆ ) = ( xˆ + yˆ )
A 4π r r 4π r
2 3
4π × 2 2 8π 2

∵ r = 12 + 12 = 2

                                                                                
Head office  Branch office 
 
fiziks, H.No. 40 D, G.F, Jia Sarai,  Anand Institute of Mathematics, 
 
Near IIT, Hauz Khas, New Delhi‐16  28‐B/6, Jia Sarai, Near IIT 
 
Phone: 011‐26865455/+91‐9871145498 Hauz Khas, New Delhi‐16 
                                                   
                                             Website: www.physicsbyfiziks.com                                                                                          
                                                            Email: fiziks.physics@gmail.com                                                                   86 
fiziks
Institute for NET/JRF, GATE, IIT‐JAM, JEST, TIFR and GRE in PHYSICAL SCIENCES 
 
Q36. A charged particle in a uniform magnetic field B = B0 eˆz starts moving from the origin with

velocity v = ( 3eˆx + 2eˆz ) m / s . The trajectory of the particle and the time t at which it reaches

2 meters above the xy - plane are

( eˆx , eˆy and eˆz are unit vectors in Cartesian-coordinate system)

(a) Helical path; t = 1 s (b) Helical path; t = 2 / 3 s

(c) Circular path; t = 1 s (d) Circular path; t = 2 / 3 s

Ans.: (a)
2m
Solution: v⊥ = 3 m / s and v = 2 m / s , thus t= = 1 sec
v

Q37. The phase difference (δ ) between input and output voltage for the following circuits (i)
C R
and (ii)

vi C vo vi C vo

will be (i) (ii)

(a) 0 and 0 (b) π / 2 and 0 < δ ≤ π / 2 respectively


(c) π / 2 and π / 2 (d) 0 and 0 < δ ≤ π / 2 respectively
Ans.: (d)
XC v 1
(i) vo = vi ⇒ o = , phase difference (δ ) is 0.
XC + XC vi 2
XC v 1 1 1
(ii) vo = vi ⇒ o = = = e − iωCR
R + XC vi 1 + R / X C 1 + iωCR 1 + (ωCR )
2

Phase difference (δ ) is 0 < δ ≤ π / 2 .

                                                                                
Head office  Branch office 
 
fiziks, H.No. 40 D, G.F, Jia Sarai,  Anand Institute of Mathematics, 
 
Near IIT, Hauz Khas, New Delhi‐16  28‐B/6, Jia Sarai, Near IIT 
 
Phone: 011‐26865455/+91‐9871145498 Hauz Khas, New Delhi‐16 
                                                   
                                             Website: www.physicsbyfiziks.com                                                                                          
                                                            Email: fiziks.physics@gmail.com                                                                   87 
fiziks
Institute for NET/JRF, GATE, IIT‐JAM, JEST, TIFR and GRE in PHYSICAL SCIENCES 
 
Q38. In the following RC circuit, the capacitor was charged in two different ways.
(i) The capacitor was first charged to 5V by moving the toggle switch to position P and
then it was charged to 10 V by moving the toggle switch to position Q .
(ii) The capacitor was directly charged to10 V , by keeping the toggle switch at
position Q .
Assuming the capacitor to be ideal, which one of the following statements is correct?
R C

P 5V
10 V
Q
(a) The energy dissipation in cases (i) and (ii) will be equal and non-zero
(b) The energy dissipation for case (i) will be more than that for case (ii)
(c) The energy dissipation for case (i) will be less than that for case (ii)
(d) The energy will not be dissipated in either case.
Ans.: (c)
1 1
Solution: The energy dissipation in cases (i) is = C ( 5 ) + C (10 − 5 ) = 25C
2 2

2 2
1
The energy dissipation in cases (ii) is = C (10 ) = 50 C
2

2
1
Q39. In the following RC network, for an input signal frequency f = , the voltage gain
2π RC
vo
and the phase angle φ between vo and vi respectively are
vi
R C

vi C
R vo

1 1 1 π 1 π
(a) and 0 (b) and 0 (c) and (d) and
2 3 2 2 3 2
Ans.: (b)
                                                                                
Head office  Branch office 
 
fiziks, H.No. 40 D, G.F, Jia Sarai,  Anand Institute of Mathematics, 
 
Near IIT, Hauz Khas, New Delhi‐16  28‐B/6, Jia Sarai, Near IIT 
 
Phone: 011‐26865455/+91‐9871145498 Hauz Khas, New Delhi‐16 
                                                   
                                             Website: www.physicsbyfiziks.com                                                                                          
                                                            Email: fiziks.physics@gmail.com                                                                   88 
fiziks
Institute for NET/JRF, GATE, IIT‐JAM, JEST, TIFR and GRE in PHYSICAL SCIENCES 
 
1 1
Solution: ∵ f = then X C = = − jR
2π RC j 2π fC
RX C − jR 2 − jR − j (1 + j ) R
ZP = = = = and Z S = R + X C = R − jR = R (1 − j )
R + X C R − jR 1 − j 2
ZP v 1 1 1 j (1 + j ) R
vo = vi ⇒ o = = = =
ZP + ZS vi 1 + Z S R (1 − j ) 2 R (1 − j ) jR − R − 2 R (1 − j )
1+ 1−
ZP − j (1 + j ) R j (1 + j ) R
2
v j (1 + j ) R j (1 + j ) R ( j − 1) 1
⇒ o = = = =
vi jR − R − 2 R (1 − j ) 3 jR − 3R 3 ( j − 1) 3
Q40. An arbitrarily shaped conductor encloses a charge q and is
surrounded by a conducting hollow sphere as shown in the figure. q
Four different regions of space, 1, 2,3 and 4 are indicated in the 1 3 4
figure. Which one of the following statements is correct? 2
(a) The electric field lines in region 2 are not affected by the
position of the charge q
(b) The surface charge density on the inner wall of the hollow sphere is uniform
(c) The surface charge density on the outer surface of the sphere is always uniform
irrespective of the position of charge q in region 1
(d) The electric field in region 2 has a radial symmetry
Ans.: (c)
Q41. Consider a small bar magnet undergoing simple harmonic motion (SHM) along the
x - axis. A coil whose plane is perpendicular to the x - axis is placed such that the magnet
passes in and out of it during its motion. Which one of the following statements is
correct? Neglect damping effects.
(a) Induced e.m.f. is minimum when the center of the bar magnet crosses the coil
(b) The frequency of the induced current in the coil is half of the frequency of the SHM
(c) Induced e.m.f. in the coil will not change with the velocity of the magnet
(d) The sign of the e.m.f. depends on the pole (N or S) face of the magnet which enters
into the coil

                                                                                
Head office  Branch office 
 
fiziks, H.No. 40 D, G.F, Jia Sarai,  Anand Institute of Mathematics, 
 
Near IIT, Hauz Khas, New Delhi‐16  28‐B/6, Jia Sarai, Near IIT 
 
Phone: 011‐26865455/+91‐9871145498 Hauz Khas, New Delhi‐16 
                                                   
                                             Website: www.physicsbyfiziks.com                                                                                          
                                                            Email: fiziks.physics@gmail.com                                                                   89 
fiziks
Institute for NET/JRF, GATE, IIT‐JAM, JEST, TIFR and GRE in PHYSICAL SCIENCES 
 
Ans.: (a)
Q42. Consider a spherical dielectric material of radius ‘ a ’ centered at origin. If the
polarization vector, P = P0 eˆx , where P0 !is a constant of appropriate dimensions, then

( eˆx , eˆy , and eˆz are unit vectors in Cartesian- coordinate system)

(a) the bound volume charge density is zero.


(b) the bound surface charge density is zero at ( 0, 0, a ) .

(c) the electric field is zero inside the dielectric


(d) the sign of the surface charge density changes over the surface.
Ans.: (a), (b), (d)
Solution: ρb = −∇.P = 0

σ b = P.nˆ = ( P0 xˆ ) .rˆ = P0 sin θ cos φ = 0 at ( 0, 0, a ) ∵θ = 0 .

Q43. For an electric dipole with moment P = p0 eˆz placed at the origin, ( p0 is a constant of

appropriate dimensions and eˆx , eˆy and eˆz are unit vectors in Cartesian coordinate system)

1
(a) potential falls as , where r is the distance from origin
r2
(b) a spherical surface centered at origin is an equipotential surface
(c) electric flux through a spherical surface enclosing the origin is zero
(d) radial component of E is zero on the xy - plane.
Ans.: (a), (c), (d)

rˆ. p p cos θ
Solution: Vdip ( r , θ ) = = .
4πε o r 2
4πε o r 2

E dip ( r ,θ ) =
p
4πε 0 r 3
( 2 cosθ rˆ + sin θθˆ ) .

                                                                                
Head office  Branch office 
 
fiziks, H.No. 40 D, G.F, Jia Sarai,  Anand Institute of Mathematics, 
 
Near IIT, Hauz Khas, New Delhi‐16  28‐B/6, Jia Sarai, Near IIT 
 
Phone: 011‐26865455/+91‐9871145498 Hauz Khas, New Delhi‐16 
                                                   
                                             Website: www.physicsbyfiziks.com                                                                                          
                                                            Email: fiziks.physics@gmail.com                                                                   90 
fiziks
Institute for NET/JRF, GATE, IIT‐JAM, JEST, TIFR and GRE in PHYSICAL SCIENCES 
 
Q44. Three infinitely-long conductors carrying currents I1 , I 2 and I 3
lie perpendicular to the plane of the paper as shown in the C3
figure. I 3 C2
I2
If the value of the integral ∫ B.dl for the loops C1 , C2 and
C1
C
I1
N
C3 are 2μ0 , 4μ0 and μ0 in the units of respectively, then
A
(a) I1 = 3 A into the paper (b) I 2 = 5 A out of the paper

(c) I 3 = 0 . (d) I 3 = 1A out of the paper


Ans.: (a), (b)
Solution: ∵ ∫ B.dl = μ0 I enc
C

⇒ I1 + I 2 = 2 , I 2 + I 3 = 4 , I1 + I 2 + I 3 = 1

⇒ I1 = −3 A , I 2 = 5 A and I 3 = −1 A .

Q45. The shape of a dielectric lamina is defined by the two curves y = 0 and y = 1 − x 2 . If the

charge density of the lamina σ = 15 y C / m 2 , then the total charge on the lamina
is…………….. C .
Ans.: 8 y
Solution: Total charge on the lamina is
1 1

(1 − x 2 ) dx
1− x 2 15
Q = ∫ σ da = ∫∫ ∫
2
15 ydxdy =
S −1
0 2 −1
1 x
15 ⎡ x3 ⎤
1
x5
⇒ Q = ∫ (1 + x 4 − 2 x 2 ) dx = ⎢ x + − 2 ⎥ −1
15 0 1
2 −1 2 ⎣ 5 3 ⎦ −1

15 ⎡ 1 2 ⎛ 1 2 ⎞ ⎤ 15 ⎡ 1 2 1 2 ⎤ 15 ⎡ 2 4⎤
⇒Q= = ⎢1 + − − ⎜ −1 − + ⎟ ⎥ = ⎢1 + − + 1 + − ⎥ = ⎢ 2 + − ⎥
2 ⎣ 5 3 ⎝ 5 3 ⎠⎦ 2 ⎣ 5 3 5 3⎦ 2 ⎣ 5 3⎦
15 16
⇒Q= × =8 C
2 15

                                                                                
Head office  Branch office 
 
fiziks, H.No. 40 D, G.F, Jia Sarai,  Anand Institute of Mathematics, 
 
Near IIT, Hauz Khas, New Delhi‐16  28‐B/6, Jia Sarai, Near IIT 
 
Phone: 011‐26865455/+91‐9871145498 Hauz Khas, New Delhi‐16 
                                                   
                                             Website: www.physicsbyfiziks.com                                                                                          
                                                            Email: fiziks.physics@gmail.com                                                                   91 
fiziks
Institute for NET/JRF, GATE, IIT‐JAM, JEST, TIFR and GRE in PHYSICAL SCIENCES 
 
Kinetic Theory, Thermodynamics
OBJECTIVE QUESTIONS

IIT-JAM-2005
5
Q1. The molar specific heat of a gas as given from the kinetic theory is R. If it is not
2
specified whether it is CP or CV, one could conclude that the molecules of the gas
(a) are definitely monatomic (b) are definitely rigid diatomic
(c) are definitely non-rigid diatomic (d) can be monatomic or rigid diatomic
Ans. : (d)
5R
Solution: If molecule is mono atomic then C p = and if molecule is rigid diatomic then
2
5R
CV = .
2
Q2. The value of entropy at absolute zero of temperature would be
(a) zero for all the materials
(b) finite for all the materials
(c) zero for some materials and non-zero for others
(d) unpredictable for any material
Ans. : (a)
Solution: If system will achieve absolute zero then it is perfectly ordered system then entropy
will be zero.
Q3. Which of the following statements is INCORRECT?
(a) Indistinguishable particles obey Maxwell-Boltzmann statistics
(b) All particles of an ideal Bose gas occupy a single energy state at T = 0
(c) The integral spin particles obey Bose-Einstein statistics
(d) Protons obey Fermi-Dirac statistics
Ans. : (a)
Solution: Distinguishable particles obey Maxwell-Boltzmann statistics.

                                                                                
Head office  Branch office 
 
fiziks, H.No. 40 D, G.F, Jia Sarai,  Anand Institute of Mathematics, 
 
Near IIT, Hauz Khas, New Delhi‐16  28‐B/6, Jia Sarai, Near IIT 
 
Phone: 011‐26865455/+91‐9871145498 Hauz Khas, New Delhi‐16 
                                                   
                                             Website: www.physicsbyfiziks.com                                                                                          
                                                            Email: fiziks.physics@gmail.com                                                                   92 
fiziks
Institute for NET/JRF, GATE, IIT‐JAM, JEST, TIFR and GRE in PHYSICAL SCIENCES 
 
IIT-JAM-2006
Q4. A solid melts into a liquid via first order phase transition. The relationship between the
pressure P and the temperature T of the phase transition is P = -2T + P0 , where P0 is a
constant. The entropy change associated with the phase transition is 1.0 J mole-1 K-1. The
⎛ dP ⎞
Clausius-Clapeyron equation for the latent heat is L = T ⎜ ⎟Δv . Here Δv = vliquid − v solid
⎝ dT ⎠
is the change in molar volume at the phase transition. The correct statement relating the
values of the volumes is
(a) vliquid = v solid (b) vliquid = v solid − 1

1
(c) vliquid = v solid − (d) v quid = v solid + 2
2
Ans. : (c)
dP
Solution: Since P = −2T + P0 ⇒ = −2
dT
⎛ dP ⎞ dL
It is given L = T ⎜ ⎟Δv ⇒ L = −2T Δv ⇒ = −2Δv
⎝ dT ⎠ dT

Since dS = 1.0 Jmole -1 K -1 1.0 J mole-1 K-1


dQ mdL 1
dS = = = 1 ⇒ 1 = −2Δv ⇒ Δv = −
T dT 2

IIT-JAM-2007
Q5. Experimental measurements of heat capacity per mole of Aluminum at low temperatures
show that the data can be fitted to the formula, CV = aT + bT 3, where a = 0.00135 JK-2
mole-1 , b = 2.48 x 10-5 JK-4 mole-1 and T is the temperature in in Kelvin. The entropy of
a mole of Aluminum at such temperatures is given by the formula
b aT b 3
(a) aT + T 3 + c, where c > 0 is a constant (b) + T + c, where c > 0 is a constant
3 2 4
b aT b 3
(c) aT + T 3 (d) + T
3 2 4

                                                                                
Head office  Branch office 
 
fiziks, H.No. 40 D, G.F, Jia Sarai,  Anand Institute of Mathematics, 
 
Near IIT, Hauz Khas, New Delhi‐16  28‐B/6, Jia Sarai, Near IIT 
 
Phone: 011‐26865455/+91‐9871145498 Hauz Khas, New Delhi‐16 
                                                   
                                             Website: www.physicsbyfiziks.com                                                                                          
                                                            Email: fiziks.physics@gmail.com                                                                   93 
fiziks
Institute for NET/JRF, GATE, IIT‐JAM, JEST, TIFR and GRE in PHYSICAL SCIENCES 
 
Ans. : (a)
cv dT aT + bT 3 b
Solution: ds = ∫ ⇒s=∫ dT = aT + T 3 + c
T T 3

IIT-JAM-2008
Q6. The chemical potential of an ideal Bose gas at any temperature is
(a) necessarily negative (b) either zero or negative
(c) necessarily positive (d) either zero or positive
Ans. : (b)
Solution: The chemical potential of an ideal Bose gas at any temperature is either zero or
negative.(zero in case of photon ).
Q7. A thermodynamic system is maintained at constant temperature and pressure. In
thermodynamic equilibrium, its
(a) Gibbs free energy is minimum (b) enthalpy is maximum
(c) Helmholtz free energy is minimum (d) internal energy is zero
Ans. : (a)
Solution: A thermodynamic system is maintained at constant temperature and pressure can be
defined by Gibbs energy dG = − SdT + VdP ≤ 0 i.e. Gibbs free energy is minimum.

IIT-JAM-2009
Q8. A box containing 2 moles of a diatomic ideal gas at temperature T0 is connected to
another identical box containing 2 moles of a monatomic ideal gas at temperature 5T0.
There are no thermal losses and the heat capacity of the boxes is negligible. Find the final
temperature of the mixture of gases (ignore the vibrational degrees of freedom for the
diatomic molecules).
(a) T0 (b) 1.5 T0 (c) 2.5 T0 (d) 3T0
Ans. : (c)
Solution: Internal energy of the system remains conserve i.e. U monoatomic + U diatomic = U mixture

                                                                                
Head office  Branch office 
 
fiziks, H.No. 40 D, G.F, Jia Sarai,  Anand Institute of Mathematics, 
 
Near IIT, Hauz Khas, New Delhi‐16  28‐B/6, Jia Sarai, Near IIT 
 
Phone: 011‐26865455/+91‐9871145498 Hauz Khas, New Delhi‐16 
                                                   
                                             Website: www.physicsbyfiziks.com                                                                                          
                                                            Email: fiziks.physics@gmail.com                                                                   94 
fiziks
Institute for NET/JRF, GATE, IIT‐JAM, JEST, TIFR and GRE in PHYSICAL SCIENCES 
 
U monotomic = n1CV1 T1 , U diatomic = n2CV2 T2

5R 3R
Cv1 = , Cv = , n1 = n2 = 2 , T1 = T0 , T2 = 5T0
2 2
2
n1CV1 + n2CV2
Let the common temperature of mixture is T and specific heat is CV = and
n1 + n2

number of moles of mixture is n = n1 + n2 , then

n1CV1T1 + n2CV2 T2 = nCV T ⇒ T = 2.5T0

1 ⎛ ∂V ⎞
Q9. Isothermal compressibility κ T of a substance is defined as κ T = ⎜ ⎟ . Its value for n
V ⎝ ∂P ⎠ T
moles of an ideal gas will be
1 n 1 n
(a) (b) (c) − (d) −
P P P P
Ans. : (c)
1 ⎛ ∂V ⎞ 1
Solution: PV = nRT and kT = ⎜ ⎟ =− .
V ⎝ ∂P ⎠T P
IIT-JAM-2010
Q10. A gas of molecules each having mass m is in thermal equilibrium at a temperature T. Let
G
vx, vy, vz be the Cartesian components of velocity, v , of a molecules. The mean value of
(v x − αv y + β v z ) is
2

(a) (1 + α 2 + β 2 ) (b) (1 − α 2 + β 2 )
k BT k BT
m m

(c) (β 2 − α 2 ) (d) (α 2 + β 2 )
k BT k BT
m m
Ans. : (a)

Solution: ( vx − α v y + β vz ) = vx2 + α 2 v y2 + β 2 v y2 − 2α vx v y + 2 β vz vx − 2βα v y vz


2

〈( vx − α v y + β vz ) 〉 = 〈 vx2 〉 + α 2 〈 v y2 〉 + β 2 〈 v y2 〉 − 2α 〈 vx 〉〈 v y 〉 + 2 β 〈 vz 〉〈 vx 〉 − 2βα 〈 v y 〉〈 vz 〉
2

                                                                                
Head office  Branch office 
 
fiziks, H.No. 40 D, G.F, Jia Sarai,  Anand Institute of Mathematics, 
 
Near IIT, Hauz Khas, New Delhi‐16  28‐B/6, Jia Sarai, Near IIT 
 
Phone: 011‐26865455/+91‐9871145498 Hauz Khas, New Delhi‐16 
                                                   
                                             Website: www.physicsbyfiziks.com                                                                                          
                                                            Email: fiziks.physics@gmail.com                                                                   95 
fiziks
Institute for NET/JRF, GATE, IIT‐JAM, JEST, TIFR and GRE in PHYSICAL SCIENCES 
 
k T
〈 vx2 〉 = 〈 v y2 〉 = 〈 vz2 〉 = B and 〈 vx 〉 = 〈 v y 〉 = 〈 vz 〉 = 0
m

(
〈( vx − α v y + β v z ) 〉 = 1 + α 2 + β 2 ) kmT
2
B

Q11. A trapped air bubble of volume V0 is released from a depth h measured from the water
surface in a large water tank. The volume of the bubble grows to 2 V0 as it reaches just
below the surface. The temperature of the water and the pressure above the surface of
water (105 N/m2) remain constant throughout the process. If the density of water is 1000
kg/ m3 and the acceleration due to gravity is 10m/ s2, then the depth h is
(a) 1 m (b) 10 m (c) 50 m (d) 100 m
Ans. : (b)
Solution: At depth h pressure P1 = P0 + ρ gh and volume V1 = V0 where

At surface pressure P2 = P0 and volume V2 = 2V0

( P0 + ρ gh )V0 = P0 2V0
P0 = 105 N / m 2 , g = 10m / s 2 ρ = 1000kg / m3
h = 10m
IIT-JAM-2011
Q12. Consider free expansion of one mole of an ideal gas in an adiabatic container from
volume V1 to V2 . The entropy change of the gas, calculated by considering a reversible

process between the original state (V1 , T ) to the final state (V2 , T ) where T is the

temperature of the system is denoted by ΔS1 . The corresponding change in the entropy of

the surrounding is ΔS 2 . Which of the following combinations is correct?

(a) ΔS1 = R ln (V1 / V2 ) , ΔS 2 = − R ln (V1 / V2 )

(b) ΔS1 = − R ln (V1 / V2 ) , ΔS 2 = R ln (V1 / V2 )

(c) ΔS1 = R ln (V2 / V1 ) , ΔS 2 = 0

(d) ΔS1 = − R ln (V2 / V1 ) , ΔS 2 = 0

                                                                                
Head office  Branch office 
 
fiziks, H.No. 40 D, G.F, Jia Sarai,  Anand Institute of Mathematics, 
 
Near IIT, Hauz Khas, New Delhi‐16  28‐B/6, Jia Sarai, Near IIT 
 
Phone: 011‐26865455/+91‐9871145498 Hauz Khas, New Delhi‐16 
                                                   
                                             Website: www.physicsbyfiziks.com                                                                                          
                                                            Email: fiziks.physics@gmail.com                                                                   96 
fiziks
Institute for NET/JRF, GATE, IIT‐JAM, JEST, TIFR and GRE in PHYSICAL SCIENCES 
 
Ans. : (c)
Solution: Free expansion is irreversible process when gas expand V1 to V2 which can be
explained by choosing any path between two state (because entropy is state function). So
one can choose reversible isothermal process.
V2
So ΔS 2 = R ln . Hence it is free expansion so entropy of surrounding is ΔS2 = 0 .
V1
Q13. A gas of molecular mass m is at temperature T. If the gas obeys Maxwell-Boltzmann
velocity distribution, the average speed of molecules is given by
k BT 2k B T 2k B T 8k B T
(a) (b) (c) (d)
m m πm πm

Ans. : (d)
IIT-JAM-2012
Q14. For a liquid to vapour phase transition at Ttr, which of the following plots between
specific Gibbs free energy g and temperature T is correct?
(a) g (b) g

Liquid Vapour
Liquid Vapour

T T
Ttr Ttr

(c) g (d) g

Liquid Vapour Liquid Vapour

T T
Ttr Ttr

Ans. : (a)
                                                                                
Head office  Branch office 
 
fiziks, H.No. 40 D, G.F, Jia Sarai,  Anand Institute of Mathematics, 
 
Near IIT, Hauz Khas, New Delhi‐16  28‐B/6, Jia Sarai, Near IIT 
 
Phone: 011‐26865455/+91‐9871145498 Hauz Khas, New Delhi‐16 
                                                   
                                             Website: www.physicsbyfiziks.com                                                                                          
                                                            Email: fiziks.physics@gmail.com                                                                   97 
fiziks
Institute for NET/JRF, GATE, IIT‐JAM, JEST, TIFR and GRE in PHYSICAL SCIENCES 
 
-11
Q15. A tiny dust particle of mass 1.4 ×10 kg is floating in air at 300K. Ignoring gravity, its
rms speed (in µm/s) due to random collisions with air molecules will be closest to
(a) 0.3 (b) 3 (c) 30 (d) 300
Ans. : (c)

3kT 3 × 1.38 × 10−23 × 300


Solution: vrms = = −11
= 30 × 10−6 m / s
m 1.4 × 10
Q16. When the temperature of a blackbody is doubled, the maximum value of its spectral
energy density, with respect to that at initial temperature, would become
1
(a) times (b) 8 times (c) 16 times (d) 32 times
16
U1 T14 T4
Solution: U ∝ T 4 ⇒ = 4 ⇒ U 2 = U1 24 = 64U1
U 2 T2 T1

IIT-JAM-2013
Q17. A blackbody at temperature T emits radiation at a peak wavelength λ . If the temperature
of the blackbody becomes 4T , the new peak wavelength is
1 1 1 1
(a) λ (b) λ (c) λ (d) λ
256 64 16 4
Ans. : (d)
λ1maxT1 λT λ
Solution: From wein Law λ1maxT1 = λ2maxT2 ⇒ λ2max = = =
T2 4T 4

Q18. Let N MB , N BE , N FD denote the number of ways in which two particles can be distributed
in two energy states according to Maxwell-Boltzmann, Bose-Einstein and Fermi-Dirac
statistics respectively. Then N MB : N BE : N FD is
(a) 4 : 3 : 1 (b) 4 : 2 : 3 (c) 4 : 3 : 3 (d) 4 : 3 : 2
Ans. : (a)
Solution: N = 2 g = 2 n = 2

                                                                                
Head office  Branch office 
 
fiziks, H.No. 40 D, G.F, Jia Sarai,  Anand Institute of Mathematics, 
 
Near IIT, Hauz Khas, New Delhi‐16  28‐B/6, Jia Sarai, Near IIT 
 
Phone: 011‐26865455/+91‐9871145498 Hauz Khas, New Delhi‐16 
                                                   
                                             Website: www.physicsbyfiziks.com                                                                                          
                                                            Email: fiziks.physics@gmail.com                                                                   98 
fiziks
Institute for NET/JRF, GATE, IIT‐JAM, JEST, TIFR and GRE in PHYSICAL SCIENCES 
 
N n 2 2
For Maxwell W = g = 2 =4
n 2
n + g −1 2 + 2 −1
For boson N = 2 g = 2 n = 2 ; W= = =3
n g −1 2 2 −1

g 2
For Fermion N = 2 g = 2 n = 2 ; W = = =1
g g −n 21

Q19. Two thermally isolated identical systems have heat capacities which vary as
C v = βT 3 (where β > 0 ). Initially one system is at 300 K and the other at 400 K. The
systems are then brought into thermal contact and the combined system is allowed to
reach thermal equilibrium. The final temperature of the combined system is………….
Ans. :
Solution: There is not a unique value of temperature rather range of temperature
The maximum temperature when work done is zero so dQ1 + dQ2 = 0

Tmax T4 ( 300 ) ( 400 )


Tmax 4 4
mβ ∫ T dT + mβ ∫ T dT = 0 ⇒ 2 max −
3 3
− =0
300 400 4 4 4

⇒ Tmax
2
=
( 300 )4 + ( 400 )4 ⇒ T = 360 K
max
2
The minimum temperature system is when process is reversible so change in entropy of
system is zero
ΔS1 + ΔS0 = 0

Tmin
mβ T 3 Tmin
dT + ∫
mβ T 3
dT = 0 ⇒ Tmin
2
=
( 300 ) + ( 400 ) ⇒ T = 353K 2 2

∫ min
300 T 400 T 2
So 353K ≤ T ≤ 360 K

                                                                                
Head office  Branch office 
 
fiziks, H.No. 40 D, G.F, Jia Sarai,  Anand Institute of Mathematics, 
 
Near IIT, Hauz Khas, New Delhi‐16  28‐B/6, Jia Sarai, Near IIT 
 
Phone: 011‐26865455/+91‐9871145498 Hauz Khas, New Delhi‐16 
                                                   
                                             Website: www.physicsbyfiziks.com                                                                                          
                                                            Email: fiziks.physics@gmail.com                                                                   99 
fiziks
Institute for NET/JRF, GATE, IIT‐JAM, JEST, TIFR and GRE in PHYSICAL SCIENCES 
 
IIT-JAM-2014
Q20. In 1- dimension, an ensemble of N classical particles has energy of the form
2
P 1
E = x + kx 2 . The average internal energy of the system at temperature T is
2m 2
3 1
(a) Nk B T (b) Nk B T (c) 3 Nk B T (d) Nk B T
2 2
Ans. : (d)
Q21. A solid metallic cube of heat capacity S is at temperature 300 K. It is brought in contact
with a reservoir at 600 K. If the heat transfer takes place only between the reservoir and
the cube, the entropy change of the universe after reaching the thermal equilibrium is
(a) 0.69 S (b) 0.54 S (c) 0.27 S (d) 0.19 S
Ans. : (d)
Q22. A real gas has specific volume v at temperature T. Its coefficient of volume expansion
and isothermal compressibility are α and k T , respectively. Its molar specific heat at
constant pressure C p and molar specific heat at constant volume C v are related as

Tvα
(a) C p = C v + R (b) C p = C v +
kT

Tvα 2
(c) C p = C v + (d) C p = C v
kT
Ans. : (c)
Q23. At atmospheric pressure (= 10 5 Pa ), aluminium melts at 550 K. As it melts, its density

decreases from 3 × 10 3 kg/m 3 to 2.9 × 10 3 kg/m 3 . Latent heat of fusion of aluminium is

24 × 103 J/kg . The melting point of aluminium at a pressure of 10 7 Pa is closest to


(a) 551.3 K (b) 552.6 K (c) 558.7 K (d) 547.4 K
Ans. : (b)

                                                                                
Head office  Branch office 
 
fiziks, H.No. 40 D, G.F, Jia Sarai,  Anand Institute of Mathematics, 
 
Near IIT, Hauz Khas, New Delhi‐16  28‐B/6, Jia Sarai, Near IIT 
 
Phone: 011‐26865455/+91‐9871145498 Hauz Khas, New Delhi‐16 
                                                   
                                             Website: www.physicsbyfiziks.com                                                                                          
                                                            Email: fiziks.physics@gmail.com                                                                   100 
fiziks
Institute for NET/JRF, GATE, IIT‐JAM, JEST, TIFR and GRE in PHYSICAL SCIENCES 
 
IIT-JAM-2015
Q24. A system consists of N number of particles, N >> 1 . Each particle can have only one of
the two energies E1 or E1 + ε (ε > 0 ) . If the system is in equilibrium at a temperature T ,
the average number of particles with energy E1 is
N N N
(a) (b) ε / kT
(c) − ε / kT
(d) Ne −ε / kT
2 e +1 e +1
Ans. : (d)
−( E2 − E1 ) −⎡⎣( E1 +ε ) − E1 ⎤⎦ −ε
Solution: N = Ne kT
=e kT
⇒ N = Ne kT

Q25. A rigid and thermally isolated tank is divided into two compartments of equal volume V ,
separated by a thin membrane. One compartment contains one mole of an ideal gas A
and the other compartment contains one mole of a different ideal gas B . The two gases
are in thermal equilibrium at a temperature T . If the membrane ruptures, the two gases
mix. Assume that the gases are chemically inert. The change in the total entropy of the
gases on mixing is
3
(a) 0 (b) R ln 2 (c) R ln 2 (d) 2 R ln 2
2
Ans. : (d)
Solution: For A , number of microstate after mixing is 2
For A , number of microstate before mixing is 1 A B
⇒ ΔS A = R ln 2 − R ln1 = R ln 2

Similarly, for B ⇒ ΔS B = R ln 2 ⇒ ΔS = ΔS A + ΔS B = 2 R ln 2
Q26. A rigid triangular molecule consists of three non-collinear atoms joined by rigid rods.
The constant pressure molar specific heat (C p ) of an ideal gas consisting of such

molecules is
(a) 6 R (b) 5 R (c) 4 R (d) 3R
Ans. : (c)
6 RT ⎛ ∂U ⎞
Solution: D.O.F = 6 ⇒ U = ⇒ CV = ⎜ ⎟ = 3R ⇒ CP = CV + R = 4 R
2 ⎝ ∂T ⎠V

                                                                                
Head office  Branch office 
 
fiziks, H.No. 40 D, G.F, Jia Sarai,  Anand Institute of Mathematics, 
 
Near IIT, Hauz Khas, New Delhi‐16  28‐B/6, Jia Sarai, Near IIT 
 
Phone: 011‐26865455/+91‐9871145498 Hauz Khas, New Delhi‐16 
                                                   
                                             Website: www.physicsbyfiziks.com                                                                                          
                                                            Email: fiziks.physics@gmail.com                                                                   101 
fiziks
Institute for NET/JRF, GATE, IIT‐JAM, JEST, TIFR and GRE in PHYSICAL SCIENCES 
 
SECTION–B: MSQ
Q27. As shown in the P − V diagram AB and CD are two isotherms at temperatures T1 and
T2 , respectively (T1 > T2 ) . AC and BD are two reversible adiabats. In this Carnot cycle,
which of the following statements are true?
P A Q1
Q Q
(a) 1 = 2
T1 T2 B
T1
(b) The entropy of the source decreases
(c) The entropy of the system increases C
Q2 D T2
(d) Work done by the system W = Q1 − Q2
V
Ans. : (a), (b) and (d)
Q28. In the thermodynamic cycle shown in the figure, one mole of a monatomic ideal gas is
taken through a cycle. AB is a reversible isothermal expansion at a temperature of 800 K
in which the volume of the gas is doubled. BC is an isobaric

Pressure
contraction to the original volume in which the temperature is
reduced to 300 K . CA is a constant volume process in which
A
P1
the pressure and temperature return to their initial values. The
C B
net amount of heat (in Joules) absorbed by the gas in one P2
complete cycle is……………. V 2V Volume
Ans. : 452
Solution: Process A → B is isothermal expansion
PA
TA = 800 K , VA , PA and TB = 800 K , VB = 2VA , PB =
2
Process B → C is isobaric
PA
PC = PB = , VC = VA , TC = 300 K
2
C → A is Isochoric
⎛V ⎞
ΔQ1 = nRTA ln ⎜ B ⎟ = 4602 J
⎝ VA ⎠

                                                                                
Head office  Branch office 
 
fiziks, H.No. 40 D, G.F, Jia Sarai,  Anand Institute of Mathematics, 
 
Near IIT, Hauz Khas, New Delhi‐16  28‐B/6, Jia Sarai, Near IIT 
 
Phone: 011‐26865455/+91‐9871145498 Hauz Khas, New Delhi‐16 
                                                   
                                             Website: www.physicsbyfiziks.com                                                                                          
                                                            Email: fiziks.physics@gmail.com                                                                   102 
fiziks
Institute for NET/JRF, GATE, IIT‐JAM, JEST, TIFR and GRE in PHYSICAL SCIENCES 
 
nγ RΔT ⎛ γ ⎞
ΔQ2 = nCP ΔT = = R ( 300 − 800) = −10344 J
(γ − 1) ⎜⎝ γ − 1⎟⎠
R R
ΔQ3 = (800 − 300) = × 500 = 6194 J
(γ − 1) (γ − 1)
Total heat exchange is Q1 + Q2 + Q3 = 452

Q29. One gram of ice at 0 o C is melted and heated to water at 39 o C . Assume that the specific
heat remains constant over the entire process. The latent heat of fusion of ice is
80 Calories/gm. The entropy change in the process (in Calories per degree) is………….
Ans. : 0.39
ML 1× 80 302 dT 302
Solution: ΔS1 = = , ΔS2 = MC ∫ ⇒ ΔS2 = 1.1ln
T 273 273 T 273
80 302
⇒ ΔS = ΔS1 + ΔS 2 ⇒ ΔS = + 1.1ln = 0.29 + 0.1 = 0.39
273 273
IIT-JAM-2016
Q30. A spherical closed container with smooth inner wall contains a monoatomic ideal gas. If
the collisions between the wall and the atoms are elastic, then the Maxwell speed-
⎛ dn ⎞
distribution function ⎜ v ⎟ for the atoms is best represented by:
⎝ dv ⎠
(a) (b)
dnv dnv
dv dv

0 v 0 v

(c) (d)
dnv dnv
dv dv

0 v 0 v
Ans. : (c)

                                                                                
Head office  Branch office 
 
fiziks, H.No. 40 D, G.F, Jia Sarai,  Anand Institute of Mathematics, 
 
Near IIT, Hauz Khas, New Delhi‐16  28‐B/6, Jia Sarai, Near IIT 
 
Phone: 011‐26865455/+91‐9871145498 Hauz Khas, New Delhi‐16 
                                                   
                                             Website: www.physicsbyfiziks.com                                                                                          
                                                            Email: fiziks.physics@gmail.com                                                                   103 
fiziks
Institute for NET/JRF, GATE, IIT‐JAM, JEST, TIFR and GRE in PHYSICAL SCIENCES 
 
dn ⎛ mv ⎞
2
Solution: ∝ v 2 exp ⎜ − ⎟
dv ⎝ 2kT ⎠
Q31. For an ideal gas, which one of the following T -S diagram is valid?
(a) (b)
T T
Isochor Isobar
Isobar Isochor
S S

(c) Isochor (d)


T T Isochor
Isobar Isobar

S S
Ans. : (a)
⎛ ∂S ⎞ ⎛ ∂S ⎞
Solution: CV = T ⎜ ⎟ CP = T ⎜ ⎟ CP > CV from the slope of T − S diagram one can plot
⎝ ∂T ⎠V ⎝ ∂T ⎠ P
isochoric and isobaric plot .
Q32. If U , F , H and G represent internal energy, Helmholtz free energy enthalpy and Gibbs
free energy respectively, then which one of the following is a correct thermodynamic
relation?
(a) dU = PdV − TdS (b) dH = VdP + TdS
(c) dF = − PdV + SdT (d) dG = VdP + SdT
Ans. : (b)
Solution: H = U + PV ⇒ dH = dU + PdV + VdP ⇒ TdS − PdV + PdV + VdP
dH = VdP + TdS

                                                                                
Head office  Branch office 
 
fiziks, H.No. 40 D, G.F, Jia Sarai,  Anand Institute of Mathematics, 
 
Near IIT, Hauz Khas, New Delhi‐16  28‐B/6, Jia Sarai, Near IIT 
 
Phone: 011‐26865455/+91‐9871145498 Hauz Khas, New Delhi‐16 
                                                   
                                             Website: www.physicsbyfiziks.com                                                                                          
                                                            Email: fiziks.physics@gmail.com                                                                   104 
fiziks
Institute for NET/JRF, GATE, IIT‐JAM, JEST, TIFR and GRE in PHYSICAL SCIENCES 
 
Q33. One mole of an ideal gas with average molecular speed v0 is kept in a container of fixed
volume. If the temperature of the gas is increased such that the average speed gets
doubled, then
(a) the mean free path of the gas molecule will increase
(b) the mean free path of the gas molecule will not change
(c) the mean free path of the gas molecule will decrease
(d) the collision frequency of the gas molecule with wall of the container remains
unchanged.
Ans. : (b)
Solution: For fixed volume if temperature is increased then pressure is also increased by same
amount so mean free path with not change
kT
λ=
2π d 2 p
Q34. The P − V diagram below shows three possible paths for an ideal
gas to reach the final state f from an initial state i . Which among i
P 3
the following statements ( s ) is (are) correct? 2
1 f
(a) The work done by the gas is maximum along path - 3 .
V
(b) Minimum change in the internal energy occurs along path - 2 .
(c) Maximum heat transfer is for path - 1
(d) Heat transfer is path independent
Ans. : (a)
Solution: work done is area under curve so it is maximum in path 3 Hence change in internal
energy is same in all path so heat exchange is also maximum in path 3
Q35. A cylinder contains 16 g of O2 . The work done when the gas is compressed to 75% of

the original volume at constant temperature of 270 C is………………. J .


[Universal gas constant R = 8.31 J / ( mole K ) ]

16
n= = 0.5, T = 300k
32
                                                                                
Head office  Branch office 
 
fiziks, H.No. 40 D, G.F, Jia Sarai,  Anand Institute of Mathematics, 
 
Near IIT, Hauz Khas, New Delhi‐16  28‐B/6, Jia Sarai, Near IIT 
 
Phone: 011‐26865455/+91‐9871145498 Hauz Khas, New Delhi‐16 
                                                   
                                             Website: www.physicsbyfiziks.com                                                                                          
                                                            Email: fiziks.physics@gmail.com                                                                   105 
fiziks
Institute for NET/JRF, GATE, IIT‐JAM, JEST, TIFR and GRE in PHYSICAL SCIENCES 
 
Ans. : 358
.75V dV 75 1
Solution: W = PdV ⇒ nRT ∫ = nRT ln = − × 8.31× 300 × ln .75 = 358 J
V V 100 2

Q36. An aluminum plate of mass 0.1 kg at 950 C is immersed in 0.5 litre of water at 200 C
kept inside an insulating container and is then removed. If the temperature of the water is
found to be 230 C , then the temperature of the aluminum plate is………… 0 C
(The specific heat of water and aluminum are 4200 J / kg − K and 900 J / kg − K

respectively, the density of water is 1000 kg / m3 )


Ans. : 94.36

( ) ( )
Solution: − M a Sa Ta f − Tai = M w Sw Tw f − Twi ⇒ −0.1× 4200 (Taf − 368 ) = 0.5 × 900 ( 296 − 293)

( )
−2100 Taf − 368 = 450 × 3 ⇒ Taf − 368 = ( ) 450
700
= −0.64

⇒ Taf = 368 − .64 = 367.36

⇒ 367.36 − 273 = 94.36


Q37. If there is a 10% decrease in the atmospheric pressure at a hill compared to the pressure
at sea level, then the change in the boiling point of water is………………. 0 C
(Take latent heat of vaporisation of water as 2270 kJ / kg and the change in the specific

volume at the boiling point to be 1.2 m3 / kg )


Ans. : 2
dP 1 L v2 − v1 0.1× 1.01× 105 × 373 × 1.2
Solution: = ⇒ dT = dP × T × = = 0.02 × 105 = 20 C
dT T v2 − v1 L 2270 × 10 3

                                                                                
Head office  Branch office 
 
fiziks, H.No. 40 D, G.F, Jia Sarai,  Anand Institute of Mathematics, 
 
Near IIT, Hauz Khas, New Delhi‐16  28‐B/6, Jia Sarai, Near IIT 
 
Phone: 011‐26865455/+91‐9871145498 Hauz Khas, New Delhi‐16 
                                                   
                                             Website: www.physicsbyfiziks.com                                                                                          
                                                            Email: fiziks.physics@gmail.com                                                                   106 
fiziks
Institute for NET/JRF, GATE, IIT‐JAM, JEST, TIFR and GRE in PHYSICAL SCIENCES 
 
Modern Physics
IIT-JAM-2005
Q1. If M e , M p and M H and are the rest masses of electron, proton and hydrogen atom in

the ground state (with energy -13.6 eV), respectively, which of the following is exactly
true? (c is the speed of light in free space)
(a) M H = M p + M e

13.6 eV
(b) M H = M p + M e −
c2
13.6 eV
(c) M H = M p + M e +
c2
13.6 eV
(d) M H = M p + M e + K , where K ≠ ± or zero
c2
Ans. : (c)

Solution: B.E. = (M p + M e − M H )c 2 ⇒ M H = M p + M e −
B.E.
where B.E. = −13.6eV .
c2
IIT-JAM-2006
Q2. Electrons of energy E coming in from x = -∞ impinge upon a potential barrier of width 2a
and height V0 centered at the origin with V0 > E , as shown in the figure below.

2m(V0 − E )
Let k = . In the region –a ≤ x ≤ a, the electrons is a linear combination of
V

V0

−a a x

(a) e kx and e − kx (b) e ikx and e − kx (c) e ikx and e −ikx (d) e ikx and e kx
Ans. : (a)
                                                                                
Head office  Branch office 
 
fiziks, H.No. 40 D, G.F, Jia Sarai,  Anand Institute of Mathematics, 
 
Near IIT, Hauz Khas, New Delhi‐16  28‐B/6, Jia Sarai, Near IIT 
 
Phone: 011‐26865455/+91‐9871145498 Hauz Khas, New Delhi‐16 
                                                   
                                             Website: www.physicsbyfiziks.com                                                                                          
                                                            Email: fiziks.physics@gmail.com                                                                   107 
fiziks
Institute for NET/JRF, GATE, IIT‐JAM, JEST, TIFR and GRE in PHYSICAL SCIENCES 
 
Solution: Since V0 > E in region -a ≤ x ≤ a Schrodinger equation is given by

∂ 2ψ 2
∂ 2ψ 2m(Vo − E ) ∂ 2ψ
− + Voψ = Eψ ⇒ − ψ = 0 ⇒ − k 2ψ = 0
2m ∂x 2
∂x 2 2
∂x 2

2m(V0 − E )
where k = .

Thus solution e kx and e − kx .which is exponential in nature .


Q3. The relation between angular frequency ω and wave number k for given type of waves is
ω2 = α k +β k3. The wave number k 0 for which the phase velocity equals the group
velocity is
α ⎛1⎞ α α ⎛1⎞ α
(a) 3 (b) ⎜ ⎟ (c) (d) ⎜ ⎟
β ⎝3⎠ β β ⎝2⎠ β
Ans. : (c)
dω ω
Solution: Group velocity Vg = and phase velocity is V p =
dk k
ω 2 = α k +β k 3 ………(A)

Differentiating both side we get 2ω. = α + 3β k 2
dk
Now dividing both side by k we will get
ω dω α α
2 . = + 3β k ⇒ 2V p .Vg = + 3β k
k dk k k
For k = k0 and V p = Vg
1
α⎛ α 3β k0 ⎞ 2
2V = + 3β k0 ⇒ V p = ⎜
2
p + ⎟
k0 ⎝ 2 k0 2 ⎠
1
⎛α ⎞ ω 2
From equation (A) V p = = ⎜ + β k0 ⎟
k ⎝ k0 ⎠
1 1
⎛ α 3β k 0 ⎞ 2
⎛α ⎞ 2
α β k0 α
Thus ⎜ + ⎟ = ⎜ + β k0 ⎟ ⇒ − = 0 ⇒ k0 =
⎝ 2k0 2 ⎠ ⎝ k0 ⎠ 2 k0 2 β

                                                                                
Head office  Branch office 
 
fiziks, H.No. 40 D, G.F, Jia Sarai,  Anand Institute of Mathematics, 
 
Near IIT, Hauz Khas, New Delhi‐16  28‐B/6, Jia Sarai, Near IIT 
 
Phone: 011‐26865455/+91‐9871145498 Hauz Khas, New Delhi‐16 
                                                   
                                             Website: www.physicsbyfiziks.com                                                                                          
                                                            Email: fiziks.physics@gmail.com                                                                   108 
fiziks
Institute for NET/JRF, GATE, IIT‐JAM, JEST, TIFR and GRE in PHYSICAL SCIENCES 
 
Q4. A particle of rest mass m0 is moving uniformly in a straight line with relativistic velocity
βc, where c is the velocity of light in vacuum and 0<β<1. The phase velocity of the de
Broglie wave associated with the particle is
c c
(a) βc (b) (c) c (d)
β β2
Ans. : (b)
Solution: E 2 = p 2 c 2 + m02 c 4

dE E c2 c2 c
2E = 2 pc 2 ⇒ E.vg = pc 2 ⇒ = = vp ⇒ vp = ⇒
dp p vg βc β

Q5. A neutron of mass mn = 10-27 kg is moving inside a nucleus to be a cubical box of size10-
14
m with impenetrable walls. Take ћ ≈ 10-14 Js and 1 MeV ≈ 10-13 J. An estimate of the
energy in MeV of the neutron is
1 1
(a) 80 MeV (b) MeV (c) 8 MeV (d) MeV
8 80
Ans:

( )
2
3π 2 2 3 ×10 × 10−14 3 ×10 × 10−28
Solution: E = = = = 15 × 1027 J
( )
−27 −28
2mn a 2 2 × 10−27 × 10−14 2
2 × 10 ×10

IIT-JAM-2007
Q6. The following histogram represents the binding energy per particle (B.E./A) in MeV as a
function of the mass number A of a
nucleus. A nucleus with mass number
8
A=180 fissions into two nuclei of
6
equal masses. In the process B.E.
A 4
(a) 180 MeV of energy is released
(b) 180 MeV of energy is absorbed 2
(c) 360 MeV of energy is released
(d) 360 MeV of energy is absorbed 40 80 120 160 200
A
                                                                                
Head office  Branch office 
 
fiziks, H.No. 40 D, G.F, Jia Sarai,  Anand Institute of Mathematics, 
 
Near IIT, Hauz Khas, New Delhi‐16  28‐B/6, Jia Sarai, Near IIT 
 
Phone: 011‐26865455/+91‐9871145498 Hauz Khas, New Delhi‐16 
                                                   
                                             Website: www.physicsbyfiziks.com                                                                                          
                                                            Email: fiziks.physics@gmail.com                                                                   109 
fiziks
Institute for NET/JRF, GATE, IIT‐JAM, JEST, TIFR and GRE in PHYSICAL SCIENCES 
 
Ans. : (c)
A A
Solution: A → + or 180 → 90 + 90
2 2
Product B.E = 90 × 6 + 90 × 6 = 1080 MeV
B.E. of nucleus A = 180 × 4 = 720 MeV
Since B.E of the product nucleus is greater than the nucleus A hence in this process
energy is released and that is = (1080 − 720 )MeV = 360 MeV .
Q7. The black body spectrum of an object O1 is such that its radiant intensity (i.e., intensity
per unit wavelength interval) is maximum at a wavelength of 200 nm. Another object O2
has the maximum radiant intensity at 600 nm. The ratio of power emitted per unit area by
O1 to that of O2 is
1 1
(a) (b) (c) 9 (d) 81
81 9
Ans. : (d)
λ1 T2 T1
Solution: From Wein’s law λT = k where k is constant thus = =3
λ2 T1 T2

P1 T14
Power ( P ) is proportional to T 4 ⇒ = = 81
P2 T24
Q8. A particle is confined in a one dimensional box with impenetrable walls at x = ±a. Its
energy eigenvalue is 2 eV and the corresponding eigenfunction is as shown below.

−a
0 +a
The lowest possible energy of the particle is
(a) 4 eV (b) 2 eV (c) 1 eV (d) 0.5 eV
Ans. : (d)
Solution: The given state is representation of first exited state whose energy is is 2 eV.
If En is energy of nth state and E0 is energy of ground state then En = n 2 E0 .

So E2 = 4 E0 = 2eV ⇒ E0 = 0.5eV

                                                                                
Head office  Branch office 
 
fiziks, H.No. 40 D, G.F, Jia Sarai,  Anand Institute of Mathematics, 
 
Near IIT, Hauz Khas, New Delhi‐16  28‐B/6, Jia Sarai, Near IIT 
 
Phone: 011‐26865455/+91‐9871145498 Hauz Khas, New Delhi‐16 
                                                   
                                             Website: www.physicsbyfiziks.com                                                                                          
                                                            Email: fiziks.physics@gmail.com                                                                   110 
fiziks
Institute for NET/JRF, GATE, IIT‐JAM, JEST, TIFR and GRE in PHYSICAL SCIENCES 
 
IIT-JAM-2008
Q9. A photon of wavelength λ is incident on a free electron at rest and is scattered in the
backward direction. The functional shift in its wavelength in terms of the Compton
wavelength λc of the electron is
λC 2λ C 3λC 2λ C
(a) (b) (c) (d)
2λ 3λ 2λ λ
Ans. : (d)
Solution: Δλ = λc (1 − cos θ )

When photon scatter in backward direction then θ = π so Δλ = 2λc

Δλ 2λC
Functional shift is =
λ λ
Q10. In an intertial frame S, a stationary rod makes an angle θ with the x-axis. Another interial
frame S’ moves with a velocity v with respect to S along the common x-x’ axis. As
observed from S’ the angle made by the rod with the x’-axis is θ’. Which of the following
statement is correct?
(a) θ’ < θ
(b) θ’ > θ
(c) θ’ < θ if v is negative and θ’ > θ if v is positive
(d) θ’ > θ if v is negative and θ’ < θ if v is positive
Ans. : (b)

v2
Solution: lx = l0 cos θ 1 − , l y = l0 sin θ
c2
ly tan θ
tan θ ' = = ⇒ θ' >θ
lx v2
1−
c2
Q11. The activity of a radioactive sample is decreased to 75% of the initial value after 30 days.
The half-life (in days) of the sample is approximately
[You may use ln3 ≈ 1.1, ln4 ≈1.4]
(a) 38 (b) 45 (c) 59 (d) 69
                                                                                
Head office  Branch office 
 
fiziks, H.No. 40 D, G.F, Jia Sarai,  Anand Institute of Mathematics, 
 
Near IIT, Hauz Khas, New Delhi‐16  28‐B/6, Jia Sarai, Near IIT 
 
Phone: 011‐26865455/+91‐9871145498 Hauz Khas, New Delhi‐16 
                                                   
                                             Website: www.physicsbyfiziks.com                                                                                          
                                                            Email: fiziks.physics@gmail.com                                                                   111 
fiziks
Institute for NET/JRF, GATE, IIT‐JAM, JEST, TIFR and GRE in PHYSICAL SCIENCES 
 
Ans. : (d)

1 ⎛ R ⎞ 1 ⎛ R0 ⎞ 1 ⎛4⎞ 1
Solution: λ = ln⎜ 0 ⎟ = ln⎜ ⎟⎟ = ln⎜ ⎟ = (1.4 − 1.1) = 1
t ⎝ R ⎠ 30 ⎜⎝ 3 / 4 R0 ⎠ 30 ⎝ 3 ⎠ 30 100

0.693 0.693
T1/ 2 = = = 69.3 day.
λ 1/100
IIT-JAM-2009
Q12. A wave packet in a certain medium is constructed by superposing waves of frequency ω
around ω0 = 100 and the corresponding wave-number k with k0 = 10 as given in the table
below
ω k
81.00 9.0
90.25 9.5
100.00 10.0
110.25 10.5
121.00 11.0
Find the ratio vε/vp of the group velocity vε and the phase velocity vp.
1 3
(a) (b) 1 (c) (d) 2
2 2
Ans. : (d)
ω0
Solution: For ω = ω0 = 100 and k = k0 = 10 the phase velocity is v p = =10
k0
Δω ω2 − ω1 110.25 − 90.25
The group velocity is vg = = = = 20
Δk k2 − k1 10.5 − 9.5

vg 20
= =2
vp 10

Q13. Two spherical nuclei have mass numbers 216 and 64 with their radii R1 and R2 ,

R1
respectively. The ratio is
R2
(a) 1.0 (b) 1.5 (c) 2.0 (d) 2.5
                                                                                
Head office  Branch office 
 
fiziks, H.No. 40 D, G.F, Jia Sarai,  Anand Institute of Mathematics, 
 
Near IIT, Hauz Khas, New Delhi‐16  28‐B/6, Jia Sarai, Near IIT 
 
Phone: 011‐26865455/+91‐9871145498 Hauz Khas, New Delhi‐16 
                                                   
                                             Website: www.physicsbyfiziks.com                                                                                          
                                                            Email: fiziks.physics@gmail.com                                                                   112 
fiziks
Institute for NET/JRF, GATE, IIT‐JAM, JEST, TIFR and GRE in PHYSICAL SCIENCES 
 
Ans. : (d)
1/ 3 1/ 3
R ⎛A ⎞ ⎛ 216 ⎞ 6
Solution: 1 = ⎜⎜ 1 ⎟⎟ =⎜ ⎟ = = 1.5
R2 ⎝ A2 ⎠ ⎝ 64 ⎠ 4

IIT-JAM-2010
Q14. A particle of mass m is confined in a two-dimensional infinite square well potential of
25π 2 2
side a. The eigen-energy of the particle in a given state is E = . The state is
ma 2
(a) 4-fold degenerate (b) 3-fold degenerate
(c) 2-fold degenerate (d) Non-degenerate
Ans. : (d)
Solution: The eigen-energy of the particle in a given state is given by
π2 2
E= 2
(nx2 + n y2 ) where nx = 1, 2,3... n y = 1, 2, 3...
2ma
25π 2 2
E= can be obtained by nx = 5 and n y = 5 which is non degenerate .
ma 2
Q15. For a wave in a medium the angular frequency ω and the wave vector k are related by,
ω 2 = (ω 02 + c 2 k 2 ) where ω 0 and c are constants. The product of group and phase
velocities, i.e v g .v p is

(a) 0.25 c2 (b) 0.4 c2 (c) 0.5 c2 (d) c2


Ans. : (d)
Solution: ω 2 = (ω 02 + c 2 k 2 )

dω ω dω
2ω. = 2c 2 k ⇒ . = c 2 ⇒ v p .vg = c 2
dk k dk

                                                                                
Head office  Branch office 
 
fiziks, H.No. 40 D, G.F, Jia Sarai,  Anand Institute of Mathematics, 
 
Near IIT, Hauz Khas, New Delhi‐16  28‐B/6, Jia Sarai, Near IIT 
 
Phone: 011‐26865455/+91‐9871145498 Hauz Khas, New Delhi‐16 
                                                   
                                             Website: www.physicsbyfiziks.com                                                                                          
                                                            Email: fiziks.physics@gmail.com                                                                   113 
fiziks
Institute for NET/JRF, GATE, IIT‐JAM, JEST, TIFR and GRE in PHYSICAL SCIENCES 
 
Q16. Three identical non-interacting particles, each of spin ½ and mass m, are moving in a
one-dimensional infinite potential well given by,

V ( x ) = ⎧⎨0 for 0 < x < a


⎩∞ for x ≤ 0 and x ≥ a
The energy of the lowest energy state of the system is
π2 2
2π 2 2 3π 2 2 5π 2 2
(a) (b) (c) (d)
ma 2 ma 2 ma 2 2ma 2
Ans. : (c)
1
Solution: Spin s = means particles are fermions and it will obey Pauli Exclusion Principle.
2
Degeneracy g = 2s + 1 ⇒ g = 2 means in every state maximum 2 identical particle can be
adjusted. If we have three fermions, then in ground state two fermions will adjust and in
next higher level one fermion will adjusted. Thus the energy of the lowest energy state of
π2
4π 2 2 6π 2 2 3π 2 2
2
the system is 2 × + = =
2ma 2 2ma 2 2ma 2 ma 2

IIT-JAM-2011
Q17. The wave function of a quantum mechanical particle is given by
3 4
ψ ( x ) = ϕ1 ( x ) + ϕ 2 ( x )
5 5

where φ1 ( x ) and φ2 ( x ) are eigenfunctions with corresponding energy eigenvalues

−1eV and −2 eV , respectively. The energy of the particle in the state ψ is


−41 −11 36 −7
(a) eV (b) eV (c) eV (d) eV
25 5 25 5

Ans. : (a)
ψ Hψ 9 16 −41
Solution: 〈 E 〉 = = −1eV × + −2eV × = eV
ψψ 25 25 25

                                                                                
Head office  Branch office 
 
fiziks, H.No. 40 D, G.F, Jia Sarai,  Anand Institute of Mathematics, 
 
Near IIT, Hauz Khas, New Delhi‐16  28‐B/6, Jia Sarai, Near IIT 
 
Phone: 011‐26865455/+91‐9871145498 Hauz Khas, New Delhi‐16 
                                                   
                                             Website: www.physicsbyfiziks.com                                                                                          
                                                            Email: fiziks.physics@gmail.com                                                                   114 
fiziks
Institute for NET/JRF, GATE, IIT‐JAM, JEST, TIFR and GRE in PHYSICAL SCIENCES 
 
Q18. Light described by the equation E = (90 V/m) [sin (6.28 x 1015 s-1) t + sin (12.56 x 1015 s-
1
) t] is incident on a metal surface. The work function of the metal is 2.0 eV. Maximum
kinetic energy of the photoelectrons will be
(a) 2.14 eV (b) 4.28 eV (c) 6.28 eV (d) 12.56 eV
Ans. : (c)
Solution: K max = ω − W

For given wave maximum kinetic energy is for highest ω so ω = 12.56 × 1015 sec −1

6.6 × 10−34 J s× 12.56 × 1015 s −1 82.8 × 10−19 J


ω= = eV = 8.24eV
2π 6.28 × 1.6 × 10−19
K max = ω − W ⇒ 8.24eV − 2eV = 6.24eV

IIT-JAM-2012
Q19. Light takes 4 hours to cover the distance from Sun to Neptune. If you travel in a
spaceship at a speed 0.99c (where c is the speed of light in vacuum), the time (in minutes)
required to cover the same distance measured with a clock on the spaceship will be
approximately
(a) 34 (b) 56 (c) 85 (d) 144
Ans. : (a)

Solution: l = ct0
v2
1 − 2 = c × 4 × 60 × 60 1 −
( 0.99c ) = c × 4 × 60 × 60 × .14m
2

c c2
c × 4 × 60 × 60 × .14
t= min = 33.9 = 34 min
.99c × 60
Q20. 60
27 Co is a radioactive nucleus of half-life 2 ln 2 × 10 8 s . The activity of 10g of 60
27 Co in
disintegrations per second is
1 1
(a) × 1010 (b) 5 × 1010 (c) × 1014 (d) 5 × 1014
5 5
Ans. : (d)

                                                                                
Head office  Branch office 
 
fiziks, H.No. 40 D, G.F, Jia Sarai,  Anand Institute of Mathematics, 
 
Near IIT, Hauz Khas, New Delhi‐16  28‐B/6, Jia Sarai, Near IIT 
 
Phone: 011‐26865455/+91‐9871145498 Hauz Khas, New Delhi‐16 
                                                   
                                             Website: www.physicsbyfiziks.com                                                                                          
                                                            Email: fiziks.physics@gmail.com                                                                   115 
fiziks
Institute for NET/JRF, GATE, IIT‐JAM, JEST, TIFR and GRE in PHYSICAL SCIENCES 
 
Solution: R = λN where N =
10
60
× 5 × 10 23 ( )

0.693 0.693 0.693 0.693 0.693


λ= = = = = = 5 × 10 −9 s −1
T1 / 2 2 ln 2 × 10 8
2 × 2.303 × 0.3010 × 10 8
1.386 × 10 8
1.386 × 10 8

10
(
Thus R = 5 × 10 −9 × × 5 × 10 23 = 4.2 × 1014 .
60
)

IIT-JAM-2013
Q21. Electric field component of an electromagnetic radiation varies with time
as E = a(cos ω 0 t + sin ωt cos ω 0 t ) , where a is a constant and the values of ω and ω 0 are

1 × 1015 s −1 and 5 × 1015 s −1 respectively. This radiation falls on a metal of work


function 2eV . The maximum kinetic energy (in eV ) of photoelectrons is
(a) 0.64 (b) 1.30 (c) 1.70 (d) 1.95
Ans. : (b)
Solution: K max = ω − W

For given wave maximum kinetic energy is for highest ω so ω0 = 5 × 1015 sec −1

6.6 × 10−34 J s× 5 × 1015 s −1 33 × 10−19 J


ω= = eV = 3.28
2π 6.28 ×1.6 × 10−19
K max = ω − W ⇒ 3.28eV − 2eV = 1.28eV

Q22. A free particle of mass m is confined to a region of length L . The de Broglie wave
associated with the particle is sinusoidal in nature as given in the figure. The energy of
the particle is

0
L/3 2L / 3 L

Ans. :
Solution: If wavelength of standing wave is λ and length of wall is L then from the figure

                                                                                
Head office  Branch office 
 
fiziks, H.No. 40 D, G.F, Jia Sarai,  Anand Institute of Mathematics, 
 
Near IIT, Hauz Khas, New Delhi‐16  28‐B/6, Jia Sarai, Near IIT 
 
Phone: 011‐26865455/+91‐9871145498 Hauz Khas, New Delhi‐16 
                                                   
                                             Website: www.physicsbyfiziks.com                                                                                          
                                                            Email: fiziks.physics@gmail.com                                                                   116 
fiziks
Institute for NET/JRF, GATE, IIT‐JAM, JEST, TIFR and GRE in PHYSICAL SCIENCES 
 
3λ 2L
=L ⇒λ = .
2 3
h
If p is momentum and λ is wavelength then from de-Broglie hypothesis p = thus
λ
3h
p= .
2L
When particle is confined into a box then total energy is only kinetic energy which is
p2 3h 9h 2
given by E = put the value of p = one will get E = .
2m 2L 8mL2

IIT-JAM-2014
Q23. In a photoelectric effect experiment, ultraviolet light of wavelength 320 nm falls on the
photocathode with work function of 2.1 eV. The stopping potential should be close to
(a) 1.8 V (b) 1.6 V (c) 2.2 V (d) 2.4V
Ans. : (a)
Q24. Four particles of mass m each are inside a two dimensional square box of side L. If each
state obtained from the solution of the Schrodinger equation is occupied by only one
h2
particle, the minimum energy of the system in units of is
mL2
5 11 25
(a) 2 (b) (c) (d)
2 2 4
Ans. : (b)
IIT-JAM-2015
Q25. A particle with energy E is incident on a potential given by
⎧ 0, x<0
V ( x) = ⎨ .
⎩ V0 , x≥0

The wave function of the particle for E < V0 in the region x > 0 (in terms of positive

constants A, B and k ) is

(a) Ae kx + Be − kx (b) Ae − kx (c) Ae ikx + Be − ikx (d) Zero

                                                                                
Head office  Branch office 
 
fiziks, H.No. 40 D, G.F, Jia Sarai,  Anand Institute of Mathematics, 
 
Near IIT, Hauz Khas, New Delhi‐16  28‐B/6, Jia Sarai, Near IIT 
 
Phone: 011‐26865455/+91‐9871145498 Hauz Khas, New Delhi‐16 
                                                   
                                             Website: www.physicsbyfiziks.com                                                                                          
                                                            Email: fiziks.physics@gmail.com                                                                   117 
fiziks
Institute for NET/JRF, GATE, IIT‐JAM, JEST, TIFR and GRE in PHYSICAL SCIENCES 
 
Ans. : (b)
d 2ψ ΙΙ
2
Solution: For x > 0 ; − + V0ψ ΙΙ = Eψ ΙΙ ; E < V0
2m d

2m (V0 − E )
ψ ΙΙ = Bekx + Ae − kx where k = 2

ψ ΙΙ → 0 as x → ∞ ⇒ A = 0 ⇒ ψ ΙΙ = Ae − kx

Q26. A system comprises of three electrons. There are three single particle energy levels
accessible to each of these electrons. The number of possible configurations for this
system is
(a) 1 (b) 3 (c) 6 (d) 7
Ans. : (d)
1
Solution: For electron spin is . So in one single state two electrons can be adjusted the number
2
of ways are
Ground First Second
1 2 1 0
2 2 0 1
3 1 2 0
4 1 0 2
5 0 1 2
6 0 2 1
7 1 1 1
So, number of ways are 7.

                                                                                
Head office  Branch office 
 
fiziks, H.No. 40 D, G.F, Jia Sarai,  Anand Institute of Mathematics, 
 
Near IIT, Hauz Khas, New Delhi‐16  28‐B/6, Jia Sarai, Near IIT 
 
Phone: 011‐26865455/+91‐9871145498 Hauz Khas, New Delhi‐16 
                                                   
                                             Website: www.physicsbyfiziks.com                                                                                          
                                                            Email: fiziks.physics@gmail.com                                                                   118 
fiziks
Institute for NET/JRF, GATE, IIT‐JAM, JEST, TIFR and GRE in PHYSICAL SCIENCES 
 
Q27. The variation of binding energy per nucleon with respect to the mass number of nuclei is
shown in the figure.
9
8

Average binding energy per


7

nucleon (MeV)
6
5
4
3
2
1
0
20 40 60 80 100 120 140 160 180 200 220 240
Number of nucleons in nucleus, A
Consider the following reactions:
(i) 238
92 U →82
206
Pb + 10 P + 22n (ii) 238
92 U→ Pb + 8 24He + 6e −
206
82

238
Which one of the following statements is true for the given decay modes of 92 U?
(a) Both (i) and (ii) are allowed (b) Both (i) and (ii) are forbidden
(c) (i) is forbidden and (ii) is allowed (d) (i) is allowed and (ii) is forbidden
Ans. : (c)
Q28. A nucleus has a size of 10 −15 m . Consider an electron bound within a nucleus. The
estimated energy of this electron is of the order of
(a) 1 MeV (b) 10 2 MeV (c) 10 4 MeV (d) 10 6 MeV
Ans. : (d)
h 6.6 ×10−34
Solution: p = = = 6.6 × 10−19 kgm / sec
λ 10−15
p2 44 × 10−38
∵E = = = 2.4 × 10−7 Joule
2me 2 × 9.1× 10−31

2.4 × 10−7
⇒E= −19
eV = 1.5 × 1012 eV = 1.5 ×106 MeV
1.6 ×10

                                                                                
Head office  Branch office 
 
fiziks, H.No. 40 D, G.F, Jia Sarai,  Anand Institute of Mathematics, 
 
Near IIT, Hauz Khas, New Delhi‐16  28‐B/6, Jia Sarai, Near IIT 
 
Phone: 011‐26865455/+91‐9871145498 Hauz Khas, New Delhi‐16 
                                                   
                                             Website: www.physicsbyfiziks.com                                                                                          
                                                            Email: fiziks.physics@gmail.com                                                                   119 
fiziks
Institute for NET/JRF, GATE, IIT‐JAM, JEST, TIFR and GRE in PHYSICAL SCIENCES 
 
SECTION–B: MSQ
Q29. A particle is moving in a two dimensional potential well
V ( x, y ) = 0, 0 ≤ x ≤ L, 0 ≤ y ≤ 2 L
= ∞, elsewhere

which of the following statements about the ground state energy E1 and ground state
eigenfunction ϕ 0 are true?
2
π2 5 2π 2
(a) E1 = (b) E1 =
mL2 8mL2
2 πx πy 2 πx πy
(c) ϕ0 = sin sin (d) ϕ 0 = cos cos
L L 2L L L 2L
Ans. : (b) and (c)

π2 ⎛ nx2 n y2 ⎞
2
Solution: En = ⎜ + ⎟
2m ⎜⎝ L2 4 L2 ⎟⎠

⎛ 1 1 ⎞ 5π 2 2 π2 2
Ground state nx = 1, n y = 1 ⇒ Ex = ⎜ + ⎟=
2m ⎝ L2 4 L2 ⎠ 8mL2

2 2 sin π x sin π y
Wave function ψ = ⋅ ⋅
L 2L L 2L
SECTION–C: NAT (Numerical Answer Type)
Q30. A particle is in a state which is a superposition of the ground state ϕ 0 and the first

excited state ϕ1 of a one-dimensional quantum harmonic oscillator. The state is given by


1 2
Φ= ϕ0 + ϕ1 . The expectation value of the energy of the particle in this state (in
5 5
units of ω , ω being the frequency of the oscillator) is…………
Ans. : 1.3
⎛ 1⎞ ⎛ ω⎞ 1 ⎛ 3 ω⎞ 4
Solution: ∵ En = ⎜ n + ⎟ ω and P ⎜ ⎟ = , P ⎜⎝ 2 ⎟⎠ = 5
⎝ 2⎠ ⎝ 2 ⎠ 5
ω 1 3 ω 4 13 ω
⇒ E = × + × = = 1.3 ω
2 5 2 5 10
                                                                                
Head office  Branch office 
 
fiziks, H.No. 40 D, G.F, Jia Sarai,  Anand Institute of Mathematics, 
 
Near IIT, Hauz Khas, New Delhi‐16  28‐B/6, Jia Sarai, Near IIT 
 
Phone: 011‐26865455/+91‐9871145498 Hauz Khas, New Delhi‐16 
                                                   
                                             Website: www.physicsbyfiziks.com                                                                                          
                                                            Email: fiziks.physics@gmail.com                                                                   120 
fiziks
Institute for NET/JRF, GATE, IIT‐JAM, JEST, TIFR and GRE in PHYSICAL SCIENCES 
 
Q31. In the hydrogen atom spectrum. the ratio of the longest wavelength in the Lyman series
(final state n = 1 ) to that in the Balmer series (final State n = 2 ) is…………..
Ans. : 0.185

1 ⎛ 1 1⎞
Solution: According to Bohr Theory = R⎜ 2 − 2 ⎟
λL ⎝ n f ni ⎠ n=3

The longest wavelength in the Lyman series is n=2



1 ⎛1 1 ⎞ ⎛3⎞ 4
⇒ = R ⎜ − 2 ⎟ = R ⎜ ⎟ ⇒ λL =
λL ⎝1 2 ⎠ ⎝4⎠ 3R n =1

The longest wavelength in the Balmer series is
1 ⎛ 1 1⎞ ⎛1 1⎞ ⎛9−4⎞ 1 ⎛ 5 ⎞ 36
⇒ = R⎜ 2 − 2 ⎟ = R⎜ − ⎟ = R⎜ ⎟⇒ = R ⎜ ⎟ ⇒ λB =
λB ⎝2 3 ⎠ ⎝4 9⎠ ⎝ 36 ⎠ λB ⎝ 36 ⎠ 5R
λL 4 5R 5
⇒ = × = = 0.185
λB 3R 36 27
Q32. X − rays of wavelength 0.24 nm are Compton scattered and the scattered beam is

observed at an angle of 60 o relative to the incident beam. The Compton wavelength of


the electron is 0.00243 nm . The kinetic energy of scattered elections in eV is……………
Ans. : 25
Solution: λ = 0.24 nm, λC = 0.00243 and θ = 600

∵ λ ′ − λ = λC (1 − cos θ ) ⇒ λ ′ = λ + λC (1 − cos θ )

⎛ 1⎞ 1
⇒ λ ′ = 0.24 + 0.00243 ⎜1 − ⎟ = 0.24 + 0.00243 × = 0.24 + 0.00121 = 0.2412nm
⎝ 2⎠ 2
Kinetic Energy of scattered electron
hc hc ⎛ 1 1 ⎞ 1
K .E. = − = 6.6 ×10−34 × 3 × 108 ⎜ − ⎟ × −9 Joules
λ λ′ ⎝ 0.24 0.2412 ⎠ 10
19.8 × 10−26 19.8 × 10−26
⇒ K .E. = ( 4.17 − 4.15 ) = × 0.02 = 396 × 10−20 Joules
10−9 10−9
396 ×10−20
⇒ K .E. = eV = 24.75 eV
1.6 ×10−19
                                                                                
Head office  Branch office 
 
fiziks, H.No. 40 D, G.F, Jia Sarai,  Anand Institute of Mathematics, 
 
Near IIT, Hauz Khas, New Delhi‐16  28‐B/6, Jia Sarai, Near IIT 
 
Phone: 011‐26865455/+91‐9871145498 Hauz Khas, New Delhi‐16 
                                                   
                                             Website: www.physicsbyfiziks.com                                                                                          
                                                            Email: fiziks.physics@gmail.com                                                                   121 
fiziks
Institute for NET/JRF, GATE, IIT‐JAM, JEST, TIFR and GRE in PHYSICAL SCIENCES 
 
IIT-JAM-2016
Q33. Consider a free electron ( e ) and a photon ( ph ) both having 10 eV of energy. If λ and

P represent wavelength and momentum respectively, then


(mass of electron = 9.1× 10−31 kg ; speed of light = 3 × 108 m / s )
(a) λe = λ ph and Pe = Pph (b) λe < λ ph and Pe > Pph

(c) λe > λ ph and Pe < Pph (d) λe > λ ph and Pe < Pph

Ans. : (c)
E h hc
Solution: For photon p ph = , λ ph = =
c p E

E 2 − m2c 4 h hc
For electron pe = , λe = =
c p E 2 − m2c 4
SECTION–B: MSQ
Q34. A slit has width ‘ d ’ along the x -direction. If a beam of electrons, accelerated in
y -direction to a particular velocity by applying a potential difference of 100 ± 0.1 kV

passes through the slit, then, which of the following statement ( s ) is (are) correct?

(a) The uncertainty in the position of the electrons in x -direction before passing the slit.

(b) The momentum of electrons in x - direction is ∼ immediately after passing the slit
d
(c) The uncertainty in the position of electrons in y - direction before passing the list is
zero
(d) The presence of the slit does not affect the uncertainty in momentum of electrons in
y - direction
Ans. : (b) and (d)

                                                                                
Head office  Branch office 
 
fiziks, H.No. 40 D, G.F, Jia Sarai,  Anand Institute of Mathematics, 
 
Near IIT, Hauz Khas, New Delhi‐16  28‐B/6, Jia Sarai, Near IIT 
 
Phone: 011‐26865455/+91‐9871145498 Hauz Khas, New Delhi‐16 
                                                   
                                             Website: www.physicsbyfiziks.com                                                                                          
                                                            Email: fiziks.physics@gmail.com                                                                   122 
fiziks
Institute for NET/JRF, GATE, IIT‐JAM, JEST, TIFR and GRE in PHYSICAL SCIENCES 
 
Solution: The electrons beam before slit is will collimated in y - direction.
x
Thus, before slit
Px
Py = P and Px = 0 θ
Py y
also Δx → ∞ as ΔPx = 0 d

Thus options (a) and (c) are not correct.

Now, after the slit ΔPx = d as a result ΔPx = =


Δx d

i.e. Px ≅
d
Thus option (b) is correct.
Whereas presence of slit does not affect the uncertainty in momentum in y - direction.
Thus option (d) is also correct.
Q35. A free particle of energy E collides with a one-dimensional square potential barrier of
height V and width W . Which one of the following statement(s) is/are correct?
(a) For E > V , the transmission coefficient for the particle across the barrier will always
be unity
(b) For E < V , the transmission coefficient changes more rapidly with W than with V
(c) For E < V , if V is doubled, the transmission coefficient will also be doubled.
(d) Sum of the reflection and the transmission coefficients is always one
Ans. : (b) and (d)
Solution: R + T = 1
2
⎛ E −V − E ⎞
R = ⎜⎜ ⎟⎟
⎝ E −V + E ⎠
SECTION–C: NAT (Numerical Answer Type)
Q36. A particular radioisotope has a half-life of 5 days. In 15 days the probability of decay in
percentage will be…………..
Ans. : 87.5

                                                                                
Head office  Branch office 
 
fiziks, H.No. 40 D, G.F, Jia Sarai,  Anand Institute of Mathematics, 
 
Near IIT, Hauz Khas, New Delhi‐16  28‐B/6, Jia Sarai, Near IIT 
 
Phone: 011‐26865455/+91‐9871145498 Hauz Khas, New Delhi‐16 
                                                   
                                             Website: www.physicsbyfiziks.com                                                                                          
                                                            Email: fiziks.physics@gmail.com                                                                   123 
fiziks
Institute for NET/JRF, GATE, IIT‐JAM, JEST, TIFR and GRE in PHYSICAL SCIENCES 
 
t / T1/ 2 15/ 5
⎛1⎞ ⎛1⎞ N0
N = N0 ⎜ ⎟ = N0 ⎜ ⎟ =
⎝2⎠ ⎝2⎠ 8
N0 − N 7
In 15 days the probability of decay = ×100 = × 100 = 87.5%
N0 8
Q37. In photoelectric experiment both sodium (work function = 2.3 eV ) and tungsten (work
function = 4.5 eV ) metals were illuminated by an ultraviolet light of same wavelength. If
the stopping potential for tungsten is measured to be 1.8V , the value of the stopping
potential for sodium will be………….. V .
Ans. : 4
Solution: For tungsten eVs = hν − Wt ⇒ hν = eVs + Wt = 1.8 + 4.5 = 6.3

For sodium eVs = hν − Ws = 6.3 − 2.3 = 4eV

Q38. The de Broglie wavelength of a relativistic electron having 1 MeV of energy

is………….. ×10−12 m . (Take the rest mass energy of the electron to be 0.5 MeV . Plank

constant = 6.63 ×10−34 Js , speed of light = 3 × 108 m / s , Electronic charge

= 1.6 × 10−19 C )
Ans. : 1.43

( )
2
E 2 − m0 c 2 1 − .25
Solution: E 2 = p 2 c 2 + m0 c ( )
2 2
⇒ p=
c 2
=
c
=
.75MeV
c
=

h 6.6 ×10−34 × 3 × 108 19.8 × 10−13


λ= = −13
= = 14.34 ×10−13 m = 1.43 × 10−12 m
p .75 × 1.6 ×10 1.38
Q39. X -ray of 20 keV energy is scattered inelastically from a carbon target. The kinetic

energy transferred to the recoiling electron by photons scattered at 900 with respect to the
incident beam is……………. keV .
(Planck constant = 6.6 × 10−34 Js, Speed of light = 3 × 108 m / s, electron mass =

9.1× 10−31 kg. Electronic charge = 1.6 × 10−19 C )


Ans. : 0.77

                                                                                
Head office  Branch office 
 
fiziks, H.No. 40 D, G.F, Jia Sarai,  Anand Institute of Mathematics, 
 
Near IIT, Hauz Khas, New Delhi‐16  28‐B/6, Jia Sarai, Near IIT 
 
Phone: 011‐26865455/+91‐9871145498 Hauz Khas, New Delhi‐16 
                                                   
                                             Website: www.physicsbyfiziks.com                                                                                          
                                                            Email: fiziks.physics@gmail.com                                                                   124 
fiziks
Institute for NET/JRF, GATE, IIT‐JAM, JEST, TIFR and GRE in PHYSICAL SCIENCES 
 
h h π
Solution: ∵ λ ' − λ = (1 − cos θ ) ⇒ λ ' − λ = ∵θ =
mc mc 2
λ' λ 1 1 1 1 1 1 1 1 1
⇒ − = 2
⇒ '− = 2
⇒ '= + 2⇒ +
hc hc mc E E mc E E mc 20keV .5MeV

⇒ E ' = 19.23keV
Recoil velocity of electron E − E ' = 0.77keV

                                                                                
Head office  Branch office 
 
fiziks, H.No. 40 D, G.F, Jia Sarai,  Anand Institute of Mathematics, 
 
Near IIT, Hauz Khas, New Delhi‐16  28‐B/6, Jia Sarai, Near IIT 
 
Phone: 011‐26865455/+91‐9871145498 Hauz Khas, New Delhi‐16 
                                                   
                                             Website: www.physicsbyfiziks.com                                                                                          
                                                            Email: fiziks.physics@gmail.com                                                                   125 
fiziks
Institute for NET/JRF, GATE, IIT‐JAM, JEST, TIFR and GRE in PHYSICAL SCIENCES 
 
Solid State Physics, Devices and Electronics
OBJECTIVE QUESTIONS
IIT-JAM-2005
Q1. A circuit and the signal applied at its input terminals (Vi) are shown in figure below.
Which one of the options correctly describes the output waveform (V0). (Assume all the
devices used are ideal).
+ 2V
C
0 t Vi D Vo

− 2V

(a) + 2V (b)

0 t 0 t

− 2V − 2V

(c) (d)
4V 0 t

0 t - 4V
Ans. : (c)
Q2. The susceptibility of a diamagnetic material is
(a) positive and proportional to temperature
(b) negative and inversely proportional to temperature
(c) negative and independent ofo temperature
(d) positive and inversely proportional to temperature
Ans. : (b)

                                                                                
Head office  Branch office 
 
fiziks, H.No. 40 D, G.F, Jia Sarai,  Anand Institute of Mathematics, 
 
Near IIT, Hauz Khas, New Delhi‐16  28‐B/6, Jia Sarai, Near IIT 
 
Phone: 011‐26865455/+91‐9871145498 Hauz Khas, New Delhi‐16 
                                                   
                                             Website: www.physicsbyfiziks.com                                                                                          
                                                            Email: fiziks.physics@gmail.com                                                                   126 
fiziks
Institute for NET/JRF, GATE, IIT‐JAM, JEST, TIFR and GRE in PHYSICAL SCIENCES 
 
Q3. Which of the following statements is correct for NaCl crystal structure?
(a) It is simple cubic lattice with one atom basis
(b) It is a face-centered cubic lattice with one atom basis
(c) It is a simple cubic lattice with two atom basis
(d) It is a face-centered cubic lattice with two atom basis
Ans. : (d)
IIT-JAM-2006
Q4. In a crystalline solid, the energy band structure (E-k relation) for an electron of mass m is
2
k (2k − 3)
given by E = . The effective mass of the electron in the crystal is
2m
2 m
(a) m (b) m (c) (d) 2m
3 2
Ans. : (c)
2
Solution: The expression of effective mass of electron in solid is m * =
d 2E
dk 2
2
dE d 2E 2
2 2 m
= ( 4k − 3 ) ⇒ 2 = ( 4 ) = ⇒ m* =
dk 2m dk 2m m 2
Q5. The truth table for the given circuit is
J

(a) J K Q (b) J K Q
0 0 1 0 0 1
0 1 0 0 1 0
1 0 1 1 0 0
1 1 0 1 1 1

                                                                                
Head office  Branch office 
 
fiziks, H.No. 40 D, G.F, Jia Sarai,  Anand Institute of Mathematics, 
 
Near IIT, Hauz Khas, New Delhi‐16  28‐B/6, Jia Sarai, Near IIT 
 
Phone: 011‐26865455/+91‐9871145498 Hauz Khas, New Delhi‐16 
                                                   
                                             Website: www.physicsbyfiziks.com                                                                                          
                                                            Email: fiziks.physics@gmail.com                                                                   127 
fiziks
Institute for NET/JRF, GATE, IIT‐JAM, JEST, TIFR and GRE in PHYSICAL SCIENCES 
 
(c) J K Q (d) J K Q
0 0 0 0 0 0
0 1 1 0 1 1
1 0 0 1 0 1
1 1 1 1 1 0

Ans. : (c)
Solution: Q = J .K + J .K = K
Q6. In an intrinsic semiconductor, the free carrier
concentration n (in cm-3) varies with temperature T
36.5
(in Kelvin) as shown in the figure below. The
35.5
band gap of the semiconductor is (use Boltzmann
ln(n ) 34.5
constant k B = 8.625 x 10-5 eVK-1 )
33.5
(a) 1.44 eV
32.5
(b) 0.72 eV
(c) 1.38 eV
2 3 1000 / Τ
(d) 0.69 eV
Ans. : (d)

⎛ E ⎞ n ⎡ Eg ⎛ 1 1 ⎞⎤
Solution: Since ni = n = N c N v exp ⎜ − g ⎟ ⇒ 1 = exp ⎢ ⎜ − ⎟⎥
⎝ 2kT ⎠ n2 ⎣ 2k ⎝ T2 T1 ⎠ ⎦

⎛n ⎞ ⎛ 1 1⎞
⇒ Eg = 2k ln ⎜ 1 ⎟ ⎜ − ⎟ = 2 × 8.625 × 10−5 ( 36.5 − 32.5 ) ( 0.003 − 0.0002 )
⎝ n2 ⎠ ⎝ T2 T1 ⎠

⇒ Eg = 2 × 8.625 × 10−5 × 4 1× 10−3 = 0.69eV

IIT-JAM-2007
Q7. Fermi energy of a certain metal M1 is 5 eV. A second metal M2 has an electron which is
6% higher than that of M1. Assuming that the free electron theory is valid for both the
metals, the Fermi energy of M2 is closet to
(a) 5.6 eV (b) 5.2 eV (c) 4.8 eV (d) 4.4 eV
Ans. : (b)
                                                                                
Head office  Branch office 
 
fiziks, H.No. 40 D, G.F, Jia Sarai,  Anand Institute of Mathematics, 
 
Near IIT, Hauz Khas, New Delhi‐16  28‐B/6, Jia Sarai, Near IIT 
 
Phone: 011‐26865455/+91‐9871145498 Hauz Khas, New Delhi‐16 
                                                   
                                             Website: www.physicsbyfiziks.com                                                                                          
                                                            Email: fiziks.physics@gmail.com                                                                   128 
fiziks
Institute for NET/JRF, GATE, IIT‐JAM, JEST, TIFR and GRE in PHYSICAL SCIENCES 
 

(3π n)
2
2/3
The expression of Fermi energy is E F = 2

2m
Let us consider that n1 , E F1 is the concentration of electron and Fermi energy in metal M1

and n2 , E F2 is in metal M2.

Given n 2 = n1 + 0.06n1 = 1.06n1

⎛ 2 ⎞
( )
2/3
3π 2 n2
EF2 ⎜⎝ 2m ⎟
⎠ = ( n2 ) = (1.06n1 ) = 1.06 2/3 = 1.0396
2/3 2/3

= ( )
EF1 ⎛ 2 2/3 ⎞ ( n1 ) ( n1 )
2/3 2/3


2 m
3π 2 n1 ( ) ⎟
⎝ ⎠
EF2 = 1.0396 × EF1 = 1.0396 × 5eV = 5.2 eV

Q8.

Fig. (i) Fig. (ii)


Figure (i) and (ii) represent respectively,
(a) NOR, NOR (b) NOR, NAND
(c) NAND, NAND (d) OR, NAND

Ans. : (c)
IIT-JAM-2008
Q9. The ratio of the second-neighbour distance to the nearest-neighbour distance in an fcc
lattice is
(a) 2 2 (b) 2 (c) 3 (d) 2
Ans. : (d)
Solution: In FCC lattice the first nearest is at distance = 2a / 2 = a / 2
Whereas the second nearest at distance of = a
The ratio of the second-neighbour distance to the nearest-neighbour distance is
a
= 2
a/ 2
                                                                                
Head office  Branch office 
 
fiziks, H.No. 40 D, G.F, Jia Sarai,  Anand Institute of Mathematics, 
 
Near IIT, Hauz Khas, New Delhi‐16  28‐B/6, Jia Sarai, Near IIT 
 
Phone: 011‐26865455/+91‐9871145498 Hauz Khas, New Delhi‐16 
                                                   
                                             Website: www.physicsbyfiziks.com                                                                                          
                                                            Email: fiziks.physics@gmail.com                                                                   129 
fiziks
Institute for NET/JRF, GATE, IIT‐JAM, JEST, TIFR and GRE in PHYSICAL SCIENCES 
 
Q10. Consider a doped semiconductor having the electron and the hole mobilities μn and μp,
respectively. Its intrinsic carrier density is ni. The hole concentration p for which the
conductivity is minimum at a given temperature is

μn μp μp μn
(a) ni (b) ni (c) ni (d) ni
μp μn μn μp

Ans. : (a)
⎛ n2 ⎞
(
Solution: conductivity σ = e nμn + pμ p = e ⎜ i μn + pμ p ⎟
⎜ p ⎟ )
⎝ ⎠

For min imum conductivity, = 0 ⇒ p = ni μn / μ p
dp

Q11. The logic expression for the output Y of the following circuit is

P
Y
Q

R
S

(a) P + Q + QR + S (b) P + Q + QR + S

(c) P + Q + QR + S (d) P + Q + QR + S
Ans. : (a)

P P+Q P + Q + QR P + Q + QR + S
Y
Q

R P + Q + QR + S
QR
S

                                                                                
Head office  Branch office 
 
fiziks, H.No. 40 D, G.F, Jia Sarai,  Anand Institute of Mathematics, 
 
Near IIT, Hauz Khas, New Delhi‐16  28‐B/6, Jia Sarai, Near IIT 
 
Phone: 011‐26865455/+91‐9871145498 Hauz Khas, New Delhi‐16 
                                                   
                                             Website: www.physicsbyfiziks.com                                                                                          
                                                            Email: fiziks.physics@gmail.com                                                                   130 
fiziks
Institute for NET/JRF, GATE, IIT‐JAM, JEST, TIFR and GRE in PHYSICAL SCIENCES 
 
IIT-JAM-2009
Q12. Monochromatic X-rays of wavelength 1 Ǻ are incident on a simple cubic crystal. The first
order Bragg reflection from (311) plane occurs at angle of 30 0 from the plane. The lattice
parameter of the crystal in Ǻ is

11
(a) 1 (b) 3 (c) (d) 11
2
Ans. : (d)
Solution: Expression of Bragg´s law is 2d sin θ = nλ ,
where, d is the inter-planar spacing defined for cubic lattice of lattice constant a in term
a
of Miller indices (h k l) as d =
h + k2 + l2
2

The lattice parameter a is


λ 1 1
a= h2 + k 2 + l 2 = 3 2 + 12 + 12 = × 11 = 11
2 × sin θ 2 × sin 30 0
2 ×1/ 2
Q13. Which one of the following is an INCORRECT Boolean expression?
(a) P Q + PQ = Q (b) (P + Q )(P + Q ) = P

(c) P(P + Q ) = Q (d) (P Q R + P Q R + PQ R + PQ R ) = Q


Ans. : (c)
(a) PQ + PQ = ( P + P ) Q = 1.Q = Q

(b) ( P + Q ) ( P + Q ) = P + QQ = P

(c) P ( P + Q ) = P + PQ = P (1 + Q ) = P

(d) ( PQR + PQR + PQR + PQR ) = PQ ( R + R ) + PQ ( R + R ) = PQ + PQ = ( P + P ) Q = Q

Q14. A battery with a constant emf ε and internal resistance ri provides power to an external

circuit with a load resistance made up by combining resistance RL and 2 RL in parallel.

For what value of RL will the power delivered to the load be maximum?

ri ri 2 3
(a) R L = (b) RL = (c) RL = ri (d) RL = ri
4 2 3 2

                                                                                
Head office  Branch office 
 
fiziks, H.No. 40 D, G.F, Jia Sarai,  Anand Institute of Mathematics, 
 
Near IIT, Hauz Khas, New Delhi‐16  28‐B/6, Jia Sarai, Near IIT 
 
Phone: 011‐26865455/+91‐9871145498 Hauz Khas, New Delhi‐16 
                                                   
                                             Website: www.physicsbyfiziks.com                                                                                          
                                                            Email: fiziks.physics@gmail.com                                                                   131 
fiziks
Institute for NET/JRF, GATE, IIT‐JAM, JEST, TIFR and GRE in PHYSICAL SCIENCES 
 
Ans. : (d)
RL × 2 RL 2
Solution: Since RL and 2 RL are in parallel so load R = = RL .
RL + 2 RL 3
2
⎛ ε ⎞
Power through load P = I R = ⎜ ⎟ R
2

⎝ ri + R ⎠
For maximum power through load

( r + R ) ε 2 − ε 2 R × 2 ( ri + R ) = 0 ⇒ r + R − 2 R = 0 ⇒ R = r
2
dP
=0⇒ i (i )
( ri + R )
4 i
dR

2 3
Thus R = ri ⇒ RL = ri ⇒ RL = ri
3 2
IIT-JAM-2010
Q15. The following are the plots of the temperature dependence of the magnetic susceptibility
for three different samples.
a b c
χ (T ) χ (T ) χ (T )

0 T 0 T 0 T
The plots a, b and c correspond to
(a) ferromagnet, paramagnet and diamagnet, respectively
(b) paramagnet, diamagnet and ferromagnet, respectively
(c) ferromagnet, diamagnet and paramagnet, respectively
(d) diamagnet, paramagnet and ferromagnet, respectively
Ans. : (b)

                                                                                
Head office  Branch office 
 
fiziks, H.No. 40 D, G.F, Jia Sarai,  Anand Institute of Mathematics, 
 
Near IIT, Hauz Khas, New Delhi‐16  28‐B/6, Jia Sarai, Near IIT 
 
Phone: 011‐26865455/+91‐9871145498 Hauz Khas, New Delhi‐16 
                                                   
                                             Website: www.physicsbyfiziks.com                                                                                          
                                                            Email: fiziks.physics@gmail.com                                                                   132 
fiziks
Institute for NET/JRF, GATE, IIT‐JAM, JEST, TIFR and GRE in PHYSICAL SCIENCES 
 
Q16. The value of θ at which the first-order peak in X-ray (λ =1.53Ǻ) diffraction
corresponding to (111) plane of a simple cubic structure with the lattice constant, α =
2.65 Ǻ, is approximately
(a) 15o (b) 30o (c) 45o (d) 60o
Ans. : (b)
Solution: Expression of Bragg´s law is
2d sin θ = nλ
where, d is the inter-planar spacing defined for cubic lattice of lattice constant a in term
a
of Miller indices (h k l) as d =
h2 + k 2 + l 2
λ λ 1.53 1 ⎛1⎞
sin θ = = h2 + k 2 + l 2 = × 3 = ⇒ θ = sin −1 ⎜ ⎟ = 300
2d 2a 2 × 2.65 2 ⎝2⎠

IIT-JAM-2010
Q17. Consider the following truth table
A B C F
The logic expression for F is
0 0 0 1
(a) AB + BC + CA 0 0 1 0
0 1 0 0
(b) A B + AC + B C 0 1 1 0
1 0 0 1
(c) CA B + AB
1 0 1 1
(d) C ( A + B ) + AB 1 1 0 1
1 1 1 0
Ans. : (d)
C C
1 0
A B 00 1

A B 01
AC
AB 11 1

AB 10 1 1

AB BC

                                                                                
Head office  Branch office 
 
fiziks, H.No. 40 D, G.F, Jia Sarai,  Anand Institute of Mathematics, 
 
Near IIT, Hauz Khas, New Delhi‐16  28‐B/6, Jia Sarai, Near IIT 
 
Phone: 011‐26865455/+91‐9871145498 Hauz Khas, New Delhi‐16 
                                                   
                                             Website: www.physicsbyfiziks.com                                                                                          
                                                            Email: fiziks.physics@gmail.com                                                                   133 
fiziks
Institute for NET/JRF, GATE, IIT‐JAM, JEST, TIFR and GRE in PHYSICAL SCIENCES 
 
IIT-JAM-2011
Q18. An X-ray diffraction (XRD) experiment is carried out on a crystalline solid having FCC
structure at room temperature. The solid undergoes a phase transformation on cooling to -
20oC and shows orthorhombic structure with small decreas in its unit cell lengths as
compared to the FCC unit cell lengths. As a result the (311) line of the XRD pattern
corresponding to the FCC system
(a) will split into a doublet
(b) will split into a triplet
(c) will remain unchanged
(d) will split into two separate doublets
Ans. : (b)
a
Solution: In FCC the inter-planar spacing is defined as d =
h + k2 + l2
2

1
whereas in Orthorhombic the inter-planar spacing is d =
h2 k 2 l 2
+ +
a2 b2 c2
For (311), numbers XRD peaks in FCC is derived from Bragg’s law as
λ λ λ λ
sin θ = = h2 + k 2 + l 2 = 3 2 + 12 + 12 = 11
2d 2a 2a 2a
Thus only one peak apeears in FCC.
λ λ
In Orthorhombic, sin θ = =
2d h 2
k2 l2
2 2 + 2 + 2
a b c
For (3 1 1), there will be three peaks corresponding to
⎛ ⎞ ⎛ ⎞ ⎛ ⎞
⎜ ⎟ ⎜ ⎟ ⎜ ⎟
−1 ⎜ λ ⎟ ,θ = sin −1 ⎜ λ ⎟ and θ = sin −1 ⎜ λ ⎟
θ1 = sin
⎜ 3 12 12
2 ⎟ 2
⎜ 1 2
32 12 ⎟ 3
⎜ 1 12 32
2 ⎟
⎜ 2 2
+ 2+ 2 ⎟ ⎜ 2 2
+ 2
+ 2 ⎟ ⎜2 2 + 2 + 2 ⎟
⎝ a b c ⎠ ⎝ a b c ⎠ ⎝ a b c ⎠

                                                                                
Head office  Branch office 
 
fiziks, H.No. 40 D, G.F, Jia Sarai,  Anand Institute of Mathematics, 
 
Near IIT, Hauz Khas, New Delhi‐16  28‐B/6, Jia Sarai, Near IIT 
 
Phone: 011‐26865455/+91‐9871145498 Hauz Khas, New Delhi‐16 
                                                   
                                             Website: www.physicsbyfiziks.com                                                                                          
                                                            Email: fiziks.physics@gmail.com                                                                   134 
fiziks
Institute for NET/JRF, GATE, IIT‐JAM, JEST, TIFR and GRE in PHYSICAL SCIENCES 
 
Q19. Which of the following circuit does not satisfy the Boolean expression AB + A B = F
(a)
Α
Β
F

(b) Α
Β
F

(c) Α
Β
F

Α
(d)
Β
F

Ans. : (d)
(a) F = AB + AB

(b) F = ( A + B ) . AB = ( A + B ) . A + B = AB + AB ( )
(
(c) F = ( A + B ) . A + B = AB + AB )
(d) F = AB + AB

                                                                                
Head office  Branch office 
 
fiziks, H.No. 40 D, G.F, Jia Sarai,  Anand Institute of Mathematics, 
 
Near IIT, Hauz Khas, New Delhi‐16  28‐B/6, Jia Sarai, Near IIT 
 
Phone: 011‐26865455/+91‐9871145498 Hauz Khas, New Delhi‐16 
                                                   
                                             Website: www.physicsbyfiziks.com                                                                                          
                                                            Email: fiziks.physics@gmail.com                                                                   135 
fiziks
Institute for NET/JRF, GATE, IIT‐JAM, JEST, TIFR and GRE in PHYSICAL SCIENCES 
 
IIT-JAM-2012
Q20. An X-ray beam of wavelength 1.54 Å is diffracted from the (110) planes of a solid with a
cubic lattice of lattice constant 3.08 Å. The first-order Bragg diffraction occurs at
⎛1⎞ ⎛ 1 ⎞ ⎛1⎞ ⎛ 1 ⎞
(a) sin −1 ⎜ ⎟ (b) sin −1 ⎜ ⎟ (c) sin −1 ⎜ ⎟ (d) sin −1 ⎜ ⎟
⎝4⎠ ⎝2 2⎠ ⎝2⎠ ⎝ 2⎠
Ans. : (b)
Solution: Expression of Bragg´s law is 2d sin θ = nλ
where, d is the inter-planar spacing defined for cubic lattice of lattice constant a in term
a
of Miller indices (h k l) as d =
h + k2 + l2
2

λ λ 1.54 1 ⎛ 1 ⎞
sin θ = = h2 + k 2 + l 2 = × 2= ⇒ θ = sin −1 ⎜ ⎟
2d 2a 2 × 3.08 2 2 ⎝2 2⎠
Q21. The Boolean expression P + P Q , where P and Q are the inputs to a circuit, represents the
following logic gate
(a) AND (b) NAND (c) NOT (d) OR
Ans. : (d)
IIT-JAM-2013
Q22. The fraction of volume unoccupied in the unit cell of the body centered cubic lattice is

8 − 3π 3π 6 − 2π π
(a) (b) (c) (d)
8 8 6 3 2
Ans. : (a)
Solution: The effective number of atoms in bcc structure is E F

1 1 1 1
neff = nc + n f + ni = × 8 + × 0 + 1 = 2 Α Β
8 2 8 2
The radius of atom and lattice constant are related as
a
a 3 Η
r= G
4 a
The volume occupide by the atoms in unit cell is D C
a

                                                                                
Head office  Branch office 
 
fiziks, H.No. 40 D, G.F, Jia Sarai,  Anand Institute of Mathematics, 
 
Near IIT, Hauz Khas, New Delhi‐16  28‐B/6, Jia Sarai, Near IIT 
 
Phone: 011‐26865455/+91‐9871145498 Hauz Khas, New Delhi‐16 
                                                   
                                             Website: www.physicsbyfiziks.com                                                                                          
                                                            Email: fiziks.physics@gmail.com                                                                   136 
fiziks
Institute for NET/JRF, GATE, IIT‐JAM, JEST, TIFR and GRE in PHYSICAL SCIENCES 
 
3
4π ⎛a 3⎞ 3 π 3 3
2× ⎜⎜ ⎟⎟ a = a
3 ⎝ 4 ⎠ 8

Thus amount of volume unoccupied in the unit cell is

π 3 ⎛8−π 3 ⎞ 3
= volume of unit cell − filled volume = a 3 − a 3 = ⎜⎜ ⎟a

8 ⎝ 8 ⎠
Thus the fraction of volume unoccupied in the unit cell of the body centered cubic lattice
⎛ 8 −π 3 ⎞
is ⎜⎜ ⎟

⎝ 8 ⎠
Q23. For an ideal op-amp circuit given below, the dc gain 1 kΩ
+
and the cut off frequency respectively are 1
μF
(a) 1 and 1 kHz 2π

(b) 1 and 100 Hz
10 kΩ
(c) 11 and 1 kHz 1 kΩ
(d) 11 and 100 Hz

Ans. : (b)
10 1 1
Solution: DC Gain = 1 + = 11 and f H = ⇒ fH = = 1kHz
1 2π RC 2π × 1×
1

Q24. A variable power supply 5V − 20V is connected to a Zener diode specified by a
breakdown voltage of 10V (see figure). The ratio of the maximum power to the minimum
power dissipated across the load resistor is
500 Ω

5V − 20 V 1 kΩ

                                                                                
Head office  Branch office 
 
fiziks, H.No. 40 D, G.F, Jia Sarai,  Anand Institute of Mathematics, 
 
Near IIT, Hauz Khas, New Delhi‐16  28‐B/6, Jia Sarai, Near IIT 
 
Phone: 011‐26865455/+91‐9871145498 Hauz Khas, New Delhi‐16 
                                                   
                                             Website: www.physicsbyfiziks.com                                                                                          
                                                            Email: fiziks.physics@gmail.com                                                                   137 
fiziks
Institute for NET/JRF, GATE, IIT‐JAM, JEST, TIFR and GRE in PHYSICAL SCIENCES 
 
Ans. :
1000
Solution: When V = 5V ⇒ open circuit voltage Vi = × 5 = 3.33 < VZ = 10V
1500
Vi 2
⇒ VL = Vi = 3.33V ⇒ PL ,min = .
RL
1000
When V = 20V ⇒ open circuit voltage Vi = × 20 = 13.33 > VZ = 10V
1500

VZ2 PL,max VZ2 ⎛ 10 ⎞ 2


⇒ V L = V z = 10V ⇒ PL ,max = ⇒ = =⎜ ⎟ = 9.2
RL PL ,min Vi 2 ⎝ 3.33 ⎠

IIT-JAM-2014

Q25. Octal equivalent of decimal number ( 478 )10 is

(a) 736 8 (b) 6738 (c) 637 8 (d) 367 8


Ans. : (a)
Solution: 7368 = 7 × 82 + 3 × 81 + 6 × 80 = 448 + 24 + 6 = 47810

6738 = 6 × 82 + 7 × 81 + 3 × 80 = 384 + 56 + 3 = 44310

6378 = 6 × 82 + 3 × 81 + 7 × 80 = 384 + 24 + 7 = 41510

3678 = 3 × 82 + 6 × 81 + 7 × 80 = 192 + 48 + 7 = 24710


Q26. In an ideal operational amplifier depicted below, the potential at node A is
25 kΩ
5 kΩ A _
5V
+
IV

(a) 1 V (b) 0 V (c) 5 V (d) 25 V


Ans. : (b)
Solution: Node A is virtually grounded.

                                                                                
Head office  Branch office 
 
fiziks, H.No. 40 D, G.F, Jia Sarai,  Anand Institute of Mathematics, 
 
Near IIT, Hauz Khas, New Delhi‐16  28‐B/6, Jia Sarai, Near IIT 
 
Phone: 011‐26865455/+91‐9871145498 Hauz Khas, New Delhi‐16 
                                                   
                                             Website: www.physicsbyfiziks.com                                                                                          
                                                            Email: fiziks.physics@gmail.com                                                                   138 
fiziks
Institute for NET/JRF, GATE, IIT‐JAM, JEST, TIFR and GRE in PHYSICAL SCIENCES 
 
Q27. To operate a npn transistor in active region, its emitter-base and collector- base junction
respectively, should be
(a) forward biased and reversed biased (b) forward biased and forward biased
(c) reversed biased and forward biased (d) reversed biased and reversed biased
Ans. : (a)
Q28. Diamond lattice can be considered as a combination of two fcc lattice displaced along the
body diagonal by one quarter of its length. There are eight atoms per unit cell. The
packing fraction of the diamond structure is
(a) 0.48 (b) 0.74 (c) 0.34 (d) 0.68
Ans. : (c)
Q29. Thermal neutrons (energy = 300 k B = 0.025 eV) are sometimes used for structural
determination of materials. The typical lattice spacing of a material for which these can
be used is
(a) 0.01 nm (b) 0.05 nm (c) 0.1 nm (d) 0.15 nm
Q30. A sine wave of 5V amplitude is applied at
Vin 1 kΩ Vout
the input of the circuit shown in the figure.
Which of the following waveforms
represents the output most closely? 3V
5V 5V
(a) (b)
-3V
-5V

(c) 3V (d) 3V
-3V
-5V

Ans. : (d)
Solution: If Vin < 3 V , diode is OFF and V0 = Vin .

If Vin > 3 V , diode is ON and V0 = 3 V .

                                                                                
Head office  Branch office 
 
fiziks, H.No. 40 D, G.F, Jia Sarai,  Anand Institute of Mathematics, 
 
Near IIT, Hauz Khas, New Delhi‐16  28‐B/6, Jia Sarai, Near IIT 
 
Phone: 011‐26865455/+91‐9871145498 Hauz Khas, New Delhi‐16 
                                                   
                                             Website: www.physicsbyfiziks.com                                                                                          
                                                            Email: fiziks.physics@gmail.com                                                                   139 
fiziks
Institute for NET/JRF, GATE, IIT‐JAM, JEST, TIFR and GRE in PHYSICAL SCIENCES 
 
Q31. 1011 binary input have been applied at X 3 X 2 X 1 X 0 input in the shown logic circuit made

of XOR gates. The binary output Y3Y2Y1Y0 of the circuit will be


X3 Y3
X2 Y2

X1 Y1

X0 Y0

(a) 1101 (b) 1010 (c) 1111 (d) 0101


Ans. : (a)
1 X3 Y3 1
0 X2 Y2 1

1 X1 Y1 0

1 X0 Y0 1

IIT-JAM-2015
Q32. Temperature dependence of resistivity of a metal can be described by
(a) R
(b) R

T T
(c) R (d) R

T
T
                                                                                
Head office  Branch office 
 
fiziks, H.No. 40 D, G.F, Jia Sarai,  Anand Institute of Mathematics, 
 
Near IIT, Hauz Khas, New Delhi‐16  28‐B/6, Jia Sarai, Near IIT 
 
Phone: 011‐26865455/+91‐9871145498 Hauz Khas, New Delhi‐16 
                                                   
                                             Website: www.physicsbyfiziks.com                                                                                          
                                                            Email: fiziks.physics@gmail.com                                                                   140 
fiziks
Institute for NET/JRF, GATE, IIT‐JAM, JEST, TIFR and GRE in PHYSICAL SCIENCES 
 
Ans. : (a)
Solution: Electrical resistivity of metal varies as
ρ ∝T5 (For T << θ D )

ρ ∝T (For T >> θ D )

where θ D is the Debye temperature. Thus, correct answer is option (a)

Q33. A Zener regulator has an input voltage in the range 15V − 20V and a load current in the
range of 5 mA − 20 mA . If the Zener voltage is 6.8V , the value of the series resistor
should be RS
V0

+
15 − 20 V 6 .8 V

(a) 390 Ω (b) 420 Ω (c) 440 Ω (d) 460 Ω


Ans. : Some data is missing. (No answer is possible)
Q34. Which of the following circuits represent the Boolean expression

S = P + QR + Q P
(a) P (b) P
S S
Q Q

(c) P (d) P
Q Q
S S
R R

Ans. : (b)

(
Solution: S = P + QR + QP = P + QR QP = ( P + QR ) ( QP ) = PQ )( )

                                                                                
Head office  Branch office 
 
fiziks, H.No. 40 D, G.F, Jia Sarai,  Anand Institute of Mathematics, 
 
Near IIT, Hauz Khas, New Delhi‐16  28‐B/6, Jia Sarai, Near IIT 
 
Phone: 011‐26865455/+91‐9871145498 Hauz Khas, New Delhi‐16 
                                                   
                                             Website: www.physicsbyfiziks.com                                                                                          
                                                            Email: fiziks.physics@gmail.com                                                                   141 
fiziks
Institute for NET/JRF, GATE, IIT‐JAM, JEST, TIFR and GRE in PHYSICAL SCIENCES 
 
P P
(a ) S = PQ = P + Q (b) S = PQ
Q Q

(c) P
(d )
P
Q P+Q Q PQ
S = ( P + Q) R S = PQR
R R
R R

Q35. Miller indicates of a plane in cubic structure that contains all the directions [100], [011] and
[111] are
(a) (011) (b) (101) (c) (100 ) (d) (110 )
Ans. : (a)
Solution: The name of the plane containing all the directions
y [111]
[100] , [011] & [111] is ( 0 11) or ( 01 1 )
The best suitable answer is option (a) [ 011]
x

[100]
z

SECTION–B: MSQ
Q36. In an ideal Op-Amp circuit shown below R1 = 3k Ω, R2 = 1k Ω and Vi = 0.5sin ω t
(in Volt). Which of the following statements are true? V +
i
V0
(a) The current through R1 = The current through R2 −
R2 P
(b) The potential at P is V0
R1 R1
R2
(c) The amplitude of V0 is 2V

(d) The output voltage V0 is in phase with Vi


Ans. : (a), (c) and (d)

                                                                                
Head office  Branch office 
 
fiziks, H.No. 40 D, G.F, Jia Sarai,  Anand Institute of Mathematics, 
 
Near IIT, Hauz Khas, New Delhi‐16  28‐B/6, Jia Sarai, Near IIT 
 
Phone: 011‐26865455/+91‐9871145498 Hauz Khas, New Delhi‐16 
                                                   
                                             Website: www.physicsbyfiziks.com                                                                                          
                                                            Email: fiziks.physics@gmail.com                                                                   142 
fiziks
Institute for NET/JRF, GATE, IIT‐JAM, JEST, TIFR and GRE in PHYSICAL SCIENCES 
 
VR
Solution: Voltage at P is VP = 0 2 .
R1 + R2

V0 R2
Vo −
Vo − VP R1 + R2 V0 R1 V0
Current through R1 is I1 = = = =
R1 R1 R1 ( R1 + R2 ) ( R1 + R2 )

VP V0
and Current through R2 is I 2 = = .
R2 ( R1 + R2 )

Thus I1 = I 2 . Option (a) is true

V0 R2
The potential at P is VP = . Option (b) is not true.
R1 + R2

⎛ R ⎞ ⎛ 3⎞
V0 = ⎜ 1 + 2 ⎟ Vi = ⎜1 + ⎟ 0.5sin ωt = 2sin ωt ⇒ Vm = 2 V . Option (c) is true
⎝ R1 ⎠ ⎝ 1⎠
Option (d) is true
SECTION–C: NAT (Numerical Answer Type)
Q37. In the given circuit VCC = 10V and β = 100 for n − p − n transistor. The collector voltage
VCC
VC (in volts) is………….
1K

100 K VC

+
5V

Ans. : 5.7
5 − 0.7
Solution: I B = = 4.3 × 10−5 A ⇒ I C = β I B = 4.3 mA
100 × 10 3

⇒ VC = VCC − I C RC = 10 − 4.3 = 5.7 V

Q38. A diode at room temperature (kT = 0.025 eV ) with a current of 1μA has a forward bias
voltage V F = 0.4V . For V F = 0.5V , the value of the diode current (in μA ) is…………..
                                                                                
Head office  Branch office 
 
fiziks, H.No. 40 D, G.F, Jia Sarai,  Anand Institute of Mathematics, 
 
Near IIT, Hauz Khas, New Delhi‐16  28‐B/6, Jia Sarai, Near IIT 
 
Phone: 011‐26865455/+91‐9871145498 Hauz Khas, New Delhi‐16 
                                                   
                                             Website: www.physicsbyfiziks.com                                                                                          
                                                            Email: fiziks.physics@gmail.com                                                                   143 
fiziks
Institute for NET/JRF, GATE, IIT‐JAM, JEST, TIFR and GRE in PHYSICAL SCIENCES 
 
Ans. : 54.5

I2 (e − 1) ( e0.5/ 0.025 − 1) ( e 20 − 1)
V2 / VT

Solution: I = I 0 ( eV / VT − 1) ⇒ = V /V = = = 54.5 ⇒ I 2 = 54.5 μ A


I1 ( e 1 T − 1) ( e0.4 / 0.025 − 1) ( e16 − 1)

Q39. GaAs has a diamond structure. The number of Ga-As bonds per atom which have to be
broken to fracture the crystal in the (001) plane is………..
Ans. : 4
Solution: Diamond structure has tetrahedral bond. To fracture the diamond structure along
( 0 0 1) plane, four bonds need to be broken.

IIT-JAM-2016
Q40. The solution of the Boolean equation Y = A + B + AB is
(a) 1 (b) AB (c) A B (d) A + B
Ans. : (b) and (d) both are correct

Solution: Y = A + B + AB = A.B + A + B = A 1 + B + B = A + B or AB ( )
Q41. If a constant voltage +V is applied to the input of the following OPAMP circuit for a
time t , then the output voltage V0 will approach
C

R

+V
+ Vo

(a) +V exponentially (b) −V exponentially


(c) +V linearly (d) −V linearly
Ans. : (d)
V −0 d ( 0 − V0 ) dV V V
Solution: ∵ I R = I C ⇒ =C ⇒ 0 =− ⇒ V0 = − t+c
R dt dt RC RC

                                                                                
Head office  Branch office 
 
fiziks, H.No. 40 D, G.F, Jia Sarai,  Anand Institute of Mathematics, 
 
Near IIT, Hauz Khas, New Delhi‐16  28‐B/6, Jia Sarai, Near IIT 
 
Phone: 011‐26865455/+91‐9871145498 Hauz Khas, New Delhi‐16 
                                                   
                                             Website: www.physicsbyfiziks.com                                                                                          
                                                            Email: fiziks.physics@gmail.com                                                                   144 
fiziks
Institute for NET/JRF, GATE, IIT‐JAM, JEST, TIFR and GRE in PHYSICAL SCIENCES 
 
SECTION–B: MSQ
Q42. A pn junction was formed with a heavily doped 1018 cm −3 p -region and lightly doped ( )
(10 14
)
cm −3 n - region. Which of the following statement ( s ) is (are) correct?

(a) The width of the depletion layer will be more in the n - side of the junction
(b) The width of the depletion layer will be more in the p - side of the junction
(c) The width of the depletion layer will be same on the both side of the junction
(d) If the pn junction is reverse biased, then the width of the depletion region increase
Ans. : (a), (d)
SECTION–C: NAT (Numerical Answer Type)
Q43. The addition of two binary numbers 1000.01 and 0001.11 in binary representation
is………….
Ans. : 1010
Q44. The number of second-nearest neighbor ions to a Na + ion in NaCl crystal is__________.
Ans. : 12
Cl −
2a a Na +
Solution: The 2nd nearest neighbour is at distance = =
2 2
3× 8
The number of 2nd nearest neighbour = = 12
2

Q45. The output voltage V0 of the OPAMP circuit given below is…………….. V
2R

R

R Vo
1V +
R
2V
R
3V
Ans. : 6

                                                                                
Head office  Branch office 
 
fiziks, H.No. 40 D, G.F, Jia Sarai,  Anand Institute of Mathematics, 
 
Near IIT, Hauz Khas, New Delhi‐16  28‐B/6, Jia Sarai, Near IIT 
 
Phone: 011‐26865455/+91‐9871145498 Hauz Khas, New Delhi‐16 
                                                   
                                             Website: www.physicsbyfiziks.com                                                                                          
                                                            Email: fiziks.physics@gmail.com                                                                   145 
fiziks
Institute for NET/JRF, GATE, IIT‐JAM, JEST, TIFR and GRE in PHYSICAL SCIENCES 
 
2R
⎛ 2R ⎞
Solution: V0 = ⎜1 + V
⎟ 1 = 3V1 where
⎝ R ⎠ R

R/2 R/2 R/2 R
V1 = ×1 + ×2+ ×3 1V +
R+R/2 R+R/2 R+R/2
R
2V
1 1 1 R
V1 = ×1 + × 2 + × 3 = 2V ⇒ V0 = 6 V 3V
3 3 3
Q46. In the circuit given below, the collector to emitter voltage VCE is…………… V .

(Neglect VBE , take β = 100 ) VCC = +10 V

5k Ω

5kΩ

VCE

5kΩ

10 k Ω

Ans. : 2.5
5 V 5
Solution: VB = × 10 = 5 V ⇒ VE = VB − VBE ≈ 5 V ⇒ I E = E = = 0.5 mA
5+5 RE 10

VCE = VCC − I C ( RC + RC ) = 10 − 0.5 ( 5 + 10 ) = 2.5 V

Q47. X -ray diffraction of a cubic crystal gives an intensity maximum for Bragg angle 200
corresponding to the (110 ) plane. The lattice parameter of the crystal is………….. nm .

(Consider wavelength of X − ray = 0.15 nm )


Ans. : 0.31
Solution: According to Bragg’s law 2d sin θ = nλ
λ 0.5 ×10−9 0.15 × 10−9
For n = 1 , 2d sin θ = λ ⇒ d = = ⇒ d = = 0.219 nm
2sin θ 2 × sin 200 2 × 0.342

                                                                                
Head office  Branch office 
 
fiziks, H.No. 40 D, G.F, Jia Sarai,  Anand Institute of Mathematics, 
 
Near IIT, Hauz Khas, New Delhi‐16  28‐B/6, Jia Sarai, Near IIT 
 
Phone: 011‐26865455/+91‐9871145498 Hauz Khas, New Delhi‐16 
                                                   
                                             Website: www.physicsbyfiziks.com                                                                                          
                                                            Email: fiziks.physics@gmail.com                                                                   146 
fiziks
Institute for NET/JRF, GATE, IIT‐JAM, JEST, TIFR and GRE in PHYSICAL SCIENCES 
 
a a
Now, d = =
h +k +l
2 2 2
2

∴ a = 2 d = 0.31 nm ⇒ a = 0.31 nm

                                                                                
Head office  Branch office 
 
fiziks, H.No. 40 D, G.F, Jia Sarai,  Anand Institute of Mathematics, 
 
Near IIT, Hauz Khas, New Delhi‐16  28‐B/6, Jia Sarai, Near IIT 
 
Phone: 011‐26865455/+91‐9871145498 Hauz Khas, New Delhi‐16 
                                                   
                                             Website: www.physicsbyfiziks.com                                                                                          
                                                            Email: fiziks.physics@gmail.com                                                                   147 
fiziks
Forum for CSIR-UGC JRF/NET, GATE, IIT-JAM, GRE in PHYSICAL SCIENCES

Tata Institute of Fundamental Research


Entrance test for the Ph.D. Programme: Sample Questions
PHYSICAL SCIENCES
SYLLABUS
In general, the syllabus for the Admissions Tests in Physics (Written and Interview) is
typically a course of studies in Physics as the main subject in the Undergraduate and
Masters levels in an Indian University. The questions in the written test (multiplechoice
type) and the interviews are distributed over the following areas:
Classical Mechanics;
Mathematics relevant to Physics;
Electricity and Magnetism;
Quantum Mechanics; Heat,
Thermodynamics and Statistical Physics;
General Physics;
Modern Physics;
Electronics and Experimental Physics.
SAMPLE QUESTIONS

(NOTE: Wrong answers will get you negative marks)


1. The function f(z) = z2 + 1 is integrated over a circle of unit radius in the complex z
plane. What is the value of the integral?
[a] 1 [b] i [c] 0

2. Charged particles are beamed into a region having a uniform electric field of
10 3 Newton/Coulomb and a uniform magnetic field of 10-2 Newton/(ampere meter). The
electric and magnetic fields are at right angles to each other and the beam of particles is
directed perpendicular to both of them, so that the electrical and magnetic forces on an
ion oppose each other. The speed of those ions that are unaffected through this region is:
[a] 104 m/s [b] 105 m/s [c] 106 m/s

fiziks c/o Anand Institute of mathematics, 28-B/6 Jia Sarai


Near IIT, Hauz Khas, New Delhi, PIN- 110016 (INDIA)
Phone: 011-32718565, +91-9871145498
Website: http://www.physicsbyfiziks.com 1
Email: fiziks.physics@gmail.com
fiziks
Forum for CSIR-UGC JRF/NET, GATE, IIT-JAM, GRE in PHYSICAL SCIENCES

3. The electron in a free Hydrogen atom is initially in the state with quantum numbers
n = 3 and 1 = 2. It makes an electric dipole transition to a lower energy state. Which of
the given states could it finally be in?
[a] n = 3, 1 = 0 [b] n = 2, 1 = 2 [c] n = 2, 1 = 1

4. N particles are distributed amongst three levels having energies 0, kT and 2kT. If
the total equilibrium energy of the system is approximately 425kT, what is the value
of N?
[a] 1001 [b] 335 [c] 425 [d] 390 [e] 181

fiziks c/o Anand Institute of mathematics, 28-B/6 Jia Sarai


Near IIT, Hauz Khas, New Delhi, PIN- 110016 (INDIA)
Phone: 011-32718565, +91-9871145498
Website: http://www.physicsbyfiziks.com 2
Email: fiziks.physics@gmail.com
fiziks
Forum for CSIR-UGC JRF/NET, GATE, IIT-JAM, GRE in PHYSICAL SCIENCES

GS-2010 (Physics)
TATA INSTITUTE OF FUNDAMENTAL RESEARCH
Written Test in PHYSICS - December 19, 2009
Duration: Three hours (3 hours)

Name: ________________________________ Ref. Code: ____________

Please read all instructions carefully before you attempt the questions.

1. Please fill-in details about name, reference code etc. on the question paper and answer
sheet. The Answer Sheet is machine-readable. Read the instructions given on the reverse
of the answer sheet before you start filling it up. Use only HB pencils to fill-in the answer
sheet.

2. Indicate your ANSWER ON THE ANSWER SHEET by blackening the appropriate


circle for each question. Do not mark more than one circle for any question: this will be
treated as a wrong answer.

3. This test comes in two sections, Section A and Section B, both of which contain
multiple choice-type questions. Only ONE of the options given at the end of each
question is correct. Section A contains 20 questions, each with 4 options, and Section B
contains 8 questions, each with 6 options. The maximum marks are 60 for Section A plus
40 for Section B, totaling to 100. Marking shall be as follows:
(i) If the answer is correct: +3 marks in Section A; +5 marks in Section B
(ii) If the answer is incorrect: -1 mark in both Section A & B
(iii) If the answer is not attempted: 0 marks in both Section A & B
(iv) If more than one box is marked: 0 marks in both Section A & B
Note that negative marking as indicated above will be implemented.

fiziks c/o Anand Institute of mathematics, 28-B/6 Jia Sarai


Near IIT, Hauz Khas, New Delhi, PIN- 110016 (INDIA)
Phone: 011-32718565, +91-9871145498
Website: http://www.physicsbyfiziks.com 1
Email: fiziks.physics@gmail.com
fiziks
Forum for CSIR-UGC JRF/NET, GATE, IIT-JAM, GRE in PHYSICAL SCIENCES

4. As a rough guideline, the time spent on questions in Section A should be about 5


minutes each; questions in Section B should take about 10 minutes each. Obviously,
some questions may take a little less time while others may require a little more.

5. We advise you to first mark the correct answers on the QUESTION PAPER and then
to TRANSFER these to the ANSWER SHEET only when you are sure of your choice.

6. Rough work may be done on blank pages of the question paper. If needed, you may
ask for extra rough sheets from an Invigilator.

7. Use of calculators is permitted. Calculator which plots graphs is NOT allowed.


Multiple-use devices such as cell phones, smart phones etc., CANNOT be used for this
purpose.

8. Do NOT ask for clarifications from the invigilators regarding the questions. They have
been instructed not to respond to any such inquiries from candidates. In case a
Correction/clarification is deemed necessary; the invigilator(s) will announce it publicly.

fiziks c/o Anand Institute of mathematics, 28-B/6 Jia Sarai


Near IIT, Hauz Khas, New Delhi, PIN- 110016 (INDIA)
Phone: 011-32718565, +91-9871145498
Website: http://www.physicsbyfiziks.com 2
Email: fiziks.physics@gmail.com
fiziks
Forum for CSIR-UGC JRF/NET, GATE, IIT-JAM, GRE in PHYSICAL SCIENCES

GS-2010 (Physics)
A SECTION: 20 x 3= 60 Marks
Q.1 The matrix
1 1 0
1 1 1
0 1 1 

can be related by a similarity transformation to the matrix
1 1 1 2 1 0 
(a) 1 1 1 (b) 1 1 1
1 1 0  0 1 2 
 

1 1 0  1 1 0
(c)   1 1 1  (d)   1 1 1
0 1 1 0 1 1 
 
Q.2 A car tyre is slowly pumped up to a pressure of 2 atmospheres in an environment at
15o C. At this point, it bursts. Assuming the sudden expansion of the air (a mixture of O2
and N2) that was inside the tyre to be adiabatic, its temperature after the burst is
(a) – 55o C (b) – 37o C (c) – 26o C (d) + 9 o C
Q.3 A small meteor approaches the Earth. When it is at a large distance, it has velocity v∞ and
impact parameter b. If Re is the radius of the Earth and v0 is the escape velocity, the
condition for the meteor to strike the Earth is

(a) b  Re 1  v0 / v  (b) b  Re 1  v0 / v 


2 2

(c) b  Re 1  v0 / v 
2
(d) b  Re v0 / v 

Q.4 Consider a very, very thin wire of uniformly circular cross section. The diameter of the
wire is of the order of microns. The correct equipment required to measure the precise
value of resistivity of this wire is
(a) ammeter, voltmeter, scale, slide calipers
(b) ammeter, magnet, screw gauge, thermometer
(c) voltmeter, magnet, screw gauge, scale
(d) ammeter, voltmeter, scale, monochromatic laser source

fiziks c/o Anand Institute of mathematics, 28-B/6 Jia Sarai


Near IIT, Hauz Khas, New Delhi, PIN- 110016 (INDIA)
Phone: 011-32718565, +91-9871145498
Website: http://www.physicsbyfiziks.com 3
Email: fiziks.physics@gmail.com
fiziks
Forum for CSIR-UGC JRF/NET, GATE, IIT-JAM, GRE in PHYSICAL SCIENCES

Q.5 A function f(x) is defined in the range – 1 ≤ x ≤ 1 by


1  x for x  0
f(x) =
1  x for x  0
The first terms in the Fourier series approximating this function are
1 4 4
(a)  2 cos x  cos 3x  .......
2  9 2
1 4 4
(b)  2 sin x  2 sin 3x  .......
2  9
4 4
(c) 2
cos x  2 cos 3x  .......
 9
1 4 4
(d)  2 cos x  2 cos 3x  .......
2  9

60 60
Q.6 A lead container contains 1 gm of a 27 Co radioactive source. It is known that a 27 Co
nucleus emits a β particle of energy 316 KeV followed by two γ emissions of energy 1173
and 1333 KeV respectively. Which of the following experimental methods would be the
60
best way to determine the lifetime of this 27 Co source?
(a) Measure the change in temperature of the source
(b) Measure the weight of the source now and again after one year
(c) Measure the recoil momentum of the nucleus during β emission
(d) Measure the number of γ photons emitted by this source

Q.7 A beam of hydrogen molecules travels in the z direction with a kinetic energy of 1 eV.
The molecules are in an excited state, from which they decay and dissociate into two
hydrogen atoms. When one of the dissociated atoms has its final velocity perpendicular to
the z direction, its kinetic energy is always 0.8 eV. The energy released in the dissociative
reaction is
(a) 0.26 eV (b) 2.6 eV (c) 0.36 eV (d) 3.6 eV

fiziks c/o Anand Institute of mathematics, 28-B/6 Jia Sarai


Near IIT, Hauz Khas, New Delhi, PIN- 110016 (INDIA)
Phone: 011-32718565, +91-9871145498
Website: http://www.physicsbyfiziks.com 4
Email: fiziks.physics@gmail.com
fiziks
Forum for CSIR-UGC JRF/NET, GATE, IIT-JAM, GRE in PHYSICAL SCIENCES

Q.8 Two parallel plates of metal sandwich a dielectric pad of thickness d, forming an ideal
capacitor of capacitance C. The dielectric pad is elastic, having a spring constant k. If an
ideal battery of voltage V across its terminals is connected to the two plates of this
capacitor, the fractional change  d d  1 in the gap between the plates is
1
CV
(a) zero (b)  2 2
kd
1 1
CV 2 CV 2
(c)  22 (d)  22
kd  CV 2 kd  CV 2

Q.9 When white light is scattered from a liquid, a strong absorption line is seen at 400 nm,
and two emission lines are observed, one of which is at 500 nm, and another in the infra-
red portion of the spectrum. The wavelength of this second emission line is
(a) 900 nm (b) 2000 nm (c) 100 nm (d) 222 nm

Q10 A detector is used to count the number of γ rays emitted by a radioactive source. If the
number of counts recorded in exactly 20 seconds is 10000, the error in the counting rate
per second is
(a) ± 5.0 (b) ± 22.4 (c) ± 44.7 (d) ± 220.0

Q.11 Consider a standard chess board with 8x8 squares. A piece starts from the lower left
corner, which we shall call Square (1, 1). A single move of this piece corresponds to
either one step right, i.e. to Square (1, 2) or one step forwards, i.e. to Square (2, 1). If it
continues to move according to these rules, the number of different paths by which the
piece can reach the Square (5, 5) starting from the Square (1, 1) is
(a) 120 (b) 72 (c) 70 (d) 45

fiziks c/o Anand Institute of mathematics, 28-B/6 Jia Sarai


Near IIT, Hauz Khas, New Delhi, PIN- 110016 (INDIA)
Phone: 011-32718565, +91-9871145498
Website: http://www.physicsbyfiziks.com 5
Email: fiziks.physics@gmail.com
fiziks
Forum for CSIR-UGC JRF/NET, GATE, IIT-JAM, GRE in PHYSICAL SCIENCES

Q.12 The uppermost graph in the set below shows the variation of current v/s voltage applied
across a copper conductor at temperature T1. Which of the graphs below – marked (a),
(b), (c) or (d) – will show the possible variation of the I–V curve for the same conductor
at another temperature T2 > T1?

0 V
1 2 3 4 5
 
(a) (b)

2 2

1 1

0 V 0 V
1 2 3 4 5 1 2 3 4 5
 
(c) (d)
2 2

1 1

0 V 0 V
1 2 3 4 5 1 2 3 4 5

fiziks c/o Anand Institute of mathematics, 28-B/6 Jia Sarai


Near IIT, Hauz Khas, New Delhi, PIN- 110016 (INDIA)
Phone: 011-32718565, +91-9871145498
Website: http://www.physicsbyfiziks.com 6
Email: fiziks.physics@gmail.com
fiziks
Forum for CSIR-UGC JRF/NET, GATE, IIT-JAM, GRE in PHYSICAL SCIENCES

Q.13 A ray of light is incident on a right-angled prism as shown in the figure below. The lower
surface of this prism is coated with a gel. If the incident ray makes angles (marked in
degrees) as shown in the figure, the refractive index of the gel must be

Air Air
Prism
45
Ray 29 74 45 Ray
Gel Air

(a) 1.40 (b) 1.46 (c) 1.50 (d) 1.52

Q.14 A particle P1 is confined in a one-dimensional infinite potential well with walls at x = ±1.
Another particle P2 is confined in a one-dimensional infinite potential well with walls at
x = 0, 1. Comparing the two particles, one can conlude that
(a) the no. of nodes in the nth excited state of P1 is twice that of P2
(b) the no. of nodes in the nth excited state of P1 is half that of P2
(c) the energy of the nth level of P1 is the same as that of P2
(d) the energy of the nth level of P1 is one quarter of that of P2
Q.15 A charged particle is in the ground state of a one-dimensional harmonic oscillator
potential, generated by electrical means. If the power is suddenly switched off, so that the
potential disappears, then, according to quantum mechanics,
(a) the particle will shoot out of the well and move out towards infinity in one of the two
possible directions
(b) the particle will stop oscillating and as time increases it may be found farther and
farther away from the centre of the well
(c) the particle will keep oscillating about the same mean position but with increasing
amplitude as time increases
(d) the particle will undergo a transition to one of the higher excited states of the
harmonic oscillator

fiziks c/o Anand Institute of mathematics, 28-B/6 Jia Sarai


Near IIT, Hauz Khas, New Delhi, PIN- 110016 (INDIA)
Phone: 011-32718565, +91-9871145498
Website: http://www.physicsbyfiziks.com 7
Email: fiziks.physics@gmail.com
fiziks
Forum for CSIR-UGC JRF/NET, GATE, IIT-JAM, GRE in PHYSICAL SCIENCES

Q.16 The pV diagram given below represents a


P

A D

B C

V
(a) Carnot refrigerator (b) Carnot engine
(c) gas turbine refrigerator (d) gas turbine engine

Q.17 In the laboratory, four point charges +Q, -Q, +Q, -Q are placed at the four ends of a
horizontal square of side a, as shown in the figure below. The number of neutral points
(where the electric field vanishes) is
(a) ∞
a
(b) 4 Q Q
a a
(c) 1

(d) zero Q Q
a

Q.18 Coherent monochromatic light falling through a small aperture produced a Fraunhofer
diffraction pattern as shown below

fiziks c/o Anand Institute of mathematics, 28-B/6 Jia Sarai


Near IIT, Hauz Khas, New Delhi, PIN- 110016 (INDIA)
Phone: 011-32718565, +91-9871145498
Website: http://www.physicsbyfiziks.com 8
Email: fiziks.physics@gmail.com
fiziks
Forum for CSIR-UGC JRF/NET, GATE, IIT-JAM, GRE in PHYSICAL SCIENCES

By looking at this diffraction pattern carefully one can guess that the shape of the
aperture was
(a) (b)

(c) (d)

Q.19 In the circuit given below, a person measures 9.0 V across the battery, 3.0 V across the 2
MΏ resistor RA and 4.5 V across the unknown resistor R B, using an ordinary voltmeter
which has a finite input resistance r. Assuming that the battery has negligible internal
resistance, it follows that (i) the resistance RB and and (ii) the input resistance r of the
voltmeter are, in MΏ, R A  2

(a) RB = 3.0, r = 6.0


(b) RB = 2.5, r = 7.5 R
(c) RB = 4.0, r = 12.0
(d) RB = 4.5, r = 10.0

fiziks c/o Anand Institute of mathematics, 28-B/6 Jia Sarai


Near IIT, Hauz Khas, New Delhi, PIN- 110016 (INDIA)
Phone: 011-32718565, +91-9871145498
Website: http://www.physicsbyfiziks.com 9
Email: fiziks.physics@gmail.com
fiziks
Forum for CSIR-UGC JRF/NET, GATE, IIT-JAM, GRE in PHYSICAL SCIENCES

Q.20 A heavy mass m is suspended from two identical steel wires of length l, radius r and
Young’s modulas Y, as shown in the figure below. When the mass is pulled down by a
distance x (x << l) and released, it undergoes elastic oscillations in the vertical direction
with a time period

2 ml
(a)
r 2Y cos 2  / 2 

l cos / 2  l l
(b) 2
g 
2ml
(c)
Yr 2 m x
2 mgl
(d)
r 2Y

fiziks c/o Anand Institute of mathematics, 28-B/6 Jia Sarai


Near IIT, Hauz Khas, New Delhi, PIN- 110016 (INDIA)
Phone: 011-32718565, +91-9871145498
Website: http://www.physicsbyfiziks.com 10
Email: fiziks.physics@gmail.com
fiziks
Forum for CSIR-UGC JRF/NET, GATE, IIT-JAM, GRE in PHYSICAL SCIENCES

B SECTION: (8 x 5=40 Marks)


Q.1 The wave function ψ of a quantum mechanical system described by a Hamiltonian Ĥ
can be written as a linear combination of Φ1 and Φ2 which are the eigenfunction of Ĥ
with eigenvalues E1 and E 2 respectively. At t = 0, the system is prepared in the state
4 3
0   1   2 and then allowed to evolve with time. The wavefunction at time
5 5
1
T h / E1  E 2  will be (accurate to within a phase)
2
4 3 4 3
(a) 1   2 (b) Φ1 (c) 1   2
5 5 5 5
3 4 3 4
(d) Φ2 (e) 1   2 (f) 1   2
5 5 5 5
Q.2 Light transmitted along an optical fibre incurs losses due to Rayleigh scattering from
inhomogeneities. If a fibre of given length transmits 50% of the monochromatic light
coupled into it at a wavelength of 1350 nm, the transmitted fraction for the same fibre at
1550 nm will be
(a) 55% (b) 57% (c) 62% (d) 67% (e) 74% (f) 87%
Q.3 A quantum system has three energy levels-0.12 eV, -0.2 eV and -0.44 eV respectively.
Three electrons are distributed among these levels. At a temperature of 1727o C the
system has total energy -0.68 eV. The free energy of the system is approximately
(a) + 1.5 eV (b) +0.3 eV (c) -0.1 eV
(d) -0.3 eV (e) -1.0 eV (f) -1.5 eV
Q.4. An atom is capable of existing in two states: a ground state of mass M and an excited
state of mass M + ∆. If the transition from the ground state to the excited state proceeds
by the absorption of a photon, the photon frequency in the laboratory frame (where the
atom is initially at rest ) is
c 2 c 2    Mc 2
(a) (b) 1   (c)
h h  2M  h

c 2    Mc 2    Mc 2   
(d) 1   (e) 1   (f) 1  
h  2M  h  2M  h  2M 

fiziks c/o Anand Institute of mathematics, 28-B/6 Jia Sarai


Near IIT, Hauz Khas, New Delhi, PIN- 110016 (INDIA)
Phone: 011-32718565, +91-9871145498
Website: http://www.physicsbyfiziks.com 11
Email: fiziks.physics@gmail.com
fiziks
Forum for CSIR-UGC JRF/NET, GATE, IIT-JAM, GRE in PHYSICAL SCIENCES

Q.5 A plot of the common-emitter characteristics of a silicon n-p-n transistor is shown below.
Given this information and assuming that there will be a 0.7 V drop across a forward
biased silicon p-n junction, the approximate value of the output voltage Vout for an input
voltage Vin = 2V in the adjacent circuit will be
30
I b  50A VCC  15V
25
Collector Current Ic (mA)

I b  40A R L  500
20
R b  65k Vout
I b  30A
15 Vin
I b  20A
10
Base Current I b  10A
5

0 16
0 2 4 6 8 10 12 14
Collector-to-Emitter Voltage VCE (V)

(a) 4V (b) 6V (c) 8V


(d)10 V (e) 12V (f) 14V
Q.6 Measurement of the electric field (E) and the magnetic field (B) in a plane-polarized
electromagnetic wave in vacuum led to the following:
E E E B B B B E
  0,  and   0, 
x y z t x y z t
It follows that
(a) E  Eiˆ, B  Bˆj and the wave was travelling along k̂

(b) E  Eˆj , B  Biˆ and the wave was travelling along k̂

(c) E  Eˆj , B  Bkˆ and the wave was travelling along  iˆ

(d) E  Ekˆ, B  Biˆ and the wave was travelling along ĵ

(e) E  Eiˆ, B  Bkˆ and the wave was travelling along  ĵ

(f) the wave was travelling along  k̂ but direction of E and B are not uniquely defined

fiziks c/o Anand Institute of mathematics, 28-B/6 Jia Sarai


Near IIT, Hauz Khas, New Delhi, PIN- 110016 (INDIA)
Phone: 011-32718565, +91-9871145498
Website: http://www.physicsbyfiziks.com 12
Email: fiziks.physics@gmail.com
fiziks
Forum for CSIR-UGC JRF/NET, GATE, IIT-JAM, GRE in PHYSICAL SCIENCES

Q.7 A mass m travels in a straight line with velocity v0 perpendicular to a uniform stick of
mass m and length L, which is initially at rest. The distance from the centre of the stick to
the path of the travelling mass is h (see figure). Now the travelling mass m collides
elastically with the stick, and the centre of the stick and mass m are observed to move
with equal speed v after the collision. Assuming that the travelling mass m can be treated
mL2
as a point mass, and the moment of inertia of the stick about its center is I  , it
12
follows that the distance h must be
m 
m v

h
vo
v
L

L L L
(a) (b) (c)
2 4 6
L L
(d) (e) (f) zero
3 3

Q.8 The binding energy per nucleon for 235U is 7.6 MeV. The 235
U nucleus undergoes fission
to produce two fragments, both having binding energy per nucleon 8.5 MeV. The energy
released, in Joules, from the complete fission of 1 Kg of 235U is, therefore,
(a) 8000 (b) 10 35 (c) 450
(d) 20000 (e) 8.7 x 1013 (f) 5.0 x 108

fiziks c/o Anand Institute of mathematics, 28-B/6 Jia Sarai


Near IIT, Hauz Khas, New Delhi, PIN- 110016 (INDIA)
Phone: 011-32718565, +91-9871145498
Website: http://www.physicsbyfiziks.com 13
Email: fiziks.physics@gmail.com
PHY - X
GS-2011 (Physics)
TATA INSTITUTE OF FUNDAMENTAL RESEARCH
Written Test in PHYSICS
December 12, 2010
Duration : Three hours (3 hours)

Please read all instructions carefully before you attempt the questions.

1. Please fill-in details about name, reference code etc. on the answer sheet. The Answer Sheet is
machine-readable. Read the instructions given on the reverse of the answer sheet before you
start filling it up. Use only HB pencils to fill-in the answer sheet.

2. Indicate your ANSWER ON THE ANSWER SHEET by blackening the appropriate circle for each
question. Do not mark more than one circle for any question : this will be treated as a wrong
answer.

3. This test comes in two sections, Section A and Section B, both of which contain multiple choice-
type questions. Only ONE of the options given at the end of each question is correct. Section A
contains 20 questions, each with 4 options, and Section B contains 8 questions, each with 6
options. The maximum marks are 60 for Section A plus 40 for Section B, totalling to 100.
Marking shall be as follows :
(i) If the answer is correct : +3 marks in Section A; +5 marks in Section B
(ii) If the answer is incorrect : -1 mark in both Section A & B
(III) If the answer is not attempted : 0 marks in both Section A & B
(iv) If more than one box is marked : 0 marks in both Section A & B
Guesswork and marking of random choices may lead to reduced scores in the test.

4. As a rough guideline, the time spent on questions in Section A should be about 5 minutes each;
questions in Section B should take about 10 minutes each. Obviously, some questions may take
a little less time while others may require a little more.

5. We advise you to first mark the correct answers on the QUESTION PAPER and then to
TRANSFER these to the ANSWER SHEET only when you are sure of your choice.

6. Rough work may be done on blank pages of the question paper. If needed, you may ask for
extra rough sheets from an Invigilator.

7. Use of calculators is permitted. Calculator which plots graphs is NOT allowed. Use of
wristwatches and electronic calculators is permitted, but multiple-use devices such as cell
phones, smartphones etc., CANNOT be used for this purpose.

8. Do NOT ask for clarifications from the invigilators regarding the questions. They have been
instructed not to respond to any such inquiries from candidates. In case a
correction/clarification is deemed necessary, the invigilator(s) will announce it publicly.

9. List of useful physical constants is given on the next sheet.


Useful Constants and Unit Conversions
Symbol Definition/Name Value

c Speed of light in vacuum 3.0 × 108 m s−1


~ reduced Planck constant (h/2π) 1.05 × 10−34 J s
GN Gravitational constant 6.67 × 10−11 m3 Kg−1 s−2
e Electron charge (magnitude) 1.60 × 10−19 C
ǫ0 Permittivity of free space 8.85 × 10−12 F m−1
µ0 permeability of free space 4π × 10−7 N A−2
kB Boltzmann constant 8.62 × 10−5 eV K−1 = 1.38 × 10−23 J K−1
me Electron mass 0.5 MeV/c2
mn Neutron mass ≈ 2000 me ≈ mp
mp Proton mass ≈ 2000 me ≈ mn
NA Avogadro number 6.023 × 1023 mol−1
R = kB NA Gas constant 8.31 J mol−1 K−1
γ = CP /CV ratio of specific heats : monatomic gas 1.67
: diatomic gas 1.40
g Acceleration due to gravity (sea level) 9.8 m s−1
Re Radius of the Earth 6 400 Km
Rs Radius of the Sun 700 000 Km
σ Stefan-Boltzmann constant 5.67 × 10−8 W m−2 K−4
~c Conversion constant 197 MeV fm = 3.16 × 10−26 J m
α = e2 /4πǫ0 ~c Fine structure constant 1/137
a0 = 4πǫ0 ~2 /e2 me Bohr radius 0.51 Å
Ionisation energy of H atom 13.6 eV

1 Å Ångstrom unit 10−10 m


1 eV electron Volt 1.60 × 10−19 J
1T Tesla 104 Gauss
1W Watt 1 J s−1
1 bar atmospheric pressure 1.01 × 105 Pa = 1.01 × 105 N m−2
1 a.m.u. atomic mass unit 931.49 MeV/c2
Rough Work
GS-2011 (Physics) X

A Marks 20 × 3 = 60
Time: 100 minutes (approx.)

A1. The infinite series


x3 x5 x7
+
x+ + + ...
3 5 7
where −1 < x < +1, can be summed to the value

(b) ln 1− π4 tan−1 x (c) 12 ln (1+x)/(1−x) (d) 21 ln (1−x)/(1+x)


    
(a) tanh x

A2. A 100 page book is known to have 200 printing errors distributed randomly through
the pages. The probability that one of the pages will be found to be completely free of
errors is closest to

(a) 67 % (b) 50 % (c) 25 % (d) 13 %

A3. Consider the matrix  


1 0 0
M= 0 0 −1 
0 −1 0
A 3-dimensional basis formed by eigenvectors of M is
           
2 1 0 1 0 0
(a)  1 ,  −1  and  1  (b)  1 ,  1  and  1 
−1 1 −1 −1 1 −1

           
0 0 0 2 0 −1
(c)  0 ,  1  and  1  (d)  1 ,  1  and  1 
0 1 −1 −1 1 1

A4. Two solid spheres S1 and S2 of the same uniform density fall from rest under gravity in
a viscous medium and, after some time, reach terminal velocities v1 and v2 respectively.
If the masses of S1 and S2 are m1 and m2 respectively, and v1 = 4 v2 , then the ratio
m1 /m2 is

(a) 1/8 (b) 1/4 (c) 4 (d) 8

X-1
A5. The dynamics of a particle of mass m is described in terms of three generalized coor-
dinates ξ, η and ϕ. If the Lagrangian of the system is
" ! #
1 ξ˙2
η̇ 2
1
L = m (ξ + η) 2
+ 2 + 4 ξ η ϕ̇2 + k (ξ + η)2
8 ξ η 8

where k is a constant, then a conserved quantity in the system will be

(a) (m+ k) (ξ˙ + η̇)  (b) m ξ η ϕ̇  


(c) m ξ˙2/η 2 + η̇ 2 /ξ 2 ˙ 2 + η̇/η 2
(d) m (ξ + η) ξ/ξ

A6. A scientist is given two heavy spheres made of the same metal, which have the same
diameter and weight, and is asked to distinguish the spheres, without damaging them in
any way. Though the spheres look identical, one of them is actually a hollow spherical
shell, while the other is a set of concentric shells mounted on four thin rods of the same
metal (see figure).

To make this distinction, the scientist must perform an experiment where each sphere
is
(a) set rotating under the action of a constant torque
(b) made into the bob of a long simple pendulum and set oscillating
(c) immersed fully in a non-corrosive liquid and then weighed
(d) given the same electric charge Q and the potential is measured

A7. A narrow beam of light of wavelength 589.3 nm from a sodium lamp is incident normally
on a diffraction grating of transmission type. If the grating constant is 1 000 000 m−1 ,
the number of principal maxima observed in the transmitted light will be

(a) 7 (b) 5 (c) 3 (d) 1

A8. A closed, thermally-insulated box contains one mole of an ideal monatomic gas G in
thermodynamic equilibrium with blackbody radiation B. The total internal energy of
the system is U = UG + UB where UG and UB (∝ T 4 ) are the energies of the ideal gas
and the radiation respectively. If UG = UB at a certain temperature T0 K, then the
energy required to raise the temperature from T0 K to (T0 + 1) K, in terms of the gas
constant R, is

(a) 7.5R (b) 6 R (c) 1.5 R (d) 0.33R

X-2
A9. The phase diagram of a pure substance is given in the figure below, where ‘T’ denotes
the triple point and ‘C’ denotes the critical point.

Temperature
γ

T β

Pressure

The phase transitions occurring along the lines marked α, β and γ are

(a) α = melting; β = condensation; γ = sublimation


(b) α= sublimation; β = vaporisation; γ = melting;
(c) α= melting; β = vaporisation; γ = condensation
(d) α= sublimation; β = melting; γ = vaporisation

A10. The current read by the ammeter (A) in the circuit given below is
6V
5 5

100 100
A
100 100

100
(a) 27.3 mA (b) 100.0 mA (c) 54.5 mA (d) 50.0 mA

A11. The sign of the majority charge carriers in a doped silicon crystal is to be determined
experimentally. In addition to a voltage supply, the combination of instruments needed
to perform the experiment is

(a) Thermometer, Voltmeter and Ammeter


(b) Pickup Coil, Voltmeter and Ammeter
(c) Magnet, Voltmeter and Ammeter
(d) Heater, Magnet and Thermometer

X-3
A12. A small but very powerful bar magnet falls from rest under gravity through the centre
of a horizontal ring of conducting wire, as shown in the figure below (on the left). The
speed-versus-time graph, in arbitrary units, of the magnet will correspond most closely
to which of the four plots below (on the right)?

(a) (b)

Speed
Speed
0 0
Time Time

(c) (d)

Speed
Speed

0 0
Time Time

A13. The spectra of electromagnetic radiation emitted by distant objects like stars and galax-
ies give important clues about their physical properties. In this context, a correct state-
ment is that

(a) the nuclear structure of the distant objects cannot be determined from lines in the
visible region of the spectrum
(b) absorption lines in the spectra of distant objects do not carry information about
their motion in a direction transverse to the line of sight
(c) the wavelengths in the emission spectrum of an element in a star are always the
same as those found in laboratory experiments
(d) absorption spectra cannot be used to determine which molecules are present in the
distant objects

A14. Given that the ionization energies of Hydrogen (1 H) and Lithium (3 Li) are 13.6 eV and
5.39 eV, respectively, the effective nuclear charge experienced by the valence electron
of a 3 Li atom may be estimated in terms of the proton charge e as

(a) 0.63 e (b) 1.26 e (c) 1.59 e (d) 3.00 e

A15. Two identical non-interacting particles, each of mass m and spin 21 , are placed in a one-
dimensional box of length L. In quantum mechanics, the lowest possible value of the
total energy of these two particles is ǫ0 . If, instead, four such particles are introduced
into a similar one-dimensional box of length 2L, then the lowest possible value of their
total energy will be

(a) 2ǫ0 (b) 5ǫ0 /4 (c) 3ǫ0 /2 (d) ǫ0

X-4
A16. An excited atomic electron undergoes a spontaneous transition

3d3/2 → 2p1/2

The interaction responsible for this transition must be of the type

(a) electric dipole (E1) OR magnetic quadrupole (M2)


(b) electric dipole (E1) OR magnetic dipole (M1)
(c) electric quadrupole (E2) OR magnetic quadrupole (M2)
(d) electric quadrupole (E2) OR magnetic dipole (M1)

A17. A fast-moving 14 N nucleus collides with an α particle at rest in the laboratory frame,
giving rise to the reaction
14
N + α → 17 O + p
Given the masses 14.00307 a.m.u. and 16.99913 a.m.u. for 14 N and 17 O nuclei respec-
tively, and 4.00260 a.m.u. and 1.00783 a.m.u. for α and p respectively, the minimum
kinetic energy in the laboratory frame of the 14 N nucleus must be

(a) 4.20 MeV (b) 1.20 MeV (c) 5.41 MeV (d) 1.55 MeV

A18. An unmagnetised sample of iron is placed in a magnetic field H which varies with time
as shown in the plot below.
0.6
H (kilo oersted)

0.3
0.0
-0.3
-0.6
0 2 4 6 8
Time (min)
The magnetisation M of this iron sample is continuously measured and also plotted as
a function of time. The appearance of this plot will be closest to

(a) (b) (c) (d)


+1 +1 +1 +1
M (arb. units)

0 0 0 0

-1 -1 -1 -1
0 2 4 6 8 0 2 4 6 8 0 2 4 6 8 0 2 4 6 8
Time (min) Time (min) Time (min) Time (min)

X-5
A19. The figure below shows the Bragg diffraction pattern for X-rays of wavelength 1.54 Å
incident on two crystalline Silicon thin film Samples A and B. The dashed line corre-
sponds to a normal Sample A and the continuous line corresponds to another Sample
B, which is modified due to differences in the growth conditions.

Diffracted X-ray intensity (arb. units)


normal
!X-ray=1.54Ao z (A) modified
2 (B)

Si Sample

28 29 30
2 (deg)
These plots suggest that the modified sample B is
(a) stretched in all directions by 3%
(b) compressed in all directions by 3%
(c) stretched in the z direction by 1% and possibly compressed in x & y directions
(d) compressed in the z direction by 1% and possibly stretched out in x & y directions

A20. The digital electronic circuit shown below (left side) has some problem and is not
performing as intended. The voltage at each pin as a function of time is shown in the
adjacent figures.
+5V
Pin 1
0

+5V
Pin 4
0

1 2 4
+5V
A 6 9 8
5 Y Pin 5 0
B
+5V
Pin 6 0

+5V

Pin 8
0

The problem in the about circuit may be that

(a) the Pin 6 is shorted to ground (b) the input inverter is shorted
(c) the Pin 8 is clamped to +5 V (d) OR gate is used instead of AND gate

X-6
GS-2011 (Physics) X

B Marks 8 × 5 = 40
Time: 80 minutes (approx.)

B1. The trace of the real 4 × 4 matrix U = exp(A), where


 
0 0 0 π/4
 0 0 −π/4 0 
A=  0

π/4 0 0 
−π/4 0 0 0
is equal to

(a) 2 2 (b) π/4 (c) exp(iϕ) for ϕ = 0, π
(d) zero (e) π/2 (f) 2

B2. A region of space is divided into two parts by a plane P, as shown in the figure below.
A particle of mass m passes from Region I to Region II, where it has speed v1 and v2
respectively. There is a constant potential U1 in Region I and U2 in Region II.
P
Region I Region II

v2
θ2
θ1

v1
U1 U2

Let T1 be the kinetic energy of the particle in Region I. If the trajectory of the particle
is inclined to the normal to the plane P by angles θ1 and θ2 , as shown in the figure then
the ratio sin θ1 / sin θ2 is given by

p p p
(a) 1 − T1 /(U1 − U2 ) (b) 1 + T1 /(U1 − U2 ) (c) 1 − (U1 + U2 )/T1
p p p
(d) 1 + (U1 + U2 )/T1 (e) 1 − T1 /(U1 − U2 ) (f) 1 + (U1 − U2 )/T1

X-7
B3. The electric field of an electromagnetic wave of angular frequency ω propagating in a
medium with conductivity σ, permittivity ǫ and permeability µ is given by
 
E = E0 exp − i(ωt − kx)
p
where the imaginary part of the complex propagation constant k is ω µǫ/2 multiplied
by the factor
q 1/2 hp i1/2 hp i1/2
p
(a) 1 + σ/ωǫ + 1 (b) 1 + σ/ωǫ + 1 (c) 1 + (σ/ωǫ)2 − 1

hp i1/2 q 1/2 hp i1/2


p
(d) 1 + (σ/ωǫ)2 + 1 (e) 1+ σ/ωǫ − 1 (f) 1 + σ/ωǫ − 1

B4. A system having N non-degenerate energy eigenstates is populated by N identical spin-


zero particles and 2N identical spin-half particles. There are no interactions between
any of these particles. If N = 1000, the entropy of the system is closest to

(a) 13.82 kB (b) zero (c) 693.1 kB


(d) 1000 kB (e) 5909.693 kB (f) 6909 kB

B5. The Michelson interferometer in the figure below can be used to study properties of
light emitted by distant sources.

Fixed Mirror
Movable
Mirror
Beam Splitter
Source

PhotoDetector

A Source S1 , when at rest, is known to emit light of wavelength 632.8 nm. In this
case, if the Movable Mirror is translated through a distance d, it is seen that 99,565
interference fringes pass across the Photo-Detector. For another Source S2 , moving at
an uniform speed of 1.5 × 107 m s−1 towards the interferometer along the straight line
joining it to the Beam Splitter, one sees 100,068 interference fringes pass across the
Photo-Detector for the same displacement d of the Movable Mirror. It follows that S2 ,
in its own rest frame, must be emitting light of wavelength

(a) 661.9 nm (b) 662.8 nm (c) 598.9 nm


(d) 631.2 nm (e) 599.6 nm (f) 628.0 nm

X-8
B6. A particle of mass m is placed in the ground state of a one-dimensional harmonic
oscillator potential of the form
1
V (x) = kx2
2
where the stiffness constant k can
√ be varied externally. The ground state wavefunction
has the form ψ(x) ∝ exp(−ax2 k) where a is a constant. If, suddenly, the parameter
k is changed to 4k, the probability that the particle will remain in the ground state of
the new potential is

(a) 0.47 (b) 0.06 (c) 0.53


(d) 0.67 (e) 0.33 (f) 0.94

B7. A Cloud Chamber of width 0.01 m is filled with pure nitrogen gas (N2 ) at normal tem-
perature and pressure. A beam of α particles, when incident normally on the chamber,
make tracks which are visible under strong illumination. Whenever an α particle (42 He)
has a nuclear collision with a 147 N nucleus, the track shows a distinct bend. The radius
of a nucleus is given by r = r0 A1/3 where r0 = 1.217 × 10−15 m and A is the atomic
mass number. If the α particles move at non-relativistic speeds, and the total number
of incident α particles is 107 , the number of such distinct bends is approximately

(a) 100 (b) 200 (c) 300


(d) 400 (e) 500 (f) 600

B8. The three electronic circuits marked (i), (ii) and (iii) in the figure below can all work
as logic gates, where the input signals are either 0 V or 5 V and the output is VO .

(i) (ii) (iii)


+V
+V
A
VO
B
A
A VO
B
B

VO

Identify the correct combination of logic gates (i), (ii), (iii) in the options given below.

(a) NOR, XOR, AND (b) OR, NAND, NOR (c) NAND, AND, XOR
(d) XOR, AND, NAND (e) AND, OR, NOR (f) NOR, NAND, OR

X-9
Rough Work

X-10
Correct Answers are ticked in green.
PHY - X
GS-2012 (Physics)
TATA INSTITUTE OF FUNDAMENTAL RESEARCH
Written Test in PHYSICS - December 11, 2011
Duration : Three hours (3 hours)

Name : _______________________________________________ Ref. Code : ____________

Please read all instructions carefully before you attempt the questions.

1. Please fill-in details about name, reference code etc. on the question paper and answer sheet.
The Answer Sheet is machine-readable. Read the instructions given on the reverse of the
answer sheet before you start filling it up. Use only HB pencils to fill-in the answer sheet.

2. Indicate your ANSWER ON THE ANSWER SHEET by blackening the appropriate circle for each
question. Do not mark more than one circle for any question : this will be treated as a wrong
answer.

3. This test comes in two sections, Section A and Section B, both of which contain multiple choice-
type questions. Only ONE of the options given at the end of each question is correct. Section A
contains 20 questions, each with 4 options, and Section B contains 10 questions, each with 5
options. The maximum marks are 60 for Section A plus 40 for Section B, totaling to 100.
Marking shall be as follows :
(i) If the answer is correct : +3 marks in Section A; +4 marks in Section B
(ii) If the answer is incorrect : -1 mark in both Section A & B
(III) If the answer is not attempted : 0 marks in both Section A & B
(iv) If more than one box is marked : 0 marks in both Section A & B
Note that negative marking as indicated above will be implemented.

4. As a rough guideline, the time spent on questions in Section A should be about 5 minutes each;
questions in Section B should take about 8 minutes each. Obviously, some questions may take
a little less time while others may require a little more.

5. We advise you to first mark the correct answers on the QUESTION PAPER and then to
TRANSFER these to the ANSWER SHEET only when you are sure of your choice.

6. Rough work may be done on blank pages of the question paper. If needed, you may ask for
extra rough sheets from an Invigilator.

7. Use of calculators is permitted. Calculator which plots graphs is NOT allowed. Multiple-use
devices such as cell phones, smartphones etc., CANNOT be used for this purpose.

8. Do NOT ask for clarifications from the invigilators regarding the questions. They have been
instructed not to respond to any such inquiries from candidates. In case a
correction/clarification is deemed necessary, the invigilator(s) will announce it publicly.

9. List of useful physical constants is given on the next sheet.


Useful Constants and Unit Conversions
Symbol Definition/Name Value

c Speed of light in vacuum 3.0 × 108 m s−1


~ reduced Planck constant (h/2π) 1.05 × 10−34 J s
GN Gravitational constant 6.67 × 10−11 m3 Kg−1 s−2
e Electron charge (magnitude) 1.60 × 10−19 C
ǫ0 Permittivity of free space 8.85 × 10−12 F m−1
µ0 permeability of free space 4π × 10−7 N A−2
kB Boltzmann constant 8.62 × 10−5 eV K−1 = 1.38 × 10−23 J K−1
= 0.7 cm−1 K−1
me Electron mass 0.5 MeV/c2 = 9.1 × 10−31 Kg
mn Neutron mass 939.6 MeV
mp Proton mass 938.2 MeV
NA Avogadro number 6.023 × 1023 mol−1
R = kB NA Gas constant 8.31 J mol−1 K−1
γ = CP /CV ratio of specific heats : monatomic gas 1.67
: diatomic gas 1.40
g Acceleration due to gravity (sea level) 9.8 m s−1
Re Radius of the Earth 6 400 Km
Rs Radius of the Sun 700 000 Km
σ Stefan-Boltzmann constant 5.67 × 10−8 W m−2 K−4
~c Conversion constant 197 MeV fm = 3.16 × 10−26 J m
α = e2 /4πǫ0 ~c Fine structure constant 1/137
a0 = 4πǫ0 ~2 /e2 me Bohr radius 0.51 Å
Ionisation energy of H atom 13.6 eV
r0 nuclear radius r = r0 A1/3 1.2 fm

1 Å Ångstrom unit 10−10 m


1 eV electron Volt 1.60 × 10−19 J
1T Tesla 104 Gauss
1W Watt 1 J s−1
1 bar atmospheric pressure 1.01 × 105 Pa = 1.01 × 105 N m−2
1 a.m.u. atomic mass unit 1.66 × 10−27 Kg
GS-2012-X (Physics)

A Section
Marks: 20 × 3 = 60 Time: 100 minutes (approx.)

A1. Two different 2 × 2 matrices A and B are found to have the same eigenvalues. It is
then correct to state that A = SBS −1 where S can be a

(a) traceless 2 × 2 matrix (b) Hermitian 2 × 2 matrix


(c) unitary 2 × 2 matrix (d) arbitrary 2 × 2 matrix

A2. The function f (x) represents the nearest integer less than x, e.g.
f (3.14) = 3 .
The derivative of this function (for arbitrary x) will be given in terms of the integers
n as f ′ (x) =
P P P
(a) 0 (b) n δ(x − n) (c) n |x − n| (d) n f (x − n)

A3. Two masses M1 and M2 (M1 < M2 ) are suspended from a perfectly rigid horizontal
support by a system of three taut massless wires W1 , W2 and W3 , as shown in the
figure. All the three wires have identical cross-sections and elastic properties and are
known to be very strong.
11111111111111111111111
00000000000000000000000
00000000000000000000000
11111111111111111111111
00000000000000000000000
11111111111111111111111
30 0
W1 60 0 W2

M1

W3

M2

If the mass M2 is increased gradually, but without limit, we should expect the wires to
break in the following order:

(a) first W2 , then W1 (b) first W1 , then W2


(c) first W2 , then W3 (d) first W3

1
A4. A high-velocity missile, travelling in a horizontal line with a kinetic energy of 3.0 Giga-
Joules (GJ), explodes in flight and breaks into two pieces A and B of equal mass. One
of these pieces (A) flies off in a straight line perpendicular to the original direction in
which the missile was moving and its kinetic energy is found to be 2.0 GJ. If gravity
can be neglected for such high-velocity projectiles, it follows that the other piece (B)
flew off in a direction at an angle with the original direction of

(a) 30◦ (b) 33◦ 24′ (c) 45◦ (d) 60◦

A5. Consider a spherical planet, rotating about an axis passing through its centre. The
velocity of a point on its equator is veq . If the acceleration due to gravity g measured
at the equator is half of the value of g measured at one of the poles, then the escape
velocity for a particle shot upwards from that pole will be
√ √
(a) veq /2 (b) veq / 2 (c) 2 veq (d) 2 veq

A6. A dynamical system with two degrees of freedom, has generalised coordinates q1 and
q2 , and kinetic energy
T = λ q̇1 q̇2
If the potential energy is V (q1 , q2 ) = 0, the correct form of the Hamiltonian for this
system is

(a) p1 p2 /λ (b) λ q̇1 q̇2


(c) (p1 q̇1 + p2 q̇2 )/2 (d) (p1 q2 + p2 q1 )/2

A7. An ideal liquid of density 1 gm/cc is flowing at a rate of 10 gm/s through a tube with
varying cross-section, as shown in the figure.

B
A

Two pressure gauges attached at the points A and B (see figure) show readings of PA
and PB respectively. If the radius of the tube at the points A and B is 0.2 cm and
1.0 cm respectively, then the difference in pressure (PB − PA ), in units of dyne cm−2 ,
is closest to

(a) 100 (b) 120 (c) 140 (d) 160

2
A8. Unpolarised light of intensity I0 passes successively through two identical linear po-
larisers A and B, placed such that their polarisation axes are at an angle of 45◦ (see
figure) with respect to one another.

450
I0 IT

A B

Assuming A and B to be perfect polarisers (i.e. no absorption losses), the intensity of


the transmitted light will be IT =
√ √
(a) I0 /4 (b) I0 /2 2 (c) I0 /2 (d) I0 / 2

A9. Three equal charges Q are successively brought from infinity and each is placed at one
of the three vertices of an equilateral triangle. Assuming the rest of the Universe as a
whole to be neutral, the energy E0 of the electrostatic field will increase, successively,
to
E0 + ∆1 , E0 + ∆1 + ∆2 , E0 + ∆1 + ∆2 + ∆3
where ∆1 : ∆2 : ∆3 =

(a) 1 : 2 : 3 (b) 1 : 1 : 1 (c) 0 : 1 : 1 (d) 0 : 1 : 2

A10. Five sides of a hollow metallic cube are grounded and the sixth side is insulated from
the rest and is held at a potential Φ (see figure).

11111
00000
00000
11111
00000
11111
00000
11111 Φ
O11111
00000
00000
11111
00000
11111
00000
11111
00000
11111

The potential at the center O of the cube is

(a) 0 (b) Φ/6 (c) Φ/5 (d) 2 Φ/3

3
A11. Consider a sealed but thermally conducting container of total volume V , which is in
equilibrium with a thermal bath at temperature T . The container is divided into two
equal chambers by a thin but impermeable partition. One of these chambers contains

0110
an ideal gas, while the other half is a vacuum (see figure).

0000
11111010
0000
1111
0000
1111 10
0000
1111
0000
1111
0000
1111
gas
0000
1111
0000
1111
vacuum

0000
1111
0000
1111
0000
1111
0000
1111
If the partition is removed and the ideal gas is allowed to expand and fill the entire
container, then the entropy per molecule of the system will increase by an amount

(a) 2kB (b) kB ln (1/2) (c) kB ln 2 (d) (kB ln 2)/2

A12. When a gas is enclosed in an impermeable box and heated to a high temperature T ,
some of the neutral atoms lose an electron and become ions. If the number density of
neutral atoms, ions and electrons is Na , Ni and Ne , respectively, these can be related
to the average volume Va occupied by an atom/ion and the ionisation energy E by the
relation

(a) Ne (Na + Ni ) = (Na /Va ) exp (−E/kB T )


(b) Na (Ne + Ni ) = (Na /Va ) exp (−E/kB T )
(c) Ne Ni = (Na /Va ) exp (+E/kB T )
(d) Ne Ni = (Na /Va ) exp (−E/kB T )

A13. In a scanning tunnelling microscope, a fine Platinum needle is held close to a metallic
surface in vacuum and electrons are allowed to tunnel across the tiny gap δ between
the surface and the needle. The tunnelling current I is related to the gap δ, through
positive constants a and b, as

(a) I = a − b δ (b) I = a + b δ
(c) log I = a − b δ (d) log I = a + b δ

4
A14. A particle in a one-dimensional potential has the wavefunction
 
1 −|x|
ψ(x) = √ exp
a a
where a is a constant. It follows that for a positive constant V0 , the potential V (x) =

(a) V0 x2 (b) V0 |x| (c) −V0 δ(x) (d) −V0 /|x|

A15. Consider the high excited states of a Hydrogen atom corresponding to large values of
the principal quantum number (n ≫ 1). The wavelength λ of a photon emitted due to
an electron undergoing a transition between two such states with consecutive values of
n (i.e. ψn+1 → ψn ) is related to the wavelength λα of the Kα line of Hydrogen by

(a) λ = n3 λα /8 (b) λ = 3n3 λα /8


(c) λ = n2 λα (d) λ = 4λα /n2

A16. A proton is accelerated to a high energy E and shot at a nucleus of Oxygen (168 O). In
order to penetrate the Coulomb barrier and reach the surface of the Oxygen nucleus,
E must be at least

(a) 3.6 MeV (b) 1.8 MeV (c) 45 keV (d) 180 eV

A17. A monochromatic beam of X-rays with wavelength λ is incident at an angle θ on a


crystal with lattice spacings a and b as sketched in the figure below.

θ θ

A condition for there to be a maximum in the diffracted X-ray intensity is



(a) 2 a2 + b2 sin θ = λ (b) 2 b cos θ = λ
(c) 2 a cos θ = λ (d) (a + b) sin θ = λ

5
A18. Suppose the energy band diagram of a certain pure crystalline solid is as shown in the
figure below, where the energy (E) varies with crystal momentum (k) as E ∝ k 2 .

At finite temperatures the bottom of the conduction band (CB) is partially filled with
electrons (e) and the top of the valence band (VB) is partially filled with holes (h).
If an electric field is applied to this solid, both e and h will start moving. If the time
between collisions is the same for both e and h, then

(a) e and h will move with the same speed in opposite directions
(b) h will on an average achieve higher speed than e
(c) e will on an average achieve higher speed than h
(d) e and h will recombine and after a while there will be no flow of charges

A19. Consider the circuit shown below.

A
Y
B
The minimum number of NAND gates required to design this circuit is

(a) 6 (b) 5 (c) 4 (d) 3

6
A20. Consider the following circuit:
R

C
V in −
V out
+

If the waveform given below is fed in at Vin ,


V in
+VP

−VP

then the waveform at the output Vout will be


V out V out
(a) (b)

V out V out
(c) (d)

7
GS-2012-X (Physics)

B Section
Marks: 10 × 4 = 40 Time: 80 minutes (approx.)

B1. Consider the integral


+p2
dx
Z
p
−p2 x2 − p2
where p is a constant. This integral has a real, nonsingular value if

(a) p < −1 (b) p > 1 (c) p = 1 (d) p → 0 (e) p → ∞

B2. If we model the electron as a uniform sphere of radius re , spinning uniformly about an
axis passing through its centre with angular momentum Le = ~/2, and demand that
the velocity of rotation at the equator cannot exceed the velocity c of light in vacuum,
then the minimum value of re is

(a) 19.2 fm (b) 0.192 fm (c) 4.8 fm (d) 1960 fm (e) 480 fm

B3. The intensity of light coming from a distant star is measured using two identical in-
struments A and B, where A is placed in a satellite outside the Earth’s atmosphere,
and B is placed on the Earth’s surface. The results are as follows:

colour wavelength intensity at A intensity at B


(nm) (nanoWatts) (nanoWatts)
green 500 100 50
red 700 200 x

Assuming that there is scattering, but no absorption of light in the Earth’s atmosphere
at these wavelengths, the value of x can be estimated as

(a) 137 (b) 147 (c) 157 (d) 167 (e) 177

8
B4. Consider three identical infinite straight wires A, B and C arranged in parallel on a
plane as shown in the figure.

x
I I
I

d d

A B C

The wires carry equal currents I with directions as shown in the figure and have mass
per unit length m. If the wires A and C are held fixed and the wire B is displaced by
a small distance x from its position, then it (B) will execute simple harmonic motion
with a time period
q q q
m d 2πm d πm d
  
(a) 2π πµ 0 I
(b) 2π µ0 I
(c) 2π µ0 I
q q
m d
(e) 2π µm0 Id
 
(d) 2π 2πµ 0 I

B5. The normalized wavefunctions of a Hydrogen atom are denoted by ψn,ℓ,m (~x), where
n, ℓ and m are, respectively, the principal, azimuthal and magnetic quantum numbers
respectively. Now consider an electron in the mixed state
1 2 2
Ψ(~x) = ψ1,0,0 (~x) + ψ2,1,0 (~x) + ψ3,2,−2 (~x)
3 3 3
The expectation value hEi of the energy of this electron, in electron-Volts (eV) will be
approximately

(a) −1.5 (b) −3.7 (c) −13.6 (d) −80.1 (e) +13.6

B6. The strongest three lines in the emission spectrum of an interstellar gas cloud are found
to have wavelengths λ0 , 2λ0 and 6λ0 respectively, where λ0 is a known wavelength. From
this we can deduce that the radiating particles in the cloud behave like

(a) free particles (b) particles in a box (c) harmonic oscillators


(d) rigid rotators (e) hydrogenic atoms

9
B7. When light is emitted from a gas of excited atoms, the lines in the spectrum are
Doppler-broadened due to the thermal motion of the emitting atoms.
The Doppler width of an emission line of wavelength 500 nanometres (nm) emitted by
an excited atom of Argon (40
20 A) at room temperature (27 C) can be estimated as

(a) 5.8 × 10−4 nm (b) 3.2 × 10−4 nm (c) 3.2 × 10−3 nm


(d) 2.5 × 10−3 nm (e) 1.4 × 10−3 nm

B8. In a nuclear reactor, Plutonium (239


94 Pu) is used as fuel, releasing energy by its fission
into isotopes of Barium ( 54 Ba) and Strontium (91
146
38 Sr) through the reaction

239
94 Pu + 10 n −→146
56 Ba +
91
38 Sr + 3 × 10 n

The binding energy (B.E.) per nucleon of each of these nuclides is given in the table
below:
239 146 91
Nuclide 94 Pu 54 Ba 38 Sr

B.E. per nucleon (MeV) 7.6 8.2 8.6

Using this information, one can estimate the number of such fission reactions per second
in a 100 MW reactor as

(a) 3.9 × 1018 (b) 7.8 × 1018 (c) 5.2 × 1019


(d) 5.2 × 1018 (e) 8.9 × 1017

B9. Metallic Copper is known to form cubic crystals and the lattice constant is measured
from X-ray diffraction studies to be about 0.36 nm. If the specific gravity of Copper is
8.96 and its atomic weight is 63.5, one can conclude that

(a) the crystals are of simple cubic type


(b) the crystals are of b.c.c. type
(c) the crystals are of f.c.c. type
(d) the crystals are a mixture of f.c.c. and b.c.c. types
(e) there is insufficient data to distinguish between the previous options

10
B10. The voltage regulator circuit shown in the figure has been made with a Zener diode
rated at 15 V, 200 mW. It is required that the circuit should dissipate 150 mW power
across the fixed load resistor RL .

238 Ω

RL
Vi VO

For stable operation of this circuit, the input voltage Vi must have a range

(a) 17.5 V — 20.5 V (b) 15.5 V — 20.5 V


(c) 15.5 V — 22.5 V (d) 17.5 V — 22.5 V
(e) 15.0 V — 22.5 V

Rough Work

11
GS-2012 (PHYSICS) PHY-X
ANSWER SHEET
Please see reverse for instructions on filling of answer sheet.

Question Paper Set : X Y

Name Reference Code :


Ref Code 1 { { { { {
Address 2 { { { { {
3 { { { { {
4 { { { { {
5 { { { { {
Phone 6 { { { { {
Email 7 { { { { {
8 { { { { {
Would you like to be considered for Biology also : 9 { { { { {
YES { NO { 0 { { { { {

SECTION-A SECTION-B
(a) (b) (c) (d) (a) (b) (c) (d) (e)
1 { { { { 1 { { { { {
2 { { { { 2 { { { { {
3 { { { { 3 { { { { {
4 { { { { 4 { { { { {
5 { { { { 5 { { { { {
6 { { { { 6 { { { { {
7 { { { { 7 { { { { {
8 { { { { 8 { { { { {
9 { { { { 9 { { { { {
10 { { { { 10 { { { { {
11 { { { {
12 { { { {
13 { { { {
14 { { { {
15 { { { {
16 { { { {
17 { { { {
18 { { { {
19 { { { { ____________________
Signature of the Student
20 { { { {
INSTRUCTIONS

The Answer Sheet is machine-readable. Apart from filling in the details on the answer sheet,
please make sure that the Reference Code is filled by blackening the appropriate circles in
the box provided on the right-top corner. Only use HB pencils to fill-in the answer sheet.

e.g. if your reference code is 15207 :

Also, the multiple choice questions are to be answered by blackening the appropriate circles
as described below

e.g. if your answer to question 1 is (b)


and your answer to question 2 is (d)
then ..........
PHY - X
Correct answers are ticked in green.

GS-2013 (Physics)
TATA INSTITUTE OF FUNDAMENTAL RESEARCH

WrittenTestinPHYSICSͲDecember9,2012
Duration:Threehours(3hours)


Name:_______________________________________________Ref.Code:____________


Pleasereadallinstructionscarefullybeforeyouattemptthequestions.

1. Please fillͲin details about name, reference code etc.  on the question paper and answer sheet.  The
Answer Sheet is machineͲreadable.  Read the instructions given on the reverse of the answer sheet
beforeyoustartfillingitup.UseonlyHBpencilstofillͲintheanswersheet.

2. Indicate yourANSWERON THE ANSWER SHEETbyblackening the appropriate circlefor eachquestion.
Donotmarkmorethanonecircleforanyquestion:thiswillbetreatedasawronganswer.

3. This test consists of three parts, Section A, Section B and Section C.  You must answer questions
accordingtotheinstructionsbelow:
Candidatesapplyingfor MustAnswer ShouldNotAttempt
IntegratedPh.D. SectionAandSectionB SectionC
Ph.D. SectionAandSectionC SectionB

 The test contains multiple choiceͲtype questions.  Only ONE of the options given at the end of each
questioniscorrect.SectionAcontains25questions,SectionBandSectionCcontain15questionseach.

Markingshallbeasfollows:
 (i) Iftheansweriscorrect:    +3marks
 (ii) Iftheanswerisincorrect:   Ͳ1mark
 (III) Iftheanswerisnotattempted:  0marks
 (iv) Ifmorethanoneboxismarked:  0marks
 Notethatnegativemarkingasindicatedabovewillbeapplicable.

4. Asaroughguideline,thetimespentonquestionsinSectionAshouldbeabout5minuteseach;questions
inSectionBandCshouldtakeabout8minuteseach.Obviously,somequestionsmaytakealittleless
timewhileothersmayrequirealittlemore.

5. WeadviseyoutofirstmarkthecorrectanswersontheQUESTIONPAPERandthentoTRANSFERtheseto
theANSWERSHEETonlywhenyouaresureofyourchoice.

6. Roughworkmaybedoneonblankpagesofthequestionpaper.Ifneeded,youmayaskforextrarough
sheetsfromanInvigilator.

7. Useofcalculatorsispermitted.CalculatorwhichplotsgraphsisNOTallowed.MultipleͲusedevicessuch
ascellphones,smartphonesetc.,CANNOTbeusedforthispurpose.

8. DoNOTaskforclarificationsfromtheinvigilatorsregardingthequestions.Theyhavebeeninstructednot
torespondtoanysuchinquiriesfromcandidates.Incaseacorrection/clarificationisdeemednecessary,
theinvigilator(s)willannounceitpublicly.

9. Listofusefulphysicalconstantsisgivenonthenextpage.
USEFUL CONSTANTS
Symbol Name/Definition Value
ܿ speed of light in vacuum 3 × 108 m s-1
԰ reduced Planck constant (= ݄/2ߨ) 1.04 × 10 െ34 J s
‫ܰܩ‬ gravitational constant 6.67 × 10 െ11 m3 kg-1 s-2
ߝ0 permittivity of free space 8.85 × 10 െ12 F m-1
ߤ0 permeability of free space 4ߨ × 10െ7 N A-2
݁ electron charge (magnitude) 1.6 × 10 െ19 C
݉݁ electron mass 9.1 × 10െ31 kg
= 0.5 MeV/c 2

ܽ0 Bohr radius 0.51 Å


ionisation potential of H atom 13.6 eV
ܰ‫ܣ‬ Avogadro number 6.023 × 1023 mol-1
݇‫ܤ‬ Boltzmann constant 1.38 × 10 െ23 J K-1
= 8.62 × 10െ5 eV K-1
ܴ = ܰ‫ ܤ݇ ܣ‬gas constant 8.31 J mol-1 K-1
ߛ = ‫ ݌ܥ‬/‫ ܸܥ‬ratio of specific heats: monatomic gas 1.67
diatomic gas 1.40
ߪ Stefan-Boltzmann constant 5.67 × 10െ8 W m-2 K-4
ߙ fine structure constant (= ݁ 2 /4ߨߝ0 ԰ܿ) 1/137
݃ acceleration due to gravity 9.8 m s-2
ܴ‫ܧ‬ radius of the Earth 6.4 × 103 Km
ܴܵ radius of the Sun 7 × 105 Km
݉‫݌‬ proton mass (ൎ 2000 ݉݁ ) 1.7 × 10െ27 kg
= 938.2 MeV/c 2

݉݊ neutron mass (ൎ 2000 ݉݁ ) 1.7 × 10െ27 kg


= 939.6 MeV/c 2

UNIT CONVERSIONS
Symbol Name/Definition Value
1 A.U. mean distance of Earth from Sun 1.5 × 109 km
1 a.m.u. atomic mass unit 1.6 × 10െ27 kg
= 931.5 MeV/c 2

1 eV electron Volt 1.6 × 10െ19 J


԰ܿ conversion constant 3.16 × 10െ26 J m-1
= 0.1973 GeV fm
1T Tesla 104 gauss
1 bar mean atmospheric pressure at 00 C 1.01 × 105 Pa (= N m-2)

1
GS-2013-X (Physics)
Section A
To be attempted by ALL candidates.

1. The value of the integral


λ

න ݀‫ ݔ ݔ‬9 expሺെ‫ ݔ‬2 ሻ


0
is

(a) 20160 (b) 12 (c) 18 (d) 24

2. A two-dimensional vector ‫ܣ‬റ(‫ )ݐ‬is given by

‫ܣ‬റ(‫݅ = )ݐ‬Ƹ sin 2‫ ݐ‬+ ݆Ƹ cos 3‫ ݐ‬.


Which of the following graphs best describes the locus of the tip of the
vector, as ‫ ݐ‬is varied from 0 to 2ߨ ?

(a) (c)

݆Ƹ

݅Ƹ

(b) (d)

3. The differential equation


݀2 ‫ݕ‬ ݀‫ݕ‬
െ 2 +‫ = ݕ‬0
݀‫ ݔ‬2 ݀‫ݔ‬
has the complete solution, in terms of arbitrary constants A and B,
(a) ‫ ܣ‬exp ‫ ݔ‬+ ‫ ݔ ܤ‬exp ‫ݔ‬ (c) ‫ ܣ‬exp ‫ ݔ‬+ ‫ ݔܤ‬expሺെ‫ݔ‬ሻ
(b) ‫ ܣ‬exp ‫ ݔ‬+ ‫ ܤ‬expሺെ‫ݔ‬ሻ (d) ‫ݔ‬ሼ‫ ܣ‬exp ‫ ݔ‬+ ‫ ܤ‬expሺെ‫ݔ‬ሻሽ

2
4. A stone is dropped vertically from the top of a tower of height 40 m. At
the same time a gun is aimed directly at the stone from the ground at a
horizontal distance 30 m from the base of the tower and fired. If the
bullet from the gun is to hit the stone before it reaches the ground, the
minimum velocity of the bullet must be, approximately,

(a) 57.4 ms-1 (b) 27.7 ms-1 (c) 17.7 ms-1 (d) 7.4 ms-1

5. Consider the uniform solid right


cone depicted in the figure on the
right. This cone has mass ‫ ܯ‬and
a circular base of radius ‫ݎ‬. If the
moment of inertia of the cone
about an axis parallel to the X
axis passing through the centre
of mass OC.M. (see figure) is given
by
3
‫ܯ‬ሺ4‫ ݎ‬2 + ݄2 ሻ
80

then the moment of inertia about another axis parallel to the X axis, but
passing through the point Or (see figure), is

3 1
(a) ‫ܯ‬ሺ4‫ ݎ‬2 + ݄2 ሻ (c) ‫ܯ‬ሺ23‫ ݎ‬2 + 2݄2 ሻ
80 20

3 1
(b) ‫ܯ‬ሺ2‫ ݎ‬2 + ݄2 ሻ (d) ‫ܯ‬ሺ15‫ ݎ‬2 + 4݄2 ሻ
40 30

6. Two planets A and B move around the Sun in elliptic orbits with time
periods ܶ‫ ܣ‬and ܶ‫ ܤ‬respectively. If the eccentricity of the orbit of B is ߝ
and its distance of closest approach to the Sun is ܴ , then the
maximum possible distance between the planets is
‫ݎ‬ െ‫ݎ‬
[Eccentricity of an ellipse: ߝ = ݉ܽ‫] ݊݅݉ ݔ‬
‫ ݔܽ݉ݎ‬+‫݊݅݉ݎ‬

3/2
1 + ߝ2 ܶ 1+ߝ ܶ‫ܣ‬3
(a) 2
൭1 + ‫ܣ‬3/2 ൱ ܴ (c) ඨ ቆ1 + 3 ቇ ܴ
1െߝ ܶ 1െߝ ܶ‫ܤ‬
‫ܤ‬

2/3
1+ߝ ܶ 1 + ߝ2 ܶ
2/3
(b) ൭1 + ‫ܣ‬2/3 ൱ ܴ (d) ඨ ൭1 + ‫ܣ‬2/3 ൱ ܴ
1െߝ ܶ 1െߝ 2
‫ܤ‬ ܶ ‫ܤ‬

3
7. A spaceship S blasts off from the Earth. After some time, Earth station
informs the crew that they have settled into a constant velocity 0.28ܿ
radially outward from the Earth, but unfortunately they are on a head-
on collision course with an asteroid A at a distance of 15 light-minutes
coming in towards the Earth along the same radius (see figure below).

Earth ݀
S A

‫ݑ‬ ‫ݒ‬
Instruments on-board the spaceship immediately estimate the speed of
the asteroid to have a constant value 0.24ܿ . It follows that the
maximum time (in minutes) available to the crew to evacuate the ship
before the collision is

(a) 60 (b) 30 (c) 29 (d) 63

8. A point charge ‫ ݍ‬sits at a corner of a cube of side ܽ,


as shown in the figure on the right. The flux of the
electric field vector through the shaded side is ‫ݍ‬

‫ݍ‬ ‫ݍ‬ ‫ݍ‬ ‫ݍ‬


(a) (b) (c) (d)
8ߝ0 16ߝ0 24ߝ0 6ߝ0

9. A parallel plate capacitor of circular cross section with radius ‫݀ ب ݎ‬,


where ݀ is the spacing between the plates, is charged to a potential ܸ
and then disconnected from the charging circuit. If, now, the plates
are slowly pulled apart (keeping them parallel) so that their separation
is increased from ݀ to ݀ Ԣ , the work done will be

2 2
(a) ߨߝ0 ‫ܸ ݎ‬ ݀ ߨߝ0 ‫ ݎ‬2 ܸ 2 ݀Ԣ
൬1 െ ൰ (c)
2݀ ݀Ԣ 2݀ ݀
2 2
(b) ߨߝ0 ‫݀ ܸ ݎ‬Ԣ ߨߝ0 ‫ ݎ‬2 ܸ 2 ݀
ቆ െ 1ቇ (d)
2݀ ݀ 2݀ ݀Ԣ

4
10. In the laboratory frame two electrons are shot at each other with
equal and opposite velocities ‫ݑ‬ሬറ1 and ‫ݑ‬ሬറ2 respectively, but not along the
same straight line, as shown below.
‫ݖ‬
‫ݑ‬ሬറ1 ‫ݔ‬
‫ݑ‬ሬറ2

Each electron will be acted on by the Coulomb repulsion due to the


other, as well as the Lorentz force due to its own motion in the
magnetic field created by the other. Which of the diagrams given
below best describes the final velocities ‫ݒ‬റ1 and ‫ݒ‬റ2 of these electrons?
[You may assume that the electrons are distinguishable.]

‫ݖ‬ ‫ݖ‬
‫ݒ‬റ1 ‫ݒ‬റ2
(a) (c)
‫ݔ‬ ‫ݔ‬
‫ݒ‬റ1
‫ݒ‬റ2

‫ݖ‬ ‫ݖ‬
‫ݒ‬റ1 ‫ݒ‬റ1 …
‫ݔ‬
(b) ‫ݔ‬ (d)
‫ݒ‬റ2
~ ‫ݒ‬റ2

11. A certain amount of fluid with heat capacity ‫ ܨܥ‬Joules/0C is initially at


a temperature 0o C. It is then brought into contact with a heat bath at a
temperature of 100o C, and the system is allowed to come into
equilibrium. In this process, the entropy (in Joules/0C) of the Universe
changes by

(a) 100 ‫ܨܥ‬ (b) 0 (c) 0.055 ‫ܨܥ‬ (d) 0.044 ‫ܨܥ‬

12. A monatomic gas is described by the equation of state


‫ ݌‬ሺܸ െ ܾ݊ሻ = ܴ݊ܶ
where ܾ and ܴ are constants and other quantities have their usual
meanings. The maximum density (in moles per unit volume) to which
this gas can be compressed is

(a) 1ൗ (b) ܾ (c) 1ൗ (d) infinity


ܾ݊ ܾ

5
13. In a quantum mechanical system, an observable A is represented by an
operator ‫ܣ‬መ. If |߰‫ ۄ‬is a state of the system, but not an eigenstate of ‫ܣ‬መ ,
then the quantity
2
‫ = ݎ‬ൻ߰ห‫ܣ‬መห߰ൿ െ ൻ߰ห‫ܣ‬መ 2ห߰ൿ

satisfies the relation

(a) ‫ < ݎ‬0 (b) ‫=ݎ‬0 (c) ‫>ݎ‬0 (d) ‫ݎ‬൒0

14. Consider a quantum mechanical system with three linear operators ‫ܣ‬መ,
‫ܤ‬෠ and ‫ܥ‬መ, which are related by

‫ܣ‬መ ‫ܤ‬෠ െ ‫ܥ‬መ = ‫ܫ‬መ

where ‫ܫ‬መ is the unit operator. If ‫ܣ‬መ = ݀ൗ݀‫ ݔ‬and ‫ܤ‬෠ = ‫ݔ‬, then ‫ܥ‬መ must be

݀ ݀ ݀
(a) zero (b) (c) െ‫ݔ‬ (d) ‫ݔ‬
݀‫ݔ‬ ݀‫ݔ‬ ݀‫ݔ‬

15. A particle of energy ‫ ܧ‬moves in one dimension under the influence of a


potential ܸሺ‫ݔ‬ሻ. If ‫ܸ > ܧ‬ሺ‫ݔ‬ሻ for some range of ‫ݔ‬, which of the following
graphs can represent a bound state wave function of the particle?

(a) (c)

(b) (d)

6
16. In a Davisson-Germer experiment, a collimated beam of electrons of
energy 54 eV, at normal incidence on a given crystal, shows a peak at a
reflection angle of 400. If the electron beam is replaced by a neutron
beam, and the peak appears at the same value of reflection angle, then
the energy of the neutrons must be

(a) 330 eV (b) 33 eV (c) 0.3 eV (d) 0.03 eV

17. The velocity of an electron in the ground state of a hydrogen atom is ‫ ܪݒ‬.
If ‫ ݌ݒ‬be the velocity of an electron in the ground state of positronium,
then

(a) ‫ܪݒ = ݌ݒ‬ (b) ‫ = ݌ݒ‬2‫ܪݒ‬ (c) ‫ܪݒ = ݌ݒ‬ൗ (d) ‫ = ݌ݒ‬ξ2‫ܪݒ‬
2

18. Consider an ensemble of microscopic quantum mechanical systems


with two energy levels ‫ܧ‬1 and ‫ܧ‬2 , where ‫ܧ‬1 < ‫ܧ‬2. Which of the following
graphs best describes the temperature dependence of the average
energy ‫ ۄܧۃ‬of the system?

‫ۄܧۃ‬ ‫ۄܧۃ‬
1 1
ሺ‫ܧ‬ + ‫ܧ‬2 ሻ ሺ‫ܧ‬ + ‫ܧ‬2 ሻ
2 1 2 1
(a) (c)

‫ܧ‬1 ‫ܧ‬1

ܶ ܶ

‫ۄܧۃ‬ ‫ۄܧۃ‬
1 1
ሺ‫ܧ‬ + ‫ܧ‬2 ሻ ሺ‫ܧ‬ + ‫ܧ‬2 ሻ
2 1 2 1
(b) (d)

‫ܧ‬1 ‫ܧ‬1

ܶ ܶ

7
19. A ray of light is incident on the
surface of a thin prism at a small
angle ߠ1 with the normal, as shown ߙ
in the figure on the right. The
material of the prism has refractive
index n and you may assume the ߜ
outside refractive index to be unity.
If the (small) apex angle of the prism ߠ1
is ߙ , the deviation angle ߜ (angle
between the incident and exited ray; n
see figure) is given by

(a) ߙ (b) ߙ݊ (c) ߙሺ݊ + 1ሻ (d) ߙሺ݊ െ 1ሻ

20. A parallel beam of light of wavelength ߣ is


incident on a transmission grating with
groove spacing ݀, at an angle ߠ݅ , as shown
in the figure on the left. The plane of
incidence is normal to the grooves. After
diffraction, the transmitted beam is seen to
be at an angle ߙ relative to the normal.
Which of the following conditions must be
satisfied for this to happen?

(a) ݀ሺsin ߠ݅ െ sin ߙሻ = ݊ߣ (c) 2݀ sinሺߠ݅ െ ߙሻ = ݊ߣ


(b) ݀ሺsin ߠ݅ + sin ߙሻ = ݊ߣ (d) 2݀ sinሺߙ + ߠ݅ ሻ = ݊ߣ

21. Let ‫ ܰܧ‬be the energy released when one mole of pure 235U undergoes
controlled fission, and ‫ ܥܧ‬be the energy released when one mole of pure
carbon undergoes complete combustion. The ratio ‫ ܰܧ‬/‫ ܥܧ‬will have the
order of magnitude

(a) 104 (b) 108 (c) 109 (d) 106

8
22. The entropy ܵ of a black hole is known to be of the form
ܵ = ߙ݇‫ܣ ܤ‬
where ‫ ܣ‬is the surface area of the black hole and ߙ is a constant, which
can be written in terms of ܿ (velocity of light in vacuum), ԰ (reduced
3ODQFN·VFRQVWDQW DQG‫ ܰܩ‬1HZWRQ·VFRQVWDQWRIJUDYLWDWLRQ 7DNLQJWKH
radius of the black hole as
2‫ܯ ܰܩ‬
ܴ=
ܿ2
it follows that the entropy ܵ is
[ߣ is a numerical constant]

2 2
(a) ‫ܤ݇ ܯ ܰܩ‬ (b) ԰ܿ݇‫ܤ‬ (c) ‫ܰܩ‬2 ‫ ܯ‬2 ݇‫ܤ‬ (d) ‫ ܯ ܰܩ‬2 ݇‫ܤ‬
ߣሺ԰ܿሻ4 ߣ‫ܯ ܰܩ‬ ߣ԰ܿ 4 ߣ԰ܿ

23. The circuit depicted on the right has


been made with a silicon n-p-n
transistor.

Assuming that there will be a 0.7 V drop


across a forward-biased silicon p-n
junction, the power dissipated across
the transistor will be, approximately,

(a) 53 mW (b) 94 mW (c) 17 mW (d) 67 mW

24. An input of 1.0 V DC is given to the ideal Op-Amp circuit depicted


below. What will be the output voltage?

(a) 10.0 V (b) ² 9.0 V (c) 1.0 V (d) 0 V

9
25. The circuit shown below uses only NAND gates. Find the final output.

(a) A XOR B (c) A AND B


(b) A OR B (d) A NOR B

10
GS-2013-X (Physics)
Section B
To be attempted only by candidates for Integrated Ph.D. programme.
(Candidates for Ph.D. programme will get no credit for attempting this section.)

26. Consider the surface corresponding to the equation

4‫ ݔ‬2 + ‫ݕ‬2 + ‫ = ݖ‬0

A possible unit tangent to this surface at the point ሺ1,2, െ8ሻ is


1 2 4 8 1
(a) ݅Ƹ െ ݆Ƹ (c)
9
݅Ƹ െ 9
݆Ƹ + 9
݇෠
ξ5 ξ5
1 4 1 3 4
(b) ݆Ƹ െ ݇෠ (d) െ ݅Ƹ + ݆Ƹ െ ݇෠
5 5 ξ5 ξ5 ξ5

27. A particle with time-varying mass


݉ሺ‫ݐ‬ሻ = ݉0 ൫1 െ ‫ݐ‬ൗ߬ ൯,
where ݉0 and ߬ are positive constants, moves along the ‫ݔ‬-axis under
the action of a constant positive force ‫ ܨ‬for 0 ൑ ‫߬ < ݐ‬. If the particle is at
rest at time ‫ = ݐ‬0, then at time ‫ݐ = ݐ‬, its velocity ‫ ݒ‬will be

߬‫ܨ‬ ‫ݐ‬ ‫ݐܨ‬ ‫ ݐ‬െ1


(a) െ log ൬1 െ ൰ (c) ൬1 െ ൰
݉0 ߬ ݉0 ߬
‫ݐܨ‬ ‫ݐ‬ ߬‫ܨ‬ ‫ݐ‬
(b) െ log (d) ൬1 െ ൰
݉0 ߬ ݉0 ߬

‫ݖ‬
28. A ball of mass ݉ slides under gravity
without friction inside a semicircular ‫ݔ‬
depression of radius ܽ inside a fixed
block placed on a horizontal surface, as
shown in the figure. The equation of
motion of the ball in the x-direction will
be

݃ ‫ݔ‬2 ݃
(a) ‫ݔ‬ሷ = ‫ݔ‬ඨ1 െ 2 (c) ‫ݔ‬ሷ = െ ‫ݔ‬
ܽ ܽ ܽ

݃ ݃ ‫ݔ‬2
(b) ‫ݔ‬ሷ = ‫ݔ‬ (d) ‫ݔ‬ሷ = െ ‫ݔ‬ඨ1 െ 2
ܽ ܽ ܽ

11
29. A particle P, of rest mass ‫ ܯ‬and energy ‫ܧ‬, suddenly decays into two
particles A and B of rest masses ݉‫ ܣ‬and ݉‫ ܤ‬respectively, and both
particles move along the straight line in which P was moving. A
possible energy ‫ ܣܧ‬of the particle A will be

‫ܧ‬ ݉‫ ܣ‬െ ݉‫ ܤ‬2 ‫ܧ‬ ݉‫ ܣ‬+ ݉‫ ܤ‬2


(a) ൜1 + ቀ ቁ ൠ (c) ቊ1 + ൬ ൰ ቋ
2 ‫ܯ‬ 2 ‫ܯ‬
2 2
(b) ‫ ܧ‬ቊ1 െ ቆ݉‫ ܣ‬െ ݉‫ ܤ‬ቇቋ (d) ‫ܧ‬ ݉‫ܣ‬2 െ ݉2‫ܤ‬
ቊ1 + ቆ ቇቋ
2 ‫ܯ‬2 2 ‫ܯ‬2

30. Consider two charges +ܳ and Ȃ ܳ placed at the ሺܺ, ܻሻ


points (ܽ, 0) and (െܽ, 0) in a plane, as shown in
െܳ +ܳ
the figure on the right. If the origin is moved to
the point ሺܺ, ܻሻ , the magnitude of the dipole ሺെܽ, 0ሻ ሺܽ, 0ሻ
moment of the given charge distribution with
respect to this origin will be

(a) ܳඥሺܽ െ ܺሻ2 + ‫ݕ‬2 െ ܳඥሺܽ + ܺሻ2 + ‫ݕ‬2 (c) ܳሺܽ െ ܺሻ െ ܳ(െܽ + ܺ)

(b) 2ܳܽ (d)


2ܳܽඥܺ 2 + ܻ 2

31. A plane electromagnetic wave travelling in a vacuum is characterised


by the electric and magnetic fields

ሬറ = ݅Ƹ ሺ30ߨ Vmെ1 ሻ exp ݅ሺ߱‫ ݐ‬+ ݇‫ݖ‬ሻ


‫ܧ‬

ሬറ = ݆Ƹ ሺ ‫ܪ‬0 Amെ1 ሻ exp ݅ሺ߱‫ ݐ‬+ ݇‫ݖ‬ሻ


‫ܪ‬

If ߱, ݇ > 0, the value of ‫ܪ‬0 must be

(a) 2ߨ (b) 0.67 (c) 0.25 (d) 0.94

32. Which of the following classic experiments provides unambiguous


proof that the Earth is a non-inertial frame of reference with
respect to the fixed stars?

(a) &ŝnjĞĂƵ͛ƐƌŽƚĂƚŝŶŐǁŚĞĞůĞdžƉĞƌŝŵĞŶƚ
(b) &ŽƵĐĂƵůƚ͛ƐƉĞŶĚƵůƵŵĞdžƉĞƌŝŵĞŶƚ
(c) EĞǁƚŽŶ͛Ɛcoin-and-feather experiment
(d) Michelson-Morley experiment

12
33. A cross-shaped opening is illuminated by a parallel beam of white light.
A thin plano-convex cylindrical glass lens is placed 20 cm in front of it,
as shown in the figure below.

The radius of curvature of the curved surface of the lens is 5 cm and


1.5 is the refractive index of glass. On a screen placed as shown at the
plane where a real image forms on the other side of the lens, the image
of the opening will appear as

(a) (c)

(b) (d)

34. A classical ideal gas, consisting of ܰ particles (ܰ ՜ λ) is confined in a


box of volume ܸ at temperature ܶ and pressure ‫݌‬. The probability that,
at any instant of time, a small sub-volume ‫ݒ‬0 becomes totally void (i.e.
no particles inside), due to a spontaneous statistical fluctuation, is

െ‫ݒ‬0 ‫ݒ‬0 െ‫ܸ݌‬ൗ


(a) exp ቆ ൗܸ ቇ exp ቆ
(c)
ܸ ܰܶቇ

(d) ‫ݒ݌‬0
െܰ‫ݒ‬0ൗ ൗܰܶ
(b) exp ቆ
ܸቇ

13
35. The ‫ ܸ݌‬-diagram for a Carnot cycle executed by an ideal gas with
‫ ܲܥ‬Τ‫ > ߛ = ܸܥ‬1 is shown below. Note that 1, 2, 3 and 4 label the change-
over points in the cycle.
‫݌‬
1
2

4
If, for this cycle,
3
ܸ
ܶ2 ‫݌‬2 ܺ
=൬ ൰
ܶ3 ‫݌‬3
then ܺ =
(a) 1 െ 1ൗ (b) 0 (c) 1 (d) െ 1ൗߛ
ߛ

36. A harmonic oscillator has the wave function,

1
߰ሺ‫ݔ‬, ‫ݐ‬ሻ = ൣ3߮0 ሺ‫ݔ‬, ‫ݐ‬ሻ െ 2ξ2߮1 ሺ‫ݔ‬, ‫ݐ‬ሻ + 2ξ2߮2 ሺ‫ݔ‬, ‫ݐ‬ሻ൧
5
where ߮݊ ሺ‫ݔ‬, ‫ݐ‬ሻ is the eigenfunction belonging to the n-th energy
eigenvalue ൫݊ + 12൯԰߱. The expectation value ‫ ۄܧۃ‬of energy for the state
߰ሺ‫ݔ‬, ‫ݐ‬ሻ is

(a) 1.58 ԰߱ (b) 0.46 ԰߱ (c) ԰߱ (d) 1.46 ԰߱

37. An energy eigenstate of the Hydrogen atom has the wave function

11 3/2 ‫ݎ‬
߰݊κ݉ ሺ‫ݎ‬, ߠ, ߮ሻ = ൰ sin ߠ cos ߠ exp ൤െ ൬
൬ + ݅߮൰൨
81ξߨ ܽ0 3ܽ0

where ܽ0 is the Bohr radius. The principal ( ݊ ), azimuthal ( κ ) and


magnetic (݉) quantum numbers corresponding to this wave function
are

(a) ݊ = 3, κ = 2, ݉ = 1 (c) ݊ = 3, κ = 2, ݉ = െ 1
(b) ݊ = 2, κ = 1, ݉ = 1 (d) ݊ = 2, κ = 1, ݉ = ±1

14
38. The Curie temperature of a
single crystal of PrGe is
known to be 41 K. The
magnetization data of this
sample is measured at 1.8 K
for the magnetic field applied
parallel to the [001] direction
is shown in the figure on the
left. At a temperature of 38
K, the hysteresis loop in the
figure will

(a) have the same width (c) decrease in width

(b) increase in width (d) shrink to a line

39. Consider two energies of a free electron gas in a metal at an absolute


temperature ܶ, viz.,
‫ܧ‬± = ‫ ܨܧ‬± ο
where ‫ ܨܧ‬is the Fermi level. If the corresponding electron populations
݊ሺ‫ܧ‬± ሻ satisfy the relation ݊ሺ‫ܧ‬െሻΤ݊ሺ‫ܧ‬+ሻ = 2, then ο =

(a) ݇‫ ܶ ܤ‬ln 2 (b) 2݇‫ܶ ܤ‬ (c) ݇‫ܶ ܤ‬ൗ (d) ݇‫ܶ ܤ‬


2

40. The figure on the right


shows the current-
voltage characteristics
of a diode over a range
of voltage and current
where it is safe to
operate the diode.

When this diode is used in the circuit on the extreme right, the
approximate current, in mA, through the diode will be

(a) 0 (b) 8.3 (c) 16.7 (d) 25

15
GS-2013-X (Physics)
Section C
To be attempted only by candidates for Ph.D. programme.
(Candidates for Integrated Ph.D. programme will get no credit for attempting this section.)

41. The integral


λ
න ݀‫ߜ ݔ‬ሺ‫ ݔ‬2 െ ߨ 2 ሻ cos ‫ݔ‬
െλ

evaluates to
1ൗ െ 1ൗߨ
(a) െ1 (b) 0 (c) ߨ (d)

42. If ‫ ݔ = ݖ‬+ ݅‫ ݕ‬then the function

݂ሺ‫ݔ‬, ‫ݕ‬ሻ = ሺ1 + ‫ ݔ‬+ ‫ݕ‬ሻሺ1 + ‫ ݔ‬െ ‫ݕ‬ሻ + ܽሺ‫ ݔ‬2 െ ‫ݕ‬2 ሻ െ 1 + 2݅‫ݕ‬ሺ1 െ ‫ ݔ‬െ ܽ‫ݔ‬ሻ

where ܽ is a real parameter, is analytic in the complex ‫ ݖ‬plane if ܽ =

(a) െ1 (b) +1 (c) 0 (d) ݅

43. A particle of mass m moves in one dimension under the influence of a


potential energy
2 4
‫ݔ‬ ‫ݔ‬
ܸሺ‫ݔ‬ሻ = െܽ ቀ κ ቁ + ܾ ቀ κ ቁ

where ܽ and ܾ are positive constants and κ is a characteristic length.


The frequency of small oscillations about a point of stable equilibrium
is

(a) 1 ܾ (c) 1 ܽ2
ඨ ඨ
2ߨκ ݉ ߨκ ܾ݉

1 ܽ
(b) ට (d) 2ܾ 1
ߨκ ݉ ඨ
ߨκ ݉ܽ

16
44. If a central force acting on a particle of mass m is given by

݇
‫ܨ‬ሺ‫ݎ‬ሻ = െ
‫ݎ‬2
where ‫ ݎ‬is the distance of the particle from the origin and k is a
positive constant, the Hamiltonian for the system, in spherical polar
coordinates, will have the form

1 ݇
(a) ݉൫‫ݎ‬ሶ 2 + ‫ ݎ‬2 ߠሶ 2 + ‫ ݎ‬2 sin2 ߠ ߮ሶ 2 ൯ +
2 ‫ݎ‬
(b) ‫ݎ݌‬2 ‫ߠ݌‬2 ‫߮݌‬2 csc2 ߠ ݇
+ + +
2݉ 2݉‫ ݎ‬2 2݉‫ ݎ‬2 ‫ݎ‬
1
(c)
2
൫‫ ݎ‬2 ‫ݎ݌‬2 + ‫ߠ݌‬2 + ‫߮݌‬2 + 2݉݇‫ݎ‬൯
2݉‫ݎ‬
(d) ‫ݎ݌‬2 ‫ߠ݌‬2 ‫߮݌‬2 ݇
+ + +
2݉ 2݉ 2݉ ‫ݎ‬

45. The magnetic vector potential ‫ܣ‬റሺ‫ݎ‬റሻ corresponding to a uniform


ሬറ is taken in the form
magnetic field ‫ܤ‬

1
‫ܣ‬റ = ሬറ × ‫ݎ‬റ
‫ܤ‬
2
where ‫ݎ‬റ is the position vector. If the electric field has the time-
dependent form ‫ܧ‬ሬറ = ‫ܧ‬ሬറ0 ሺ‫ݎ‬റሻ݁ ݅߱‫ ݐ‬, where ߱ is a constant, the gauge choice
corresponding to this potential is a

(a) Lorenz gauge (c) Coulomb gauge


(b) non-linear gauge (d) time-varying gauge

46. A binary star is observed to consist of a blue star B (peak wavelength


400 nm) and a red star R (peak wavelength 800 nm) orbiting each
other. As observed from the Earth, B and R appear equally bright.
Assuming that the stars radiate as perfect blackbodies, it follows that
the ratio of volumes ܸ‫ ܤ‬/ܸܴ of the two stars is

1ൗ 1ൗ
(a) 64 (b) 64 (c) 16 (d) 16

17
47. An system at temperature ܶ has three energy states 0, ±ߝ. The entropy
of the system in the low temperature (ܶ ՜ 0) and high temperature
(ܶ ՜ λ) limits are, respectively,

(a) ܵܶ՜0 = 0 and ܵܶ՜ь = ݇B expሺെ3ሻ


(b) ܵܶ՜0 = ܵܶ՜ь = ݇B ln 3
(c) ܵܶ՜0 = 0 and ܵܶ՜ь = ݇B ln 3
(d) ܵܶ՜0 = 0 and ܵܶ՜ь = 3݇B /2

48. The state |߰‫ ۄ‬of a quantum mechanical system, in a certain basis, is
represented by the column vector

1/ξ2
|߰‫ = ۄ‬ቌ 0 ቍ
1/ξ2

The operator ‫ܣ‬መ corresponding to a dynamical variable A, is given, in the


same basis, by the matrix

1 1 1
‫ܣ‬መ = ൭1 2 1൱
1 1 2

If, now, a measurement of the variable A is made on the system in the


state |߰‫ۄ‬, the probability that the result will be +1 is

(a) 1ൗ (b) 1 (c) 1ൗ


2 (d) 1ൗ
4
ξ2

49. A spin-½ particle A decays to two other particles B and C. If B and C


are of spin-½ and spin-1 respectively, then a complete list of the
possible values of the orbital angular momentum of the final state (i.e.
B + C) is

(a) 0, 1 (b) 1ൗ , 3ൗ (c) 0, 1, 2 (d) 0, ±1


2 2

18
50. When a pure element is vaporised and placed in a uniform magnetic
field ‫ܤ‬0 , it is seen that a particular spectral line of wavelength ߣ ,
corresponding to a ‫ = ܬ‬1 ՜ ‫ = ܬ‬0 transition, gets split into three
components ߣ, ߣ ± οߣ . It follows that the Landé g-factor for the
transition ‫ = ܬ‬1 ՜ ‫ = ܬ‬0 is given by

(a) ݃ =
݄ܿ οߣ2 ݄ܿ ߣ2
(c) ݃=
ߤ‫ܤ ܤ‬0 ߣ ߤ‫ܤ ܤ‬0 οߣ
݄ܿ ߣ ݄ܿ οߣ
(b) ݃ = (d) ݃=
ߤ‫ܤ ܤ‬0 οߣ2 ߤ‫ܤ ܤ‬0 ߣ2

51. The rate of deposition of a


dielectric thin film on a thick
dielectric substrate was
monitored by the following
experiment: a laser beam of
wavelength ߣ = 633 nm, at near-
normal incidence ߠ݅ , was reflected
from the thin film (see inset figure
on the right), and the reflection
coefficient ܴ was measured. As
the film thickness increased ܴ
varied with time as shown on the

right. The refractive index of the film is 3.07 and is less than that of the
substrate. Using the graph, the approximate thickness of the film at
the end of 25 seconds can be estimated to be

(a) 0.017 Ǎm (b) 0.26 Ǎm (c) 0.51 Ǎm (d) 2.2 Ǎm

52. The negative image on the right represents a very


small portion of the night sky at a very high
resolution. Notice the broken ring(s) around the
central bright object in the middle of the picture.
These are most likely to be due to

(a) debris from a smaller object torn apart by tidal forces


(b) gas clouds forming the remnant of a supernova explosion
(c) ice collected on the lens used for taking the picture
(d) gravitational lensing of a distant object by the central massive object

19
53. The magnetic susceptibility ߯ of three samples A, B and C, is
measured as a function of their absolute temperature ܶ, leading to the
graphs shown below.
1ൗ A
߯

ܶ
0

From these graphs, the magnetic nature of the samples can be


inferred to be

(a) A : anti-ferromagnet B : paramagnet C : ferromagnet


(b) A : diamagnet B : paramagnet C : anti-ferromagnet
(c) A : paramagnet B : anti-ferromagnet C : ferromagnet
(d) A : anti-ferromagnet B : diamagnet C : paramagnet

54. A gold foil, having ܰሺ0ሻ number of 197Au nuclides per cm 2, is irradiated
by a beam of thermal neutrons with a flux of ‫ ܨ‬neutrons-cm-2-s-1. As a
result, the nuclide 198Au, with a half-life ߬ of several years, is produced
by the reaction

197 198
Au + ݊ ՜ Au + ߛ
which has a cross section of ߪ cm2. Assuming that the gold foil has
100% abundancy of 197Au nuclides, the maximum number of 198Au
nuclides that can accumulate at any time in the foil is proportional to

(a) ߪ߬‫ܰ ܨ‬ሺ0ሻ (c) 1


ܰሺ0ሻ
ߪ߬‫ܨ‬
߬ ߪ‫ܨ‬
(b) ܰሺ0ሻ (d) ܰሺ0ሻ
ߪ‫ܨ‬ ߬

20
55. The process of electron capture

‫ ݌‬+ ݁ െ ՜ ݊ + ߥ݁
takes place at the quark level through the Feynman diagram
݀ ߥ݁ ‫ݑ‬ ݁െ

(a) (c)
ܹ ܹ
‫ݑ‬ ݁െ ݀ ߥ݁

time
‫ݑ‬ ݁െ ‫ݑ‬
݁െ

(b) (d)
ܼ ܹ
݀ ߥ݁
ߥ݁ ݀

21
Correct answers are ticked in green.
PHY - X
GS-2014 (Physics)
TATA INSTITUTE OF FUNDAMENTAL RESEARCH

WrittenTestinPHYSICSͲDecember8,2013
Duration:Threehours(3hours)


Name:_______________________________________________Ref.Code:____________


Pleasereadallinstructionscarefullybeforeyouattemptthequestions.

1. Please fillͲin details about name, reference code etc.  on the question paper and answer sheet.  The
Answer Sheet is machineͲreadable.  Read the instructions given on the reverse of the answer sheet
beforeyoustartfillingitup.UseonlyHBpencilstofillͲintheanswersheet.

2. Indicate yourANSWERON THE ANSWER SHEETbyblackening the appropriate circlefor eachquestion.
Donotmarkmorethanonecircleforanyquestion:thiswillbetreatedasawronganswer.

3. This test consists of three parts, Section A, Section B and Section C.  You must answer questions
accordingtotheinstructionsbelow:
Candidatesapplyingfor MustAnswer ShouldNotAttempt
IntegratedPh.D. SectionAandSectionB SectionC
Ph.D. SectionAandSectionC SectionB

 The test contains multiple choiceͲtype questions.  Only ONE of the options given at the end of each
questioniscorrect.SectionAcontains25questions,SectionBandSectionCcontain15questionseach.

Markingshallbeasfollows:
 (i) Iftheansweriscorrect:    +3marks
 (ii) Iftheanswerisincorrect:   Ͳ1mark
 (III) Iftheanswerisnotattempted:  0marks
 (iv) Ifmorethanoneboxismarked:  0marks
 Notethatnegativemarkingasindicatedabovewillbeapplicable.

4. Asaroughguideline,thetimespentonquestionsinSectionAshouldbeabout5minuteseach;questions
inSectionBandCshouldtakeabout8minuteseach.Obviously,somequestionsmaytakealittleless
timewhileothersmayrequirealittlemore.

5. WeadviseyoutofirstmarkthecorrectanswersontheQUESTIONPAPERandthentoTRANSFERtheseto
theANSWERSHEETonlywhenyouaresureofyourchoice.

6. Roughworkmaybedoneonblankpagesofthequestionpaper.Ifneeded,youmayaskforextrarough
sheetsfromanInvigilator.

7. Useofcalculatorsispermitted.CalculatorwhichplotsgraphsisNOTallowed.MultipleͲusedevicessuch
ascellphones,smartphonesetc.,CANNOTbeusedforthispurpose.

8. DoNOTaskforclariĮcationsfromtheinvigilatorsregardingthequestions.Theyhavebeeninstructednot
torespondtoanysuchinquiriesfromcandidates.Incaseacorrection/clariĮcationisdeemednecessary,
theinvigilator(s)willannounceitpublicly.

9. Listofusefulphysicalconstantsisgivenonthenextpage.
USEFUL CONSTANTS
Symbol Name/Definition Value
ܿ speed of light in vacuum ͵ ൈ ͳͲ଼ m s-1
԰ reduced Planck constant (ൌ ݄Ȁʹߨ) ͳǤͲͶ ൈ ͳͲିଷସ J s
‫ܩ‬ே gravitational constant ͸Ǥ͸͹ ൈ ͳͲିଵଵ m3 kg-1 s-2
ߝ଴ permittivity of free space ͺǤͺͷ ൈ ͳͲିଵଶ F m-1
ߤ଴ permeability of free space Ͷߨ ൈ ͳͲି଻ N A-2
݁ electron charge (magnitude) ͳǤ͸ ൈ ͳͲିଵଽ C
݉௘ electron mass ͻǤͳ ൈ ͳͲିଷଵ kg
ൌ ͲǤͷ MeV/c 2

ܽ଴ Bohr radius 0.51 Å


ionisation potential of H atom 13.6 eV
ܰ஺ Avogadro number ͸ǤͲʹ͵ ൈ ͳͲଶଷ mol-1
݇஻ Boltzmann constant ͳǤ͵ͺ ൈ ͳͲିଶଷ J K-1
ൌ ͺǤ͸ʹ ൈ ͳͲିହ eV K-1
ܴ ൌ ܰ஺ ݇஻ gas constant ͺǤ͵ͳ J mol-1 K-1
ߛ ൌ ‫ܥ‬௣ Ȁ‫ܥ‬௏ ratio of specific heats: monatomic gas 1.67
diatomic gas 1.40
ߪ Stefan-Boltzmann constant ͷǤ͸͹ ൈ ͳͲି଼ W m-2 K-4
ߙ fine structure constant (ൌ ݁ ଶ ȀͶߨߝ଴ ԰ܿ) 1/137
݃ acceleration due to gravity 9.8 m s-2
ܴா radius of the Earth ͸ǤͶ ൈ ͳͲଷ Km
ܴௌ radius of the Sun ͹ ൈ ͳͲହ Km
݉௣ proton mass (ൎ ʹͲͲͲ ݉௘ ) ͳǤ͹ ൈ ͳͲିଶ଻ kg
ൌ ͻ͵ͺǤʹ MeV/c 2

݉௡ neutron mass (ൎ ʹͲͲͲ ݉௘ ) ͳǤ͹ ൈ ͳͲ ିଶ଻ kg


ൌ ͻ͵ͻǤ͸ MeV/c 2

UNIT CONVERSIONS
Symbol Name/Definition Value
1 A.U. mean distance of Earth from Sun ͳǤͷ ൈ ͳͲଽ km
1 a.m.u. atomic mass unit ͳǤ͸ ൈ ͳͲିଶ଻ kg
ൌ ͻ͵ͳǤͷ MeV/c 2

1 eV electron Volt ͳǤ͸ ൈ ͳͲିଵଽ J


԰ܿ conversion constant ͵Ǥͳ͸ ൈ ͳͲିଶ଺ J m-1
ൌ ͲǤͳͻ͹͵ GeV fm
1T Tesla ͳͲସ gauss
1 bar mean atmospheric pressure at 00 C ͳǤͲͳ ൈ ͳͲହ Pa (= N m-2)

1

GS-2014-X (Physics)
Section A
TobeattemptedbyALLcandidates.

1. The directed beam from a small but powerful searchlight placed on the
ground tracks a small plane flying horizontally at a fixed height ݄ above the
ground with a uniform velocity ‫ ݒ‬, as shown in the figure below.

‫ݒ‬

݄

߱

If the searchlight starts rotating with an instantaneous angular velocity ߱଴


at time ‫ ݐ‬ൌ Ͳ when the plane was directly overhead, then at a later time ‫ݐ‬, its
instantaneous angular velocity ߱ሺ‫ݐ‬ሻ is given by
߱଴
(a) ߱଴ ‡š’ሺെ߱଴ ‫ݐ‬ሻ (b)
ͳ ൅ –ƒ ߱଴ ‫ݐ‬
߱଴ ߱଴
(c) ͳ ൅ ߱଴ଶ ‫ ݐ‬ଶ (d) ͳ െ ߱ ‫ ݐ‬൅ ଵ߱ଶ ‫ ݐ‬ଶ
଴ ଶ ଴

2. The Conservation Principles for energy, linear momentum and angular


momentum arise from the necessity that

(a) the laws of physics should not involve infinite quantities.

(b) internal forces on a body should cancel out, by Newton’s (third) law of
action and reaction.

(c) physical measurements should be independent of the origin and


orientation of the coordinate system.

(d) the laws of physics should be independent of the state of rest or motion
of the observer.

2

3. A uniform ladder of length ʹ‫ ܮ‬and mass ݉ leans
against a wall in a vertical plane at an angle ߠ to the
horizontal. The floor is rough, having a coefficient of
static friction ߤ. A person of mass ‫ ܯ‬stands on the
ladder at a distance ‫ ܦ‬from its base (see figure). If ʹ‫ܮ‬
the wall is frictionless, the maximum distance ‫ܦ‬
ߠ
( ‫ܦ‬୫ୟ୶ ) up the ladder that the person can reach
ߤ
before the ladder slips is
݉ ݉ ݉
(a) ʹߤ‫ ܮ‬ቀͳ ൅ ቁ –ƒ ߠ (b) ቄʹߤ ቀͳ ൅ ቁ –ƒ ߠ െ ቅ ‫ܮ‬
‫ܯ‬ ‫ܯ‬ ‫ܯ‬
݉
(c) ߤ‫ߠ ƒ– ܮ‬ (d) ʹߤ‫ܮ‬ –ƒ ߠ
‫ܯ‬

4. The product ࡹࡺ of two Hermitian matrices ࡹ and ࡺ is anti-Hermitian. It


follows that

(a) ሼࡹǡ ࡺሽ ൌ Ͳ (b) ሾࡹǡ ࡺሿ ൌ Ͳ (c) ࡹற ൌ ࡺ (d) ࡹற ൌ ࡺିଵ

5. A student is asked to find a series approximation for the function


݂ሺ‫ݔ‬ሻin the domain െͳ ൑ ‫ ݔ‬൑ ൅ͳ , as indicated by the thick line in the figure
below.

݂ሺ‫ݔ‬ሻ
൅ͳ

൅ଷ

‫ݔ‬
െͳ Ͳ ൅ͳ
The student represents the function by a sum of three terms

ߨ‫ݔ‬ ߨ‫ݔ‬
݂ሺ‫ݔ‬ሻ ൎ ܽ଴ ൅ ܽଵ …‘• ൅  ܽଶ •‹
ʹ ʹ

Which of the following would be the best choices for the coefficients ܽ଴ , ܽଵ
and ܽଶ ?

ͳ ʹ ʹ
(a) ܽ଴ ൌ ͳǡ ܽଵ ൌ െ ǡ ܽଶ ൌ Ͳ (b) ܽ଴ ൌ ǡ ܽଵ ൌ െ ǡ ܽଶ ൌ Ͳ
͵ ͵ ͵
ʹ ʹ ͳ
(c) ܽ଴ ൌ  ǡ ܽଵ ൌ Ͳǡ ܽଶ ൌ െ (d) ܽ଴ ൌ െ ǡ ܽ ൌ Ͳǡ ܽଶ ൌ െͳ
͵ ͵ ͵ ଵ

3

6. The probability function for a variable which assumes only positive values is

exp

where  0. The ratio 〈 〉/ , where is the most probable value and 〈 〉 is
the mean value of the variable , is

1  1
(a) 2 (b) (c) (d) 1
1  

7. A solid spherical conductor has a conical hole made at A 


one end, ending in a point B, and a small conical
projection of the same shape and size at the opposite
side, ending in a point A. A cross-section through the
centre of the conductor is shown in the figure on the B 
right.

If, now, a positive charge is transferred to the sphere,


then

(a) the charge density at both A and B will be undefined.

(b) the charge density at A will be the same as the charge density at B.

(c) the charge density at A will be more than the charge density at B.

(d) the charge density at B will be more than the charge density at A.

8. Solving Poisson’s equation / for the electrostatic potential


in a region with a constant charge density , two students find different
answers, viz.

        and           

The reason why these different solutions are both correct is because

(a) space is isotropic and hence and are physically equivalent.

(b) we can add solutions of Laplace’s equation to both and .

(c) the electrostatic energy is infinite for a constant charge density.

(d) the boundary conditions are different in the two cases.


 
9. A short solenoid with ݊ turns per unit
P
length has diameter ‫ ܦ‬and length
‫ ܮ‬ൌ ͺ‫ܦ‬Ȁͳͷ , as shown in the figure, ଶ
‫ ܪ‬ൌ ଷ‫ܦ‬
and it carries a constant current ‫ ܫ‬.

The magnetic field ‫ ܤ‬at a point P on ଼


‫ ܮ‬ൌ ଵହ‫ܦ‬
the axis of the solenoid at a distance
‫ ܪ‬ൌ ʹ‫ܦ‬Ȁ͵ from its near end (see
figure) is ‫ܦ‬
[use ‫ ݔ݀ ׬‬ሺͳ ൅ ‫ݔ‬ ଶ ሻିଷȀଶ
ൌ ‫ݔ‬ሺͳ ൅ ‫ݔ‬ ଶ ሻିଵȀଶ
]

Ͷ Ͷ
(a) ߤ ݊‫ܫ‬ (b) ߤ ݊‫ܫ‬
͸ͷ ଴ ͳ͵ ଴

ʹͶ ͳͳʹ
(c) ߤ ݊‫ܫ‬ (d) ߤ ݊‫ܫ‬
ͳͷ ଴ ͸ͷ ଴

10. A particle moving in one dimension has the un-normalised wave function
‫ݔ‬ଶ
߰ሺ‫ݔ‬ሻ ൌ ‫ ’š‡ ݔ‬ቆെ ቇ
Oଶ
where O is a real constant. The expectation value of its momentum is ‫ ۄ݌ۃ‬ൌ

԰ ‫ݔ‬ଶ ԰ଶ ԰
(a) ‡š’ ቆെ ଶቇ
(b) െʹ
O O O ଶ O
԰
(c) ‡š’ሺെͳሻ (d) zero
O

11. A particle of mass ݉ and charge ݁ is in the ground state of a one-


dimensional harmonic oscillator potential in the presence of a uniform
external electric field ‫ ܧ‬. The total potential felt by the particle is
ͳ
ܸሺ‫ݔ‬ሻ ൌ ݇‫ ݔ‬ଶ െ ݁‫ݔܧ‬
ʹ
If the electric field is suddenly switched off, then the particle will

(a) make a transition to any harmonic oscillator state with ‫ ݔ‬ൌ െ݁‫ܧ‬Ȁ݇ as
origin without emitting any photon.
(b) make a transition to any harmonic oscillator state with ‫ ݔ‬ൌ Ͳ as origin
and absorb a photon.
(c) settle into the harmonic oscillator ground state with ‫ ݔ‬ൌ Ͳ as origin
after absorbing a photon.
(d) oscillate back and forth with initial amplitude ݁‫ܧ‬Ȁ݇ , emitting multiple
photons as it does so.

5

12. Consider the Hamiltonian
‫ ܪ‬ൌ ݂ ߪറ ή ‫ݔ‬റ
Here ‫ݔ‬റ is the position vector, ݂ is a constant and ߪറ ൌ ൫ߪ௫ ǡ ߪ௬ ǡ ߪ௭ ൯ , where
ߪ௫ ǡ ߪ௬ ǡ ߪ௭ are the three Pauli matrices. The energy eigenvalues are

(a) ݂ ቀඥ‫ ݔ‬ଶ ൅ ‫ ݕ‬ଶ  േ ‫ݖ‬ቁ (b) ݂ሺ‫ ݔ‬േ ݅‫ݕ‬ሻ

(c) േ݂ඥ‫ ݔ‬ଶ ൅ ‫ ݕ‬ଶ ൅ ‫ ݖ‬ଶ (d) േ݂ሺ‫ ݔ‬൅ ‫ ݕ‬൅ ‫ݖ‬ሻ

13. One mole of an ideal gas undergoes the cycle ACBA shown in the ‫ܸ݌‬diagram
below.

‫݌‬

A
‫݌‬ଵ 

‫݌‬ଶ  B
C
ܸ
ܸଵ  ܸଶ ܸଷ

One of the curved lines in the cycle represents an isothermal change at


temperature ܶ, while the other represents an adiabatic change. The net heat
gained by the gas in this cycle is

ܸଶ ܸଷ
(a) െ‫݌‬ଶ ሺܸଷ െ ܸଶ ሻ ൅ ܴܶ Ž (b) െ‫݌‬ଶ ሺܸଷ െ ܸଶ ሻ ൅ ܴܶ Ž
ܸଵ ܸଵ

ଵିఊ ଵିఊ ܸଷ
(c) െ‫݌‬ଶ ሺܸଷ െ ܸଶ ሻ ൅ ߛܴܶ൫ܸଶ െ ܸଵ ൯ (d) ሺ‫݌‬ଵ ܸଵ െ ‫݌‬ଶ ܸଶ ሻ െ ܴܶ Ž
ܸଵ

14. An ideal gas at a temperature ܶ is enclosed in a rigid container whose walls


are initially at temperature ܶଵ , where ܶଵ ൏ ܶ. The walls are covered on the
outside with perfect thermal insulation and the system is allowed to come to
equilibrium. The pressure exerted by the gas on the walls of the container

(a) remains constant throughout.

(b) is lower at the initial stage than at the final stage.

(c) is higher at the initial stage than at the final stage.

(d) is the same at the initial and final stages.

6

15. Consider the CO molecule as a system of two point particles which has both
translational and rotational degrees of freedom. Using classical statistical
mechanics, the molar specific heat ‫ܥ‬௏ of CO gas is given in terms of the
Boltzmann constant ݇஻ by

ͷ ͵ ͳ
(a) ݇ (b) ʹ݇஻ (c) ݇ (d) ݇
ʹ ஻ ʹ ஻ ʹ ஻

16. In the laboratory frame, two observers A and B are A


moving along the sides of an equilateral triangle with ܿȀʹ
equal speeds ܿȀʹ, as shown in the figure.
ܿȀʹ
The speed of Bas measured by Awill be B

Ͷ ξͷ
(a) ξ͵ ܿ (b) ܿ (c) ξͳ͵
ܿ (d) ܿ
ʹ ͵ξ͵ ͹ ͵

17. Two telescopes X and Y have identical objective lenses, but the single-lens
eyepiece of X is converging whereas the single-lens eyepiece of Y is diverging.
If the magnification ‫ ܯ‬of these two telescopes for objects at infinity is the
same, the lengths ‫ܮ‬௑  and ‫ܮ‬௒ of the two telescopes (length of a telescope is
defined as the distance between the objective lens and the eyepiece) must be
in the ratio
‫ܮ‬௑ Τ‫ܮ‬௒ ൌ

ʹ‫ ܯ‬൅ ͳ ʹ‫ ܯ‬െ ͳ ‫ܯ‬൅ͳ ‫ܯ‬െͳ


(a) (b) (c) (d)
ʹ‫ ܯ‬െ ͳ ‫ܯ‬൅ͳ ‫ܯ‬െͳ ‫ܯ‬൅ͳ

18. A lens can be constructed using a flat circular glass plate whose refractive
index ݊ varies radially, i.e. ݊ ൌ ݊ሺ‫ݎ‬ሻ, where ‫ ݎ‬is the radial distance from the
centre of the plate. In order to make a convex lens by this method ݊ሺ‫ݎ‬ሻ should
vary (in terms of positive constants ݊ሺͲሻ and ߙ) as

(a) ݊ሺͲሻ െ ߙȀ‫ ݎ‬ଶ (b) ݊ሺͲሻ െ ߙȀ‫ݎ‬ (c) ݊ሺͲሻ െ ߙ‫ݎ‬ (d) ݊ሺͲሻ െ ߙ‫ ݎ‬ଶ

19. A solid sample has the property that, when cooled below a certain
temperature, it expels any small applied magnetic field from within the
material. Which of the following best describes this sample in the cooled
state?

(a) Paramagnet (b) Diamagnet

(c) Ferromagnet (d) Anti-ferromagnet

7

20. A beam of atoms moving in a certain direction can be slowed down if they
absorb photons from a laser beam moving in the opposite direction and
subsequently spontaneously emit photons isotropically. For a beam of
Sodium atoms (mass number ‫ ܣ‬ൌ ʹ͵) with speed 600 ms-1, if a laser beam of
wavelength 589 nm is used, the number of such absorption and emission
cycles needed to bring a Sodium atom to rest would be approximately

(a) ͳǤ͵ ൈ ͳͲହ (b) ͳǤ͵ ൈ ͳͲସ (c) ʹǤͳ ൈ ͳͲଷ (d) ʹǤͳ ൈ ͳͲସ

21. In a laboratory, the double-slit experiment is performed with free non-


relativistic electrons, each having energy ‫ ܧ‬, emitted from a source S (see
figure below). The screen consists of a uniform sheet of charge-sensitive
pixels of size ‫ݎ‬. If the slit-screen distance is ‫ ݖ‬and the spacing between slits is
݀ , which of the following restrictions on the electron energy ‫ ܧ‬should be
satisfied so that the fringes can be distinctly observed?

screen
݀
S
‫ݖ‬

ͳ ݄‫ ݖ‬ଶ ͳ ݄‫ ݖ‬ଶ
(a) ‫ܧ‬ ൑  ൬ ൰ (b) ‫ ܧ‬൒ ൬ ൰
ʹ݉௘ ‫݀ݎ‬ ʹ݉௘ ‫݀ݎ‬

݄‫ݖ‬ ݄‫ݖ‬
(c) ‫ ܧ‬൑ ܿ ൬ ൰ (d) ‫ ܧ‬൒ ܿ ൬ ൰
‫݀ݎ‬ ‫݀ݎ‬

22. An alpha particle of energy ‫ ܧ‬is shot towards a gold nucleus ( ଵଽ଻ ଻ଽ— ). At
distances much larger than the nuclear size ܴே , the dominant force is the
Coulomb repulsion, but at distances comparable to the nuclear size the
dominant force is the strong nuclear attraction. These combine to form a
potential barrier of height ܸ஼ . If ‫ ܧ‬൏  ܸ஼ , the probability that the alpha particle
will fuse with the gold nucleus can be written (in terms of a dimensionless
positive constant ݇ ) as

݇‫ܧ‬ ‫ܧ‬ ܸ݇஼


(a) zero (b) (c) ݇ ൬ͳ െ ൰ (d) ‡š’ ൬െ ൰
ඥ݇ ଶ ‫ ܧ‬ଶ ൅ ܸ஼ଶ ܸ஼ ‫ܧ‬

8

23. Consider the following circuit.

Which of the graphs given below is a correct representation of ܸ୭୳୲ ?

(a) (b)
Vout

Vout

time (t) time (t)


(c) (d)
Vout
Vout

time (t) time (t)

24. In the following circuit, the resistance  ଶ is doubled.

It follows that the current through  ଶ

(a) remains the same. (b) is halved.


(c) is doubled. (d) is quadrupled.

9

25. A control circuit needs to be designed to save on power consumption by an
air-conditioning unit A in a windowless room with a single door. The room is
fitted with the following devices:
1. a temperature sensor T, which is enabled (  ൌ ͳ ) whenever the
temperature falls below a pre-set value;
2. a humidity sensor H, which is enabled ( ൌ ͳ) whenever the humidity falls
below a certain pre-set value;
3. a sensor D on the door, which is triggered (  ൌ ͳ) whenever the door
opens.
Which of the following logical circuits will turn the air-conditioning unit off
( ൌ Ͳ) whenever the door is opened or when both temperature and humidity
are below their pre-set values?

(a) (b) D
D 

T T A
A
H
H
(c) (d) D
D 

T A T A

H H

10

GS-2014-X (Physics)
Section B
TobeattemptedonlybycandidatesforIntegratedPh.D.programme.
(Candidates for Ph.D. programme will get NO credit for attempting this section.)

26. A body of mass ݉ falls from rest at a height ݄ under gravity (acceleration
due to gravity ݃) through a dense medium which provides a resistive force
‫ ܨ‬ൌ െ݇‫ ݒ‬ଶ , where ݇ is a constant and ‫ ݒ‬is the speed. It will hit the ground
with a kinetic energy

݉ଶ ݃ ʹ݄݇ ݉ଶ ݃ ʹ݄݇
(a) ‡š’ ൬െ ൰ (b) –ƒŠ
ʹ݇ ݉ ʹ݇ ݉
݉ଶ ݃ ʹ݄݇ ݉ଶ ݃ ʹ݄݇
(c) ൜ͳ ൅ ‡š’ ൬െ ൰ൠ (d) ൜ͳ െ ‡š’ ൬െ ൰ൠ
ʹ݇ ݉ ʹ݇ ݉

27. A weight ܹ is suspended from a rigid support by a


hard spring with stiffness constant ‫ ܭ‬. The spring is
‫ܭ‬
enclosed in a hard plastic sleeve, which prevents
ܹ
horizontal motion, but allows vertical oscillations (see
figure). A simple pendulum of length κ with a bob of
mass ݉ (݉݃ ‫ ) ܹ ا‬is suspended from the weight ܹ and
is set oscillating in a horizontal line with a small κ
amplitude. After some time has passed, the weight ܹ is
observed to be oscillating up and down with a large
amplitude (but not hitting the sleeve).
It follows that the stiffness constant ‫ ܭ‬must be ݉

Ͷܹ ʹܹ ܹ ܹ
(a) ‫ܭ‬ൌ (b) ‫ ܭ‬ൌ (c) ‫ ܭ‬ൌ (d) ‫ ܭ‬ൌ
κ κ κ ʹκ

28. In spherical polar coordinates ‫ݎ‬റ ൌ ሺ‫ݎ‬ǡ ߠǡ ߮ሻ the delta function ߜሺ‫ݎ‬റଵ െ ‫ݎ‬റଶ ሻ can
be written as

(a) ߜሺ‫ݎ‬ଵ െ ‫ݎ‬ଶ ሻ ߜሺߠଵ െߠଶ ሻߜሺ߮ଵ െ ߮ଶ ሻ


ͳ
(b) ߜሺ‫ݎ‬ଵ െ ‫ݎ‬ଶ ሻߜሺ…‘• ߠଵ െ …‘• ߠଶ ሻ ߜሺ߮ଵ െ ߮ଶ ሻ
‫ݎ‬ଵଶ
ͳ
(c) ߜሺ‫ݎ‬ଵ െ ‫ݎ‬ଶ ሻߜሺ…‘• ߠଵ െ …‘• ߠଶ ሻ ߜሺ߮ଵ െ ߮ଶ ሻ
ȁ‫ݎ‬റଵ െ ‫ݎ‬റଶ ȁଶ
ͳ
(d) ߜሺ‫ݎ‬ଵ െ ‫ݎ‬ଶ ሻߜሺߠଵ െߠଶ ሻ ߜሺ߮ଵ െ ߮ଶ ሻ
‫ݎ‬ଵଶ …‘• ߠଵ

11

‫ݖ‬
29. An electric dipole is constructed by fixing two circular charged
rings, each of radius ܽ, with an insulating contact (see figure). ൅ܳ
One of these rings has total charge ൅ܳ and the other has total
charge Ȃ ܳ . If the charge is distributed uniformly along each െܳ
ring, the dipole moment about the point of contact will be
ܳܽ
(a) ‫ݖ‬Ƹ (b) Ͷߨܳܽ ‫ݖ‬Ƹ
ߨ
(c) ʹܳܽ‫ݖ‬Ƹ (d) zero

30. A spherical conductor, carrying a total charge ܳ , spins uniformly and very
rapidly about an axis coinciding with one of its diameters. In the diagrams
given below, the equilibrium charge density on its surface is represented by
the thickness of the shaded region. Which of these diagrams is correct?

(a) (b)

(c) (d)

31. A rigid rotator is in a quantum state described by the wavefunction

͵
߰ሺߠǡ ߮ሻ ൌ  ඨ •‹ ߠ •‹ ߮
Ͷߨ
where ߠ and ߮ are the usual polar angles. If two successive measurements of
‫ܮ‬௭ are made on this rotator, the probability that the second measurement will
yield the value ൅԰ is

(a) ͲǤʹͷ (b) ͲǤ͵͵ (c) ͲǤͷ (d) negligible

12

32. A particle in the ʹ‫ ݏ‬state of hydrogen has the wave function

ͳ ͳ ଷȀଶ ‫ݎ‬ ‫ݎ‬


߰ଶ௦ ሺ‫ݎ‬ሻ ൌ ൰ ൬ʹ െ ൰‡š’ ൬െ
൬ ൰
Ͷξʹߨ ܽ଴ ܽ଴ ʹܽ଴
where ‫ ݎ‬is the radial coordinate w.r.t. the nucleus as origin and ܽ଴ is the Bohr
radius. The probability ܲ of finding the electron somewhere inside a sphere of
radius Oܽ଴ centred at the nucleus, is best described by the graph

(a) (b)
ܲ ܲ

O
  O 

(c) (d)

ܲ ܲ

O O

33. A thermally-insulated container of volume ܸ଴ is divided into two equal halves


by a non-permeable partition. A real gas with equation of state

ܽଶ
ܾ ଷ ቆ‫ ݌‬൅ ቇ ൌ ܴ݊ܶ
ܸଷ
where ܽ and ܾ are constants, is confined to one of these halves at a
temperature ܶ଴ . The partition is now removed suddenly and the gas is allowed
to expand to fill the entire container. The final temperature of the gas, in
terms of its specific heat ‫ܥ‬௏ , will be

͵ܽଶ ʹܽଶ
(a) ܶ଴ െ (b) ܶ଴ െ
ʹ‫ܥ‬௏ ܸ଴ଶ ͵‫ܥ‬௏ ܸ଴ଶ

͵ܽଶ ʹܽଶ
(c) ܶ଴ ൅ (d) ܶ଴ ൅
ʹ‫ܥ‬௏ ܸ଴ଶ ͵‫ܥ‬௏ ܸ଴ଶ

13

34. A manufactturer is ab
ble to offe
er two mod
dels of he
eat-conserv
ving windo
ows, as
de
escribed below.
b

Window A is a simplle pane off glass, 4 mm thick


W k. Window w B, on the other
hand, consiists of two extremelyy thin panees of glass,, separated d by an airr gap of
2 mm, as shown
s in the figure above. If the thermmal conduc ctivity of glass
g is
kn
nown to beb 0.8 Wm m K and that of airr is 0.025 Wm K , then the ratio of
-1 -1 -1 -1

heat flow ܳ஺ through Window A to the hea at flow ܳ஻ through W Window B is i given
y
by
ܳ஺

ܳ஻

ͳ ͳ
(a
a) (b) (c) Ͷ (d) ͳ͸
ͳ͸ Ͷ

n a low te
35. In emperature experim ment, the resistance
r of a senssor is use
ed as a
th
hermometeer. In order to have better
b senssitivity in the
t range 100 mK too 1.0 K,
w
which mateerial would
d make thee best senssor?

(a
a) insulator (b) p-n juncttion

(c
c) pure se
emiconduc
ctor (d) metal

36. For a pure germaniu um semiconductor co ooled in liiquid nitro


ogen, the average
a
deensity of conduction
c n electrons is about ݊ ൌ ͳͲଵଶ pe er cm3. At this temperature
th
he electron n and ho ole mobilitties are equal
e and have the e common n value

ߤ ൌ ͷǤͲ ൈ ͳͲ Ͳ cm V s . If a po
2 -1 -1 otential of 100 V is applied
a acrross a 1 cm
m cube
ed germanium sample, the currrent obserrved can be estimate
off this coole ed as

a) 80 PA
(a (b) 160 mA

(c
c) 16 mA
A (d) 16 A

14

37. A glass plate P (refractive index ݊௉ ൌ ͳǤͷͶ) is coated with a dielectric material
C with the refractive index ݊஼ ൌ ͳǤ͸. In order to have enhanced reflection from
this coated glass for near-normal incident light of wavelength O, the thickness
of the coating material C must be

(a) even multiples of OΤʹ݊஼ (b) even multiples of OΤͶ݊஼

(c) odd multiples of OΤͶ݊஼ (d) integral multiples of OΤͶ݊஼

38. In the following circuit, the AC source is an ideal voltage source. What is the
amplitude of the steady state current through the inductor at resonance?

(a) ܸ௢ ඥ‫ ܥ‬Τ‫ܮ‬ (b) ܸ௢ Τܴ

(c) ܸ௢ ඥ‫ ܥ‬Τሺܴଶ ‫ ܥ‬൅ ʹ‫ܮ‬ሻ (d) zero

39. A standard radioactive source is known to decay by emission of ߛ rays. The


source is provided to a student in a thick sealed capsule of unbreakable
plastic and she is asked to find out the half-life. Which of the following
would be the most useful advice to the student?

(a) The half-life cannot be measured because the initial concentration of


the source is not given.

(b) Mount the source in front of a gamma ray detector and count the
number of photons detected in one hour.

(c) Measure the mass of the source at different times with an accurate
balance having a least count of 1 mg. Plot these values on a curve and
fit it with an exponential decay law.

(d) Mount the source in front of a gamma ray detector and count the
number of photons detected in a specific time interval. Repeat this
experiment at different times and note how the count changes.

15

40. Cosmic ray muons generated at the top of the Earth’s atmosphere decay
according to the radioactive decay law
ͲǤ͸ͻ͵‫ݐ‬
ܰሺ‫ݐ‬ሻ ൌ ܰሺͲሻ ‡š’ ቆെ ቇ
ܶଵȀଶ
where ܰሺ‫ݐ‬ሻ is the number of muons at time ‫ݐ‬, and ܶଵȀଶ ൌ ͳǤͷʹPs is the proper
half-life of the muon. Immediately after generation, most of these muons
shoot down towards the Earth’s surface. Some of these muons decay on the
way, but their interaction with the atmosphere is negligible.

An observer on the top of a mountain of height 2.0 km above mean sea level
detects muons with the speed ͲǤͻͺܿ over a period of time and counts 1000
muons. The number of muons of the same speed detected by an observer at
mean sea level in the same period of time would be

(a) 232 (b) 539

(c) 839 (d) 983

16

GS-2014-X (Physics)
Section C
TobeattemptedonlybycandidatesforPh.D.programme.
(Candidates for Integrated Ph.D. programme will get NO credit for attempting this section.)

41. The integral



݀‫ݔ‬

Ͷ ൅ ‫ݔ‬ସ

evaluates to

ߨ ߨ ߨ
(a) ߨ (b) (c) (d) ͺ
ʹ Ͷ

42. The solution of the integral equation


݂ሺ‫ݔ‬ሻ ൌ ‫ ݔ‬െ න ݀‫݂ ݐ‬ሺ‫ݐ‬ሻ



has the graphical form

(a) (b)

(c) (d)

17

43. Assume that the Earth is a uniform sphere of radius ܴ, rotating about its
axis with a uniform angular velocity ߱ . A rocket is launched from the
Equator in a direction due North. If it keeps on flying at a uniform speed ‫ݒ‬
(neglecting air resistance), the highest latitude that can be achieved is
ߨ ߨ ܴ߱
(a) (b) െ ሺߨ െ ʹሻ
ʹ ʹ ‫ݒ‬
ߨ ܴ߱ ߨ ʹܴ߱
(c) െ ሺߨ ൅ ʹሻ (d) ൬ͳ െ ൰
ʹ ‫ݒ‬ ʹ ‫ݒ‬

44. A particle P of mass ݉ moves under the influence of a central potential,


centred at the origin O, of the form
݇
ܸሺ‫ݎ‬ሻ ൌ െ
͵‫ ݎ‬ଷ
where ݇ is a positive constant
‫ݑ‬ P

O

If the particle P comes in from infinity with initial velocity ‫ ݑ‬and impact
parameter ܾ (see figure), then the largest value of ܾ for which the particle gets
captured by the potential is
ଵȀ଺
͵݇ ଶ ݇ ଵȀଷ
(a) ቆ ቇ (b) ൬ ൰
݉ଶ ‫ ݑ‬ସ ͵݉‫ݑ‬
ଵȀ଺ ଵȀଷ
ʹ݇ ଶ ʹ݇
(c) ቆ ଶ ସቇ (d) ൬ ൰
݉ ‫ݑ‬ ͵݉‫ݑ‬

45. The instantaneous electric and magnetic fields created at a distance ‫ ݎ‬by a
point source at the origin are given by
‫ݐ߱ •‘… ܣ‬ ‫ݐ߱ •‘… ܤ‬
‫ܧ‬ሬറ ൌ ߠ෠‫ܪ‬
ሬറ ൌ ߮ො
ʹߨߝ଴ ‫ݎ‬ ߤ଴ ‫ݎ‬
where ߱ǡ ‫ܣ‬ǡ ‫ ܤ‬are constants, and the unit vectors ൫‫ݎ‬Ƹ ǡ ߠ෠ ǡ ߮
ො൯ form an orthonormal
set. The time-averaged power radiated by the source is
߱ߝ଴ ܿଷ ʹߨ߱
(a) ‫ܤܣ‬ (b) ‫ܤܣ‬ (c) ܿ ଶ ‫ܤܣ‬ (d) ‫ܤܣ‬
ߤ଴ ʹߨ ܿ

18

46. A particle is confined to a one-dimensional box of length ‫ܮ‬. If a vanishingly
thin but strongly repulsive partition is introduced in the exact centre of the
box, and the particle is allowed to come to its ground state, then the
probability density for finding the particle will appear as

(a) (b)

(c) (d)

47. A spin-2 nucleus absorbs a spin-½ electron and is then observed to decay to a
stable nucleus in two stages, recoiling against an emitted invisible particle in
the first stage and against an emitted spin-1 photons in the second stage. If
the stable nucleus is spinless, then the set of all possible spin values of the
invisible particle is

(a) ሼͳΤʹ ǡ ͷΤʹሽ (b) ሼ͵Τʹ ǡ ͹Ȁʹሽ

(c) ሼͳΤʹ ǡ ͵Τʹ ǡ ͷȀʹሽ (d) ሼͳΤʹ ǡ ͵Τʹ ǡ ͷΤʹ ǡ ͹Ȁʹሽ

48. A gas of photons is enclosed in a container of fixed volume at an absolute


temperature ܶ. Noting that the photon is a massless particle (i.e., its energy
and momentum are related by ‫ ܧ‬ൌ ‫) ܿ݌‬, the number of photons in the
container will vary as

(a) ܶ (b) ܶ ଶ (c) ܶ ଷ (d) ܶ ସ

19

49. In a Stern-Gerlach experiment with spin-½ particles, the beam is found to
form two spots on the screen, one directly above the other. The experimenter
now makes a hole in the screen at the position of the upper spot. The
particles that go through this hole are then passed through another Stern-
Gerlach apparatus but with its magnets rotated by 90 degrees counter-
clockwise about the axis of the beam direction. Which of the following shows
what happens on the second screen?

(a) (b)

 
(c) (d)

50. The ground state electronic configuration for a carbon atom is


ሺͳ‫ݏ‬ሻଶ ሺʹ‫ݏ‬ሻଶ ሺʹ‫݌‬ሻଶ .
The first excited state of this atom would be achieved by

(a) re-alignment of the electron spins within the ʹ‫ ݌‬orbital.


(b) transition of an electron from the ʹ‫ ݏ‬orbital to the ʹ‫ ݌‬orbital.
(c) transition of an electron from the ʹ‫ ݌‬orbital to the ͵‫ ݏ‬orbital.
(d) transition of an electron from the ʹ‫ ݏ‬orbital to the ͵‫ ݏ‬orbital.

51. Which of the following statement best explains why the specific heat of
electrons in metals is much smaller than that expected in a non-interacting
(free) electron gas model?

(a) The mass of electron is much smaller than that of the ions in the
crystal.

(b) The Pauli exclusion principle restricts the number of electrons which
can absorb thermal energy.

(c) Electron spin can take only two different values.

(d) Electrons in a metal cannot be modelled as non-interacting.

20

52. In a beta decay experiment, an electromagnet M and a detector D are used to
measure the energy of electrons (ߚ ି ), as shown in the figure.

The detector D is capable of detecting either electrons ሺߚ ି ሻ or positrons ሺߚ ା ሻ.


Now the ߚ ି source is replaced with a ߚ ା source, and we would like to
measure the energy of the positrons (ߚ ା ) using the same setup. Which of the
following is correct?

(a) This can be done quite easily, if the polarity of current in the coils of the
electromagnet is reversed.

(b) This can be done trivially, without changing anything, since the
detector D can detect either ߚ ି or ߚ ା .

(c) There is no way to do this with the given set up, since ߚ ା will have to be
converted into ߚ ି , which is obviously not possible.

(d) This cannot be done since the magnet does not have a symmetric
shape.

53. It is well-known that the energy of the Sun arises from the fusion of
hydrogen nuclei (protons) inside the core of the Sun. This takes place
through several mechanisms, each resulting in emission of energy.
Which of the following reactions is NOT possible during the proton fusion
inside the Sun?

(a) ଵ
ଵ ൅ ଵଵ  ՜  ଶଶ ‡

(b) ଶ
ଵ ൅ ଵଵ  ՜  ଷଶ ‡

(c) ଵ
ଵ ൅ ଵଵ  ՜  ଶଵ ൅ ݁ ା ൅ ߥ௘

(d) ଵ
ଵ ൅ ଵଵ  ൅ ଵଵ ൅ ଵଵ  ՜  ସଶ ‡ ൅ ʹ݁ ା

21

54. A group of alien astronomers far away from the solar system tries to find out
if the Sun (visible to them as a small yellow star) has planets orbiting around
it. The method they use is to look for wobbles in the motion of the Sun
induced by the planet(s) revolving around it (if any). To detect this motion,
they build a high-resolution spectrometer which can measure the Doppler
shift in frequency of a 600 nm line in the solar spectrum with an accuracy of
1 in ͳͲ଺ .
Given that the Sun has a mass ʹ ൈ ͳͲଷ଴ kg and that Earth (mass ͸ ൈ ͳͲଶସ kg,
orbital velocity ͵ ൈ ͳͲସ ms-1) and Jupiter (mass ʹ ൈ ͳͲଶ଻ kg, orbital velocity
ͳǤͷ ൈ ͳͲସ ms-1) are two typical planets, one could predict that the experiments
conducted by the aliens would find

(a) evidence for both the planets Earth and Jupiter.

(b) evidence for the planet Jupiter, but not for the planet Earth.

(c) no evidence for any planets orbiting the Sun.

(d) evidence for planets, but will not be able to tell how many.

55. The interaction strength of the recently-discovered Higgs boson (mass


approximately 125 GeV/ ܿ ଶ ) with any other elementary particle is
proportional to the mass of that particle. Which of the following decay
processes will have the greatest probability?

(a) Higgs boson decaying to a top quark + a top anti-quark.

(b) Higgs boson decaying to a bottom quark + a bottom anti-quark.

(c) Higgs boson decaying to an electron and a positron.

(d) Higgs boson decaying to a neutrino-antineutrino pair.

22

DO NOT WRITE ON THIS SHEET

23

Correct answers are ticked in green.
PHY - X
GS-2015 (Physics)
TATA INSTITUTE OF FUNDAMENTAL RESEARCH
Written Test in PHYSICS - December 14, 2014
Duration : Three hours (3 hours)

Name : _______________________________________________ Ref. Code : ____________

Please read all instructions carefully before you attempt the questions.

1. Please fill-in details about name, reference code etc. on the question paper and answer sheet. The
Answer Sheet is machine-readable. Use only black/blue ball point pen to fill-in the answer sheet.

2. Indicate your ANSWER ON THE ANSWER SHEET by blackening the appropriate circle for each question.
Do not mark more than one circle for any question : this will be treated as a wrong answer.

3. This test consists of three parts, Section A, Section B and Section C. You must answer questions
according to the instructions below :
Candidates applying for Must Answer Should Not Attempt
Integrated Ph.D. Section A and Section B Section C
Ph.D. Section A and Section C Section B

The test contains multiple choice-type questions. Only ONE of the options given at the end of each
question is correct. Section A contains 25 questions, Section B and Section C contain 15 questions each.

Marking shall be as follows :


(i) If the answer is correct : +3 marks
(ii) If the answer is incorrect : -1 mark
(III) If the answer is not attempted : 0 marks
(iv) If more than one box is marked : 0 marks
Note that negative marking as indicated above will be applicable.

4. As a rough guideline, the time spent on questions in Section A should be about 5 minutes each; questions
in Section B and C should take about 8 minutes each. Obviously, some questions may take a little less
time while others may require a little more.

5. We advise you to first mark the correct answers on the QUESTION PAPER and then to TRANSFER these to
the ANSWER SHEET only when you are sure of your choice.

6. Rough work may be done on blank pages of the question paper. If needed, you may ask for extra rough
sheets from an Invigilator.

7. Use of calculators is permitted. Calculator which plots graphs is NOT allowed. Multiple-use devices such
as cell phones, smartphones etc., CANNOT be used for this purpose.

8. Do NOT ask for clariĮcations from the invigilators regarding the questions. They have been instructed not
to respond to any such inquiries from candidates. In case a correction/clariĮcation is deemed necessary,
the invigilator(s) will announce it publicly.

9. List of useful physical constants is given on the next page.


USEFUL CONSTANTS
Symbol Name/Definition Value
ܿ speed of light in vacuum ͵ ൈ ͳͲ଼ m s-1
԰ reduced Planck constant (ൌ ݄Ȁʹߨ) ͳǤͲͶ ൈ ͳͲିଷସ J s
‫ܩ‬ே gravitational constant ͸Ǥ͸͹ ൈ ͳͲିଵଵ m3 kg-1 s-2
‫ٖܯ‬ solar mass ͳǤͻͺͻ ൈ ͳͲଷ଴ kg
ߝ଴ permittivity of free space ͺǤͺͷ ൈ ͳͲିଵଶ F m-1
ߤ଴ permeability of free space Ͷߨ ൈ ͳͲି଻ N A-2
݁ electron charge (magnitude) ͳǤ͸ ൈ ͳͲିଵଽ C
݉௘ electron mass ͻǤͳ ൈ ͳͲିଷଵ kg
ൌ ͲǤͷ MeV/c 2

ܽ଴ Bohr radius 0.51 Å


ionisation potential of H atom 13.6 eV
ܰ஺ Avogadro number ͸ǤͲʹ͵ ൈ ͳͲଶଷ mol-1
݇஻ Boltzmann constant ͳǤ͵ͺ ൈ ͳͲିଶଷ J K-1
ൌ ͺǤ͸ʹ ൈ ͳͲିହ eV K-1
ܴ ൌ ܰ஺ ݇஻ gas constant ͺǤ͵ͳ J mol-1 K-1
ߛ ൌ ‫ܥ‬௣ Ȁ‫ܥ‬௏ ratio of specific heats: monatomic gas 1.67
diatomic gas 1.40
ߪ Stefan-Boltzmann constant ͷǤ͸͹ ൈ ͳͲି଼ W m-2 K-4
ߙ fine structure constant (ൌ ݁ ଶ ȀͶߨߝ଴ ԰ܿ) 1/137
݃ acceleration due to gravity 9.8 m s-2
ܴா radius of the Earth ͸ǤͶ ൈ ͳͲଷ Km
ܴௌ radius of the Sun ͹ ൈ ͳͲହ Km
݉௣ proton mass (ൎ ʹͲͲͲ݉௘ ) ͳǤ͹ ൈ ͳͲିଶ଻ kg
ൌ ͻ͵ͺǤʹ MeV/c 2

݉௡ neutron mass (ൎ ʹͲͲͲ݉௘ ) ͳǤ͹ ൈ ͳͲିଶ଻ kg


ൌ ͻ͵ͻǤ͸ MeV/c 2

UNIT CONVERSIONS
Symbol Name/Definition Value
1 A.U. mean distance of Earth from Sun ͳǤͷ ൈ ͳͲଽ km
1 a.m.u. atomic mass unit ͳǤ͸ ൈ ͳͲିଶ଻ kg
ൌ ͻ͵ͳǤͷ MeV/c 2

1 eV electron Volt ͳǤ͸ ൈ ͳͲିଵଽ J


1T Tesla ͳͲସ gauss
1 bar mean atmospheric pressure at 00 C ͳǤͲͳ ൈ ͳͲହ Pa (= N m-2)
1Å Ångstrom unit 10-8 cm
԰ܿ conversion constant ͵Ǥͳ͸ ൈ ͳͲିଶ଺ J m-1
ൌ ͲǤͳͻ͹͵ GeV fm


GS-2015-X (Physics)
Section A
To be attempted by ALL candidates.

1. Which of the following vectors is parallel to the surface ‫ ݔ‬ଶ ‫ ݕ‬൅ ʹ‫ ݖݔ‬ൌ Ͷ at the
pointሺʹǡ െʹǡ͵ሻ?

(a) ൅͸ଓƸ െ ʹଔƸ െ ͷ݇෠  (b) ൅͸ଓƸ ൅ ʹଔƸ ൅ ͷ݇෠

(c) െ͸ଓƸ െ ʹଔƸ ൅ ͷ݇෠ (d) ൅͸ଓƸ െ ʹଔƸ ൅ ͷ݇෠

2. A random number generator outputs ൅ͳ or െͳ with equal probability every


time it is run. After it is run 6 times, what is the probability that the sum of
the answers generated is zero? Assume that the individual runs are
independent of each other.

(a) 1/2 (b) 5/6 (c) 5/16 (d) 15/32

3. It is required to construct the quantum theory of a particle of mass ݉ moving


in one dimension ‫ ݔ‬under the influence of a constant force ‫ ܨ‬. The
characteristic length-scale in this problem is
ଵȀଷ
԰ ԰ଶ ԰ ଵȀଷ ݉‫ܨ‬
(a) (b) ቆ ቇ (c) ൬ ଶ ൰ (d)
݉‫ܨ‬ ݉‫ܨ‬ ݉ ‫ܨ‬ ԰ଶ

4. A particle slides on the inside surface of a frictionless


cone. The cone is fixed with its tip on the ground and
its axis vertical, as shown in the figure on the right.
The semi-vertex angle of the cone is ߙ. If the particle
moves in a circle of radius ‫ݎ‬଴ , without slipping
downwards, the angular frequency ߱ of this motion will
be

(a) ݃ (b) ݃ (c) ݃ (d) ݃


ඨ ඨ ඨ ඨ
‫ݎ‬଴ …‘• ߙ ‫ݎ‬଴ •‹ ߙ ‫ݎ‬଴ …‘– ߙ ‫ݎ‬଴ –ƒ ߙ


5. A car starts from rest and accelerates under a force ‫ ܨ‬increasing linearly in
time as ‫ ܨ‬ൌ ܽ‫ ݐ‬where ܽ is a constant. At time ‫ݐ‬ଵ ൐ Ͳ, the force ‫ ܨ‬is suddenly
switched off. At a later time ‫ݐ‬ଶ ൐ ‫ݐ‬ଵ , brakes are applied resulting in a force ‫ܨ‬Ԣ
whose magnitude increases linearly with time, ‫ܨ‬Ԣ ൌ  െܽሺ‫ ݐ‬െ ‫ݐ‬ଶ ሻ where ܽ is the
same constant as before. Which of the following graphs would best represent
the change in the position of the car ‫ݔ‬ሺ‫ݐ‬ሻ with time?

(a) (b)

(c) (d)

6. In the Earth’s atmosphere, a localised low-pressure region (shaded in


diagrams) develops somewhere in the southern hemisphere. Which one of the
following diagrams represents the correct air flow pattern as observed from a
satellite?

(a) (b)

(c) (d)


7. The focal length in air of a thin lens made of glass of refractive index ͳǤͷ is ℓ.
When immersed in water (refractive index = 4/3), its focal length becomes

(a) Ͷℓ (b) ℓȀͶ (c) ͵ℓȀͶ (d) ͶℓȀ͵

8. A light beam of intensity ‫ܫ‬଴ passes at normal incidence through a flat plate of
plastic kept in air. If reflection at the interface reduces the intensity by 20%
and absorption on passing through the plate reduces the intensity by 2%, the
intensity of the emergent beam will be about

(a) ͲǤ͸Ͳ‫ܫ‬଴ (b) ͲǤ͸͵‫ܫ‬଴ (c) ͲǤ͸ͷ‫ܫ‬଴ (d) ͲǤ͹ͺ‫ܫ‬଴

9. A light beam is propagating through a medium with index of refraction ͳǤͷ. If


the medium is moving at constant velocity ͲǤ͹ܿ in the same direction as the
beam, what is the velocity of light in the medium as measured by an observer
in the laboratory? (ܿ ൌ velocity of light in vacuum)

(a) ͲǤͻ͵ܿ (b) ͲǤͻͺܿ (c) ͲǤͻ͸ܿ (d) ͲǤͻܿ

10. Two blackbodies radiate energy at temperatures ܶଵ and ܶଶ ( ܶଵ  ൐  ܶଶ ). The


energy emitted per unit time per unit solid angle per unit surface area of a
blackbody in the frequency range ߥ to ߥ ൅ ݀ߥ is given by ‫ܤ‬ሺߥሻ݀ߥ. Which one of
the following graphs has the correct form?

(a) (b)

(c) (d)


11. In a cold country, in winter, a lake was freezing slowly. It was observed that
it took 2 hours to form a layer of ice 2 cm thick on the water surface.
Assuming a constant thermal conductivity throughout the layer, the
thickness of ice would get doubled after

(a) 2 more hours. (b) 4 more hours.

(c) 6 more hours. (d) 8 more hours.

12. Which of the following graphs qualitatively describes the pressure ܲ of a gas
of non-interacting fermions in thermal equilibrium at a constant volume as a
function of temperature ?

(a) ܲ (b) ܲ

ܶ ܶ

(c) ܲ (d)
ܲ

ܶ ܶ

13. The electrostatic potential ߮ሺ‫ݎ‬ሻ of a distribution of point charges has the
form ߮ሺ‫ݎ‬ሻ ‫ି ݎ  ן‬ଷ at a distance ‫ ݎ‬from the origin ሺͲǡͲǡͲሻ, where ‫ƒ ب ݎ‬. Which
of the following distributions can give rise to this potential ?

(a) (b)

(c) (d)


14. Two semi-infinite solenoids placed next to each other are separated by a small
gap of width W as shown in the figure.
ܹ

‫ܫ‬଴  ‫ܫ‬଴ 
The current ‫ܫ‬଴  in the solenoids flows in the direction as shown. If the
solenoids have a circular cross-section of radius ܴ and are filled with a
magnetic material of permeability ߤሺߤ ൐ ߤ଴ ሻ , then the magnetic energy
densities ‫ݑ‬௜ inside the solenoid and ‫ݑ‬௚ in the gap are best related by

(a) ‫ݑ‬௚ ൐ ‫ݑ‬௜ (b) ‫ݑ‬௚ ൏ ‫ݑ‬௜ (c) ‫ݑ‬௚ ൌ ܿ‫ݑ‬௜ (d) ‫ݑ‬௚ ൐ ܿ‫ݑ‬௜

15. A light source has a small filament at the centre of a spherical glass bulb of
radius 5 cm and negligible thickness. If this source emits 100 Watts of power
in the form of spherical electromagnetic waves, the r.m.s. electric field ‫ܧ‬at
the surface of the bulb (in units of Volt/m) will be approximately

(a) ͳͷͶ͹ (b) ͳͲͻͶ (c) ͳͲͻǤͶ (d) ͳͷǤͶ͹

16. A particle is moving in one dimension under a potential ܸሺ‫ݔ‬ሻ such that, for
large positive values of x, ܸሺ‫ݔ‬ሻ ൎ ݇‫ ݔ‬ఉ , where ݇ ൐ Ͳ and ߚ ൒ ͳ . If the
wavefunction in this region has the form ߰ሺ‫ݔ‬ሻ̱ ‡š’ሺെ‫ ݔ‬ఒ ሻ , which of the
following is true ?

ߚ
(a) ߣ ൌ  ൅ ͳ (b) ߣ ൌ ߚ
ʹ

ߣ ൌ ʹߚ െ ʹ ߚଶ
(c) (d) ߣ ൌ
ʹ

17. The ground state energy of a particle of mass ݉ in a three-dimensional


cubical box of side κ is not zero but ͵݄ଶ Ȁͺ݉κଶ . This is because

(a) the ground state has no nodes in the interior of the box.

(b) this is the most convenient choice of the zero level of potential energy.

(c) position and momentum cannot be exactly determined simultaneously.

(d) the potential at the boundaries is not really infinite, but just very large.


18. A one-dimensional box contains a particle whose ground state energy is ߳. It
is observed that a small disturbance causes the particle to emit a photon of
energy ݄ߥ ൌ ͺ߳, after which it is stable. Just before emission, a possible state
of the particle in terms of the energy eigenstates ሼ߰ଵ ǡ ߰ଶ ǡ ǥ ሽ would be
߰ଵ െ ߰ଶ ߰ଶ ൅ ʹ߰ଷ
(a) (b)
ξʹ ξͷ
െͶ߰ସ ൅ ͷ߰ହ
ξʹ߰ଵ െ ͵߰ଶ ൅ ͷ߰ହ
(c) (d)
ξͶͳ ͸

19. A sample of ordinary hydrogen ( ଵଵ ) gas in a discharge tube was seen to emit
the usual Balmer spectrum. On careful examination, however, it was found
that the ‫ܪ‬ఈ line in the spectrum was split into two fine lines, one an intense
line at 656.28 nm, and the other a faint line at 656.04 nm. From this, one can
conclude that the gas sample had a small impurity of
ଶ ଷ ସ
(a) ଵ (b) ଵ (c) ଶ ‡ (d) ଶ 

20. An electron makes a transition from the valence band to the conduction
band in an indirect band gap semiconductor. Which of the following is NOT
true?

(a) The energy of the electron increases.

(b) A phonon is involved in the process.

(c) A photon is absorbed in the process.

(d) There is no momentum change in the electron.

21. Which of the following radioactive decay chains is it possible to observe?


ଶ଴଺ ଶ଴ଶ
(a) ଼ଶ„ ՜ ଼଴ ‰ ՜  ଶ଴ଶ
଻ଽ—

ଶଵ଴ ଶଵ଴
(b) ଼ଷ‹ ՜ ଼ସ‘ ՜  ଶ଴଺
଼ଶ„

ଶଵସ ଶଵ଴
(c) ଼଼ƒ ՜ ଼଺ ՜  ଶ଴଻
଼ଶ„

ଶ଴଺ ଶ଴ଶ
(d) ଼ଶ„ ՜ ଼଴ ‰ ՜  ଶ଴ଶ
଻ଽ—

22. Which of the following is the best technique for measuring the effective mass
of an electron in a semiconductor?

(a) Resistivity measurements (b) X-ray diffraction experiment

(c) Cyclotron resonance (d) Millikan’s oil drop experiment


23. Two LCR circuits (A) and (B) are shown below where ‫ܥ‬௖ ‫ܥ ا‬. At time ‫ ݐ‬ൌ Ͳ, a
charge ܳ is put on the capacitor C.

Which of the following statements is correct?

(a) The charge Q will decay faster in (A)

(b) The charge Q will decay faster in (B)

(c) The charge Q will decay at the same rate in (A) and (B)

(d) The relative decay rates cannot be predicted without knowing the exact
values of L, C, R and ‫ܥ‬௖

24. In the circuit shown below, the op-amp is powered by a bipolar supply of
േͳͲ.

Which one of the following graphs represents ܸ୭୳୲ correctly?

(a) (b)

(c) (d)


25. All resistors in the circuit on the right have a
tolerance of േͷ%.

Assuming a diode drop of 0.7 V, which of the


following is the lowest possible value of the collector
voltage?

(a) 3.1 V (b) 4.1 V (c) 4.7 V (d) 5.2 V

DO NOT WRITE IN THIS SPACE


GS-2015-X (Physics)
Section B
To be attempted only by candidates for Integrated Ph.D. programme.
(Candidates for Ph.D. programme will get NO credit for attempting this section.)

26. Consider the differential equation


݀ଶ ‫ݕ‬ ݀‫ݕ‬

ൌ െͶ ൬‫ ݕ‬൅ ൰
݀‫ݔ‬ ݀‫ݔ‬
with the boundary condition that ‫ݕ‬ሺ‫ݔ‬ሻ ൌ Ͳ at ‫ ݔ‬ൌ ͳȀͷ . When plotted as a
function of ‫ݔ‬, for ‫ ݔ‬൒ Ͳ, we can say with certainty that the value of ‫ݕ‬

(a) oscillates from positive to negative with amplitude decreasing to zero

(b) has an extremum in the range Ͳ ൏ ‫ ݔ‬൏ ͳ

(c) first increases, then decreases to zero

(d) first decreases, then increases to zero

27. In a transmission diffraction grating, there are ͳͲସ lines/mm. Which of the
following ranges of wavelength (in nm) will produce at least one principal
maximum?

(a) ͳ െ ʹͲͲ (b) ʹͲͳ െ ͷͲͲ (c) ͷͲͳ െ ͳͲͲͲ (d) ͳͲͲͳ െ ͷͲͲͲ

28. Two cylinders A and B of the same length L


and outer radius R were placed at the same
height ݄ on an inclined plane at an angle ߮
with the horizontal (see figure). Starting from
‫ܮ‬
rest, each cylinder was allowed to roll down
the plane without slipping. It was found that
A reached the end of the inclined plane ݄
earlier than B. Which of the following
possibilities could be true? ߮

(a) A is hollow and made of copper; B is hollow and made of copper;


B is heavier than A.

(b) A is solid and made of copper; B is solid and made of aluminium.

(c) A is hollow and made of aluminium; B is solid and made of aluminium.

(d) A is solid and made of copper; B is hollow and made of copper.


29. A thin uniform rod of length ʹ݈ and mass ‫ ܯ‬is
pivoted at one end P on a horizontal plane (see
figure). A ball of mass ݉ ‫ ܯ ا‬and speed ‫ݒ‬଴
strikes the free end of the rod perpendicularly
and bounces back with velocity ‫ݒ‬௙ along the
original line of motion as shown in the fig. If the
collision is perfectly elastic the magnitude of‫ݒ‬௙ is

(a) ‫ ܯ‬െ Ͷ݉ (b) ‫ ܯ‬൅ Ͷ݉ (c) ‫ ܯ‬െ ͵݉ (d) ‫ ܯ‬൅ ͵݉
‫ݒ‬  ‫ݒ‬  ‫ݒ‬ ‫ݒ‬
‫ ܯ‬൅ Ͷ݉ ଴ ‫ ܯ‬െ Ͷ݉ ଴ ‫ ܯ‬൅ ͵݉ ଴ ‫ ܯ‬െ ͵݉ ଴

30. A collimated beam of pions originate from an accelerator and propagates in


vacuum along a long straight beam pipe. The intensity of this beam was
measured in the laboratory after a distance of ͹ͷ m and found to have
dropped to one-fourth of its intensity at the point of origin. If the proper half-
life of a pion is ͳǤ͹͹ ൈ ͳͲି଼ s, the speed of the pions in the beam, as measured
in the laboratory, must be

(a) ͲǤͻͻܿ (b) ͲǤͻͺܿ (c) ͲǤͻ͹ܿ (d) ͲǤͻ͸ܿ

31. The equation of state of a gas is given by


ܴܶ ܾ
ܸൌ െ
ܲ ܶ
where ܴ is the gas constant and ܾ is another constant parameter. The
specific heat at constant pressure ‫ܥ‬௉ and the specific heat at constant
volume ‫ܥ‬௏ for this gas is related by ‫ܥ‬௉ െ ‫ܥ‬௏ ൌ

ܴܶ ଶ
(a) ܴ (b) ܴ ቆͳ ൅ ቇ
ܾܲ

ܾܲ ଶ ܾܲ ଶ
(c) ܴ ൬ͳ ൅ ൰ (d) ܴ ൬ͳ െ ൰
ܴܶ ଶ ܴܶ ଶ

32. An ideal diatomic gas is initially at a temperature ܶ ൌ ͲԨ. Then it expands


reversibly and adiabatically to 5 times its volume. Its final temperature will be
approximately

(a) െͳͺͲԨ (b) െͳͷͲԨ (c) െͳ͵ͲԨ (d) ͲԨ


33. Consider an infinitely long cylinder of radius ܴ , placed along the ‫ ݖ‬-axis,
which carries a static charge density ߩሺ‫ݎ‬ሻ  ൌ ݇‫ݎ‬, where ‫ ݎ‬is the perpendicular
distance from the axis of the cylinder and ݇ is a constant. The electrostatic
potential ߶ሺ‫ݎ‬ሻ inside the cylinder is proportional to

ʹ ‫ݎ‬ଷ ‫ݎ‬
(a) െ ቆ ଷ ൅ ͳቇ (b) െʹ Ž ቀ ቁ
͵ ܴ ܴ

ʹ ‫ݎ‬ଷ ܴ
(c) െ ቆ ଷ െ ͳቇ (d) െʹ Ž ൬ ൰
͵ ܴ ‫ݎ‬

34. A solid spherical conductor encloses 3 cavities, a


cross-section of which are as shown in the figure. ൅‫ݍ‬
A net charge ൅‫ ݍ‬resides on the outer surface of B
the conductor. Cavities A and C contain point C
A െ‫ݍ‬
charges ൅‫ ݍ‬and Ȃ ‫ݍ‬, respectively. ൅‫ݍ‬

The net charges on the surfaces of these


cavities are

(a) ‫ ܣ‬ൌ  െ‫ݍ‬ǡ ‫ ܤ‬ൌ ‫ݍ‬ǡ ‫ ܥ‬ൌ Ͳ (b) ‫ ܣ‬ൌ  െ‫ݍ‬ǡ ‫ ܤ‬ൌ Ͳǡ ‫ ܥ‬ൌ  െ‫ݍ‬

(c) ‫ ܣ‬ൌ  ൅‫ݍ‬ǡ ‫ ܤ‬ൌ Ͳǡ ‫ ܥ‬ൌ  െ‫ݍ‬ (d) ‫ ܣ‬ൌ  െ‫ݍ‬ǡ ‫ ܤ‬ൌ Ͳǡ ‫ ܥ‬ൌ  ൅‫ݍ‬

35. 1000 neutral spinless particles are confined in a one-dimensional box of


length 100 nm. At a given instant of time, if 100 of these particle have
energy Ͷ߳଴ and the remaining 900 have energy ʹʹͷ߳଴ , then the number of
particles in the left half of the box will be approximately

(a) ͸ʹͷ (b) ͷͲͲ

(c) ͶͶͳ (d) ͳͲͲ

36. A one-dimensional quantum harmonic oscillator is in its ground state


݉߱ ଵȀସ ି௠ఠ௫ మ Ȁଶ԰
߰଴ ሺ‫ݔ‬ሻ ൌ ቀ ቁ ݁
ߨ԰
Two experiments, [A] and [B], are performed on the system. In [A], the
frequency ߱of the oscillator is suddenly doubled, while in [B] the frequency
߱ is suddenly halved. If ‫݌‬஺ and ‫݌‬஻ denote the probability in each case that
the system is found in its new ground state immediately after the frequency
change, which of the following is true?

(a) ‫݌‬஺  ൌ  ξʹ‫݌‬஻ (b) ‫݌‬஺  ൌ ʹ‫݌‬஻

(c) ʹ‫݌‬஺  ൌ  ‫݌‬஻ (d) ‫݌‬஺  ൌ  ‫݌‬஻


37. In the basic band structure theory of crystalline solids, which of the
following leads to energy gaps in the allowed electronic energy values?

(a) Electron spin (b) Bragg reflection

(c) Electron-electron interaction (d) Electron-phonon interaction

38. The material inside a box is either a metal or a semiconductor. If ܴሺൌ ͳπሻ is
the resistance of the material, which of the following experiments CANNOT
distinguish whether it is a metal or a semiconductor?

(a) Measurement of ܴ using power supplies of different frequencies.

(b) Measurement of absorption spectrum in the energy range ͲǤͳ െ ʹ eV.

(c) Measurement of ܴ at different temperatures.

(d) Measurement of ܴ in the presence of different magnetic fields.

39. To measure the voltage in the range Ͳ െ ͷ V with a precision of ͷ mV, the
minimum number of bits required in a digital voltmeter is

(a) ͻ (b) ͳͲ (c) ͳͳ (d) ͳʹ

40. A building has three overhead water tanks, each fitted with a sensor (S1, S2,
S3) which goes to 0 when the water level in the tank falls below a set value
and remains 1 otherwise. A common pump is used to raise water from an
underground storage tank to these overhead tanks. Of the following circuits,
which one will turn on (P = 1) the pump only when at least two of the tanks
have water level below the set value?

(a) (b)
S1 S1
S2 S2

S3 P S3 P

(c) S1 (d)
S2 S1
S3 S2

S1 P S3 P
S2
S3


GS-2015-X (Physics)
Section C
To be attempted only by candidates for Ph.D. programme.
(Candidates for Integrated Ph.D. programme will get NO credit for attempting this section.)

41. The integral


ଶగ
݀ߠ

଴ ͳ െ ʹܽ …‘• ߠ ൅ ܽଶ
where Ͳ ൏ ܽ ൏ ͳ, evaluates to
ʹߨ ʹߨ Ͷߨ
(a) ʹߨ (b) (c) (d)
ͳ ൅ ܽଶ ͳ െ ܽଶ ͳ െ ܽଶ

42. The generating function for a set of polynomials in ‫ ݔ‬is given by


݂ሺ‫ݔ‬ǡ ‫ݐ‬ሻ ൌ ሺͳ െ ʹ‫ ݐݔ‬൅ ‫ ݐ‬ଶ ሻିଵ
The third polynomial (order ‫ ݔ‬ଶ ) in this set is

(a) ʹ‫ ݔ‬ଶ ൅ ͳ (b) ʹ‫ ݔ‬ଶ െ ‫ݔ‬

(c) Ͷ‫ ݔ‬ଶ ൅ ͳ (d) Ͷ‫ ݔ‬ଶ െ ͳ

43. A particle moves under the influence of a central potential in an orbit ‫ ݎ‬ൌ ݇ߠ ସ ,
where ݇ is a constant and ‫ ݎ‬is the distance from the origin. It follows that the
angle ߠ varies with time ‫ ݐ‬as

(a) ߠ ‫ݐ  ן‬ଵȀଽ (b) ߠ ‫ݐ  ן‬ଵȀ଼ (c) ߠ ‫ݐ  ן‬ଵȀ଻ (d) ߠ ‫ݐ  ן‬ଵȀ଺

44. In a system with two degrees of freedom, if ሺ‫݌‬ǡ ‫ݍ‬ሻ are the canonical
coordinates, then which of the following transformations to ሺܲǡ ܳሻ is
canonical?
ͳ ʹ‫ݍ‬ ͳ ‫݌‬
(a) ܲ ൌ  ሺ‫݌‬ଶ ൅  ‫ ݍ‬ଶ ሻǡ ܳ ൌ –ƒିଵ (b) ܲ ൌ  ሺ‫݌‬ଶ ൅  ‫ ݍ‬ଶ ሻǡ ܳ ൌ …‘– ିଵ
ʹ ‫݌‬ ʹ ‫ݍ‬
ͳ ‫ݍ‬ ͳ ‫݌‬
(c) ܲ ൌ  ሺ‫݌‬ଶ ൅  ‫ ݍ‬ଶ ሻǡ ܳ ൌ •‹ିଵ (d) ܲ ൌ  ሺ‫݌‬ଶ ൅  ‫ ݍ‬ଶ ሻǡ ܳ ൌ …‘• ିଵ 
ʹ ʹ‫݌‬ ʹ ‫ݍ‬


45. In a monatomic gas, the first excited state is only ͳǤͷ eV above the ground
state, while the other excited states are much higher up. The ground state is
doubly-degenerate, while the first excited state has a four-fold degeneracy. If
now, the gas is heated to a temperature of ͹ͲͲͲ K, the fraction of atoms in the
excited state will be approximately

(a) ͲǤͳͶ (b) ͲǤͲ͹ (c) ͲǤͶʹ (d) ͲǤ͵

46. Measurement of the magnitudes of the electric field (‫ )ܧ‬and the magnetic
field (‫ )ܤ‬in a plane-polarised electromagnetic wave in vacuum leads to the
following results
߲‫ܧ‬ ߲‫ܤ‬ ߲‫ܤ‬ ͳ ߲‫ܧ‬
ൌ െ  ൌെ ଶ
߲‫ݕ‬ ߲‫ݐ‬ ߲‫ݕ‬ ܿ ߲‫ݐ‬
at all points where the measurement is made. In this case the electric vector
ሬԦ and the wave vector ݇ሬԦ (with magnitude ݇ሻcan be
‫ܧ‬ሬԦ , the magnetic vector ‫ܤ‬
written in terms of the unit vectors ሺ‫ݔ‬ොǡ ‫ݕ‬ොǡ ‫ݖ‬Ƹ ሻ along the Cartesian axes as

ሬԦ ൌ ‫ݔܧ‬ොǡ‫ܤ‬
(a) ‫ܧ‬ ሬԦ ൌ ݇‫ݖ‬Ƹ
ሬԦ ൌ ‫ݕܤ‬ොǡ ݇ ሬԦ ൌ ‫ݔܧ‬ොǡ‫ܤ‬
(b) ‫ܧ‬ ሬԦ ൌ െ݇‫ݕ‬ො
ሬԦ ൌ ‫ݖܤ‬Ƹ ǡ݇

(c) ‫ܧ‬ሬԦ ൌ ‫ݔܧ‬ොǡ‫ܤ‬ ሬԦ ൌ ݇‫ݕ‬ො


ሬԦ ൌ െ‫ݖܤ‬Ƹ ǡ݇ ሬԦ ൌ െ‫ݕܧ‬ොǡ ‫ܤ‬
(d) ‫ܧ‬ ሬԦ ൌ െ݇‫ݔ‬ො
ሬԦ ൌ െ‫ݖܤ‬Ƹ ǡ ݇

47. A two-state quantum system has two observables ‫ ܣ‬and ‫ܤ‬. It is known that
the observable ‫ ܣ‬has eigenstates ȁߙଵ ‫ ۄ‬and ȁߙଶ ‫ ۄ‬with eigenvalues ܽଵ and ܽଶ
respectively, while ‫ ܤ‬has eigenstates ȁߚଵ ‫ ۄ‬and ȁߚଶ ‫ۄ‬with eigenvalues ܾଵ and ܾଶ
respectively, and that these eigenstates are related by
͵ Ͷ Ͷ ͵
ȁߚଵ ‫ ۄ‬ൌ ȁߙଵ ‫ ۄ‬െ ȁߙଶ ‫ۄ‬ȁߚଶ ‫ ۄ‬ൌ ȁߙଵ ‫ ۄ‬൅ ȁߙଶ ‫ۄ‬
ͷ ͷ ͷ ͷ
Suppose a measurement is made of the observable ‫ ܣ‬and a value ܽଵ is
obtained. If the observable ‫ ܤ‬is now measured, the probability of obtaining the
value ܾଵ will be

(a) ͲǤͺͲ (b) ͲǤ͸Ͷ (c) ͲǤ͸Ͳ (d) ͲǤ͵͸

48. An rigid rotator has the wave function

߰ሺߠǡ ߮ሻ ൌ ܰൣʹܻ݅ଵǡ଴ ሺߠǡ ߮ሻ ൅  ሺʹ ൅ ݅ሻܻଶǡିଵ ሺߠǡ ߮ሻ ൅ ͵ܻ݅ଵǡଵ ሺߠǡ ߮ሻ൧


where ܻ௟ǡ௠ ሺߠǡ ߮ሻ are the spherical harmonics, and N is a normalization
constant. If ‫ܮ‬ሬԦ is the orbital angular momentum operator, and ‫ܮ‬േ ൌ ‫ܮ‬௫ േ
݅‫ܮ‬௬ the expectation value of ‫ܮ‬ା ‫ ିܮ‬is

(a) ʹͳ԰ଶ Ȁͻ (b) ʹ͵԰ଶ Ȁͻ (c) ʹͷ԰ଶ Ȁͻ (d) Ͳ


49. In the ground state electronic configuration of nitrogen ( ଵସ଻) the ‫ܮ‬, ܵ and ‫ܬ‬
quantum numbers are

(a) ‫ ܮ‬ൌ ͳ, ܵ ൌ ͳȀʹ, ‫ ܬ‬ൌ ͳȀʹ (b) ‫ ܮ‬ൌ ͳ, ܵ ൌ ͳȀʹ, ‫ ܬ‬ൌ ͵Ȁʹ

(c) ‫ ܮ‬ൌ Ͳ, ܵ ൌ ͳȀʹ, ‫ ܬ‬ൌ ͳȀʹ (d) ‫ ܮ‬ൌ Ͳ, ܵ ൌ ͵Ȁʹ, ‫ ܬ‬ൌ ͵Ȁʹ

50. Solar radiation tends to push any particle inside solar system away from the
Sun. Consider a spherical dust particle of specific gravity 6.0 and no angular
momentum about the Sun. What should be its minimum radius so that it
does not escape from the solar system? Take the solar luminosity to be
͵Ǥͺ ൈ ͳͲଶ଺ W.

(a) ͳͲି଺ ߤm (b) ͲǤͲͳ ߤm (c) ͲǤͳ ߤm (d) ͳͲ ߤm

51. Bosonic excitations of ferromagnets have a dispersion relation ߳ ൌ ߛ݇ ଶ, where


߳ is the energy and ݇ is the wavevector of the excitation. Assuming a system of
such non-interacting bosonic excitations, at low temperature ܶ, the specific
heat ‫ܥ‬௏ of a three-dimensional ferromagnet will be proportional to

(a) ܶ (b) ܶ ଷȀଶ (c) ܶ ହȀଶ (d) ܶଷ

52. In the semi-empirical mass formula, the volume (ܸ), surface (ܵ), coulomb (‫)ܥ‬,
and pairing (ܲ) contributions to the binding energy of a nucleus ஺௓ vary with
mass number ‫ ܣ‬as

(a) ܸ ‫ܣ ן‬ǡ ܵ ‫ܣ ן‬ଶȀଷ ǡ‫ିܣ ן ܥ‬ଵȀଷ ǡ ܲ ‫ିܣ  ן‬ଷȀସ

(b) ܸ ‫ܣ ן‬ǡ ܵ ‫ܣ ן‬ଵȀଷ ǡ‫ିܣ ן ܥ‬ଵȀଷ ǡ ܲ ‫ିܣ  ן‬ଷȀସ

(c) ܸ ‫ܣ ן‬ǡ ܵ ‫ିܣ ן‬ଶȀଷ ǡ ‫ܣ ן ܥ‬ଵȀଷ ǡܲ ‫ିܣ  ן‬ଷȀସ

(d) ܸ ‫ܣ ן‬ଶ ǡܵ ‫ܣ ן‬ଶȀଷ ǡ‫ିܣ ן ܥ‬ଵȀଷ ǡ ܲ ‫ିܣ  ן‬ଷȀସ

53. In an experiment, ଵଽ଻଻ଽ— nuclei were bombarded with neutrons leading to


formation of ଻ଽ—, which is unstable. The half-life of ଵଽ଼
ଵଽ଼
଻ଽ— was measured to
be ʹǤʹͷ days and it was found later that this measured half-life was an
underestimate by 10%. The corresponding percentage error in the estimated
population of ଵଽ଼
଻ଽ— after 9 days is

(a) 10% (b) 25% (c) 2.5% (d) 15%


54. You are given the following circuit and two instruments: a voltmeter and an
ammeter both with 0.001 % accuracy in their readings.

Which of the following methods will result in the most accurate reading for
the current without interrupting the current in the circuit?

(a) Use voltmeter to measure voltage across points B and C

(b) Use the ammeter to measure current at point B

(c) Use voltmeter to measure voltage across points A and B

(d) Use voltmeter to measure voltage across points A and C

55. Consider the following reactions involving elementary particles:


(A) ߨ ି ൅ ‫ ݌‬՜  ‫ ି ܭ‬൅ ߑ ା
(B)‫ ି ܭ‬൅ ‫ ݌‬՜  ‫ ି ܭ‬൅ ߩା

Which of the following statements is true for strong interactions?

(a) (A) and (B) are both forbidden

(b) (B) is allowed but (A) is forbidden

(c) (A) is allowed but (B) is forbidden

(d) (A) and (B) are both allowed

DO NOT WRITE IN THIS SPACE


Correct answers are ticked in green.
PHY - X
GS-2016 (Physics)
TATA INSTITUTE OF FUNDAMENTAL RESEARCH 
WrittenTestinPHYSICSͲDecember13,2015
Duration:Threehours(3hours)
Name:_______________________________________________Ref.Code:____________

Pleasereadallinstructionscarefullybeforeyouattemptthequestions.

1. Please fill in details about name, reference code etc. on the question paper and answer sheet. The Answer
Sheet is machine-readable. Use only black/blue ball point pen to fill in the answer sheet.
2. This test consists of three parts, Section A, Section B and Section C. You must answer questions
according to the program you are applying for :
Candidates applying for Must Answer Should Not Attempt
Integrated Ph.D. Section A and Section B Section C
Ph.D. Section A and Section C Section B
Section A contains 25 questions, Section B and Section C contain 15 questions each. Note that the test
contains multiple-choice questions as well as numerical-type questions (36-40 in section B and 51-55 in
section C) where you have to fill in numbers in the answer-sheet.
4. Indicate your ANSWER ON THE ANSWER SHEET by filling in the appropriate circle or circles
completely for each question.
For multiple-choice questions, only ONE of the options given at the end of each question is correct. Do
not mark more than one circle for any multiple choice question : this will be treated as a wrong answer.
For number-type questions, the answers should be indicated on the answer sheet by filling in circles for
appropriate numbers. All three circles for each answer should be filled to get the credit. Detailed
instructions are given inside the question paper.
5. The marking shall be as follows:
Multiple-choice Numerical
If the answer is correct : +3 +5
If the answer is incorrect : -1 0
If the answer is not attempted : 0 0
If more than one circle is marked (only for multiple-choice): 0 Not applicable

Note that there is negative marking for multiple-choice questions, but not for the numerical-type
questions.
6. We advise you to first mark the correct answers on the QUESTION PAPER and then to TRANSFER these
to the ANSWER SHEET only when you are sure of your choice.
7. Rough work may be done on blank pages of the question paper. If needed, you may ask for extra rough
sheets from an Invigilator.
8. Use of calculators is permitted. Calculator which plots graphs is NOT allowed. Multiple-use devices such
as cell phones, smartphones etc., CANNOT be used for this purpose.
9. Do NOT ask for clarifications from the invigilators regarding the questions. They have been instructed not
to respond to any such inquiries from candidates. In case a correction/clarification is deemed necessary, the
invigilator(s) will announce it publicly.
10. List of useful physical constants is given on the next page. Make sure to use only these values while
answering the questions.
1
USEFUL CONSTANTS
Symbol Name/Definition Value
ܿ speed of light in vacuum ͵ǤͲͲ ൈ ͳͲ଼ m s-1
԰ reduced Planck constant (ൌ ݄Ȁʹߨ) ͳǤͲͶ ൈ ͳͲିଷସ J s
‫ܩ‬ே gravitational constant ͸Ǥ͸͹ ൈ ͳͲିଵଵ m3 kg-1 s-2
‫ٖܯ‬ solar mass ͳǤͻͻ ൈ ͳͲଷ଴ kg
ߝ଴ permittivity of free space ͺǤͺͷ ൈ ͳͲିଵଶ F m-1
ߤ଴ permeability of free space ͳǤʹ͸ ൈ ͳͲି଺ N A-2
݁ electron charge (magnitude) ͳǤ͸Ͳ ൈ ͳͲିଵଽ C
݉௘ electron mass ͻǤͳͲ ൈ ͳͲିଷଵ kg
ൌ ͲǤͷͲͲ MeV/c 2

ܽ଴ Bohr radius 0.510 Å


ionisation potential of H atom 13.6 eV
ܰ஺ Avogadro number ͸ǤͲʹ ൈ ͳͲଶଷ mol-1
݇஻ Boltzmann constant ͳǤ͵ͺ ൈ ͳͲିଶଷ J K-1
ൌ ͺǤ͸ʹ ൈ ͳͲିହ eV K-1
ܴ ൌ ܰ஺ ݇஻ gas constant ͺǤ͵ͳ J mol-1 K-1
ߛ ൌ ‫ܥ‬௣ Ȁ‫ܥ‬௏ ratio of specific heats: monatomic gas 1.67
diatomic gas 1.40
ߪ Stefan-Boltzmann constant ͷǤ͸͹ ൈ ͳͲି଼ W m-2 K-4
ߙ fine structure constant (ൌ ݁ ଶ ȀͶߨߝ଴ ԰ܿ) 1/137
݃ acceleration due to gravity 9.80 m s-2
ܴா radius of the Earth ͸ǤͶͲ ൈ ͳͲଷ Km
ܴௌ radius of the Sun ͹ǤͲͲ ൈ ͳͲହ Km
݉௣ proton mass (ൎ ʹͲͲͲ݉௘ ) ͳǤ͹Ͳ ൈ ͳͲିଶ଻ kg
ൌ ͻ͵ͺǤͲ MeV/c 2

ିଶ଻ kg
݉௡ neutron mass (ൎ ʹͲͲͲ݉௘ ) ͳǤ͹Ͳ ൈ ͳͲ
ൌ ͻͶͲǤͲ MeV/c 2

UNIT CONVERSIONS
Symbol Name/Definition Value
1 A.U. mean distance of Earth from Sun ͳǤͷͲ ൈ ͳͲଽ km
1 a.m.u. atomic mass unit ͳǤ͸Ͳ ൈ ͳͲିଶ଻ kg
ൌ ͻ͵ʹǤͲ MeV/c 2

1 eV electron Volt ͳǤ͸Ͳ ൈ ͳͲିଵଽ J


1T Tesla ͳͲସ gauss
1 bar mean atmospheric pressure at 00 C ͳǤͲͳ ൈ ͳͲହ Pa (= N m-2)
1Å Ångstrom unit 10-8 cm
԰ܿ conversion constant ͵Ǥͳ͸ ൈ ͳͲିଶ଺ J m-1
ൌ ͲǤͳͻ͹ GeV fm

2
GS-2016-X (Physics)
Section A
To be attempted by ALL candidates.

1. If ‫ ݔ‬is a continuous variable which is uniformly distributed over the real line
from ‫ ݔ‬ൌ Ͳ to ‫ ݔ‬՜ λ according to the distribution
݂ሺ‫ݔ‬ሻ ൌ ‡š’ሺെͶ‫ݔ‬ሻ
then the expectation value of
…‘• Ͷ‫ݔ‬
is

(a) zero (b) ͳȀʹ (c) ͳȀͶ (d) ͳΤͳ͸

2. If the eigenvalues of a symmetric ͵ ൈ ͵ matrix A are Ͳǡ ͳǡ ͵ and the


corresponding eigenvectors can be written as
ͳ ͳ ͳ
൭ͳ൱ǡ ൭ Ͳ ൱ǡ൭െʹ൱
ͳ െͳ ͳ
respectively, then the matrix ‫ۯ‬ସ is
(a) Ͷͳ െͺͳ ͶͲ (b) െͺʹ െͺͳ ͹ͻ
൭െͺͳ Ͳ െͺͳ൱ ൭െͺͳ ͺͳ െͺͳ൱
ͶͲ െͺͳ Ͷͳ ͹ͻ െͺͳ ͺ͵
(c) ͳͶ െʹ͹ ͳ͵ (d) ͳͶ െͳ͵ ʹ͹
൭െʹ͹ ͷͶ െʹ͹൱ ൭െͳ͵ ͷͶ െͳ͵൱
ͳ͵ െʹ͹ ͳͶ ʹ͹ െͳ͵ ͳͶ

3. In a triangular lattice a particle moves from a lattice


point to any of its 6 neighbouring points with equal
probability, as shown in the figure on the right.

The probability that the particle is back at its


starting point after 3 moves is

(a) ͷȀͳͺ (b) ͳȀ͸ (c) ͳȀͳͺ (d) ͳȀ͵͸

4. A ball is dropped vertically from a height ‫ ܪ‬on to a plane surface and


permitted to bounce repeatedly along a vertical line. After every bounce, its
kinetic energy becomes a quarter of its kinetic energy before the bounce. The
ball will come to rest after time

(a) infinity (b) ሺʹ‫ܪ‬Τ݃ሻଵȀଶ (c) ʹሺʹ‫ܪ‬Τ݃ሻଵȀଶ (d) ͵ሺʹ‫ܪ‬Τ݃ሻଵȀଶ

3
5. In a moving car, the wheels will skid if the brakes are applied too suddenly.
This is because

(a) the inertia of the car will carry it forward.


(b) the momentum of the car must be conserved.
(c) the impulsive retarding force exceeds the limiting force of static friction.
(d) the kinetic friction will suddenly get converted to static friction.

6. An aircraft, which weighs ͳʹͲͲͲ kg when unloaded, is on a relief mission,


carrying ͶͲͲͲ food packets weighing 1 kg each. The plane is gliding
horizontally with its engines off at a uniform speed of ͷͶͲ kmph when the first
food packet is dropped. Assume that the horizontal air drag can be neglected
and the aircraft keeps moving horizontally. If one food packet is dropped every
second, then the distance between the last two packet drops will be

(a) ͳǤͷ Km (b) ʹͲͲ m (c) ͳͷͲ m (d) ͳͲͲ m

7. Imagine that a narrow tunnel is excavated through the


Earth as shown in the diagram on the left and that the
݀
mass excavated to create the tunnel is extremely small
ܴ compared to Earth’s mass ‫ܯ‬.
A person falls into the tunnel at one end. at time ‫ ݐ‬ൌ Ͳ.
Assuming that the tunnel is frictionless, the person will

(a) fall straight through, escaping Earth's gravity at time ʹߨඥܴ ଷΤ‫ܯܩ‬
(b) describe simple harmonic motion with period ʹߨሺ݀Τܴ ሻඥܴଷ Τ‫ܯܩ‬
(c) describe simple harmonic motion with period ʹߨඥሺܴ െ ݀ሻଷ Τ‫ܯܩ‬
(d) describe simple harmonic motion with period ʹߨඥܴ ଷΤ‫ܯܩ‬

8. A grounded conducting sphere of radius ܽ is placed with its centre at the


origin. A point dipole of dipole moment ‫݌‬Ԧ ൌ ‫݇݌‬෠ is placed at a distance ݀ along
the ‫ݔ‬-axis, where ଓƸǡ ݇෠ are the units vector along the ‫ ݔ‬and ‫ݖ‬-axes respectively.
This leads to the formation of an image dipole of strength ‫݌‬Ԧᇱ at a distance ݀ᇱ
from the centre along the ‫ݔ‬-axis. If ݀ᇱ ൌ ܽଶ Ȁ݀, then ‫݌‬Ԧᇱ ൌ

ܽସ ‫݌‬ ܽଷ ‫݌‬ ܽଶ ‫݌‬ ܽ‫݌‬


(a) െ ݇෠ (b) െ ݇෠ (c) െ ݇෠ (d) െ ݇෠
݀ସ ݀ଷ ݀ଶ ݀

4
9. A long, solid dielectric cylinder of radius a is permanently polarized so that
the polarization is everywhere radially outward, with a magnitude proportional

ሬԦ ൌ  ܲ଴ ‫ݎݎ‬Ƹ . The bound
to the distance from the axis of the cylinder, i.e., ܲ

charge density in the cylinder is given by

(a) െܲ଴ (b) ܲ଴ (c) െܲ଴ Ȁʹ (d) ܲ଴ Ȁʹ

10. A circular loop of fine wire of radius ܴcarrying a current ‫ ܫ‬is placed in a
uniform magnetic field ‫ ܤ‬perpendicular to the plane of the loop. If the
breaking tension of the wire is ܶ௕ , the wire will break when the magnetic field
exceeds

(a) ܶ௕ Ȁ‫ܴܫ‬ (b) ܶ௕ Ȁʹߨ‫ܴܫ‬

(c) ߤ଴ ܶ௕ Τʹߨ‫ܴܫ‬ (c) ߤ଴ ܶ௕ ΤͶߨ‫ܴܫ‬

11. A gas of non-interacting particles, each of rest mass ͳ MeV, is at a


temperature ܶ ൌ ʹǤͲ ൈ  ͳͲ଻ K and has an average particle density ݊ ൌ ʹǤ͹ ൈ ͳͲଷସ
cm-3. We can obtain a reasonably correct treatment of this system

(a) only by using special relativity as well as quantum mechanics.

(b) by neglecting quantum mechanics but not special theory of relativity.

(c) by neglecting special relativity but not quantum mechanics.

(d) by neglecting both special relativity and quantum mechanics.

12. A one-dimensional harmonic oscillator of mass ݉ and natural frequency ߱is


in the quantum state

ͳ ݅ ݅
ȁ߰‫ ۄ‬ൌ ȁͲ‫ ۄ‬൅  ȁͳ‫  ۄ‬൅  ȁʹ‫ۄ‬
ξ͵ ξ͵ ξ͵

at time ‫ ݐ‬ൌ Ͳ, where ȁ݊‫ ۄ‬denotes the eigenstate with eigenvalue ቀ݊ ൅ ଶቁ ԰߱.

It follows that the expectation value ‫ ۄݔۃ‬of the position operator ‫ݔ‬ො is

ͳ
(a) ‫ݔ‬ሺͲሻ ൤…‘• ߱‫ ݐ‬൅  •‹ ߱‫ݐ‬൨ (b) ‫ݔ‬ሺͲሻሾ…‘• ߱‫ ݐ‬െ •‹ ߱‫ݐ‬ሿ
ξ͵
ͳ ͳ
(c) ‫ݔ‬ሺͲሻ ൤…‘• ߱‫ ݐ‬െ  •‹ ߱‫ݐ‬൨ (d) ‫ݔ‬ሺͲሻ ൤…‘• ߱‫ ݐ‬൅  •‹ ߱‫ݐ‬൨
ʹ ξʹ

5
13. A particle is confined inside a one-
dimensional potential well ܸሺ‫ݔ‬ሻ , as shown ܸሺ‫ݔ‬ሻ
on the right.

One of the possible probability distributions


ȁ߰ሺ‫ݔ‬ሻȁଶ for an energy eigenstate for this Ͳ ‫ݔ‬
particle is

(a) ȁ߰ሺ‫ݔ‬ሻȁଶ (b) ȁ߰ሺ‫ݔ‬ሻȁଶ

‫ݔ‬ ‫ݔ‬
Ͳ Ͳ

(c) ȁ߰ሺ‫ݔ‬ሻȁଶ (d) ȁ߰ሺ‫ݔ‬ሻȁଶ

Ͳ ‫ݔ‬

‫ݔ‬
Ͳ

14. The energy per oscillator of an isolated system of a large number of identical
non-interacting fermions in a one-dimensional harmonic oscillator potential is
ͷ԰߱ȀͶ , where ߱ is the angular frequency of the harmonic oscillator. The
entropy of the system per oscillator is given by

(a) ͲǤʹͷ (b) ͲǤͷ͸ (c) ͲǤ͸͵ (d) ͲǤ͹ͷ

15. In the temperature range ͳͲͲ െ ͳͲͲͲ C, the molar specific heat of a metal
varies with temperature ܶ (measured in degrees Celsius) according to the
formula ‫ܥ‬௣ ൌ  ሺͳ ൅ ܶȀͷሻ J-deg C-1-mol-1. If ͲǤʹ kg of the metal at ͸ͲͲ C is
brought in thermal contact with ͲǤͳ kg of the same metal at ͵ͲͲ C, the final
equilibrium temperature, in deg C, will be
[Assume that no heat is lost due to radiation and/or other effects.]

(a) Ͷ͸͸ (b) ͷ͸͹ (c) ͵ͺ͵ (d) ͷͳͻ

16. In a fixed target experiment, a proton of total energy 200 GeV is bombarded

on a proton at rest and produces a nucleus ஺௓ܰ and its anti-nucleus ஺௓ܰ
ܼ
ഥ൅‫݌‬൅‫݌‬
‫ ݌‬൅ ‫ ݌‬՜ ‫ ܼܰܣ‬൅ ‫ܰܣ‬
The heaviest nucleus ஺௓ܰthat can be created has atomic mass number ‫ ܣ‬ൌ

(a) ͳͷ (b) ͻ (c) ͷ (d) Ͷ

6
17. In a simple cubic lattice of lattice constant 0.287 nm, the number of atoms
per mm2 along the 111 plane is

(a) ʹǤͳͳ ൈ  ͳͲଵଷ (b) ͳǤ͹͵ ൈ  ͳͲଵଷ  (c) ͳǤʹͻ ൈ  ͳͲଵଷ (d) ͳǤʹͳ ൈ  ͳͲଵଷ 

18. In a glass fibre, light propagates by total internal reflection from the inner
surface. A very short pulse of light enters a perfectly uniform glass fibre at
‫ ݐ‬ൌ Ͳand emerges from the other end of the fibre with negligible losses. If the
refractive index of the glass used in the fibre is 1.5 and its length is exactly
1.0 km, the time ‫ ݐ‬at which the output pulse will have completely exited the
fibre will be

(a) ͷǤͲߤ• (b) ͹Ǥͷߤ• (c) ʹͷ݊• (d) ͹ͷͲ݊• 

19. In an ionization experiment conducted in the laboratory, different singly-


charged positive ions are produced and accelerated simultaneously using a
uniform electric field along the ‫ݔ‬-axis. If we need to determine the masses of
various ions produced, which of the following methods will NOT work

(a) Detect them at a fixed distance from the interaction point along ‫ݔ‬-axis
and measure their time of arrival.

(b) Apply a uniform magnetic field along ‫ݕ‬-axis and measure the deviation.

(c) Apply a uniform electric field along ‫ݕ‬-axis and measure the deviation.

(d) Apply a uniform electric field along ‫ݕ‬-axis and a (variable) uniform
magnetic field along ‫ݖ‬-axis simultaneously and note the zero deviation.

20. An observer stands at the edge of a swimming pool, as sketched in the figure
below.

This observer will perceive the pool as

(a) (b)

(c) (d)

7
21. Consider a process in which atoms of Actinium-226 ( ଶଶ଺
଼ଽ…) get converted to
ଶଶ଺
atoms of Radium-226 ( ଼଼ƒ) and the yield of energy is ͲǤ͸Ͷ MeV per atom.
This occurs through

‘–Š‫ ݌‬՜ ݊ ൅ ݁൅ ൅ ߥ݁  ‘–Š‫ ݌‬՜ ݊ ൅ ݁൅ ൅ ߥ݁ 


(a) and ‫ ݌‬൅  ݁ ି ՜ ݊ ൅ ߥ௘ (b) and ݊ ՜ ‫ ݌‬൅ ݁ ି ൅ ߥҧ௘

(c) Only ‫ ݌‬՜ ݊ ൅ ݁ ା ൅ ߥ௘ (d) Only ‫ ݌‬൅  ݁ ି ՜ ݊ ൅ ߥ௘

22. Two homonuclear diatomic molecules produce different rotational spectra,


even though the atoms are known to have identical chemical properties. This
leads to the conclusion that the atoms must be

(a) isotopes, i.e. with the same atomic number.

(b) isobars, i.e. with the same atomic weight.

(c) isotones, i.e. with the same neutron number.

(d) isomers, i.e. with the same atomic number and weight.

23. In the generalized operational


amplifier circuit shown on the right, ͷͲͲ k:
ͷ k:
the op. amp. has a very high input

impedance ( ܼ ൐ ͷͲM:) and an open

gain of 1000 for the frequency range
ܸଵ
under consideration. Assuming that ܼ ܸଶ
the op. amp. draws negligible current,
the voltage ratio ܸଶ Ȁܸଵ is
approximately

(a) െͳͻͲ (b) െͳͻͲ (c) െͻͲ (d) ͺͲ

24. In the transistor circuit shown on the


right, assume that the voltage drop LOAD
between the base and the emitter is ܴଶ
0.5 V.
͹Ǥͷ V
What will be the ratio of the
ܴଵ
resistances ܴଶ Ȁܴଵ , in order to make
ʹ k:
this circuit function as a source of
constant current, ‫ ܫ‬ൌ ͳ݉‫?ܣ‬

(a) ͶǤͷ (b) ͵ǤͲ (c) ʹǤͷ (d) ʹǤͲ

8
25. For the circuit depicted on the right, the input
voltage ܸ௜ is a simple sinusoid as shown ܴ
below, where the time period is much smaller ܸ௜ ܸ௢
compared to the time constant of this circuit. ‫ܥ‬

The voltage ܸ௢ across ‫ ܥ‬is best represented by


(a) (b)

(c) (d)

DO NOT WRITE IN THIS SPACE

9
GS-2016-X (Physics)
Section B
To be attempted only by candidates for Integrated Ph.D. programme.
(Candidates for Ph.D. programme will get NO credit for attempting this section.)

26. The integral



݀‫ݔ‬ ‫ݔ‬ ‫ݔ‬
න ൤‡š’ ൬െ ൰ െ ‡š’ ൬െ ൰൨
଴ ‫ݔ‬ ξ͵ ξʹ
evaluates to

(a) zero (b) ʹǤͲ͵ ൈ ͳͲିଶ (c) ʹǤͲ͵ ൈ ͳͲିଵ (d) ʹǤͲ͵

27. A uniform solid wheel of mass ‫ ܯ‬and radius‫ ݎ‬is halted


at a step of height h as shown in the figure. The
minimum force ‫ܨ‬, applied horizontally at the centre of ‫ܨ‬
the wheel, necessary to raise the wheel over this step is ‫ݎ‬
݄

ඥ݄ሺʹ‫ ݎ‬െ ݄ሻ ඥ݄ሺʹ‫ ݎ‬൅ ݄ሻ


(a) ‫݃ܯ‬ (b) ‫݃ܯ‬
‫ݎ‬൅݄ ‫ݎ‬െ݄

(c) ඥ݄ሺ‫ ݎ‬൅ ݄ሻ (d) ඥ݄ሺʹ‫ ݎ‬െ ݄ሻ


‫݃ܯ‬ ‫݃ܯ‬
‫ݎ‬െ݄ ‫ݎ‬െ݄

28. In a futuristic scenario, two spaceships, A and B, are running a race, where
they start from the same point (marked START) but fly in opposite directions
at constant speeds close to the speed of light. An observer fixed at the
starting point observes that they both cross the points marked END, which
are equidistant from the starting point, at the same time. Afterwards this
observer receives messages from both spaceships.

Which of the following could be true?

(a) Both A and B agree that A won the race.

(b) A and B both claim to have won the race.

(c) Both A and B agree that they crossed the end point simultaneously.

(d) A thinks B won the race while B thinks A won the race.

10
29. The equation of state for a gas is given by
ߙܰ ଶ
ቈ‫ ݌‬൅ ൬ ൰ ቉ ሺܸ െ ߚܰሻ ൌ ܰ݇஻ ܶ
ܸ
where ܲǡ ܸǡ ܶǡ ܰǡ and ݇஻ represent pressure, volume, temperature, number of
atoms and the Boltzmann constant, respectively, while ߙ and ߚ are
constants specific to the gas.

If the critical point C corresponds to a point of inflexion of the ‫݌‬-ܸ curve,


then the critical volume ܸ௖ and critical pressure ‫݌‬௖ for this gas are given by

(a) ܸ௖ ൌ ͵ߚܰǡ ‫݌‬௖ ൌ ߙ ଶ Τ͵ߚ ଶ (b) ܸ௖ ൌ ͵ߚܰǡ ‫݌‬௖ ൌ ߙ Τʹ͹ߚଶ

(c) ܸ௖ ൌ ͵ߚܰǡ ‫݌‬௖ ൌ ͺߙ ଶ Τʹ͹ߚ (d) ܸ௖ ൌ ͵ߚܰǡ ‫݌‬௖ ൌ ߙ ଶ Τʹ͹ߚ ଶ

30. The lattice constant of a material is of the order of a ߤ݉, and its bond energies
are of the order of an eV. The bulk modulus of such a material, in Pascals, is
of the order of

(a) ͳͲିଵ (b) ͳͲିଷ (c) ͳͲି଺ (d) ͳͲଽ

31. Two harmonic oscillators A and B are in excited eigenstates with the same
excitation energy E, as measured from their respective ground state energies.
The natural frequency of A is twice that of B.

If the wavefunction of B is as sketched in the above picture, which of the


following would best represent the wavefunction of A ?

(a) (b)

(c) (d)

11
32. Three positively charged particles lie on a straight line at positions Ͳǡ ‫ ݔ‬and ͳͲ
as indicated in the figure below. Their charges are ܳǡ ʹܳǡ and Ͷܳ cm
respectively.
ܳ ʹܳ Ͷܳ

Ͳ ‫ݔ‬ ͳͲ
If the charges at ‫ ݔ‬ൌ Ͳ and ‫ ݔ‬ൌ ͳͲ are fixed and the charge at ‫ݔ‬is movable, the
system will be in equilibrium when ‫ ݔ‬ൌ

(a) ͺ (b) ʹ (c) ʹͲΤ͵ (d) ͳͲΤ͵

33. Consider the following system. Two circular


loops of wire are placed horizontally, having a ܴ
common axis passing vertically through the
centre of each coil (see figure). The lower loop
has radius ‫ ݎ‬and carries a current ݅ as shown in
‫ݔ‬
the figure. The upper loop has a radius ܴ (ܴ ‫)ݎ ب‬
and is at a distance ‫ )ܴ ب ݔ(ݔ‬above it.

If the lower loop is held fixed and the upper loop


moves upwards with a uniform velocity ‫ ݒ‬ൌ
݀‫ݔ‬Ȁ݀‫ݐ‬, then the induced e.m.f. and the direction
of the induced current in this loop will be ݅
‫ݎ‬

(a) ͵݅ߤ଴ ߨ ଶ ‫ ݎ‬ଶ ܴଶ ‫ݒ‬Τʹ‫ ݔ‬ସ ; anti-clockwise (b) ʹ݅ߤ଴ ߨ ଶ ‫ ݎ‬ଶ ܴଶ ‫ݒ‬Τʹ‫ ݔ‬ସ ; clockwise

(c) ͵݅ߤ଴ ߨ ଶ ‫ ݎ‬ଶ ܴଶ ‫ݒ‬Τʹ‫ ݔ‬ଷ ; anti-clockwise (d) ʹ݅ߤ଴ ߨ ଶ ‫ ݎ‬ଶ ܴଶ ‫ݒ‬Τ͵‫ ݔ‬ଷ ; clockwise

34. A student in the laboratory is provided with a bunch of standard resistors as


well as the following instruments
 Voltmeter accurate to 0.1 V  Thermometer accurate to 0.1 C
 Ammeter accurate to 0.01 A  Stop watch accurate to 0.05 s
 Constant current source (ideal)  Constant voltage source (ideal)
Using this equipment (and nothing else), the student is expected to measure
the resistance ܴ of one of the given resistors. The least accurate result would
be obtained by

(a) measuring the Joule heating.

(b) passing a constant current and measuring the voltage across it.

(c) measuring the current on application of a constant voltage across it.

(d) the Wheatstone bridge method.

12
35. In a digital circuit for three input signals (A, B and C) the final output (Y)
should be such that for inputs
A B C
0 0 0
0 0 1
0 1 0
the output (Y) should be low and for all other cases it should be high.

Which of the following digital circuits will give such output?

(a) (b)

(c) (d)

PLEASE READ CAREFULLY BEFORE PROCEEDING FURTHER


The following questions (36 – 40) must be answered by integers of 3 digits each. You may
round off, e.g. ૚૛૜Ǥ ૙ ൑ ࢞ ൏ ͳʹ૜Ǥ ૞ as ࢞ ൌ ૚૛૜ and e.g. ૚૛૜Ǥ ૞ ൑ ࢞ ൑ ૚૛૝Ǥ ૙ as
࢞ ൌ ૚૛૝ and so on. Answer these questions on the OMR by filling in bubbles as you did
for your reference code. Use only values of constants given in the table ‘USEFUL
CONSTANTS’.
Note that if the answer is, e.g. 25, you must fill in 025 and if it is, e.g. 5, you must fill in
005. If it is 0, you must fill in 000. If the zeros are not filled in (where required), the answer
will be not be credited.
There are NO NEGATIVE MARKS for these questions.

36. On a planet having the same mass and diameter as the Earth, it is
observed that objects become weightless at the equator. Find the time period of
rotation of this planet in minutes (as defined on the Earth). Answer = 085

13
37. The function ‫ݕ‬ሺ‫ݔ‬ሻ satisfies the differential equation
݀‫ݕ‬
‫ݔ‬ ൌ ‫ݕ‬ሺŽ ‫ ݕ‬െ Ž ‫ ݔ‬൅ ͳሻ
݀‫ݔ‬
with the initial condition ‫ݕ‬ሺͳሻ ൌ ͵. What will be the value of ‫ݕ‬ሺ͵ሻ? Answer = 081

38. Two containers are maintained at the same temperature and are filled
with ideal gases whose molecules have mass ݉ଵ and ݉ଶ respectively. The
mean speed of molecules of the second gas is ͳͲ times the r.m.s. speed of
the molecules of the first gas. Find the ratio of ݉ଵ Ȁ݉ଶ , to the nearest
integer. Answer = 118

39. A particle is confined in a one-dimensional box


of unit length, i.e. ‫ ܮ‬ൌ ͳ, i.e. in a potential ܸሺ‫ݔ‬ሻ
Ͳ ‹ˆͲ ൏ ‫ ݔ‬൏ ͳ
ܸሺ‫ݔ‬ሻ ൌ ൝ ߰ሺ‫ݔ‬ሻ
λ ‘–Š‡”™‹•‡
The energy eigenvalues of this particle are
denoted ‫ܧ‬଴ , ‫ܧ‬ଵ , ‫ܧ‬ଶ , ‫ܧ‬ଷ , ...
‫ݔ‬
Ͳ ͳȀʹ ͳ
In a particular experiment, the wavefunction of
this particle, at ‫ ݐ‬ൌ Ͳ, is given by
ξʹ‹ˆͲ ൏ ‫ ݔ‬൏ ͳȀʹ
߰ሺ‫ݔ‬ሻ ൌ ൝ Answer = 000
Ͳ ‘–Š‡”™‹•‡

If, simultaneously, i.e. at ‫ ݐ‬ൌ Ͳ, a measurement of the energy of the particle is


made, find ͳͲͲ‫݌‬ଷ , where ‫݌‬ଷ is the probability that the measurement will yield
the energy ‫ܧ‬ଷ .

40. Consider a sawtooth waveform which rises linearly from Ͳ Volt to ͳ Volt in ͳͲ
ns and then decays linearly to Ͳ V over a period of ͳͲͲ ns. Find the r.m.s.
voltage in units of milliVolt? Answer = 577

DO NOT WRITE IN THIS SPACE

14
GS-2016-X (Physics)
Section C
To be attempted only by candidates for Ph.D. programme.
(Candidates for Integrated Ph.D. programme will get NO credit for attempting this section.)

41. The value of the integral


•‹ ‫ݖ‬
ර ݀‫ݖ‬
஼ ‫଺ݖ‬
where ‫ ܥ‬is the circle of centre z = 0 and radius =1

(a) ݅ߨ (b)݅ߨΤͳʹͲ (c) ݅ߨΤ͸Ͳ (d) െ ݅ߨΤ͸

42. In a Rutherford scattering experiment, the number ܰ of particles scattered in


a direction ߠ , i.e. ݀ܰȀ݀ߠ , as a function of the scattering angle ߠ (in the
laboratory frame) varies as
ߠ ߠ ߠ
(a) …•… ସ (b) …•… ଶ …‘–
ʹ ʹ ʹ
ߠ ߠ ߠ
(c) …•… ଶ –ƒଶ (d) •‡… ସ
ʹ ʹ ʹ

43. In a simple stellar model, the density ߩof a spherical star of mass ‫ܯ‬varies
according to the distance ‫ ݎ‬from the centre according to
‫ݎ‬ଶ
ߩሺ‫ݎ‬ሻ ൌ ߩ଴ ቆͳ െ ଶ ቇ
ܴ
where ܴ is the radius of the star. The gravitational potential energy of this star
(in terms of Newton’s constant ‫ܩ‬ே ) will be

(a) െ ‫ܩ‬ே ‫ܯ‬ଶ ΤͶߨܴ  (b) െ ͵‫ܩ‬ே ‫ܯ‬ଶ Τͷܴ

(c) െ ͷ‫ܩ‬ே ‫ܯ‬ଶΤ͹ܴ (d) െ ͵‫ܩ‬ே ‫ܯ‬ଶ Τ͹ܴ

44. A particle moving in one dimension is confined inside a rigid box located
between ‫ ݔ‬ൌ െܽȀʹ and ‫ ݔ‬ൌ ܽȀʹ. If the particle is in its ground state
ߨ‫ݔ‬
߰଴ ሺ‫ݔ‬ሻ ൌ  ඥʹΤܽ …‘•
ܽ
the quantum mechanical probability of its having a momentum ‫ ݌‬is given by

ͺ԰ସ ‫ܽ݌‬ ߨ ଶ ԰ସ ‫ܽ݌‬


(a) ଶ ଶ ଶ ଶ ଶ
…‘• ଶ (b) ଶ ଶ ଶ ଶ ଶ
•‹ଶ
ሺߨ ԰ െ ‫ ܽ ݌‬ሻ ʹ԰ ሺߨ ԰ െ ‫ ܽ ݌‬ሻ ʹ԰

ʹ԰ସ ‫ܽ݌‬ ͳ͸԰ସ


(c) ଶ ଶ ଶ ଶ ଶ
…‘• ଶ (d)
ሺߨ ԰ ൅ ‫ ܽ ݌‬ሻ ʹ԰ ሺߨ ଶ ԰ଶ െ ‫݌‬ଶ ܽଶ ሻଶ

15
45. Consider two spin-1/2 identical particles A and B, separated by a distance ‫ݎ‬,
interacting through a potential
ܸ଴
ܸሺ‫ݎ‬ሻ ൌ  ܵԦ஺ Ǥ ܵԦ஻
‫ݎ‬
where ܸ଴ is a positive constant and the spins are ܵԦ஺ǡ஻ ൌ ߪԦ ൌ ൫ߪ௫ ǡ ߪ௬ ǡ ߪ௭ ൯ in terms
of the Pauli spin matrices. The expectation values of this potential in the spin-
singlet and triplet states are
ܸ଴ ܸ଴ ͵ܸ଴ ܸ଴
(a) ‹‰Ž‡– ‫  ׷‬െ ǡ ”‹’Ž‡– ‫ ׷‬ (b) ‹‰Ž‡– ‫  ׷‬െ ǡ ”‹’Ž‡– ‫  ׷‬
͵‫ݎ‬ ‫ݎ‬ ‫ݎ‬ ‫ݎ‬
͵ܸ଴ ܸ଴ ܸ଴ ͵ܸ଴
(c) ‹‰Ž‡– ‫ ׷‬ ǡ ”‹’Ž‡– ‫  ׷‬െ (d) ‹‰Ž‡– ‫  ׷‬െ ǡ ”‹’Ž‡– ‫ ׷‬
‫ݎ‬ ‫ݎ‬ ‫ݎ‬ ‫ݎ‬

46. Two semi-infinite slabs A and B of dielectric constant ߳஺ and ߳஻ meet in a


plane interface, as shown in the figure below.

A B
‫ܧ‬ሬԦ஻ 
ߠ஻ 
ߠ஺ 

‫ܧ‬ሬԦ஺ 
߳஺  ߳஻ 

If the electric field in slab A makes an angle ߠ஺ with the normal to the
boundary and the electric field in slab B makes an angle ߠ஻ with the same
normal (see figure), then

߳஺ ߳஺
(a) …‘•ߠ୅ ൌ  …‘•ߠ୆ (b) •‹ߠ஺ ൌ  •‹ ߠ஻
߳஻ ߳஻
߳஺ ߳஻
(c) –ƒߠ୅ ൌ  –ƒߠ୆ (d) •‹ߠ஺ ൌ  •‹ ߠ஻
߳஻ ߳஺

47. The Weizsäcker semi-empirical mass formula for an odd nucleus with ܼ
protons and ‫ ܣ‬nucleons may be written as
‫ܯ‬ሺܼǡ ‫ܣ‬ሻ ൌ  ߙଵ ‫ ܣ‬൅  ߙଶ ‫ܣ‬ଶȀଷ ൅  ߙଷ ܼ ൅  ߙସ ܼ ଶ
where the ߙ௜ are constants independent of ܼǡ ‫ ܣ‬. For a given ‫ ܣ‬, if ܼ஺  is the
number of protons of the most stable isobar, the total energy released when
an unstable nuclide undergoes a single ߚ ି decay to (ܼ஺ ǡ ‫ )ܣ‬is

(a) ߙଷ (b) ߙସ (c) ߙସ െ ߙଷ (d) ߙଵ ൅ ߙଶ

16
48. In the experiment shown in figure (i) below, the emitted electrons from the
cathode (C) are made to pass through the mercury vapor filled in the tube by
accelerating them using a grid (G) at potential V, positive w.r.t. the cathode.
The electrons are collected by the anode (A).

Voltage

(i) (ii)

The variation of electron current (I) as a function of V is given in figure (ii). The
shape of this curve must be interpreted as due to

(a) ionization of mercury atoms.


(b) an emission line from mercury atoms.
(c) attachment of electrons to mercury atoms.
(d) resonant backscattering of electrons to cathode from grid.

49. The dispersion relation for electrons in the conduction band of a ݊ -type
semiconductor has the form ‫ܧ‬ሺ݇ሻ ൌ ܽ݇ ଶ ൅ ܾ where ܽ and ܾ are constants. It
was observed that the cyclotron resonance frequency of such electrons is
߱଴ ൌ ͳǤͺ ൈ ͳͲଵଵ rad s-1, when placed in a magnetic field ‫ ܤ‬ൌ ͲǤͳ W m-2. It follows
that the constant ܽ must be about

(a) ͳͲିଷ଺ (b) ͳͲିଶ଼ (c) ͳͲିଷଶ (d) ͳͲିଷ଼

50. Consider the hyperon decay ሺͳሻ߉ ՜ ݊ ൅  ߨ ଴ followed by ሺʹሻߨ ଴ ՜ ߛߛ . If the


isospin component, baryon number and strangeness quantum numbers are
denoted by ‫ܫ‬௭ , ‫ ܤ‬and ܵ respectively, then which of the following statements is
completely correct?

(a) In (1) ‫ܫ‬௭ is not conserved, ‫ ܤ‬is conserved, ܵ is not conserved;


In (2) ‫ܫ‬௭ is conserved, ‫ ܤ‬is conserved, ܵ is conserved.

(b) In (1) ‫ܫ‬௭ is conserved, ‫ ܤ‬is not conserved, ܵ is not conserved;


In (2) ‫ܫ‬௭ is conserved, ‫ ܤ‬is conserved, ܵ is conserved.

(c) In (1) ‫ܫ‬௭ is not conserved, ‫ ܤ‬is conserved, ܵ is not conserved;


In (2) ‫ܫ‬௭ is not conserved, ‫ ܤ‬is conserved, ܵ is conserved.

(d) In (1) ‫ܫ‬௭ is not conserved, ‫ ܤ‬is conserved, ܵ is conserved;


In (2) ‫ܫ‬௭ is conserved, ‫ ܤ‬is conserved, ܵ is conserved.

17
PLEASE READ CAREFULLY BEFORE PROCEEDING FURTHER
The answer to the following questions (51 – 55) must be answered by integers of 3 digits
each. You may round off, e.g. ૚૛૜Ǥ ૙ ൑ ࢞ ൏ ͳʹ૜Ǥ ૞ as ࢞ ൌ ૚૛૜ and e.g. ૚૛૜Ǥ ૞ ൑ ࢞ ൑
૚૛૝Ǥ ૙ as ࢞ ൌ ૚૛૝ and so on. Answer these questions on the OMR by filling in bubbles as
you did for your reference code. Use only values of constants given in the table ‘USEFUL
CONSTANTS’.
Note that if the answer is, e.g. 25, you must fill in 025 and if it is, e.g. 5, you must fill in
005. If it is 0, you must fill in 000. If the zeros are not filled in (where required), the answer
will be not be credited.
There are NO NEGATIVE MARKS for these questions.

51. Given the infinite series


ሺ݊ ൅ ͳሻሺ݊ ൅ ʹሻ ௡
‫ݕ‬ሺ‫ݔ‬ሻ ൌ ͳ ൅ ͵‫ ݔ‬൅ ͸‫ ݔ‬ଶ ൅ ͳͲ‫ ݔ‬ଷ ൅ ‫ ڮ‬൅ ‫ ݔ‬൅ ‫ ڮ‬Answer = 343
ʹ
find the value of ‫ݕ‬ሺ‫ݔ‬ሻ for ‫ ݔ‬ൌ ͸Ȁ͹.

52. A quantum mechanical plane rotator consists of two rigidly connected


particles of mass m and connected by a massless rod of length d is rotating in
the x-y plane about their centre of mass. Suppose that the initial state of the
rotor is given by
߰ሺ߮ǡ ‫ ݐ‬ൌ Ͳሻ ൌ ‫•‘… ܣ‬ଶ ߮ǡ Answer = 004
where ߮ is the angle between one mass and the x axis, while ‫ ܣ‬is a
normalization constant. Find the expectation value of ͵‫ܮ‬෠ଶ௭ in this state, in
units of ԰ଶ .

53. A continuous monochromatic (ߣ ൌ ͸ͲͲ ) laser beam is chopped into
0Ǥ ͳ• pulses using some sort of shutter. Find the resultant linewidth ߂ߣ of
the beam in units of ͳͲିଷ . Answer = 012

54. N distinguishable particles are distributed among three states having


energies ‫ ܧ‬ൌ Ͳǡ ݇஻ ܶ and ʹ݇஻ ܶ respectively. If the total equilibrium energy of
the system is ͳ͵ͺǤͲ͸݇஻ ܶ, find the number ܰof particles. Answer = 325

18
55. The circuit shown below contains an unknown device X.
100 :

100 :
Answer = 012
ࡵ 2V

The current-voltage characteristics of the device X were determined and are


shown in the plot given below.

Determine the current I (in mA) flowing through the device X.

DO NOT WRITE IN THIS SPACE

19
DO NOT WRITE IN THIS SPACE

20
PHY – X
GS-2017-X (Physics)
TATA INSTITUTE OF FUNDAMENTAL RESEARCH
Wrtten Test in PHYSICS – December 11, 2016
Duration : 3 (Three) Hours

NAME:_____________________________________REF. CODE:________
Please Read These Instructions Carefully Before Attempting the Questions
1. Please fill in details about name, reference code etc. on the answer sheet and
question paper. Use only blue/black ballpoint pen. The Answer Sheet is
machine –readable and will not read other colours.
2. This test consists of three parts: Section A, Section B and Section C. You must
answer questions according to the programme you are applying for.
Candidates applying for Must answer Should not attempt
Integrated M.Sc.-Ph.D. Section A + Section B Section C
Ph.D. Section A + Section C Section B
Section A has 25 questions : 1 – 15 are multiple-choice; 16 – 25 are numerical.
Section B has 15 questions : 26 – 35 are multiple-choice; 36 – 40 are symbolic.
Section C has 15 questions : 41 – 50 are multiple-choice; 51 – 55 are symbolic.
3. Indicate your ANSWER ON THE ANSWER SHEET as follows.
Multiple choice questions have four options (a), (b), (c) and (d), of which only
one option is correct. Indicate the answers by filling up the bubble on the
Answer Sheet corresponding to the correct option. If more than one bubble is
filled in, it will be treated as not answered.
Numerical questions have answers which are 3 (three) digit integers. Indicate
the answers by filling in the corresponding bubbles on the Answer Sheet.
Unless all three bubbles for a given question are filled, it will be treated as not
answered. (See inside for details.)
Symbolic questions have answers which are a number, a short formula or a
word. Indicate the answers by writing in the boxes on the Answer Sheet next
to the appropriate question numbers. (See inside for details.)
4. The marking for these questions shall be as follows.
If the answer is Multiple-choice Numerical Symbolic
Correct ൅͵ ൅ͷ ൅ͷ
Incorrect െͳ Ͳ Ͳ
Not attempted Ͳ Ͳ Ͳ
Multiple options marked Ͳ  
Note that only multiple-choice type questions have negative marking.
continued on next page…
5. Candidates are advised to mark the Answer Sheet only when they are sure of
the answer. Till then, they may mark the answers on the question paper.
6. Rough work may be done on blank pages of the question paper. If needed,
you may ask for extra sheets from the invigilators.
7. Use of scientific, non-programmable calculators is permitted. Calculators
which plot graphs are NOT allowed. Multiple-use devices, such as cell phones,
smartphones, etc. CANNOT be used as calculators.
8. Do NOT ask the invigilators for clarifications regarding the questions. They
have been instructed not to respond to any such queries. In case a
correction/clarification is deemed necessary, it will be announced in the
examination hall.
9. A list of useful physical constants is given on the next page. Make sure to use
only these values in answering the questions, especially those of numeric
type.
USEFUL CONSTANTS
Symbol Name/Definition Value
ܿ speed of light in vacuum ͵ ൈ ͳͲ଼ m s-1
԰ reduced Planck constant (ൌ ݄Ȁʹߨ) ͳǤͲͶ ൈ ͳͲିଷସ Js
‫ܩ‬ே gravitational constant ͸Ǥ͸͹ ൈ ͳͲିଵଵ m3 kg-1 s-2
‫ٖܯ‬ solar mass ͳǤͻͺͻ ൈ ͳͲଷ଴ kg
ߝ଴ permittivity of free space ͺǤͺͷ ൈ ͳͲ ିଵଶ F m-1
ߤ଴ permeability of free space Ͷߨ ൈ ͳͲି଻ N A-2
݁ electron charge (magnitude) ͳǤ͸ ൈ ͳͲିଵଽ C
݉௘ electron mass ͻǤͳ ൈ ͳͲିଷଵ kg
ൌ ͲǤͷ MeV/c 2

ܽ଴ Bohr radius 0.51 Å


ionisation potential of H atom 13.6 eV
ܰ஺ Avogadro number ͸ǤͲʹ͵ ൈ ͳͲଶଷ mol-1
݇஻ Boltzmann constant ͳǤ͵ͺ ൈ ͳͲିଶଷ J K-1
ൌ ͺǤ͸ͳ͹͵ ൈ ͳͲିହ eV K-1
ܴ ൌ ܰ஺ ݇஻ gas constant ͺǤ͵ͳ J mol-1 K-1
ߛ ൌ ‫ܥ‬௣ Ȁ‫ܥ‬௏ ratio of specific heats: monatomic gas 1.67
diatomic gas 1.40
ߪ Stefan-Boltzmann constant ͷǤ͸͹ ൈ ͳͲି଼ W m-2 K-4
ߙ fine structure constant (ൌ ݁ ଶ ȀͶߨߝ଴ ԰ܿ) 1/137
݃ acceleration due to gravity 9.8 m s-2
ܴா radius of the Earth ͸ǤͶ ൈ ͳͲଷ Km
ܴௌ radius of the Sun ͹ ൈ ͳͲହ Km
݉௣ proton mass (ൎ ʹͲͲͲ݉௘ ) ͳǤ͹ ൈ ͳͲିଶ଻ kg
ൌ ͻ͵ͺǤʹ MeV/c 2

݉௡ neutron mass (ൎ ʹͲͲͲ݉௘ ) ͳǤ͹ ൈ ͳͲିଶ଻ kg


ൌ ͻ͵ͻǤ͸ MeV/c 2

UNIT CONVERSIONS
Symbol Name/Definition Value
1 A.U. mean distance of Earth from Sun ͳǤͷ ൈ ͳͲଽ km
1 a.m.u. atomic mass unit ͳǤ͸ ൈ ͳͲିଶ଻ kg
ൌ ͻ͵ͳǤͷ MeV/c 2

1 eV electron Volt ͳǤ͸ ൈ ͳͲିଵଽ J


1T Tesla ͳͲସ gauss
1 bar mean atmospheric pressure at 00 C ͳǤͲͳ ൈ ͳͲହ Pa (= N m-2)
1Å Ångstrom unit 10-8 cm
԰ܿ conversion constant ͵Ǥͳ͸ ൈ ͳͲିଶ଺ J m-1
ൌ ͲǤͳͻ͹͵ GeV fm
GS-2017-X (Physics)
Section A
Q. 1 – 25 : to be attempted by ALL candidates

PLEASE READ CAREFULLY BEFORE PROCEEDING FURTHER

The following questions (1 – 15) are all of multiple-choice type. For


every question, four options (a), (b), (c) and (d) are given, of which only
one is correct. Indicate the correct option on the OMR by filling the
bubble.

A correct answer will be awarded ൅૜ marks and an incorrect answer will


be awarded െ૚ mark. If the question is not attempted, no marks will be
awarded.

1. Denote the commutator of two matrices ‫ ܣ‬and ‫ ܤ‬by ሾ‫ܣ‬ǡ ‫ܤ‬ሿ ൌ ‫ ܤܣ‬െ ‫ ܣܤ‬and the
anti-commutator by ሼ‫ܣ‬ǡ ‫ܤ‬ሽ ൌ ‫ ܤܣ‬൅ ‫ܣܤ‬Ǥ

If ሼ‫ܣ‬ǡ ‫ܤ‬ሽ ൌ Ͳ, we can write ሾ‫ܣ‬ǡ ‫ܥܤ‬ሿ ൌ

(a) െ‫ܤ‬ሾ‫ܣ‬ǡ ‫ܥ‬ሿ (b) ‫ܤ‬ሼ‫ܣ‬ǡ ‫ܥ‬ሽ (c) െ‫ܤ‬ሼ‫ܣ‬ǡ ‫ܥ‬ሽ (d) ሾ‫ܣ‬ǡ ‫ܥ‬ሿ‫ܤ‬

2. Consider the waveform ‫ݔ‬ሺ‫ݐ‬ሻ shown in the diagram below.


‫ݔ‬
ͳ
െʹܶ െܶ
‫ݐ‬
Ͳ ܶ ʹܶ ͵ܶ

െͳ
The Fourier series for ‫ݔ‬ሺ‫ݐ‬ሻwhich gives the closest approximation to this waveform is

ʹ ߨ‫ͳ ݐ‬ Ͷߨ‫ͳ ݐ‬ ͵ߨ‫ݐ‬


(a) ‫ݔ‬ሺ‫ݐ‬ሻ ൌ ൤…‘• െ …‘• ൅ …‘• ൅  ǥ൨
ߨ ܶ ʹ ܶ ͵ ܶ
ʹ ߨ‫ͳ ݐ‬ ʹߨ‫ͳ ݐ‬ ͵ߨ‫ݐ‬
(b) ‫ݔ‬ሺ‫ݐ‬ሻ ൌ ൤െ•‹ ൅ •‹ െ •‹ ൅  ǥ൨
ߨ ܶ ʹ ܶ ͵ ܶ
ʹ ߨ‫ͳ ݐ‬ ʹߨ‫ͳ ݐ‬ ͵ߨ‫ݐ‬
(c) ‫ݔ‬ሺ‫ݐ‬ሻ ൌ ൤•‹ െ •‹ ൅ •‹ ൅  ǥ൨
ߨ ܶ ʹ ܶ ͵ ܶ
ʹ ʹߨ‫ͳ ݐ‬ Ͷߨ‫ͳ ݐ‬ ͸ߨ‫ݐ‬
(d) ‫ݔ‬ሺ‫ݐ‬ሻ ൌ ൤െ…‘• ൅ …‘• െ …‘• ൅  ǥ൨
ߨ ܶ ʹ ܶ ͵ ܶ
3. A solid tetrahedron (solid with four plane sides) has the following projections
(drawn to scale) when seen from three different sides:

Left Top Right


When viewed from the front, its projection will be

(a) (b)

(c) (d)

4. A small elastic ball of mass ݉ is placed at the apex of a Ͷͷ௢ inclined plane as
shown in the figure below.

The ball is allowed to slip without friction down the plane (along the dotted
line), hit the ground (as shown) and bounce along it. If the height of the inclined
plane is ݄ and the coefficient of restitution between the ball and the ground is
ͲǤͷ, then the distance AB, as marked on the figure, will be

(a) ͵݄ (b) ʹ݄ (c) ൫ͳ ൅ ξʹ൯݄ (d) ͵ξʹ݄

5. Two masses ͵݉ and ʹ݉ are suspended vertically by


identical massless springs, each of stiffness constant ݇. ݇
The mass ʹ݉ is suspended from the mass ͵݉ and the
͵݉
mass ͵݉is suspended from a rigid support, as shown in
the figure. If only vertical motion is permitted, the ݇
frequencies of small oscillations of this system are ʹ݉

݇ ݇ ݇ ݇ ݇ ͵݇ ʹ݇ ͵݇
(a) ඨ ǡඨ (b) ඨ ǡඨ (c) ඨ ǡඨ (d) ඨ ǡඨ
݉ ͸݉ ʹ݉ ͵݉ ݉ ʹ݉ ͵݉ ʹ݉
6. Two long hollow conducting cylinders, each of height ݄, are placed concentrically
on the ground, as shown in the figure (top view). The outer cylinder is grounded,
while the inner cylinder is insulated. A positive charge (the black dot in the figure)
is placed between the cylinders at a height ݄Ȁʹ from the ground.

Which of the following figures gives the most accurate representation (top view) of
the lines of force ?

(a) (b)

(c) (d)

7. A common model for the distribution of charge in a hydrogen atom has a point-like
proton of charge ൅‫ݍ‬଴ at the centre and an electron with a static charge density
distribution
‫ݍ‬଴ ିଶ௥Ȁ௔
ߩሺ‫ݎ‬ሻ ൌ  െ ݁
ߨܽଷ
ሬԦ at ‫ ݎ‬ൌ ܽ due to this system of charges
where ܽ is a constant. The electric field ‫ܧ‬
will be

ͷ‫ݍ‬଴ ͷ‫ݍ‬଴
(a) െ Ͷߨ߳ ݁ ଶ ܽଶ ‫ݎ‬Ƹ (b) െ ‫ݎ‬Ƹ
଴ Ͷߨ߳଴ ݁ܽଶ

ͷ‫ݍ‬଴ ͵‫ݍ‬଴
(c) Ͷߨ߳ ݁ ଶ ܽଶ ‫ݎ‬Ƹ (d)  ‫ݎ‬Ƹ
଴ Ͷߨ߳଴ ݁ ଶ ܽଶ
8. A rectangular metallic loop with sides ‫ܮ‬ଵ and ‫ܮ‬ଶ is
placed in the vertical plane, making an angle ߮
with respect to the ‫ݔ‬-axis, as shown in the figure,
and a spatially uniform magnetic field ‫ܤ‬ ሬԦ  ൌ ‫ݕܤ‬ො is
applied. The loop is free to rotate about the‫ݖ‬Ƹ axis
(shown in the figure with a double line). The
magnetic field changes with time at a constant rate
݀‫ܤ‬
ൌߢ
݀‫ݐ‬
If the resistance of the loop is R, the torque ߬
required to prevent the loop from rotating will be

ሺ‫ܮ‬ଵ ‫ܮ‬ଶ ሻଶ ሺ‫ܮ‬ଵ ‫ܮ‬ଶ ሻଶ


(a) െߢ‫ܤ‬ •‹ ʹ߮ ‫ݖ‬Ƹ (b) ߢ‫ܤ‬ •‹ ߮ …‘• ߮ ‫ݖ‬Ƹ
ʹܴ ܴ
ሺ‫ܮ‬ଵ ‫ܮ‬ଶ ሻଶ ሺ‫ܮ‬ଵ ‫ܮ‬ଶ ሻଶ
(c) ߢ‫ܤ‬ •‹ ߮ ‫ݖ‬Ƹ (d) െߢ‫ܤ‬ •‹ ߮ ‫ݖ‬Ƹ
ʹܴ ܴ

9. Consider the 1-D asymmetric double-well potential ܸሺ‫ݔ‬ሻas sketched below.


ܸሺ‫ݔ‬ሻ

‫ݔ‬

The probability distribution ‫݌‬ሺ‫ݔ‬ሻ of a particle in the ground state of this potential is
best represented by

(a) ‫݌‬ሺ‫ݔ‬ሻ (b) ‫݌‬ሺ‫ݔ‬ሻ

‫ݔ‬ ‫ݔ‬

(c) ‫݌‬ሺ‫ݔ‬ሻ (d) ‫݌‬ሺ‫ݔ‬ሻ

‫ݔ‬ ‫ݔ‬
10. The normalized wave function of a particle can be written as
ஶ ௡
ͳ
Ȳሺ‫ݔ‬ሻ ൌ ܰ ෍ ൬ ൰ ߮௡ ሺ‫ݔ‬ሻ
௡ୀ଴
ξ͹
where ߮௡ ሺ‫ݔ‬ሻ are the normalized energy eigenfunctions of a given Hamiltonian. The
value of ܰ is

(a) ඥͳΤ͹ (b) ඥ͸Τ͹

(c) ඥ͵Τ͹ (d) ට൫͸ െ ʹξ͹൯Τ͹

11. Consider a system of non-interacting particles with integer angular momentum ‫ܬ‬
at a temperature ܶ. This system is placed in a magnetic field ‫ ܤ‬in the z direction.
The energy of a state with ‫ܬ‬௭ ൌ ݉԰ is
‫ܧ‬௠ ൌ ݉ߤ஻ ‫ܤ‬
with ߤ஻ ൐ ͲǤ The fractional magnetization of the particles as a function of ߤ஻ ‫ܤ‬Ȁ݇஻ ܶ
can be represented as

(a) (b)

(c) (d)
12. The separation between neighbouring absorption lines in a pure rotational
spectrum of the hydrogen bromide (HBr) molecule is ʹǤʹ͵ meV. If this molecule is
considered as a rigid rotor and the atomic mass number of Br is 80, the
corresponding absorption line separation in deuterium bromide (DBr) molecule, in
units of meV, would be

(a) ʹǤʹ͵Ͷ (b) ͳǤͳͳͷ (c) ͳǤͳʹͺ (d) ͶǤͶ͸ͳ

13. Consider a 2-D square lattice. The ratio of the kinetic energy of a free electron at a
corner of the first Brillouin zone (‫ܧ‬௖ ) to that of an electron at the midpoint of a side
face of the same zone (‫ܧ‬௠ ) is ‫ܧ‬௖ Τ‫ܧ‬௠ ൌ

(a) ͳȀʹ (b) ʹ (c) ξʹ (d) 1

14. A current source produces a square wave ‫ܫ‬ሺ‫ݐ‬ሻ of ͳǤͲ V peak-to-peak voltage and is
used to drive the RC circuit shown below.

Which of the following represents the correct voltage across the capacitor C ?

(a) (b)

(c) (d)
15. The output (Y) of the following circuit will be

(a) ‫ܣ‬ҧ ൅ ‫ ܤ‬൅ ‫ܥ‬ҧ (b) ‫ܣ‬ҧ (c) ‫ܤ‬ത (d) ‫ܥ‬ҧ


Section A continues (to be answered by ALL candidates)


PLEASE READ CAREFULLY BEFORE PROCEEDING FURTHER

The answers to the following questions (16 – 25) are all integers of 3 (three) digits
each. You may round off decimal parts, e.g. ૚૛૛Ǥ ૞ ൑ ࢞ ൏ ૚૛૜Ǥ ૞ǡas࢞ ൌ ૚૛૜ and e.g.
૚૛૜Ǥ ૞ ൑ ࢞ ൏ ૚૛૝Ǥ ૞as ࢞ ൌ ૚૛૝ and so on.

Use only values of constants given in the table ‘USEFUL CONSTANTS’.

Answer these questions on the OMR by filling in bubbles as you did for your
reference code. Note that if the answer is, e.g. 25, you must fill in 025 and if it is,
e.g. 5, you must fill in 005. If it is 0, you must fill in 000. If the zeros are not filled
in (where required), the answer will not be credited.

A correct answer will be awarded ൅૞ marks.


There are NO NEGATIVE MARKS for these questions.

16. The matrix


ͳͲͲξʹ ‫ݔ‬ Ͳ
ቌ െ‫ݔ‬ Ͳ െ‫ ݔ‬ቍ
Ͳ ‫ݔ‬ ͳͲͲξʹ
where ࢞ ൐ Ͳ, is known to have two equal eigenvalues. Find the value of ‫ݔ‬.

Ans = 050

17. A space telescope in orbit around the Earth discovers a new planet, which is
observed to move around the Sun by an angle of ͶǤ͹ʹ milliradians in a year.
Assuming a circular orbit, estimate the distance, in A.U., of the planet from the
Sun.
Ans = 121
18. A system of particles occupying single-particle levels and obeying Maxwell-
Boltzmann statistics is in thermal equilibrium with a heat reservoir at
temperatureܶ. If the population distribution in the non-degenerate energy levels is
as shown in the table below, what would be the temperature of the system in
degree Kelvin?

Energy (eV) PopulationΨ


͵ͲǤ͵Ͳ ͵Ǥͳ͸
Given +5 marks ʹͳǤ͸Ͳ ͺǤ͸ͻ
to all students. ͳ͵ǤͲͳ ʹ͵ǤͷͶ
ͶǤ͵ͳ ͸ͶǤ͸ͳ

19. A thermally isolated container stores ܰଶ gas at ʹ͹ǤʹͶԨ at one atmospheric


pressure. Suddenly the pressure of the gas is increased to two atmospheric
pressures. Assuming ܰଶ to behave as an ideal gas, estimate the change in
temperature of the gas, in Celsius degrees (Ԩ).
Ans = 066

20. A beam of plane microwaves of wavelength 12 cm strikes the surface of a dielectric


at Ͷͷ୭ . If the refractive index of the dielectric is 4/3, what will be the wavelength, in
units of mm, of the microwaves inside the dielectric?
Ans = 090

21. A quantum mechanical system consists of a one-dimensional infinite box, as


indicated in the figures below.

‫ܧ‬଴

3 (three) identical non-interacting spin-½ particles, are first placed in the box, and
the ground state energy of the system is found to be ‫ܧ‬଴ ൌ 18 eV. If 7 (seven) such
identical particles are placed in the box, what will be the ground state energy, in
units of eV?

Ans = 132
22. The energy of an electron in the ground state of the ‡ atom is െ͹ͻ‡. Considering
the Bohr model of the atom, what would be 10 times the first ionization potential
for a ‡ା ion, in units of eV?
Ans = 246

23. Cosmic ray muons, which decay spontaneously with proper lifetime ʹǤʹߤ s, are
produced in the atmosphere, at a height of 5 km above sea level. These move
straight downwards at 98% of the speed of light.
Find the percent ratio ͳͲͲ ൈ ሺܰ஺ Ȁܰ஻ ሻ of the number of muons measured at the top of
two mountains A and B, which are at heights 4,848 m and 2,682 m respectively
above mean sea level.
Ans = 052

24. A signal is to be sent from a coaxial cable with impedance 40 Ω into a second
coaxial cable with impedance 60 Ω. We can prevent reflection at the joint between
the cables, by adding an impedance in parallel to the second cable. What should be
the value, in units of Ohms (Ω), of that impedance?
Ans = 120

25. An AC voltage source has an internal resistance of 50 Ω and is specified to deliver


an rms voltage of 50 V to a matched load. If you connect this AC source to a
cathode-ray oscilloscope with 1 MΩ input setting, what will be the peak-to-peak
voltage you observe?

Ans = 283
GS-2017-X (Physics)
Section B
Q. 26 – 40 : to be attempted only by all candidates for
Integrated M.Sc.-Ph.D. programme.
(Candidates for Ph.D. programme will get NO credit for attempting this section.)

PLEASE READ CAREFULLY BEFORE PROCEEDING FURTHER

The following questions (26 – 35) are all of multiple-choice type. For
every question, four options (a), (b), (c) and (d) are given, of which only
one is correct. Indicate the correct option on the OMR by filling in the
bubble next to the correct label.

A correct answer will be awarded ൅૜ marks and an incorrect answer will


be awarded െ૚ mark. If the question is not attempted, no marks will be
awarded.

26. A unitary matrix ܷ is expanded in terms of a Hermitian matrix ‫ܪ‬, such that

ܷ ൌ ݁ ௜గுȀଶ
If we know that

ͳൗ Ͳ ξ͵ൗ
ʹ ʹ
‫ܪ‬ൌ‫Ͳ ۇ‬ ͳ Ͳ ‫ۊ‬
ξ͵ Ͳ െ ͳൗʹ‫ی‬
‫ ۉ‬ൗʹ

then ܷ must be

݅ ͳΤʹ ξ͵Τʹ ݅ Τʹ Ͳ ݅ξ͵Τʹ


(a) ቌ ͳΤʹ ݅ ͳ Τ ʹቍ (b) ቌ Ͳ ݅ Ͳ ቍ
ξ͵Τʹ ͳ Τʹ ݅ ݅ξ͵Τʹ Ͳ െ ݅ Τʹ

ͳ Ͳ ξ͵ ʹ݅ ͳ ξ͵Τʹ
(c) ቌ Ͳ ʹ Ͳቍ (d) ቌ ͳ ʹ݅ Ͳ ቍ
ξ͵ Ͳ െͳ ξ͵Τʹ Ͳ ʹ݅
27. A liquid is flowing through a capillary tube of inner radius r under the influence of
an external pressure P. The uncertainties in the measurements of P and r are found
to be 2% and 1%, respectively. The uncertainty in the flow of liquid per second is

(a) 4.47% (b) 2.23% (c) 2.83% (d) ͵Ǥ͸ͳΨ

28. A uniform solid sphere ଵ of radius ‫ ݎ‬and mass ݉ is rolling S1


without slipping on top of another sphere ଶ of radius ܴ, as N ‫ݎ‬
shown in the figure. Initially, ଵ was at rest directly on top ܴߠ P
of ଶ , and then it started rolling down under the influence
of gravity. The point of contact P subtends an
instantaneous angle ߠ from the topmost point N of the S2
lower sphere at the centre of the lower sphere.
At what minimum value of ߠ will the spheres lose contact?

ͷ ͷ ʹ ͳʹ
(a) …‘•ିଵ (b) …‘•ିଵ
ିଵ
(c) …‘• ͵ (d) …‘•ିଵ
ͳʹ ͳ͵ ͳ͵

29. An electromagnetic wave in free space is described by


ͳ
‫ܧ‬ሬԦ ሺ‫ݔ‬ǡ ‫ݕ‬ǡ ‫ݖ‬ǡ ‫ݐ‬ሻ ൌ  ‫ݖ‬Ƹ ‫ܧ‬଴ …‘• ൫݇‫ ݔ‬െ ξ͵݇‫ ݕ‬െ ʹ߱‫ݐ‬൯
ʹ
The Poynting vector associated with this wave is along the direction

(a) ‫ݔ‬ො ൅ ξ͵‫ݕ‬ො (b) ξ͵‫ݔ‬ො ൅  ‫ݕ‬ො

(c) െξ͵‫ݔ‬ො ൅  ‫ݕ‬ො (d) ‫ݔ‬ො െ ξ͵‫ݕ‬ො

30. Electrons in a given system of hydrogen atoms are described by the wave function
߰ሺ‫ݎ‬ǡ ߠǡ ߮ሻ ൌ ͲǤͺΨଵ଴଴ ൅ ͲǤ͸݁ ௜గΤଷ Ψଷଵଵ
where the Ψ௡ℓ௠ denote normalized energy eigenstates. If ൫‫ܮ‬෠௫ ǡ ‫ܮ‬෠௬ ǡ ‫ܮ‬෠௭ ൯ are the
components of the orbital angular momentum operator, the expectation value of ‫ܮ‬෠ଶ௫
in this system is

(a) ͳǤͷ԰ଶ (b) ͲǤ͵͸԰ଶ (c) ͲǤͳͺ԰ଶ (d) Zero


31. In two dimensions, two metals A and B, have the number density of free
electrons in the ratio
݊஺ ‫݊ ׷‬஻  ൌ ͳ ‫ʹ ׷‬

The ratio of their Fermi energies is

(a) ʹǣ ͵ (b) 1:2 (c) ͳǣ Ͷ (d) ͳǣ ͺ

32. One mole of monoatomic ideal gas is


initially at pressure ܲ଴ and volume ܸ଴ .
The gas then undergoes a three-stage
cycle consisting of the following
processes:
i) An isothermal expansion till it
reaches volume ʹܸ଴, and heat Q flows
into the gas
ii) An isobaric compression back to the
original volume ܸ଴
The corresponding P-V diagram is
iii) An isochoric increase in pressure till shown above.
the original pressure ܲ଴ is regained.

The efficiency of this cycle can be expressed as

Ͷܳ ൅ ʹܴܶ଴ Ͷܳ ൅ ʹܴܶ଴
(a) ߳ ൌ (b) ߳ ൌ
Ͷܳ ൅ ܴܶ଴ Ͷܳ െ ͵ܴܶ଴
Ͷܳ െ ʹܴܶ଴ Ͷܳ െ ʹܴܶ଴
(c) ߳ ൌ  (d) ߳ ൌ 
Ͷܳ ൅ ܴܶ଴ Ͷܳ ൅ ͵ܴܶ଴

33. A deuteron of mass ‫ ܯ‬and binding energy ‫ ܤ‬is struck by a gamma ray photon of
energy‫ܧ‬ఊ , and is observed to disintegrate into a neutron and a proton. If ‫ا ܤ‬
‫ ܿܯ‬ଶ , the minimum value of ‫ܧ‬ఊ must be

‫ܤ‬ଶ ‫ܤ‬ଶ
(a) ʹ‫ ܤ‬൅ (b) ‫ܤ‬൅
ʹ‫ ܿܯ‬ଶ ‫ ܿܯ‬ଶ

ͳ ‫ܤ‬ଶ ͳ ‫ܤ‬ଶ
(c)  ቆ͵‫ ܤ‬൅ ቇ (d) ቆʹ‫ ܤ‬൅ ቇ
ʹ ‫ ܿܯ‬ଶ ʹ ‫ ܿܯ‬ଶ
34. Light passes through a narrow slit and gives the
Fraunhofer diffraction pattern shown in the
adjacent figure.

Which of the following could be the shape of the


slit?

(a) (c)

L S
(b) (d)

Z +
35. For exact calculation and minimum complexity, two four-digit binary numbers can
be added with
(a) 1 full adder and 3 half-adders (b) 2 full adders and 2 half-adders
(c) 3 full adders and 1 half-adder (d) 4 full adders

Part B continues on next page


(to be attempted by all candidates for Int. M.Sc.-Ph.D. programme)
PLEASE READ CAREFULLY BEFORE PROCEEDING FURTHER

Each of the following questions (36 – 40) must be answered by a word or


a number or a simple mathematical expression, which must be written
down clearly and legibly using only black/blue ballpoint pen. If there
are any cancellations, overwriting or erasures, the question will be
considered un-answered.

Use only values of constants given in the table ‘USEFUL CONSTANTS’.

A correct answer will be awarded ൅૞ marks.


There are NO NEGATIVE MARKS for these questions.
36. Evaluate the expression
஺ ௫೙షభ ௫೙షమ ௫య ௫మ ௫భ

݊Ǩ න ݀‫ݔ‬௡ିଵ න ݀‫ݔ‬௡ିଶ න ݀‫ݔ‬௡ିଷ ǥන ݀‫ݔ‬ଶ න ݀‫ݔ‬ଵ න ݀‫ݔ‬଴ 


଴ ଴ ଴ ଴ ଴ ଴
Ans = A^n
37. In outer space, where the effects of gravity can be neglected, a
drop of liquid assumes a spherical shape. However, when
disturbed it undergoes shape oscillations (see figure). The
frequency ߥ of oscillation of a drop depends on its equilibrium
radius, its density and the surface tension.

What would be the numerical value of the ratio ߥୌ୥ Ȁߥୌమ ଴ of the
frequencies of oscillation between a drop of mercury (Hg) and
a drop of water (H2O) of the same equilibrium radius ?

You may use the following data:


Liquid Density in gm cm-3 Surface tension in N m-1
water 1.0 0.073
mercury 13.6 0.487
Ans = 0.7
38. Consider the following situations.

A B

݀
‫ݍ‬
݀ ‫ݍ‬

In situation A, two semi-infinite earthed conducting planes meet at right-angles.


A point charge ‫ݍ‬, is placed at a distance ݀ from each plane, as shown in the figure
A. The magnitude of the force exerted on the charge ‫ ݍ‬is denoted ‫ܨ‬஺ .
In situation B, the same charge ‫ ݍ‬is kept at the same distance ݀ from an infinite
earthed conducting plane, as shown in the figure B. The magnitude of the force
exerted on the charge ‫ ݍ‬is denoted ‫ܨ‬஻ .
Find the numerical ratio ‫ܨ‬஺ Τ‫ܨ‬஻

Ans = 0.9142
39. Two identical bosons may occupy any of two energy levels Ͳ, ߝ, where ߝ ൐ Ͳ. The
lowest energy state is doubly-degenerate and the excited state is non-degenerate.
Assume that the two-particle system is in thermal equilibrium at a temperature T.
Calculate the average energy ‫ۄܧۃ‬. What will be the leading term of

‫ۄܧۃ‬
ߝ
‡š’ ቀെ ቁ
݇஻ ܶ
at low temperature?
Ans = 2*epsilon/3
40. Which digital logic gate is mimicked by the following silicon diode and silicon
transistor circuit?

Ans = AND or (A.B)


GS-2017-X (Physics)
Section C
Q. 41 – 55 : to be attempted only by candidates for Ph.D. programme.
(Candidates for Integrated M.Sc.-Ph.D. programme will get NO credit
for attempting this section.)

PLEASE READ CAREFULLY BEFORE PROCEEDING FURTHER

The following questions (41 – 50) are all of multiple-choice type. For
every question, four options (a), (b), (c) and (d) are given, of which only
one is correct. Indicate the correct option on the OMR by filling in the
bubble next to the correct label.

A correct answer will be awarded ൅૜ marks and an incorrect answer will


be awarded െ૚ mark. If the question is not attempted, no marks will be
awarded.

41. The value of the integral



݀‫ݔ‬

଴ ‫ݔ‬ସ ൅ Ͷ
Is
ߨ ߨ ߨ
(a) ߨ (b) (c)  (d) 
ʹ Ͷ ͺ

42. The Lagrangian of a system described by a single generalised coordinate ‫ݍ‬is


ͳ
‫ ܮ‬ൌ ‫ݍ‬ሶ •‹ଶ ‫ݍ‬
ʹ
Its Hamiltonian is
(a) ‘–†‡ˆ‹‡† (b) Zero
ͳ
(c) െ‫ݍ‬ሶ •‹ଶ ‫ݍ‬ (d) ‫ݍ‬ሶ ൬‫ ݌‬െ •‹ଶ ‫ݍ‬൰
ʹ

43. A one-dimensional quantum harmonic oscillator of natural frequency ߱ is in


thermal equilibrium with a heat bath at temperatureܶ. The mean value ‫ ۄܧۃ‬of
the energy of the oscillator can be written as

԰߱ ԰߱ ԰߱ ԰߱
(a) •‡…Š൬ ൰ (b) …•…Š൬ ൰
ʹ ʹ݇஻ ܶ ʹ ʹ݇஻ ܶ
԰߱ ԰‫ݓ‬ ԰߱ ԰߱
(c) …‘–Š൬ ൰ (d) –ƒŠ ൬ ൰
ʹ ʹ݇஻ ܶ ʹ ʹ݇஻ ܶ
44. Consider a spherical shell with radius ܴ such that
the potential on the surface of the shell in spherical
coordinates is given by,
ܸሺ‫ ݎ‬ൌ ܴǡ ߠǡ ߮ሻ ൌ ܸ଴ …‘• ଶ ߠ P
ܴ ߠ
where the angle ߠ is shown in the figure. There are
no charges except for those on the shell. The C
potential outside the shell at the point P a distance
ʹܴ away from its center C (see figure) is

ܸ଴ ܸ଴
(a) ܸ ൌ ሺͳ ൅ …‘•ଶ ߠሻ (c) ܸ ൌ ሺͳ ൅ ʹ …‘•ଶ ߠሻ
ͺ ͺ

ܸ଴ ܸ଴
(b) ܸ ൌ  ሺͳ െ …‘•ଶ ߠሻ (d) ܸ ൌ ሺെʹ …‘• ߠ ൅ …‘• ଷ ߠሻ
Ͷ ʹ

45. A quantum mechanical system which has stationary states ȁͳ‫ ۄ‬, ȁʹ‫ ۄ‬and ȁ͵‫ ۄ‬,
corresponding to energy levels 0 eV, 1 eV and 2 eV respectively, is perturbed by
a potential of the form
ܸ෠ ൌ ߝȁͳ‫͵ۃۄ‬ȁ ൅ ߝȁ͵‫ͳۃۄ‬ȁ
where, in eV, Ͳ ൏ ߝ ‫ͳ ا‬.
The new ground state, correct to order ߝ, is approximately.

ߝ ߝ ߝ
(a) ቀͳ െ ቁ ȁͳ‫ ۄ‬൅ ȁ͵‫ۄ‬ (b) ȁͳ‫ ۄ‬൅ ȁʹ‫ ۄ‬െ ߝȁ͵‫ۄ‬
ʹ ʹ ʹ

ߝ ߝ
(c) ȁͳ‫ ۄ‬൅ ȁ͵‫ۄ‬ (d) ȁͳ‫ ۄ‬െ ȁ͵‫ۄ‬
ʹ ʹ

46. Hydrogen atoms in the atmosphere of a star are in thermal equilibrium, with an
average kinetic energy of 1 eV. The ratio of the number of hydrogen atoms in the
2nd excited state (݊ ൌ ͵) to the number in the ground state (݊ ൌ ͳ) is

(a) ͵Ǥͳ͸ ൈ ͳͲିଵଵ (b) ͳǤ͵͵ ൈ ͳͲି଼

(c) ͵Ǥͳ͸ ൈ ͳͲି଼ (d) ͷǤ͸ʹ ൈ ͳͲି଺


47. A photomultiplier tube is used to detect identical light pulses each of which
consists of a fixed number of photons. The photoelectric efficiency is 10%, i.e. a
photon has 10% probability of causing the emission of a detectable photo-
electron. The photomultiplier gain is ͳͲ଺ .

‫ܫ‬ሺ‫ݐ‬ሻ

‫ܫ‬୫ୟ୶
‫ݐ‬
ʹͲ•
The typical output current, as a function of time, is shown by the figure below for
a few pulses, where ‫ܫ‬୫ୟ୶ is 80 μA. It follows that the number of photons in each
pulse is

(a) ͷ ൈ ͳͲ଺ (b) ͷ


(c) ͺͲͲ (d) ͷͲ

48. A subatomic particle ߰ and its excited state ߰ ‫ כ‬have rest masses 3.1 GeVȀܿ ଶ and
3.7 GeVȀܿ ଶ respectively. A table of its assigned quantum numbers is given below.

Angular Momentum Parity C-Parity Isospin Electric charge


‫ ܬ‬ൌ ͳ ܲ ൌ  െͳ ‫ ܥ‬ൌ  െͳ ‫ ܫ‬ൌ Ͳ ܳ ൌ Ͳ
If ߨ ଴‫ כ‬is an excited state of ߨ ଴ with a mass of about 1.3 GeVȀܿ ଶ , which of the
following reactions is possible when the above quantum numbers are conserved?

(a) ߰ ‫ כ‬՜ ߛߛ (b) ߰ ‫ כ‬՜ ߰ߨ ା ߨ ି


(c) ߰ ‫ כ‬՜  ߨ ଴ ߨ ଴ (d) ߰ ‫ כ‬՜ ߰ߨ ଴‫כ‬

49. The cosmic microwave background radiation in the Universe has a blackbody
distribution corresponding to a temperature 2.735 K. In a certain cosmological
model, it was assumed that the universe consists purely of radiation and is
undergoing adiabatic expansion. In this model it was predicted that the volume
of the Universe will be tripled in the next ͳͲଵ଴ yrs. The corresponding blackbody
radiation temperature would be

(a) 0.9116 K (b) 2.078 K


(c) 1.896 K (d) 1.526 K
50. The following circuit is fed with an input sine wave of frequency ͷͲ Hz.

Which of the following graphs (solid line is input and dashed line is output) best
represents the correct situation?

(a) (b)

(c) (d)

Part C continues on next page


(to be attempted by all candidates for Ph.D. programme)
PLEASE READ CAREFULLY BEFORE PROCEEDING FURTHER

Each of the following questions (51 – 55) must be answered by a word or


a number or a simple mathematical expression, which must be written
down clearly and legibly using only black/blue ballpoint pen. If there
are any cancellations, overwriting or erasures, the question will be
considered un-answered.

Use only values of constants given in the table ‘USEFUL CONSTANTS’.

A correct answer will be awarded ൅૞ marks.


There are NO NEGATIVE MARKS for these questions.
51. Write down ‫ݔ‬ሺ‫ݐ‬ሻ, where ‫ݔ‬ሺ‫ݐ‬ሻ is the solution of the following differential equation

݀ ݀
൬ ൅ ʹ൰ ൬ ൅ ͳ൰ ‫ ݔ‬ൌ ͳǡ
݀‫ݐ‬ ݀‫ݐ‬

with the boundary conditions


݀‫ݔ‬ ͳ
ฬ ൌ Ͳǡ‫ݔ‬ሺ‫ݐ‬ሻȁ௧ୀ଴ ൌ െ
݀‫ ݐ‬௧ୀ଴ ʹ
Ans = exp(-2t) -2exp(-t) + 1/2
52. A particle of mass ݉, confined to one dimension ‫ݔ‬, is in the ground state of a
harmonic oscillator potential with a normalized wave function

ʹܽ ସ మ
Ȳ଴ ሺ‫ݔ‬ሻ ൌ ൬ ൰ ݁ ି௔௫
ߨ

where ܽ ൌ ݉߱Τʹ԰. Find the expectation value of ‫ ଼ ݔ‬in terms of the parameter ܽ.
Ans = 105/(256*a^4)
53. Electrons in a metal are scattered by both impurities and phonons. The impurity
scattering time is ͺ ൈ ͳͲିଵଶ s and the phonon scattering time is ʹ ൈ ͳͲିଵଶ s. Taking
the density of electrons to be ͵ ൈ ͳͲଵସ m-3, find the conductivity of the metal in
units of A V-1 m-1. [Assume that the effective mass of the electrons is the same as
that of a free electron.]
Ans = 13.5 x 10-6

54. In a theoretical model of the nucleus, the binding energy per nucleon was
predicted as shown in the figure below.

If a nucleus of mass number A ൌ ʹͶͲ undergoes a symmetric fission to two


daughter nuclei each of mass number A ൌ ͳʹͲ, write down the amount of energy
released in this process, in units of MeV, using this theoretical model.

Ans = 240
55. Assume that the crystal structure of metallic copper (Cu) results in a density of
atoms ߩେ୳ ൌ ͺǤͶ͸ ൈ  ͳͲଶ଼m-3. Each Cu atom in the crystal donates one electron to
the conduction band, which leads, for the 3-D Fermi gas, to a density of states

ͳ ʹ݉‫ כ‬ଷȀଶ ଵȀଶ


݃ሺߝሻ ൌ ଶ ൬ ଶ ൰ ߝ
ʹߨ ԰

where ݉‫ כ‬is the effective mass of the conduction electrons. In the low temperature
limit (i.e. ܶ ൌ Ͳ‫)ܭ‬, find the Fermi energy ‫ܧ‬ி , in units of eV. You may assume ݉‫כ‬
to be equal to the free electron mass ݉௘ .

Ans 6.87

You might also like